You are on page 1of 1064

HUGE RESOURCE OF ANAESTHESIA TESTS AND EXAMINATION QUESTIONS

From the Internet (www.manbit.com, www.anaesthesiauk.com)

2005 Pcs, Hungary


For every person who wants to pass..

Contents:

TESTS 1, 2, 3, 4, 5, 6, 7, 8, 9, 10, 11, 12, 13, 14, 15, 16, 17, 18, 19, 20, 21, 22, 23, 24, 25, 26, 27, ANATOMY ANAESTHESIA MONITORING CARDIORESPIRATORY PHYSIOLOGY CLINICAL INVESTIGATION GENERAL ANAESTHESIA OPIATES AND OTHERS MEDICINE ECHOCARDIOGRAPHY OBSTETRIC ANAESTHESIA PEDIATRIC ANAESTHESIA PERFUSION PHARMACOLOGY OTHER PHYSIOLOGY REGIONAL AND SPECIALTY RELAXANT AND LOCAL ANAESTHETICS RENAL AND NEUROPHYSIOLOGY STATISTICS SURGERY TRAUMA TRIVIAL PURSUITS X-RAY ECG GENERAL PRIMARY MCQS MCQS A-K MCQ PAPERS PHYSIOLOGY MCQS PHARMACOLOGY MCQS pgs :3-66 pgs: 66-129 pgs: 130-160 pgs: 160-173 pgs: 174-260 pgs. 260-309 pgs: 309-395 pgs: 396-401 pgs: 402-426 pgs: 426-450 pgs: 450-471 pgs: 471-492 pgs: 492-499 pgs: 499-564 pgs: 564-595 pgs: 595-607 pgs: 607-617 pgs: 617-642 pgs: 642-660 pgs: 661-677 pgs: 677-689 pgs: 690-706 pgs: 707-776 pgs: 776-859 pgs: 859-900 pgs: 901-922 pgs: 922-937

ORAL QUESTIONS 28, 29, OSCE AND VIVA QUESTIONS FINAL FRCA QUESTIONS pgs: 937-1020 pgs: 1020-1064

ANATOMY TESTS Number: 84 In adults, the angle at which the right main bronchus leaves the carina is: A. 15 degrees. B. 20 degrees. C. 25 degrees. D. 30 degrees. E. 35 degrees. Select the single best answer ABCDE Correct Answer: C In adults, the angle between the left and right main bronchi and the midline are 45 and 25 degrees respectively. In infants, these are 47 and 30 degrees. LATTO, I.P & ROSEN, M. (EDS); Difficulties in Tracheal Intubation, Balliere Tindall, pp 910. Number: 134 Which of the following are true of a cervical rib? A. It occurs commonly. B. It is apparent on palpation in the supraclavicular region. C. It originates from the 7th cervical vertebra. D. It commonly causes compression of the subclavian artery and brachial plexus. E. All of the above. Select the single best answer ABCDE Correct Answer: C In 0.5-1% of individuals, the costal elements of the 7th cervical vertebra form projections called cervical ribs. Commonly they have a head, neck, and tubercle, with varying amounts of body. They extend into the posterior triangle of the neck where they may be free anteriorly, or be attached to the first rib and/or sternum. Usually these ribs cause no symptoms, and are diagnosed after incidental finding on CXR. In some cases, the subclavian artery and the lower trunk of the brachial plexus are kinked where they pass over the cervical rib. Compression of these structures between this extra rib and the anterior scalene muscle may produce symptoms of nerve and arterial compression, producing the "neurovascular compression syndrome".

Often the tingling, numbness, and impaired circulation to the upper limb do not appear until the age of puberty when the neck elongates and the shoulders tend to droop slightly. MOORE, K.L.; Clinically Oriented Anatomy, Williams and Wilkins, 1980, pp 701. Number: 135 With respect to the intercostal nerves: 1. They represent the dorsal rami of the thoracic spinal nerves. 2. The twelfth intercostal nerve is also known as the subcostal nerve. 3. Anteriorly they run in the costal groove on the upper margin of the rib. 4. Each is connected to a ganglion of the sympathetic trunk. A: 1,2,3 Correct B: 1,3 Correct C: 2,4 Correct D: 4 Correct E: All Correct ABCDE Correct Answer: C The intercostal nerves represent the ventral rami of the first 11 thoracic spinal nerves. The twelfth, being below the 12th rib is subcostal, hence is called the subcostal nerve. Each intercostal nerve is connected to a ganglion of the sympathetic trunk by rami communicantes to and from which it carries preganglionic and postganglionic fibres which innervate blood vessels, sweat glands, and muscles. The 3rd to 6th intercostal nerves behave typically. A typical nerve enters the intercostal space between the parietal pleura and internal intercostal membrane and muscle. At first it runs along the middle of the intercostal space, then at the angle of the rib, it passes between the internal and innermost intercostal muscles (which begins here). Here it enters the costal groove on the inferior margin of the rib, where it runs with the intercostal vessels. It continues forward giving off several branches and terminates about 1 cm from the sternum as an anterior cutaneous branch. The branches comprise: (1) A collateral branch which arises near the angle of the rib and runs along the upper margin of the rib below to supply the intercostal muscles. (2) A lateral cutaneous branch which arises beyond the angle of the rib and pierces the internal and external intercostal muscles about halfway around the thorax. The cutaneous branches divide into anterior and posterior branches which supply the skin of the thoracic and abdominal walls. (3) Muscular branches supply the subcostal, transversus thoracis, levator costae, and serratus posterior muscles. Note that the intercostal nerves do not supply the muscles connecting the pectoral muscles, trapezius, lattisimus dorsi, rhomboids, or levator scapulae. These muscles are supplied by the accessory nerve and the cervical and brachial plexuses.

The other intercostal nerves have some individual characteristics. (1)The first intercostal nerve usually has no lateral cutaneous branch. It divides into a large upper and small lower part. The upper part joins the brachial plexus; the lower part becomes the 1st intercostal nerve. (2) The second intercostal nerve may also contribute a small branch to the brachial plexus. Its lateral cutaneous branch is called the intercostobrachial nerve which supplies sensory innervation to the floor of the axilla and communicates with the medial cutaneous nerve of the arm to supply the medial aspect of the arm as far as the elbow. (3) The 7th to 11th intercostal nerves supply the abdominal as well as the thoracic wall. References MOORE, K.L.; Clinically Oriented Anatomy, Williams and Wilkins, 1980, pp 29-30. Number: 136 The musculocutaneous nerve: A. Is a branch of the lateral cord of the brachial plexus. B. Leaves the plexus at the point at which it crosses the first rib. C. Receives fibres from the 8th cervical spinal root. D. Only contains sensory fibres. E. None of the above. Select the single best answer ABCDE Correct Answer: A The musculocutaneous nerve is the major terminal branch of the lateral cord of the brachial plexus. The lateral cord contains fibres from the superior and middle trunks which represent the spinal roots of the 5th to 7th cervical vertebrae. It leaves the plexus as the cords give off their branches to the major nerves of the arm. The cords correspond to the point at which the plexus emerges from behind the clavicle. It courses the axilla in the coracobrachialis muscle, and then descends obliquely and laterally between the biceps and brachialis muscles, sending motor fibres to all of these. It terminates in the forearm as the lateral antebrachial cutaneous nerve supplying sensation to the lateral aspect of the forearm. WINNIE, A.P.; Plexus Anesthesia, vol 1, Churchill Livingstone, 1984, p 224 & pp 16-28. Number: 137 Which of the following are branches of the brachial plexus? 1. Lateral pectoral nerve. 2. Medial pectoral nerve. 3. Medial brachial cutaneous nerve. 4. The intercostobrachial nerve.

A: 1,2,3 Correct B: 1,3 Correct C: 2,4 Correct D: 4 Correct E: All Correct ABCDE Correct Answer: A The intercostobrachial nerve represents the lateral cutaneous branch of the second intercostal nerve. It forms a loop and runs with the medial brachial cutaneous nerve, together supplying sensory innervation of the medial aspect of the arm as far as the elbow. All the others represent infraclavicular branches of the brachial plexus. WINNIE, A.P.; Plexus Anesthesia, vol 1, Churchill Livingstone, 1984, pp 19-42.

Which of the following structures accompany the median nerve in the carpal tunnel? A. Flexor carpi ulnaris. B. Flexor digitorum profundis. C. The ulnar artery. D. All of the above. E. None of the above. Select the single best answer ABCDE Correct Answer: B The carpal tunnel is an osseofibrous tunnel formed by the flexor retinaculum in the wrist. The median nerve and tendons of the long flexor muscles of the digits pass through it. MOORE, K.L.; Clinically Oriented Anatomy, Williams and Wilkins, 1980, p 767. Number: 139 Transection of the facial nerve as it passes through the facial canal will be evidenced by: 1. Inability to wink the eye. 2. Inability to whistle. 3. Dribbling. 4. Inability to raise the eyebrows.

A: 1,2,3 Correct B: 1,3 Correct C: 2,4 Correct D: 4 Correct E: All Correct ABCDE Correct Answer: A All the muscles of facial expression are supplied by the facial nerve. The frontalis is part of the a scalp muscle called the occipito-frontalis. It elevates the eyebrows. This muscle receives part of its innervation from the contralateral motor cortex and preservation of this manoeuvre in the presence of ipsilateral facial weakness is suggestive of an upper motor neuron lesion occurring in the motor cortex or internal capsule, before the contralateral fibres join the ipsilateral fibres and become the facial nerve. The facial nerve also innervates the buccinator, hence difficulty chewing, whistling, and withholding dribble may occur. MOORE, K.L.; Clinically Oriented Anatomy, Williams and Wilkins, 1980, pp 878-82. Number: 139 Transection of the facial nerve as it passes through the facial canal will be evidenced by: 1. Inability to wink the eye. 2. Inability to whistle. 3. Dribbling. 4. Inability to raise the eyebrows. A: 1,2,3 Correct B: 1,3 Correct C: 2,4 Correct D: 4 Correct E: All Correct ABCDE Correct Answer: A All the muscles of facial expression are supplied by the facial nerve. The frontalis is part of the a scalp muscle called the occipito-frontalis. It elevates the eyebrows. This muscle receives part of its innervation from the contralateral motor cortex and preservation of this manoeuvre in the presence of ipsilateral facial weakness is suggestive of an upper motor neuron lesion occurring in the motor cortex or internal capsule, before the contralateral fibres join the ipsilateral fibres and become the facial nerve. The facial nerve also innervates the buccinator, hence difficulty chewing, whistling, and withholding dribble may occur.

MOORE, K.L.; Clinically Oriented Anatomy, Williams and Wilkins, 1980, pp 878-82. Number: 141 Which of the following structures pass through the foramen magnum? 1. The vertebral arteries. 2. The glossopharyngeal nerves. 3. Spinal arteries supplying the upper portion of the spinal cord. 4. The vagi. A: 1,2,3 Correct B: 1,3 Correct C: 2,4 Correct D: 4 Correct E: All Correct ABCDE Correct Answer: B The structures passing through the foramen magnum are: (1)The spinal cord- the junction of the spinal cord and medulla of the brainstem. (2)The spinal roots of the accessory nerves (cranial nerves IX) (3)The meningeal branches of the upper cervical nerves (C1to C3) (4)The meninges (5)The vertebral arteries ascending to supply parts of the brain (6)The anterior and posterior spinal arteries descending to supply the upper part of the spinal cord. Four other foramina exist in the posterior cranial fossa. These are the jugular foramen, the hypoglossal canal, the condylar canal, and the internal acoustic meatus. The contents of the jugular foramen are: (1)The superior bulb of the internal jugular vein. The sigmoid sinus enters into this. The jugular glomus is a small ovoid body consisting of chemoreceptor tissue which is enclosed in the adventitia of the jugular bulb. (2)The glossopharyngeal nerve (3)The vagus (4)The accessory nerve (5)The inferior petrosal sinus on its way to the upper end of the internal jugular vein. References MOORE, K.L.; Clinically Oriented Anatomy, Williams and Wilkins, 1980, pp 911-13.

Number: 141 Which of the following structures pass through the foramen magnum? 1. The vertebral arteries. 2. The glossopharyngeal nerves. 3. Spinal arteries supplying the upper portion of the spinal cord. 4. The vagi. A: 1,2,3 Correct B: 1,3 Correct C: 2,4 Correct D: 4 Correct E: All Correct ABCDE Correct Answer: B The structures passing through the foramen magnum are: (1)The spinal cord- the junction of the spinal cord and medulla of the brainstem. (2)The spinal roots of the accessory nerves (cranial nerves IX) (3)The meningeal branches of the upper cervical nerves (C1to C3) (4)The meninges (5)The vertebral arteries ascending to supply parts of the brain (6)The anterior and posterior spinal arteries descending to supply the upper part of the spinal cord. Four other foramina exist in the posterior cranial fossa. These are the jugular foramen, the hypoglossal canal, the condylar canal, and the internal acoustic meatus. The contents of the jugular foramen are: (1)The superior bulb of the internal jugular vein. The sigmoid sinus enters into this. The jugular glomus is a small ovoid body consisting of chemoreceptor tissue which is enclosed in the adventitia of the jugular bulb. (2)The glossopharyngeal nerve (3)The vagus (4)The accessory nerve (5)The inferior petrosal sinus on its way to the upper end of the internal jugular vein. References MOORE, K.L.; Clinically Oriented Anatomy, Williams and Wilkins, 1980, pp 911-13.

Number: 142 Which of the following are true statements concerning the blood supply of the spinal cord via the anterior spinal artery? A. A single pair of arteries supply the anterior region for the entire length of the cord. B. No collateral blood supply exists. C. Thrombosis results in paraplegia and paraesthesia. D. All of the above. E. None of the above. Select the single best answer ABCDE Correct Answer: E The anterior spinal artery is a single vessel lying in the pia mater in front of the anterior median fissure. It arises from the junction of two small arteries which are branches of each vertebral arteries at the level of the foramen magnum. It descends along the entire length of the spinal cord, receiving small communications from the intercostal and lumbar arteries. To provide the extra blood supply needed in the thoracic and lumbar enlargements, the communicating branches at the level of T1 and T2 are larger than the others (the arteries of Adamkiewicz). Thrombosis of the anterior spinal artery results in the syndrome in which there is paraplegia without involvement of the modalities subserved by the posterior columns: joint position, touch, and vibration sense. The posterior spinal arteries are two or three in number on each side and originate in the posterior inferior cerebellar arteries at the base of the brain. They supply the posterior columns of the cord. There are no anastomoses between the anterior and posterior arteries: in fact, the vascularization of the cord comprises three distinct territories- one anterior, and two posterior. LEE, J.A. ET AL; Sir Robert Macintosh's Lumbar Puncture and Spinal Analgesia, 5th Ed., Churchill Livingstone, 1985, p 53. Number: 143 A palsy of the third cranial nerve will result in which apparent gaze of the eyeball? A. Inferolaterally. B. Inferomedially. C. Superolaterally. D. Superomedially. E. Laterally. Select the single best answer

10

ABCDE Correct Answer: A The cranial nerves, CN III (oculomotor), IV (trochlear), and VI (abducens) supply the muscles of the orbit. They enter the orbit through the superior orbital fissure. CN IV supplies the superior oblique which rotates the eyeball in an inferolateral direction; CN VI supplies the lateral rectus which abducts the eye; and CN III supplies the levator palpebrae superioris which elevate the eyelid, and the medial , superior, and inferior rectus muscles, which rotate the eyeball in the same direction. A palsy of CN III will result in the unopposed action of CN IV and VI. The eye will appear to gaze "down and out". MOORE, K.L.; Clinically Oriented Anatomy, Williams and Wilkins, 1980, p 970. Number: 144 Which of the following structures would be encountered if a needle were passed directly back at a site two fingerbreadths medial and inferior to the anterior superior iliac spine? 1. The aponeurosis of the external oblique muscle. 2. The transversalis fascia. 3. Scarpa's fascia. 4. Camper's fascia. A: 1,2,3 Correct B: 1,3 Correct C: 2,4 Correct D: 4 Correct E: All Correct ABCDE Correct Answer: E The fascia of the anterior abdominal wall is divided into superficial and deep. The superficial fascia consists of a fatty superficial layer (Camper's fascia) and membranous deeper layer (Scarpa's fascia). The deep fascia forms a layer over the external oblique muscle. The transversalis fascia lines the entire abdominal wall. It covers the deep surface of the transversalis muscle and its aponeurosis. MOORE, K.L.; Clinically Oriented Anatomy, Williams and Wilkins, 1980, pp 127-9.

11

Number: 145 A needle inserted directly backwards into the sacral hiatus: 1. Will penetrate the sacro-coccygeal ligament. 2. Will encounter the 3rd sacral vertebral segment. 3. Is likely to remain extradurally. 4. Is at the level of the posterior superior iliac spines. A: 1,2,3 Correct B: 1,3 Correct C: 2,4 Correct D: 4 Correct E: All Correct ABCDE Correct Answer: B The sacral hiatus is formed by the failure of fusion of the 5th and often the 4th vertebral laminae. It is covered by the sacro-coccygeal ligament. The sacral canal ends at the sacral hiatus. It follows the curve of the sacrum and in cross section is triangular. In the average adult, the dural sac extends to the lower border of the 2nd sacral vertebra which is at the level of the posterior superior iliac spines. This discrepancy between the dural sac and the sacral canal is exploited in caudal anaesthesia. The filum terminale extends from the cord, through the dura to the coccygeal periosteum. The canal contains in addition to the dural sac, the sacral nerves, loose fat, veins, and the filum terminale. LEE, J.A. ET AL; Sir Robert Macintosh's, Lumbar Puncture and Spinal Analgesia, 5th Ed., Churchill Livingstone, 1985, p 66. Number: 146 Which of the following arteries form part of the "circle of Willis"? 1. Anterior communicating artery. 2. Posterior communicating arteries. 3. Internal carotid artery. 4. Basilar arteries. A: 1,2,3 Correct B: 1,3 Correct C: 2,4 Correct D: 4 Correct E: All Correct ABCDE

12

Correct Answer: E The " circle of Willis " is formed at the base of the brain and represents the fusion of the four major arterial systems supplying the brain: the internal carotid arteries, and the vertebral arteries. It is formed by the anterior communicating artery, the anterior cerebral, a short segment of each internal carotid, the posterior communicating, the posterior cerebral and the basilar arteries. The middle cerebral artery is a continuation of the internal carotid artery and is not part of the circle. MOORE, K.L.; Clinically Oriented Anatomy, Williams and Wilkins, 1980, pp 949-50. Number: 147 Which of the following are contained within the carotid sheath? 1. The common carotid artery. 2. The external carotid artery. 3. The internal carotid artery. 4. The sympathetic trunk. A: 1,2,3 Correct B: 1,3 Correct C: 2,4 Correct D: 4 Correct E: All Correct ABCDE Correct Answer: B The carotid sheath is a tubular, fascial condensation that extends from the base of the skull to the root of the neck. It is formed by the fascial extensions of the cervical fascia, and its fibres fuse with the prevertebral fascia. It contains several structures: 1. The common and internal carotid arteries. 2. The internal jugular vein. 3. The vagus nerve. 4. The superior root of the ansa cervicalis is sometimes embedded within the carotid sheath. 5. Deep cervical lymph nodes. The cervical part of the sympathetic trunk runs posterior to the sheath. MOORE, K.L.; Clinically Oriented Anatomy, Williams and Wilkins, 1980, p 1128.

13

Number: 148 Which of the following statements are true? A. The most prominent spinous process palpable represents T1. B. The tip of the spine of T9 is opposite the inferior angle of the scapula. C. The dimples overlying the posterior superior iliac spines are on a line crossing the termination of the dural sac in the spinal canal at S2. D. The lower end of the spinal cord is opposite the lower border of the body of L2 and sometimes extends a little below this. E. All of the above. Select the single best answer ABCDE Correct Answer: C The spinous process of C7 (vertebra prominens) is generally easily palpated and represents the most prominent spinous process. The tip of the spine of T7 is opposite the inferior angle of the scapula when the arms are held by the side. The lower end of the spinal cord is opposite the lower border of the body of L1 and sometimes extends a little below this. LEE, J.A. ET AL; Sir Robert Macintosh's, Lumbar Puncture and Spinal Analgesia, 5th Ed., Churchill Livingstone, 1985, p 38. Number: 149 With respect to the extradural space: 1. The spinal dura mater is loosely attached to the foramen magnum and permits some spread of local anaesthetic agent cephalad during a "high" extradural block. 2. It terminates with the filum terminale at the lower border of the second sacral vertebra. 3. It is of uniform width from the anterior to posterior compartments. 4. The extradural veins mainly occupy the antero-lateral compartment. A: 1,2,3 Correct B: 1,3 Correct C: 2,4 Correct D: 4 Correct E: All Correct ABCDE Correct Answer: D

14

The dura mater, although continuous, can be described in two parts, cranial and spinal. The cranial dura mater consists of two layers, endosteal and meningeal (opponents of this view state that the endosteal layer is actually the inner periosteal lining), closely united except where they enclose the great venous sinuses which drain the blood from the brain. At the foramen magnum, the endosteal layer is reflected back to become continuous with the periosteum on the outer surface of the bone. In the vertebral canal, it is represented by the periosteal lining of the vertebrae. The meningeal layer invests the brain and folds inwards to form the tentorium cerebelli and falx cerebri. In the vertebral canal, the spinal dura mater represents the downward continuation of the meningeal layer of the cranial dura mater. It is firmly attached around the circumference of the foramen magnum. The spread of methylene blue injected into the extradural space has been observed. The upward spread is limited by the attachment of the spinal dura mater to the foramen magnum. The spinal dura mater invests the spinal cord forming a "dural sac" between itself and the arachnoid mater. The dura mater and its sac terminate at the lower border of the second sacral spinal vertebra. The potential space which is the extradural space extends with the filum terminale caudad to this; the lowermost point of the extradural space occurs at the sacral hiatus and the filum terminale inserts into the periosteum at the back of the coccyx. This discrepancy is exploited in caudal anaesthesia. The width of the extradural space largely depends upon the amount of fat contained at any site. This is greatest in the median plane posteriorly where the summit of the vertebral arch is commonly separated from the rounded posterior aspect of the dura by 5-6 mm, and anterolaterally where it is continuous with the pads of fat surrounding the spinal nerves in the intervertebral foramina. Between the posterior-lateral walls of the lumbar vertebral canal and the dura, the space is narrower, and the fat is less evident. Anteriorly, in a thin subject, the space is only potential, since here the dura lies close to the posterior longitudinal ligament on the posterior aspects of the vertebral bodies The extradural veins form a plexus which is most dense in the antero-lateral compartment. LEE, J.A. ET AL; Sir Robert Macintosh's, Lumbar Puncture and Spinal Analgesia, 5th Ed., Churchill Livingstone, 1985, pp 53-60. Number: 150 The depth of the subarachnoid space in the thoracic region is: A. 1 mm. B. 3 mm. C. 5 mm. D. 9 mm. E. 12 mm. Select the single best answer ABCDE Correct Answer: B

15

In the cervical and thoracic regions of the spinal cord. the subarachnoid space is annular and has a depth of only 3 mm between the arachnoid mater and the pia mater which is adherent to the spinal cord. The spinal cord terminates at the lower border of L1 (or upper border of L2 in some texts). At this point, the subarachnoid space becomes circular and has a diameter of approximately 15 mm. LEE, J.A. ET AL; Sir Robert Macintosh's, Lumbar Puncture and Spinal Analgesia, 5th Ed., Churchill Livingstone, 1985, p 60 Number: 151 In a neonate, the spinal cord terminates at the lower border of: A. T12. B. L1. C. L2. D. L3. E. L4. Select the single best answer ABCDE Correct Answer: D In early foetal life, the spinal cord is as long as the vertebral canal. During development, however, increase in the length of the cord does not keep pace with the growth of the vertebrae. At birth the tip of the spinal cord has risen from the level of the second coccygeal vertebra to the lower border of the third lumbar vertebra. LEE, J.A ET AL; Sir Robert Macintosh's, Lumbar Puncture and Spinal Analgesia, 5th Ed., Churchill Livingstone, 1985, p 64. Number: 152 Which of the following statements concerning the innervation of the abdomen by the autonomic nervous system are true? 1. Afferents accompanying sympathetic fibres enter the spinal cord between T5 and L1. 2. All splanchnic sympathetic nerves pierce the crura of the diaphragm. 3. The coeliac plexus receives sympathetic and parasympathetic fibres. 4. Afferent impulses accompanying parasympathetic fibres cannot be abolished by a block to T5. A: 1,2,3 Correct B: 1,3 Correct C: 2,4 Correct D: 4 Correct

16

E: All Correct ABCDE Correct Answer: E The visceral motor nerve supply to the abdominal and pelvic organs are derived from both sympathetic and parasympathetic components of the autonomic nervous system. Sympathetic preganglionic fibres leave the spinal cord between the levels of T5 and L1 or L2. After leaving the paravertebral ganglia they fuse to form three splanchnic nerves on each side. These enter the abdomen by piercing the crura of the diaphragm and end in the coeliac plexus and the other pre-aortic plexuses from which they reach the viscera with the arterial supply. Visceral afferent impulses from the abdomen travel along fibres which accompany the efferent sympathetic fibres, pass through the paravertebral sympathetic ganglia and proceed up the splanchnic nerves to enter the spinal nerves. They enter the spinal cord via the dorsal root ganglia between T5 and L2. The visceral parasympathetic nerves leave the central nervous system in two distinct parts; cranial and sacral. That section of the cranial parasympathetic outflow which supplies the viscera consists of fibres in the vagus nerves which run their course outside the vertebral canal to enter the abdomen, passing through a hole in the diaphragm with the oesophagus at the level of the 10th thoracic vertebra. Here they innervate the stomach, and communicate freely with the coeliac plexus through which they are distributed to the rest of the alimentary canal up to the distal part of the transverse colon. Afferent impulses accompany the vagi as evidenced the inability to abolish "visceral" type sensation with a block of spinal segments to T5. LEE, J.A .ET AL; Sir Robert Macintosh's, Lumbar Puncture and Spinal Analgesia, 5th Ed., Churchill Livingstone, 1985, p 78-9. Number: 153 Which cranial nerves contribute to the sensory innervation of the tongue? 1. V. 2. VII. 3. IX. 4. XII. A: 1,2,3 Correct B: 1,3 Correct C: 2,4 Correct D: 4 Correct E: All Correct ABCDE Correct Answer: A

17

Sensation to the anterior two thirds of the tongue is supplied by the lingual branch of the mandibular nerve, the third division of the trigeminal nerve (CN V). This nerve also carries the taste fibres of the chorda tympani branch of the facial nerve (CN VII). Common sensation and taste to the posterior one third of the tongue is supplied by the glossopharyngeal nerve (CN IX). A few fibres of the superior laryngeal branch of the vagus (CN X) also supply sensation to this area. The hypoglossal nerve (CN XII) provides motor innervation to all the muscles of the tongue except palatoglossus which is innervated by CN XI via the pharyngeal plexus. MORRIS, I.R; " Functional Anatomy of the Airway ", Emergency Medicine Clinics of North America, vol 6, no 4, Nov. 1988, p 650. Number: 154 The cricoid cartilage in an infant is located at the level of which cervical vertebra? A. C1. B. C2. C. C3. D. C4. E. C5. Select the single best answer ABCDE Correct Answer: D In infants, the glottis is located two to three segments higher than in adults. The cricoid cartilage in infants is at the level of C4 as opposed to C6 in adults. ROGERS, M.C. ET AL (EDS); Principles and Practice of Anesthesiology, Mosby, 1993, p 442. Number: 155 Which of the following structures are contained within the posterior mediastinum? 1. The thoracic duct. 2. Oesophagus. 3. Azygos vein. 4. The thymus gland. A: 1,2,3 Correct B: 1,3 Correct C: 2,4 Correct D: 4 Correct E: All Correct

18

ABCDE Correct Answer: A The posterior mediastinum is the part of the mediastinum located posterior to the fibrous pericardium, below the fourth thoracic vertebra. Its lower part lies at a lower level than the anterior part of the diaphragm, but at the level of the diaphragm posteriorly. It contains the thoracic aorta, oesophagus, thoracic duct, azygos and hemiazygos veins, posterior intercostal arteries and some intercostal veins. The thymus gland is contained in the superior mediastinum. References MOORE, K.L.; Clinically Oriented Anatomy, Williams and Wilkins, 1980, p 99. Number: 156 The oesophagus: A. Extends for 1.5 cm below the diaphragm. B. Pierces the diaphragm to the left of the midline. C. Forms a groove in the left lobe of the liver. D. All of the above. E. None of the above. Select the single best answer ABCDE Correct Answer: D The oesophagus pierces the diaphragm just to the left of the midline and extends for 1.5 cm intraabdominally before joining the stomach. It forms a groove on the surface of the left lobe of the liver before joining the stomach. References MOORE, K.L.; Clinically Oriented Anatomy, Williams and Wilkins, 1980, pp 158-9. Number: 185 The internal jugular vein: 1. Is lateral to the carotid artery at the level of the cricoid cartilage. 2. Is postero-lateral to the carotid artery at the base of the skull. 3. Is anterior to the carotid artery at the base of the neck. 4. Lies outside the carotid sheath.

19

A: 1,2,3 Correct B: 1,3 Correct C: 2,4 Correct D: 4 Correct E: All Correct ABCDE Correct Answer: A Emerging from the base of the skull through the jugular foramen, the internal jugular vein enters the carotid sheath dorsally to the internal carotid artery (CA). Initially, it runs posterolateral to the internal, then the common carotid artery, beneath the sternocleidomastoid muscle. As it approaches the base of the neck, its relationship with the CA becomes lateral, and then anterior. References ROHEN, J.W. & YOKOCHI; Color Atlas of Anatomy, Igaku-Shoin, 1988 . Number: 209 In this Left Anterior Oblique view of the heart, the artery indicated by the figure "2" is: A. Left Anterior Descending. B. 1st Diagonal C. Obtuse Marginal D. Circumflex E. Septal Select the single best answer

ABCDE

20

Correct Answer: B 1. Left Anterior Descending. 2. 1st Diagonal 3. Obtuse Marginal 4. Circumflex

Number: 214 A patient who has suffered a penetrating wound above the clavicle and who has evidence of degeneration in the axillary, musculocutaneous and long thoracic nerves will may have severed which ventral ramus? A. C4 B. C6 C. C8 D. T1 E. None of the above. Select the single best answer ABCDE Correct Answer: B Number: 215 Which nerve usually accompanies the external jugular vein as it crosses the surface of the sternocleidomastoid muscle. A. Spinal accessory. B. Great auricular. C. Lesser occipital. D. Greater occipital. D. All of the above. Select the single best answer ABCDE Correct Answer: B The great auricular nerve winds around the posterior border of the sternomastoid (where it is occasionally palpable as a small nodules) and ascends obliquely across that muscle onto the parotid gland, where it divides to supply the skin over the gland and over the mastoid process, together with both surfaces of the auricle.

21

Number: 216 Which nerve constitutes the afferent (sensory) supply to the piriform fossa? A. Glossopharyngeal (IX) B. External laryngeal C. Internal laryngeal D. Hypoglossal (XII) E. Recurrent Laryngeal Select the single best answer ABCDE Correct Answer: C The internal laryngeal nerve, is afferent from the mucous membrane of the larynx. The area of mucosa supplied extends from the epiglottis and the back of the tongue down to the vocal folds. Simulation of the internal laryngeal nerve results in sensations of touch and pain. The nerve pierces the thyrohyoid membrane above the superior laryngeal artery and divides into terminal branches. A twig is given to the transverse arytenoid muscle, but whether these fibers are motor or proprioceptive is disputed). The internal laryngeal nerve ends by joining branches of the recurrent laryngeal nerve. The anastomosis may take place behind or in the substance of the posterior crico-arytenoid and the connection may pierce the inferior constrictor of the pharynx. Number: 216 Which nerve constitutes the afferent (sensory) supply to the piriform fossa? A. Glossopharyngeal (IX) B. External laryngeal C. Internal laryngeal D. Hypoglossal (XII) E. Recurrent Laryngeal Select the single best answer ABCDE Correct Answer: C The internal laryngeal nerve, is afferent from the mucous membrane of the larynx. The area of mucosa supplied extends from the epiglottis and the back of the tongue down to the vocal folds. Simulation of the internal laryngeal nerve results in sensations of touch and pain. The nerve pierces the thyrohyoid membrane above the superior laryngeal artery and divides into terminal branches. A twig is given to the transverse arytenoid muscle, but whether these fibers are motor or proprioceptive is disputed). The internal laryngeal nerve ends by joining branches of the recurrent laryngeal nerve. The anastomosis may take place behind or in the substance of the posterior crico-arytenoid and the connection may pierce the inferior constrictor of the pharynx.

22

Number: 218 The dorsal scapular nerve innervates: A. trapezius B. levator scapulae C. latissimus dorsi D. splenius cervicis E. None of the above Select the single best answer ABCDE Correct Answer: B The dorsal scapular nerve (Nerve to the rhomboids) arises mainly from C5, pierces scalenus medius, runs deep to levator scapulae (which it usually supplies) and finally enters the deep surface of the rhomboids. Levatot scapulae may also be supplied directly from C3 and C4 by superficial branches. Number: 219 The nerve which is most closely associated with the radial artery in the forearm is the : A. Median nerve. B. Superficial radial nerve. C. Deep radial nerve. D. Anterior interosseous nerve. E. None of the above. Select the single best answer ABCDE Correct Answer: B

23

Number: 220 Which nerve accompanies the deep palmar arch? A. Deep branch of the radial nerve B. Superficial branch of the radial nerve C. Deep branch of the ulnar nerve D. Superficial branch of the ulnar nerve E. None of the above. Select the single best answer ABCDE Correct Answer: C Number: 225 Hilton's law, applied to the knee joint, would predict innervation of the joint by: 1. The femoral nerve 2. The tibial nerve 3. The obturator nerve 4. The Common Peroneal nerve A: 1,2,3 Correct B: 1,3 Correct C: 2,4 Correct D: 4 Correct E: All Correct ABCDE Correct Answer: A Hilton's law states that the innervation of a joint is by all nerves which supply muscles which act across the same joint. Number: 234 Which nerve root, if compressed by a herniated disc, would result in a diminished knee jerk? A. L1 B. L2 C. L3 D. L4 E. L5 Select the single best answer

24

ABCDE Correct Answer: D L4 nerve root compression results a diminished knee jerk. Number: 273 The thyroid gland: 1. Has C cells that are derived from the ultimobranchial body 2. Is at the level of the fifth to seventh cervical and first thoracic vertebrae 3. May have accessory nodules in the tongue 4. Develops from the endoderm between the second and third pharyngeal pouches A: 1,2,3 Correct B: 1,3 Correct C: 2,4 Correct D: 4 Correct E: All Correct ABCDE Correct Answer: A It develops from the endoderm between the first and second pharyngeal pouches. Number: 275 Which of the following statements about the anatomy of the coronary arteries are correct: 1. Tthe anterior two thirds of the interventricular septum are supllied by the anterior descending artery 2. The atrioventricular node is typically supplied by the right coronary artery 3. The circumflex artery runs in the left atrioventricular groove 4. The left coronary artery arises from the anterior aortic sinus A: 1,2,3 Correct B: 1,3 Correct C: 2,4 Correct D: 4 Correct E: All Correct ABCDE Correct Answer: A

25

The right coronary artery arises from the anterior aortic sinus. Number: 299 The corticospinal tract: 1. Runs on the anterior aspect of the medulla. 2. Originates predominantly from the cortical cells of the precentral gyrus. 3. Runs in the pyramid. 4. Decussates in the midbrain. A: 1,2,3 Correct B: 1,3 Correct C: 2,4 Correct D: 4 Correct E: All Correct ABCDE Correct Answer: A The decussation is in the hindbrain. Number: 335 All the following muscles have an insertion on the radius, EXCEPT: A. Supinator B. Pronator quadratus C. Pronator teres D. Biceps brachii E. Brachialis Select the single best answer ABCDE Correct Answer: E Brachialis arises from the front of the lower two-thirds of the humerus and the medial intermuscular septum. Its upper fibres clasp the deltoid insertion; some fibres arise from the lower part of the spiral groove. The broad muscle flattens to cover the anterior part of the elbow joint and is inserted by mixed tendon and muscle fibres into the coronoid process and tuberosity of the ulna.

26

Number: 336 Which artery and region that it primarily supplies are incorrectly paired: A. Right gastroepiploic : greater curvature B. Left gastric cardiac part of stomach C. Right colic ascending colon D. Right gastric : pyloris of stomach E. Middle colic: descending colon Select the single best answer ABCDE Correct Answer: E The middle colic supplies the transverse colon.

Number: 336 Which artery and region that it primarily supplies are incorrectly paired: A. Right gastroepiploic : greater curvature B. Left gastric cardiac part of stomach C. Right colic ascending colon D. Right gastric : pyloris of stomach E. Middle colic: descending colon Select the single best answer ABCDE Correct Answer: E The middle colic supplies the transverse colon. Number: 337 The recurrent (or inferior) laryngeal nerve innervates all the intrinsic laryngeal muscles, EXCEPT: A. Lateral cricoarytenoid B. Posterior cricoarytenoid C. Cricothyroid D. Vocalis E. Aryepiglottis Select the single best answer

27

ABCDE Correct Answer: C Cricothyroid Cricothyroid arises from the arch of the cricoid backwards to fan out towards its attachment to the inferior horn and lower border of the thyroid lamina. Its contraction causes the arch of the cricoid and the Adam's apple to approach each other. The effect of contraction is to lengthen the vocal fold. Number: 338 With regard to Pectoralis Major: 1. It is supplied by the pectoral nerves. 2. It can abduct and laterally rotate the humerus. 3. Its contraction is used in testing the mobility of a breast lump. 4. Its lower border forms the posterior axillary fold. A: 1,2,3 Correct B: 1,3 Correct C: 2,4 Correct D: 4 Correct E: All Correct ABCDE Correct Answer: B Pectoralis Major is a medial rotator of the arm. In combination with latissimus dorsi it is also a powerful adductor of the arm. Number: 339 The ulnar nerve: 1. Supplies flexor carpi ulnaris muscle. 2. Supplies abductor pollicis brevis. 3. Arises from medial cord of the brachial plexus 4. Supplies the 2nd lumbrical muscle. A: 1,2,3 Correct B: 1,3 Correct C: 2,4 Correct D: 4 Correct E: All Correct

28

ABCDE Correct Answer: B In 95% of cases flexor carpi ulnaris is supplied by the ulnar nerve. The muscles of the thenar eminence (abductor pollicis brevis, flexor pollicis brevis and opponens pollicis) are supplied by the median nerve. The ulnar nerve is the largest branch of the medial cord of the brachial plexus. The 1st and 2nd lumbricals are supplied by the median nerve. Number: 356 In this plan of the right lumbar plexus, The nerve marked 'A' is: A. Ilio-Hypogastric. B. Lateral cutaneous nerve of the thigh. C. Femoral. D. Obturator. E. Genitofemoral Select the single best answer

29

ABCDE Correct Answer: D The lumbar plexus is derived from the anterior primary rami of the 1st, 2nd, 3rd and part of the 4th lumbar nerve roots. About 50 percent of subjects receive an additional contribution from TI2. In much the same way as the brachial plexus, the lumbar plexus may be prefixed, with its lowest contribution from L3, or postfixed, when it extends to L5. The plexus assembles in front of the transverse processes of the lumbar vertebrae within the substance of the psoas major. LI, joined in 50 per cent of cases by a branch from TI2, divides into an upper and lower division. The upper division gives rise to the iliohypogastric and ilioinguinal nerves; the lower joins a branch from L2 to form the genito-femoral nerve. The rest of L2, together with L3 and the contribution to the plexus from L4, divide into dorsal and ventral divisions. Dorsal divisions L2 and 3 form the lateral cutaneous nerve of the thigh and L2, 3 and 4 form the femoral nerve. The ventral branches join into the obturator nerve (L2, 3, 4) and, when present, the accessory abturator nerve (L3, 4). 30

Number: 357 In this plan of the right lumbar plexus, The nerve marked 'B' is: A. Ilio-Hypogastric. B. Lateral cutaneous nerve of the thigh. C. Femoral. D. Obturator. E. Genitofemoral Select the single best answer

ABCDE Correct Answer: C The lumbar plexus is derived from the anterior primary rami of the 1st, 2nd, 3rd and part of the 4th lumbar nerve roots. About 50 percent of subjects receive an additional contribution from TI2. In much the same way as the brachial plexus, the lumbar plexus may be prefixed, with its lowest contribution from L3, or postfixed, when it extends to L5. The plexus assembles in front of the transverse processes of the lumbar vertebrae within the substance of the psoas major. LI, joined in 50 per cent of cases by a branch from TI2, divides into an upper and lower division. The upper division gives rise to the iliohypogastric and ilioinguinal nerves; the lower joins a branch from L2 to form the genito-femoral nerve. The rest of L2, together with L3 and the contribution to the plexus from L4, divide into dorsal and ventral divisions. Dorsal divisions L2 and 3 form the lateral cutaneous nerve of the thigh and L2, 3 and 4 form the femoral nerve. The ventral branches join into the obturator nerve (L2, 3, 4) and, when present, the accessory abturator nerve (L3, 4). Number: 358 In this plan of the right lumbar plexus, The nerve marked 'C' is: A. Ilio-Hypogastric. B. Lateral cutaneous nerve of the thigh. C. Femoral. D. Obturator. E. Genitofemoral Select the single best answer

31

ABCDE Correct Answer: B The lumbar plexus is derived from the anterior primary rami of the 1st, 2nd, 3rd and part of the 4th lumbar nerve roots. About 50 percent of subjects receive an additional contribution from TI2. In much the same way as the brachial plexus, the lumbar plexus may be prefixed, with its lowest contribution from L3, or postfixed, when it extends to L5. The plexus assembles in front of the transverse processes of the lumbar vertebrae within the substance of the psoas major. LI, joined in 50 per cent of cases by a branch from TI2, divides into an upper and lower division. The upper division gives rise to the iliohypogastric and ilioinguinal nerves; the lower joins a branch from L2 to form the genito-femoral nerve. The rest of L2, together with L3 and the contribution to the plexus from L4, divide into dorsal and ventral divisions. Dorsal divisions L2 and 3 form the lateral cutaneous nerve of the thigh and L2, 3 and 4 form the femoral nerve. The ventral branches join into the obturator nerve (L2, 3, 4) and, when present, the accessory abturator nerve (L3, 4).

Number: 359 In this view of the dorsal surface of the left first rib, the structure crossing the rib at point 'A' is: A. The Subclavian vein. B. The Subclavian artery. C. The Brachial plexus. D. The Thoracic duct. E. The Scalenus Anterior. Select the single best answer

32

ABCDE Correct Answer: A The upper surface of the shaft of the 1st rib slopes down at about 45 degrees, and is at the root of the neck. It is grooved obliquely at its greatest lateral convexity. The groove between 'B' and 'C' is called the subclavian groove. It lodges the lower trunk of the brachial plexus. The fibres in contact with the rib are all from T1, and the C8 fibres lie above them, not yet intermingled. The subclavian artery has its upward convexity Iying more transversely. The artery does not lie in the groove, and it touches only the outer border of the rib. Between the groove and the tubercle the large quadrangular area of the upper surface gives attachment to scalenus anterior. Number: 360 In this view of the dorsal surface of the left first rib, the muscle inserted at point 'B' is: A. Scalenus Medius. B. Scalenus Posterior. C. Posterior belly of Omo-hyoid. D. Scalenus Anterior. E. None of the above. Select the single best answer 33

ABCDE Correct Answer: D The upper surface of the shaft of the 1st rib slopes down at about 45 degrees, and is at the root of the neck. It is grooved obliquely at its greatest lateral convexity. The groove between 'B' and 'C' is called the subclavian groove. It lodges the lower trunk of the brachial plexus. The fibres in contact with the rib are all from T1, and the C8 fibres lie above them, not yet intermingled. The subclavian artery has its upward convexity Iying more transversely. The artery does not lie in the groove, and it touches only the outer border of the rib. Between the groove and the tubercle the large quadrangular area of the upper surface gives attachment to scalenus anterior.

Number: 363 In this view of the dorsal surface of the left first rib, the muscle inserted at point 'C' is: A. Scalenus Medius. B. Scalenus Posterior. C. Posterior belly of Omo-hyoid. D. Scalenus Anterior. E. None of the above. Select the single best answer

ABCDE Correct Answer: A The scalenus medius: The scalenus medius arises from the posterior tubercles and costo-transverse lamellae of all the cervical vertebrae and is inserted into the quadrangular area between the neck and subclavian groove of the first rib . Number: 365 With regard to the brachial plexus the nerve labelled 'A' is: A. Lateral Pectoral. B. Musculocutaneous. C. Suprascapular. D. Median E. Nerve to subclavius

34

Select the single best answer

ABCDE Correct Answer: B The musculocutaneous nerve: The musculocutaneous nerve (C5, 6, 7) is the continuation of the lateral cord after this has given off the lateral head of the median nerve at the lower border of pectoralis minor. Because of its derivation from the lateral cord, the nerve naturally lies lateral to the axillary artery. It first supplies and then pierces coracobrachialis, then descends downwards and laterally between biceps and brachialis, supplying both these muscles. The nerve emerges between biceps tendon and brachioradialis, pierces the deep fascia of the antecubital fossa and continues downwards as the lateral cutaneous nerve of the forearm.

35

Number: 366 With regard to the brachial plexus the nerve labelled 'B' is: A. Lateral Pectoral. B. Musculocutaneous. C. Suprascapular. D. Median E. Nerve to subclavius Select the single best answer ABCDE Correct Answer: D The median nerve: The median nerve (C5, 6, 7, 8, TI) carries fibres from all the roots of the brachial plexus; C5, 6 and 7 from the lateral head, which is derived from the lateral cord of the plexus, and C8 and TI from the medial head, derived from the medial cord. The two origins of the nerve unite in front of the third part of the axillary artery. The nerve descends through the arm first on the lateral side of the brachial artery, then on its medial side, crossing the artery at the mid-point of the upper arm at the insertion of coracobrachialis. Usually the nerve passes across the front of the artery, but occasionally crosses behind it. Number: 367 With regard to the brachial plexus the nerve labelled 'E' is: A. Phrenic B. Nerve to Serratus Anterior C. Suprascapular. D. Nerve to rhomboids E. Nerve to subclavius Select the single best answer ABCDE Correct Answer: B Nerve to Serratus Anterior: The long thoracic nerve (nerve to serratus anterior) arises from the roots of the brachial plexus (C5, 6, 7). The branches from C5 and 6 join in the scalenus medius muscle and emerge from its lateral border as a single trunk which enters the axilla by passing over the first digitation of serratus anterior. The contribution from C7 also passes over the first digitation of serratus

36

anterior and joins the former nerve on the medial wall of the axilla (i.e., on the surface of serratus anterior) to form the nerve to serratus anterior. The nerve lies behind the midaxillary line (i.e., behind the lateral branches of the intercostal arteries) on the surface of the muscle, deep to the fascia, and is thus protected in operations on the axilla. The muscle is supplied segmentally; C5 into the upper two digitations, C6 into the next two, and C7 into the lower four digitations. Number: 381 The structure marked 'A' on this lateral view of the pharynx is: A. Superior Constrictor. B. Middle Constrictor. C. Inferior Constrictor. D. Hyoglossus. E. Buccinator. Select the single best answer

ABCDE

37

Correct Answer: C Inferior Constrictor: The inferior constrictor, which is the thickest of the pharyngeal constrictors, arises from the side of the cricoid, from the tendinous arch over the cricothyroid muscle and from the oblique line on the lamina of the thyroid cartilage. The muscle consists functionally of two parts: the lower portion, arising from the cricoid (the cricopharyageus), which acts as a sphincter and the upper portion, with obliquely placed fibres which arise from the thyroid cartilage, which has a propulsive action. Number: 383 The structure marked 'C' on this lateral view of the pharynx is: A. Superior Constrictor. B. Middle Constrictor. C. Inferior Constrictor. D. Hyoglossus. E. Buccinator. Select the single best answer ABCDE Correct Answer: D Hyoglossus: The hyoglossus muscle arises from the length of the greater horn of the hyoid bone and from the body of that bone lateral to genio-hyoid. It extends as a quadrilateral sheet on the side of the tongue; its upper border, interdigitating at right angles with the fibres of stylo-glossus, is attached to the side of the tongue. Number: 387 The structure marked 'B' on this lateral view of the pharynx is: A. Superior Constrictor. B. Middle Constrictor. C. Inferior Constrictor. D. Hyoglossus. E. Buccinator. Select the single best answer ABCDE Correct Answer: A

38

The Superior Constrictor arises from the medial pterygoid plate and is inserted into the pharyngeal ligament. Number: 395 The phrenic nerve: 1. Arises predominantly from the third cervical nerve 2. Runs in front of the root of the lung 3. Is a purely motor nerve 4. Iinnervates the diaphragm from below A: 1,2,3 Correct B: 1,3 Correct C: 2,4 Correct D: 4 Correct E: All Correct ABCDE Correct Answer: C The phrenic nerve (C3, 4, 5) is, the most important branch of the cervical plexus. It provides the motor innervation of the diaphragm (apart from a clinically insignificant contribution to the crura from T11 and T12) and transmits proprioceptive sensory fibres from the central part of the diaphragm. In addition filaments are supplied to the pleura and pericardium. Number: 419 Which artery is NOT a branch of the internal carotid artery? A. Middle cerebral B. Anterior cerebral. C. Posterior cerebral. D. Choroidal. E. Retinal. Select the single best answer ABCDE Correct Answer: C The posterior cerebral arteries are terminal branches of the basilar artery.

39

Number: 420 Afferent fibres from the carotid sinus travel via the: A. Glossopharyngeal nerve. B. Vagus nerve. C. Cervical sympathetics. D. Recurrent laryngeal nerve. E. Accessory nerve. Select the single best answer ABCDE Correct Answer: A The sinu-carotid nerve (a branch of the glosspharyngeal nerve) supplies the carotid sinus and carotid body. The fibres pass centrally to the vasomotor centre. Number: 421 The radial artery : 1. Lies between flexor carpi radialis and brachioradialis at the wrist. 2. Passes beneath the flexor retinaculum. 3. Accompanies superficial branches of the radial nerve at the wrist. 4. Has no branches in the forearm. A: 1,2,3 Correct B: 1,3 Correct C: 2,4 Correct D: 4 Correct E: All Correct ABCDE Correct Answer: B The radial artery has a recurrent branch at the elbow which anastomoses with branches from the ulnar and interosseous arteries. It also supplies muscular branches to all the muscles which it passes. The flexor retinaculum contains the median nerve and flexor tendons of the thumb and fingers.

40

Number: 432 The inferior vena cava: 1. Is formed at the 5th lumbar vertebral level. 2. Passes through the tendinous part of the diaphragm. 3. Passes behind the horizontal part of the duodenum. 4. Has a valve at its opening in the right atrium. A: 1,2,3 Correct B: 1,3 Correct C: 2,4 Correct D: 4 Correct E: All Correct ABCDE Correct Answer: E The inferior vena cave is a large, valveless, venous trunk that receives the blood from the lower limbs, and much of the blood from the back and from the walls and contents of the abdomen and pelvis It is formed by the junction of the two common iliac veins, slightly below and to the right of the bifurcation of the aorta. It ascends at the right of the aorta, through the central tendon of the diaphragm, and empties into the right atrium. From below upward it lies behind peritoneum (crossed by the root of the mesentery and right gonadal vessels), duodenum and pancreas, portal vein, epiploic foramen, and the liver, The right renal artery crosses behind it. The tributaries of the inferior vena cava are the common iliac, gonadal, renal, suprarenal, inferior phrenic, lumbar, and hepatic veins. Number: 433 With regard to the foetal circulation: 1. Most of the blood bypasses the liver in the ductus venosus. 2. The ductus arteriosus connects the root of the right pulmonary trunk to the descending aorta. 3. The valve of the inferior vena cava helps to direct the blood through the foramen ovale. 4. The umblical arteries arises from the external iliac artery. A: 1,2,3 Correct B: 1,3 Correct C: 2,4 Correct D: 4 Correct E: All Correct

41

ABCDE Correct Answer: B The economy of the foetal circulation is improved by three short-circuiting arrangements, all of which cease to function at the time of birth. The three short-circuiting structures are the ductus venosus, the forarnen ovale and the ductus arteriosus. The Ductus Venosus. Oxygenated blood returns from the placenta by the (left) umbilical vein, which joins the left branch of the portal vein in the porta hepatis. This oxygenated blood short-circuits the sinusoids of the liver; it is conveyed directly to the inferior vena cava by a channel called the ductus venosus. The ductus venosus lies along the inferior surface of the liver, between the attached layers of the lesser omentum. After birth, when blood no longer flows along the thrombosed umbilical vein, the blood in the ductus venosus clots and the ductus venosus becomes converted into a fibrous cord, the ligamentum venosum, lying deep in the cleft bounding the caudate lobe of the liver. The intra-abdominal part of the umbilical vein persists as a fibrous cord, the ligamentum teres. The two are continuous. The Foramen Ovale. The interatrial septum of the foetal heart is patent, being perforated by the foramen ovale. Blood brought to the right atrium by the inferior vena cava is directed by its 'valve' through the aperture in the inter-atrial septum and so enters the left atrium. The oxygenated placental blood is thus made to by-pass the right ventricle and the airless lungs, and is directed into the left ventricle and aorta and so to the carotid arteries. After birth this aperture, the foremen ovale, is closed by approximation and overlap of flanges of cardiac muscle which act from below and above like shutters. The two flanges are the septum primum and the septum secundum. The two flanges overlap and adhere together, so closing the interatrial septum. The site of union is marked by a shallow depression, the fossa ovalis, in the right side of the interatrial septum. After closure of the foramen ovale all the blood in the right atrium perforce passes into the right ventricle and so to the lungs. The Ductus Arteriosus. It has already been noted that oxygenated blood in the umbilical vein passes via the ductus venosus, inferior vena cava and right atrium through the foremen ovale to the left side of the heart locally, of a raised oxygen tension. and so to the head. Venous blood from the head is returned by way of the brachio-cephalic veins to the superior vena cava. In the right atrium this venous blood stream crosses the stream of oxygenated blood brought there via the inferior vena cava. The two streams of blood scarcely mix with each other. The de-oxygenated blood from the superior vena cave passes through the right atrium into the right ventricle and so into the pulmonary trunk. It now short-circuits the airless lungs by the ductus arteriosus. This is a thick artery joining the left branch of the pulmonary trunk to the aorta, distal to the origin of the three branches of the aortic arch. The de-oxygenated blood thus passes distally along the aorta and via the umbilical arteries to the placenta to be re-oxygenated. After birth the ductus arteriosus is occluded by contraction of its muscular walls. It persists as a fibrous band, the ligamentum arteriosum, which connects the commencement of the left pulmonary artery to the concavity of the arch of the aorta. After the closure of the ductus arteriosus blood from the right ventricle perforce circulates through the lungs.

42

Number: 436 In this diagram of the cervical plexus, the nerve labelled 'A' is: A. Phrenic. B. Supraclavicular. C. Descendens cervicalis. D. Descendens hypoglossi. E. Anterior cutaneous nerve of the neck. Select the single best answer ABCDE Correct Answer: A Phrenic. The phrenic nerve (C3, 4, 5) is the most important branch of the cervical plexus. It provides the motor innervation of the diaphragm (apart from a clinicallyinsignificant contribution to the crura from T11 and T12) and transmits proprioceptive sensory fibres from the central part of the diaphragm. In addition filaments are supplied to the pleura and pericardium. The principal component of the nerve is derived from the anterior primary ramus of C4 but contributions are also provided from C3 and C5. The three roots of the nerve join at the lateral border of scalenus anterior and then the fully constituted nerve runs downwards and medially across the anterior face of the muscle, covered by, and showing through, the prevertebral fascia. On scalenus anterior the phrenic nerve is overlapped by the internal jugular vein and sternomastoid, and is crossed by the inferior belly of the omohyoid and by the transverse cervical and transverse scapular vessels. On the left side, in addition, the nerve is crossed by the thoracic duct. Number: 437 In this diagram of the cervical plexus, the nerve labelled 'B' is: A. Phrenic. B. Supraclavicular. C. Descendens cervicalis. D. Descendens hypoglossi. E. Anterior cutaneous nerve of the neck. Select the single best answer

43

ABCDE Correct Answer: C Descendens Cervicalis. The ansa cervicalis lies on the front of the internal jugular vein and gives branches to the infrahyoid muscles. It is formed by union of superior and inferior rami. The superior ramus (descendens hypoglossi) is a branch of the hypoglossal nerve given off where the nerve loops just below the posterior belly of the digastric muscle, on the occipital, external carotid and lingual arteries. It runs down on the front of the internal jugular vein. It contains only C1 fibres, which have hitch-hiked along the hypoglossal nerve. The inferior ramus (descendens cervicalis) is formed by union of a branch each from C2 and C3 in the cervical plexus. The single nerve so formed spirals from behind around the internal jugular vein and runs down to join the superior ramus at a variable level. Sometimes a wide loop is formed over the lower part of the vein and the branches arise from the loop. Sometimes the two nerves join, Y-shaped, high up and the branches are given off from the stem of the Y. In either case they are distributed to the infrahyoid muscles (sterno-hyoid, stereo-thyroid and omo-hycid) segmentally, Cl, C2 and C3 from above down. Number: 438 In this diagram of the cervical plexus, the nerve labelled 'C' is: A. Phrenic. B. Supraclavicular. C. Descendens cervicalis. D. Descendens hypoglossi. E. Anterior cutaneous nerve of the neck. Select the single best answer ABCDE Correct Answer: D Descendens hypoglossi. The ansa cervicalis lies on the front of the internal jugular vein and gives branches to the infrahyoid muscles. It is formed by union of superior and inferior rami. The superior ramus (descenders hypoglossi) is a branch of the hypoglossal nerve given off where the nerve loops just below the posterior belly of the digastric muscle, on the occipital, external carotid and lingual arteries. It runs down on the front of the internal jugular vein. It contains only C1 fibres, which have hitch-hiked along the hypoglossal nerve. The inferior ramus (descendens cervicalis) is formed by union of a branch each from C2 and C3 in the cervical plexus. The single nerve so formed spirals from behind around the internal

44

jugular vein and runs down to join the superior ramus at a variable level. Sometimes a wide loop is formed over the lower part of the vein and the branches arise from the loop. Sometimes the two nerves join, Y-shaped, high up and the branches are given off from the stem of the Y. In either case they are distributed to the infrahyoid muscles (sterno-hyoid, stereo-thyroid and omo-hycid) segmentally, Cl, C2 and C3 from above down. Number: 439 At birth, the spinal cord is most likely to terminate at: A. L1. B. L2. C. L3. D. L4. E. L5. Select the single best answer ABCDE Correct Answer: C L3. By adulthood it has migrated upwards to the L1-2 region. Number: 440 Which of the following nerves contribute to the supply of the external auditory meatus: 1. Auriculo-temporal. 2. Great auricular. 3. Auricular branch of the vagus. 4. Lesser occipital. A: 1,2,3 Correct B: 1,3 Correct C: 2,4 Correct D: 4 Correct E: All Correct ABCDE Correct Answer: B The nerve supply of the external auditory meatus is from the auriculo-temporal nerve overlapped by the facial nerve and the auricular branch of the vagus.

45

The skin of the auricle is supplied by the great auricular and the auriculo-termporal nerves. The great auricular nerve supplies the whole of the cranial surface and the lateral surface below the meatus with fibres from C2. The auriculo-temporal nerve supplies the outer surface of the tympanic membrane, the external acoustic meatus and the skin of the auricle above this level. The auricular branch of the vagus supplies the posteroinferior quadrant of the tympanic membrane and skin of the adjoining meatus and a small area of skin of the cranial surface near the mastoid. The facial nerve by branches from the tympanic plexus also supplies the cutaneous surface of the tympanic membrane and external meatus, and these areas show vesicles in cases of facial herpes. The lesser occipital nerve (C2) overlaps the great auricular nerve at the upper margin of the cranial surface. The lesser occipital nerve (C2) is a slender branch that hooks around the accessory nerve and runs up along the posterior border of stereo-mastoid to supply the posterior part of the neck below the superior nuchal line (i.e., over the upper part of stereo-mastoid). It may overlap to the tip of the auricle. Number: 471 The greater splanchnic nerves usually synapse in the: A. Superior mesenteric ganglion B. Coeliac ganglion C. Hypogastric plexus D. Inferior mesenteric ganglion E. Ganglia of the lower thoracic sympathetic trunk Select the single best answer ABCDE Correct Answer: B The greater splachnic nerves arises from T5-T9 and pass forwards and downwards on the sides of the vertebral bodies. They pierce the crus of the diaphragm and then join the coeliac ganglion. Number: 474 The sixth cranial nerve: 1. Supplies the superior oblique muscle 2. Passes through the superior orbital fissure 3. Innervates the lacrimal gland 4. May be involved in an injury to the petrous part of the temporal bone

46

A: 1,2,3 Correct B: 1,3 Correct C: 2,4 Correct D: 4 Correct E: All Correct ABCDE Correct Answer: C The abducent nerve, like the trochlear nerve, supplies only one eye muscle, the lateral rectus.Its nucleus is part of the somatic efferent column and lies immediately deep to the floor of the 4th ventricle in the upper part of the pons. From this nucleus, fibres pass through the pontine tegmentum to emerge on the base of the brain at the junction of the pons and medulla. The nerve then passes forwards to enter the cavernous sinus. Here it lies lateral to the internal carotid artery and medial to the IIIrd, IVth and Vth cranial nerves. Passing through the tendinous ring just below the IIIrd nerve, it enters the orbit to pierce the deep surface of the lateral rectus. Because of its long and oblique intracranial course, the Vith nerve is frequently involved in basal skull injuries. If damaged, diplopia and a covergent squint are the result. Number: 514 Concerning a typical intercostal nerve: 1. It arises from the posterior primary ramus of a thoracic spinal nerve. 2. It lies superior to the artery in the costal groove. 3. It lies between the internal and external intercostal muscles. 4. It is both motor and sensory. A: 1,2,3 Correct B: 1,3 Correct C: 2,4 Correct D: 4 Correct E: All Correct ABCDE Correct Answer: D The intercostal nerves are the primary anterior rami of the thoracic spinal nerves. They are mixed, motor (to the intercostal muscle) and sensory (serving skin). From above downwards in the costal groove lie the intercostal vein, artery and nerve. The vein, artery and nerve lie between the internal intercostal and the transverse thoracis muscles.

47

Number: 534 In this posterior view of the right popliteal fossa, the structure labelled 'A' is: A. Biceps Femoris. B. Semimembranosus. C. Semitendinosus. D. Gracilis. E. Sartorius. Select the single best answer ABCDE Correct Answer: A Biceps Femoris. - The long head originates from a common tendon with semitendinosus from the superior medial quadrant of the posterior portion of the ischial tuberosity; and the short head from the lateral lip of the linea aspera, lateral supracondylar ridge of femur, and lateral intermuscular septum of the thigh. It is inserted into the fibular head; also the lateral collateral ligament and lateral tibial condyle. Its action is to flex the knee, and also rotate the tibia laterally; the long head also extends the hip joint. The long head is innervated by tibial nerve; and the short head by common peroneal nerve. Its arterial supply is from perforating branches of the profunda femoris artery, the inferior gluteal artery, and superior muscular branches of the popliteal artery.

48

Number: 535 In this posterior view of the right popliteal fossa, the structure labelled 'B' is: A. Tibial N. B. Lateral Cutaneous N. of the calf. C. Sural Communicating N. D. Posterior Tibial N. E. Sural N. Select the single best answer ABCDE Correct Answer: A Tibial N. Number: 536 In this posterior view of the right popliteal fossa, the structure labelled 'C' is: A. Tibial N. B. Lateral Cutaneous N. of the calf. C. Sural Communicating N. D. Posterior Tibial N. E. Sural N. Select the single best answer 49

ABCDE Correct Answer: C Sural Communicating N. Number: 537 In this posterior view of the right popliteal fossa, the structure labelled 'D' is: A. Biceps Femoris. B. Semimembranosus. C. Semitendinosus. D. Gracilis. E. Sartorius. Select the single best answer ABCDE Correct Answer: C Semitendinosus originates from a common tendon with long head of biceps femoris on the superior medial quadrant of the posterior portion of the ischial tuberosity. It is inserted into the superior aspect of the medial portion of tibial shaft. Its action is to extend the thigh, flex the knee, and rotate the tibia medially - especially when the knee is flexed. It is innervated by the tibial nerve and draws its blood supply from perforating branches of profunda femoris, the inferior gluteal artery, and superior muscular branches of the popliteal artery. Number: 538 In this posterior view of the right popliteal fossa, the structure labelled 'E' is: A. Biceps Femoris. B. Semimembranosus. C. Semitendinosus. D. Gracilis. E. Sartorius. Select the single best answer ABCDE Correct Answer: B Semimembranosus takes origin from the superior lateral quadrant of the ischial tuberosity and is inserted onto the posterior surface of the medial tibial condyle. It acts to extend the thigh, flex the knee, and rotate the tibia medially, especially when the knee is flexed.

50

It is innervated by the tibial nerve and derives its arterial supply from perforating branches of the profunda femoris artery, inferior gluteal artery, and the superior muscular branches of the popliteal artery. Number: 577 Which of the following vertebrae has the most prominent spinous process? A. T1. B. T2. C. C7. D. T11. E. T12. Select the single best answer ABCDE Correct Answer: C C7 - 'Vertebra Prominens'. Number: 586 The femoral sheath: 1. Contains the femoral nerve, artery and vein 2. Has the inguinal ligament as its anterior border 3. Has the lacunar ligament as its lateral border 4. Has the pectineal ligament as its posterior border A: 1,2,3 Correct B: 1,3 Correct C: 2,4 Correct D: 4 Correct E: All Correct ABCDE Correct Answer: C The femoral nerve lies outside the femoral sheath which contains the artery, vein and lymphatics. The lacunar ligament is on the medial border of the femoral sheath.

51

Number: 712 In the case of a patient sitting upright with his arms by his side, a line drawn between the tips of the scapulae will correspond to the vertebral body of: A. T6. B. T7. C. T8. D. T9. E. T10. Select the single best answer ABCDE Correct Answer: B Vertebra Prominens is C7. A line drawn between the tips of the scapulae corresponds to the vertebral body of T7. A line drawn between the superior margin of the iliac crests is level with the vertebral body of L4. Number: 748 A patient presents with a history of low back pain and sciatica. The pain radiates to the little toe, the ankle reflex is absent and the patient has difficulty in everting the foot. Which nerve root is likely to be trapped? A. L3 B. L4 C. L5 D. S1 E. S2 Select the single best answer ABCDE Correct Answer: D The root supply to the peroneal muscles (which control eversion of the foot and which also participate in the the reflex arc of the ankle jerk reflex) is S1 via the tibial and superficial peroneal nerves. The sensory dermatome of the S1 root gives innervation to the postero-lateral aspect of the leg and foot down to and including the little toe and sole of foot.

52

Number: 777 Which of the following are branches of the MEDIAL CORD of the brachial plexus? 1. Ulnar Nerve. 2. Axillary Nerve. 3. Medial Pectoral Nerve. 4. Radial Nerve. A: 1,2,3 Correct B: 1,3 Correct C: 2,4 Correct D: 4 Correct E: All Correct ABCDE Correct Answer: B The axillary nerve is a branch of the posterior cord. The radial nerve is a branch of the posterior cord. Number: 778 Which of the following are branches of the POSTERIOR CORD of the brachial plexus? 1. Dorsal scapular nerve. 2. Nerve to serratus anterior. 3. Nerve to subclavius. 4. Upper subscapular nerve. A: 1,2,3 Correct B: 1,3 Correct C: 2,4 Correct D: 4 Correct E: All Correct ABCDE Correct Answer: D The dorsal scapular nerve, nerve to serratus anterior and the nerve to subclavius all arise from the roots of the brachial plexus.

53

Number: 779 Which of the following is a branch of the LATERAL CORD of the brachial plexus? A. Suprascapular nerve. B. Lower subscapular nerve. C. Medial pectoral nerve. D. Musculo-cutaneous nerve. E. Upper subscapular nerve. Select the single best answer ABCDE Correct Answer: D The suprascapular nerve arises from the upper trunk of the plexus. The upper and lower subscapular nerves arise from the posterior cord of the plexus. The medial pectoral nerve arise from the medial cord of the plexus. Number: 779 Which of the following is a branch of the LATERAL CORD of the brachial plexus? A. Suprascapular nerve. B. Lower subscapular nerve. C. Medial pectoral nerve. D. Musculo-cutaneous nerve. E. Upper subscapular nerve. Select the single best answer ABCDE Correct Answer: D The suprascapular nerve arises from the upper trunk of the plexus. The upper and lower subscapular nerves arise from the posterior cord of the plexus. The medial pectoral nerve arise from the medial cord of the plexus.

54

Number: 831 The right middle lobe bronchus: 1. Divides into superior and inferior segmental bronchi. 2. Is medial to the right middle lobe artery. 3. Arises from the posterior aspect of the right main bronchus. 4. Is about 1.5cms in length. A: 1,2,3 Correct B: 1,3 Correct C: 2,4 Correct D: 4 Correct E: All Correct ABCDE Correct Answer: C The right middle lobe bronchus divides into medial and lateral segmental bronch. It is about 1.5cms in length and arises from the anterior aspect of the right main bronchus. The right middle lobe artery runs on its lateral side and the right middle lobe vein on its medial side. See: Anesthesia for Thoracic Surgery.2nd ed.Jonathan L. Benumof. ISBN: 0721644678. Publisher: WB Saunders. Number: 832 In the left lung, the number of bronchopulmonary segments is: A. 7. B. 8. C. 9. D. 10. E. 11. Select the single best answer ABCDE Correct Answer: B This is a 'trick' question. In one sense, there are 10 bronchopulmonary segments in the left lung, but the apical and posterior segments of the upper lobe and the anterior and medial basal segments of the lower lobe both combine into single segments. Hence the correct answer is 8. These are: Upper lobe: Apico-posterior

55

Anterior Lingula: Superior Inferior Lower Lobe: Superior ('Apical') Anterior-medial basal Lateral basal Postero-basal See: Anesthesia for Thoracic Surgery.2nd ed.Jonathan L. Benumof. ISBN: 0721644678. Publisher: WB Saunders. Number: 833 In the right lung, the number of bronchopulmonary segments is: A. 7. B. 8. C. 9. D. 10. E. 11. Select the single best answer ABCDE Correct Answer: D There are 10 bronchopulmonary segments in the right lung. These are: Upper lobe: Apical Posterior Anterior Middle Lobe: Medial Lateral Lower Lobe: Superior ('Apical') Anterior basal Posterior basal Medial basal

56

Lateral basal See: Anesthesia for Thoracic Surgery.2nd ed.Jonathan L. Benumof. ISBN: 0721644678. Publisher: WB Saunders. Number: 854 The incidence of probe-patent foramen ovale (PFO) in the general population is approximately: A. 5%. B. 10%. C. 15%. D. 20%. E. 25%. Select the single best answer ABCDE Correct Answer: E The incidence of PFO in the normal population is approximately 25%, as determined by postmortem examination. See: Hagen PT, Scholz DG, Edwards WD. Incidence and size of patent foramen ovale during the first 10 decades of life: an autopsy study of 965 normal hearts. Mayo Clin Proc 1984; 59: 17 20. Sweeney LJ, Rosenquist GC. The normal anatomy of the atrial septum in the human heart. Am Heart J 1979; 98: 1949. Number: 857 Abduction of the vocal cords is achieved by contraction of the: A. Posterior cricoarytenoids. B. Lateral cricoarytenoids. C. Interarytenoids. D. Cricothyroids. E. Thyroarytenoids. Select the single best answer ABCDE Correct Answer: A

57

The posterior cricoarytenoids are the (only) cord abductors. They are innervated by the recurrent laryngeal nerve. Number: 858 Adduction of the vocal cords is achieved by contraction of the: A. Posterior cricoarytenoids. B. Lateral cricoarytenoids. C. Aryepiglottics. D. Cricothyroid. E. Thyroarytenoid. Select the single best answer ABCDE Correct Answer: B The lateral cricoarytenoids (and the interarytenoids) are the cord adductors. The lateral cricoarytenoids are innervated by the recurrent laryngeal nerve, while the interarytenoids are innervated by both the recurrent and superior laryngeal nerves. Number: 860 In this posterior view of the intrinsic muscles of the larynx, the muscle labelled 'A' is: A. Interarytenoid. B. Posterior cricoarytenoid. C. Aryepiglottic. D. Thyroarytenoid. E. Thyroepiglottic. Select the single best answer

58

ABCDE Correct Answer: C Muscle 'A' is the Aryepiglottic. It is innervated by the recurrent laryngeal nerve.

Number: 861 In this posterior view of the intrinsic muscles of the larynx, the muscle labelled 'B' is: A. Interarytenoid. B. Posterior cricoarytenoid. C. Aryepiglottic. D. Thyroarytenoid. E. Thyroepiglottic. Select the single best answer ABCDE Correct Answer: A Muscle 'B' is the transverse component of the Interarytenoid. It is innervated by the recurrent laryngeal nerve and probably by branches from the superior laryngeal nerve.

59

Number: 861 In this posterior view of the intrinsic muscles of the larynx, the muscle labelled 'B' is: A. Interarytenoid. B. Posterior cricoarytenoid. C. Aryepiglottic. D. Thyroarytenoid. E. Thyroepiglottic. Select the single best answer ABCDE Correct Answer: A Muscle 'B' is the transverse component of the Interarytenoid. It is innervated by the recurrent laryngeal nerve and probably by branches from the superior laryngeal nerve. Number: 862 In this posterior view of the intrinsic muscles of the larynx, the muscle labelled 'C' is: A. Interarytenoid. B. Posterior cricoarytenoid. C. Aryepiglottic. D. Thyroarytenoid. E. Thyroepiglottic. Select the single best answer ABCDE Correct Answer: B Muscle 'C' is the posterior cricoarytenoid. It is innervated by the recurrent laryngeal nerve.

Number: 879 Klumpke's paralysis may be associated with: 1. Wasting of the small muscles of the hand. 2. Horner's syndrome. 3. Sensory loss on the medial side of the arm. 4. An inability to extend the metacarpophalangeal joints.

60

A: 1,2,3 Correct B: 1,3 Correct C: 2,4 Correct D: 4 Correct E: All Correct ABCDE Correct Answer: A Klumpke's paralysis is caused by upward traction on the arm (as may occur in forcible breech delivery or, during anaesthesia, by wide abduction of the limb when an arm board is used) which results in damage to the lowest root of the brachial plexus (T1). This provides the segmental supply of the intrinsic muscles of the hand. The hand assumes a clawed appearance because of the unopposed actions of the long flexors and extensors of the fingers. (The extensors, which are inserted into the bases of the proximal phalanges, extend the metacarpophalangeal joints; flexor profundus and sublimis, inserted respectively into the distal and middle phalanges, flex the interphalangeal joints.) An area of sensory loss may be present on the inner aspect of the arm and upper forearm. Traction may also tear the white ramus communicans from T1 to the stellate ganglion, so that there may be an associated Horner's syndrome. Number: 934 With respect to the veins of the upper limb found at the cubital fossa: 1. The basilic vein lies occupies a lateral position. 2. The cephalic vein accompanies the brachial artery above the cubital fossa. 3. Generally, the most prominent vein in the cubital fossa is the cephalic. 4. The aponeurosis of the biceps muscle is the non-vascular structure most commonly encountered at venepuncture. A: 1,2,3 Correct B: 1,3 Correct C: 2,4 Correct D: 4 Correct E: All Correct ABCDE Correct Answer: D At the cubital fossa, the veins of the upper limb are divided into deep and superficial. The deep brachial veins are arranged in pairs which accompany the brachial artery. They begin at

61

the elbow by the union of the venae comitantes (companions) of the ulnar and radial arteries and end in the axillary vein. The brachial veins contain valves and are connected at intervals by short transverse branches. The superficial veins are the cephalic (medial) and basilic (lateral) veins. They are linked by the median cubital vein anterior to the bicipital aponeurosis. The latter vein is most commonly used for venepuncture, however, considerable anatomic variation exists. MOORE, K.L; Clinically Oriented Anatomy, Williams and Wilkins, 1982, pp 740-1. Number: 949 The pudendal nerve: 1. Crosses the greater sciatic foramen. 2. Crosses the lesser sciatic foramen. 3. Gives off the inferior rectal (inferior haemorrhoidal) nerve. 4. Gives off the posterior cutaneous nerve of the thigh. A: 1,2,3 Correct B: 1,3 Correct C: 2,4 Correct D: 4 Correct E: All Correct ABCDE Correct Answer: A See: "Anatomy for Anaesthetists" Ellis and Feldman ISBN 0865427216 According to these authors: "The pudendal nerve (S2, 3, 4) provides the principal innervation of the perineum; its course is complex, passing from the pelvis, briefly through the gluteal region, along the side-wall of the ischiorectal fossa and through the deep perineal pouch to end by supplying the skin of the external genitalia. Arising as the lower main division of the sacral plexus (although dwarfed by the giant sciatic nerve), the pudendal nerve leaves the pelvis through the greater sciatic foramen below piriformis. It appears briefly in the buttock region, accompanied laterally by the internal pudendal vessels, merely to cross the dorsurn of the ischial spine and straightway disappear through the lesser sciatic foramen into the perineum, The nerve now traverses the lateral wall of the ischiorectal fossa, accompanied by the internal pudendal vessels, and lies within a distinct fascial. compartment on the medial aspect of obturator internus termed the pudendal canal (Alcock's canal). Within the canal, it first gives off the inferior rectal nerve which crosses the fossa to innervate the external anal sphincter and the perianal skin, then divides into the perineal nerve and the dorsal nerve of the penis.

62

The perineal nerve is the larger of the two. It bifurcates almost at once; its deeper branch enters the deep pouch and there supplies sphincter urethrae and the other muscles of the anterior perineum the ischio-cavernosus, bulbospongiosus and the superficial and deep transverse perinei. Its more superficial branch innervates the skin of the posterior aspect of the scrotum. The dorsal nerve of the penis (or clitoris) traverses the deep perineal pouch, pierces the perineal membrane near its apex, then penetrates the suspensory ligament of the penis to supply the dorsal aspect of this structure." The posterior cutaneous nerve of the thigh arises directly from the S1-3 roots of the sacral plexus. Number: 950 The sphenoplatine ganglion: 1. Receives parasympathetic inflow from the greater superficial petrosal nerve. 2. Provides secretomotor fibres to the lacrimal gland. 3. Receives sympathetic inflow via the deep petrosal nerve. 4. Provides parasympathetic supply to the pupillary sphincter. A: 1,2,3 Correct B: 1,3 Correct C: 2,4 Correct D: 4 Correct E: All Correct ABCDE Correct Answer: A See: "Anatomy for Anaesthetists" Ellis and Feldman ISBN 0865427216 According to these authors: "The sphenopalatine ganglion (which is associated closely with the maxillary nerve) is deeply placed in the upper part of the pterygopalatine fossa. It receives para-sympathetic, sympathetic and sensory nerve fibres. The parasympathetic component is derived from the greater superficial petrosal nerve which originates from the geniculate ganglion of the facial nerve (VII). This nerve traverses the petrous temporal bone then runs in a groove on the anterior surface of the bone deep to the trigeminal ganglion to enter the foramen lacerum. Here it is joined by the deep petrosal nerve to form the nerve of the pterygoid canal (the vidian nerve) which passes through the pterygoid canal to reach the sphenopalatine ganglion. These parasympathetic fibres, having arrived at the ganglion, have not completed their complicated journey. They are transmitted via the zygomaticotemporal branch of the

63

maxillary nerve to the lacrimal branch of the ophthalmic nerve by which they arrive at their final destination as secretomotor fibres to the lacrimal gland. Sympathetic fibres, derived from the internal carotid plexus, form the deep petrosal nerve which, as described above, reaches the ganglion via the nerve of the pterygoid canal. The sensory component is derived from the two sphenopalatine branches of the maxillary nerve. The sensory and sympathetic (vasoconstrictor) branches of the ganglion are distributed to the nose, nasopharynx, palate and orbit via the following branches: (i) The long sphenopalatine nerve passes medially through the sphenopalatine foramen, crosses the roof of the nasal cavity, then passes downwards and forwards along the nasal septum, grooving the vomer as it does so, to reach the incisive foramen and thence the mucous membrane of the roof of the mouth. It supplies filaments to the posterior part of the nasal roof, to the nasal septum, and to those parts of the gums and anterior part of the hard palate which are in relation to the incisor teeth. (ii) The short sphenopalatine branches also pass medially through the sphenopalatine foramen; they supply sensory fibres to the superior and middle conchae and to the posterior part of the nasal septum. (iii) The greater palatine nerve descends through the greater palatine canal, then emerges on to the hard palate from the greater palatine foramen just posterior to the palatomaxillary suture. It innervates the mucosa of the gums and hard palate as far forward as the level of the canine teeth. Other fibres pass backwards to serve both aspects of the soft palate and nasal branches pierce openings in the perpendicular plate of the palatine bone to supply the region of the inferior nasal concha. (iv) The lesser palatine nerves, two or occasionally three in number, pass through the greater palatine canal in company with the greater palatine nerve but emerge through separate lesser palatine foramina, which perforate the inferior and medial aspects of the tubercle of the palatine bone. They supply the soft palate, uvula and tonsil. (v) The pharyngeal nerve passes backwards through the pharyngeal canal in the posterior wall of the pterygopalatine fossa to supply an area of nasopharyngeal mucosa immediately behind the orifice of the Eustachian tube. (vi) The orbital branches are small; they constitute two or three fine twigs which pass through the superior orbital fissure to supply the adjacent periosteum and perhaps also to carry some secretomotor fibres from the sphenopalatine ganglion to the lacrimal gland." The parasympathetic supply to the pupillary sphincter arises from the ciliary ganglion. Interest in this neurological backwater has recently been rekindled by pain doctors! See: Klein RN, Burk DT, Chase PF. Anatomically and physiologically based guidelines for use of the sphenopalatine ganglion block versus the stellate ganglion block to reduce atypical facial pain.

64

Cranio. 2001 Jan;19(1):48-55. Cohen S, Trnovski S, Zada Y. A new interest in an old remedy for headache and backache for our obstetric patients: a sphenopalatine ganglion block. Anaesthesia. 2001 Jun;56(6):6067. Number: 983 The sphenoplatine ganglion: 1. Is situated in the pterygopalatine fossa. 2. Provides sensory innervation to the soft palate. 3. Receives sympathetic inflow via the deep petrosal nerve. 4. Provides sympathetic innervation of the nasopharynx. A: 1,2,3 Correct B: 1,3 Correct C: 2,4 Correct D: 4 Correct E: All Correct ABCDE Correct Answer: E See: "Anatomy for Anaesthetists" Ellis and Feldman ISBN 0865427216 According to these authors: "The sphenopalatine ganglion (which is associated closely with the maxillary nerve) is deeply placed in the upper part of the pterygopalatine fossa. It receives para-sympathetic, sympathetic and sensory nerve fibres. The parasympathetic component is derived from the greater superficial petrosal nerve which originates from the geniculate ganglion of the facial nerve (VII). This nerve traverses the petrous temporal bone then runs in a groove on the anterior surface of the bone deep to the trigeminal ganglion to enter the foramen lacerum. Here it is joined by the deep petrosal nerve to form the nerve of the pterygoid canal (the vidian nerve) which passes through the pterygoid canal to reach the sphenopalatine ganglion. These parasympathetic fibres, having arrived at the ganglion, have not completed their complicated journey. They are transmitted via the zygomaticotemporal branch of the maxillary nerve to the lacrimal branch of the ophthalmic nerve by which they arrive at their final destination as secretomotor fibres to the lacrimal gland. Sympathetic fibres, derived from the internal carotid plexus, form the deep petrosal nerve which, as described above, reaches the ganglion via the nerve of the pterygoid canal.

65

The sensory component is derived from the two sphenopalatine branches of the maxillary nerve. The sensory and sympathetic (vasoconstrictor) branches of the ganglion are distributed to the nose, nasopharynx, palate and orbit via the following branches: (i) The long sphenopalatine nerve passes medially through the sphenopalatine foramen, crosses the roof of the nasal cavity, then passes downwards and forwards along the nasal septum, grooving the vomer as it does so, to reach the incisive foramen and thence the mucous membrane of the roof of the mouth. It supplies filaments to the posterior part of the nasal roof, to the nasal septum, and to those parts of the gums and anterior part of the hard palate which are in relation to the incisor teeth. (ii) The short sphenopalatine branches also pass medially through the sphenopalatine foramen; they supply sensory fibres to the superior and middle conchae and to the posterior part of the nasal septum. (iii) The greater palatine nerve descends through the greater palatine canal, then emerges on to the hard palate from the greater palatine foramen just posterior to the palatomaxillary suture. It innervates the mucosa of the gums and hard palate as far forward as the level of the canine teeth. Other fibres pass backwards to serve both aspects of the soft palate and nasal branches pierce openings in the perpendicular plate of the palatine bone to supply the region of the inferior nasal concha. (iv) The lesser palatine nerves, two or occasionally three in number, pass through the greater palatine canal in company with the greater palatine nerve but emerge through separate lesser palatine foramina, which perforate the inferior and medial aspects of the tubercle of the palatine bone. They supply the soft palate, uvula and tonsil. (v) The pharyngeal nerve passes backwards through the pharyngeal canal in the posterior wall of the pterygopalatine fossa to supply an area of nasopharyngeal mucosa immediately behind the orifice of the Eustachian tube. (vi) The orbital branches are small; they constitute two or three fine twigs which pass through the superior orbital fissure to supply the adjacent periosteum and perhaps also to carry some secretomotor fibres from the sphenopalatine ganglion to the lacrimal gland." Interest in this neurological backwater has recently been rekindled by pain doctors! See: Klein RN, Burk DT, Chase PF. Anatomically and physiologically based guidelines for use of the sphenopalatine ganglion block versus the stellate ganglion block to reduce atypical facial pain. Cranio. 2001 Jan;19(1):48-55. Cohen S, Trnovski S, Zada Y. A new interest in an old remedy for headache and backache for our obstetric patients: a sphenopalatine ganglion block. Anaesthesia. 2001 Jun;56(6):6067.

66

Number: 999 All of the following nerves arise from cords of the brachial plexus, EXCEPT: A. Dorsal scapular (Nerve to rhomboids). B. Lateral pectoral. C. Thoracodorsal. D. Median. E. Ulnar. Select the single best answer ABCDE Correct Answer: A The Dorsal Scapular Nerve (nerve to the rhomboids). From the brachial plexus a posterior branch from the C5 root passes through scalenus medius, runs down deep to levator scapulae (which it supplies) and lies on the serratus posterior superior muscle to the medial side of the descending branch of the transverse cervical artery. It supplies each rhomboid on the deep surface. ANAESTHESIA MONITORING TESTS Number: 3 The train-of-four (TOF) ratio measured at the adductor pollicis (AP) which is commonly accepted as correlating with adequate reversal of neuromuscular blockade (NMB) is: A. 0.5. B. 0.6. C. 0.7. D. 0.8. E. 0.9. Select the single best answer ABCDE Correct Answer: C Torda has recently published a superb review entitled "Monitoring Neuromuscular Transmission". According to this author "In fact, what constitutes adequate recovery is in some dispute. The commonly accepted figure of 0.7 for the TOF ratio of the AP as 'adequate reversal' appears to have been introduced about 25 years ago. More recent work has called this 'standard' into serious doubt. The criteria for choosing 0.7 as the critical ratio were that patients were able to produce peak inspiratory pressure of -25 cm H20 and that their ventilation was normal. Swallowing and coughing, however, requires better neuromuscular recovery, equivalent to an inspiratory pressure of ~45 cm H20. The group from the University of Copenhagen, which is often quoted in this review, suggested 0.8 as the minimum adequate level of recovery. Bevan

67

has suggested that upper airway obstruction was possible following small doses of NMB drugs and may cause morbidity. Eriksson et al have shown that at TOF ratio of the AP of 0.7 there is significant weakness of the muscles of swallowing and contrast media entered the laryngeal vestibule in 25% of subjects under the study conditions. They concluded that patients with TOF ratio < 0.9 are at increased risk of pulmonary aspiration. Another possible source of adverse effects is the demonstrated 25% reduction in hypoxic respiratory response following partial NMB with the TOF ratio at 0.7. The significance of partial paralysis as a risk factor in postoperative pulmonary complications has been prospectively documented." Thus, although I have cited 0.7 as the 'correct' answer, 0.9 is probably more appropriate. See: Torda TA. Monitoring Neuromuscular Transmission. Anaesth. Int. Care. 2002, April (30): 123-133. Number: 9 The pattern of neural stimulation which is most useful for monitoring profound neuromuscular blockade (NMB) is: A. Train-Of-Four (TOF). B. Double Burst Stimulation (DBS). C. 50 hz tetanic burst (T50). D. 100 hz tetanic burst (T100). E. Post-tetanic Count (PTC). Select the single best answer ABCDE Correct Answer: E Torda has recently published an excellent review entitled "Monitoring Neuromuscular Transmission". The order of utility (least appropriate to most appropriate) appears to be: TOF, DBS, T50, T100, PTC - that is in the order A,B,C,D,E above. According to Torda, "Intense NMB cannot be quantified by twitch or TOF stimulation. Even before there is a return of a response to TOF, some quantitation of the NMB can be made by exploiting the Post-Tetanic Facilitation (PTF) phenomenon. The post-tetanic count (PTC) is elicited by administering a 5 s, 50 Hz tetanic train, followed after a 3 s pause by twitches at 1 Hz. A count of 10 or 11 twitches coincides usually with the reappearance of the first response to TOF stimulation. This can be used to maintain intense NMB but of course, it is not useful in predicting reversability of the NMB or the absence of residual blockade." A post-tetanic burst technique has also been described which is as good as the PTC for the monitoring of prfound NMB. See: Torda TA. Monitoring Neuromuscular Transmission. Anaesth. Int. Care. 2002, April (30): 123-133. Number: 40

68

This tracing was recorded from the distal lumen of a Swan-Ganz catheter inserted to a distance of 32 cms via the left subclavian vein with balloon inflated. The waveform shown below most probably represents: A. An unwedged pulmonary artery pressure trace. B. A wedged pulmonary artery pressure trace from a patient with mitral incompetence. C. A right ventricular pressure trace. D. 'Cannon' waves in a right atrial pressure trace. E. A coronary sinus pressure trace. Select the single best answer

ABCDE Correct Answer: C This is a typical right ventricular pressure trace. Note the low diastolic pressure (similar to the central venous pressure) which tends to rise through the diastolic interval as the ventricle becomes filled. Note also that the systolic wave is timed to the 'T' wave of the ECG. - If this were a wedged trace from the pulmonary artery of a patient with mitral incompetence, the systolic wave would be timed after the 'T' wave. If this were an unwedged pulmonary artery pressure trace, the diastolic pressure would be higher and tend to fall during the diastolic interval. Cannon waves cannot occur in the presence of regular sinus rhythm (such as is the case here).

69

Number: 42 Which of the following units are BASIC SI units of measurement? 1. Kilometre. 2. Candela. 3. Watt. 4. Kilogram. A: 1,2,3 Correct B: 1,3 Correct C: 2,4 Correct D: 4 Correct E: All Correct ABCDE Correct Answer: C There are seven basic units of measurement in the SI system. These are: Unit of length: meter: The meter is the length of the path travelled by light in vacuum during a time interval of 1/299 792 458 of a second. Unit of mass: kilogram: The kilogram is the unit of mass; it is equal to the mass of the international prototype of the kilogram. Unit of time: second: The second is the duration of 9 192 631 770 periods of the radiation corresponding to the transition between the two hyperfine levels of the ground state of the cesium 133 atom. Unit of electric current: ampere: The ampere is that constant current which, if maintained in two straight parallel conductors of infinite length, of negligible circular cross-section, and placed 1 meter apart in vacuum, would produce between these conductors a force equal to 2 x 10-7 newton per meter of length. Unit of thermodynamic temperature: kelvin: The kelvin, unit of thermodynamic temperature, is the fraction 1/273.16 of the thermodynamic temperature of the triple point of water. Unit of amount of substance: mole: 1. The mole is the amount of substance of a system which contains as many elementary entities as there are atoms in 0.012 kilogram of carbon 12; its symbol is "mol." 2. When the mole is used, the elementary entities must be specified and may be atoms, molecules, ions, electrons, other particles, or specified groups of such particles. Unit of luminous intensity: candela: The candela is the luminous intensity, in a given direction, of a source that emits monochromatic radiation of frequency 540 x 1012 hertz and that has a radiant intensity in that direction of 1/683 watt per steradian.

70

A permanent transvenous pacemaker set 'DDDR': 1. 2. 3. 4. Can change the pacing rate during exercise. Will inhibit Atrial Pacing in the presence of atrial systole. Can be made to go faster by tapping over the box. Can be inhibited by surgical diathermy.

A: 1,2,3 Correct B: 1,3 Correct C: 2,4 Correct D: 4 Correct E: All Correct ABCDE Correct Answer: E The 4 letters refer to the chamber being paced, the chamber sensed, the pacing mode and whether or not the device is rate adaptive. The devices can be reset either with a programmer, or if immediate conversion to VOO is required, with a simple magnet. If the unit is rate adaptive, tapping over the box (which is interpreted as muscle activity by the pacemaker) will temporarily increase the rate. An increase in heart rate is a major determinant of increased cardiac output during exercise and various sensing techniques have been used to trigger rate adaption during exercise: - If P waves are present SA node activity can be used. - If P waves are absent: 1. Muscle activity can be detected piezo-electrically. 2. Minute ventilation changes can be detected by measuring the cyclical change in impedance between the electrode tip and the pulse generator. 3. QT shortening (which occurs during exercise as a result of neurohumoral mechanisms), can be measured. A fifth category is used by some pacemakers and refers to the antiarrhythmic functions of the device. Thus the performance criteria of all pacemakers can be defined according to the following table: Chamber Paced Chamber Sensed Response to sensing Antiarrhythmic Function O = None O = None A = Atrial P = Pacing V = Ventricle S = Shock O = None A = Atrial V = Ventricle O = None T = Triggered I = Inhibited Programmability O = None P = Simple M = Multi

71

D = Dual (A+V) D = Dual (P+S)

D = Dual (A+V)

D = Dual (T+I)

R = Rate modulated

The antiarrhythmic functions include overdrive pacing for VT and SVT and the capacity to deliver a 25 J shock for VF. Prys-Roberts, C; Current Opinion in Anesthesiology, 4, 1991, pp 130-133. Zaiden, J.R. Pacemakers. Anesthesiology, 60, 1984, pp 319 et seq. Baig, MW and Perins, EJ. The Haemodynamics of Cardiac Pacing. Progress in cardiovascular disease. (1991). 33: pp283 et seq. Number: 80 What is the largest size endotracheal tube that will pass readily through the lumen of a size 3 laryngeal mask airway (LMA)? A. B. C. D. E. 6.00 mm (Internal Diameter (ID). 6.50 mm ID. 7.00 mm ID. 7.5 mm ID. 8.0 mm ID. Select the single best answer

ABCDE Correct Answer: A A size 3 or 4 LMA will allow up to a size 6.00 mm ID endotracheal tube to be passed without an introducer. Reference: ATKINSON, R.S. ET AL (EDS); Lee's Synopsis of Anaesthesia, 11th Ed., Butterworth, 1993, p227. Dr Douglas Fahlbusch has supplied the information that: 1. An adult size 5 LMA takes a 7.0 ID Endotracheal tube (ETT). In paediatric practice: 1. A size 2.5 LMA will allow the passage of a 5.0 mm ID ETT or a 4.0mm outside diameter flexible bronchoscope with an ETT loaded on it. 2.A size 1 LMA will allow the passage of a 3.5 mm ID ETT which will protrude 8 cm from grille using ETT adaptor

72

Number: 81 With respect to electronic defibrillators, which of the following are associated with increased transthoracic impedance? 1. Triggering during inspiration. 2. Repeated shocks. 3. Inadequate use of coupling gel. 4. Small paddle size. A: 1,2,3 Correct B: 1,3 Correct C: 2,4 Correct D: 4 Correct E: All Correct ABCDE Correct Answer: B Defibrillation is accomplished by passage of sufficient electrical current through the heart. Current flow (amperes) is determined by the energy chosen (joules) and the transthoracic impedance or resistance to current flow (ohms). The average transthoracic impedance in adults is 70-80 ohms. Recommended energy requirements for cardioversion and defibrillation are calculated on this assumption. If transthoracic impedance is unexpectedly high, insufficient current may be generated from shocks using standard energy levels. Transthoracic impedance is increased with inspiration, large paddle size, inadequate use of coupling gel, excessive gel outside of the paddles which allows the current to dissipate, and inadequate force applied by the operator. Transthoracic impedance decreases with successive shocks. Theoretically, the energy required on successive shocks should not need to be increased, however, the recommendations are that they be increased. (ie 200, 300, 360 J). References 1992 National Conference on Cardiopulmonary Resuscitation (CPR) and Emergency Cardiac Care (ECC), JAMA - Symposium issue, vol 268, no 16, October 28th, 1992, p 2212. Number: 82 With reference to perioperative myocardial ischaemia as detected by the ECG, which of the following are correct? 1. QRS complexes with a taller R wave exhibit greater ST depression in response to ischaemia. 2. Minor degrees of ST depression often result by going from a .05 Hz cutoff to a O.5 Hz cutoff. 3. It may mimic changes produced by subarachnoid haemorrhage. 4. The "monitor" mode is preferable to the "diagnostic" mode.

73

A: 1,2,3 Correct B: 1,3 Correct C: 2,4 Correct D: 4 Correct E: All Correct ABCDE Correct Answer: A The taller the R wave, the greater the amount of ST depression that will occur during an ischaemic episode. This phenomena is known as the "R wave gain factor", and is a well known cause of greater amounts of ST depression in response to ischaemia. Treadmill studies have been made more sensitive and specific by normalizing for the interpatient variability in R wave amplitude. It is well known that subarachnoid haemorrhage can produce a variety of ECG changes, including T inversion or flattening, ST depression or elevation, and QT prolongation. Monitor and diagnostic modes on ECG monitors alter the frequencies filtered out to produce the final displayed signal. Monitor mode usually filters out frequencies below 4 Hz (0.5 Hz in some machines), whereas diagnostic mode filters frequencies below 0.05 -0.1 Hz. Although a higher filtration cutoff renders the baseline more stable, an isoelectric ST segment may become elevated or depressed, mimicking ischaemia. Therefore, going from a 0.05 to 0.5 Hz cutoff may produce ST depression. Diagnostic mode is preferred for this reason. MILLER, R.D (ED); Anesthesiolgy, Churchill Livingstone, 2nd Ed., 1986. ASA Annual Refresher Course Lectures, 1991. HOLLENBERG, M.ET AL;" Influence of R wave amplitude on exercise induced ST depression: need for a gain factor correction when interpreting stress electrocardiograms ", Am J Cardiol, 56, 1985, pp 13-17. SLOGOFF, S & KEATS, A.S; " Does perioperative myocardial ischaemia lead to postoperative myocardial infarction? ", Anesthesiology, 62, 1985, pp 107-114. Number: 83 When using a pulmonary artery flotation catheter: 1. Complete heart block can be precipitated in patients with left bundle branch block (LBBB). 2. Measurement of cardiac output is equally as accurate with room temperature and ice cold injectates. 3. Obesity increases the gradient between PCWP and left atrial pressure. 4. Mixed venous blood samples can be drawn from the port distal to the wedged balloon. A: 1,2,3 Correct B: 1,3 Correct C: 2,4 Correct

74

D: 4 Correct E: All Correct ABCDE Correct Answer: A Complete heart block is a well documented complication of these devices in patients with preexisting LBBB, possibly due to mechanical trauma to the vulnerable subendocardial portion of the right bundle. Although the signal to noise ratio is improved by using ice cold injectate, room temperature injectates are equally as accurate for measurement of cardiac output by thermodilution. Moreover, ice cold solutions may precipitate arrhythmias (atrial fibrillation and bradyarrhythmias). The assumption made when interpreting PCWP measurements as an index of left atrial pressure (LAP), is a continuous column of blood between the wedged balloon and the left atrium, with the pressure equilibrated throughout. This correlation becomes tenuous in the presence of high airway pressures and pulmonary vascular disease. The PCWP-LAP gradient will be increased by PEEP, CPAP, airway obstruction, obesity and hypovolemia. A wedged catheter can only conceivably draw blood from an area distal to itself, and therefore may contain some pulmonary capillary blood, which would artificially elevate the mixed venous oxygen tension. BLITT, C.D.; Monitoring in Anesthesia and Critical Care Medicine, Churchill Livingstone, New York, 1985. Number: 157 Which of the following are methods of measuring the content of oxygen in a gas mixture? 1. Paramagnetic analysis 2. Mass spectrometry. 3. Fuel cell analysis 4. Katharometry. A: 1,2,3 Correct B: 1,3 Correct C: 2,4 Correct D: 4 Correct E: All Correct ABCDE Correct Answer: A

75

The measurement of gases by katharometry is dependent upon the property of the gas to be thermally conducted. If resistance elements are incorporated into an electric circuit, and the gas to be measured introduced across the circuit, them the change in temperature will cause a change in electrical resistance, resulting in a potential difference. This potential difference is related to the content of the gas. CO2 and helium, but not oxygen are measured in this way. References SYKES, M.K. VICKERS, M.D. & HULL, C,J; Principles of Measurement and Monitoring in Anaesthesia and Intensive Care, 3rd Ed., Blackwell, 1991, p 228. Number: 158 With regard to a pneumotachograph: 1. It is a fixed orifice, variable pressure device. 2. It measures laminar flow. 3. The pressure drop across the resistance component is generally low. 4. It may be inaccurate in infants. A: 1,2,3 Correct B: 1,3 Correct C: 2,4 Correct D: 4 Correct E: All Correct ABCDE Correct Answer: E A pneumotachograph is a rapid response, fixed orifice/variable pressure device for measuring either constant or intermittent gas flows. It incorporates a flow chamber, resistance component, pressure transducer, and thermostat. The principle is that laminar flow across the resistance component will cause a pressure drop across the resistance which is determined by the flow and the resistance. When the resistance is known, the flow is directly related to the pressure gradient (this is an analog of Ohm's Law). The transducer may be associated with a microprocessor, amplifier, display, and recording facility. These would be seen in units located within pneumotachographs used in respiratory function testing, or in modern spirometers. The pressure drop is kept to a minimum by anticipating the range of flows and adjusting the resistance accordingly. It is usually of the order of 1-2 mm Hg. This is necessary in order to preserve laminar flow (and hence the direct relationship between pressure and flow ), and to minimize any additional resistance in the unit which may cause inaccuracies in measurement. The dead space is of the order of 16 mls which may cause inaccuracies at low tidal volumes (ie. neonates) References

76

SYKES, M.K. VICKERS, M.D. & HULL, C,J; Principles of Measurement and Monitoring in Anaesthesia and Intensive Care, 3rd Ed., Blackwell, 1991, pp 201-2. Number: 159 What arterial-end tidal CO2 tension difference would be expected to exist in a fit, young patient undergoing general anaesthesia? A. 0 mm Hg. B. 5 mm Hg. C. 10 mm Hg. D. 15 mm Hg. E. No single value can be expected; it is highly variable in this population. Select the single best answer ABCDE Correct Answer: B In a healthy awake subject, the end-tidal CO2 tension (ETCO2) is an indirect measure of arterial CO2 tension (PaCO2). The theoretical basis for this is that gas sampled at endexpiration represents alveolar gas which is in equilibrium with pulmonary end-capillary (arterial) blood. An "arterial-end expiratory PCO2 difference" has been observed to occur during anaesthesia in otherwise healthy, young subjects. This is generally of the order of 3-4 mm Hg. It is attributed to an increase in dead space ventilation during anaesthesia. In elderly subjects and those with preexisting lung disease, it has been recorded as high as 18 mmHg. Other causes include the supine position, positive pressure ventilation, and surgery. Paradoxically, in some patients (eg pregnant women at term), the ETCO2 can exceed the arterial CO2. References ROGERS, M.C ET AL (EDS); Principles and Practice of Anesthesiology, Mosby, 1993, pp 786. Number: 160 The safety coding features existing for all enflurane vapourizers and refill bottles are: A. A single pin and groove. B. Two pins and one groove. C. A groove only. D. An individualized shape for the block. E. None of the above. Select the single best answer

77

ABCDE Correct Answer: A Three identification devices exist to prevent inadvertent use of the wrong volatile anaesthetic agent. These are: (1) Colour coding- Trichloroethylene is coloured blue. In addition to this, all agents are packaged with individualized coloured labels. (2) Special key or pin filling devices- The commonly used agents are poured by specific key filling devices. In these, the pouring tube has a special block on the end of it, and a square cavity on the vapourizer to receive the filling block. Specificity is achieved by the position of the special groove on one side of the block. This groove mates with a short pin protruding from the wall of the square socket. The lower face of the block has liquid and air holes which allow the vapourizer to fill when the liquid agent in the bottle is inverted. (3) Individualized bottle necks and threads- There is a common agreement amongst manufacturers that the bottles for each agent have necks with the same screw thread regardless of which company produces them. Each agent has a different size neck and thread. References RUSSELL, W.J; Equipment for Anaesthesia and Intensive Care, 1983, pp 81-2. Number: 161 A Semi-rigid oxygen mask receiving an oxygen flow of 10 L/min will deliver what concentration of oxygen to the patient? A. 30%. B. 40%. C. 50%. D. 60%. E. 70%. Select the single best answer ABCDE Correct Answer: D A semi-rigid oxygen mask is an example of a variable performance system. Here, the fractional inspired oxygen concentration is dependent on the flow rate, patient ventilation, and device factors, including the fit of the mask. These devices will only be able to deliver 100% oxygen if the flow rate exceeds the patient's peak inspiratory flow rate. This is generally five times the resting minute volume or 25-35 L/min.

78

References T.E. OH; Intensive Care Manual, 3rd Ed., p 131. T.E. OH & DUNCAN, A.W; " Oxygen Therapy ", The Medical Journal of Australia, vol 149, August 1, 1988. (REVIEW) Number: 162 In an " in-series ", twin system soda lime, what is the most reliable sign of exhaustion (the "break point") of the upstream canister? A. Mixed expired CO2 concentration of 4%. B. Colour change throughout canister. C. Cooling of the first (upstream) canister. D. Warming of the second (downstream) canister. E. None of the above. Select the single best answer ABCDE Correct Answer: D Generally, a soda lime is considered at the end of its useful life when about 0.5% of CO2 is leaking through, with a mixed expired gas concentration of 4%. This is called the "break point". This is reached gradually, as the efficiency of the soda lime decreases with time. The "break point" of a single canister is of less importance when it is used in series with a second canister. This is because the first or upstream canister will absorb CO2 preferentially and be exhausted before there is significant utilization of the second or downstream canister. In this situation, exhaustion of the first will not be realized as an increasing mixed expired concentration of CO2. Exhaustion of the upstream canister will be suggested by its cooling or colour change, but the most reliable indication is probably warming of the downstream canister which is now receiving a larger load of CO2. References RUSSELL, W.J; Equipment for Anaesthesia and Intensive Care, 1983, pp 11-12. Number: 163 Which is the most efficient breathing system during controlled ventilation? A. Mapleson A. B. Mapleson B. C. Mapleson C. D. Mapleson D. E. Mapleson E. Select the single best answer

79

ABCDE Correct Answer: D The Mapleson A circuit is the only circuit which has the fresh gas flow distally located near the reservoir bag, The pressure release valve is located near the patient. The performance of this circuit differs dramatically between situations of spontaneous and controlled ventilation. During spontaneous ventilation with the valve fully open, the patient inhales only fresh gas flow. During exhalation, all expired gases are directed out the valve, and the reservoir/expiratory limb is again filled with fresh gas flow. This circuit provides the least amount of rebreathing in this situation. During controlled ventilation with the valve fully closed, expired gases will be directed into the reservoir limb so that on inspiration rebreathing will occur. To prevent rebreathing in this situation, very large gas flows are required and the valve must be partially open. In the Mapleson B circuit, the fresh gas flow and pressure release valve are located close to the patient; the reservoir limb is not the source of fresh gas as in the type A circuit. This circuit is less efficient than the A in spontaneous breathing and also allows rebreathing in controlled ventilation. In the Mapleson C circuit, fresh gas flow, valve, and reservoir bag are all located close to the patient, hence rebreathing is reduced in both situations. It provides less rebreathing than the A during controlled ventilation. The Mapleson D circuit features a fresh gas flow which is located close to the patient and a pressure release valve and reservoir bag which are located distally. During spontaneous ventilation, the fresh gas flow is directed toward the patient, and any overflow is directed into the expiratory limb. During expiration, expired gas is directed up the expiratory limb and is diluted with fresh gas flow. If the fresh gas flow is sufficiently high to fill the expiratory limb, or when the fresh gas volume in the expiratory limb is equal to the tidal volume, then no rebreathing of expired gas occurs during inspiration. During controlled ventilation, the expiratory limb is pressurized by occlusion. Fresh gas and some expiratory gas flow toward the patient. The greater the fresh gas flow, the less rebreathing occurs. During spontaneous ventilation, the Mapleson D circuit requires higher fresh gas flow than the Mapleson A circuit to ensure no rebreathing; however, this is the most efficient circuit for CO2 elimination during controlled ventilation. References: ROGERS, M.C ET AL (EDS); Principles and Practice of Anesthesiology, Mosby, 1993, pp 2157-9. Number: 164 When a sphygmomanometer is attached to the common gas outlet of an anaesthetic machine and the oxygen flow meter is turned on, in the absence of leaks upstream: A. A flow of 100 ml/min maintains a pressure of 25 mm Hg. B. A flow of 500 ml/min maintains a pressure of 25 mm Hg. C. A flow of 100 ml/min maintains a pressure of 50 mm Hg.

80

D. A flow of 500 ml/min maintains a pressure of 50 mm Hg. E. None of the above. Select the single best answer ABCDE Correct Answer: A The Australian protocol for checking leaks upstream from the common gas outlet recommends the use of a sphygmomanometer attached to the common gas outlet. The oxygen flow meter is turned on to a flow of 100 ml/min. In the absence of significant leaks, this should maintain a pressure of 25 mm Hg. References ANDREWS, J; " Checking Anaesthesia Machines ", Australian Anaesthesia 1990, pp 162-3. Number: 165 During the use of uncuffed endotracheal tubes in paediatric patients, a gas leak should ideally be elicited with an inspiratory pressure of: A. 10 cm H2O. B. 20 cm H2O. C. 30 cm H2O. D. 40 cm H2O. E. 50 cm H2O. Select the single best answer ABCDE Correct Answer: B The air leak test around an endotracheal tube is performed by slowly increasing airway pressure while noting the airway pressure at which audible air begins to leak around the endotracheal tube into the glottis. The ideal range is considered 15-20 cm H2O, with a safe range of 10-30 cm H2O. References ROGERS, M.C. ET AL (EDS); Principles and Practice of Anesthesiology, Mosby, 1993, pp 2162. Number: 166 Which of the following materials are flammable in air? 1. Polyethylene. 2. Polyvinylchloride (PVC). 3. Polymethylmethacrylate. 4. Polytetrafluoroethylene (Teflon).

81

A: 1,2,3 Correct B: 1,3 Correct C: 2,4 Correct D: 4 Correct E: All Correct ABCDE Correct Answer: B Polyethylene and polymethylmethacrylate are flammable in air. Teflon requires 100% oxygen for flammability. PVC requires 26.3% oxygen in nitrogen to support combustion. One implication of the latter is that if an inspired oxygen concentration of 25% is used, then a sustained flame from an ignited PVC endotracheal tube is not possible. This is of particular relevance to laser surgery of the oral cavity. References WOLF, G.L.; " Flammability of Medical Equipment in Oxygen Enriched Atmospheres ", Australian Anaesthesia 1990, pp 149-50. Number: 167 A filling ratio is used when filling cylinders of : 1. CO2. 2. Cycloproprane. 3. N2O. 4. Entonox. A: 1,2,3 Correct B: 1,3 Correct C: 2,4 Correct D: 4 Correct E: All Correct ABCDE Correct Answer: A Filling ratios are used in the filling of cylinders whose contents are liquified under pressure. Cylinders containing oxygen, nitrogen, argon, helium, and air are filled under pressure to a nominal pressure of about 13,700 kPa. Because they remain in the gaseous state, cylinder pressure alone can be used as a guide to filling. N2O, CO2, and cycloproprane are liquifiable gases. When these cylinders are filled under pressure, gas entering the cylinder will form liquid, with the vapour pressure remaining unchanged. Pressure cannot be used as a guide to filling. These cylinders are thus weighed. The nominal weight at which the cylinder is considered " filled " is given by the filling ratio. The filling ratio of the gas is defined as the

82

weight of the gas in the cylinder to the water capacity at 15 degrees celsius. The filling ratio is an arbitrary value based on the expected behaviour of the agent. For instance, the filling ratio of N2O is 0.67 in tropical and sub-tropical climates (Australia), and 0.75 in temperate climates (United Kingdom). The filling of Entonox requires a special procedure because oxygen will remain as a gas and N2O will liquify under pressure. The mixture is manufactured by compressing the correct amount of N2O by weight in a cylinder. The pressure (about 5,000 kPa) causes some of it to liquify. Oxygen is then introduced into the cylinder and bubbles through the N2O. Initially, N2O is vapourized and oxygen dissolves into this mixture. Gradually, the cylinder pressure rises and the liquid evaporates. A cylinder pressure of about 12,000 kPa will be reached when the oxygen concentration reaches 50%. References RUSSELL, W.J.; Equipment for Anaesthesia and Intensive Care, 1983, p 16. Number: 168 What is the maximum allowable negative pressure applied across the interface of an active scavenging system? A. 0.1 cm H2O. B. 0.5 cm H2O. C. 1 cm H2O. D. 5 cm H2O. E. 10 cm H2O. Select the single best answer ABCDE Correct Answer: B An active scavenging system uses vacuum to remove waste gases. Interfaces are essential in active scavenging systems to ensure that the suction is not directly applied to the patient. An interface should ensure that no more than 0.5 cm H2O of negative pressure, nor 3.5 cm H2O of positive pressure are applied to the patient's circuit at any time. References RUSSELL, W.J.; Equipment for Anaesthesia and Intensive Care, 1983, p 123. Number: 169 What is the most important determinant of CO2 elimination during ventilation with an anaesthetic circuit not using a CO2 absorber? A. The type of circuit (Mapleson A,B,C,D,E,F) B. The mode of ventilation (spontaneous versus controlled).

83

C. The minute ventilation. D. The fresh gas flow. E. All of the above are equally important and must be considered within context. Select the single best answer ABCDE Correct Answer: D The elimination of CO2 during ventilation with a circuit which does not incorporate a CO2 absorber is determined by the extent of rebreathing which occurs. The type of circuit is important but this is usually dependent upon the mode of ventilation; the Mapleson A circuit ia associated with the least amount of rebreathing during spontaneous ventilation, but is inefficient during controlled ventilation. Under these conditions the D circuit is considered most efficient. In relation to this, the mode of ventilation is also important but variable. Respiratory rate, peak inspiratory flow rate, and tidal volume all influence elimination of CO2. The minute ventilation is the primary determinant of CO2 elimination when a CO2 absorber is in the circuit ( a reciprocal relationship exists which is independent of fresh gas flow), but is not a major determinant in the absence of CO2 absorption. In this situation, the fresh gas flow is the most important determinant of rebreathing, and hence, CO2 elimination. References ROGERS, M.C. ET AL (EDS); Principles and Practice of Anesthesiology, Mosby, 1993, pp 2159. Number: 170 Which of the following traces require a prompt response during the continuous measurement of intracranial pressure (ICP)? 1. Flattened trace. 2. C waves. 3. A waves. 4. B waves. A: 1,2,3 Correct B: 1,3 Correct C: 2,4 Correct D: 4 Correct E: All Correct ABCDE Correct Answer: B Pulsations from the choroid plexus are transmitted to the CSF in the ventricular and subarachnoid spaces and produce a waveform similar, but of lower amplitude, to the arterial pressure. A normal waveform rises sharply initially, then slopes toward the baseline. The

84

amplitude remains under 15 mm Hg. Several abnormal waveforms may be observed whilst monitoring a patient with intracranial pathology. (1) A waves- vary in intensity and amplitude, range from 50-100 mm Hg, and last 5-20 minutes. These are also known as "plateau" waves and occur when ICP rises rapidly above 50 mm Hg. They should be interpreted as a deterioration in the patient's condition which requires immediate treatment. (2) B waves-these are sharp rhythmic fluctuations that can occur in conjunction with changes in respiration and/or blood pressure. They sometimes precede A waves. They may rise as high as 50 mmHg, and usually occur every 50 seconds to 2 minutes. Whilst they may herald serious rises in ICP, B waves generally do not require prompt treatment. (3) C waves- these are also rhythmic fluctuations that correlate with changes in respiration and blood pressure, but are not as sharp or as rapid as B waves. They are small and rapid; usually occurring at a rate of four to eight per minute, and are not clinically significant. Flattening of the trace usually represents excessive damping which will reduce the usefulness of the measurement. References LUCHKA, S.; " Working with ICP monitors ", RN, April, 1993, pp 34-7. Number: 170 Which of the following traces require a prompt response during the continuous measurement of intracranial pressure (ICP)? 1. Flattened trace. 2. C waves. 3. A waves. 4. B waves. A: 1,2,3 Correct B: 1,3 Correct C: 2,4 Correct D: 4 Correct E: All Correct ABCDE Correct Answer: B Pulsations from the choroid plexus are transmitted to the CSF in the ventricular and subarachnoid spaces and produce a waveform similar, but of lower amplitude, to the arterial pressure. A normal waveform rises sharply initially, then slopes toward the baseline. The amplitude remains under 15 mm Hg. Several abnormal waveforms may be observed whilst monitoring a patient with intracranial pathology.

85

(1) A waves- vary in intensity and amplitude, range from 50-100 mm Hg, and last 5-20 minutes. These are also known as "plateau" waves and occur when ICP rises rapidly above 50 mm Hg. They should be interpreted as a deterioration in the patient's condition which requires immediate treatment. (2) B waves-these are sharp rhythmic fluctuations that can occur in conjunction with changes in respiration and/or blood pressure. They sometimes precede A waves. They may rise as high as 50 mmHg, and usually occur every 50 seconds to 2 minutes. Whilst they may herald serious rises in ICP, B waves generally do not require prompt treatment. (3) C waves- these are also rhythmic fluctuations that correlate with changes in respiration and blood pressure, but are not as sharp or as rapid as B waves. They are small and rapid; usually occurring at a rate of four to eight per minute, and are not clinically significant. Flattening of the trace usually represents excessive damping which will reduce the usefulness of the measurement. References LUCHKA, S.; " Working with ICP monitors ", RN, April, 1993, pp 34-7. Number: 171 With respect to storage of medical gases: 1. Air cylinders have black and white shoulders on grey bodies. 2. Some oxygen cylinders are painted green. 3. Nitrous oxide cylinders cool more quickly than oxygen cylinders. 4. Helium cylinders are painted orange. A: 1,2,3 Correct B: 1,3 Correct C: 2,4 Correct D: 4 Correct E: All Correct ABCDE Correct Answer: A Orange coloured cylinders are used for cycloproprane and mid brown cylinders are used for helium. Both air and nitrogen cylinders have a grey body; air has quartered black and white shoulders and nitrogen has all black shoulders. Although the Australian standard for oxygen is as described above, cylinders held on aircraft originating in the USA may actually be green. Adiabatic cooling occurs as a gas expands. It occurs in cylinders of both oxygen and nitrous oxide, however additional cooling occurs in nitrous oxide due the latent heat of vapourization as liquid enters the vapour phase.

86

References RUSSELL, W.J.; Equipment for Anaesthesia and Intensive Care, 1983. Number: 172 With regard to vacuum insulated evaporators (VIE) used in the storage of bulk oxygen: 1. Oxygen is stored mainly as a gas. 2. A temperature of about -150 degrees exists in the tank. 3. The oxygen content can be calculated by measuring the tank pressure. 4. The tanks are insulated with concentric steel shells and an evacuated space between. A: 1,2,3 Correct B: 1,3 Correct C: 2,4 Correct D: 4 Correct E: All Correct ABCDE Correct Answer: C The oxygen is stored mainly as a liquid by cooling. This cooling is achieved through the latent heat of vapourization generated as oxygen enters the gas phase on leaving the cylinder, and through insulation of the cylinder walls. If oxygen leaves too rapidly, rapid cooling occurs which may result in a pressure fall within the tank. This is regulated by a separate liquid oxygen drain which vapourizes in an external heat exchanger connected to the gas distribution system. The tank contents can be calculated from the weight but not the pressure, as the oxygen is mainly in the liquid form. References RUSSELL, W.J.; Equipment for Anaesthesia and Intensive Care, 1983, pp 24-6. Number: 173 Which of the following are true of rotameters? 1. There is generally a pressure drop across the rotating bobbin. 2. Temperature changes may cause inaccuracy. 3. Changes in density are more likely to cause inaccuracy in cyclopropane than viscosity. 4. Changes in density are more likely to cause inaccuracy in oxygen than viscosity. A: 1,2,3 Correct B: 1,3 Correct C: 2,4 Correct D: 4 Correct E: All Correct

87

ABCDE Correct Answer: C Temperature alters the rotameter mainly by its effect on viscosity and density. An increase in temperature from 10-35 degrees will, for most common medical gases, increase viscosity by 7%, and decrease density by 8%. The influence of viscosity is greatest at low flow, and of density at high flow. This is because of their respective effects on laminar and turbulent flow which occur at these ranges. At low flow, when viscosity is important, an increase in temperature causes an increase in viscosity and therefore a tendency to overread. The effects on density result in a tendency to underread at high flow. Normally, however, a change in temperature of a few degrees has no effect on accuracy. The range of flow for cyclopropane is within that of laminar flow, and that of air and oxygen is in turbulent flow. Rotameters are examples of variable orifice/constant pressure flowmeters. They are calibrated for a specific gas and have an accuracy of 3% at 1 ATM atmospheric pressure. References RUSSELL, W.J.; Equipment for Anaesthesia and Intensive Care, 1983, p 54. Number: 178 With regard to electrical safety in the operating room: 1. An isolated circuit provides total protection against microshock. 2. The minimum current applied externally that will cause VF is 1 ampere. 3. A core balance relay (ground fault detector) will alarm if an appliance has a broken ground wire. 4. Alternating current is at its most dangerous between 10-200 Hz. A: 1,2,3 Correct B: 1,3 Correct C: 2,4 Correct D: 4 Correct E: All Correct ABCDE Correct Answer: D The isolated circuit is powered by an additional transformer, but the return (neutral) wire is not connected to ground. In other words, there will be no potential difference between the live wire and ground. Therefore, the ideal isolated circuit would permit no current to flow if a bridge were to be created between the live wire and earth. This imparts a margin of safety to an individual who accidentally touches a live wire - such as may occur with a faulty appliance with a broken live wire which contacts the instrument casing. However, the ideal isolated circuit does not exist. Any appliance powered by alternating current will develop a small current in its casing due to the to and fro movement of electrons

88

in the circuitry. (Capacitance Current). A small potential difference is thus generated between the the circuit case and by default, between the circuit and ground since the casing is connected to a ground wire. As appliances are added to the circuit, the potential difference between the circuit and ground will increase, decreasing the isolation of the circuit. The leakage current under these circumstances may be enough to cause microshock if applied directly to the heart through conductors such as external pacemaker wires or saline filled central lines although the current will be too small to be detectable to the touch of the hand. Protection against microshock - which can occur with currents as small as 100 microamperes lies in the design of appliances that are to be attached to the patient. Macroshock requires significantly larger currents to cause VF. This is usually of the order of 100 millamperes. A core balance relay is designed to detect a difference in the current between the active and neutral wires and will shut off the current and/or alarm when the threshold difference is exceeded. Any difference will be due to current leaking to ground, such as may occur when a fault exists and the active wire contacts the casing of the appliance. Therefore, to function properly, such a relay requires an intact ground wire and will not function correctly if this wire is broken. Alternating current is most dangerous between 10-200 Hz, requiring significantly greater currents to cause VF outside these limits. References: BUCZKO, G.B. & MCKAY, W.P.S; " Electrical Safety in the operating room. ", Can. An. Soc. J. Anesth. 1987;34: pp 315-322. Number: 198 When using anaesthetic vaporisers: 1. The pumping effect is most marked at low flow rates. 2. In series, an ethrane vaporiser should be upstream of a halothane one. 3. With Vaporiser in Circle (VIC) as FGF is increased at a constant setting, the circuit concentration falls. 4. At high altitude a variable bypass vaporiser must be set to a higher dialled concentration to achieve the same delivered partial pressure compared to sea level. A: 1,2,3 Correct B: 1,3 Correct C: 2,4 Correct D: 4 Correct E: All Correct ABCDE Correct Answer: A

89

The pumping effect tends to increase the concentrations delivered by a vaporiser during IPPV. It is most marked with increasing back pressure, low FGF (cf pressurising effect which is converse) and lower dial setting of the vaporiser. It is minimised by modifications such as reducing the volume of the vaporising chamber relative to the bypass and increasing the volume of the inflow to the vaporising chamber. In series, the vaporiser delivering the agent with the lowest vapour pressure and highest MAC should be placed upstream. This will reduce the amount of cross contamination between the vaporisers. With VIC at low FGF, gas is recirculated more frequently through the vaporiser and takes up more agent with each passage. As the FGF is increased, so this effect becomes less marked and the concentration of the agent in the circuit falls. The effect of volatile agents on the brain is determined by their alveolar partial pressure, not the concentration. Because vapour pressure is independent of ambient pressure, the partial pressure of vapour delivered by a vaporiser at altitude will be the same as at sea level, although the concentration will rise since the partial pressure of the carrier gas will be lower. Therefore no adjustment to the dial setting is needed at altitude. Number: 202 When considering the detection of anaesthetic incidents: 1. Monitors are far more effective than anaesthetists at the initial detection of the impending incident. 2. A stethoscope is more cost:effective than a capnograph. 3. The ECG is useful at detecting an event before organ damage has occurred. 4. The capnograph and oximeter equally effective at initial detection. A: 1,2,3 Correct B: 1,3 Correct C: 2,4 Correct D: 4 Correct E: All Correct ABCDE Correct Answer: C References: Which Monitor? An analysis of 2000 incident reports. Webb RK et al. Anaesth. Intens Care 1993; 21: 529-542 This paper is one of several important papers in an issue of Anaesthesia and Intensive Care which was devoted to critical incident analysis. The net effect of the issue was (to say the least!) overwhelming and the creator of this question is still plodding dutifully through it.

90

Of the first 2000 incidents examined in the Australian Incident Monitoring Study, about half of the incidents were first detected by humans (48%) and half (52%) by one or other monitor. In the same study, the authors attempted to predict the likelihood of an individual monitor detecting an incident had it been allowed to evolve and the likelihood of this detection occurring before the potential for organ damage had occurred. On this basis, a stethoscope was virtually as effective as a capnograph, although the former was not as effective at early detection. Bearing in mind the cost differential, it must represent a more cost:effective instrument. An ECG was surprisingly deemed to be virtually ineffective at detecting critical incidents before the potential for organ damage had occurred - notwithstanding the fact that it apparently first detected 19% of monitor detected incidents. The capnograph first detected 24% of monitor detected incidents whereas the oximeter first detected 27%.

Number: 240 During the insertion of a pulmonary artery catheter, the incidence of ventricular arrhythmias can be reduced if: 1. The patient is placed 5 degrees head up. 2. The Internal Jugular route is used to insert the catheter. 3. The patient is rolled slightly to the right. 4. Lignocaine is given prior to passage of the catheter. A: 1,2,3 Correct B: 1,3 Correct C: 2,4 Correct D: 4 Correct E: All Correct ABCDE Correct Answer: B Positioning the patient 5 degrees head up with slight right lateral tilt during flotation of the catheter facilitates rapid transit of the catheter through the right ventricular outflow tract and reduces the likelihood of ventricular arrhythmias. The route of insertion and the 'prophylactic' administration of lignocaine has no effect on the incidence of arrhythmias. Keusch DJ, Winters S, and Thys DM. The patient's position influences the incidence of dysrhythmias during pulmonary artery catheterization. Anesthesiology 70:582-584, 1989.

91

Number: 243 A pulmonary artery catheter is contraindicated in a patient with:

1. A prosthetic Tricuspid Valve. 2. Tetralogy of Fallot. 3. Latex Allergy. 4. Type II Heparin Induced Thrombocytopenia. A: 1,2,3 Correct B: 1,3 Correct C: 2,4 Correct D: 4 Correct E: All Correct ABCDE Correct Answer: A The use of Heparin-bonded catheters is contraindicated in patients with Type II Heparin Induced Thrombocytopenia, but unbonded catheters can be safely used for short periods. 'Latex Free' catheters are not yet generally available for use in patients with latex allergy. As the balloons of all the manufacturers' catheters are made from latex, they should not be used in those with a history of allergy. Number: 249 The risk of pulmonary artery rupture associated with the use of a pulmonary artery catheter is higher in the presence of: 1. Advanced age. 2. Female gender. 3. Pulmonary hypertension. 4. Mitral stenosis, A: 1,2,3 Correct B: 1,3 Correct C: 2,4 Correct D: 4 Correct E: All Correct ABCDE

92

Correct Answer: E The risk factors which have been identified as increasing the likelihood of rupture include: Advanced age, female gender, pulmonary hypertension, mitral stenosis, anticoagulation and peripheral placement of the catheter tip. Most reported cases of rupture occur in association with balloon inflation or catheter manipulation. The incidence of pulmonary artery rupture may be reduced by: Staff training - in particular, teaching the importance of gradual balloon inflation, of watching the PA waveform continuously during balloon inflation and of stopping balloon inflation immediately a 'wedged' trace is observed. The use of a 'pilot' balloon which has a slightly lower compliance than that of the catheter balloon. If abnormal resistance is ignored during balloon inflation, a pilot balloon reduces the maximum attainable pressure within the catheter balloon from 1700 mm Hg to about 1000 mm Hg. Number: 250 Score: The incidence of sepsis related to the use of central venous catheters ('CRS') is increased if: 1. The line is left in place more than three days. 2. The internal jugular rather than the subclavian route is used. 3. A multi- rather than single lumen catheter is used. 4. Antibiotic prophylaxis is not used. A: 1,2,3 Correct B: 1,3 Correct C: 2,4 Correct D: 4 Correct E: All Correct ABCDE Correct Answer: A The longer a catheter is in place, the more likely it is to become infected. Furthermore, the risks of infection increase sharply if the PAC is in place for more than 3 days. If the risk of bacteraemia is expressed in terms of 'risk per catheter day', an incidence of 6-8 cases per 1000 catheter days is typical. The internal jugular route is associated with a fourfold increase in the risk of CRS as compared to the subclavian route.

93

Antibiotic prophylaxis has little effect on the incidence of CRS. Mermel LA; McCormick RD; Springman SR; Maki DG. The pathogenesis and epidemiology of catheter-related infection with pulmonary artery Swan-Ganz catheters: a prospective study utilizing molecular subtyping. Am J Med, 91(3B):197S-205S 1991 Number: 297 The likelihood of radial artery thrombosis complicating radial artery cannulation is increased if: 1. An 18G rather than a 20G catheter is used. 2. A polypropylene catheter has been used. 3. The patient suffers from Raynaud's phenomenon. 4. Aspirin has been administered preoperatively. A: 1,2,3 Correct B: 1,3 Correct C: 2,4 Correct D: 4 Correct E: All Correct ABCDE Correct Answer: A The incidence of radial artery thrombosis after cannulation has been extensively studied. Thrombosis has been found to correlate with the following factors: prolonged duration of cannulation, larger catheters, and smaller radial artery size (that is, a greater proportion of the artery is occupied by the catheter). Other factors associated with thrombosis in adults (but not in children) include polypropylene catheters and tapered catheters. The incidence of thrombosis is not affected by the technique of cannulation but is lowered with aspirin before treatment. Bedford RF: Radial arterial function following percutaneous cannulation with 18- and 20gauge catheters, Anesthesiology 47:37-39, 1977. Davis FM and Steward JM: Radial artery cannulation, BR J Anaesth 52:674-684, 1980. Bedford RF and Ashford TP: Aspirin pretreatment prevents post-cannulation radial-artery thrombosis, Anesthesiology 51:176-178, 1979. Number: 301 If holding a live electrical wire, what is the maximum current flow which an adult can tolerate before it becomes impossible to let go?

94

A. 1 mA. B. 5 mA. C. 10-20 mA. D. 50 mA. E. 100-300 mA. Select the single best answer ABCDE Correct Answer: C Following a 1-second contact with a 60-Hz source in an adult, a current level of: 1 mA will be just perceptible. 5 mA is accepted as the maximum harmless current. 10-20 mA is the maximum "Let-go" current before sustained muscle contraction occurs. 50 mA causes pain, possible fainting, but allows cardio-respiratory function to continue. 100-300 mA causes ventricular fibrillation, but the respiratory centre remains intact. Number: 320 Dead-space and alveolar gas flowing through standard anaesthetic circuit tubing will be completely mixed at a point: A. 0.25 metres along the tube. B. 0.50 metres along the tube. C. 1.0 metres along the tube. D. 1.25 metres along the tube. E. 1.50 metres along the tube. Select the single best answer ABCDE Correct Answer: C Spoerel WE: Rebreathing and carbon dioxide elimination with the Bain circuit, Can Anaesth Soc J 27:357-361, 1980. The pattern of gas flow through the circuit is almost always turbulent because of the corrugations in the tubing. This promotes both radial mixing and longitudinal mixing. In documenting performance of one circuit, Spoerel has demonstrated complete mixing of dead space and alveolar gas after gas has passed through one meter of such tubing.

95

Number: 398 With regard to an oxygen rotameter calibrated in the range 0 - 10 lpm at an indicated flow of 0.5 lpm: 1. The flow rate will still be 0.5 lpm if Nitrous Oxide is substituted for Oxygen. 2. The flow rate will still be 0.5 lpm if Helium is substituted for Oxygen. 3. Gas flow in the device will be orificial. 4. Gas flow in the device will be laminar. A: 1,2,3 Correct B: 1,3 Correct C: 2,4 Correct D: 4 Correct E: All Correct ABCDE Correct Answer: C Rotameters are examples of constant pressure, variable orifice flowmeters. At low flow rates, gas flow is essentially laminar and Poiseuille's law applies. When the flow rate is greater, turbulence occurs and flow becomes orificial. Flowmeters are not interchangeable among gases, and if a gas is passed through a rotameter for which it has not been calibrated, the flows shown are likely to be incorrect. However, at low flows, flow rates of gases with similar viscosities are read identically (e.g., oxygen and helium, 202 and 194 micropoise, respectively); and at high flows, gases of similar density (e.g., nitrous oxide and carbon dioxide, both of which have molecular weights of 44) are read identically. Number: 457 The most accurate measure of LVEDP on a wedged pulmonary artery pressure trace is: A. The mean value of the PAOP at the end of expiration. B. The peak of the 'A' wave at the end of expiration. C. The peak of the 'V' wave at the end of expiration. D. The peak of the 'A' wave at the end of inspiration. E. The mean value of the PAOP at the end of inspiration. Select the single best answer ABCDE

96

Correct Answer: B The peak of the 'A' wave at the end of expiration. - This point corresponds to the point of maximal distension of the ventricle and is the best measure of LVEDP (The 'Z' point). All intrathoracic intravascular pressure measurements should be made at the same phase of respiration. The end-expiratory point is generally agreed to be the most appropriate. Number: 464 A paralysed patient, under general anaesthesia is undergoing IPPV at a fixed rate and constant tidal volume. 200 mls of apparatus deadspace are inserted at the endotracheal tube connector. Which of the following statements are true? 1. End-tidal PCO2 will increase. 2. End-tidal to arterial PCO2 difference will increase. 3. Arterial PCO2 difference will increase. 4. Alveolar deadspace will increase. A: 1,2,3 Correct B: 1,3 Correct C: 2,4 Correct D: 4 Correct E: All Correct ABCDE Correct Answer: B The addition of apparatus ('serial') deadspace under these circumstances will increase the arterial and end-tidal PCO2, but there is no reason why alveolar ('Parallel') deadspace should change. Similarly, there is no reason why the difference between End-tidal to arterial PCO2 should be altered. Number: 495 To prevent rebreathing during spontaneous ventilation, the 'Bain' co-axial circuit requires a fresh gas flow of: A. 70 ml/kg/min. B. 100 ml/kg/min. C. 2.5 to 3 times minute ventilation.

97

D. 2.5 to 3 times alveolar ventilation. E. None of the above. Select the single best answer ABCDE Correct Answer: C Several studies have evaluated the fresh gas flow (FGF) requirements of the Bain system. When interpreting these data it is important to distinguish the FGF which can sustain normocarbia as opposed to the FGF which will effectively prevent rebreathing. (The latter being greater than the former). It has been found that during spontaneous ventilation, a FGF of 100 ml/kg/min produces normocarbia at the cost of increased minute ventilation. The FGF required to prevent rebreathing during spontaneous ventilation is 2.5 to 3 times the minute ventilation. During controlled ventilation the Bain circuit behaves more as a Mapleson D, and a FGF flow of 70 ml/kg/min results in normocarbia, provided that minute ventilation is adequate. See: Bain JA, Spoerel WE: A streamlined anaesthetic system, Can Anaesth Soc J 19:426, 1972. Conway CM, Seeley HF, and Barnes PK: Spontaneous ventilation with the Bain anaesthetic system, Br J Anaesth 49:1245, 1977. Bain JA, Spoerel WE: Flow requirements for a modified Mapleson D system during controlled ventilation, Can Anaesth Soc J 20:629, 1973. Number: 497 Score: The Mapleson 'A' circuit: 1. Is more efficient during spontaneous (SV) than controlled ventilation (IPPV). 2. Is available in coaxial form. 3. Can be safely used at a fresh gas flow equal to the alveolar ventilation during SV. 4. Is more efficient than the 'Bain' circuit during IPPV. A: 1,2,3 Correct B: 1,3 Correct C: 2,4 Correct D: 4 Correct E: All Correct

98

ABCDE Correct Answer: A The Mapleson 'A' circuit is the most efficient circuit available for use during spontaneous respiration and the least efficient during IPPV. A coaxial version of it (The 'Lack' circuit) is available. See also: Br J Anaesth 1998 Feb;80(2):263-9 Mapleson WW The elimination of rebreathing in various semi-closed anaesthetic systems. 1954. Number: 498 A variable bypass vaporiser set to deliver 1% halothane is in use at an altitude of ~ 10000 feet (Ambient pressure of 500 mm Hg). Which of the following statements are true? A. The delivered concentration of halothane will be about 0.66%. B. The delivered concentration of halothane will be about 1.33%. C. The delivered concentration represents a lower MAC value than would be delivered at sea level. D. The delivered concentration represents a higher MAC value than would be delivered at sea level. E. None of the above. Select the single best answer ABCDE Correct Answer: D Suppose that the vaporiser is set to deliver 1% halothane (1.33 MAC at 760 mm Hg atmospheric pressure), and is being used at an ambient pressure of 500 mm Hg and at a temperature of 20 C. In the vaporising chamber, halothane has an SVP of 243 mm Hg which represents 48.6 volumes percent (243/500) of the chamber flow. The vaporiser, set to deliver 1% at one atmosphere, creates a splitting ratio of 46:1 between bypass and vaporising chamber flows. If the total gas flow to the vaporiser is 4700 ml/min, then 100 ml/min of carrier gas passes through the vaporising chamber. This now represents 51.4% of the volume there because halothane represents the other 48.6 volumes percent (100 - 51.4). Emerging from the vaporising chamber is 100 ml/min of carrier gas plus 95 ml/min of halothane vapor ([100/51.4] 48.6). When the vaporising chamber and bypass flows merge, the 95 ml/min of halothane vapor are diluted in a total volume of 4795 ml/min (4600 + 100 + 95 ml), giving a halothane concentration of 1.98 volumes percent, or approximately 2% of the (hypobaric) atmosphere by volume. This appears to be double the dialed-in concentration in terms of volumes percent.

99

However, partial pressures must be considered because it is the tension of the anesthetic agent that is important. If halothane represents 1.98% of the gas mixture by volume, its partial pressure in the emerging mixture is 1.98% 500, or 9.90 mm Hg. In terms of anesthetic potency, this represents 1.74 MAC (9.90/5.7), because the MAC of halothane is 5.7 mm Hg. Thus, in theory, a halothane vaporiser used at a pressure of 500 mm Hg (10,000 feet) set to 1% (vol/vol) delivers twice the set concentration in terms of volumes percent but only 1.3 times the anesthetic potency in MAC (1.74/1.33). See Eisenkraft JB: Vaporizers and vaporization of volatile anesthetics. In 'Progress in anesthesiology', vol 2, San Antonio, Texas, 1989 Number: 505 The Jackson Rees modification of Ayre's 'T' piece used during spontaneous ventilation: 1. Is effective at conserving heat and moisture. 2. Offers little resistance to ventilation. 3. Will effectively eliminate rebreathing if the fresh gas flow (FGF) is twice the minute ventilation. 4. Requires a higher fresh gas flow than during controlled ventilation A: 1,2,3 Correct B: 1,3 Correct C: 2,4 Correct D: 4 Correct E: All Correct ABCDE Correct Answer: C 1. Because the circuit needs a high FGF it is not effective at conserving heat and moisture. 2. It is a very low resistance circuit. 3. When the fresh gas flow is less than the peak inspiratory flow, some inspired gas will be entrained from the expiratory limb. Thus rebreathing can only be avoided completely if FGF exceeds the peak inspiratory flow. For most patients this usually requires a fresh gas flow that is in excess of three times the minute ventilation. 4. During IPPV a lower FGF can be selected. Rebreathing will occur, but normocarbia can still be achieved. As FGF is reduced, arterial PCO2 becomes less dependent on minute ventilation and more dependent on fresh gas flow. See also: Rees GJ: Anaesthesia in the newborn, Br Med J 1:1419, 1950.

100

Number: 506 A gas mixture which is fully saturated with water vapour at 37 degrees centigrade, 1 atmosphere absolute has a water content of: A. 22 mgs/L. B. 33 mgs/L. C. 44 mgs/L. D. 55 mgs/L. E. 66 mgs/L. Select the single best answer ABCDE Correct Answer: C Air fully saturated with water vapour at 37C contains 44 mgs/L of water. The content can be calculated on the basis of Avogadro's law. (1 g molecular weight of any gas or vapour occupies 22.4 L at standard temperature and pressure.)

Number: 524 Laser radiation is: 1. Monochromatic. 2. Coherent. 3. Collimated. 4. Confined to the visible portion of the spectrum. A: 1,2,3 Correct B: 1,3 Correct C: 2,4 Correct D: 4 Correct E: All Correct ABCDE Correct Answer: A The word "laser" is an acronym for Light Amplification of the Stimulated Emission of Radiation. Laser light is produced when energy is directed at a "lasing medium." The medium lends its name to the laser, for instance in the case of the CO2 laser, electrical energy is aimed at carbon dioxide molecules. When the lasing medium is stimulated electrically, the electrons in the molecular orbit are excited and change orbital patterns in such a way as to emit energy in the form of light. Light

101

produced in this fashion is known as "coherent radiation" because it is spatially and temporally coherent, collimated (the beam does not diverge), and monochromatic. Coherent light can be focused into spots in which the power density is so great that the concentrated light can cut and vapourise tissue. Number: 526 Soda lime: 1. Is mainly composed of calcium hydroxide. 2. Can produce carbon monoxide if dry. 3. Requires the presence of water to absorb carbon dioxide. 4. Can be used to absorb sevoflurane. A: 1,2,3 Correct B: 1,3 Correct C: 2,4 Correct D: 4 Correct E: All Correct ABCDE Correct Answer: A "Wet" soda lime is composed of calcium hydroxide (about 80%), sodium hydroxide and potassium hydroxide (about 5%), water (about 15%), and small amounts of inert substances such as silica and clay for hardness. The potassium hydroxide and sodium hydroxide function as 'catalysts' to speed the initial absorption of carbon dioxide - forming sodium and potassium carbonate. The sodium and potassium carbonates then react with the calcium hydroxide to form calcium carbonate and water, thereby regenerating sodium and potassium hydroxides. Soda lime is exhausted when all the hydroxides have become carbonates. Soda lime can absorb 19% of its weight in carbon dioxide. Both soda lime and baralyme can produce carbon monoxide if flushed with dry gases for long periods of time. (Moon RE, Ingram C, Brunner EA, et al: Spontaneous generation of carbon monoxide within anesthesia circuits, Anesthesiology 75:A873, 1991) Soda lime does not absorb volatile anaesthetic agents. The relevant chemical reactions for soda lime are: CO2 + H2O -> H2CO3 H2CO3 + 2NaOH -> Na2CO3 + 2H2O H2CO3 + 2KOH -> K2CO3 + 2H2O Na2CO3 + Ca(OH)2 -> 2NaOH + CaCO3

102

or NK2CO3 + Ca(OH)2 -> 2KOH + CaCO3 Sevoflurane is unstable in the presence of soda lime, but is not usefully absorbed by it. (Tanifuji Y, Takagi MS, Kobayashi K, et al: The interaction between sevoflurane and soda lime or Baralyme, Anesth Analg 68:S285, 1989) Number: 527 With regard to the monitoring of non-depolarising neuromuscular blockade (NMB): 1. A post-tetanic count of 5 correlates with a train-of-four ratio of approximately 0.7. 2. A post-tetanic count of greater than 10 is generally considered suitable for extubation. 3. The ability to sustain head lift for 5 seconds corresponds < 50% of receptor occupancy by the relaxant. 4. The ability to sustain head lift for 3 seconds is a reliable sign of the adequacy of reversal of NMB. A: 1,2,3 Correct B: 1,3 Correct C: 2,4 Correct D: 4 Correct E: All Correct ABCDE Correct Answer: D The post-tetanic count is used for the assessment of profound NMB. Its use is only appropriate when the response to single twitch, train-of-four (ToF) and tetanic stimuli are all absent. The fourth twitch of ToF does not appear until 12-15 post-tetanic twitches are present. When the ToF ratio is about 0.6, patients can sustain a head lift for 3.0 seconds (See : Ali HH, Utting JE, and Gray TC: Quantitative assessment of residual antidepolarizing block (part I), Br J Anaesth 43:473, 1971. and Ali HH, Wilson RS, Savarese JJ et al: The effect of tubocurarine on directly elicited train-offour muscle response and respiratory measurements in humans, Br J Anaesth 47:570, 1975.) 70-80% of receptors have to be occupied by neuromuscular blocking drug before the response to nerve stimulation is affected. Accordingly, during recovery from neuromuscular blockade even with normal inspiratory force, vital capacity, protrusion of the tongue and sustained head lift for 3 seconds, 70-80% of all receptors can still be occupied by the neuromuscular blocker. The ability to sustain head lift for 3 seconds is a good sign of the adequacy of reversal of NMB

103

Number: 550 The following capnograph trace was obtained from an intubated patient who was ventilated using a circle system with the absorber on. The trace is most compatible with: A. A correctly positioned tube. B. Inspiratory valve malfunction. C. Oesophageal intubation. D. Expiratory valve malfunction. E. Right main bronchial intubation. Select the single best answer

ABCDE Correct Answer: E The tube is in the right main bronchus. - Under these circumstances the left lung is poorly ventilated (ie has a low overall V:Q ratio) and a long overall time constant. Thus the initial plateau represents gas exchange in the right lung and the later plateau that in the left.

Number: 553 Which statement is true concerning medical gas cylinders: A. They are made of titanium / aluminium alloy. B. The colour and shape of the plastic disc around the neck of the cylinder identifies the gas it contains. C. The tare weight is the weight of the cylinder when full. D. The filling ratio is the weight of the fluid in the cylinder divided by the weight of the cylinder and fluid together. E. The cylinders attached to the anaesthetic machine are usually size E. Select the single best answer 104

ABCDE Correct Answer: E Cylinders are made of lightweight molybdenum steel (a chromium : steel alloy). The year when the cylinder was last tested can be identified from the shape and colour of the disc. The body and shoulder of the cylinder are colour-coded to identify the gas it contains. Tare weight is the weight of the cylinder when empty. The filling ratio is the weight of the fluid in the cylinder divided by the weight of water required to fill the cylinder. The cylinders attached to the anaesthetic machine are usually size E.

Number: 554 Which statements are true regarding the desflurane 'TEC 6' vapouriser? 1. It is a 'plenum' type vapouriser. 2. It has a warm-up period of about 3 minutes. 3. It can be used at a fresh gas flow of 500 mls/min. 4. It can provide desflurane at a concentration of up to 18%. A: 1,2,3 Correct B: 1,3 Correct C: 2,4 Correct D: 4 Correct E: All Correct ABCDE Correct Answer: E All statements are correct. Desflurane liquid is held within the vaporising chamber and is heated to 39 C by two 100W electrical heater elements. Two further heater elements warm the upper part of the vapouriser to prevent condensation. When operating temperature has been reached, the upper part of the

105

vaporising chamber contains desflurane vapour under pressure and solenoidoperated locks on the concentration dial and vapourising chamber outflow are released. Fresh gas from the flowmeters enters the vapouriser and passes through a fixed flow restrictor, to generate backpressure, sensed by two independent pressure sensors. Desflurane vapour passes through a shutoff valve and a pressure regulating valve to the concentration selection dial. The pressure of the vapour is detected also by these pressure sensors and by control of the pressureregulating valve the pressure may be matched to the backpressure generated by the fresh gas flows. This allows one single dial setting to deliver fixed concentrations of desflurane vapour on the output side of the vapouriser at varying fresh gas flows; higher flowmeter flows produce a larger measured backpressure, matched by a larger pressure of desflurane vapour, with a resultant greater flow of desflurane vapour from the vapouriser matching the increase in fresh gas flow. The concentration selector has a dial release bar on the back, which is compressed when initially setting a desired concentration. The dial is graduated in increments of 1% between 0 and 10%, and in increments of 2% between 10 and 18%. An interim stop is located at 12%, which requires depression of the dial release bar to bypass, in a manner similar to that used on the 7% Enfluratec 3 vaporizer. The desflurane Tec 6 vaporizer Br. J. Anaesth. 1994; 72:470-473: G Stephen. Number: 555 Which of the following techniques are absolutely contraindicated in a patient with an automatic implantable cardioverter defibrillator (AICD)? 1. Radiotherapy. 2. Extracorporeal shock wave lithotripsy. 3. Electroconvulsive therapy. 4. Magnetic resonance imaging. A: 1,2,3 Correct B: 1,3 Correct C: 2,4 Correct D: 4 Correct E: All Correct ABCDE Correct Answer: D Magnetic resonance imaging is contraindicated in patients with AICD as strong magnetic fields produced can exert mechanical forces causing physical pain and damage to the generator, or cause inappropriate delivery of a shock. Radiotherapy (linear accelerator, betatron, radioactive cobalt) may damage the complementary metal oxide semiconductor circuits of the AICD, especially with cumulative

106

doses. Therefore, shielding of the generator should be undertaken and the device evaluated after its exposure to radiation. Theoretically, extracorporeal shock wave lithotripsy can produce mechanical and electromagnetic forces which can damage the piezoelectric crystal timing device of the AICD or cause inappropriate delivery of a discharge. However, bench analysis and a case report suggest that contralateral lithotripsy can be performed safely in patients with an AICD in situ. The safety of lithotripsy applied ipsilateral to the AICD is unknown. As electroconvulsive therapy (ECT) frequently induces transient arrhythmias, the AICD should be deactivated just before treatment and reactivated immediately after ECT. Grounding of the patient should be avoided as the delivered current may be rerouted to the heart via the defibrillator electrodes resulting in VF. Anaesthetic management of a patient with an automatic implantable cardioverter defibrillator in situ. Br. J. Anaesth. 1997; 78:102-106: P. C. A. Kam. Number: 556 Which of the following solutions would be suitable for transcervical endometrial ablation (TCEA) using electrodiathermy? 1. 1.5% Glycine. 2. Normal Saline. 3. 5% Sorbitol. 4. Distilled Water. A: 1,2,3 Correct B: 1,3 Correct C: 2,4 Correct D: 4 Correct E: All Correct ABCDE Correct Answer: B Electrodiathermy requires the use of solutions which do not conduct electricity. These include 1.5% glycine, 5% sorbitol and 5% mannitol. The most commonly used irrigation fluid is 1.5% glycine. It has good optical qualities but is hypotonic (osmolality of 1.5% glycine is 188 mosmol kg-1) and absorption into the circulation may cause fluid overload, hyponatraemia, pulmonary oedema and cerebral oedema. Sorbitol 5% is isotonic with a short half-life of 35 min but it causes toffee-like deposits on the electrode and this may make the resection technically difficult. Consequently, it is seldom used.

107

Mannitol 5%, although isotonic, has a long half-life and is also seldom used; it may cause visual distortion and crystallization on instruments. Normal saline may be used only when laser treatment is contemplated and there is no danger of electrical conduction. It is isotonic and optically clear but it may also lead to fluid overload and pulmonary oedema if absorbed in large quantities. Distilled water should never be used. Complications of hysteroscopic treatments of menorrhagia. Br. J. Anaesth. 1996; 77:305-8: Williamson, K.M.; Mushambi, M.C. Number: 564 The Raman spectrometer cannot be used to measure the partial pressure of which of the following gases? A. Oxygen. B. Helium. C. Carbon Dioxide. D. Nitrogen. E. Nitrous Oxide. Select the single best answer ABCDE Correct Answer: B When light strikes gas molecules, most of the energy scattered is absorbed and re-emitted in the same direction and at the same wavelength as the incoming beam (Rayleigh scattering). At room temperature, about 1/1,000,000 of the energy is scattered at a longer wavelength, producing a so called red-shifted spectrum. This Raman scattering can be used to measure the constituents of a gas mixture. Unlike infrared spectroscopy, Raman scattering is not limited to gas species that are polar. Carbon dioxide, oxygen, nitrogen, water vapour, nitrous oxide, and all of the volatile anaesthetic agents exhibit Raman activity. Only monatomic gases such as helium, xenon, and argon, which lack intramolecular bonds, do not exhibit Raman activity.

108

Number: 566 Which of the following nerve:muscle combinations may be used for monitoring a 'Train-offour'? 1. Ulnar nerve:Abductor pollicis brevis 2. Common peroneal nerve:Tibialis anterior 3. Ulnar nerve:Opponens pollicis 4. Facial nerve:Orbicularis oculi A: 1,2,3 Correct B: 1,3 Correct C: 2,4 Correct D: 4 Correct E: All Correct ABCDE Correct Answer: C Abductor pollicis brevis and opponens pollicis are supplied by the median nerve. The 'appropriate' combination in the hand is the ulnar nerve and Adductor pollicis. Number: 624 In this capnograph of a patient breathing spontaneously through a 'Bain' circuit there is evidence of: 1. Hypoventilation. 2. Segmentation of gas in the outer limb. 3. Rebreathing. 4. Disconnection of the inner limb at the machine end. A: 1,2,3 Correct B: 1,3 Correct C: 2,4 Correct D: 4 Correct E: All Correct

109

ABCDE Correct Answer: A The end-tidal CO2 is elevated which confirms hypoventilation. The 'blip' in the inspiratory phase of the trace reflects segmentation of gas in the outer limb as dead-space, alveolar and fresh gas fail to mix in the outer limb. - During inspiration the patient is sequentially ventilated with fresh gas which has entered the outer limb durin the expiratory pause, alveolar gas and then anatomical dead-space gas (which has less CO2 in it than the expired alveolar gas). Clearly there is re-breathing. Disconnection of the inner limb at the machine end produces almost complete re-breathing by adding a large, serial dead-space. There is no evidence of this. Number: 627 Regarding the desflurane sump in a 'TEC 6' vapouriser: 1. It is heated to 39 degrees centigrade. 2. The operating pressure in the sump is about three atmospheres absolute. 3. The vapour pressure in the sump is about 1500 mm Hg. 4. Up to 18% of the fresh gas flow passes through the sump. A: 1,2,3 Correct B: 1,3 Correct C: 2,4 Correct D: 4 Correct E: All Correct ABCDE Correct Answer: B The vapour circuit originates at the desflurane sump, which is electrically heated and thermostatically controlled to 39C. This circuit is completely separate from the fresh gas circuit and none of the fresh gas enters the sump. At 39C, the vapour pressure in the sump is approximately 1500 mm Hg absolute, or approximately 2 atmospheres.

110

Number: 628 Concerning the measurement of oxygen by a paramagnetic device: 1. The technique utilises the fact that oxygen is strongly attracted to a magnetic field. 2. The glass spheres in the device usually contain nitrogen. 3. No anaesthetic gases exhibit paramagnetism. 4. Water vapour must be removed from the sample before the measurement is made. A: 1,2,3 Correct B: 1,3 Correct C: 2,4 Correct D: 4 Correct E: All Correct ABCDE Correct Answer: E The two glass spheres are filled with a weakly diamagnetic gas such as nitrogen. Paramagnetic analysers are affected by the presence of high concentrations of diamagnetic background gases such as nitrous oxide, carbon dioxide or water vapour. The only other paramagnetic gas of occasional importance to anaesthetists is nitric oxide. For an explanation of the difference between paramagnetism and diamagnetism, visit the website at http://www.sci.kun.nl/hfml/. Number: 632 Concerning arterial PCO2 (PaCO2) and End-Tidal CO2 : Arterial CO2 tension difference (DCO2), this capnograph of a patient breathing spontaneously through a circle-absorber system suggests that: A. PaCO2 is high and DCO2 is normal. B. PaCO2 is high and DCO2 is high. C. PaCO2 is high and DCO2 is low. D. PaCO2 is normal and DCO2 is high. E. PaCO2 is normal and DCO2 is normal. Select the single best answer

111

ABCDE Correct Answer: B The End-Tidal CO2 is high (~46 mm Hg) and therefore the PaCO2 must also be high (unless DCO2 is negative which is an extremly unusual finding). Note also that the plateau phase (phase III) is still upsloping at the end of expiration. This suggests that a true end-tidal sample has not been measured and also that the patient has an increased dispersion of alveolar V/Q ratios with widely varying time constants. For all these reasons DCO2 is also likely to be increased. Number: 641 The elective use of a laryngeal mask airway has been advocated for: 1. Ventilation of children undergoing closure of an atrial septal defect. 2. Airway management for anaesthesia in the prone position. 3. Ventilation of fresh cadavers. 4. Ventilation of adults undergoing coronary artery grafting. A: 1,2,3 Correct B: 1,3 Correct C: 2,4 Correct D: 4 Correct E: All Correct ABCDE Correct Answer: A The things people do!! - Just because it's possible that surely doesn't make it right. As far as I can tell, the LMA has not yet been used (electively) in the context of coronary artery grafting in adults. Sooner or later I suppose that someone will tell me I'm wrong.

112

Number: 649 This tracing was recorded from the distal lumen of a Swan-Ganz catheter inserted to a distance of 52 cms via the left subclavian vein with balloon inflated. The waveform shown below most probably represents: A. An unwedged pulmonary artery pressure trace. B. A wedged pulmonary artery pressure trace in a patient with mitral incompetence. C. A right ventricular pressure trace. D. 'Cannon' waves in a right atrial pressure trace. E. A coronary sinus pressure trace. Select the single best answer

ABCDE Correct Answer: B This is, in fact, a wedged pulmonary artery pressure trace from a patient with severe mitral incompetence and large 'V' waves. The key to the diagnosis is the timing of the wave in relation to the 'T' wave. The normal pulmonary artery systolic peak occurs at about the time of the 'T' wave, whereas a 'V' wave occurs after the 'T' wave. This trace illustrates the dangers of using pulmonary artery catheters in patients with 'V' waves. - It is easy to imagine that someone could go on inflating the balloon in the presence of this waveform and thus rupture the pulmonary artery. Similarly, the systolic peaks of the right ventricular and coronary sinus pressure waves approximate to the 'T' wave. Cannon waves do not occur in patients (such as this) who are in normal sinus rhythm.

113

Number: 670 The value of the bispectral index which will prevent 50% of patients responding to a verbal command is: A. > 90. B. > 80 <= 90. C. > 65 <= 80. D. > 50 <= 65. E. <= 50. Select the single best answer ABCDE Correct Answer: C The BIS that prevents 50% of subjects from responding to verbal command ranges from 67 to 79. The BIS that prevents implicit memory is between 8491. Recovery of consciousness is expected as BIS rises above 90. References: Glass, PS, Bloom, M, Kearse, L, Rosow, C, Sebel, P & Manberg, P. Bispectral analysis measures sedation and memory effects of propofol, midazolam, isoflurane, and alfentanil in healthy volunteers. Anesthesiology 1997; 86, 836-847. Katoh, T, Suzuki, A & Ikeda, K. Electroencephalographic derivatives as a tool for predicting the depth of sedation and anesthesia induced by sevoflurane. Anesthesiology 1998; 88, 642650. Leslie, K, Sessler, DI, Schroeder, M & Walters, K. Propofol blood concentration and the Bispectral Index predict suppression of learning during propofol/epidural anesthesia in volunteers. Anesth Analg 1995; 81, 1269-74. Iselin-Chaves, IA, Flaishon, R, Sebel, PS et al. The effect of the interaction of propofol and alfanianil on recall, loss of consciousness, and the Bispectral Index. Anesth Analg 1998; 87, 949-955. Kearse, LA, JrC Rosow, Zaslavsky, A, Connors, P, Dershwitz, M & Denman, W. Bispectral analysis of the electroencephalogram predicts conscious processing of information during propofol sedation and hypnosis. Anesthesiology 1998; 88, 25-34. Liu, J, Singh, H & White, PF. Electroencephalogram bispectral analysis predicts the depth of midazolam-induced sedation. Anesthesiology 1996; 84, 64-69.

114

Number: 697 The International Standards Organization recommendation for the minimum humidity generated by a heat and moisture exchanger is: A. 44 mg H2O/L. B. 20 mg H2O/L. C. 25 mg H2O/L. D. 30 mg H2O/L. E. 33 mg H2O/L. Select the single best answer ABCDE Correct Answer: D At 37 degrees C, fully humidified gas contains 44 mg/L of water at a partial pressure of 47 mm Hg. Surprisingly, there are no really clear guidelines for what constitutes an 'acceptable' or 'ideal' water content of gases used in the ventilation of patients in the operating theatre or the intensive care unit. The British Standard Institution recommended in 1970 that optimum tracheal temperature and absolute humidity should be 35C and 33 mg H2O/L respectively. In 1988, the International Standards Organization (ISO) presented a draft standard for heat and moisture exchangers. They set a minimum humidity of 30 mgH2O/L for safe and effective use. The optimal humidity of the inspired gases during mechanical ventilation has been reported in adults as anywhere between 17 - 44 mgH2O/L. Number: 698 A gas which has a relative humidity (RH) of 70% at 37 degrees C is cooled to 30 degrees C, its RH at this temperature will be: A. 60%. B. 70%. C. 80%. D. 90% E. 100%. Select the single best answer

115

ABCDE Correct Answer: E An RH of 70% at 37 degrees C corresponds to an Absolute Humidity of 30 mg/L. At 30 degrees C, this water content corresponds to an RH of 100%. Thus it is never possible to attain an RH of 70% at the airway unless the ventilating gas is warmed to at least 30 degrees C. Number: 699 A 20-year-old, negro male is undergoing an exploratory laparotomy and partial hepatectomy following a motor vehicle accident. General anaesthesia is maintained with isoflurane in 100% oxygen and paralysis with vecuronium. His vital signs are: blood pressure 100/65 mmHg, temperature 34.7 C, Hb 80 gm/L. His oxygen saturation is noted to be only 85% by pulse oximetry, but is calculated at 98% on subsequent blood gas analysis. This discrepancy might be due to: 1. Anaemia. 2. Sickle Cell Disease. 3. Interference by ambient light. 4. Hypoperfusion. A: 1,2,3 Correct B: 1,3 Correct C: 2,4 Correct D: 4 Correct E: All Correct ABCDE Correct Answer: D Provided that a good waveform is present, anaemia does not significantly affect the accuracy of a pulse oximeter. See, for example: Jay GD, Hughes L, Renzi FP. Pulse oximetry is accurate in acute anemia from hemorrhage. Ann Emerg Med. 1994 Jul;24(1):32-5. In a recent study, Ortiz et al (Ortiz FO, Aldrich TK, Nagel RL, Benjamin LJ. Accuracy of pulse oximetry in sickle cell disease. Am J Respir Crit Care Med. 1999 Feb;159(2):447-51.) have concluded that "as long as strong and regular photoplethysmographic waves are present, pulse oximeters can be relied upon not to misdiagnose either hypoxemia or normoxemia in Sickle Cell Disease." Interference by ambient light tends to raise (not lower) the reading obtained by pulse oximetry.

116

Number: 703 With regard to the laminar flow of a fluid through a tube, the flow rate is directly proportional to: 1. The length of the tube. 2. The fourth power of the radius of the tube. 3. The viscosity of the fluid. 4. The pressure difference between the ends of the tube. A: 1,2,3 Correct B: 1,3 Correct C: 2,4 Correct D: 4 Correct E: All Correct ABCDE Correct Answer: C The Hagen-Poiseuille equation describes the determinants of the laminar flow of a fluid through a tube. The equation is: V = (Delta P*Pi*r4)/(8*L*mu): Where V is the flow rate, Delta P is the pressure difference between the ends of the tube, Pi is Pi, r4 is the fourth power of the radius of the tube, L is the length of the tube and mu is the viscosity of the fluid. Thus flow rate is inversely related to the length of the tube and the viscosity of the fluid. Number: 736 The average distance from skin puncture site to atrio-caval junction when the left subclavian (infraclavicular) approach is used for central venous access in an adult is: A. 12 cms. B. 15 cms. C. 18 cms. D. 21 cms. E. 24 cms. Select the single best answer

117

ABCDE Correct Answer: D The most effective way to induce arrhythmias during the insertion of a central venous (or PA) catheter is to pass the guide wire too far into the vein. Andrews et al have recently measured the distance to the atrio-caval junction from skin puncture site for various approaches to the central circulation. For the left subclavian vein the distance in adults is about 21 cms. Andrews RT, Bova DA, Venbrux AC. How much guidewire is too much? Direct measurement of the distance from subclavian and internal jugular vein access sites to the superior vena cava-atrial junction during central venous catheter placement. Crit Care Med 2000 Jan;28(1):138 - 42. Number: 741 The average distance from skin puncture site to atrio-caval junction when the right internal jugular approach is used for central venous access in an adult is: A. 10 cms. B. 13 cms. C. 16 cms. D. 19 cms. E. 22 cms. Select the single best answer ABCDE Correct Answer: C The most effective way to induce arrhythmias during the insertion of a central venous (or PA) catheter is to pass the guide wire too far into the vein. Andrews et al have recently measured the distance to the atrio-caval junction from skin puncture site for various approaches to the central circulation. For the right internal jugular vein the distance in adults is about 16 cms. Andrews RT, Bova DA, Venbrux AC. How much guidewire is too much? Direct measurement of the distance from subclavian and internal jugular vein access sites to the superior vena cava-atrial junction during central venous catheter placement. Crit Care Med 2000 Jan;28(1):138 - 42.

118

Number: 826 An adult patient is quietly breathing supplemental oxygen from a '24%' venturi mask supplied with oxygen at 2 litres per minute. The delivered oxygen concentration (FiO2) will be significantly affected if: A. The patient doubles his tidal volume. B. The patient doubles his peak inspiratory flow rate (PIFR). C. The fresh gas flow to the mask is doubled. D. The fit of the mask to the patient's face is loosened. E. None of the above. Select the single best answer ABCDE Correct Answer: E HAFOE (High Air Flow Oxygen Enrichment) masks are designed to deliver a constant inspired oxygen at a flow rate which considerably exceeds the patient's own PIFR. Thus, even if mask fit is relatively poor, the achieved FiO2 will not be affected. Doubling the fresh gas flow through the venturi will not alter the entrainment ratio of the mask, but will obviously increase the delivered gas flow to the patient. Number: 827 The completeness of pre-oxygenation is best reflected by measurement of: A. Arterial Oxygen Saturation (SaO2). B. Arterial Oxygen Partial Pressure (PaO2). C. End-tidal Oxygen concentration (ETO2). D. End-tidal Nitrogen concentration (ETN2). E. None of the above. Select the single best answer ABCDE Correct Answer: D The purpose of pre-oxygenation is the replacement of alveolar nitrogen with oxygen. This is best reflected by the ETN2. The 'next best' would be ETO2 - if your monitoring system cannot measure the former. A SaO2 of 100% merely reflects the fact that PaO2 is something over ~ 100 mm Hg and PaO2 itself is obviously affected by the proportion of venous admixture.

119

Number: 827 The completeness of pre-oxygenation is best reflected by measurement of: A. Arterial Oxygen Saturation (SaO2). B. Arterial Oxygen Partial Pressure (PaO2). C. End-tidal Oxygen concentration (ETO2). D. End-tidal Nitrogen concentration (ETN2). E. None of the above. Select the single best answer ABCDE Correct Answer: D The purpose of pre-oxygenation is the replacement of alveolar nitrogen with oxygen. This is best reflected by the ETN2. The 'next best' would be ETO2 - if your monitoring system cannot measure the former. A SaO2 of 100% merely reflects the fact that PaO2 is something over ~ 100 mm Hg and PaO2 itself is obviously affected by the proportion of venous admixture.

Number: 829 A hypothetical volatile anaesthetic agent with a saturated vapour pressure of 152 mmHg at 20C is placed in a copper kettle vapouriser. Gas flow into the vapouriser is 0.2 l/min. The patient flowmeter is delivering oxygen at 1.75 l/min. At 20C the concentration of the volatile anaesthetic agent delivered in the final mixture is: A. 0.5% B. 1% C. 2.5% D. 5% E. 10% Select the single best answer ABCDE Correct Answer: C Copper kettle vapourisers are obviously little used devices nowadays, but they do illustrate some important principles. First calculate the total volume of (fully saturated) vapour leaving the kettle each minute. We're told that the SVP of the agent is 20% of an atmosphere, so the volume leaving the kettle must be 250 mls (200mls oxygen + 50 mls of agent).

120

Now add in the fresh gas coming from the patient flow meters (1.75 l) to get a total gas volume of 2.0 l. This volume contains 50 mls of agent, so the final concentration is 50/2000 (or 2.5) %. Number: 830 The vapouriser of an ADU (Anesthesia Delivery Unit; Datex-Ohmeda, Helsinki, Finland) is loaded with a sevoflurane 'Aladin' cassette. The dial setting of the agent is 4%. The total fresh gas flow into the circuit is 5 l/minute. The delivered concentration of sevoflurane will remain within 10% of the dial setting if: 1. The fresh gas flow is changed to a 50:50 mixture of O2:N2O. 2. The fresh gas flow is reduced to 1 l/min. 3. The ambient temperature rises from 20c to 25c. 4. The fresh gas flow is increased to 10 l/min. A: 1,2,3 Correct B: 1,3 Correct C: 2,4 Correct D: 4 Correct E: All Correct ABCDE Correct Answer: A Refer to the paper by Hendrickx et al for a comprehensive review of this vapouriser. If the fresh gas flow is changed to a mixture of O2:N2O and the total flow remains at 5 l/min, the delivered concentration of sevoflurane will remain within 10% of the dial setting - as the effect of changes in gas composition only become apparent at relatively high fresh gas flows. The delivered concentration of sevoflurane remains within 10% of the dial setting down to a flow rate of about 0.5 l/min. The vapouriser is well temperature compensated within this range. At high fresh gas flows, the vapouriser tends to deliver a lower concentration of agent. Anesth Analg 2001 Aug;93(2):391-5 The adu vaporizing unit: a new vaporizer. Hendrickx JF, De Cooman S, Deloof T. In their discussion, these authors remarked "Vaporizer output also depended on carrier gas composition, dial setting, and the anesthetic used. As fresh gas flow increases, vaporizer output decreases despite the use of a fan to facilitate heat transfer to the vaporizer, probably because the amount of heat transfer to the vaporizing chamber becomes insufficient, and hence output decreases. The effect of higher fresh gas flows on vaporizer output was highly dependent on the duration of the high fresh gas flow and may have obscured the effects of carrier gas composition. Despite the use of anemometers and algorithms that electronically

121

control vaporizer output, vaporizer output was not independent of carrier gas composition. Changing carrier gas composition changes the viscosity and hence the flow rate across the anemometer: the viscosities of oxygen, air, and 70% N2O in oxygen are 210, 190, and 171 micropoises, respectively. These very same physical principles explain the effect of carrier gas composition on vaporizer output in conventional vaporizers. The performance of the Aladin cassettes is not the same for different anesthetics. Because the cassette design for the three anesthetics is similar, the effect might in part be related to differences in the relative quantities of liquid anesthetic that need to be vaporized to attain the desired concentration. " Number: 835 Xenon concentration in an anaesthetic gas mixture may be measured using: 1. Mass spectrometry. 2. Piezoelectric adsorption. 3. Thermal conductivity. 4. Infra red absorption spectroscopy. A: 1,2,3 Correct B: 1,3 Correct C: 2,4 Correct D: 4 Correct E: All Correct ABCDE Correct Answer: A According to Dingley et al "Being a noble gas, conventional methods cannot be used and only methods based on its physical properties can be applied. The Drager Physioflex device initially used a mass spectrometer. This was effective but far too expensive. Other lower cost devices have included piezoelectric adsorption, thermal conductivity and ultrasound. The piezoelectric method depends on adsorption of gas into an oil film on an oscillating quartz crystal, so changing its frequency of oscillation. The thermal conductivity method exploits the fact that xenon conducts heat far better than any other gas likely to be found in the breathing system. The ultrasonic method has been successfully used recently for the first time and exploits the fact that xenon as a very dense gas will conduct sound faster than any other gas likely to be found in the breathing system." Infra red absorption spectroscopy is only applicable to the measurement of polyatomic molecules such as CO2 or N2O.

122

Number: 875 With regard to electroencephalograph (EEG): 1. Voltages are generally in the range of 10 -100 microvolts. 2. Spontaneous EEG activity is lost when body temperature drops below ~22 centigrade. 3. Beta activity is accentuated by sedative-hypnotic drugs. 4. Theta waves occur at frequency of 3.5 - 7.5 Hz. A: 1,2,3 Correct B: 1,3 Correct C: 2,4 Correct D: 4 Correct E: All Correct ABCDE Correct Answer: E EEG voltages are generally in the range of 10 -100 microvolts. Electro-cerebral silence occurs at a temperature below 22 - 25 degrees C. Beta activity is 'fast' activity. It has a frequency of 14 and greater Hz. It is usually seen on both sides in symmetrical distribution and is most evident frontally. It is accentuated by sedative-hypnotic drugs especially the benzodiazepines and the barbiturates. It may be absent or reduced in areas of cortical damage. It is generally regarded as a normal rhythm. It is the dominant rhythm in patients who are alert or anxious or who have their eyes open. Theta activity has a frequency of 3.5 to 7.5 Hz and is classed as "slow" activity. It is abnormal in awake adults but is perfectly normal in children up to 13 years and in sleep. It can be seen as a focal disturbance in focal subcortical lesions; it can be seen in a generalised distribution in diffuse disorders, metabolic encephalopathy, deep midline disorders and some instances of hydrocephalus. Number: 941 A small air bubble in radial artery catheter system will consistently reduce the: A. Resonant frequency of the system. B. Damping of the system. C. Recorded systolic blood pressure. D. Recorded diastolic blood pressure. E. Recorded mean blood pressure. Select the single best answer

123

ABCDE Correct Answer: A Air bubbles in such a system will consistently reduce its resonant frequency and increase its damping. This will have an unpredictable effect on the final accuracy of the systolic and diastolic measurements and little (if any) effect on the accuracy of the mean measurement. Hipkins SF, Rutten AJ, Runciman WB. Experimental analysis of catheter-manometer systems in vitro and in vivo. Anesthesiology. 1989 Dec;71(6):893-906. Kleinman B, Powell S, Gardner RM. Equivalence of fast flush and square wave testing of blood pressure monitoring systems. J Clin Monit. 1996 Mar;12(2):149-54. Number: 944 Which of the following units are BASIC SI units of measurement? 1. Gram. 2. Second. 3. Watt. 4. Meter. A: 1,2,3 Correct B: 1,3 Correct C: 2,4 Correct D: 4 Correct E: All Correct ABCDE Correct Answer: C There are seven basic units of measurement in the SI system. These are: Unit of length: meter: The meter is the length of the path travelled by light in vacuum during a time interval of 1/299 792 458 of a second. Unit of mass: kilogram: The kilogram is the unit of mass; it is equal to the mass of the international prototype of the kilogram. Unit of time: second: The second is the duration of 9 192 631 770 periods of the radiation corresponding to the transition between the two hyperfine levels of the ground state of the cesium 133 atom. Unit of electric current: ampere: The ampere is that constant current which, if maintained in two straight parallel conductors of infinite length, of negligible circular cross-section, and

124

placed 1 meter apart in vacuum, would produce between these conductors a force equal to 2 x 10-7 newton per meter of length. Unit of thermodynamic temperature: kelvin: The kelvin, unit of thermodynamic temperature, is the fraction 1/273.16 of the thermodynamic temperature of the triple point of water. Unit of amount of substance: mole: 1. The mole is the amount of substance of a system which contains as many elementary entities as there are atoms in 0.012 kilogram of carbon 12; its symbol is "mol." 2. When the mole is used, the elementary entities must be specified and may be atoms, molecules, ions, electrons, other particles, or specified groups of such particles. Unit of luminous intensity: candela: The candela is the luminous intensity, in a given direction, of a source that emits monochromatic radiation of frequency 540 x 1012 hertz and that has a radiant intensity in that direction of 1/683 watt per steradian. Number: 954 Which of the following lung volumes CANNOT be measured by simple spirometry? A. Vital Capacity. B. Inspiratory Reserve Volume. C. Expiratory Reserve Volume. D. Tidal Volume. E. Residual Volume. Select the single best answer ABCDE Correct Answer: E Vital capacity, tidal volume, inspiratory reserve and expiratory reserve can all be measured with a simple spirometer. Total lung capacity, functional residual capacity and residual volume all contain a fraction (the residual volume) which cannot be measured by simple spirometry. However, if one of these volumes is measured (most commonly the FRC), the others can easily be derived. Three techniques are available for the measurement of FRC. The first employs nitrogen washout by breathing 100% oxygen. Total quantity of nitrogen eliminated is measured as the product of the expired volume collected and the concentration of nitrogen. If, for example, 4 litres of nitrogen are collected and the initial alveolar nitrogen concentration was 80%, then the initial lung volume was 5 litres. The second method uses the wash-in of a tracer gas such as helium, the concentration of which may be conveniently measured by catharometry. If, for example, 50 mI of helium is introduced into the lungs and the helium concentration is then found to be 1%, the lung volume is 5 litres.

125

The third method uses the body plethysmograph. The subject is totally contained within a gastight box and he attempts to breathe against an occluded airway. Changes in alveolar pressure are recorded at the mouth and compared with the small changes in lung volume, derived from pressure changes within the plethysmograph. Application of Boyle's law then permits calculation of lung volume. This method would include trapped gas which might not be registered by the two previous methods. Number: 955 Which of the following gases is paramagnetic? A. Nitrous Oxide. B. Nitric Oxide. C. Carbon Dioxide. D. Nitrogen. E. Helium. Select the single best answer ABCDE Correct Answer: B An atom with all of its orbitals filled, and therefore all of its electrons paired with an electron of opposite spin, will be very little affected by magnetic fields. Such atoms are called diamagnetic. Conversely, paramagnetic atoms do not have all of their electrons spin-paired and are affected by magnetic fields. In the context of anaesthesia, the two important paramagnetic gases are oxygen and nitric oxide. Number: 956 The most sensitive method available for the detection of venous air-embolism (VAE) occuring during anaesthesia is: A. Electrocardiography. B. Capnography. C. Two dimensional, trans-oesophageal echocardiophy. D. M-Mode, trans-oesophageal echocardiophy. E. Pre-cordial Doppler monitoring. Select the single best answer ABCDE Correct Answer: C Two dimensional, trans-oesophageal echocardiography provides by far the most sensitive and specific indication of VAE. In addition, it can also be used to examine the integrity of the atrial septum - which may be of considerable importance should VAE occur. However, the

126

use of the technique requires considerable skill and it is clinically inappropriate in many situations. M-Mode, trans-oesophageal echocardiophy is of no practical use in the detection of VAE. Both capnography and pre-cordial Doppler monitoring have an important role to play in the detection of clinically significant VAE. ECG changes are relatively insensitive and non-specific. Number: 965 Which of the following muscles is LEAST sensitive to the effects of a non-depolarising muscle relaxant? A. Lateral Cricoarytenoid. B. Adductor Pollicis. C. Abductor Digiti Minimi. D. Flexor Hallucis Brevis. E. Gastrocnemius. Select the single best answer ABCDE Correct Answer: A The laryngeal adductors (Lateral Cricoarytenoid and Interarytenoid) are amongst the muscles which are least sensitive to the effects of neuro-muscular blocking drugs (NMBD's). In contrast, Adductor Pollicis (AP) is relatively sensitive to the NMBD's. Thus, if it is imperative that a patient not cough or buck during a procedure, simple train-of-four monitoring of the AP may not be sufficient to guarantee 'peace and tranquility' and a technique such as post-tetanic counting should be used. Torda has recently published an excellent review entitled "Monitoring Neuromuscular Transmission". See: Torda TA. Monitoring Neuromuscular Transmission. Anaesth. Int. Care. 2002, April (30): 123-133. Number: 968 With regard to a central venous catheter: 1. The tip of the catheter should be above the level of the carina on chest X-Ray. 2. The risk of line colonisation is greater if the jugular rather than subclavian route is used. 3. The risk of catheter perforation is greater if the catheter is inserted using a peripheral route. 4. At the time of insertion, the catheter tip can be reliably positioned using electrocardiography. A: 1,2,3 Correct

127

B: 1,3 Correct C: 2,4 Correct D: 4 Correct E: All Correct ABCDE Correct Answer: E 1. Conventional wisdom dictates that the catheter tip should lie outside the pericardial reflection - in order to obviate the risk of tamponade should vascular perforation occur. This point has been found to be consistently at or below the level of the carina in a cadaveric study by Schuster et al. 2. Several studies have found that the risk of line colonisation and / or infection is significantly increased if the jugular rather than subclavian route is used for line insertion. See for example, Mermel et al. 3. The risk of catheter perforation is widely believed to be greater if the catheter is inserted using a peripheral rather than central approach - due to the greater propensity for movement by the catheter tip. This seems to be a reasonable proposition, although there does not appear to be any evidence to support it. 4. Catheter tip localisation can be reliably achieved if the catheter itself is used as an intravascular electrode. Typical changes in the morphology of the 'P' wave are observed as the catheter passes the atrio-caval junction. ('The equivalence point'). Special kits - which incorporate a suitable electrode as a guide wire are available from some manufacturers See: Schuster M, Nave H, Piepenbrock S, Pabst R, Panning B. The carina as a landmark in central venous catheter placement. Br J Anaesth. 2000 Aug;85(2):192-4. Mermel LA, McCormick RD, Springman SR, Maki DG. The pathogenesis and epidemiology of catheter-related infection with pulmonary artery Swan-Ganz catheters: a prospective study utilizing molecular subtyping. Am J Med 1991 Sep 16;91(3B):197S-205S Watters VA, Grant JP. Use of electrocardiogram to position right atrial catheters during surgery. Ann Surg. 1997 Feb;225(2):165-71.

128

Number: 977 A Bispectral Index (BIS) monitor: 1. Will reliably predict the likelihood of a haemodynamic response to a noxious stimulus. 2. Will generate a BIS index of zero in the presence of an iso-electric EEG. 3. Will reliably predict the likelihood of movement in response to a noxious stimulus. 4. Incorporates the burst suppression ratio in its analysis of the EEG signal. A: 1,2,3 Correct B: 1,3 Correct C: 2,4 Correct D: 4 Correct E: All Correct ABCDE Correct Answer: C It is important to understand that the BIS index is a statistical function that has been tuned to correlate with the degree of sedation. It does not correlate with the 'depth' of anaesthesia as judged by somatic or autonomic responses to noxious stimuli and there is abundant evidence to support this view. For example see: Driessen, JJ, Harbers, JIBM, van Egmond, J & Booij, LHDJ. Evaluation of the electroencephalographic bispectral index (BIS) during fentanyl-midazolarn anaesthesia for cardiac surgery. Does it predict haemodynamic responses during endotracheal intubation and sternotomy? Eur J Anaesthesiol 1999; 16, 622627. The EEG waveforms is analysed according to three features: 1. Burst suppression ratio - A time-domain feature that quantifies the extent of electrical silence during deep anaesthesia. 2. Relative alpha/beta ratio. This is a frequency-domain feature and contributions from these frequency bands are seen during light sedation. 3. Bicoherence of the EEG - which describes the phase coupling relations between individual waves. In simple terms, a signal with strong phase relations and a high bicoherence value implies a common generator and may be associated with moderate sedation. A flat EEG generates a BIS index of zero.

129

CARDIORESPIRATORY PHYSIOLOGY TESTS Number: 18 A morbidly obese, non-smoking patient who is otherwise well is likely to have a significant reduction in: 1. Functional Residual Capacity (FRC). 2. Forced Expiratory Volume in 1 second (FEV1). 3. Expiratory Reserve Volume (ERV). 4. Diffusing Capacity for Carbon Monoxide (DLCO). A: 1,2,3 Correct B: 1,3 Correct C: 2,4 Correct D: 4 Correct E: All Correct ABCDE Correct Answer: A Biring et al have recently examined pulmonary physiological changes of morbid obesity and concluded that "Forced vital capacity, forced expiratory volume in 1 second, expiratory reserve volume, functional residual capacity, maximum voluntary ventilation, and forced expiratory flow during midexpiratory phase were all significantly reduced." However, no abnormality in DLCO was found. See:Biring MS, Lewis MI, Liu JT, Mohsenifar Z. Pulmonary physiologic changes of morbid obesity. Am J Med Sci. 1999 Nov;318(5):293-7. Number: 30 An acclimatised mountaineer, breathing air on the summit of Mount Everest (barometric pressure 253 Torr), will have an arterial PCO2 (PaCO2) of approximately: A. 24 mm Hg. B. 20 mm Hg. C. 16 mm Hg. D. 12 mm Hg. E. 8 mm Hg. Select the single best answer ABCDE

130

Correct Answer: E See the classic study by West. - "Pulmonary gas exchange was studied on members of the American Medical Research Expedition to Everest at altitudes of 8,050 m (barometric pressure 284 Torr), 8,400 m (267 Torr) and 8,848 m (summit of Mt. Everest, 253 Torr). Thirty-four valid alveolar gas samples were taken using a special automatic sampler including 4 samples on the summit. Venous blood was collected from two subjects at an altitude of 8,050 m on the morning after their successful summit climb. Alveolar CO2 partial pressure (PCO2) fell approximately linearly with decreasing barometric pressure to a value of 7.5 Torr on the summit. For a respiratory exchange ratio of 0.85, this gave an alveolar O2 partial pressure (PO2) of 35 Torr. In two subjects who reached the summit, the mean base excess at 8,050 m was -7.2 meq/l, and assuming the same value on the previous day, the arterial pH on the summit was over 7.7. Arterial PO2 was calculated from changes along the pulmonary capillary to be 28 Torr. In spite of the severe arterial hypoxemia, high pH, and extremely low PCO2, subjects on the summit were able to perform simple tasks. The results allow us to construct for the first time an integrated picture of human gas exchange at the highest point on earth." West JB, Hackett PH, Maret KH, Milledge JS, Peters RM Jr, Pizzo CJ, Winslow RM. Pulmonary gas exchange on the summit of Mount Everest. J Appl Physiol. 1983 Sep;55(3):678-87. Number: 31 A patient in an intensive care unit has the following haemodynamic measurements made: Mean Systemic Arterial Pressure (MAP) 80 mm Hg. Mean Central Venous Pressure (CVP) 10 mm Hg. Cardiac Output (CO) 5.0 l/min. Mean Pulmonary Arterial Pressure (MPAP) 35 mm Hg. Pulmonary Artery Occlusion Pressure (PAOP) 20 mm Hg. The pulmonary vascular resistance (PVR) is: A. 14 dynes.sec.cm-5. B. 120 dynes.sec.cm-5. C. 240 dynes.sec.cm-5. D. 400 dynes.sec.cm-5. E. 1120 dynes.sec.cm-5. Select the single best answer ABCDE Correct Answer: C PVR= 80 x (MPAP-PAOP)/CO.

131

This equation is the hydraulic equivalent of Ohm's Law ie: Flow = Driving pressure / Resistance. The constant '80' being a conversion factor to convert the pressures (in mm Hg) to SI units. (More accurately the value is 79.9). The normal range for PVR in an adult is 150 - 250 dynes.sec.cm-5. The MAP and CVP are not needed for calculation of the PVR but can be used in the calculation of systemic vascular resistance according to the equation: SVR= 80 x (MAP-CVP)/CO. Number: 94 In the left lateral position, blood flow to the non-dependant lung is: A. 25%. B. 35%. C. 45%. D. 55%. E. 65%. Select the single best answer ABCDE Correct Answer: C The distribution of perfusion in the lungs varies regionally and is under the influence of posture, mode of ventilation, and type of anaesthesia. In general, a hydrostatic head of pressure exists which decreases with vertical height above the heart. In the upright position, the absolute pressure in the pulmonary artery decreases by 1 cm water/cm vertical distance up the lung. Distribution of blood flow can be summarized as follows: Normally, in the upright position, the right lung receives 55% of blood flow and the left receives 45%. In the lateral decubitus position, when the right lung is up, it receives 45% and the dependant left lung receives 55%. When the left lung is up, it receives 35% and the right dependant lung receives 65%. These values are for an awake, spontaneously breathing patient. With respect to the left lateral position only, blood flow to the non-dependant lung is: (1) 41% in an awake, spontaneously breathing patient. (2) 43% in an anaesthetised, spontaneously breathing patient. (3) 44% in a paralyzed patient, ventilated with IPPV. (4) 70% in a paralyzed patient, ventilated with IPPV, with the chest wall open. MARTIN, J; Positioning the Patient for Anaesthesia and Surgery, 1978, p138. ROGERS, M.C ET AL (EDS); Principles and Practice of Anesthesiology, Mosby, 1993, pp 1748.

132

Number: 96 What is the rate of rise of PaCO2 during breath holding? A. 1 mmHg/min. B. 3 mmHg/min. C. 5 mmHg/min. D. 7 mmHg/min. E. 10 mmHg/min. Select the single best answer ABCDE Correct Answer: B The rise of PaCO2 is most rapid in the first minute of breath holding. This felt to be due to the additional effects of equilibration between the venous and arterial blood. Thereafter a steady rise of ~ 3mmHg/min occurs. NUNN, J; Applied respiratory Physiology,3rd Ed., Butterworths, 1987. Number: 97 Which of the following are true with respect to increasing oxygen reserves through preoxygenation with 100% oxygen: 1. It is well reflected by the arterial oxygenation saturation. 2. It requires longer when using a Bain circuit. 3. It is achieved equally as well with 4 vital capacity breaths or 3 minutes of tidal volume breathing. 4. Oxygen reserves are reduced in pregnancy. A: 1,2,3 Correct B: 1,3 Correct C: 2,4 Correct D: 4 Correct E: All Correct ABCDE Correct Answer: D Arterial oxygen saturation does not reflect the amount of oxygen contained within the functional residual capacity- the major reservoir utilized during preoxygenation. This is reflected by the end-tidal N2. Non-rebreathing circuits generally achieve denitrogenation rapidly. Vital capacity breathing reduces end-tidal N2O to 6% as compared with conventional tidal volume breathing in which it is reduced to 1%. In ASA I patients, this will still prevent desaturation for up to 6 minutes of apnoea, but may cause significant reductions in the tolerated apnoeic intervals in obstetric or elderly patients. Arterial oxygen saturation falls to a mean of 75% within 1 minute of suxamethonium when oxygen is not administered prior to induction of anaesthesia. Maximizing oxygen reserve has

133

been advocated as a means of delaying this event. The major oxygen stores of the body are shown after equilibration when breathing 21% and 100% oxygen: 21% functional residual capacity = 0.21 x 2.4 L ~450 mls chemical combination with haemoglobin ~850 mls dissolved in plasma ~50 mls chemical combination with myoglobin ~200 mls 100% ~3000 mls ~950 mls ~100 mls ~200 mls

Thus, increasing oxygen reserve by " preoxygenation " is achieved predominantly through denitrogenation of the FRC and to a lesser extent through increasing saturation of arterial blood. This will be manifested using pulse oximetry as an elevation of baseline SAO2 and a delayed fall in SAO2 during apnoea. The best method of preoxygenation has been studied. The options include: (1)Preoxygenation with tidal volume breaths for 2-10 minutes. (2) " 4 vital capacity breaths. (3)Postoxygenation by assisted ventilation after apnoea has occured. (4)Any combination of the above. (1)Recommendations vary between 2 and 10 minutes. Earlier studies showed that in normal subjects, breathing 100% achieved > 98% denitrogenation in 7 minutes. Reducing alveolar N2 to 4% was felt to be acceptable however, and allowed 5-6 minutes of apnoea without desaturation. The difference in oxygen stores at 0% and 4% N2 were negligible (2.53L v 2.61L). More recently, it has been shown that in ASA I patients, tidal breathing of 100% oxygen reduced alveolar N2 to 1% after 3 minutes, and to 6% after 2 minutes. The time required will however vary with different breathing circuits. Circuits using low fresh gas flows require longer to complete denitrogenation. Non-rebreathing systems generally achieve this rapidly. Three minutes has been recommended with the Magill (Mapleson A) system using 8 L/min flow rate. One minute using 10 L/min will allow 3 minutes of apnoea before desaturation > 6% SAO2 occurs. When a circle system is used with 5 L/min flow rate, an adequate level of denitrogenation is achieved within 5 minutes. Note that these times assume a tight seal with the face mask. (2)More recently, it has been shown that four voluntary, maximal breaths of 100% oxygen over 30 seconds produced a similar level of oxygenation as 3 minutes of tidal breathing. It would appear that this technique would require a non-rebreathing circuit with a large reservoir bag and a large fresh gas flow. Satisfactory results have been obtained however, with a circle system with 5 L/minute flows and a Magill circuit with 8 L/min flows despite this observation. Here, the patient was instructed to inhale slowly to prevent the reservoir bag from collapsing. A study published in 1989 has challenged the second method on the basis that most studies performed measured oxygen content of the blood rather than actual N2 content of the lungs. This was following an earlier study which showed the maximal breath technique to be associated with a significantly shorter time to desaturation than tidal breathing for 3 minutes.The implication was that whilst they both fully saturate haemoglobin, only the latter provides adequate denitrogenation. One of the reasons suggested was that the maximal breath

134

method was assumed to represent vital capacity breathing where in reality, many patients achieved inspiratory capacity or smaller volumes. In the study cited, 3 minutes of tidal breathing was compared with eight inspiratory capacity breaths (TLC with passive exhalation) and four vital capacity breaths (exhaling to residual volume then maximal breaths). The study showed that N2 washout to 1% was achieved with 3 minutes of tidal breathing and to 6% with 2 minutes of tidal breathing, 8 inspiratory breaths or 4 vital capacity breaths. It should be noted that whilst 6% residual N2 may be adequate in an ASA I patient, it may only allow a shorter apnoeic time before desaturation in a patient with a less favourable relationship between closing capacity and FRC as seen in pregnancy, obesity and age > 44. (3)Oxygenation after induction is practiced widely however is felt to be less effective than preoxygenation because the volume of the reservoir bag is limited and the FRC is often reduced. LATTO, I.P & ROSEN, M (EDS); Difficulties in Tracheal Intubation, Balliere Tindall, pp 2023. Number: 130 Which of the following typically result from the application of an aortic cross clamp? 1. Stroke volume decreases. 2. Systemic blood pressure increases. 3. Myocardial contractility decreases. 4. Venous return decreases. A: 1,2,3 Correct B: 1,3 Correct C: 2,4 Correct D: 4 Correct E: All Correct ABCDE Correct Answer: E The anticipated consequences of application of an aortic cross-clamp include an increased ventricular afterload, a decreased venous return, and a decreased velocity and shortening of myocardial muscle fibres. Clinical reports consistently report a 15-35% reduction in stroke volume and cardiac index, coupled with an increased arterial blood pressure and up to 40% increase in systemic vascular resistance. The effect of cross-clamping on venous return is a composite of complimentary and opposing factors- diminished venous return due to exclusion of blood flow to the pelvis and lower extremities; a possible redistribution of blood flow from the inferior vena cava to the superior vena cava; and an increase in left ventricular end systolic and end-diastolic volumes. CUNNINGHAM, A.J; " Anaesthesia for abdominal aortic surgery- a review (Part 1) ", Can J Anaesth, vol 36, no 4, 1989, pp 426-44.

135

Number: 183 Pulmonary Surfactant: 1. Is produced by type II alveolar cells. 2. Is turned over so rapidly that a reduction in pulmonary blood flow can cause a decrease in surfactant production. 3. Synthesis is stimulated by thyroxine and glucocorticoids. 4. Is partly recycled by endocytosis into the synthesising cell. A: 1,2,3 Correct B: 1,3 Correct C: 2,4 Correct D: 4 Correct E: All Correct ABCDE Correct Answer: E The role of surfactant in the aetiology of respiratory distress syndrome has been recently reviewed. ARDS includes a complex series of events leading to alveolar damage, high permeability pulmonary edema, and respiratory failure. The endogenous pulmonary surfactant system is crucial to maintaining normal lung function, and only recently has it been appreciated that alterations in the surfactant system significantly contributed to the pathophysiology of the lung injury of patients with ARDS. Through a combination of analyzing broncho-alveolar lavage samples from patients with ARDS and extensive animal studies, there have been significant insights into the variety of surfactant abnormalities that can occur in injured lungs. These include altered surfactant composition and pool sizes, abnormal surfactant metabolism, and inactivation of alveolar surfactant by serum proteins present within the airspace. Positive effects of exogenous surfactant administration on acute lung injury have been reported. There is now a prospective, randomized clinical trial evaluating the efficacy of aerosolized exogenous surfactant in patients with ARDS. This trial has demonstrated improvements in gas exchange and a trend toward decreased mortality in response to the surfactant. Despite these encouraging results, there are multiple factors requiring further investigation in the development of optimal surfactant treatment strategies for patients with ARDS. Such factors include the development of optimal surfactant delivery techniques, determining the ideal time for surfactant administration during the course of injury, and the development of optimal exogenous surfactant preparations that will be used to treat these patients. With further clinical trials and continued research efforts, exogenous surfactant administration should play a useful role in the future therapeutic approach to patients with ARDS. References: NUNN, J.F; Applied Respiratory Physiology, 3rd edition , Butterworths, 1987. LEWIS. J.F; Am Rev Respir Dis, vol 147, no 1, Jan. 1993, pp 218-33.

136

Number: 183 Pulmonary Surfactant: 1. Is produced by type II alveolar cells. 2. Is turned over so rapidly that a reduction in pulmonary blood flow can cause a decrease in surfactant production. 3. Synthesis is stimulated by thyroxine and glucocorticoids. 4. Is partly recycled by endocytosis into the synthesising cell. A: 1,2,3 Correct B: 1,3 Correct C: 2,4 Correct D: 4 Correct E: All Correct ABCDE Correct Answer: E The role of surfactant in the aetiology of respiratory distress syndrome has been recently reviewed. ARDS includes a complex series of events leading to alveolar damage, high permeability pulmonary edema, and respiratory failure. The endogenous pulmonary surfactant system is crucial to maintaining normal lung function, and only recently has it been appreciated that alterations in the surfactant system significantly contributed to the pathophysiology of the lung injury of patients with ARDS. Through a combination of analyzing broncho-alveolar lavage samples from patients with ARDS and extensive animal studies, there have been significant insights into the variety of surfactant abnormalities that can occur in injured lungs. These include altered surfactant composition and pool sizes, abnormal surfactant metabolism, and inactivation of alveolar surfactant by serum proteins present within the airspace. Positive effects of exogenous surfactant administration on acute lung injury have been reported. There is now a prospective, randomized clinical trial evaluating the efficacy of aerosolized exogenous surfactant in patients with ARDS. This trial has demonstrated improvements in gas exchange and a trend toward decreased mortality in response to the surfactant. Despite these encouraging results, there are multiple factors requiring further investigation in the development of optimal surfactant treatment strategies for patients with ARDS. Such factors include the development of optimal surfactant delivery techniques, determining the ideal time for surfactant administration during the course of injury, and the development of optimal exogenous surfactant preparations that will be used to treat these patients. With further clinical trials and continued research efforts, exogenous surfactant administration should play a useful role in the future therapeutic approach to patients with ARDS. References: NUNN, J.F; Applied Respiratory Physiology, 3rd edition , Butterworths, 1987. LEWIS. J.F; Am Rev Respir Dis, vol 147, no 1, Jan. 1993, pp 218-33.

137

Number: 284 Acidosis may result in: 1. Potassium retention 2. A rise in plasma chloride 3. A low pCO2 4. Tetany A: 1,2,3 Correct B: 1,3 Correct C: 2,4 Correct D: 4 Correct E: All Correct ABCDE Correct Answer: A Alkalosis enhances and acidosis depresses renal potassium secretion, probably by inducing corresponding changes in tubular cell potassium. Chloride is required for bicarbonate secretion in the collecting duct via a bicarbonate-chloride exchanger. In acute metabolic acidosis, hyperventilation is usual and may be intense (Kussmaul respiration). Alkalosis directly enhances neuromuscular excitability; this effect, rather than the modest decrease in ionized plasma calcium induced by alkalosis, is probably the major cause of tetany. Number: 324 Concerning the membrane potential of cardiac muscle: 1. Phase 2 is associated with efflux of calcium ions 2. Phase 3 is produced by an efflux of potassium ions 3. Slowing of phase 3 decreases the QT interval 4. Verapamil blocks slow calcium currents A: 1,2,3 Correct B: 1,3 Correct C: 2,4 Correct D: 4 Correct E: All Correct ABCDE

138

Correct Answer: C Phase 2 is the plateau phase of the action potential. The slow calcium channel is open during this time. During phase 3 membrane repolarisation occurs as a result of the continued flow of potassium out of the cell. Number: 325 The gas transfer (DLCO) depends on: 1. The volume of the pulmonary capillary bed. 2. Ventilation perfusion matching 3. Haemoglobin concentration 4. Residual volume A: 1,2,3 Correct B: 1,3 Correct C: 2,4 Correct D: 4 Correct E: All Correct ABCDE Correct Answer: A The ability of gas to diffuse across the alveolar-capillary membrane is ordinarily assessed by the diffusing capacity of the lung for carbon monoxide (DLCO). In this test, a small concentration of carbon monoxide (0.3%) is inhaled, usually in a single breath that is held for approximately 10 s. The carbon monoxide is diluted by the gas already present in the alveoli and is also taken up by haemoglobin as the erythrocytes course through the pulmonary capillary system. The concentration of carbon monoxide in exhaled gas is measured, and DLCO is calculated as the quantity of carbon monoxide absorbed per minute per mmHg pressure gradient from the alveoli to the pulmonary capillaries. The value obtained for DLCO depends on the alveolar-capillary surface area available for gas exchange and on the pulmonary capillary blood volume. In addition, the thickness of the alveolar-capillary membrane, the degree of ventilation-perfusion ratio mismatching, and the patient's haemoglobin level will affect the measurement. Because of this effect of haemoglobin levels on DLCO, the measured DLCO is frequently corrected to take the patient's haemoglobin level into account. The value for DLCO can then be compared with a predicted value, based either on age, height, and gender or on the alveolar volume (VA) at which the value was obtained. Alternatively, the DLCO can be divided by VA and the resulting value for DLCO/VA compared with a predicted value.

139

Number: 334 The features of chronic mountain sickness include: 1. A decreased ventilatory response to carbon dioxide. 2. Extreme polycythaemia. 3. A decreased ventilatory response to hypoxia. 4. Thromboembolism A: 1,2,3 Correct B: 1,3 Correct C: 2,4 Correct D: 4 Correct E: All Correct ABCDE Correct Answer: E This uncommon disorder affects long-time, high-altitude residents and is known as Monge's disease. It is characterised by fatigue, dyspnoea, aches and pains, excessive polycythaemia, and thromboembolism. Hypoventilation is a prominent feature of the disorder. The condition is well recognized in miners living above 4000 metres in the Andes. Number: 342 The cardiovascular response to cooling to 31 degrees centigrade in a healthy 20 year old is likely to include: 1. Bradycardia. 2. Prolongation of the PR interval. 3. Prolongation of the QT interval. 4. Ventricular fibrillation. A: 1,2,3 Correct B: 1,3 Correct C: 2,4 Correct D: 4 Correct E: All Correct ABCDE Correct Answer: A Heart rate slows as temperature decreases to less than 33 C. This slowing results from prolongation of systole, unlike physiologic bradycardia in which diastole is prolonged.

140

Conduction velocity decreases throughout the hypothermic heart, yielding prolonged PR and QT intervals as well as a widening of the QRS complex on the electrocardiogram. Both Jpoint elevations, termed "Osborn waves," and T-wave flattening or inversions occur. Atrial fibrillation is common. It results from atrial stretching because of increased central blood volume. Ventricular fibrillation develops between 26 and 30 C . Hypothermia-induced ventricular fibrillation is refractory to pharmacologic therapy. Number: 404 A fit twenty year old male has been thoroughly preoxygenated, rendered apnoeic with thiopentone and suxamethonium and now has oxygen being insufflated into the upper airway at 500 mls/min. Given this scenario, which of the following statements are true? 1. The PCO2 will rise more in the first minute of apnoea than in the second. 2. Oxygen will be drawn into the apnoeic lung at a rate equal to the difference between O2 utilisation and CO2 production. 3. The arterial PCO2 will exceed the venous PCO2 by about 2 mm Hg after three minutes of apnoea. 4. Apnoea can be safely sustained for no longer than ten minutes. A: 1,2,3 Correct B: 1,3 Correct C: 2,4 Correct D: 4 Correct E: All Correct ABCDE Correct Answer: B The PCO2 rises more in the first minute than subsequent minutes because there is a rapid equilibration of CO2 between the alveolar and mixed venous blood. Thus in the first minute, the rise in PCO2 is about 6 mm Hg, thereafter it rises at about 3 mm Hg per minute. Oxygen is drawn into the lung ('apnoeic oxygenation') at a rate equal to the difference between the uptake of oxygen (~250 mls/min) and the rate at which CO2 is being added to the FRC. - The catch is that this rate does NOT equal CO2 production. - Because the body has a large capacity for buffering CO2, PCO2 only climbs at about 3 mm Hg per minute during apnoea. Thus the partial pressure of CO2 in the Functional Residual Capacity climbs at the same rate - which corresponds to a volume change of about 10 mls. As a result, oxygen is drawn into the lung at a rate just a little less than that of oxygen utilisation. Oxygen is transferred into the blood in the absence of CO2 exchange. The Haldane effect now operates and raises the PCO2 of the blood (without changing carbon dioxide content).

141

Properly performed, the limiting factor in apnoeic oxygenation is the maximum acceptable PCO2. In an otherwise healthy 20 year old this is likely to be more than 100 mm Hg, which would correspond to an apnoeic period of about 20 minutes. Number: 485 With regard to the Inspiratory (FiO2) to End-tidal Oxygen (FeO2) concentration difference (Fi-eO2). A. FeO2 is a good measure of the completeness of pre-oxygenation. B. (FetO2) can exceed (FiO2) during the first few minutes of Nitrous Oxide:Oxygen anaesthesia C. Fi-eO2 increases during the first hour of Nitrous Oxide:Oxygen anaesthesia. D. Fi-eO2 correlates with the rate of uptake of oxygen in the steady state. E. All of the above. Select the single best answer ABCDE Correct Answer: E Br J Anaesth 1994 Jan;72(1):116-8 Berry CB, Myles PS Preoxygenation in healthy volunteers: a graph of oxygen "washin" using end-tidal oxygraphy. Anaesth Intensive Care 1993 Aug;21(4):409-13 Machlin HA, Myles PS, Berry CB, Butler PJ, Story DA, Heath BJ End-tidal oxygen measurement compared with patient factor assessment for determining preoxygenation time. Time to adequate preoxygenation was assessed in 200 elective surgical patients, using measurement of end-tidal oxygen concentration. A variety of patient factors were assessed as to their ability to predict the time required to preoxygenate a patient. Of the 200 patients, 23 (11.5%) were unable to be adequately preoxygenated; most of these cases were due to a poor mask fit. The average time for preoxygenation was 154 seconds (range 43-364 seconds). Of those patients who could be preoxygenated, 46 (23%) required more than three minutes. Although a regression equation could be constructed to calculate time required for preoxygenation, the wide standard errors of the coefficients preclude a clinically useful predictive equation. We thus found that we could not accurately predict time required for preoxygenation and that a routine three minutes preoxygenation may not be sufficient for many patients. However, the measurement of end-tidal oxygen concentration is a very useful method of determining the end-point for preoxygenation.

142

Number: 492 The component of the pulmonary artery wedged pressure (PAWP) trace which most accurately reflects the left ventricular end-diastolic pressure is: A. The peak of the 'V' wave. B. The trough of the 'X' descent. C. The peak of the 'A' wave. D. The trough of the 'Y' descent. E. The mean value of the PAWP. Select the single best answer ABCDE Correct Answer: C The peak of the 'A' wave corresponds to the maximal end-diastolic pressure of the ventricle. Number: 520 With regard to lung volumes and capacities: A. Vital capacity represents the total lung volume. B. Inspiratory capacity is the sum of the tidal volume and the inspiratory reserve volume. C. FEV1 is typically about 90% of the vital capacity. D. Residual volume can be measured by the spirometry. E. Vital capacity is the sum of the inspiratory and the expiratory reserve volumes. Select the single best answer ABCDE Correct Answer: B Total lung volume is the vital capacity plus the residual volume. FEV1 is approximately 75-85% of the vital capacity. The functional residual capacity and residual volume can only be measured by plethysmography or indicator-dilution (eg Helium). Vital capacity is the sum of the inspiratory reserve volume, expiratory reserve volume and tidal volume.

143

Number: 522 Functional residual capacity: 1. Can be measured by the Helium dilution technique. 2. Decreases with age. 3. Decreases following prolonged exposure to 100% oxygen. 4. Exceeds closing capacity in a fit 20 year old in the supine position. A: 1,2,3 Correct B: 1,3 Correct C: 2,4 Correct D: 4 Correct E: All Correct ABCDE Correct Answer: B FRC can be measured by plethysmography or indicator-dilution (eg Helium, Oxygen). FRC remains the same or increases slightly with increasing age. Removal of the 'Nitrogen Splint' is associated with absorption atelectasis and a reduction in FRC. (The 'Critical VQ Hypothesis'). Closing capacity usually exceeds FRC in the supine position after 40 years and in the erect position after 60 years. Number: 525 The partial pressure of oxygen in the 'ideal' alveolar gas during air breathing: 1. Varies with atmospheric pressure. 2. Varies with ambient humidity. 3. Will transiently rise during induction of anaesthesia with a mixture of 21% oxygen in nitrous oxide. 4. Is greater than in the mixed expired air. A: 1,2,3 Correct B: 1,3 Correct C: 2,4 Correct D: 4 Correct E: All Correct ABCDE Correct Answer: B

144

Whatever the ambient humidity, air drawn into the respiratory tract becomes fully saturated in the alveoli at 37 degrees C. The SVP of water at this temperature is 47 mm Hg which corresponds to a concentration of 44 mg/L or a fractional concentration of 6.2%. During induction of anaesthesia with 21% oxygen in nitrous oxide, the initial rate of uptake of nitrous oxide is very rapid in comparison to oxygen, so the alveolar oxygen tension (PAO2) transiently rises in proportion to the decrease in alveolar volume. (Severinghaus JW: The rate of uptake of nitrous oxide in man, J Clin Invest 33:1183, 1954.) Expired air is a mixture of alveolar and dead space gas. By definition, dead space gas has not participated in gas exchange and therefore has a higher percentage of oxygen than alveolar air. Number: 587 A volunteer sits with his left arm in a water bath maintained at 42 centigrade and his right arm in a bath at 4 centigrade. 1. The arterial PO2 is higher in his left arm than in his right. 2. The arterial PO2 is lower in his right arm than in his aortic root. 3. The arterial O2 content is the same in both arms. 4. The arterial PO2 will be the same in both arms if temperature corrections are not applied to the measurements. A: 1,2,3 Correct B: 1,3 Correct C: 2,4 Correct D: 4 Correct E: All Correct ABCDE Correct Answer: E There is effectively no change in arterial oxygen content between the aortic root and the right and left arms. However, the arterial PO2 (measured at the actual blood temperature) will vary according to the temperature-induced shift in the haemoglobin dissociation curve. Thus, if the blood is cooled, the curve is left-shifted, haemoglobin binds oxygen more avidly and the arterial PO2 falls. The converse applies in the limb which has been warmed. Number: 600 Which of the following methods can be used to measure functional residual capacity (FRC)? 1. Helium dilution. 2. Bohr's method. 3. Body plethysmograph. 4. Fowler's method.

145

A: 1,2,3 Correct B: 1,3 Correct C: 2,4 Correct D: 4 Correct E: All Correct ABCDE Correct Answer: B FRC can be measured by plethysmography or indicator-dilution (eg Helium, Nitrogen, or Oxygen). Fowler's method is used to measure the anatomical dead space, while Bohr's method is used to measure physiological dead space. Number: 617 Which of the following conditions is least likely to be associated with an increased end-tidal to arterial CO2 tension difference? A. Fat embolism. B. Chronic Obstructive Pulmonary Disease. C. Air embolism. D. Cardiogenic shock. E. Lobar pneumonia. Select the single best answer ABCDE Correct Answer: E Conditions which are characterised by an increase in alveolar (parallel) deadspace increase the end-tidal to arterial CO2 tension difference much more markedly than conditions which are characterised by increased intrapulmonary shunting (such as lobar pneumonia), Number: 619 After 5 minutes of air breathing in a hypobaric chamber at a chamber altitude of 10,000 feet, a fit 25 year old will demonstrate: A. A reduced PaCO2 with an increased ETCO2 : Arterial CO2 tension difference. B. A reduced PaCO2 with an unchanged ETCO2 : Arterial CO2 tension difference. C. An unchanged PaCO2 with an increased ETCO2 : Arterial CO2 tension difference. D. An unchanged PaCO2 with an unchanged ETCO2 : Arterial CO2 tension difference. E. No change in end-tidal or arterial CO2 tension. Select the single best answer

146

ABCDE Correct Answer: B During air-breathing at this altitude, hyperventilation (in response to hypoxia) occurs and hence PaCO2 is reduced. However, this is not accompanied by any increase in alveolar deadspace and accordingly there will be no change in the end-tidal to arterial tension difference. (In fact alveolar deadspace might actually decrease slightly which would tend to reduce the ETCO2 : Arterial CO2 tension difference.) At the low FiO2, ventilation:perfusion mismatch in any low V/Q units would be accentuated and venous admixture will tend to increase. Number: 668 The rate of transfer of oxygen from the alveolar gas to the alveolar capillary blood increases if there is an increase in: 1. Cardiac output. 2. Mixed venous PCO2. 3. Haemoglobin. 4. Mixed venous PO2. A: 1,2,3 Correct B: 1,3 Correct C: 2,4 Correct D: 4 Correct E: All Correct ABCDE Correct Answer: A Cardiac output. An increased cardiac output increases haemoglobin flow through the lungs, and, as a result produces an almost linear increase in oxygen transfer from alveolar gas to the blood. Mixed venous PCO2. When more CO2 is presented to the lungs the Haldane effect becomes more marked and more oxygen is transferred. Haemoglobin concentration. At normal levels of alveolar PO2 haemoglobin is about 98% saturated, and the absolute amount of oxygen is proportional to the rate of flow of haemoglobin through the lungs. Mixed venous PO2. The higher the venous O2 content, the less can be taken up in approaching saturation as the blood traverses the alveolar capillaries.

147

Number: 685 With regard to this flow:volume loop (blue), which of the following diagnoses is most likely? (For comparison, a normal loop is shown in black.) A. Acute asthma. B. Chronic Obstructive Pulmonary Disease. C. Intrathoracic tracheal obstruction. D. Laryngeal carcinoma. E. Pulmonary fibrosis. Select the single best answer

148

ABCDE Correct Answer: E Pulmonary Fibrosis. - This is a classic flow:volume loop for a patient with a restrictive lung defect. The five major types of abnormal flow:volume loops are: 1. Obstructive pattern the loop is left shifted (towards the TLC) due to hyperinflation and air trapping (increased RV). Decreased expiratory flows show up in the top half of the loop with a typical "scooped out" appearance. 2. Restrictive pattern a small volume loop that is shifted towards the right. Both volumes and, to a lesser extent, flow are decreased. 3. Fixed Upper Airway Obstruction - both inspiratory and expiratory flows are decreased. The FVC is nearly normal. 4. Variable Intrathoracic Obstruction Peak expiratory flow is markedly decreased in the presence of a nearly normal FVC. 5. Variable Extrathoracic Obstruction - Peak inspiratory flow is markedly decreased in the presence of a nearly normal FVC.

Number: 715 Acclimatisation to altitude commonly results in a: 1. Decreased red cell 2,3 diphosphoglycerate concentration. 2. Decreased affinity of haemoglobin for oxygen. 3. Decreased ventilatory response to CO2. 4. Decreased arterial PCO2. A: 1,2,3 Correct B: 1,3 Correct C: 2,4 Correct D: 4 Correct E: All Correct ABCDE Correct Answer: C

149

The erythrocyte content of 2,3-DPG is INCREASED by both anaemia and arterial hypoxaemia (such as occurs following ascent to altitude). The effect of 2,3-DPG is to decrease the affinity of haemoglobin for oxygen (by shifting the oxyhaemoglobin dissociation curve to the right), causing oxygen to be released to the tissues at a higher PaO2 than would otherwise be the case. In cases of Chronic Mountain Sickness (Monge's disease), the ventilatory response to CO2 is diminished, but this is unusual. Otherwise, the ventilatory response to CO2 is unchanged. The immediate response to altitude is an increase in alveolar ventilation, caused by hypoxic stimulation of the carotid body chemoreceptors. Arterial PCO2 is decreased as a result.

Number: 763 The Left Ventricular End-Diastolic Volume (LVEDV) of an adult at rest is approximately: A. 25 - 50 mls. B. 51 - 75 mls. C. 76 - 100 mls. D. 101 - 125 mls. E. 126 - 150 mls. Select the single best answer ABCDE Correct Answer: D Wahr et al estimated the mean LVEDV (in a group of 52 normal volunteers using echocardiography) to be 112 ml +/- 27 ml. Number: 764 The Left Ventricular End-Systolic Volume (LVESV) of an adult at rest is approximately: A. 0 - 10 mls. B. 11 - 25 mls. C. 26 - 50 mls. D. 51 - 75 mls. E. 76 - 100 mls. Select the single best answer ABCDE

150

Correct Answer: C Wahr et al estimated the mean LVESV (in a group of 52 normal volunteers using echocardiography) to be 35 +/- 16 ml. Number: 765 The effect of the loss of atrial systole on cardiac output is to reduce ouput by about: A. 0 - 5% B. 6 - 10% C. 11 - 20% D. 21 - 40% E. 41 - 60% Select the single best answer ABCDE Correct Answer: D Most of the work in this area is produced from research into the effects of DDD vs VVI cardiac pacing. Typically, there is a 30% reduction in cardiac output when such a switch is made. See, for example, Am J Cardiol 1988 Feb 1;61(4):323-9. Clinical and hemodynamic comparison of VVI versus DDD pacing in patients with DDD pacemakers. Rediker DE, Eagle KA, Homma S, Gillam LD, Harthorne JW Number: 768 Which of the following conditions would be associated with the greatest alveolar deadspace in an otherwise fit 20 year old? A. Major pulmonary embolism. B. Tension pneumothorax. C. Lobar collapse. D. Morbid obesity. E. Status Asthmaticus. Select the single best answer ABCDE Correct Answer: A Remember that 'Deadspace' respresents 'wasted' ventilation or 'ventilation without perfusion'. The initial 'lesion' in pulmonary embolism is the development of unperfused but still ventilated areas of lung. - Hence the large increase in alveolar deadspace.

151

A tension pneumothorax markedly increases right-to-left shunt together with a reduction in cardiac output. Thus hypoxaemia features strongly, but alveolar deadspace (and CO2 exchange) may not be greatly changed. Lobar collapse and morbid obesity are also associated with increases in right-to-left shunt but for different reasons. In status asthmaticus there are disturbances in the distribution of ventilation (in particular, the effect of anatomical deadspace may become relatively more significant if alveoalr hypoventilation occurs) but the most important effects are caused by the mal-distribution of perfusion acting in concert with alveolar hypoventilation. Number: 773 The (absolute) rate of uptake of oxygen into the blood from the alveolar gas will increase with an increase in: 1. Haemoglobin concentration. 2. Alveolar ventilation. 3. Cardiac output 4. 2,3 diphosphoglycerate. A: 1,2,3 Correct B: 1,3 Correct C: 2,4 Correct D: 4 Correct E: All Correct ABCDE Correct Answer: A 1. At normal levels of alveolar PO2 (about 100 mm Hg or 13.3 kPa) haemoglobin will become 96% saturated, and the absolute amount of oxygen taken up will therefore be proportional to the rate of flow of haemoglobin through the lungs. 2. An increase of alveolar ventilation will raise the PO2 of alveolar gas, and therefore increase the uptake of oxygen by blood. This will be slight if the haemoglobin is already nearly saturated, but there will always be an increase. 3. Like an increase in haemoglobin concentration, an increase in cardiac output increases haemoglobin flow through the lungs, which results in a nearly proportional increase in oxygen transfer from the alvolus to blood. 4. The effect of raising 2,3-DPG is to shift the oxyhaemoglobin dissociation curve to the right, which is advantageous for the transfer of O2 from blood to tissue, but not for uptake in the lungs.

152

Number: 781 With regard to the normal heart: 1. Atrial systole contributes to about 20 - 25% of ventricular filling. 2. Coronary blood flow comprises about 5% of the cardiac output at rest. 3. Blood flow in the left anterior descending coronary artery occurs primarily during diastole. 4. Left Ventricular dP/dT max is dependent on changes in preload. A: 1,2,3 Correct B: 1,3 Correct C: 2,4 Correct D: 4 Correct E: All Correct ABCDE Correct Answer: E Number: 782 The application of Positive End-Expiratory Pressure (PEEP) to a patient with normal lungs will tend to: 1. Increase right ventricular afterload. 2. Increase left ventricular afterload. 3. Increase alveolar deadspace. 4. Reduce the end-tidal to arterial carbon dioxide tension difference. A: 1,2,3 Correct B: 1,3 Correct C: 2,4 Correct D: 4 Correct E: All Correct ABCDE Correct Answer: B Under these circumstances, PEEP tends to increase right ventricular afterload and alveolar deadspace. As a result of the increase in deadspace, the end-tidal to arterial carbon dioxide tension difference also increases. PEEP increases in intrathoracic pressure, reduces left ventricular transmural pressure during systole, and, as a direct consequence, decreases left ventricular afterload.

153

See, for example: Circulation 1995 Mar 15;91(6):1725-31: Effect of continuous positive airway pressure on intrathoracic and left ventricular transmural pressures in patients with congestive heart failure. Naughton MT, Rahman MA, Hara K, Floras JS, Bradley TD Number: 811 Which of the following conditions would inevitably be associated with a lower-than-normal arterial PO2? A. Anaemia B. Carbon monoxide poisoning C. The presence of a high oxygen-affinity haemoglobin in the red cells. D. The presence of a low oxygen-affinity haemoglobin in the red cells. E. Lung disease with intra-pulmonary shunting. Select the single best answer ABCDE Correct Answer: E The presence of anaemia only affects only oxygen content, but not saturation or PO2. In contrast, the presence of CO or an abnormal oxygen-affinity haemoglobin affects oxygen saturation and content, but not PO2. The first demonstration of a high oxygen-affinity haemoglobin was by Charache in 1966, who was investigating a patient with erythrocytosis. The patients Hb level was 19.9 g/dl and an abnormal Hb band was observed on electrophoresis. The oxygen dissociation curve (ODC) of the patient's blood was found to be significantly shifted to the left. It was therefore suggested that the patient's erythrocytosis might be a secondary compensation to a primary defect in oxygen unloading. A family study revealed 15 other members with both erythrocytosis and the abnormal Hb. This Hb was then isolated and confirmed to have a marked increased oxygen affinity. A structural analysis revealed that it was an alpha-chain variant carrying the Arg->Lys substitution at position 92. This variant was named Hb Chesapeake. There are now over 200 high oxygen-affinity haemoglobins described. Number: 812 With regard to hyper- or hypo- baric conditions: 1. The 'cabin altitude' of commercial aircraft is maintained at sea-level pressure. 2. The PO2 of a scuba diver breathing compressed air at a depth of 66 feet is higher than at sea level. 3. At an altitude of 10,000 feet, the arterial PO2 of a climber remains normal due to hyperventilation. 4. The fractional inspired concentration of oxygen is the same at the summit of Mt. Everest as it is at sea level.

154

A: 1,2,3 Correct B: 1,3 Correct C: 2,4 Correct D: 4 Correct E: All Correct ABCDE Correct Answer: C The 'cabin altitude' of commercial aircraft is maintained at 7-8000 feet. This can be an important consideration when transporting patients with 'bends' or arterial hypoxaemia. Breathing compressed air at this depth will approximately treble the sea level arterial PO2. Number: 813 The pH of a blood sample with a bicarbonate of 24 mmol/L and a PaCO2 80 mm Hg is: A. 7.10 B. 7.30 C. 7.40 D. 7.50 E. 7.60 Select the single best answer ABCDE Correct Answer: A This is calculated from the Henderson-Hasselbalch equation. pH = pK + log(HCO3-/0.03*PCO2). Even without doing the calculation, it should be apparent that a very high PaCO2 and normal bicarbonate will lead to a very low pH. Number: 814 'Shallow water blackout' occurring during a breath-holding dive is principally due to: A. Hypercarbia. B. Hypocarbia. C. Hypoxia.

155

D. The 'diving reflex'. E. None of the above. Select the single best answer

ABCDE Correct Answer: C I think this is correct!! Certainly hypoxia is the main cause of the problem, although it is possible that the diving response contributes as well. Shallow-water blackout is a sudden unconsciousness from resulting from lack of oxygen during a breath-hold dive. It can occur at any depth, but the risk is greatest during ascent. At this time, decompression is lowering an already low arterial PO2. The risk of shallow-water blackout is further increased if the diver hyperventilates just prior to the dive. Refer to the web link for a good explanation of the subject. Number: 815 Given that the barometric pressure on the summit of Mount Everest has been measured at about 253 mm Hg, the arterial PO2 of an acclimatised mountaineer at the summit without supplemental oxygen will be about: A. 20 mm Hg. B. 25 mm Hg. C. 30 mm Hg. D. 40 mm Hg. E. 50 mm Hg. Select the single best answer ABCDE Correct Answer: C The alveolar air equation tells us that: Alveolar PO2 = PAO2 = .21(253-47) - 1.2 (40). An acclimatised mountaineer is able to hyperventilate to a PCO2 of about 7.5 mm Hg under these circumstances. If ones assumes an A-a difference of 5 mm Hg, he or she will achieve a PaO2 of about 30 mm Hg.

156

Number: 866 The best indicator of the adequacy of systemic oxygen delivery in a patient with haemorrhagic shock is: A. Oxygen flux. B. Arterio-venous oxygen content difference (Ca-vO2). C. Arterial partial pressure of oxygen (PaO2). D. True mixed venous partial pressure of oxygen (PvO2), E. True mixed venous oxygen content (CvO2). Select the single best answer ABCDE Correct Answer: D This is a difficult one! The main contenders for the correct answer being options 'B', 'D' and 'E'. Oxygen flux - is clearly incorrect as it simply gives you a measure of oxygen delivery and tells you nothing about oxygen utilisation. Similarly, PaO2 gives you even less information about oxygen delivery and again tells you nothing about oxygen utilisation. Ca-vO2 gives you an insight into oxygen utilisation, but because you don't know the partial pressure of oxygen in either the arterial or venous sample (or the haemoglobin), you don't know where they are are on Hb dissociation curve and therefore don't know if arterial (and therefore tissue) hypoxaemia is present. PvO2 in the steady state will always reflect 'average' tissue PO2. Thus, a high PvO2 will always reflect an O2 supply that is adequate for the current utilisation and a low PvO2 one that is inadequate. CvO2 (like Ca-vO2) gives you no estimate of blood or tissue PO2. See, also, the interesting paper by Siggaard-Andersen et al. Siggaard-Andersen O, Ulrich A, Gothgen IH. Classes of tissue hypoxia. Acta Anaesthesiol Scand Suppl. 1995;107:137-42.

157

Number: 906 An acclimatised mountaineer, breathing air on the summit of Mount Everest (barometric pressure 253 Torr), will have an arterial PO2 (PaO2) of approximately: A. 50 mm Hg. B. 45 mm Hg. C. 40 mm Hg. D. 35 mm Hg. E. 30 mm Hg. Select the single best answer ABCDE Correct Answer: E See the classic study by West. - "Pulmonary gas exchange was studied on members of the American Medical Research Expedition to Everest at altitudes of 8,050 m (barometric pressure 284 Torr), 8,400 m (267 Torr) and 8,848 m (summit of Mt. Everest, 253 Torr). Thirty-four valid alveolar gas samples were taken using a special automatic sampler including 4 samples on the summit. Venous blood was collected from two subjects at an altitude of 8,050 m on the morning after their successful summit climb. Alveolar CO2 partial pressure (PCO2) fell approximately linearly with decreasing barometric pressure to a value of 7.5 Torr on the summit. For a respiratory exchange ratio of 0.85, this gave an alveolar O2 partial pressure (PO2) of 35 Torr. In two subjects who reached the summit, the mean base excess at 8,050 m was -7.2 meq/l, and assuming the same value on the previous day, the arterial pH on the summit was over 7.7. Arterial PO2 was calculated from changes along the pulmonary capillary to be 28 Torr. In spite of the severe arterial hypoxemia, high pH, and extremely low PCO2, subjects on the summit were able to perform simple tasks. The results allow us to construct for the first time an integrated picture of human gas exchange at the highest point on earth." West JB, Hackett PH, Maret KH, Milledge JS, Peters RM Jr, Pizzo CJ, Winslow RM. Pulmonary gas exchange on the summit of Mount Everest. J Appl Physiol. 1983 Sep;55(3):678-87. Number: 917 An awake patient, with normal lung function, is breathing spontaneously in the lateral decubitus position with the right side dependent. The proportion of blood flowing to the right lung will be approximately: A. 25% of total pulmonary flow. B. 35% of total pulmonary flow. C. 45% of total pulmonary flow. D. 55% of total pulmonary flow. E. 65% of total pulmonary flow.

158

Select the single best answer

ABCDE Correct Answer: E According to Benumof: "Thus, when the right lung is nondependent, it should receive approximately 45 per cent of total blood flow, as opposed to the 55 per cent of the total blood flow that it received in the upright and supine positions. When the left lung is nondependent, it should receive approximately 35 per cent of total blood flow, as opposed to the 45 per cent of the total blood flow that it received in the upright and supine positions." Anesthesia for Thoracic Surgery.2nd ed.Jonathan L. Benumof. ISBN: 0721644678. Publisher: WB Saunders. See also: Wulff KE, Aulin I. The regional lung function in the lateral decubitus position during anesthesia and operation. Acta Anaesthesiol Scand. 1972;16(4):195-205. Rehder K. Postural changes in respiratory function. Acta Anaesthesiol Scand Suppl. 1998;113:13-6. Number: 925 A patient in an intensive care unit has the following haemodynamic measurements made: Mean Systemic Arterial Pressure (MAP) 80 mm Hg. Mean Central Venous Pressure (CVP) 10 mm Hg. Cardiac Output (CO) 5.0 l/min. Mean Pulmonary Arterial Pressure (MPAP) 35 mm Hg. Pulmonary Artery Occlusion Pressure (PAOP) 20 mm Hg. The systemic vascular resistance (SVR) is: A. 14 dynes.sec.cm-5. B. 300 dynes.sec.cm-5. C. 350 dynes.sec.cm-5. D. 960 dynes.sec.cm-5. E. 1120 dynes.sec.cm-5. Select the single best answer ABCDE Correct Answer: E SVR= 80 x (MAP-CVP)/CO.

159

This equation is the hydraulic equivalent of Ohm's Law ie: Flow = Driving pressure / Resistance. The constant '80' being a conversion factor to convert the pressures (in mm Hg) to SI units. (More accurately the value is 79.9). The normal range for SVR in an adult is 900-1500 dynes.sec.cm-5. The MPAP and PAOP are not needed for calculation of the SVR but can be used in the calculation of pulmonary vascular resistance according to the equation: PVR= 80 x (MPAP-PAOP)/CO.

CLINICAL INVESTIGATIONS TESTS Number: 207 The Pressure:volume loop below is of: A. Acute Aortic Incompetence. B. Aortic Stenosis. C. Mitral Incompetence. D. Chronic Aortic Incompetence. E. Ventricular Septal Defect. (A normal loop - shaded - is shown for comparison) Select the single best answer

ABCDE Correct Answer: D The loop is of Chronic Aortic Incompetence. - Note the large increase in End-Diastolic volume which is associated with little increase in End-Diastolic pressure. 160

Number: 239 The following blood gas was taken 10 minutes into routine cardio-pulmonary bypass in an adult: PaO2:35 mmHg; PaCO2:34 mm Hg; pH:7.28; BXS -6.3; For the blood gas analysis shown above, the most likely diagnosis is: A. Oxygenator failure. B. Gas Blender Failure. C. Malignant Hyperpyrexia. D. Disconnection of the gas line from oxygenator. E. None of the above. Select the single best answer ABCDE Correct Answer: B There is an isolated failure of oxygen transfer. This is most probably due to the delivery of fresh gas with an inappropriately low FiO2 - ie blender failure. If the gas line has become disconnected from the oxygenator, or the oxygenator has failed, there will be a failure of both oxygen and carbon dioxide transfer. Malignant hyperpyrexia on bypass is extremely uncommon. It can cause arterial hypoxaemia, but this would inevitably be associated with extreme hypercarbia, metabolic acidosis and hyperkalaemia. Number: 559 The blue (expiratory) flow pattern is superimposed on a black flow:volume loop for a normal patient of the same height, age and weight. Both expire from vital capacity to residual volume. On the basis of this test, which statements are true of the blue patient? 1. He has a greater residual volume. 2. He has an abnormal FEV (1 sec) : VC Ratio. 3. His vital capacity is less than the black patient. 4. He will have a reversible component to his mid-expiratory flow impairment. A: 1,2,3 Correct B: 1,3 Correct C: 2,4 Correct D: 4 Correct E: All Correct

161

ABCDE Correct Answer: B 2. Although it is highly likely that this is true, a measure of time on the X axis is necessary to confirm this. 4. The reversibility (or otherwise) of expiratory flow impairment can only be measured by testing pre- and post-exposure to a bronchodilator. The five major types of abnormal flow:volume loops are: 1. Obstructive pattern the loop is left shifted (towards the TLC) due to hyperinflation and air trapping (increased RV). Decreased expiratory flows show up in the top half of the loop with a typical "scooped out" appearance. 2. Restrictive pattern a small volume loop that is shifted towards the right. Both volumes and, to a lesser extent, flow are decreased. 3. Fixed Upper Airway Obstruction - both inspiratory and expiratory flows are decreased. The FVC is nearly normal. 4. Variable Intrathoracic Obstruction Peak expiratory flow is markedly decreased in the presence of a nearly normal FVC. 5. Variable Extrathoracic Obstruction - Peak inspiratory flow is markedly decreased in the presence of a nearly normal FVC.

162

Number: 565 Which of the following diagnoses is most likely for a patient with this flow:volume loop? A. Acute bronchospasm. B. Chronic Obstructive Pulmonary Disease. C. Intrathoracic tracheal obstruction. D. Extrathoracic tracheal obstruction. E. None of the above. Select the single best answer

ABCDE Correct Answer: C Intrathoracic tracheal obstruction. The five major types of abnormal flow:volume loops are: 1. Obstructive pattern the loop is left shifted (towards the TLC) due to hyperinflation and air trapping (increased RV). Decreased expiratory flows show up in the top half of the loop with a typical "scooped out" appearance. 2. Restrictive pattern a small volume loop that is shifted towards the right. Both volumes and, to a lesser extent, flow are decreased. 3. Fixed Upper Airway Obstruction - both inspiratory and expiratory flows are decreased. The FVC is nearly normal.

163

4. Variable Intrathoracic Obstruction Peak expiratory flow is markedly decreased in the presence of a nearly normal FVC. 5. Variable Extrathoracic Obstruction - Peak inspiratory flow is markedly decreased in the presence of a nearly normal FVC. Number: 643 A junior colleague calls you into his operating theatre because he is a little bit worried about his patient. The patient is a 20 year old with Down's syndrome who is undergoing a dental clearance under relaxant general anaesthesia. The patient has been intubated with a size 8 cuffed endotracheal tube (ETT) and is being ventilated with isoflurane in 100% oxygen. 30 minutes into the procedure, the patient is noted to be a little bit dusky. A blood gas analysis (ABG) has been performed, and is reported as: PaO2: 54 mm Hg; PaCO2: 30 mm Hg; pH: 7.48 Given the scenario above, what is the most likely cause for desaturation? A. Oesophageal intubation. B. Intubation of the right main bronchus. C. Endocardial cushion defect. D. Absorption atelectasis. E. Inhalation. Select the single best answer ABCDE Correct Answer: B Common things occur commonly! The ABG is unlikely to be due to oesophageal intubation in this scenarion because the PaCO2 is low. Nevertheless, as part of your management, you must visualise the ETT and confirm that it is passing through the cords. An endocardial cushion defect is a possibility - The incidence of endocardial cushion defects in patients with Down's syndrome is about 40% and it is certainly possible that a right-to-left shunt is causing (or contributing) to hypoxia in this patient.. Absorption atelectasis cannot produce this degree of right to left shunting unless other pathology is present.

164

Number: 702 These two blood gases were taken at a temperature of 26 degrees centigrade, 15 minutes apart, during cardiopulmonary bypass for aortic valve replacement. Both are reported at the analyser temperature of 37 dgrees centigrade. Note that the first gas was taken using a much HIGHER fresh gas flow (sweep gas) rate than the second. Blood Gas 1: Fresh Gas Flow: 3.6 L/min; Pump Flow: 3.5 L/min Hb: 77 G/L PaO2: 670 mm Hg. PaCO2: 46 mm Hg. pH: 7.32. Blood Gas 2: Fresh Gas Flow: 2.7 L/min; Pump Flow: 3.5 L/min Hb: 76 G/L PaO2: 640 mm Hg. PaCO2: 33 mm Hg. pH: 7.44. The most likely explanation of the change between blood gas 1 and blood gas 2 is: A. That CO2 flooding of the open chest was stopped. B. That a muscle relaxant was administered. C. That malignant hyperpyrexia was treated. D. That a large dose of thiopentone was administered. E. That the blood gas analyser was recalibrated. Select the single best answer ABCDE Correct Answer: A CO2 flooding of the open chest was stopped. The first blood gas is quite typical for a patient in whom CO2 flooding (for the purpose of minimising the risks of air embolism) is in use. In such cases, much higher than normal Fresh Gas Flows may be required in order to achieve normocarbia. CO2 is more dense than air and therefore accumulates in the open chest wound. As such, it tends to get preferentially sucked into the cardiotomy reservoir (where again it accumulates), either through the hand-held sucker or through a vent line. Once in the cardiotomy reservoir it raises the PCO2 of the incoming venous blood significantly - the reservoir being a surprisingly good gas-exchanging device. See, for example: Nadolny EM, Svensson LG. Carbon dioxide field flooding techniques for open heart surgery. Perfusion 2000; 15: 151-153. The effects of muscle relaxants and profound anaesthesia on metabolic rate at this temperature are minimal.

165

Malignant hyperpyrexia on bypass is virtually undescribed during cardiopulmonary bypass. The pH change is consistent with the change in PCO2 which means that it is unlikely that the gas analyser was recalibrated. Number: 767 A fit young man suffers a large, left-sided, spontaneous pneumothorax with no clinical evidence of tension. If an arterial blood gas (ABG) analysis were performed on the patient while breathing room air, which of the following results would be most likely? A. PaO2 70; PaCO2 35. B. PaO2 60; PaCO2 40. C. PaO2 50; PaCO2 40. D. PaO2 40; PaCO2 30. E. PaO2 50; PaCO2 30. Select the single best answer ABCDE Correct Answer: E Option 'E' corresponds to about a 40% right-to-left shunt - which is the sort of value you might expect in this scenario. The degree of hypoxia is sufficient to provoke a ventilatory response - hence option 'C' cannot be correct. Option 'D' corresponds to a 70% right-to-left shunt which is unlikely. Download the calculator (which is included in the pulmonary artery catheter simulator) if you want to examine the effect of right-to-left shunting on arterial oxygenation. Number: 791 A previously fit 50 year old lady undergoes an uneventful laparoscopic cholecystectomy. On the first post-operative day, the intern takes a blood specimen for 'routine' biochemistry. The patient is well. The result is reported as: Sodium 110 mmol/L (Normal Range 135 -145 mmol/L) Potassium 2.6 mmol/L (Normal Range 3.5 - 5.0 mmol/L) Chloride 79 mmol/L (Normal Range 98 - 106 mmol/L) Bicarbonate 20 mmol/L (Normal Range 22 - 32 mmol/L) Osmolality 291 mosm/L (Normal Range 270 - 290 mmol/L) Glucose 60 mmol/L (Normal Range 3.5 - 7.0 mmol/L) Urea 0.04 mmol/L (Normal Range 3.0 - 8.0 mmol/L)

166

The most likely diagnosis is: A. Syndrome of Inappropriate Anti-Diuretic Hormone secretion (SIADH). B. Hypothyroidism. C. Over-administration of 5% dextrose. D. Sampling from the 'Drip' arm. E. Hypoaldosteronism. Select the single best answer ABCDE Correct Answer: D There are three important causes of hyponatraemia resulting from hormonal abnormalities SIADH, adrenal insufficiency and hypothyroidism. All of such cases of hyponatraemia are associated with a low plasma osmolality. Adrenal insufficiency and hypothyroidism may present with hyponatraemia but should not be confused with SIADH. Although decreased mineralocorticoids may contribute to the hyponatraemia of adrenal insufficiency, it is the cortisol deficiency that leads to hypersecretion of ADH both indirectly (secondary to volume depletion) and directly (cosecreted with corticotropin-releasing factor). The mechanisms by which hypothyroidism leads to hyponatraemia include decreased cardiac output and GFR and increased ADH secretion in response to haemodynamic stimuli. Given the scenario, it is very unlikey that hormonal excess or deficiency is the cause of hyponatraemia in this case. Inappropropiate prescription of free water (in the form of 5% dextrose) is a relatively common peri-operative complication, but, again given the scenario seems unlikely. This is the typical result of sampling from the 'Drip' arm in a patient receiving 5% dextrose. Note that the blood is still iso-osmolar - in contrast to most of the 'real' causes of extreme hyponatraemia. Number: 793 An 85 year old lady is brought to the emergency room having been found unconscious at home. No other history is available. A biochemical screen performed at that time shows: Sodium 144 mmol/L (Normal Range 135 -145 mmol/L) Potassium 5.0 mmol/L (Normal Range 3.5 - 5.0 mmol/L) Chloride 99 mmol/L (Normal Range 98 - 106 mmol/L) Bicarbonate 17 mmol/L (Normal Range 22 - 32 mmol/L) Osmolality 391 mosm/L (Normal Range 270 - 290 mmol/L) Glucose 65 mmol/L (Normal Range 3.5 - 7.0 mmol/L) Urea 16 mmol/L (Normal Range 3.0 - 8.0 mmol/L) Creatinine 0.2 mmol/L (Normal Range 0.05 - 0.12 mmol/L) Urinary ketones absent.

167

The most likely diagnosis is: A. Neurogenic Diabetes Insipidus. B. Primary Hypodipsia. C. Hyperosmolar, nonketotic diabetic coma. D. Simple Dehydration. E. Diabetic Ketoacidosis. Select the single best answer ABCDE Correct Answer: C This is the classic picture of hyperosmolar, nonketotic diabetic coma. Hyperosmolar, nonketotic diabetic coma is usually a complication of NIDDM. It is a syndrome of profound dehydration resulting from a sustained hyperglycaemic diuresis under circumstances in which the patient is unable to drink enough water to keep up with urinary fluid losses. Commonly, an elderly diabetic patient, often living alone or in a nursing home, develops a stroke or infection that worsens hyperglycaemia and prevents adequate water intake. The full-blown syndrome probably does not occur until volume depletion is severe enough to decrease urine output. Number: 803 The following blood gas was taken 60 minutes into routine cardio-pulmonary bypass in an adult. Sodium Nitroprusside (SNP), administered at a rate of 2 mcg/kg/min has been running for 15 minutes to assist in re-warming. PaO2: 375 mm Hg; PaCO2: 40 mm Hg; pH: 7.08; BXS: -16.3 Blood Sugar: 27.2 mmol/L; Given the scenario above, the most likely diagnosis is: A. Malignant hyperpyrexia. B. Diabetic ketoacidosis. C. Lactic acidosis. D. Acute renal failure. E. Cyanide toxicity. Select the single best answer ABCDE Correct Answer: B Diabetic Ketoacidosis. The diagnosis should be confirmed by looking for ketonuria.

168

During high infusion rates of nitroprusside, cyanide toxicity may occur. This appears to be a particular problem when doses greater than 10 mcg/kg/min (0.6 mg/kg/hour) are used. Cyanide ion combines with cytochrome C, an enzyme required for aerobic metabolism. The impairment of aerobic metabolism results in a shift to anaerobic metabolism which is manifested as metabolic acidosis, with elevated plasma lactate concentrations. Number: 804 A 65 year old woman presents to the emergency department complaining of generalised weakness. On examination she has suffered obvious weight loss (her skin creases are quite prominent) and has a blood pressure of 90/60. She has no peripheral oedema or other evidence of malnutrition. A biochemical screen performed at that time shows: Sodium 120 mmol/L (Normal Range 135 -145 mmol/L) Potassium 6.2 mmol/L (Normal Range 3.5 - 5.0 mmol/L) Chloride 87 mmol/L (Normal Range 98 - 106 mmol/L) Bicarbonate 17 mmol/L (Normal Range 22 - 32 mmol/L) Osmolality 273 mosm/L (Normal Range 270 - 290 mmol/L) The most likely diagnosis for her hyponatraemia is: A. 'Factitious' hyponatraemia. B. Addison's disease. C. Syndrome of Inappropriate Anti-diuretic Hormone Secretion (SIADH). D. Cushing's syndrome. E. Nephrotic syndrome. Select the single best answer ABCDE Correct Answer: B A. 'Factitious' hyponatraemia (such as occurs with hyperlipidaemia) does not explain the serum potassium. B. Addison's original description was of "general languor and debility, feebleness of the heart's action, irritability of the stomach and a peculiar change of the colour of the skin". The condition results from progressive destruction of the adrenals - which must involve more than 90 percent of the glands before adrenal insufficiency appears. Historically the most important cause was chronic granulomatous disease - in particular tuberculosis (but also histoplasmosis, coccidioidomycosis, and cryptococcosis). Nowadays the most frequent cause in developed countries is 'idiopathic atrophy' with necrotising adrenalitis (due to cytomegalovirus) in AIDS patients running a close second. Hyperpigmentation in Addison's disease may be striking or absent. It commonly appears as a diffuse brown, tan, or bronze darkening of parts such as the elbows or creases of the hand and

169

of areas that normally are pigmented such as the areolae about the nipples. Bluish-black patches may appear on the mucous membranes. C. The diagnosis SIADH should be suspected in patients who have hyponatraemia and a concentrated urine (osmolality >300 mmol/kg) associated with lethargy - in the absence of oedema, orthostatic hypotension and features of dehydration. It may be reasonable to suspect SIADH (secondary to a malignancy) in this woman, but the diagnosis is inconsistent with the hyperkalaemia. D. The clinical picture is not one of Cushing's syndrome. E. The absence of peripheral oedema excludes the possibility of nephrotic syndrome. Number: 810 The following blood gas was obtained 15 minutes after initiation of cardio-pulmonary bypass in an otherwise fit 60 year old man undergoing elective aortic valve replacement. At the time the measurement was made, carbon dioxide was being continually insufflated into the chest wound in order to reduce the likelihood of air embolism. pH: 7.41. pCO2: 72.5 mm Hg. pO2: 504 mm Hg. HCO3-: 41.7 mmol/L K+: 5.2 mmol/L Given this scenario, the most likely cause of the hypercarbia is: A. Oxygenator failure. B. Partial disconnection of the fresh gas supply from the oxygenator. C. Malignant hyperpyrexia (MH). D. The carbon dioxide insufflation. E. Malfunction of the blood-gas electrode. Select the single best answer ABCDE Correct Answer: E Malfunction of the blood-gas electrode. - the combination of a normal pH and large base excess is incompatible with any pathological condition. If a partial disconnection of the fresh gas supply from the oxygenator had occurred, the pH would be much lower. Similarly, MH would be associated with a base deficit and hyperkalaemia. Carbon dioxide insufflation can result in hypercarbia, but usually not of this magnitude. If the insufflated CO2 were the cause, one would expect the pH to be much lower and the base excess to be in the normal range.

170

Number: 920 A 40 year old woman with a DVT has the following coagulation profile: INR 0.9 (Normal range 0.8-1.2) APPT 90 sec (Normal range < 35 sec) Platelet count 250 x 10^6 (Normal range 150-450.) Which of the following could explain these findings? 1. Disseminated intravascular coagulation. 2. Presence of lupus anticoagulant. 3. Antithrombin III deficiency. 4. Heparin therapy. A: 1,2,3 Correct B: 1,3 Correct C: 2,4 Correct D: 4 Correct E: All Correct ABCDE Correct Answer: C Abnormal coagulation studies may be the result of anticoagulation therapy or it may be a disorder of coagulation. The latter may be congenital or acquired. In this scenario, there is an isolated rise in the APPT. This may occur in the presence of the lupus anticoagulant, however, paradoxically, thrombosis is more common than coagulopathy. The cornerstone of monitoring heparin therapy is serial measures of APPT. The standard coagulation screen includes INR, APPT, TT, and FBC to assess platelet count. In addition a skin bleeding time and ACT may be indicated in some situations. INR (correlates with Prothrombin time (PR)): involves adding the source of thromboplastin (animal brain usually) and calcium to plasma thus activating Factor VII which activates Factor X. Sequentially, Xa and Va convert prothrombin to thrombin, fibrinogen to fibrin. Thus , the INR bypasses the intrinsic pathway and is abnormal with deficencies of Factors VII, X, V, prothrombin, fibrinogen or if a coagulation inhibitor is present, for example, warfarin. APPT (Activated Partial Thromboplastin Time): contact Factors XII and XI are activated by kaolin, and phospholipid accelerates the reactions involving VIII and V. Thus Factor X is activated leading to the formation of thrombin and fibrin. The only factor not involved in this pathway is VII. This test gives prolonged times with deficencies of XII, XI, IX, X, V, prothrombin & fibrinogen or by inhibitors such as Heparin. TT (Thrombin time):involves adding a source of thrombin to plasma thus bypassing intrinsic, extrinsic and common pathways to the level of conversion of fibrinogen to fibrin. It is

171

prolonged with deficiency of fibrinogen or by inhibitors of the conversion of fibrinogen to fibrin, for example: Heparin, FDPs DISSEMINATED INTRAVASCULAR COAGULATION (DIC): in DIC a triggering event occurs which disturbs the normal balance between intravascular coagulation and fibrinolysis and leads to activation of the coagulation system with widespread deposition of fibrin and platelets and secondary activation of the fibrinolytic system. Laboratory diagnosis involves: HB - low - bleeding/hemolysis PLATELET COUNT - low - consumption/ may be normal in chronic. INR - high - markedly prolonged in acute/ may be normal in chronic. APPT - high - markedly prolonged in acute/ may be normal in chronic. TT - high - may be normal depending on the level of fibrinogen. FIBRINOGEN - low - may be normal/increased. FDPs/D-DIMER - high - reflecting fibrinolysis Some disorders associated with DIC include:infection, obstetric complications, neoplasia, shock, hepatic disease, intravascular hemolysis, vasculitis, snake venom, burns, extracorporeal circulation, metabolic diseases eg: severe diabetes, hyperlipoproteinaemia. LUPUS ANTICOAGULANT: These are inhibitors which are directed against phospholipids and inhibit the interaction between the complex of Xa, V, phospholipid and calcium, and prothrombin. They may be found in SLE, other autoimmune disorders and some haematological malignancies, but often no underlying disease state can be found. In screening the most common finding is prolonged APPT which is not corrected by the addition of plasma. Despite this, bleeding is a rare event in the absence of other haemostatic defects for example, thrombocytopenia or hypoprothrombinaemia. The most common clinical manifestations are thrombotic events and recurrent abortions. ANTITHROMBIN III: This is one of the endothelial cell factors and has an anticoagulant role. The mechanism and control of its release are poorly understood. It is the main physiological inhibitor of thrombin and Factor Xa. It reacts irreversibly with thrombin to form a complex in which both components are inactivated. This reaction is greatly enhanced in the presence of heparin, and antithrombin III is an essential cofactor in its anticoagulant effect. As a serine protease inhibitor, it is also capable of inhibiting the activity of XIIa, XIa,and IXa. As a serine protease inhibitor, it is also capable of inhibiting the activity of XIIa, XIa,and IXa. The importance of antithrombin III in the normal inhibition of clotting activation is emphasised by the high incidence of venous thrombosis found in patients with congenital antithrombin III deficiency. A patient with a DVT may have antithrombin III deficiency but heparin therapy will be limited in effectiveness as reflected by an APPT which does not rise with therapy. HEPARIN: There are several ways in which heparin can influence the coagulation system. Predominantly,it potentiates the inhibition of Factors XIIa, XIa, IXa, Xa and thrombin through its interaction with Antithrombin III. As a result the APPT and TT will be prolonged as isolated findings unless it is complicated by bleeding or HITS. Low molecular weight heparin is produced by depolymerization of the parent compound. It has an equipotent effect on thrombogenesis but a reduced effect on platelets. It is a potent inhibitor of Factor X but has a weak effect on thrombin. Subsequently, the APPT cannot be used to measure activity; instead the more complex anti Factor Xa assay is used.

172

References International Anesthesiology Clinics, vol 23, no 2, Coagulation Disorders and the Hemoglobinopathies, 1985, pp 8-9, 34 NUNN, UTTING & BROWN; Anaesthesia, Ch. 79; Acquired Haemostatic Failure. Number: 1007 The bleeding time is increased in: 1. Haemophilia. 2. Idiopathic Thrombocytopenic Purpura (ITP). 3. Coumarin overdose. 4. Vitamin C deficiency. A: 1,2,3 Correct B: 1,3 Correct C: 2,4 Correct D: 4 Correct E: All Correct ABCDE Correct Answer: C The bleeding time (BT) is a very sensitive measure of platelet function and is thus normal in both haemophilia and coumarin overdosage. In both ITP and scurvy the bleeding time is prolonged. In ITP it is prolonged because of the reduction in platelet numbers (There is effectively a linear relationship between BT and platelet count once the count is less than ~ 80,000 platelets / microlitre of blood.), while in scurvy, there is a functional platelet defect associated with impairment of prostaglandin E1 synthesis. See: Johnson GJ, Holloway DE, Hutton SW, Duane WC. Platelet function in scurvy and experimental human vitamin C deficiency. Thromb Res. 1981 Oct 1-15;24(1-2):85-93. Horrobin DF, Oka M, Manku MS. The regulation of prostaglandin E1 formation: a candidate for one of the fundamental mechanisms involved in the actions of vitamin C. Med Hypotheses. 1979 Aug;5(8):849-58. LA Harker, Hemostasis Manual, 2d ed. Philadelphia, FA Davis Company, 1974

173

GENERAL ANAESTHESIA TESTS Number: 1 With regard to explicit awareness during general anaesthesia: 1. It occurs in about 0.2% of a heterogenous population of cases. 2. It is about twice as common in those given muscle relaxants compared to those who are not. 3. About 20% of those who suffer explicit awareness feel pain. 4. About 50% of those who suffer explicit awareness manifest cardiovascular changes.

A: 1,2,3 Correct B: 1,3 Correct C: 2,4 Correct D: 4 Correct E: All Correct Correct Answer: A Awareness during anaesthesia can be classified as either 'explicit' or 'implicit'. Explicit awareness refers to the specific recall of intra-operative events, whereas implicit memories can only be recalled when special techniques (such as hypnosis) are used to demonstrate 'awareness'. The incidence of explicit awareness in a heterogenous population of cases in the United Kingdom has been reported at about 0.2% and in Sweden at 0.18% (Liu D, Thorp S, Aitkenhead AR. Incidence of awareness with recall during general anaesthesia. Anaesthesia 1991; 46:435-437. Sandin RH, Enlund G, Samuelsson P, Lennmarken C: Awareness during anaesthesia: a prospective case study. Lancet 2000 Feb 26;355(9205):707-11 ). In particular risk groups, the incidence of explicit awareness may be considerably higher - for example in cardiac surgical cases it has been reported as about 1%. Awareness is usually thought to be extremely uncommon in those who have not been paralysed, but a recent study suggests that this might not be so. (Sandin RH, Enlund G, Samuelsson P, Lennmarken C: Awareness during anaesthesia: a prospective case study. Lancet 2000 Feb 26;355(9205):707-11 ) Schwender et al recently described 45 patients who had reported explicit awareness. 8 of these patients complained of painful awareness. (Schwender D, Kunze-Kronawitter H, Dietrich P, et al; Conscious awareness during general anaesthesia: patients' perceptions, emotions, cognition and reactions. Br J Anaesth 1998; 80:133 - 139.) The traditional cardiovascular signs of awareness only occur in about 25% of cases. Auditory awareness is almost always present in those who have been aware. 174

Number: 7 In patients undergoing laparoscopy, there is a association between the incidence of POSTOPERATIVE nausea and vomiting (PONV) and a high INTRA-OPERATIVE arterial partial pressure of which of the following gases: 1. Oxygen. 2. Carbon dioxide. 3. Nitrogen. 4. Nitrous oxide A: 1,2,3 Correct B: 1,3 Correct C: 2,4 Correct D: 4 Correct E: All Correct Correct Answer: E This is a bit of a trick question as all are probably correct! 1. Oxygen. In a recent study by Goll et al, the surprising efficacy of intra-operative hyperoxia in combatting PONV was demonstrated.The authors speculated that "The efficacy of oxygen may be related to ameliorating subtle intestinal ischemia with its consequent release of emetogenic substances, including serotonin. Intestinal ischemia seems more likely during surgery than postoperatively because that is when the bowel is manipulated and compressed by retractors. To the extent that this theory is correct, intraoperative oxygen presumably contributed more to reducing the incidence of PONV than the 2 h of postoperative oxygen." See: Goll V, Akca O, Greif R, Freitag H, Arkilic CF, Scheck T, Zoeggeler A, Kurz A, Krieger G, Lenhardt R, Sessler DI. Ondansetron is no more effective than supplemental intraoperative oxygen for prevention of postoperative nausea and vomiting. Anesth Analg. 2001 Jan;92(1):112-7. 2. Carbon dioxide. Although intra-operative PaCO2 measurements have not been correlated with the incidence of PONV, there seems little doubt that 'Gasless' laparoscopy (which is associated with a lower end-tidal PaCO2) is associated with a lower incidence of PONV. See, for example: Br J Anaesth 1996 Nov;77(5):576-80: Gasless laparoscopic cholecystectomy: comparison of postoperative recovery with conventional technique. Koivusalo AM, Kellokumpu I, Lindgren L 3. Nitrogen. - This is a natural consequence of '1' above if nitrous oxide is not the balancing gas. 4. Nitrous oxide. There seems little doubt that the use of N2O increases the incidence of PONV. See, for example: Anesthesiology 1996 Nov;85(5):1055-62: Omission of nitrous

175

oxide during anesthesia reduces the incidence of postoperative nausea and vomiting. A metaanalysis. Divatia JV, Vaidya JS, Badwe RA, Hawaldar RW Kovac has recently reviewed the whole area of PONV. See: Drugs 2000 Feb;59(2):213-43: Prevention and treatment of postoperative nausea and vomiting. Kovac AL Number: 8 Mild, accidental hypothermia during the course of major surgery may lead to significant increases in: 1. The requirement for peri-operative blood transfusion. 2. The duration of stay in the anaesthetic recovery room. 3. The incidence of post-operative wound infection. 4. The incidence of post-operative myocardial ischaemia. A: 1,2,3 Correct B: 1,3 Correct C: 2,4 Correct D: 4 Correct E: All Correct Correct Answer: E Mild, accidental hypothermia (<35 centigrade) is emerging as a major cause of perioperative morbidity. It is a potent cause of coagulopathy (Johnson TD, Chen Y, Reed RL: Functional equivalence of hypothermia to specific clotting factor deficiencies. J Trauma 37:413, 1994.) which can be easily overlooked as laboratory testing is carried out at 37 centigrade. It has also been demonstrated to cause a significant increase in post-operative blood loss (Schmied H, Kurz A, Sessler DI, Kozek S, Reiter A. Mild hypothermia increases blood loss and transfusion requirements during total hip arthroplasty. Lancet. 1996 Feb 3;347(8997):289-92.). The incidence of post-operative wound infection is at least doubled in those whose body temperature falls to less than 35 centigrade (Kurz A, Sessler DI, Lenhardt R. Perioperative normothermia to reduce the incidence of surgical-wound infection and shorten hospitalization. Study of Wound Infection and Temperature Group. N Engl J Med. 1996 May 9;334(19):120915.). Similarly, duration of stay in the recovery ward is virtually doubled (Lenhardt R, Marker E, Goll V, Tschernich H, Kurz A, Sessler DI, Narzt E, Lackner F. Mild intraoperative hypothermia prolongs postanesthetic recovery. Anesthesiology. 1997 Dec;87(6):1318-23.). In a study of patients undergoing peripheral vascular surgery (Frank SM et al: Unintentional hypothermia is associated with postoperative myocardial ischemia. The Perioperative Ischemia Randomized Anesthesia Trial Study Group. Anesthesiology. 1993 Mar;78(3):46876.), it was found that the incidence of postoperative angina was greater in the hypothermic group (18%, 6 of 33) than in the normothermic group (1.5%, 1 of 67, P = 0.002). The

176

incidence of PaO2 < 80 mmHg in the arterial blood was greater in the hypothermic group (52%, 17 of 33) than in the normothermic group (30%, 20 of 67, P = Number: 14 Of the following possible complications of electro-convulsive therapy (ECT) the most common is: A. Dental damage. B. Aspiration pneumonitis. C. Bronchospasm. D. Laryngospasm. E. Hypertension Select the single best answer Correct Answer: E Tecoult and Nathan reviewed 612 electroconvulsive therapy procedures carried out under propofol anaesthesia on 75 patients. In their series, hypertension was the most common complication (15%). Laryngospasm occurred in about 5% of patients while aspiration pneumonitis, dental damage and bronchospasm complicated less than 2% of cases. Of complications not listed above, confusion and headache were reported by 33% and 10% of patients respectively. See: Tecoult E, Nathan N. Morbidity in electroconvulsive therapy. Eur J Anaesthesiol. 2001 Aug;18(8):511-8. The haemodynamic response to ECT (hypertension, tachycardia) is well-recognised, although the exact mechanism remains uncertain. See, for example, Petrides G, Maneksha F, Zervas I, Carasiti I, Francis A. Trimethaphan (Arfonad) control of hypertension and tachycardia during electroconvulsive therapy: a double-blind study. J Clin Anesth. 1996 Mar;8(2):104-9 and Weinger MB, Partridge BL, Hauger R, Mirow A. Prevention of the cardiovascular and neuroendocrine response to electroconvulsive therapy: I. Effectiveness of pretreatment regimens on hemodynamics. Anesth Analg. 1991 Nov;73(5):556-62. Number: 17 Which of the following drugs are believed to be effective in the treatment of post-operative shivering? 1. Clonidine. 2. Tramadol. 3. Pethidine. 4. Paracetamol

177

A: 1,2,3 Correct B: 1,3 Correct C: 2,4 Correct D: 4 Correct E: All Correct Correct Answer: A With the exception of paracetamol, all of these drugs may be of some use in the treatment of post-operative shivering. In a recent study, Bhatnagar et al demonstrated that tramadol was a remarkably effective form of therapy. In their discussion, they commented that "Although the aetiology of postoperative shivering is inadequately understood, various risk factors have been suggested. These include hypothermia, stress, uncontrolled pain, uninhibited spinal reflexes and decreased sympathetic activity. Many drugs have been used to treat shivering, including opioids, doxapram, tramadol, ketanserin, clonidine, propofol, physostigmine and nefopam, with opioids being the most extensively evaluated. Amongst the opioids, pethidine has been found to be most efficacious. Evidence suggests that kappa-opioid receptors play an important role in the modulation of postoperative shivering. This explains the greater efficacy of pethidine compared with equi-analgesic doses of mu-receptor opioid agonists such as morphine, fentanyl, alfentanil and sufentanil. The analgesic potential of tramadol is believed to be mediated weakly through its effect on the mu-opioid receptor, for which it has a low affinity. Of greater importance may be its effect on 5-HT3 and noradrenergic receptors, with activation of descending inhibitory pathways producing antinociception. The R (+) enantiomer of tramadol inhibits 5-HT3 uptake and enhances its release, while the L (-) enantiomer inhibits nonadrenaline uptake. Electrophysiologic, neurophysiologic and neuropharmacologic experiments in animals have established the role of noradrenaline and 5-HT3 in the control of body temperature. Activation of the nucleus raphe magnus, where 5-HT3 acts as a neurotransmitter, has an inhibitory effect on shivering. It is thus possible that the antishivering effect of tramadol is mediated by its effect on these receptors. We postulated that it was likely to have better clinical utility compared with pethidine for the management of postoperative shivering." See: Bhatnagar S, Saxena A, Kannan TR, Punj J, Panigrahi M, Mishra S. Tramadol for postoperative shivering: a double-blind comparison with pethidine. Anaesth Intensive Care. 2001 Apr;29(2):149-54. Number: 22 The prevalence of the use of 'alternative' medicines in an unselected group of adults presenting for surgery in California is approximately: A. 10%. B. 20%. C. 30%. D. 40%. E. 50%. 178

Select the single best answer Correct Answer: D Leung et al found that 39% of a group of 2560 patients awaiting surgery in the San Francisco area admitted to taking alternative medicine supplements. In about 2/3rds of these cases the supplement was a herbal medicine. Many herbal medicines (Garlic, Ginger, Ginko and Ginseng) are believed to impair platelet function and St John's Wort may have monoamine oxidase inhibitory activity. See: Leung JM, Dzankic S, Manku K, Yuan S. The prevalence and predictors of the use of alternative medicine in presurgical patients in five california hospitals. Anesth Analg. 2001 Oct;93(4):1062-8. Number: 23 The relationship between increasing age and the Minimum Alveolar Anaesthetic Concentration (MAC) of the volatile agents is best described as one that: A. Increases by 6% per decade of life. B. Does not vary with decade of life. C. Decreases by 6% per decade of life. D. Decreases by 12% per decade of life. E. Decreases unpredictably per decade of life. Select the single best answer Correct Answer: C Both Mapleson and Eger have undertaken meta-analyses which suggest that MAC decreases by 6% per decade of life in a more or less linear fashion. See: Eger EI 2nd. Age, minimum alveolar anesthetic concentration, and minimum alveolar anesthetic concentration-awake. Anesth Analg. 2001 Oct;93(4):947-53. And: Mapleson WW. Effect of age on MAC in humans: a metaanalysis. Br J Anaesth. 1996;76: 179-185. Number: 46 Which of the following significantly increase the risk of peri-operative myocardial infarction.? 1. Untreated Hypertension in the absence of Coronary Artery Disease. 2. Signs of Congestive Cardiac Failure. 3. Previous Coronary Artery Grafting.

179

4. Unstable Angina A: 1,2,3 Correct B: 1,3 Correct C: 2,4 Correct D: 4 Correct E: All Correct Correct Answer: C Preoperative findings that correlate with perioperative myocardial infarction (PMI) include severe CCF (associated with S3 or raised JVP) and unstable angina. Hypertension does not correlate despite its prevalence in this population. Several studies have concluded that prior coronary artery bypass grafting lowers the risk of PMI. Peri-operative myocardial infarction remains a major cause of morbidity and mortality associated with surgery. Many occur on the third postoperative day. Many contributing factors not directly related to anaesthesia have been suggested. These include the hypercatabolic and hypercoaguable responses to surgery, anaemia, and pain. The period of anaesthesia often accounts for a limited portion of the perioperative period, however, the separation of anaesthesia and events occurring postoperatively has not been achieved. The correlation between preoperative factors and perioperative myocardial ischaemia in cardiac and non-cardiac surgery has been extensively studied. The issue is not wholly resolved. The major studies involving cardiac surgery have looked at anaesthetic technique and intraoperative haemodynamic instability and outcome. Slogoff and Keats, and Tuman could not find any correlation between different agents or anaesthetic techniques and intraoperative ischaemia or postoperative MI. Mangano provides editorial comment highlighting the difficulty separating anaesthetic from surgical factors. In non-cardiac surgery, the highest risk occurs in thoracic, major vascular, and upper abdominal surgery. Of the extensive literature available, studies by Goldman, Rao, and Steen are prominent. Many of the studies incorporate a scoring system which correlates the presence of preoperative factors with PMI. Those factors which appear consistently in the studies are: (1) Previous MI:Numerous studies have shown that if previous AMI precedes surgery by less than 6 months, the perioperative reinfarction rate is 5-86% and mortality rate is 23-86%. After 6 months the reinfarction rate stabilizes at 2-6%. (Goldman, Steen, Rao, Tarhan, Eerola, Schoeppel, Fraser, Topkins, Sapala). A commonly quoted figure is: 5% overall reinfarction rate; 15-30% if previous MI is within 3 months; 10-15% within 3-6 months . The study by Rao is of special interest because it proposes to show a major reduction in perioperative reinfarction rate (7.7% to 1.9% overall and 36% to 5.8% within the 0-3 month group). This was attributed to invasive monitoring and rapid treatment of cardiovascular variables both intraoperatively and within the first 72 hours postoperatively. (2) Congestive Cardiac Failure.-Master, Skinner & Pearce, Goldman, Rao and Mangano all describe CCF as a risk factor for surgery. It is associated with a high rate of reinfarction. More specifically, signs of poorly controlled or decompensated left ventricular failure, such as

180

an elevated JVP, S3 or pulmonary oedema are associated with a 20% mortality from cardiovascular complications, including myocardial infarction. (3) Unstable or poorly controlled angina. (4) Aortic stenosis is the only valve lesion which has been identified as a factor which increases cardiac risk for non-cardiac surgery. It appears low on the Goldman Index and high on Detsky's Modified list. Early work by Skinner & Pearce evaluating patients with rheumatic heart disease in non-cardiac surgery found a 10% mortality in aortic valve disease versus 6% for mitral valve disease. This difference was more pronounced in intra-abdominal and intrathoracic procedures (20% v 0%). (4)Prior CABG surgery. In general, authors have reported a reduction in risk. Maher has compared outcomes from non-cardiac surgery in patients who have had prior CABG surgery versus medically treated, angiographically proven coronary artery disease (CAD). Patients with a history of previous CABG had a PMI rate of 0% compared with 5% for the latter group. Repeat revascularization is sometimes needed and this protection probably diminishes as the time from surgery increases. (5) Long standing hypertension is a marker for CAD, and is clearly associated with cardiac hypertrophy, changes in cerebral blood flow, autoregulation and baroreceptor responsiveness, and renal function. Cardiovascular responses to anaesthesia are pronounced in hypertensive patients. However, the literature does not clearly show that preoperative hypertension increases cardiac risk. In Goldman's study it was found not to correlate; whereas in Steen's it did. Antihypertensive therapy has not demonstrably reduced the incidence of PMI. (6) Therapy for ventricular dysrhythmias has not greatly reduced the incidence of sudden death . Much of the early work evaluating cardiac risk concentrated on clinical risk factors as predictors of outcome. Later work evaluated the usefulness of monitoring techniques and preoperative investigations in identifying high risk groups, and the efficacy of improved control of CAD through pharmacological intervention perioperatively. A summary of the literature available on this topic follows: Barter, 1930 - recognized that patients who underwent surgery after an AMI had a high mortality. Master, 1937,1938 - described age > 60, prior CAD, cardiomegaly and preoperative ECG abnormalities as predictive indicators of PMI; that PMI typically occurred on day 3 and had a 65% mortality. Knapp, 1962 - Topkins & Artuso 1964 demonstrated previous AMI or CVA to predispose to further events. Arkins, 1964 - found that recent AMI, poor physical condition and emergency surgery predisposed to PMI which had a mortality of 69%.

181

Skinner & Pearce, 1964 - characteristics associated with increased mortality included intraperitoneal and intrathoracic surgery, severe hypertension , aortic valve disease, cor pulmonale, poor functional class, advanced age >75, ECG abnormality including AF, BBB, and multiple surgical procedures. MAYO Clinic report, 1972 - PMI more likely after upper abdominal and intrathoracic surgery; most frequent on 3rd postoperative day; associated with high mortality and not related to type of anaesthesia. AMI within 3 months was associated with 37% reinfarction rate, 16% within 3-6 months and 4-5% after 6 months. Goldman, 1977 - Cardiac risk index compiled 9 preoperative factors as independent predictors of postoperative cardiac complications including PMI, CCF and VT. Each factor was weighted such that multiple factors were summated to give an overall score of risk. This study suggested that history and physical examination account for 29 of 53 points in assessing cardiac risk. Patients were further segregated into classes which correlated with cardiac complications. Ambiguity arose in patients intermediate scores. In 1978, he suggested that the index be used to select 'intermediate' risk patients who could proceed with further evaluation.(ie: ambulatory ECG, Dipyridamole thallium scanning and/or coronary angiography). This was widely adopted despite criticisms of the use of sample groups which were too small , operator dependence on eliciting the more significant factors, and insufficient monitoring of postoperative ischaemia. Moreover, a number of studies evaluated the predictive value of the Goldman index. One (Waters, 1981) found it to be no better than the ASA Physical Status Classification and others found it to underestimate risk especially in major vascular surgery. Non-predictive factors included smoking, diabetes, hypertension, stable angina, BBBs and stable CCF (ie: absence of S3 or raised JVP) Steen, 1978 - confirmed that recent AMI was associated with high reinfarction rate. Predictive factors of PMI in Steen's study which had not correlated well in Goldman's study included preoperative hypertension, intraoperative hypotension, and duration of anaesthesia. Predictive factors common to both studies included intra-thoracic, intra-abdominal or major vessel surgery. Non-predictive factors common to both were diabetes, preoperative angina or anaesthetic technique. Of note was the high incidence of PMI in patients with 'old' AMIs (10.8%). The conclusion drawn was that in addition to old AMIs, these patients had experienced recent silent AMIs. Statistically, this group represented 1/4 of all reported previous AMIs. Detsky, 1986 - performed one of the studies which concluded that Goldman's Cardiac Index Score underestimated actual risk. In response, he compiled the Modified Multifactorial Index. In brief, this dropped uncontrolled LVF and added class 4 angina and symptomatic aortic stenosis to the list of risk factors. It was more complicated to apply and never proved to be of greater predictive value. Gerson, 1985 - found that poor performance on exercise ECG provided greater predictive data that was available from either Goldman data or radionucleotide ventriculograms. References

182

ROGERS,M.C ET AL (EDS); Principles and Practice of Anesthesiology, Mosby, 1993, pp 168-95 -an excellent review of the topic! MILLER,R (Ed), Anesthesia, Churchill Livingstone, 3rd Ed., pp 733-4, 165-178.

Number: 47 Which of the following clinical predictors of a difficult intubation is postulated to be responsible for a grade III Mallampati view of the oral cavity? A. Temporomandibular joint disease. B. Hypognathism. C. Restricted neck extension. D. A large tongue. Select the single best answer Correct Answer: D Many investigators have attempted to determine clinical signs which could be used as predictors of subsequent difficult intubation.The Mallampati classification attempts to grade the difficulty of intubation on the preoperative ability to visualize the faucial pillars, soft palate, and base of uvula. The postulate is that the size of the base of the tongue is an important factor determining the degree of difficulty of laryngoscopy. Since it is not possible to determine the volume or size of the base of the tongue relative to the capacity of the oropharyngeal cavity, it is inferred that the base of the tongue is disproportionately large when it is able to mask the visibility of the faucial pillars and uvula. The original classification involved three grades which described visibility of pharyngeal structures when the patient protruded the tongue maximally through the open mouth in the sitting position (nb: without saying Arrrrgh!). This was later modified to describe four classes: Class 1- Faucial pillars, soft palate and uvula can be visualized. Class 2- Faucial pillars and soft palate can be visualized, but uvula is masked by the base of the tongue. Class 3-Only the soft palate can be visualized. Class 4-The soft palate cannot be visualized. (Modified) The degree of difficulty of intubation is determined by laryngoscopic findings: Grade 1- Glottis (including anterior and posterior commissures) can be exposed. Grade 2- Glottis can be partly exposed (anterior not visualized). Grade 3- Glottis can not be exposed (corniculate cartilages only can be visualized). Grade 4- Glottis including corniculate cartilages can not be exposed. Grades 1 and 2 are considered " adequate exposure ", and grades 3 and 4 " inadequate exposure ".

183

In the original paper, all class 1 views correlated with either a grade 1 or 2 larynx. Conversely, of class 3 patients, 93% were either a grade 3 or 4 larynx. See: Mallampati SR, Gatt SP, Gugino LD, Desai SP, Waraksa B, Freiberger D, Liu PL. A clinical sign to predict difficult tracheal intubation: a prospective study. Can Anaesth Soc J. 1985 Jul;32(4):429-34.

Number: 49 Which of the following may confer difficulty in anaesthetising a patient with rheumatoid arthritis? 1. Reduced mouth opening. 2 .Oedematous larynx. 3. Atlantoaxial instability. 4. Hypognathism. A: 1,2,3 Correct B: 1,3 Correct C: 2,4 Correct D: 4 Correct E: All Correct Correct Answer: E Temporo-mandibular, crico-arytenoid, and cervical spine synovitis may all contribute to difficulty in airway management in a patient with rheumatoid arthritis (RA). Hypognathism is found in juvenile onset RA. Rheumatoid arthritis typically involves diarthritic joints. These contain two opposing cartilages with a synovial lined joint space and capsule. The cervical spine, temporomandibular and cricoarytenoid joints are examples of these. Synovitis of these joints often results in a constellation of changes which render the airway increasingly difficult to manage as the disease progresses. Temporo-mandibular joint ankylosis may manifest as limited mouth opening and an obscured view of the fauces. This may make oro-tracheal intubation difficult. The jaw may be tender and difficult to 'thrust' during mask anaesthetics. Crico-arytenoid joint ankylosis has been suggested to occur in up to 26% of patients with RA.(Funk). When the cricoarytenoid joint is involved, the patient may complain of recent changes in voice, dysphagia, dysarthria, hoarseness, stridor, and a sense of fullness in the oropharynx. Visualization shows oedematous, hyperaemic arytenoid mucosa with swollen aryepiglottic folds and false cords. The cords may show decreased movement. During anaesthesia, abrupt airway obstruction may be precipitated by the anaesthetic agent or by adjunctive drug-induced diminution in tone of the laryngeal musculature.

184

Cervical spine instability may present problems during anaesthesia even in the early stages of the disease. Synovial destruction and vertebral erosion, along with ligamentous changes at the atlas/odontoid or subaxial vertebral structures, may result in subluxation or cord compression. Knowledge of the degree instability becomes critical, particularly with attempts to manipulate the neck for airway management or positioning. Because of the immobility and shortening of the neck associated with advanced disease, the trachea becomes increasing rotated with an anterolateral displacement, which rotates the larynx, making laryngoscopy increasingly difficult. Cervical joint involvement has been reported in 15-86% of patients with RA. Higher incidence occurs in those sero-positive for rheumatoid factor, those with severe erosive polyarthropathy of peripheral joints and in patients on long term steroid therapy. Whereas RA has a female predeliction, males seem to be at greater risk of cervical spine involvement. Juvenile onset RAs are not at risk. Atlanto-axial (C1-C2) are most commonly involved (1535%). They do not correlate with disease duration. Depending on the particular supporting element which is destroyed, the atlas may sublux in relation to the axis in one of four directions: (1) Anterior AAS- comprises 80% of AAS. C1 moves forward on C2 due to transverse ligament destruction. Best seen by examining the flexion view of the lateral XR. Subluxation is said to exist when the distance between the atlas and odontoid peg in the lateral flexion view is > 3mm in patients > 44 or > 4mm in those < 44 years old. (2) Posterior AAS-rare, accounting for only 3-7% AAS. C1 moves backward on C2 due to peg destruction. Best demonstrated on lateral extension XR. (3) Vertical AAS-Comprises 10-20%. The lateral masses of C1 are destroyed. The odontoid peg may then sublux up through the foramen magnum to compress the cervicomedullary junction.Lateral XR views reveal that the process is superior to MacGregor's line by > 8mm in men and > 9.5mm in women. MacGregor's line joins the upper surface of the posterior edge of the hard palate and the most caudal point on the occiput. (4) Lateral/Rotatory AAS-rare, occurring in 2-5%. C1 moves laterally or rotationally due to facet joint destruction. Seen on lateral and open mouth views. Subaxial subluxation occur in 10-20% of the RA population and correlate with disease duration. It is caused by facet joint destruction and becomes symptomatic early causing cervical column and spinal root compression because of the smaller diameter at these lower levels. The commonest level is C5-C6. Lateral views may show a step deformity >2mm. While synovial joint destruction initially leads to subluxation, it is important to bear in mind that cervical spine ankylosis may develop as a final pathological state which results in a flexion deformity of the neck. See: ROGERS,M.C ET AL (Eds); Principles and Practice of Anesthesiology, Mosby, 1993, pp 219-20. Macarthur A, Kleiman S. Rheumatoid cervical joint disease--a challenge to the anaesthetist. Can J Anaesth. 1993 Feb;40(2):154-9.

185

Funk D, Raymon F. Rheumatoid arthritis of the cricoarytenoid joints: an airway hazard. Anesth Analg. 1975 Nov-Dec;54(6):742-5. Keenan MA, Stiles CM, Kaufman RL. Acquired laryngeal deviation associated with cervical spine disease in erosive polyarticular arthritis. Use of the fiberoptic bronchoscope in rheumatoid disease. Anesthesiology. 1983 May;58(5):

Number: 51 Abnormal preoperative pulmonary function tests in a patient with severe kyphoscoliosis might include: A. Increased RV/TLC. B. Reduced FEV1/FVC. C. Reduced FEV25-75. D. Increased FRC. E. All of the above. Select the single best answer Correct Answer: A Respiratory dysfunction associated with severe kyphoscoliosis primarily involves abnormalities in lung volumes and capacities. Total lung capacity and vital capacity are reduced. This occurs at a greater rate than reductions in residual volume and functional residual capacity, hence RV/TLC increases. Airways and lung parenchyma are essentially normal as are tests reflecting these. Scoliosis refers to lateral deviation of the spine and consists of the initiating curve and a compensatory curve attempting to restore postural balance. Kyphosis refers to posterior angulation of the spine. Respiratory and cardiac compromise associated with these conditions are most common when they occur together. They correlate with the severity of the deformity. In general , they would be unexpected in the presence of < 20 degree kyphosis and < 10 degree scoliosis but may appear with milder disease. Severe kyphoscoliosis may be associated with dyspnoea, cyanosis, somnolence and corpulmonale. Alterations in lung function involve lung volumes and capacities. TLC and FRC are reduced. VC is markedly reduced with a smaller reduction in TV. The RV/TLC increases as does TV/VC. Because chest wall compliance is reduced, augmenting TV requires large increases in the work of breathing resulting in a tendency to employ shallow , rapid breathing when minute ventilation needs to be augmented. The patient's reserve may be quite limited and decompensation may occur during anaesthesia, sedation or infection. A restrictive pattern is observed with a normal FEV1/FVC. Typically, airways disease is not present, therefore tests of this should be normal (FEV25-75%, FEF50%, MEF etc.)

186

Pulmonary hypertension may develop. Compression and kinking of pulmonary vessels may cause an increase in PVR but this is more typically related to chronic hypoxaemia. There is some evidence that disordered sleep patterns and nocturnal hypoxaemia occur which contribute to this as in sleep apnoea. Cor pulmonale is a not uncommon sequel. See: Pehrsson K, Bake B, Larsson S, Nachemson A. Lung function in adult idiopathic scoliosis: a 20 year follow up. Thorax. 1991 Jul;46(7):474-8. BORDOW,R.A & MOSER,K.M (EDS);Manual of Clinical Problems in Pulmonary Medicine, Little Brown and CO, London, 3rd Ed, 1991, pp 335-6. Kafer ER. Respiratory and cardiovascular functions in scoliosis and the principles of anesthetic management. Anesthesiology. 1980 Apr;52(4):339-51. Number: 52 Which of the following has been shown to offer protection from gastric aspiration syndrome in a patient with symptoms of reflux ? 1. Cimetidine. 2. Metoclopramide. 3. Sodium citrate. 4. Atropine. Number: 53 Which of the following are true of adrenal suppression due to steroid therapy? 1. It is associated with atrophy of the adrenal glands. 2. It does not occur in patients receiving inhaled steroids. 3. It should be expected in anyone receiving > 5mg prednisolone daily. 4. Following cessation, the stress response normalises after 8 weeks. A: 1,2,3 Correct B: 1,3 Correct C: 2,4 Correct D: 4 Correct E: All Correct

Correct Answer: B

There are three components in the regulation of cortisol production. These are the basal production of cortisol by the adrenal cortex; negative feedback control of ACTH produced by 187

the hypothalamic-pituitary axis in response to plasma cortisol; and the augmented adrenal response to plasma ACTH. The administration of high doses of exogenous adrenocorticoids for prolonged periods may result in suppression of all components and atrophy of the adrenal cortex. In these cases, full function may not return for up to 9 months. The individual response to exogenous steroids is unpredictable. Many consider that suppression of adrenocortical function does not occur below a maintenance dose of 5mg of prednisolone daily or a duration of therapy less than 2 weeks regardless of maximal dosage. It has been observed after inhaled therapy. Despite these observations, however, there exists only a few reports of hypotension attributable to omission of replacement therapy in the perioperative period. Suppression of secretion of cortisol by the adrenal gland caused by exogenous steroids involves normal mechanisms of feedback inhibition. Responsiveness of the axis remains normal after a single dose and normalizes over a period of days if a short course has been given. When steroids have been given for a slightly longer period, suppression becomes a function of the duration of therapy as well as the total dose. If the dosage is high enough, adrenal atrophy and suppression of the entire axis may occur, hence even exogenous ACTH may not stimulate adrenal secretion. With lower doses, basal levels of cortisol may be low, but response to stimulation may be normal (ie:stress). Long term high dose therapy can cause profound and prolonged suppression of basal adrenal gland function, the axis, and responsiveness of the gland to ACTH. High dose therapy (50mg daily) for only 5 days may cause adrenal-pituitary-axis dysfunction. Adrenal response is better preserved with alternate day therapy. Both inhaled and topical steroids may impair the pituitary-adrenal response. Patients receiving the equivalent of 30mg/day for longer than 1-2 weeks may have impairment for up to 1 year. 9 months after discontinuation of therapy, most patients should have a normal stress response and normal basal secretion of cortisol. In general, any patient who has been receiving steroids for more than a day or two before surgery, or who has received steroids for prolonged periods within the last year, should be considered for replacement. See: Rogers, M.C et al (Eds); Principles and Practice of Anesthesiology, Mosby, 1993, pp 68-70, 1584-85. Udelsman R, Ramp J, Gallucci WT et al. Adaptation during surgical stress. A reevaluation of the role of glucocorticoids. J Clin Invest. 1986 Apr;77(4):1377-81. Number: 55 Which of the following is true of the immediate postoperative period in a young patient with previously normal lungs having upper abdominal surgery? A. Arterial oxygen tension will typically be reduced by an average of 10 mm Hg when breathing room air. B. Arterial oxygen tension will normalize after 15 minutes. C. Diffusion hypoxia is the major determinant of arterial hypoxaemia. D. The CXR will typically reveal no abnormalities. E. All of the above. Select the single best answer

188

Correct Answer: D Pulmonary dysfunction extending for a variable time into the postoperative period occurs invariably after upper abdominal surgery with the exception of young patients who have been anaesthetized for only a few minutes. It is most pronounced in the first two hours and may continue for 3-4 days. More commonly, it will begin to normalize after the first postoperative day in the absence of evolving complications. In the immediate postoperative period, arterial PaO2 will be reduced by up to 30 mm Hg when breathing room air. Many factors contribute to this event, however, intrapulmonary shunting secondary to atelectasis and exaggerated ventilation/perfusion inequality in general are felt to be the primary cause. In most cases, increasing the FIO2 to 35-40% will restore PaO2 to preoperative values. The CXR will typically appear normal unless frank pathology has developed like aspiration or CCF. After the first hour of recovery from anaesthesia, previously well patients who have undergone surgery to the limbs or body surface normally exhibit little deterioration of lung function and any apparent deterioration in lung function suggests intraoperative events including aspiration. In contrast, when the same patients undergo abdominal surgery, there is impairment of arterial oxygenation for at least 48 hours postoperatively. Exceptions include young patients anaesthetized for only a few minutes. The magnitude of the hypoxaemia is related to the size and site of the incision. Typically, if the patient breathes air, the PaO2 is reduced by up to 30 mm Hg. It has been shown that 30% of patients breathing air on the way to recovery will have SAO2 < 90% and 12% < 85%. Multiple factors contribute including low FIO2 and hypoventilation. Diffusion hypoxia contributes in the first 10 minutes.The main cause however, is an increase in intrapulmonary shunt The causes may be considered as an extension of intraoperative factors and the development of new factors. The intraoperative factors include redistribution of ventilation and perfusion in the supine, anaesthetised, and paralysed patient resulting in a decrease in FRC, encroachment of closing capacity upon FRC and an increase in scatter of V/Q relationships. Central respiratory depression, and dyscoordinate activity of respiratory muscles from residual effects of anaesthetic agents compounds this in the early postoperative period. The fall in FRC increases postoperatively and is maximal on the first to second postoperative day. Reasons include diaphragmatic splinting due to pain, abdominal distension associated with ileus and retained pneumoperitoneum and prolonged rest in the supine position. Hypoxaemia will be most severe in the first 2 hours postoperatively but will remain impaired until FRC normalizes. If the fall in FRC is complicated by sputum retention and bacterial colonization of collapsed segments , infection may prolong recovery and hypoxaemia. A CXR performed at this time will often appear within normal limits See: SPENCE, A; " Postoperative Pulmonary Complications ", in GRAY, NUNN & BROWN; Anaesthesia, pp 1149-59. Schwieger I, Gamulin Z, Suter PM. Lung function during anesthesia and respiratory insufficiency in the postoperative period: physiological and clinical implications. Acta Anaesthesiol Scand. 1989 Oct;33(7):527-34.

189

Simonneau G, Vivien A, Sartene R, Kunstlinger F, Samii K, Noviant Y, Duroux P. Diaphragm dysfunction induced by upper abdominal surgery. Role of postoperative pain. Am Rev Respir Dis. 1983 Nov;128(5):899-903. Brown LT, Purcell GJ, Traugott FM. Hypoxaemia during postoperative recovery using continuous pulse oximetry. Anaesth Intensive Care. 1990 Nov;18(4):509-16. Mitchell C, Garrahy P, Peake P. Postoperative respiratory morbidity: identification and risk factors. Aust N Z J Surg. 1982 Apr;52(2):203-9. Number: 56 With regard to diffusion hypoxia: 1. It is likely to occur during emergence from Xenon anaesthesia. 2. It is a common mechanism of hypoxia when entonox is used in labour 3. It may contribute to hypoxaemia present 1 hour following cessation of administration of N2O. 4. It may cause alveolar hypoventilation A: 1,2,3 Correct B: 1,3 Correct C: 2,4 Correct D: 4 Correct E: All Correct Correct Answer: D This term is applied to a transient decrease in Pa02 of about 5-10 mmHg associated with a return to breathing air after the use of N2O in O2. The primary cause is a dilution of the alveolar O2 content as the more soluble N2O leaves the tissues at a faster rate than N2 returns. It does not occur when a relatively insoluble gas (such as Xenon) is administered. A secondary factor, which prolongs and aggravates the effect, is alveolar hypoventilation, consequent upon dilution of the alveolar PCO2. At worst, the reduction in Pa02 attributable to this effect lasts for not more than 10 mins. Because N2O is relatively insoluble in blood, brain and muscle, discontinuation of N2O is followed by a 50% decrease in brain levels in less than 5 minutes regardless of the length of anaesthesia. Supplemental 02 given for 10 minutes after the discontinuation of N2O will prevent this effect. See: Einarsson S, Stenqvist O, Bengtsson A, Noren H, Bengtson JP. Gas kinetics during nitrous oxide analgesia for labour. Anaesthesia. 1996 May;51(5):449-52. Fink BR, Diffusion Anoxia, Anesthesiology, 16,pp 511-19, 1955.

190

Schwieger I, Gamulin Z, Suter PM. Lung function during anesthesia and respiratory insufficiency in the postoperative period: physiological and clinical implications. Acta Anaesthesiol Scand. 1989 Oct;33(7):527-34. Calzia E, Stahl W, Handschuh T, Marx T, Froba G, Georgieff M, Rademacher P. Continuous arterial P(O2) and P(CO2) measurements in swine during nitrous oxide and xenon elimination: prevention of diffusion hypoxia. Anesthesiology. 1999 Mar;90(3):829-34. Number: 57 Which of the following surgical incisions is associated with the highest risk of postoperative pulmonary complications? A. Vertical laparotomy. B. Horizontal laparotomy. C. Lateral thoracotomy. D. Median sternotomy. E. Cholecystectomy Select the single best answer Correct Answer: C Postoperative pulmonary complications as measured by impairment of oxygenation correlate strongly with the type of surgery. In decreasing order are lateral thoracotomy, median sternotomy, vertical midline and paramedian abdominal, upper lateral abdominal and lower abdominal incisions. With respect to falls in FRC, a reduction of 30% following upper abdominal surgery compared with 15% following inguinal hernia repair have been shown. Alternatively, a pulmonary complication rate of: 30-40%-upper abdominal ; 10-16% lower abdominal; <10% -non-thoracic, non-abdominal surgery SPENCE, A.; " Postoperative pulmonary complications ", chapter 96 NUNN, UTTING & BROWN, Anaesthesia. Number: 58 The following are true of Mendelson's Syndrome: 1. Critical volume of aspirate is 30 mls. 2. Critical pH of gastric aspirate is 1.5. 3. Onset of symptoms generally occurs within 30 minutes. 4. Steroids have been shown to improve outcome. A: 1,2,3 Correct B: 1,3 Correct C: 2,4 Correct D: 4 Correct E: All Correct

191

Correct Answer: B A patient is thought to be at risk when > 25-30 mls (0.4 ml/Kg) of gastric aspirate of a pH <2.5 is regurgitated. Higher volumes are thought to be tolerated if the pH is higher. It s felt that the pH is a more critical determinant of lung injury. Several studies indicate that corticosteroid therapy may provide some modification of the inflammatory response early after aspiration, but in essence, does not alter the course of the illness. Bordrow, R.A & Moser, K.M; Manual of Clinical Problems in Pulmonary Medicine, Little Brown & Co., 3rd Ed., 1991, pp 72-75. Mendelson, C.L; " The aspiration of stomach contents into the lungs during obstetric anaesthesia. ", Am J Obstet Gynecol, 52, 1946, pp 191-204. Number: 61 With respect to cardiac arrests occurring during anaesthesia: 1. Most occur during induction of anaesthesia. 2. The most common cause is failure of ventilation. 3. The most common preceding arrhythmia is bradycardia. 4. Most are considered unpreventable and untreatable A: 1,2,3 Correct B: 1,3 Correct C: 2,4 Correct D: 4 Correct E: All Correct Correct Answer: B With respect to cardiac arrest due to anaesthetic factors, most occur during the induction phase. Whilst failure of ventilation is the sequelae of many intraoperative complications, the most common cause is inappropriate drug administration; either frank overdose or patient intolerance to 'normal' dosages. Greater than 90% are heralded by bradycardia, however it was not specified whether this was solely sinus bradycardia or other bradyarrhythmias (slow idioventricular for example). Most of the arrests considered due to anaesthetic causes were also deemed preventable. Cardiac arrests due solely to anaesthesia were studied over a 20 year period. During this time, 798 arrests occurred. Most were ascribed to patient illness or surgical factors but 36 were considered to result from the anaesthesia. Contributing factors included: (1)Age - the incidence was three fold in children < 12 years. No correlation was found for advancing years.

192

(2)Time of day - no correlation was found between incidence and anaesthesia outside of normal working hours. (3)Phase of anaesthesia - the majority occurred induction. (4)ASA physical status - this comes under patient illness and not anaesthesia related. (5)Emergency surgery correlates with a sixfold increase in incidence. The causes may be considered as respiratory or non-respiratory. Respiratory causes culminate in hypoxaemia secondary to failure to ventilate for a number of reasons. The most common cause was related to the inappropriate use or overdose of an anaesthetic agent. Half of these were 'relative', occurring during induction in patients who were haemodynamically unstable with doses of anaesthetic within the usual clinical ranges. Absolute overdosage occurred in the rest.The latter group tended to have improved outcome. Cardiac arrest was preceded by bradycardia in 26/27 patients KEENAN, R.L; " Cardiac arrest due to anesthesia- a study of incidence and causes ", JAMA, vol 253, no 16, April 26, 1985, pp2373. KEENAN, R.L; " Decreasing frequency of anesthetic cardiac arrests ", J Clin. Anesth., vol 3, Sept/Oct, 1991, pp 354-7. Number: 62 Which of the following should be considered as the cause of generalized convulsions 20 minutes postoperatively? 1. Local anesthetic toxicity after use of lignocaine. 2 .Enflurane. 3. Propofol. 4. Preexisting grand mal epilepsy. A: 1,2,3 Correct B: 1,3 Correct C: 2,4 Correct D: 4 Correct E: All Correct Correct Answer: E Anaesthetic agents may have a role in the genesis of perioperative seizures. Several agents produce intraoperative EEG and clinical evidence of intraoperative seizures, but evidence that they produce postoperative seizures is comparatively weak. Enflurane produces epileptiform activity that is influenced by the depth of anaesthesia and the PaCO2. In healthy normocapnic people, EEG spiking is maximal at end-tidal concentrations of 2-3%, and grand mal seizure patterns occur at 3-6%. At a given enflurane concentration, hyperventilation increases seizure activity such that minimum epileptiform activity is approximately 1% lower at a PaCO2 of 20 mm Hg and is 1% higher at a PaCO2 of 60 mm Hg than it is at 40 mmHg. Even though

193

seizures have been reported days after enflurane anaesthesia in non-epileptic patients, EEG documentation of postoperative seizure activity is rare. Halothane, isoflurane, and N2O have also been the subject of isolated case reports of seizure like activity during exposure, but none are believed to cause postoperative seizures. It is known that propofol may activate preexisting seizure foci, but has until recently been considered not to precipitate epileptiform activity in patients without a seizure disorder. It is known to decrease the seizure duration in electroconvulsive therapy, and has been used successfully to treat status epilepticus. A growing number of reports describing convulsions, twitching, or opisthotonos has led to a challenge to its use in epileptics recently. By February, 1993, 105 reports had been received by the Australian Adverse Drug Reaction Committee. 45 of these were considered serious and included convulsions (32/45), twitching (14/45), or opisthotonos (5/45). 14 cases occurred immediately, during, or shortly after induction, and 17 cases occurred within 30 minutes to 6 hours. Symptoms lasted 20 seconds to three minutes in 16 cases, 45-180 minutes in 5 cases, and one case of status epilepticus occurred in a well controlled epileptic. Another patient continued to experience seizures intermittently for 48 hours. 5 of the patients had a history of epilepsy, 2 had had similar experiences with other drugs, and one patient had had an episode of seizures after a previous anaesthetic. References BENUMOF ,J.L & SAIDMAN, L.J; Anesthesia and Perioperative Complications, Mosby, 1992, pp 360-1. Australian Adverse Drug Reactions Bulletin, 1991, 10:8. Number: 63 With regard to laryngeal trauma associated with the placement of an endotracheal tube (ETT) 1. It is most likely to occur at the arytenoid cartilage. 2. Trauma to the vocal cords usually involves the left. 3. Vocal cord paralysis is caused by injury to the anterior branch of the recurrent laryngeal nerve. 4. Postoperative hoarseness greater than one week is likely to represent transient nerve injury. A: 1,2,3 Correct B: 1,3 Correct C: 2,4 Correct D: 4 Correct E: All Correct Correct Answer: A Laryngeal trauma due to the placement of an ETT occurs most commonly at the posterior arytenoids and involve sore throat secondary to pressure effects. Trauma found on examination most commonly involves haematomas of the vocal cords, which are likely to occur on the left as a result of placement by right handed anaesthetists. True cord paralysis is

194

generally due to a neuropraxia of the anterior branch of the recurrent laryngeal nerve as it passes between the laryngeal mucosa below and thyroid cartilage above. Postoperative hoarseness usually represents a haematoma which resolves over a period of 2-4 weeks. The placement of endotracheal tubes carries with it a number of complications. They may be classified temporally. Early (0-24 hrs)- transient sore throat and hoarseness -damage to hypoglossal or lingual nerves -glottic oedema -supraglottic oedema -retroarytenoid oedema -subglottic oedema -vocal cord paralysis Medium (24-72 hrs)-infection Late (72+ hrs)-laryngeal ulcer, granuloma and polyp -synechia of vocal cords -laryngotracheal membranes and webs -laryngeal fibrosis -tracheal fibrosis -stricture of the nostril Sore throat postoperatively is relatively common. The majority of cases are probably caused by pressure induced injury of the arytenoid mucosa. The arytenoid mucosa contacts the ETT at its point of maximal curvature and therefore exerts significant pressure in this area. Sore throat correlates with the size of the ETT(32% in 8.5-9.00mm ETTs; 18% in 6.5-7.00mm ETTs); length of intubation and variably with the use of succinylcholine. It does not correlate with the use of lignocaine ointment or spray. Several large studies of laryngeal injury after short term intubation have shown an incidence of laryngeal injury of 6-7%. The majority of these are haematomas of the vocal cords, usually the left, since the ETT is usually inserted by a right handed anaesthetist and rotated to the left. The vocal process of the arytenoid is the most likely site of damage as it sits between the cords. In one of these studies, laryngoscopy was performed immediately after extubation. Haematoma occurred in 4.5% of these. It was more common in patients suffering allergic laryngitis or when the cords were not fully relaxed at intubation This is generally a benign lesion that resolves over 2-4 weeks. It would be likely to manifest as a resolving postoperative hoarseness. Persistent postoperative hoarseness may represent granuloma or polyp formation. These are rare complications of ulceration secondary to pressure on the posterior commissure. Persistent hoarseness and odynophagia has also been reported in arytenoid dislocation and responds to reduction. Persistent hoarseness is an indication for otolaryngeal evaluation. In the study described above, laceration of the mucosa of the vocal cord occurred in only 0.8%, and subluxation of the arytenoid in only 0.1% Vocal cord paralysis is an uncommon sequelae following tracheal intubation. It is most often attributed to thyroid, carotid or head and neck surgery where direct or indirect injury to the recurrent laryngeal nerve has occurred, however,it has been reported in 3% of non-head and

195

neck cases. Additionally, it is possible to cause lasting voice changes from damage to the external laryngeal nerves. True vocal cord paralysis may be related to endotracheal intubation. One review found only 32 cases that could plausibly be attributed to the ETT and in common with other complications of intubation, is possibly associated with a known difficult intubation. The most likely cause of vocal cord paralysis, exclusive of traumatic or difficult intubation, is pressure by the cuff on the vulnerable portion of the anterior branch of the recurrent laryngeal nerve, 6-10mm below the cords as it runs between the cuff and the overlying thyroid cartilage. In this injury, cord adductors are paralyzed so that hoarseness rather than airway obstruction is produced. The latter would result from paralysis of the cord abductors which are not innervated by this nerve. These injuries are generally caused by a neuropraxia which resolves in 2-4 weeks without specific intervention. It may be prevented by positioning the cuff lower in the trachea such that it can be felt in the suprasternal notch, and by monitoring cuff pressures. The recurrent laryngeal nerve has an anterior branch which innervates the lateral cricoarytenoid muscles (vocal cord adductors), and posterior branch which innervates the posterior cricoarytenoid and interarytenoid muscles. Other causes of nerve damage during endotracheal intubation include damage to lingual or hypoglossal nerves due to pressure from the laryngoscope blade in the vallecular region behind the tongue. Right sided lingual nerve damage is more common and usually resolves over a few months. Some inconsistency exists in the literature with regard to this issue. Atkinson says that laryngeal or supraglottic haematomas usually clear up in a few days (rather than 2-4 weeks), and that the occurrence of this complication bears no relation to the difficulty of intubation. Additionally, bilateral cord paralysis occurring from pressure of the cuff on the laminae of the thyroid cartilage may result in airway obstruction requiring CPAP or reintubation. This is in contrast to the earlier statement that hoarseness is the most serious sequelae. Permanent voice changes have been reported in 3%. Laryngoscopy is recommended for hoarseness persisting greater than 1 week. References ROGERS, M.C ET AL (EDS); Principles and Practice of Anesthesiology, Mosby, 1993, p 2392-4. LATTO, I.P & ROSEN, M (Eds); Difficulties in Tracheal Intubation, Balliere Tindall, Sydney, p38, 43-6 - an excellent book covering anatomy, equipment and procedural aspects of intubation! ATKINSON, R.S; Lee's Synopsis of Anaesthesia, 11th Ed., p 231. Number: 64 Which of the following positions will prevent damage to the ulnar nerve in an anaesthetised patient. A. Supine, arm resting by side with forearms folded across chest. B. Supine, arm resting by side in supination. C. Supine, arm resting by side in pronation.

196

D. Supine, arm abducted 45 degrees in supination. E. None of the above. Select the single best answer Correct Answer: E Whilst supination of the forearm theoretically rotates the cubital tunnel away from the underlying surface, there is no evidence to confirm that any form of positioning will decrease the incidence of postoperative ulnar nerve palsy Ulnar neuropathy represents 1/3 of all claims related to nerve injury in the USA. The most common site of injury is in the cubital tunnel. Some types of surgery are associated with an exceptionally high incidence of nerve damage. One study found a 61% incidence of ulnar nerve conduction velocity slowing in cardiac surgery. Most commonly, the position of the patient during anaesthesia allows compression or elongation of the nerve in the cubital tunnel. Some positions are particularly prone to injury. For example,when the arm is tethered to the supine patient's side or abducted with the arm pronated, the elbow is rotated such that the cubital tunnel is in contact with the flat supporting surface. As a result of this rotation, the nerve is theoretically vulnerable to external compression. Conversely, supination of the forearm rotates the cubital tunnel away from the flat surface. Flexion of the elbow to greater than 90 degrees causes the arcuate ligament which forms the roof of the cubital tunnel , to become tense, thereby decreasing available space and increasing risk of compression. Despite the theoretical advantages, there is no proof that supination decreases the risk of postoperative palsy. In fact, there is evidence that ulnar nerve palsy may still occur despite accepted positioning practices. One revealing study (Kroll), examined the American Society of Anesthesiologists Closed Claims Study database to define the role of nerve damage in the overall spectrum of anesthesia-related injury that leads to litigation. Of 1,541 claims reviewed, 227 (15%) were for anesthesia-related nerve injury. Ulnar neuropathy represented one-third of all nerve injuries and was the most frequent nerve injury. Less-frequent sites of nerve injury were the brachial plexus (23%) and the lumbosacral nerve roots (16%). In a large proportion of cases, the exact mechanism of injury was unclear despite evidence of intensive investigation in the claim files. Median payment for nerve damage claims involving disabling injury was $56,000, which was significantly lower than the $225,000 median payment for claims for disabling injury not involving nerve damage (P less than 0.01). The closed claims reviewers judged that the standard of care had been met significantly more often in claims involving nerve damage than in claims not involving nerve damage. The authors conclude that nerve damage is a significant source of anesthesia-related claims but that the exact mechanism of nerve injury is often unclear. In particular, ulnar nerve injuries seemed to occur without identifiable mechanism. References STOELTING, R.K; " Postoperative ulnar nerve palsy- is it a preventable complication? ", Anesth Analg, 76, 1993, pp 7-9. PERREAULT, L ET AL;" Ulnar nerve palsy at the elbow after general anaesthesia ", Can J Anaesth, 39, 5, 1992, pp 499-503.

197

KROLL, D.A. CAPLAN, R.A. POSNER, K & CHENEY, F.W;" Nerve injury associated with anesthesia. ", Anesthesiology, vol 73, no 2, 2Aug, 1990, pp 202-7. Number: 65 Which of the following is consistent with return of muscle tone adequate to protect the airway from aspiration? A. TOF ratio of 0.7. B. No fade detectable on Double Burst Tetanic Stimulation. C. Return of normal resting minute ventilation. D. 5 sec sustained head lift. E. None of the above. Select the single best answer Correct Answer: D With respect to the assessment of recovery from muscle relaxants, tests employing the peripheral nerve stimulator are used to quantify recovery of postsynaptic receptors at the neuromuscular junction, and yield information throughout the range of recovery of 50-90% of receptors. This is consistent with considerable diminution in muscle tone. Clinical tests may support return of muscle tone adequate to support ventilation, which correlates with a maximum inspiratory effort generating -20 cmH2O. Power required to protect the airway from aspiration has been found to correlate with a generatable inspiratory pressure of -40 cmH2O. This corresponds with a sustained head lift of 5 seconds. Criteria which suggest adequate ventilatory function which may be used to determine appropriateness of extubation after anaesthesia include: (1)resting minute ventilation < 10L. (2)the ability to voluntarily double the resting minute ventilation. (3)peak negative pressure on maximal inspiration > 30cm H2O. (4)FVC >10-15 ml/Kg. A patient may however be able to maintain adequate ventilation in the presence of residual muscle weakness that is inadequate to protect the airway from aspiration. A recent study compared the requirement for both events. In it, subjects were given d-tubocurarine to cause different levels of inspiratory muscle weakness as measured by the mean inspiratory pressure (MIP) generated by a maximal inspiratory effort. Four levels of power were evaluated: -90 cmH2O (control), -60, -40, and -20 cm H2O. At each level, vital capacity (VC), ETCO2, and hand grip strength were measured and muscles of airway protection were evaluated. These comprised ability to swallow, perform a Valsalva manoeuvre, prevent obstruction of the airway and approximate the teeth. These were compared with sustained head lift and straight leg lift. At MIP - 20cmH2O, VC was 2 L, and ETCO2 was normal. Hand grip strength was 0 and muscles of airway protection were still incapacitated.

198

Swallowing returned at a MIP> -43 cm H2O, approximation of the teeth at >-42 cmH2o, airway obstruction at > -39 cm H2O, and Valsalva at > -33 cm H2O. All subjects who could perform a sustained head lift could perform the airway protective manoeuvres. Tests employing PNSs are not sensitive with respect to assessing the adequacy of ventilation and airway protection, as for most the degree of receptor blockade is profound before they become abnormal (ie:~50-90% ) Thus, although ventilation may be adequate at -25 cm H2O, the muscles of airway protection are still non-functional and further reversal of neuromuscular blockade is required. References PAVLIN, E.G; " Recovery of Airway Protection Compared with Ventilation in Humans after Paralysis with Curare ", Anesthesiology, 70, 381-85, 1989. ROBBINS,M.C ET AL (EDS); Principles and Practice of Anesthesiology, Mosby, 1993, pp 2363. MILLER,R (ED); Anesthesiology, Churchill Livingstone, 3rd Ed., 1990, p 392. Number: 66 When comparing general versus regional anaesthetic techniques, the following is/are true of regional anaesthesia: 1. Urinary nitrogen is reduced postoperatively. 2. Intraoperative blood loss is reduced. 3. GIT motility is increased. 4. Perioperative cardiac complications are reduced A: 1,2,3 Correct B: 1,3 Correct C: 2,4 Correct D: 4 Correct E: All Correct Correct Answer: A When comparing the benefits of regional over general anaesthesia, the major issues can be summarized as: - Pain relief - Postoperative morbidity - Modification of metabolic response to surgery - Pulmonary complications - Blood loss - Thromboembolic complications - Cerebral function - Cardiac complications - Gastrointestinal function and complications - Mortality

199

- Length of hospital stay Most of these parameters are considered to benefit from regional anaesthetic techniques. Whilst theoretically it would appear that perioperative myocardial ischaemia and other cardiac complications would be decreased, no proof has yet appeared in the literature. The metabolic response to surgery is characterized by an increased secretion of catabolically acting hormones (cortisol, glucagon, catecholamines), whereas the anabolically acting hormones (insulin, testosterone, growth hormone) are mainly inhibited. The resulting effect is hypermetabolism and release of substrate from peripheral stores. This is reflected in an increase in urinary nitrogen.There is a tendency to suppression of immune function and changes in coagulation and fibrinolysis resulting in a hypercoaguable state are other features. The effect of general anaesthesia on the intraoperative metabolic response to surgery is either slight or nil and there is no effect on postoperative catabolism with the exception of high dose opioid anaesthesia which reduces a major part of the response intraoperatively; etomidate, which causes intraoperative suppression of adrenocortical responses; and hypothermia, which may inhibit catabolism. In contrast, the effect of regional anaesthesia is extensive. Much of the research has been conducted with intrathecal or extradural anaesthesia. The major mechanisms involved are inhibition of nociceptive signal from the surgical area to the CNS; Blockade of reflexes involving efferent autonomic and somatic pathways, and suppression of sensitization of nociceptors and other events involved in " wind-up " phenomena. This effect is most pronounced with procedures involving the lower part of the body (gynaecologic, urologic, orthopedic) whereas the effect is less pronounced with major abdominal and thoracic procedures. Reasons for this may include insufficiently blocked vagal, phrenic, sympathetic and somatic afferents. The timing and duration of blockade are important for magnitude and duration of the modification of the postoperative stress response. Post-traumatic blockade has less effect than a block instituted before the surgical incision. A single dose has only a short inhibitory effect whereas preliminary studies suggest that blockade for 24 hours will have a prolonged effect for up to 4 days on indices of the stress response. More than 20 studies have compared intraoperative blood loss and the subsequent need for blood transfusions during general and regional anaesthesia. Most studies were undertaken during elective hip surgery and show a reduction in intraoperative blood loss of approximately 30% in patients receiving regional anaesthesia. Similar results have been obtained in studies during prostatectomy and lower limb vascular surgery. Abdominal procedures show a similar trend but results have been statistically insignificant. Seven studies have shown that regional anaesthesia reduces postoperative DVTs by approximately 50% as compared with general anaesthesia without prophylaxis.These studies usually employed continuous epidural techniques in hip surgery.The effect after abdominal surgery is less and suggests that thoracic epidural techniques may not be as effective in altering components of Virchow's Triad as lumbar, however it should be noted that in this group low dose heparin prophylaxis was utilized. No data is available on graft patency rates with patients having distal vascular procedures, however , physiologic data show increased

200

lower extremity and graft blood flow. In all studies suggest greater efficacy of continuous epidural versus single shot techniques. The most important factor leading to postoperative ileus is activation of sympathetic inhibitory reflexes. Addition of N2O to Isoflurane has been shown to impair operative working conditions and to delay passage of flatus and faeces. The effects of systemic opioids are well documented. Continuous epidural bupivacaine appears to be the most effective measure to improve postoperative ileus. It is more effective than epidural opioids alone. Few studies are available comparing mixtures of epidural local anaesthetics and opioids with the former. Intraoperatively, intraluminal pressures are elevated but evidence of increased anastomotic breakdown have not been shown, in the wake of three case reports. Colonic peristaltic activity and blood flow are increased. Despite well documented physiologic advantages, the few studies that have evaluated clinically important cardiac complications only show an insignificant trend toward a beneficial effect in patients receiving regional anaesthesia.Thus it is not possible to draw conclusions. This is of particular relevance to major abdominal vascular surgery where outcome is highly associated with cardiac complications. References ROGERS, M.C. ET AL (EDS); Principles and Practice of Anesthesiology, Mosby, 1993, pp 1218-25. Number: 101 Which of the following signs of congestive cardiac failure constitute a major risk to the surgical patient? 1. Jugular venous distension. 2. Cardiomegaly. 3. Third heart sound. 4. Basal crepitations on auscultation. A: 1,2,3 Correct B: 1,3 Correct C: 2,4 Correct D: 4 Correct E: All Correct Correct Answer: B

Congestive cardiac failure has been identified as a major risk factor for cardiovascular complications following abdominal aortic aneurysm repair and other major vascular surgery. Studies have shown that in adults undergoing non-cardiac surgery, the presence of a S3 gallop and/or jugular venous distension are associated with about a 20% incidence of fatal cardiovascular complications. Decompensated congestive failure remains a stronger predictor

201

of perioperative cardiac mortality than recent myocardial infarction. Cardiomegaly and basal crepitations alone are consistent with stable, compensated heart failure and are associated with a significantly smaller risk (5%). Other conditions that are associated with an increased incidence of fatal cardiac complications are pulmonary oedema (14%), and cardiac related dyspnoea, orthopnoea, or peripheral oedema (6%). These findings suggest that the surgical patient with congestive cardiac failure can be stratified into two groups. Patients with decompensated heart failure, as manifested by S3, raised JVP, or pulmonary oedema comprise a high risk group with a 15-20% mortality from cardiorespiratory complications. Pulmonary venous congestion in these patients causes peribronchiolar oedema and areas of airway closure, resulting in intrapulmonary shunting. This further impairs cardiac function. Decreased cardiac output can also promote a rapid increase in the depth of anaesthesia and concomitant changes in circulatory dynamics and vital organ perfusion during induction of anaesthesia. The combined cardiopulmonary dysfunction predisposes to intraoperative hypoxaemia, hypotension, overt pulmonary oedema, metabolic acidosis and malignant arrhythmias. Finally, limited reserve results in poor tolerance of increased afterload as seen with surgically induced sympathetic stimulation, aortic cross clamping, etc. In contrast, patients with stable, haemodynamically compensated heart failure make up a relatively low risk group in which the incidence of fatal complications approaches 5%. These patients may exhibit one or more signs of diminished cardiac reserve, such as orthopnoea, basilar crepitations, or cardiomegaly. However, they are adequately compensated at rest. Operative mortality in these patients is determined less by existing haemodynamic compromise than by underlying cardiac pathology or associated medical conditions. In exception, if ejection fraction is documented to be < 35% in patients undergoing major vascular procedures, then the risk of perioperative myocardial infarction is significantly increased regardless of whether signs of decompensated heart failure are present. References MILLER, R. (ED); Anesthesiology, 3rd Ed., Churchill Livingstone, 1990, pp 1699-1700. Number: 104 A neurological assessment is required in the recovery room to exclude an intraoperative cerebrovascular accident. Which of the following are consistent with the normal postoperative state? 1. Hyperreflexia in the biceps muscles. 2. Fixed dilated pupils. 3. Bilaterally upgoing toes (positive Babinski reflex). 4. The presence of primitive reflexes (grasp, snout, palmomental). A: 1,2,3 Correct B: 1,3 Correct C: 2,4 Correct D: 4 Correct E: All Correct

202

Correct Answer: A 40%-100% of neurologically normal patients have an absent pupillary response to light 20 minutes after anaesthesia, and in 10% this can be absent for 40 minutes. Biceps and quadriceps hyperreflexia, unsustained ankle clonus and a positive Babinski reflex occur in a large number of patients recovering from anaesthesia, and in , many cases, abnormalities are present when the patients are fully awake. Abnormal reflexes occur more commonly after enflurane or halothane than N2O/opiate anaesthesia. A variety of other transient abnormalities have been reported during recovery from anaesthesia; opisthotonus and difficulty with eye opening have been associated with propofol, extrapyramidal reactions with droperidol, and seizures with several agents. Certain findings should be considered abnormal. These include unilateral reflex changes, and primitive reflexes- grasp, snout, palmomental, and Hoffman's. Ophthalmoplegia may represent basilar artery thrombosis. BENUMOF, J.L. & SAIDMAN, L.J.; Anesthesia and Perioperative Complications, Mosby, 1992, p 361. Number: 105 Which of the following conditions require endocarditis antibiotic prophylaxis ? 1. Mitral valve prolapse. 2. Presence of a permanent pacemaker. 3. Ventricular septal defect. 4. Previous coronary artery bypass graft surgery. A: 1,2,3 Correct B: 1,3 Correct C: 2,4 Correct D: 4 Correct E: All Correct Correct Answer: B Patients who have any form of valvular heart disease, intracardiac or intravascular shunts should have antibiotic prophylaxis against endocarditis. Endocarditis has occured in a sufficiently significant number of patients with hypertrophic cardiomyopathy (subvalvular aortic stenosis, asymmetric septal hypertrophy), and mitral valve prolapse to include such patients in a prophylaxis regime. Previous coronary artery bypass graft surgery or ligation of a patent ductus arteriosus arteriosus greater than 6 months previously, or presence of a pacemaker are considered to be associated with a low risk of endocarditis and do not require prophylaxis. In summary: Endocarditis prophylaxis recommended:

203

prosthetic cardiac valves most congenital cardiac malformations surgically constructed systemic-pulmonary shunts rheumatic and other acquired valvular dysfunction idiopathic hypertrophic subaortic stenosis past history of endocarditis mitral valve prolapse Endocarditis prophylaxis not recommended: Isolated secundum atrial septal defect patent ductus arteriosus ligated more than six months earlier postoperative coronary artery bypass graft pacemaker SHULMAN, S.T. ET AL; " Prevention of Bacterial Endocarditis ", Circulation, vol 70, no 6, Dec. 1994, 1123A-1127A. Number: 106 A patient with myasthenia gravis who is managed on oral neostigmine can be expected to have a prolonged response to which of the following agents during anaesthesia: A. Succinylcholine. B. Atracurium. C. Lignocaine. D. Vecuronium. Select the single best answer Correct Answer: A Anticholinesterases can inhibit plasma cholinesterase activity with a subsequent decrease in the metabolism of ester local anaesthetic agents, and the hydrolysis of succinylcholine. As in non-myasthenic patients, the duration of succinylcholine activity is inversely related to the plasma cholinesterase activity. In MG there is a decrease in the number of functional acetylcholine receptors available. This can decrease the response to the neurotransmitter and other depolarizing agents like succinylcholine and decamethonium, and cause marked sensitivity to non-depolarizing agents secondary to a with a decrease in the " safety " margin. These events are typically manifest as a need to increase the amount of available acetylcholine with neostigmine during general therapy, and atropinization during anaesthesia; a larger paralyzing dose of succinylcholine as compared with the normal patient; and smaller doses of non-depolarizing agents compared with the normal patient. Because of the decreased number of receptors, succinylcholine may not effectively depolarize the endplate resulting in " resistance ". The ED50 and ED95 in myasthenic patients is 2.0 and

204

2.6 times that of the normal patient. Thus, high doses of succinylcholine may be required for rapid sequence intubation. The endplate potential may not reach the threshold required for inducing a depolarizing " phase 1 " block and may readily induce a " phase 2 " block. Also if plasma cholinesterase has been inhibited, then hydrolysis may be decreased. Both of these events may result in potentiation and prolongation of its activity. The reduction of the number of receptors at the neuromuscular junction, and consequent reduction in the " safety margin " makes these patients extremely sensitive to nondepolarizing muscle relaxants. One tenth of the normal paralysing dose may be sufficient to paralyse a patient with MG, however, this varies with the severity of the disease. Also, this may not apply to intermediate acting agents, vecuronium and atracurium. Both of these are eliminated normally, and their short half lives allow the initial dose to be titrated against need with the expectation of full reversal at the end of the procedure as normal. References BAKARA, A; " Anaesthesia and Myasthenia Gravis ", Can J Anaesth, vol 39, no 5, 1992, pp 476-86. Number: 210 Which of the following are true of Latex allergy: 1. The incidence in Health Care Workers is a least three times that in those not exposed to latex. 2. Type IV reactions are more common than Type I reactions. 3. Is four times as common in women as opposed to men. 4. Is more common in those allergic to bananas. A: 1,2,3 Correct B: 1,3 Correct C: 2,4 Correct D: 4 Correct E: All Correct

Correct Answer: E Number: 302 A 75 year old patient with stable angina whose exercise capacity is limited to walking 200 metres before angina supervenes is in the ASA class of: A. I B. II C. III D. IV E. V

205

Correct Answer: C The American Society of Anesthesiologists classification is: I. No organic disease II. Mild or moderate systemic disease without functional impairment III. Organic disease with definite functional impairment IV. Severe disease that is life-threatening V. Moribund patient, not expected to survive Dripps RD, Lamont A, and Eckenhoff JE: The role of anesthesia in surgical mortality, JAMA 178:261, 1961. Number: 303 The most common intraoperative complication reported during anaesthesia in ASA I or II patients is: A. Dysrhythmia. B. Hypotension. C. Hypertension. D. Syringe Swap. E. Airway Obstruction Select the single best answer Correct Answer: A In 1986 Cohen et al. described the findings of a large prospective survey that identified major and minor complications in 112,721 patients over a 9-year period. More than 80% of the patients were healthy American Society of Anesthesiologists (ASA) physical status class I or II. All types of surgery and anesthetic techniques were included. The study indicated that 9% of all patients had at least one intraoperative complication. The two most common complications encountered in adults were dysrhythmias (3.9%) and hypotension (2.7%). The next most common were respiratory problems (1%), followed by incidents related to the drugs administered or to the surgical procedure performed. Cardiac arrest was very uncommon. It is of interest that during the period 1975 to 1978, 7.6% of patients experienced some intraoperative complication, whereas during the period 1979 to 1983, the incidence had increased to 10.6%. The authors of this study suggested that at least part of this increase was a result of improved monitoring, which permitted better detection of problems, rather than reflecting an actual increase in the overall incidence of intraoperative complications. In a similar study conducted to determine the incidence of intraoperative anesthetic complications in children, Cohen, Cameron, and Duncan found that 35% of pediatric patients experienced adverse perioperative events, of which 8.5% occurred during the intraoperative period. Cohen MM, Duncan PG, Pope WDB et al: A survey of 112,000 anaesthetics at one teaching hospital (1975-83), Can J Anaesth 33:22, 1986.

206

Cohen MM, Cameron CB, and Duncan PG: Pediatric anesthesia morbidity and mortality in the perioperative period, Anesth Analg 70:160, 1990. Number: 332 The appropriate size of laryngeal mask airway for a 40 kg patient is likely to be: A. 1. B. 2. C. 3. D. 4. E. 5. Select the single best answer Correct Answer: C The manufacturer recommends: Size 1: up to 6.5kgs. Size 2. 6.5 - 20 kgs. Size 2.5. 20 - 30 kgs. Size 3. 30 - 50 kgs. Size 4. 50 - 75 kgs. Size 5. Over 75 kgs. Number: 341 A healthy 20 year old is to be anaesthetised in a hospital at an altitude of 10,000 feet: 1. Nitrous oxide can be expected to be a useful component of a general anaesthetic. 2. The concentration of halothane delivered by a 'Tec' type vapouriser will be accurate. 3. His resting PCO2 is likely to be 40 mm Hg. 4. His arterial PO2 breathing air will be about 65 mm Hg. Select the single best answer Correct Answer: D At an altitude of 10,000 feet, the inspired PO2 is reduced to 110 mm Hg and PaO2 to 65 mm Hg in air. At this hypoxic level, alveolar ventilation increases and PaCO2 is steadily reduced (34 mm Hg at rest). The effective anesthetic power of nitrous oxide is also reduced as total barometric pressure decreases. It has been shown that analgesia induced by 50% nitrous oxide is reduced by nearly 50% at 5000 feet and it becomes insignificant at 10,000 feet. Therefore, nitrous oxide is not a useful anesthetic gas at altitude. Safar and Tenicela and James and White condemn the use of nitrous oxide for anesthesia at altitude.

207

The saturated vapor pressure of a volatile anesthetic agent depends only on temperature and is practically independent of total environmental pressure. Consequently, for a given vapouriser temperature, the concentration of a given mass of vapor increases as the barometric pressure is reduced, because the same mass of volatile agent is vaporized in less and less dense carrier gas. However, the partial pressure (expressed in mm Hg) of the agent remains unchanged, and so does its biological effect on the neural tissue where the anesthetic effect is produced. Safar P, Tenicela R: High altitude physiology in relation to anesthesia and inhalation therapy, Anesthesiology 25:515-531, 1964. James MFM, White JF: Anesthetic considerations at moderate altitude, Anesth Analg 63:1097-1105, 1984. Number: 397 Nerve injury in relation to anaesthesia is most likely to occur in the distribution of the: A. Supraorbital nerve. B. Mental nerve C. Ulnar nerve. D. Radial nerve E. Median nerve Select the single best answer Correct Answer: C Ulnar nerve injury is by far the most commonly reported neurological deficit reported in 'closed claims' analyses. For example, the percentage incidence in a survey of 227 cases by Kroll was: Ulnar 34 Brachial plexus 23 Lumbosacral nerve root 6 Spinal cord 6 Sciatic 5 Median 4 Radial 3 Femoral 3 Multiple nerves 2 Other nerves 5 Overall the risks of major neurologic deficits after any form of anesthesia are exceedingly rare. In a recent review of nerve injury in 227 patients associated with anesthesia, the mechanisms of injury often remained unclear despite extensive analysis.

208

The sensitivity of the ulnar nerve to damage is because of its superficial position as it passes behind the epicondyle of the humerus. Injury can occur because of hyperflexion of the elbow, which overstretches the nerve in a comatose patient, direct compression of the nerve against a hard object such as an armboard or side rail of the operating room table, or in relation to the use of automatically cycled blood pressure cuffs. Alexander GD: Mechanism of ulnar nerve injury, Anesthesiology 73:1294-1295, 1990. Kroll DA et al: Anesthesiology 73:202-207, 1990 Number: 399 The most likely adverse outcome of anaesthesia in closed claim studies is: A. Nerve damage. B. Airway trauma. C. Aspiration. D. Myocardial infarction E. Stroke. Select the single best answer In the American Society of Anesthesiologists Closed Claims Project, nerve damage was far more common than the other outcomes listed here. The percentage incidence of some adverse outcomes of 1541 settled claims is shown below. Death 37 Nerve damage 15 Brain damage 12 Low-severity injuries 11 Airway trauma 4 Pneumothorax 3 Eye damage 3 Aspiration 3 Pulmonary edema 3 Stroke 2 Hepatic dysfunction 2 Myocardial infarction 1 Number: 406 Intraneural pressure in the ulnar nerve at the elbow is highest if the arm is positioned: A. Elbow at side and extended, wrist supinated. B. Elbow at side and extended, wrist pronated. C. Elbow at side and flexed. D. Elbow flexed and above shoulder. E. None of the above.

209

Select the single best answer Correct Answer: D The intraneural pressure in the ulnar nerve is at its highest when the shoulder is fully abducted and the elbow is flexed. With the elbow at the side and extended, intraneural pressure is about 7 mm Hg. With the elbow at the side and flexed, intraneural pressure is about 11 mm Hg. With the elbow above the shoulder and flexed, intraneural pressure is about 47 mm Hg. Pechan J; Julis I: Pressure measurement in the ulnar nerve. A contribution to the pathophysiology of the cubital tunnel syndrome. J. Biomech 8: 75-79, 1975. Pronation and / or extension of the wrist tend to further increase intraneural pressure. It seems reasonable to assume that measures which minimise the rise of intraneural pressure during anaesthesia will reduce the risk of ulnar neuropathy, although, as yet, there is no clear evidence to support this view. Number: 409 With regard to critical incidents in the recovery ward: 1. They are associated with significantly more adverse outcomes than those occurring in theatre. 2. Reintubation is necessary in ~ 0. 1% of critical respiratory incidents in the recovery room. 3. The complication rate for removal of artificial airways in recovery is about 15%. 4. About 70% of critical incidents in the recovery room are circulatory problems. A: 1,2,3 Correct B: 1,3 Correct C: 2,4 Correct D: 4 Correct E: All Correct Correct Answer: A Van der Walt JH, Webb RK, Osborne GA, et al. Recovery room incidents in the first 2000 incident reports. Anaesthesia Intensive Care 1993; 21 :650-652. Vaughan RS. Airway Management in the recovery room. Anaesthesia 1997; 52:617-618. Number: 441 Which of the following statements are true regarding Broncho - Alveolar Lavage for a patient with pulmonary alveolar proteinosis?

210

1. A double-lumen endotracheal tube is absolutely indicated. 2. The right lung should be lavaged first. 3. Pre-oxygenation of the lavaged lung will improve the effectiveness of the procedure. 4. Lavage can be safely performed with isotonic saline infused at a height of 70 cm above the midaxillary line. A: 1,2,3 Correct B: 1,3 Correct C: 2,4 Correct D: 4 Correct E: All Correct Correct Answer: B Pulmonary Alveolar Proteinosis (PAP) is a condition in which the alveoli are filled with granular material that stains with Periodic Acid Schiff reagent, but there is no evidence of inflammation and septal architecture is relatively normal. The intra-alveolar material is a combination of surfactant phospholipid produced by alveolar type II pneumocytes and of other proteins and immunoglobulins found in alveolar lining fluid. The cytoplasm of alveolar macrophages appears engorged with inclusions (lamellar bodies). Whole-lung lavage provides relief from both the dyspnoea and the progressive deterioration in arterial oxygenation which affects many patients. It may also may provide long-term benefit. Broncho - Alveolar Lavage (BAL) is performed under general anaesthesia using warm, isotonic saline infused by gravity from a height of 30 cm above the midaxillary line. The resultant hydrostatic pressure of the lavage fluid with this setup is 30 mm Hg - a pressure at which the seal on the endobronchial cuff can still remain effective. The left lung, being smaller, should be lavaged first so that during the first period of one lung ventilation the largest lung is responsible for gas transfer. Pre-oxygenation is recommended in order to remove insoluble nitrogen from the alveoli and enhance distribution of the saline. The non- lavaged lung is ventilated with 100% O2. When lavage fluid ceases to flow (usually after 1000 ml in an adult), drainage is accomplished by clamping the inflow line and unclamping the drainage line, which runs to a collection bottle placed 20 cm below the midaxillary line. The application of mechanical chest percussion and vibration to the lavaged hemithorax before drainage enhances the yield of each lavage and thus shortens the time required for the procedure. Initially the lavage fluid is cloudy, turbid, and has a light-brown colour. Sediment collects at the bottom of the drainage bottle. The cycle of filling and drainage is then repeated until the lavage effluent becomes clear. It is extremely important to record accurately volumes delivered to and retrieved from each tidal lavage. Total lavage fluid volumes of 10 to 20 L are usually employed. After the effluent

211

lavage fluid becomes clear, the procedure is terminated, the lavaged lung thoroughly suctioned, and ventilation reestablished. The compliance of the lavaged lung is much less than that of the nonlavaged lung at this time. Large tidal volumes of 15 to 20 ml/kg to the lavaged side alone are necessary to reexpand the alveoli. The non-lavaged side is temporarily clamped after a large inspiration of 100% O2. Number: 443 The maximum amount of air recommended for use in a size 2 laryngeal mask airway is: A. 2 mls. B. 5 mls. C. 7 mls. D. 10 mls. E. 15 mls. Select the single best answer Correct Answer: D The manufacturer recommends: Size 1.0 1.5 2.0 2.5 3.0 4.0 5.0 Weight Air in Cuff under 5 kg 4 ml 5 to 10 kg 7 ml 10 to 20 kg 10 ml 20 to 30 kg 14 ml 30 kg to small adult 20 ml adult 30 ml big adult 40 ml

Number: 444 With regard to aspiration risk and the use of the laryngeal mask airway (LMA). 1. It has an overall incidence of about 1:5000 cases. 2. Gastric reflux to the level of the mid-oesophagus is greater when an LMA as opposed to face mask is used. 3. The risk may be increased by the effect of the mask on lower oesophageal sphincter tone. 4. The incidence Is increased if extubation is delayed until the patient is awake. A: 1,2,3 Correct B: 1,3 Correct C: 2,4 Correct D: 4 Correct E: All Correct

212

Correct Answer: A The incidence of aspiration associated with the laryngeal mask airway: a meta-analysis of published literature. Brimacombe JR; Berry A. J Clin Anesth, 7(4):297-305 1995 Jun STUDY OBJECTIVE: To determine the incidence of pulmonary aspiration with the laryngeal mask airway (LMA). Number: 478 With regard to latex-induced anaphylaxis: 1. A 'Neoprene' reservoir bag can be safely used in a patient who has had a previous reaction to latex. 2. Anaphylaxis is more likely to occur in a patient who is allergic to bananas. 3. Allergy is usually IgE-mediated. 4. RAST testing is more sensitive than intradermal testing in confirming the allergy. A: 1,2,3 Correct B: 1,3 Correct C: 2,4 Correct D: 4 Correct E: All Correct Correct Answer: A 'Neoprene' is completely synthetic and can be safely used in the breathing circuit. Allergic manifestations commonly depend on the route of exposure to latex: Cutaneous exposure causes contact pruritus, urticaria, eczema and dermatitis, whereas airborne exposure causes rhinitis, conjunctivitis and asthma. Mucosal or intravenous exposure results in angioedema and anaphylaxis. - However, any route of exposure may cause anaphylaxis. Ask about a history of atopy (hay fever, asthma) and multiple drug allergies as these patients are more likely to be allergic also to latex. Patients with certain food allergies (e.g.avocado, banana, chestnut) are also more likely to be allergic. The reaction is IGE-mediated. Skin-prick testing is less sensitive than intradermal test but more sensitive than RAST. Radioallergosorbent test (RAST) - an in-vitro test for IgE antibodies in the patient's serum is positive in 65 - 95% of cases. The Allergenicity of Latex Gloves:

213

Allergenicity of latex gloves is related to both the quantity, and the type of protein antigens they contain. Allergenicity can be suppressed by washing and steam sterilising the gloves during manufacture. Cornstarch powder is the most common lubricant and is associated with reactions due to the adsorption of antigens to the cornstarch particles, producing aerosolized latex antigen. Powder-free gloves are less allergenic. Note that the powder itself does not cause allergy. "Hypoallergenic" gloves are usually made from latex and should not be used to manage latexallergic patients. There is no standard for "hypoallergenic" latex gloves, which are often mistakenly assumed to be non-latex. Read the labelling carefully! Number: 486 With regard to desflurane: A. It has a boiling point of 29.2 Deg. C. B. Has a blood:gas partition coefficient higher than isoflurane. C. Has been reported as a triggering agent for malignant hyperthermia. D. Is associated with a low incidence of airway reflex responses when used for gaseous induction. E. None of the above. Select the single best answer Correct Answer: C ADesflurane is a volatile anaesthetic that combines low blood gas solubility (blood/gas partition coefficient = 0.42 at 37 degrees C), moderate potency (MAC = 6-7%), and high volatility (vapour pressure = 681 mmHg at 20 degrees C, boiling point = 23.5 degrees C). Anesthesiology 1999 Apr;90(4):1208-9 Garrido S, Fraga M, Martin MJ, Belda J Malignant hyperthermia during desflurane-succinylcholine anesthesia for orthopedic surgery. nesth Analg 1998 Jun;86(6):1328-31 Allen GC, Brubaker CL Human malignant hyperthermia associated with desflurane anesthesia Number: 487 Sevoflurane: 1. Is a halogenated ether. 2. Is flammable. 3. Has a MAC of about 2 vol%. 4. Has a blood/gas partition coefficient of about 3.

214

A: 1,2,3 Correct B: 1,3 Correct C: 2,4 Correct D: 4 Correct E: All Correct Correct Answer: B Sevoflurane is a halogenated methyl isopropyl ether. It is potent, non explosive and non flammable. It reacts with soda lime to form traces of a related ether which has not been shown to have any toxic effect on animals chronically exposed to it in a closed system. Induction of anaesthesia with sevoflurane is rapid and smooth, as predicted by a blood/gas partition coefficient of about 0.6 and an acceptable odour which allows the use of concentrations of up to 10%. Its MAC has been reported to vary between 1.7 and 2.3 vol %. Sevoflurane causes dose-dependent cardiovascular and respiratory depression. Its effect on the cerebral circulation is similar to that of isoflurane. The extent of biotransformation is similar to that of enflurane, but its low solubility and rapid elimination confine this to the period of inhalation. No toxic effects on the kidneys, liver and haematopoietic system have been found. Br J Hosp Med 1997 Jan 15-Feb 5;57(1-2):43-6 Grounds RM, Newman PJ 'Sevoflurane.' Number: 488 Carbon Monoxide (CO) is more likely to be produced by carbon dioxide absorbent in a circle system if: 1. Enflurane rather than sevoflurane is the volatile agent in use. 2. High concentrations of volatile agents are in use. 3. The carbon dioxide absorbent is 'baralyme' rather than soda lime. 4. The carbon dioxide absorbent is very moist A: 1,2,3 Correct B: 1,3 Correct C: 2,4 Correct D: 4 Correct E: All Correct Correct Answer: A Anesth Analg 1995 Jun;80(6):1187-93 Fang ZX, Eger EI 2nd, Laster MJ, Chortkoff BS, Kandel L, Ionescu P Carbon monoxide production from degradation of desflurane, enflurane, isoflurane, halothane, and sevoflurane by soda lime and Baralyme.

215

Anecdotal reports suggest that soda lime and Baralyme brand absorbent can degrade inhaled anesthetics to carbon monoxide. We examined the factors that govern CO production and found that these include: 1) The anesthetic used: for a given minimum alveolar anesthetic concentration (MAC)multiple, the magnitude of CO production (greatest to least) is desflurane > or = enflurane > isoflurane >> halothane = sevoflurane. 2) The absorbent dryness: completely dry soda lime produces much more CO than absorbent with just 1.4% water content, and soda lime containing 4.8% or more water (standard soda lime contains 15% water) generates no CO. In contrast, both completely dry Baralyme and Baralyme with 1.6% water produce high concentrations of CO, and Baralyme containing 4.7% water produces concentrations equaling those produced by soda lime containing 1.4% water. Baralyme containing 9.7% or more water and standard Baralyme (13% water) do not generate CO. 3) The type of absorbent: at a given water content, Baralyme produces more CO than does soda lime. 4) The temperature: an increased temperature increases CO production. 5) The anesthetic concentration: more CO is produced from higher anesthetic concentrations. These results suggest that CO generation can be avoided for all anesthetics by using soda lime with 4.8% (or more) water or Baralyme with 9.7% (or more) water, and by using inflow rates of less than 2-3 L/min. Such inflow rates are low enough to ensure that the absorbent does not dry out. See also: Anesth Analg 1995 Jul;81(1):144-6 Baum J, Sachs G, v d Driesch C, Stanke HG Carbon monoxide generation in carbon dioxide absorbents. Number: 502 Ulnar nerve injury occurring in association with anaesthesia is: 1. More common when regional rather than general anaesthesia is used. 2. More common in patients with an 'extreme' body habitus. 3. More common in women rather than men. 4. The most common single nerve injury. A: 1,2,3 Correct B: 1,3 Correct C: 2,4 Correct D: 4 Correct E: All Correct

216

Correct Answer: C Ulnar nerve injury is by far the most common nerve injury reported in association with anaesthesia. It occurs predominantly in men (75:25), who are either fat or thin and who have undergone general anaesthesia. See: Anesthesiology 1999 Apr;90(4):1062-9 Cheney FW, Domino KB, Caplan RA, Posner KL Nerve injury associated with anesthesia: a closed claims analysis. Anesthesiology 1994 Dec;81(6):1332-40 Warner MA, Warner ME, Martin JT Ulnar neuropathy. Incidence, outcome, and risk factors in sedated or anesthetized patients. Number: 504 Which of the following can be considered a 'normal' reaction to suxamethonium given to a child? 1. Masseter spasm. 2. Myoglobinuria. 3. Elevation of creatine phosphokinase (CPK). 4. Myalgia A: 1,2,3 Correct B: 1,3 Correct C: 2,4 Correct D: 4 Correct E: All Correct Correct Answer: E Masseter spasm is no longer thought to be an early sign of malignant hyperthermia. It is more likely that it reflects relative 'underdosing' of the drug. See: Br J Anaesth 1990 Apr;64(4):488-92 Leary NP, Ellis FR Masseteric muscle spasm as a normal response to suxamethonium. Succinylcholine, particularly in infants and small children, will result in damage to the muscle cell as evidenced by increased levels of creatine phosphokinase (CPK) and by myoglobinaemia and in some cases myoglobinuria. This is a normal response to succinylcholine. It can be minimized or prevented by the pretreatment with nondepolarizing muscle relaxants (curare or atracurium 0.05 mg/kg). This pretreatment will result in a pronounced reduction in the levels of CPK and so on and a considerable reduction in myalgias. Number: 510 The following exist as liquids at room temperature when stored in pressurised cylinders:

217

1. Helium 2. Nitrous oxide 3. Entonox 4. Carbon dioxide A: 1,2,3 Correct B: 1,3 Correct C: 2,4 Correct D: 4 Correct E: All Correct Correct Answer: C The critical temperature of a gas is the temperature above which the gas cannot be liquefied by pressure alone. In the case of both carbon dioxide and nitrous oxide this temperature exceeds room temperature at 31 and 36.5 degrees C respectively. Therefore when cylinder pressure is higher than critical pressure, these compounds exists in the cylinder as a mixture of gas and liquid. Under these circumstances the cylinder pressure gauge will not give a true indication of cylinder content. Number: 511 With regard to the pin index system of medical gas cylinders: 1. There are seven possible pin positions. 2. Air and oxygen share a common pin position. 3. The positions for oxygen are two and five. 4. Keying is permitted using either one or two pins. A: 1,2,3 Correct B: 1,3 Correct C: 2,4 Correct D: 4 Correct E: All Correct Correct Answer: A The Pin Index Safety System has two (5-mm) stainless steel pins on the cylinder yoke connector just below the fitting for the valve outlet port. There are seven different pin positions, depending on the type of gas in the cylinder. The yoke connector for an oxygen cylinder, for example, has pins at positions 2 and 5 whereas that for air is 1 and 5. Two pins must ALWAYS be present on the yoke.

218

Number: 519 Gastric emptying is enhanced by: A. The presence of fat in the duodenum. B. The presence of acid in the duodenum. C. Stress. D. An increase in vagal tone. E. The administration of atropine. Select the single best answer Correct Answer: D The presence of fat, acid or hypertonic solutions in the duodenum initiates a neurallymediated inhibitory enterogastric reflex, thus delaying the rate of gastric emptying. Gastric emptying is also under control of the autonomic nervous system such that a relative increase in vagal tone hastens gastric emptying while an increase in sympathetic tone will decrease it. Thus, anti-muscarinic agents delay gastric emptying. Number: 539 The value of the bispectral index which is thought to be most appropriate for the performance of minor, body-surface surgery, in a patient anaesthetised with propofol is: A. > 90. B. > 80 <= 90. C. > 70 <= 80. D. > 60 <= 70. E. <= 60. Select the single best answer Correct Answer: E During propofol-induced sedation, BIS values may be maintained above 75 to prevent airway obstruction and hypoxia. During propofol intravenous anaesthesia, BIS values from 40 to 60 have been proposed to maintain the desired level of hypnosis, with values below 50 associated with an insignificant probability of recall. Singh H : Eur J Anaesthesiol 1999 Jan;16(1):31-6 Bispectral index (BIS) monitoring during propofol-induced sedation and anaesthesia. Gajraj RJ, Doi M, Mantzaridis H, Kenny GN : Br J Anaesth 1998 Jan;80(1):46-52 Analysis of the EEG bispectrum, auditory evoked potentials and the EEG power spectrum during repeated transitions from consciousness to unconsciousness.

219

Number: 557 In the case of a 'typical' brachial plexus injury complicating cardiac surgery via a median sternotomy: 1. Sensory features are usually present. 2. Motor features are usually present. 3. The lower roots are usually affected. 4. The long-term prognosis is poor. A: 1,2,3 Correct B: 1,3 Correct C: 2,4 Correct D: 4 Correct E: All Correct

Correct Answer: B Brachial plexus injury occurring after cardiac surgery is a distinct entity from that which complicates other forms of surgery. The former is characterised by a predominance of sensory features confined to the lower roots of the plexus and the latter by a predominance of motor signs in the upper and middle roots. The former is far more frequent than the latter, with an excellent prognosis for recovery. The prognosis of brachial plexopathy after non-cardiac surgery may be worse in males, diabetics, those with injury to all roots of the plexus and when, in addition to the motor deficit, there is sensory loss and pain or dysaesthesia. Prognosis of intraoperative brachial plexus injury: a review of 22 cases. Br. J. Anaesth. 1997; 79:440-44: Ben-David, B.; Stahl, S Number: 567 Which of the following routine pre-operative laboratory tests have been shown to have a significant impact on the anaesthetic management of healthy patients scheduled for elective outpatient surgery? A. Serum electrolytes. B. Full blood count. C. Urinalysis. D. Coagulation studies. Select the single best answer Correct Answer: E

220

JAMA 1985 Jun 28;253(24):3576-81 The usefulness of preoperative laboratory screening. Kaplan EB, Sheiner LB, Boeckmann AJ, Roizen MF, Beal SL, Cohen SN, Nicoll CD We assessed the usefulness of routine laboratory screening of preoperative patients. Computer-readable laboratory, demographic, and discharge diagnostic data were assembled for 2,000 patients undergoing elective surgery over a four-month period, and randomly selected samples of patients were studied. Several tests ordered by protocol and performed by the laboratory at the time of admission were examined in these samples, including complete blood cell count, differential cell count, prothrombin time, partial thromboplastin time, platelet count, six-factor automated multiple analysis, and glucose level. Sixty percent of these routinely ordered tests would not have been performed if testing had only been done for recognizable indications, and only 0.22% of these revealed abnormalities that might influence perioperative management. Chart review indicated that these few abnormalities were not acted on nor did they have adverse surgical or anesthetic consequences. In the absence of specific indications, routine preoperative laboratory tests contribute little to patient care and could reasonably be eliminated Number: 568 Which statement is correct with regard to dantrolene sodium? A. It is a neuromuscular blocking agent. B. It causes phlebitis if administered into a peripheral vein.. C. It has an elimination half-life of 2-3 hours. D. It causes significant myocardial depression. E. It significantly enhances the effects of other neuromuscular blocking agents. Select the single best answer Correct Answer: B Unlike neuromuscular blocking agents, dantrolene acts within the muscle cell itself by reducing the level of intracellular ionised calcium. 'Minor' complications of dantrolene administration include nausea, phlebitis and transient (~24 hrs) muscle weakness. The half-life of intravenous dantrolene is approximately 12 hours. In clinical doses, this decrease in intracellular calcium has been shown to have little effect on myocardial contractility. The dosage of a neuromuscular blocking agent need not be changed significantly after dantrolene administration

221

Number: 578 With regard to ondansetron: 1. It increases gastric motility. 2. It has anti-dopaminergic activity. 3. It is relatively ineffective in the treatment of chemotherapy-induced nausea. 4. It is a 5-HT 3 receptor antagonist. A: 1,2,3 Correct B: 1,3 Correct C: 2,4 Correct D: 4 Correct E: All Correct Correct Answer: D In normal volunteers, single I.V. doses of 0.15 mg/kg of ondansetron had no effect on esophageal motility, gastric motility, lower oesophageal sphincter pressure, or small intestinal transit time. Ondansetron is a selective 5-HT 3 receptor antagonist. While ondansetron's mechanism of action has not been fully characterised, it is not a dopamine-receptor antagonist. Serotonin receptors of the 5-HT 3 type are present both peripherally on vagal nerve terminals and centrally in the chemoreceptor trigger zone of the area postrema. Ondansteron is extremely effective in the treatment of chemotherapy-induced emesis. It is not certain whether ondansetron's antiemetic action in chemotherapy-induced emesis is mediated centrally, peripherally, or in both sites. However, cytotoxic chemotherapy appears to be associated with release of serotonin from the enterochromaffin cells of the small intestine. In humans, urinary 5-HIAA (5-hydroxyindoleacetic acid) excretion increases after cisplatin administration in parallel with the onset of emesis. The released serotonin may stimulate the vagal afferents through the 5-HT 3 receptors and initiate the vomiting reflex. Number: 582 Drugs which are known to be triggering agents for malignant hyperthermia (MH) include: A. Haloperidol B. Ketamine C. Etomidate D. Atropine E. None of the above. Select the single best answer

222

Correct Answer: E The known triggering agents for MH comprise all the inhalational anaesthetic agents and suxamethonium. The drugs known to be safe include: atropine, benzodiazepines, bupivacaine, droperidol, lignocaine, nitrous oxide, non-depolarising muscle relaxants, opioids and the intravenous induction agents. Atropine can cause hyperpyrexia (particularly in children), but this is a feature of a central anticholinergic syndrome rather than malignant hyperthermia. Haloperidol can precipitate the neurolept malignant syndrome. In some of these patients the caffeine-contracture test may be positive for malignant hyperthermia susceptibility. This syndrome differs from malignant hyperthermia in that central dopamine depletion is responsible for the clinical picture. Number: 609 Which of the following therapies are probably effective in the treatment of post-operative nausea and vomiting (PONV)? 1. Ondansetron. 2. Acupuncture. 3. Droperidol. 4. Metoclopramide. A: 1,2,3 Correct B: 1,3 Correct C: 2,4 Correct D: 4 Correct E: All Correct Correct Answer: A With the exception of metoclopramide (which is emerging as a drug with little other than placebo effect in this regard), all have been found to be more or less effective therapies for PONV. Acupuncture (at the P6 acupuncture point which lies about four centimetres up the arm from the wrist creases) has been shown to reduce nausea and vomiting quite effectively in adults but not children. See: Lee A, Done ML. The use of non-pharmacologic techniques to prevent postoperative nausea and vomiting: a meta-analysis. Anesth Analg. 1999 Jun;88(6):1362-9.

223

Visit the 'Bandolier' website (www.jr2.ox.ac.uk/bandolier) and in particular pages www.jr2.ox.ac.uk/bandolier/band71/b71-9.html and www.jr2.ox.ac.uk/bandolier/band71/b718.html for comment on this topic. Number: 648 With regard to intravenous cannulation: 1. The use of local anaesthesia for cannulation diminishes pain on insertion even when a 20G cannula is used. 2. Antibiotic bonding of the cannula will reduce the rate of catheter infection. 3. Cannulation in the sitting position increases the likelihood of vasovagal symptoms. 4. Polyurethane cannulae are associated with a lower incidence of thrombophlebitis than teflon cannulae. A: 1,2,3 Correct B: 1,3 Correct C: 2,4 Correct D: 4 Correct E: All Correct Correct Answer: E Langham and Harrison have demonstrated that even when small cannulae (22G) are used, it is kinder to use local anaesthetic! See: Langham BT, Harrison DA Local anaesthetic: does it really reduce the pain of insertion of all sizes of venous cannula? Anaesthesia. 1992 Oct;47(10):890-1. As with central venous catheters, the incidence of catheter-related infection appears to be reduced when antibiotic bonded cannulae are used. See: Kamal GD, Pfaller MA, Rempe LE, Jebson PJ. Reduced intravascular catheter infection by antibiotic bonding. A prospective, randomized, controlled trial. JAMA. 1991 May 8;265(18):2364-8. The effect of posture on the likelihood of vasovagal symptoms has been explored by Rapp and Pavlin ( Rapp SE, Pavlin DJ, Nessly ML, Keyes H. Effect of patient position on the incidence of vasovagal response to venous cannulation. Arch Intern Med. 1993 Jul 26;153(14):1698-704.). Polyurethane cannulae are associated with a lower incidence of thrombophlebitis than teflon cannulae - see: Gaukroger PB, Roberts JG, Manners TA. Infusion thrombophlebitis: a prospective comparison of 645 Vialon and Teflon cannulae in anaesthetic and postoperative use. Anaesth Intensive Care. 1988 Aug;16(3):265-71. Number: 659 The incidence of anaesthesia-related dental trauma in a patient with 'good' dentition is about: A. 0.0001% B. 0.001% C. 0.01%

224

D. 0.1% E. 1.0% Select the single best answer Correct Answer: D This subject has recently been the subject of comprehensive review by Owen and WaddellSmith. Damage to teeth has long been associated with general anaesthesia and especially endotracheal intubation. Over sixty years ago, Magill recommended using a piece of adhesive plaster to protect the teeth when using the laryngoscope, and noted that with "blind (nasal) intubation" there was no risk of damage to teeth. Reviews of claims made to medical defence organisations in England, data from the Risk Management Foundations, closed claims analysis by the National Association of Insurance Examiners in the U.S.A and a review of anaesthesia-related claims in South Australian public hospitals between 1989 and 1998 confirm that dental injuries are the most common anaesthetic-related event reported, accounting for up to a third of incidents.Surveys of anaesthesia-related dental trauma show an incidence from 0.02% to 0.7%). The incidence of dental damage in patients with otherwise "good" dentition is approximately 0.08%. Macintosh described the curved blade most widely used today, observing that the straight blade occasionally "jeopardizes the patient's upper teeth". It was claimed that the open top of the curved blade decreased the risk of dental injury. The plethora of subsequent descriptions of devices and techniques to facilitate endotracheal intubation prompted Sykes to write "There is no living anaesthetist who holds the distinction of no having designed one or more! Despite the progress in intubation techniques, damage to teeth is still the commonest cause of complaint against anaesthetists. See: Anaesth Intensive Care 2000; 28: -133-145. Dental Trauma Associated with Anaesthesia. H. Owen, I. Waddell-Smith. Number: 660 The incidence of difficult intubation (> 2 attempts at laryngoscopy required) in a general surgical population (excluding obstetrics) is about: A. 0.05% B. 0.2% C. 1.0% D. 2.0% E. 5.0% Select the single best answer Correct Answer: D

225

Rose and Cohen determined the factors predictive of difficult intubation in a study of 18,205 patients scheduled for tracheal intubation under direct laryngoscopy. Tracheal intubation was difficult (> 2 laryngoscopies) in 1.8% and awkward (< or = 2 laryngoscopies) in 2.5%. This approach was a failure in 0.3%, and surgery was postponed in 0.05%. In this study an alternative approach to direct laryngoscopy, was the first choice in 353 patients. See: Rose DK, Cohen MM. The airway: problems and predictions in 18,500 patients. Can J Anaesth. 1994 May;41(5 Pt 1):372-83. Number: 682 Which of the following is most likely to occur within 5 minutes of the accidental insertion of an endotracheal tube into the right main bronchus of a ventilated patient: A. Hypotension. B. Severe hypercarbia. C. An increase in the requirement for the volatile agent. D. An increased inflation pressure. E. Collapse of the right upper lobe. Select the single best answer Correct Answer: D Hypotension, hypercarbia and increased requirements for volatile agents are not features of inadvertent bronchial intubation. RUL collapse usually takes longer than 5 minutes to occur. Number: 684 During the repair of idiopathic scoliosis in a 20-year-old, you decide to reduce blood loss by the combined use of isovolaemic haemodilution and induced hypotension with sodium nitroprusside. Your monitoring includes an arterial line and an oximetric pulmonary artery catheter. You are ventilating the patient with isoflurane in 30% oxygen. One hour into the procedure you note that the blood pressure is 85/60 mm Hg and the pulse rate is 115 bpm. You perform an arterial blood gas analyses which reveals, amongst other things: haemoglobin 61 G/L, PaO2 95 mm Hg, BXS -2.5. The mixed venous saturation is 59%. Given the scenario above, the most appropriate initial therapy is A. Institution of positive end-expiratory pressure at 10 crnH2O. B. Cessation of the sodium nitroprusside.

226

C. Administration of methylene blue for the treatment of possible cyanide toxicity. D. Retransfusion of some of the autologous blood. E. Ventilation with 100% oxygen. Select the single best answer Correct Answer: D Institution of positive end-expiratory pressure at 10 crnH2O is likely to reduce cardiac output and not be helpful at all. The sodium nitroprusside appears to be doing its job. The patient doesn't really have any signs of cyanide toxicity. The mixed venous oxygen saturation is unacceptably low. Given that it is not due to arterial hypoxaemia, the obvious option is to retransfusion of some of the autologous blood. Number: 689 A unit of Fresh Frozen Plasma (FFP): 1. Will tend to reverse the effect of warfarin. 2. Poses the same infection risk as a unit of packed cells. 3. Can prcipitate an anaphylactic reaction. 4. Contains the same concentration of factor VIII as a unit of cryoprecipitate A: 1,2,3 Correct B: 1,3 Correct C: 2,4 Correct D: 4 Correct E: All Correct

Correct Answer: B FFP is an acellular product and therefore does not carry the risk of transmission of CMV (which is an intracellular infection). Otherwise, the infection transmission risk is the same as for a unit of packed cells. FFP contains stable coagulation factors and plasma proteins - that is: fibrinogen, antithrombin, albumin, protein C and protein S. Patients who are IgA-deficient and require plasma support should receive FFP collected from IgA-deficient donors because the risk of anaphylaxis is increased if normal donors are used. Cryoprecipitate contains about half the factor VIII activity of FFP in one-tenth the original volume.

227

Indications for FFP transfusion include correction of coagulopathies, including the rapid reversal of coumadin; supplying deficient plasma proteins; and treatment of thrombotic thrombocytopenic purpura. Although FFP is an excellent volume expander, it should not be routinely used in this role. FFP is an acellular component and therefore does not transmit intracellular infections, i.e., cytomegalovirus (CMV). Patients who are IgA-deficient and require plasma support should receive FFP collected from IgA-deficient donors because the risk of anaphylaxis is increased with normal donors. Number: 690 A unit of whole blood which has been stored for 24 hours will have normal levels of factor(s): 1. V. 2. VII. 3. X. 4. VIII. A: 1,2,3 Correct B: 1,3 Correct C: 2,4 Correct D: 4 Correct E: All Correct Correct Answer: A O'Neill et al recently examined the effect of 24 hours of storage on the clotting factors. They found that there were no significant changes in factors V, VII, and X; fibrinogen; antithrombin III; protein C; and protein S over the 24-hour storage period. At 8 hours, factor VIII activity was reduced by 13 percent, and it was reduced by a further15 to 20 percent after the 24 hours. See: O'Neill EM et al Transfusion 1999 May;39(5):488-91 Effect of 24-hour whole-blood storage on plasma clotting factors Number: 704 Which organ is at greatest risk of ischaemia under conditions of normovolaemic haemodilution? A. Brain B. Lung C. Liver D. Heart E. Kidney Select the single best answer

228

Correct Answer: D The heart is the organ at greatest risk as, under basal conditions, it has the highest oxygen extraction ratio. Number: 710 Generally accepted techniques for reducing the incidence of suxamethonium-induced muscle pains include pre-treatment with: 1. Pancuronium 0.3 mg/kg. 2. Lignocaine 1.5 mg/kg. 3. Suxamethonium 0.05 mg/kg. 4. Midazolam 0.1 mg/kg. A: 1,2,3 Correct B: 1,3 Correct C: 2,4 Correct D: 4 Correct E: All Correct Correct Answer: A Pretreatment with various non-depolarising relaxants, lignocaine and suxamethonium itself have all been shown to be effective in reducing the incidence of suxamethonium-induced muscle pains although the efficacy of a 'self-taming' dose of suxamethonium is questionable. In contrast, induction agents (thiopentone, propofol) and benzodiazepines are all ineffective. Number: 716 A 32 year old ex-heroin addict with hepatitis C requires an urgent laparotomy for oversewing of a perforated duodenal ulcer. The patient is undergoing treatment with naltrexone. The results of pre-operative coagulation studies include an International Normalised Ration for the prothrombin time of 1.8 and a platelet count of 85,000/cubic mm. Given this scenario, which of the following post-operative analgesic techniques would be most appropriate? A. Morphine-based patient-controlled analgesia (PCA). B. Local anaesthetic-based epidural analgesia. C. Ketamine PCA. D. Intramuscular Methadone. E. Fentanyl-based epidural analgesia. Select the single best answer

229

Correct Answer: C There are surprisingly few reports addressing the problems of providing anaesthesia and postoperative analgesia for patients undergoing treatment with naltrexone. Given the options listed here, a ketamine based analgesic technique would seem most appropriate (notwithstanding the fact that some effects of ketamine are mediated via opiate mu and delta receptors). See, for example: Latasch L, Freye E. Opioid receptors-mediated respiratory effects and antinociception after S(+)-ketamine. Acta Anaesthesiol Belg. 1993;44(3):93-102. The use of opiates in patients receiving naltrexone is likely to be ineffective and is probably also inappropriate. An epidural technique is contra-indicated given the presence of a coagulopathy Number: 717 A 75 year old lady weighing about 65Kgs, requires internal fixation of a fractured neck of femur. At your preoperative visit you ascertain that she is in good health and that her only significant medical condition is hypertension for which she takes captopril. Her blood pressure at the time of this visit is 140/85. You decide to give her a general anaesthetic. The patient receives her usual captopril prior to surgery and is premedicated with temazepam 10 mgs orally. Anaesthesia is induced with propofol 80 mgs, midazolam 2.5 mgs and fentanyl 50 mcgs. Muscle relaxation is secured with vecuronium 6mgs, the trachea is intubated and the patient ventilated with sevoflurane in oxygen. 3 minutes after induction the blood pressure has fallen to 80/50 and the monitor shows sinus rhythm at a rate of 80 bpm. You treat this by the administration of 500 mls of Hartmanns solution together with 1 mg of metaraminol. The blood pressure rises to 100/50 while the heart rate remains unchanged. You decide to commence 500 mls of Normal Serum Albumen (NSA). Five minutes later the blood pressure has fallen to 55/30 with a heart rate of 110 bpm. There is no evidence of bronchospasm or of a rash. Given this scenario, the most likely cause of her hypotension is: A. Hypovolaemia. B. Anaphylaxis. C. Fat embolism. D. Acute coronary ischaemia. E. Administration of albumen to a patient on an ACE inhibitor. Select the single best answer Correct Answer: E

230

Anaphylaxis and the administration of albumen to a patient on an ACE inhibitor are the most likely diagnoses in this case. Given the absence of respiratory or cutaneous manifestations of anaphylaxis, the latter is more likely. It should be remembered that patients on ACE inhibitors may demonstrate an exaggerated hypotensive response to induction of anaesthesia - unlike, say, a patient on beta blockers. (Coriat P et al; Influence of chronic angiotensin-converting enzyme Inhibition on anesthetic Induction; Anesthesiology 81:299-307, 1994.) In this case, the felony has been compounded by the concurrent administration of NSA! Albumen solutions contain low levels of Hageman Factor - a bradykinin activator. Bradykinin is an autocoid which stimulates the release of nitric oxide thereby inducing vasodilatation. It is normally very rapidly hydrolysed by ACE and usually has minimal systemic effects. However, in the presence of ACE inhibition, profound systemic effects commonly occur. Exaggerated hypotension in patients on ACE inhibitors has also been reported in response to the adminstration of colloid solutions and a similar mechanism has been suggested. See, for example: Powell CG, Unsworth DJ and McVey FK; Severe hypotension associated with Angiotensin-Converting Enzyme inhibition in anaesthesia. Anaesth. Intens. Care 1998; 26:107-109. Fat embolism is very unusual with this type of fracture. Hypovolaemia and primary coronary ischaemia seem equally unlikely given the scenario. Number: 726 The absorption of glycine during trans-urethral prostatectomy: 1. Can be detected by expired gas analysis. 2. Is a likely cause of confusion. 3. Is a likely cause of prolonged hypertension. 4. Is a likely cause of transient blindness. A: 1,2,3 Correct B: 1,3 Correct C: 2,4 Correct D: 4 Correct E: All Correct Correct Answer: C Glycine itself is not detectable in expired alveolar gas. However, if a trace amount of ethanol is added to the solution, the absorption can be measured by expired-breath testing. See: Hahn RG, Olsson J. Ethanol monitoring of the transurethral resection syndrome. J Clin Anesth. 1996 Dec;8(8):652-5. Absorption of glycine is suggested by the occurrence of either an acute confusional state or transient blindness. Surprisingly, the degree of visual disturbance does not appear to correlate

231

with the plasma level of glycine. See: Mizutani AR, Parker J, Katz J, Schmidt J. Visual disturbances, serum glycine levels and transurethral resection of the prostate. J Urol. 1990 Sep;144(3):697-9. Transient hypertension may occur in the early stages of glycine absorption, but hypovolaemia and hypotension then supervene. See: Hahn RG. Fluid and electrolyte dynamics during development of the TURP syndrome. Br J Urol. 1990 Jul;66(1):79-84. Number: 730 With regard to postobstructive (negative pressure) pulmonary oedema: 1. The oedema fluid has a protein content consistent with a transudate. 2. The syndrome complicates about one in a thousand general anaesthetics. 3. Follow-up echocardiography is generally worthwhile. 4. Haemoptysis may be a prominent feature. A: 1,2,3 Correct B: 1,3 Correct C: 2,4 Correct D: 4 Correct E: All Correct Correct Answer: C At the time of writing (July 2000) over a 100 cases of postobstructive pulmonary oedema have been reported in the world literature. The terms "postobstructive pulmonary oedema", "negative pressure pulmonary oedema" and "laryngospasm induced pulmonary oederna" have been used. The common pattern in these cases is the occurrence of an episode of airway obstruction at emergence from general anaesthesia, followed by the rapid onset of respiratory distress, haemoptysis and bilateral radiological changes consistent with pulmonary oedema. Oedema fluid analysis has consistently shown a high protein content (oedema: plasma albumin ratio). This suggests that the fluid is an exudate and is evidence for disruption of capillary integrity rather than transudation occurring as a result of simple rearrangement of Starling forces. The largest reported series is by Deepika who reviewed 30 cases in one institution and arrived at an incidence of 0.094%. See: Deepika K, Kenaan CA, Barrocas AM, Fonseca JJ, Bikazi GB. Negative pressure pulmonary edema after acute upper airway obstruction. J Clin Anesth. 1997 Aug;9(5):403-8. McConkey has also recently written a good review. See: McConkey PP. Postobstructive pulmonary oedema--a case series and review. Anaesth Intensive Care. 2000 Feb;28(1):72-6. Follow-up echocardiography is generally normal and therefore not worthwhile. When laryngospasm occurs in the recovery period from general anaesthesia, postobstructive pulmonary oedema will complicate 5-10% of such cases.

232

Number: 735 The incidence of ipsilateral phrenic nerve paralysis in patients undergoing carotid endarterectomy under combined superficial and deep cervical plexus block is: A. Less than 5%. B. 5 - 49% C. 50 - 70% D. 71% - 95% E. More than 95%. Select the single best answer Correct Answer: C The incidence of ipsilateral phrenic nerve paralysis following combined superficial and deep cervical plexus block seems to be about 55 - 60%. - See for example: G. EMERY, G. HANDLEY, M. J. DAVIES:, P. H. MOONEY Anaesth Intensive Care 1998; 26: 377-381 Incidenceof Phrenic Nerve Block and Hypercapnia in Patients Undergoing Carotid Endarterectomy Under Cervical Plexus Block. The incidence is similar when supraclavicular brachial plexus blockade is instituted. Number: 737 Which of the following food allergies in a patient scheduled for surgery coronary artery grafting are of potential concern to the anaesthetist? 1. Avocado. 2. Fish. 3. Bananas 4. Egg. A: 1,2,3 Correct B: 1,3 Correct C: 2,4 Correct D: 4 Correct E: All Correct Correct Answer: E Allergy to avocado, bananas or chestnuts has a significant correlation with allergy to latex ("Latex-fruit syndrome"). The antigens responsible for this have not been fully identified, but are probably class I chitinases. The ASA website (see website link) deals with this problem extensively.

233

Protamine is manufactured from fish roe and for this reason, patients who are allergic to fish are at increased risk of the development of protamine hypersensitivity. There is also an increased risk of an adverse reaction to protamine in patients treated with neutral protamine Hagedorn (NPH) insulin. 'Diprivan' is formulated with egg-lecithin and should probably not be administered to those with a proven allergy to eggs. Number: 755 In the management of an anaesthetised patient with severe anaphylaxis: 1. Treatment of hypovolaemia is of major importance. 2. H2 receptor antagonists may have a role as adjunctive therapy. 3. Adrenaline is the drug of choice for the treatment of bronchospasm. 4. Aminophylline is strongly indicated if bronchospasm is refractory to treament with adrenaline A: 1,2,3 Correct B: 1,3 Correct C: 2,4 Correct D: 4 Correct E: All Correct Correct Answer: A Aggressive treatment of hypovolaemia and the administration of adrenaline are central to the successful management of anaphylaxis. The venodilatory response of histamine is mediated through both H1 and H2 receptor subtypes and for this reason, H2 receptor antagonists may have a role as adjunctive therapy. The role of aminophylline in the therapy of refractory bronchospasm is at best debatable. There is little evidence to support its use in anaphylaxis. If bronchospasm persists despite the use of large doses of adrenaline, it seems more reasonable to contemplate drug therapy with volatile agents, ketamine or magnesium and the use of other therapeutic techniques such as special modes of ventilation and even short term cardiopulmonary bypass. Number: 761 Which of the following drugs may precipitate myotonia in a patient with myotonic dystrophy? 1. Neostigmine. 2. Potassium chloride. 3. Suxamethonium 4. Vecuronium. A: 1,2,3 Correct B: 1,3 Correct

234

C: 2,4 Correct D: 4 Correct E: All Correct Correct Answer: A Neostigmine: Anticholinesterase drugs used to antagonise the effects of the nondepolarising neuromuscular blocking drugs may also precipitate myotonia, presumably because myotonic muscle has increased sensitivity to the stimulatory effects of acetylcholine. Potassium: The administration of potassium worsens clinical myotonia. Normal and myotonic muscle respond differently to increased serum concentrations of potassium. In normal muscle, an increase in serum potassium concentration increases muscular excitability and spontaneous discharges. Myotonic muscle shows a biphasic response. Initially, a decreased excitability can be demonstrated, but as serum potassium increases, increased sensitivity is seen. Durelli and colleagues have suggested that there is a differential effect of potassium on potassium channels on the muscle membrane surface and on the Ttubules of myotonic muscle. It would seem wise to avoid potassiumcontaining solutions. Suxamethonium: Although some authors have reported a normal response to suxamethonium, others report a generalised myotonic response resulting in difficulties in tracheal intubation and ventilation . The response appears to be dose dependent. Suxamethonium in the myotonic patient has a dual effect. It blocks neuromuscular transmission in the normal manner , but also acts directly on the muscle causing contraction . In addition, the increase in serum potassium concentration after administration of suxamethonium may further contribute to the development of myotonia. Several reports describing relaxation of myotonia after suxamethonium have appeared, but the mechanism is unclear. A typical generalised myotonic response to suxamethonium consists of the rapid development of jaw, abdominal and chest rigidity with arching of the cervical and lumbar spines. Ventilation and intubation may be difficult or impossible for 45 min. Furthermore, because the myotonia is caused by a primary defect of the muscle, nondepolarising neuromuscular blocking drugs do not abolish the generalised contractions. It is therefore recommended that suxamethonium is avoided in myotonic patients. Vecuronium: Nondepolarising neuromuscular blocking agents, when effective, appear to behave normally. Number: 790 The cost of a dose of a drug from societys perspective is equal to the purchase ('acquisition') cost of the drug plus the cost of treating the complications of the drug and the cost of treating any adverse outcomes. Dexter et al have recently analysed the 'true' cost of suxamethonium in this manner In the context of the statement above, the 'true' cost of a dose of suxamethonium, expressed as a proportion of acquisition cost is about: A. Equal to the acquisition cost. B. Ten times the acquisition cost. C. Twenty times the acquisition cost.

235

D. Two hundred times the acquisition cost. E. Five hundred times the acquisition cost. Select the single best answer Correct Answer: D In the report by Dexter et al, the 'true' or 'societal' cost of a drug was estimated using a pharmaco-economic measurement system which included the cost of treating complications and adverse outcomes. Suxamethonium was chosen as the drug for study, and the cost estimate included the cost of treating complications such as malignant hyperpyrexia, anaphylaxis, bradycardia causing cardiac arrest etc. Their estimate of true cost of a single dose of suxamethonium was that it was approximately two hundred times the acquisition cost (USD 0.19 compared to USD 37.00). There is no reason why the technique cannot be applied to any other drug (or therapeutic technique). See: Anesth Analg 2001 Mar;92(3):693-9: Cost identification analysis for succinylcholine. Dexter F, Gan TJ, Naguib M, Lubarsky DA. (Note that there is an arithmetic error in the published abstract of this paper). Number: 792 Since the earliest reports in 1982, there have been more than 24 reported cases of survival after failed attempts at cardio-pulmonary resuscitation. This phenomenon has been variously referred to as "spontaneous return of circulation" or "Lazarus syndrome". %The proportion of patients who survive this experience neurologically intact is about: A. 5%. B. 10%. C. 20%. D. 30% E. 50 Select the single best answer Correct Answer: D At the time of writing, (March 2001) there have been 26 case reports of Lazarus syndrome and 9 of these patients have survived to be neurologically intact. The mechanism of "spontaneous return of the circulation" is believed to be the removal of the effect of 'Auto PEEP' when the resuscitation attempt is abandoned.

236

The message is - If it happens to you it is probably worth re-commencing the resuscitation process! See: Ben-David B, Stonebraker VC, Hershman R, Frost CL, Williams HK. Survival after failed intraoperative resuscitation: a case of "Lazarus syndrome". Anesth Analg. 2001 Mar;92(3):690-2. Number: 809 The Chief Executive Officer of your local private hospital asks you for your advice on cost containment in the Post-Anaesthesia Care Unit (PACU). The strategy which is most likely to be effective is: A. The application of restrictions on the prescription of expensive drugs. B. The aggressive treatment of post-operative nausea and vomiting. C. Scheduling the distribution of admissions (ie.arranging the operating theatre schedule to optimise PACU admission patterns). D. The application of restrictions on the use of sophisticated monitoring equipment. Select the single best answer Correct Answer: C Cost containment and reduction have become major goals in health care. To decrease costs, hospital managers need to know the principal determinants of cost. However, these determinants are not always obvious, despite widespread beliefs. Dexter and Tinker recently analysed the costs of running a PACU and explored the effects of varying different cost inputs on overall running costs. Their principal findings are outlined below: Supplies and medications accounted for only 2% of PACU charges. Personnel costs, which depend on the peak number of patients in the PACU, accounted for almost all PACU costs. If nausea and vomiting could have been eliminated in each patient who suffered this complication, without causing sedation, the total time to discharge for all patients would have been decreased by less than 4.8% (95% confidence interval < 7.3%). Arrival rates to and times to discharge from the PACU followed triangular and log-normal distributions, respectively. Computer simulations, using published times to discharge for drugs with faster recovery, such as propofol, showed that the use of these drugs would only decrease PACU costs if operating rooms were consistently scheduled to run later each day. Such earlier discharge also might be beneficial if used at night, but only if the PACU could close after a single patient leaves. However, reasonably achievable decreases in the times to discharge for all patients undergoing general anesthesia are unlikely to substantively decrease PACU costs. In contrast, arranging an operating room schedule to optimize admission rates would greatly affect the number of PACU nurses needed. They concluded that "Anesthesiologists have little control over PACU economics via choice of anesthetic drugs. The major determinant of PACU costs is the distribution of admissions."

237

See: Anesthesiology Anesthesiology 82:94-101, 1995: Analysis of Strategies to Decrease Postanesthesia Care Unit Costs: Dexter F and Tinker JH. Number: 817 The most effective way to prevent pain on injection of propofol in an adult is to: A. Warm the drug to 37 centigrade before injection. B. Mix ~ 35 mg of lignocaine with the drug before injection. C. Cool the drug to 4 centigrade before injection. D. Inject ~ 35 mg of lignocaine as a 'Bier's Block' ~ 30-120 secs before injection of propofol. E. Inject 10 mg of metoclopramide through the same vein before injection of propofol. Select the single best answer Correct Answer: D A recent meta-analysis has been performed by Picard and Tramer. Their findings were that approximately 70% of all control patients reported some degree of pain or discomfort on injection with propofol alone. In some trials, all controls reported pain. The most effective analgesic method was IV lignocaine, given as a Biers block before the injection of propofol. Of 100 the patients treated with lignocaine 40 mg with a rubber tourniquet at the forearm for 30 to 120 s before the injection of propofol, approximately 60 (NNT 1.6) would not have any pain who would have had pain had they not received lignocaine. See: Picard P. and Tramr MR: Prevention of Pain on Injection with Propofol: A Quantitative Systematic Review Anesth Analg 2000 90: 963-969. Metoclopramide is also a surprisingly effective method for reducing pain on injection. This presumably reflects its structural similarity to the local anaesthetics. See: Pang WW, Mok MS, Chang DP, Huang MH. Local anesthetic effect of tramadol, metoclopramide, and lidocaine following intradermal injection. Reg Anesth Pain Med 1998;23:5803. Liaw WJ, Pang WW, Chang DP, Hwang MH. Pain on injection of propofol: the mitigating influence of metoclopramide using different techniques. Acta Anaesth Scand 1999;43:247. Number: 821 Which of the following drugs may be safely used in a patient with proven Acute Intermittent Porphyria? 1. Aminocaproic Acid. 2. Propofol. 3. Morphine. 4. Metformin.

238

A: 1,2,3 Correct B: 1,3 Correct C: 2,4 Correct D: 4 Correct E: All Correct Correct Answer: E Patients with the acute forms of porphyrias are at risk of developing life-threatening attacks of porphyria on exposure to certain commonly prescribed drugs. All the acute porphyrias are inherited as mendelian autosomal dominants, and each may be linked to lowered activity of one of the enzymes of the haem biosynthetic pathway: in Acute Intermittent Porphyria - a decrease in porphobilinogen deaminase; in Variegate Porphyria - a decrease in protoporphyrinogen oxidase; and in Hereditary Coproporphyria - a decrease in coproporphyrinogen oxidase. My suggestion is that you specifically confirm the safety (or otherwise) of every drug you wish to use in any patient with the disease. See the excellent review by James and Hift entitled 'Porphyrias' in Br J Anaesth 2000 Jul;85(1):143-53 A couple of web-sites which may help you are: The Canadian Porphyria Foundation at http://www.cpf-inc.ca and The University of Queensland Porphyria Research Unit at http://www.uq.edu.au/porphyria/ Number: 834 With regard to Xenon: 1. It has a MAC of about 70%. 2. It is a potent analgesic. 3. It is an odourless gas. 4. It has a blood/gas partition coefficient similar to nitrous oxide. A: 1,2,3 Correct B: 1,3 Correct C: 2,4 Correct D: 4 Correct E: All Correct Correct Answer: A See: Dingley J, Ivanova-Stoilova TM, Grundler S, Wall T. Xenon: recent developments. Anaesthesia. 1999 Apr;54(4):335-46. Xenon is a colourless, odourless, tasteless monatomic gas. It has an atomic number of 54 and a molecular weight of 131.3. It has nine stable isotopes and many artificial isotopes. It freezes at -111.9 C and boils at -107.1 C. Xenon is four times as dense as air and 3.4 times as dense

239

as N2O. It is nonflammable and does not support combustion. Its oil/water solubility coefficient is 20.0 and it has the highest coefficient of any of the noble gases, being the only one with anaesthetic properties at atmospheric pressure. It has an extremely low blood/gas partition coefficient of 0.14, even compared to nitrous oxide (0.47) or sevoflurane (0.65). Xenon diffuses freely through rubber and there can be significant losses of gas by this route during anaesthesia. It has a MAC of about 70% and is a good analgesic. Number: 842 The force required during the application of cricoid pressure is in the range: A. < 20 Newtons. B. 20 - 45 Newtons. C. 45 - 60 Newtons. D. 60 - 85 Newtons. E. > 85 Newtons. Select the single best answer Correct Answer: B The force required during the application of cricoid pressure appears to be in the range of 20 45 Newtons. See: 1. Wraight WJ, Chamney AR, Howells TH. The determination of an effective cricoid pressure. Anaesthesia. 1983 May;38(5):461-6. 2. Skinner HJ, Bedforth NM, Girling KJ, Mahajan RP. Effect of cricoid pressure on gastrooesophageal reflux in awake subjects. Anaesthesia. 1999 Aug;54(8):798-800. 3. Vanner RG, Pryle BJ. Regurgitation and oesophageal rupture with cricoid pressure: a cadaver study. Anaesthesia. 1992 Sep;47(9):732-5. Number: 843 A force of 30 Newtons applied to the cricoid region is believed to be effective in preventing regurgitation when intra-oesophageal pressure rises to: A. 20 mm Hg. B. 40 mm Hg. C. 60 mm Hg. D. 80 mm Hg. E. 100 mm Hg. Select the single best answer Correct Answer: B

240

See: 1. Wraight WJ, Chamney AR, Howells TH. The determination of an effective cricoid pressure. Anaesthesia. 1983 May;38(5):461-6. 2. Skinner HJ, Bedforth NM, Girling KJ, Mahajan RP. Effect of cricoid pressure on gastrooesophageal reflux in awake subjects. Anaesthesia. 1999 Aug;54(8):798-800. 3. Vanner RG, Pryle BJ. Regurgitation and oesophageal rupture with cricoid pressure: a cadaver study. Anaesthesia. 1992 Sep;47(9):732-5. The latter authors reported that "The efficacy of cricoid pressure was studied in 10 adult cadavers. The oesophageal pressure that would result in regurgitation during measured values of cricoid pressure was determined. Oesophageal pressure, recorded by a 2 mm diameter oesophageal tube, was increased by oesophageal distension with saline, and incremental levels of cricoid force, 20, 30 and 40 Newtons, were applied with a cricoid yoke. With each 10 Newton increment of cricoid force there was a significant rise in the oesophageal pressure required to provoke regurgitation (p < 0.01). Thirty Newtons of cricoid force prevented regurgitation of saline in all cadavers with oesophageal pressures of up to 40 mmHg. Rupture of the oesophagus occurred in three cadavers: one at 30 and two at 40 Newtons of cricoid force, but there was no rupture at 20 Newtons of cricoid force. In the other seven cadavers oesophageal pressures were also studied with a 4.6 mm diameter (14 FG) oesophageal tube, which did not reduce the efficacy of cricoid pressure in preventing regurgitation." See also the report by Tournadre et al on the effect of cricoid force on lower oesophageal sphincter tone. Tournadre JP, Chassard D, Berrada KR, Bouletreau P. Cricoid cartilage pressure decreases lower esophageal sphincter tone. Anesthesiology 1997; 86: 79. Note the trade offs, - low range cricoid force is less effective in preventing regurgitation and has a disproportionately large effect in reducing lower oesophageal sphincter tone, whereas higher forces are more effective, but appear to increase the risk of oesophageal rupture. Number: 845 The incidence of pulmonary aspiration in the perioperative period in adults who have undergone general anaesthesia is approximately: A. 1 in a 100. B. 3 in a 1000. C. 1 in a 1000. D. 3 in 10000. E. 1 in 10000. Select the single best answer See: Ng A, Smith G: Anesth Analg 2001;93:494-513. Gastroesophageal Reflux and Aspiration of Gastric Contents in Anesthetic Practice. According to the authors of this excellent review: "Examination of the literature suggests that the incidence of pulmonary aspiration in the perioperative period is relatively infrequent and that there has been little change in the last few years. In 1986, a study of Scandinavian Teaching Hospitals suggested that the incidence of aspiration varied between 0.7 and 4.7 per

241

10,000 general anesthetics. A report published one decade later suggested that the incidence was 2.9 per 10,000 at a Norwegian hospital. Studies from the Mayo Clinic indicated that the incidence of aspiration is similar in adults (3.1 per 10,000) and children (3.8 per 10,000) although another study from the United States suggested that the incidence of aspiration in children was more frequent (10.2 per 10,000)." Number: 847 A 70 year old man is scheduled for elective repair of an inguinal hernia under general anaesthesia. Which of the following screening tests performed as part of the pre-operative assessment are most likely to be predictive of an adverse perioperative outcome? A. Haemoglobin < 10 g/ dL. B. Creatinine > 1.5 mg/dL (~0.13 mmol/L). C. Sodium > 147 mmol/L. D. Potassium < 3.5 mmol/L. E. Glucose > 200 mg/dL (~11 mmol/L). Select the single best answer Correct Answer: C All of these variables are associated with a higher than expected incidence of adverse perioperative outcomes - with a serum sodium of more than 147 mmol/L being the most powerful predictor. However, none of them are particularly useful or cost-effective investigations if used routinely. Refer to the important paper by Dzankic et al and the editorial by Fleisher in the same issue of Anesthesia & Analgesia. Both these groups of authors agree that "routine preoperative laboratory testing for hemoglobin, creatinine, glucose, and electrolytes based on age alone may not be indicated in geriatric patients." See: Dzankic S, Pastor D, Gonzalez C, Leung JM. The prevalence and predictive value of abnormal preoperative laboratory tests in elderly surgical patients. Anesth Analg. 2001 Aug;93(2):301-8. Fleisher LA. Routine laboratory testing in the elderly: is it indicated? Anesth Analg. 2001 Aug;93(2):249-50 Number: 880 Professional document P24 of the Australia and New Zealand College of Anaesthetists outlines the standards required for the safe conduct of sedation for endoscopy. Which of the following drugs is NOT required to be immediately available in an area where patients undergo such sedation?

242

A. Dextrose 50% B. Lignocaine C. Flumazenil D. Naloxone E. Calcium chloride Select the single best answer Correct Answer: E The document requires that immediately available emergency drugs should include at least: adrenaline, atropine, dextrose 50%, lignocaine, flumazenil and naloxone. Number: 881 Which of the following drugs is contraindicated in a patient with known malignant hyperpyrexia (MH)? A. Droperidol. B. Propofol. C. Bupivacaine. D. Ketamine. E. Chlorpromazine. Select the single best answer Correct Answer: E Droperidol, propofol, bupivacaine and ketamine can all be safely used in patients with MH. In a recent review, Wappler commented that "Further evaluation is required about the potency of certain psychotropic substances in triggering symptoms of MH, e.g. phenothiazines, monoamine oxidase inhibitors and tricyclic antidepressants. These substances should not be given to MHS individuals, as malignant neuroleptic syndrome might be induced, which presents with similar symptoms to MH (fever, muscle rigidity and tachycardia). This is caused by a deregulation of central dopamine receptors as opposed to MH where peripheral metabolism is deranged. However, there have been no reports of MH associated with usage of these drugs outside anaesthetic practice. Furthermore, no case of MH-like symptoms after phenothiazines, monoamine oxidase inhibitors or tricyclic antidepressants has been verified by an in vitro contracture test on the patient concerned." See: Wappler F. Malignant hyperthermia. Eur J Anaesthesiol. 2001 Oct;18(10):632-52 and Hopkins PM. Malignant hyperthermia: advances in clinical management and diagnosis. Br J Anaesth. 2000 Jul;85(1):118-28. Number: 882

243

An In Vitro Contracture Test (IVCT) on a muscle specimen obtained from a patient with suspected malignant hyperpyrexia (MH) is being conducted. Preparation and processing for IVCT are standardised according to the protocol of the European MH Group. Which of the following agents should the muscle be exposed to? 1. Caffeine. 2. Suxamethonium. 3. Halothane. 4. Dibucaine A: 1,2,3 Correct B: 1,3 Correct C: 2,4 Correct D: 4 Correct E: All Correct Correct Answer: B "The muscle sample is split into separate bundles and spread out between two electrodes. It is then stimulated using a supramaximal current and the resulting muscle twitches measured. Only viable muscle preparations with a twitch response greater than 10 mN are used for IVCT. After equilibration, halothane is introduced into the system using a vapouriser, in increasing concentrations. The same procedure is carried out on a second muscle bundle but using cumulative concentrations of caffeine instead of halothane. Both tests are repeated with fresh muscle samples." (Wappler F. Malignant hyperthermia. Eur J Anaesthesiol. 2001 Oct;18(10):632-52). See also: Hopkins PM. Malignant hyperthermia: advances in clinical management and diagnosis. Br J Anaesth. 2000 Jul;85(1):118-28. Number: 883 Which of the following agents is a known trigger for Malignant Hyperthermia (MH) in a susceptible patient? A. Droperidol. B. Sevoflurane. C. Ketamine. D. Pancuronium. E. Ondansetron Select the single best answer Correct Answer: B

244

Although case reports of MH associated with the use of sevoflurane are few, there is little doubt that all volatile anaesthetics (including halothane, enflurane, isoflurane, sevoflurane and desflurane) can trigger MH. Number: 884 The incidence of Malignant Hyperthermia Susceptibility (MHS) in an unselected population of children and adolescents is in the order of: A. 1:1000. B. 1:5000. C. 1:15000. D. 1:50000. E. 1:100000. Select the single best answer Correct Answer: C The incidence of MHS is estimated to be 1:15 000 for children and adolescents and 1:50 000 150 000 for adults in North America and Europe. See: Gronert GA, Antognini JF, Pessah IN. Malignant Hyperthermia. In: Miller RD, eds. Anesthesia, 5th edn. New York: Churchill Livingstone, 2000: 10331052 Number: 885 Which of the following agents is LEAST likely to precipitate contracture in a muscle specimen obtained from a patient susceptible to Malignant Hyperthermia (MH)? A. Caffeine. B. Isoflurane. C. 4-chloro-m-cresol. D. Ryanodine. Select the single best answer Correct Answer: E Caffeine (and halothane) are used as triggering agents in the In Vitro Contracture Test protocol of the European MH Group. Isoflurane is the second most potent of the volatile agents at inducing contracture and there are numerous case reports of MH associated with the use of this drug. 4-chloro-m-cresol and ryanodine are emerging as important trigger agents in patients with equivocal responses to caffeine or halothane. Haloperidol may augment the response to trigger agents but is still probably safe for use in patients with MH.

245

Number: 887 A 50 year old woman, is scheduled for an emergency ovarian cystectomy. She reports that she regularly takes the herbal medicine Ginseng (Panax Ginseng) but is otherwise well. The patient is at increased risk of: 1. Intra-operative cardiac arrhythmias. 2. Intra-operative hypoglycaemia. 3. Prolongation of the effects of non-depolarising muscle relaxants. 4. Peri-operative bleeding. A: 1,2,3 Correct B: 1,3 Correct C: 2,4 Correct D: 4 Correct E: All Correct Correct Answer: C See the systematic review by: Ang-Lee MK, Moss J, Yuan CS. Herbal medicines and perioperative care. JAMA. 2001 Jul 11;286(2):208-16. Ginseng usage is associated with an increased incidence of hypoglycaemia and peri-operative bleeding. The use of garlic, ginger and ginko has also been associated with an increase in peri-operative bleeding. (It seems that herbs that begin with 'G' are anti-coagulant!) Number: 889 The incidence of persistent ulnar neuropathy complicating general anaesthesia in patients undergoing non-cardiac surgery is about: A. 1:100. B. 1:250. C. 1:2500. D. 1:5000. E. 1:10,000. Select the single best answer Correct Answer: C The definitive study is that by Warner et al who reviewed this complication in a study of 1,129,692 procedures undertaken at the Mayo clinic and found an incidence of 1:2729 cases.

246

However, it should be noted that the incidence of transient neuropathy (duration less than 3 months) is considerably higher than this - probably in the order of 1:250. Warner MA, Warner ME, Martin JT. Ulnar neuropathy. Incidence, outcome, and risk factors in sedated or anesthetized patients. Anesthesiology. 1994 Dec;81(6):1332-40. Warner MA, Warner DO, Matsumoto JY, Harper CM, Schroeder DR, Maxson PM. Ulnar neuropathy in surgical patients. Anesthesiology. 1999 Jan;90(1):54-9. Number: 890 A patient sustains a mixed motor and sensory ulnar neuropathy following the repair of an abdominal aortic aneurysm under general anaesthesia. Three months post-operatively the patient still has signs of the neuropathy. The likelihood of a complete recovery from this injury is about: A. 10% B. 20% C. 30% D. 40% E. 50% Select the single best answer Correct Answer: E Warner et al found that in a group of the 382 patients who sustained such an injury, "53% had regained complete motor function and sensation and were asymptomatic. An additional six percent had regained complete motor function and sensation but still had pain as a symptom. The remaining 41% of the patients had persistent deficits with or without pain at 1 yr. In general, pain was perceived in the ulnar nerve distribution; however, 23% of patients with persistent pain for more than 1 yr described intermittent episodes of generalized forearm and hand discomfort or aching. Early identification of ulnar neuropathy symptoms in these patients did not appear to result in a greater chance of improvement for these symptoms." See: Warner MA, Warner ME, Martin JT. Ulnar neuropathy. Incidence, outcome, and risk factors in sedated or anesthetized patients. Anesthesiology. 1994 Dec;81(6):1332-40. Number: 914 With regard to the relationship between hypothermia and the Minimum Alveolar Anaesthetic Concentration (MAC) of the volatile agents it is believed that MAC: A. Increases by ~5% per degrees centigrade decrease. B. Does not vary with body temperature. C. Decreases by ~5% per degrees centigrade decrease. D. Decreases by ~10% per degrees centigrade decrease. E. Decreases unpredictably with body temperature Select the single best answer

247

Correct Answer: C Surprsingly little is known about this subject - with most of the work having been undertaken in animals. According to Eger: "In animals, and presumably humans, MAC for potent inhaled anesthetics decreases with decreasing body temperature by approximately 4% to 5% per degrees centigrade decrease, doing so in a rectilinear manner, with complete elimination of the requirement for anesthesia at 20C. However, this relationship does not extend to N2O; MAC for N2O does not change materially with decreasing temperature. " See: Eger EI 2nd. Age, minimum alveolar anesthetic concentration, and minimum alveolar anesthetic concentration-awake. Anesth Analg. 2001 Oct;93(4):947-53. Number: 919 Recognised complications of the use of 6% Hydroxy Ethyl Starch (Hetastarch) include: 1. Suppression of the pituitary-adrenal axis. 2. Pruritus. 3. Hypercalcaemia 4. Increased perioperative bleeding. A: 1,2,3 Correct B: 1,3 Correct C: 2,4 Correct D: 4 Correct E: All Correct Correct Answer: C There is no suggestion that hetastarch interferes with the integrity of the pituitary-adrenal axis. Delayed pruritus is well recognised. - Kimme et al found that: "After Hetastarch treatment, we observed pruritus in 54% of the patients." and also noted that in other investigations the incidence has been found to vary between 20% and 64%. 6% hetastarch is calcium-free and, if anything, has a tendency to cause hypocalcaemia. Hetastarch inhibits platelet function by reducing the availability of the functional receptor for fibrinogen on the platelet surface. It may also impair coagulation by prolonging the partial thromboplastin times and decreasing factor VIII activity and fibrinogen levels. See: Kimme P, Jannsen B, Ledin T, Gupta A, Vegfors M. High incidence of pruritus after large doses of hydroxyethyl starch (HES) infusions. Acta Anaesthesiol Scand. 2001 Jul;45(6):686-9.

248

Wilkes MM, Navickis RJ, Sibbald WJ. Ann Thorac Surg 2001 Aug;72(2):527-33; Albumin versus hydroxyethyl starch in cardiopulmonary bypass surgery: a meta-analysis of postoperative bleeding Number: 930 The most common major complication of blood transfusion in the US and western Europe is: A. Transmission of Hepatitis A. B. Transmission of Hepatitis B. C. Transmission of Hepatitis C. D. Transmission of Human Immunodeficiency Virus (HIV). E. ABO incompatibility as a result of transfusion error. Select the single best answer Correct Answer: E See the case report and review by Krombach et al. According to these authors, "Human error has long been identified as a major source of ABO incompatibility-related transfusion fatalities. However, reports on this issue in the anesthesiology literature are sparse, and many anesthesiologists may be unaware that they are key players, as at least 50% of all blood units, by estimate, are transfused by anesthesiologists. In the United States, the frequency of avoidable transfusion fatalities attributable to misidentification of the pretransfusion blood sample, the blood unit, or the recipient has been reported to range from 1 per 600,000 to 1 per 800,000 transfusions. The estimated incidence of nonfatal transfusion errors in the United States ranges from 1 in 12,000 to 1 in 19,000 transfusions. The United Kingdoms Serious Hazards of Transfusion (SHOT) group, which tracks adverse events on a voluntary basis and currently covers more than 90% of all red cell usage in the United Kingdom, recently reported an error incidence of 1 in 16,000 (335 errors per 5.5 million units of red blood cells transfused between 1996 and 1999). Transfusion errors accounted for 54% of all transfusion-related complications, and ABO incompatibility as a possible life-threatening condition was reported 97 times, leading to 4 deaths (1 per 1,400,000 transfusions) and 29 cases of immediate major morbidity (1 per 200,000 transfusions). By contrast, the SHOT initiative recorded only 19 cases of confirmed transfusion-transmitted infection (1 per 300,000 transfusions), including four cases of hepatitis B, three cases of hepatitis C, and one case of HIV. Eleven cases (one hepatitis A infection, one malaria, and nine bacteremias) were attributable to infections for which no testing of donations is performed. In the United States, the aggregated risk of transmission of HIV, human T-cell lymphotropic virus, hepatitis C virus, and hepatitis B virus is 1 per 34,000 blood units transfused, or less than half the risk of transfusion error (1 in 12,000 to 1 in 19,000 transfusions). On the basis of current estimates of the risks of transfusion-transmitted infections and transfusion errors, an anesthesiologist of a major general hospital or trauma center who transfuses an average 500 U of packed red cells per year can be estimated to transmit HIV infection once in 1000 years, hepatitis C once in 200 years, hepatitis B once in 120 years, and

249

to administer blood to the wrong recipient once in 30 years, or once within his professional lifetime. In a university hospital with an annual volume of 20,000 blood transfusions or more, transfusion error is likely to occur once every year. " References: Krombach J, Kampe S, Gathof BS, Diefenbach C, Kasper SM. Human error: the persisting risk of blood transfusion: a report of five cases. Anesth Analg. 2002 Jan;94(1):154-6. Williamson L, Cohen H, Love E, Jones H, Todd A, Soldan K. The Serious Hazards of Transfusion (SHOT) initiative: the UK approach to haemovigilance. Vox Sang 2000;78 Suppl 2:291-5 Number: 933 When used as an anti-emetic for prophylaxis of Post-Operative Nausea and Vomiting (PONV), a single dose of dexamethasone: 1. Has no clinically relevant side-effects if administered to an otherwise healthy patient. 2. Exerts its effects for at least 12 hours. 3. Is more effective if administered with a 5HT3 antagonist. 4. Should be given to an adult in a dose of 0.5 mg/kg. A: 1,2,3 Correct B: 1,3 Correct C: 2,4 Correct D: 4 Correct E: All Correct Correct Answer: A Readers are referred to the excellent systematic review by Henzi et al. The drug appears to have no clinically relevant side-effects when used in otherwise healthy patients. It is more effective if used in combination with a 5HT3 antagonist and exerts its effects for about 24 hours. The appropriate dose in an adult seems to be either 8 or possibly 16 mgs. In their review, Henzi et al concluded that "In the surgical setting, a single prophylactic dose of dexamethasone is antiemetic compared with placebo without evidence of any clinically relevant toxicity in otherwise healthy patients. Late (i.e., up to 24 hours) efficacy seems to be most pronounced. It is likely that the best prophylaxis of PONV currently available is achieved by combining dexamethasone with a 5-HT3 receptor antagonist." References: Henzi I, Walder B, Tramer MR. Dexamethasone for the prevention of postoperative nausea and vomiting: a quantitative systematic review. Anesth Analg. 2000 Jan;90(1):186-94.

250

Number: 936 Which of the following drugs are believed to be effective in the treatment of post-operative shivering? 1. Clonidine. 2. Doxapram. 3. Pethidine. 4. Ketanserin. A: 1,2,3 Correct B: 1,3 Correct C: 2,4 Correct D: 4 Correct E: All Correct

Correct Answer: E All of these drugs are effective forms of treatment. The appropriate doses in an adult are: Clonidine 150 micrograms, doxapram 100 mg, pethidine 25 mg and ketanserin 10 mg. The topic of post-operative shivering has recently been subject to systematic review by Kranke et al. These authors noted that "Shivering, as nausea or vomiting, never becomes chronic and it is unlikely to kill a patient. However, in shivering postoperative patients, left ventricular systolic work index and oxygen consumption index may be increased (54). It is, therefore, encouraging that some simple and inexpensive interventions are effective in the treatment of this adverse effect of anesthesia and surgery. Two shivering patients need to be treated with meperidine 25 mg, clonidine 150 g, or doxapram 100 mg for one to stop shivering within five minutes who would have continued to shiver had they all received a placebo. This degree of efficacy relates to the "average" shivering adult patient in the postoperative period." See also the recent study Bhatnagar (who was investigating the effect of tramadol on shivering) - who observed that "Although the aetiology of postoperative shivering is inadequately understood, various risk factors have been suggested. These include hypothermia, stress, uncontrolled pain, uninhibited spinal reflexes and decreased sympathetic activity. Many drugs have been used to treat shivering, including opioids, doxapram, tramadol, ketanserin, clonidine, propofol, physostigmine and nefopam, with opioids being the most extensively evaluated. Amongst the opioids, pethidine has been found to be most efficacious. Evidence suggests that kappa-opioid receptors play an important role in the modulation of postoperative shivering. This explains the greater efficacy of pethidine compared with equi-analgesic doses of mu-receptor opioid agonists such as morphine, fentanyl, alfentanil and sufentanil." See: Kranke P, Eberhart LH, Roewer N, Tramer MR. Pharmacological Treatment of Postoperative Shivering: A Quantitative Systematic Review of Randomized Controlled Trials. Anesth Analg. 2002 Feb;94(2):453-460.

251

Bhatnagar S, Saxena A, Kannan TR, Punj J, Panigrahi M, Mishra S. Tramadol for postoperative shivering: a double-blind comparison with pethidine. Anaesth Intensive Care. 2001 Apr;29(2):149-54. Number: 940 Glucocorticoid cover may be required in patients with the following diseases: 1. Sarcoidosis. 2. Polyarteritis nodosa. 3. Systemic lupus erythematosus 4. Thrombocytopenic purpura. A: 1,2,3 Correct B: 1,3 Correct C: 2,4 Correct D: 4 Correct E: All Correct Correct Answer: E Short or long term therapy with steroids may be part of the management of all of these diseases. Glucocorticoid cover is required to replace the 'basal' level of cortisol if the patient's normal oral intake is disrupted, and to increase plasma cortisol levels to a level consistent with the expected 'stress' response to surgery. The magnitude of the response correlates to a degree with the magnitude of injury from trauma or the extent of surgery, inflammation etc. In general, after surgery, plasma cortisol increases 5-6x within 6 hours postoperatively, with levels falling by 24 hours unless the stress response continues. ROGERS, M.C ET AL (EDS); Principles and Practice of Anesthesiology, Mosby, 1993, pp 68-70, 1584-85. Jabbour SA. Steroids and the surgical patient. Med Clin North Am. 2001 Sep;85(5):1311-7. Udelsman R, et al. Adaptation during surgical stress. A reevaluation of the role of glucocorticoids. J Clin Invest. 1986 Apr;77(4):1377-81. Number: 947 Which of the following manoeuvres were included in Sellick's original description of cricoid pressure? 1. Extending the head 2. Applying one hand behind the neck to optimize traction of the oesophagus against the vertebral body of C5 3. Applying cricoid pressure with the thumb and the index finger 4. Applying pressure with the loss of the lash reflex.

252

A: 1,2,3 Correct B: 1,3 Correct C: 2,4 Correct D: 4 Correct E: All Correct Correct Answer: B In Sellick's original description, the stomach was emptied first with an oro-gastric tube and the patient was preoxygenated. The patient lies supine with slight head down, so that if active vomiting occurs, gravity will direct it away from the airway. The head was extended in the tonsillectomy position. This increases the anterior convexity of the cervical spine, stretches the oesophagus, and prevents lateral displacement when pressure is applied to the cricoid cartilage. Before induction, the cricoid is palpated to establish landmarks by the assistant. It is lightly held whilst the patient is awake. As anaesthesia begins, moderate pressure is exerted with the thumb and second finger. As soon as consciousness is lost, firm pressure is applied and maintained until the cuff of the endotracheal tube is inflated. Sellick demonstrated that this manoeuvre could prevent passive regurgitation of contrast material from the stomach up to a pressure of 100 cm H2O. References SELLICK, B.A;" Cricoid pressure to control regurgitation of stomach contents during induction of anaesthesia", The Lancet, August 19th, 1961, pp 404-6. Number: 962 A 50 year old man with chronic renal failure (CRF) presents for creation of an arterio-venous fistula. He is receiving no drugs which are known to interfere with coagulation. His coagulation profile is reported as follows: Bleeding time 18 mins (3 - 5 mins). Platelet count: 110 (150 - 350 K/ml). APTT: 30 secs (30 - 40 secs). PT: 15 secs (11 - 14 secs). In this context, the most appropriate management of his bleeding diathesis is to: A. Transfuse 6 units of platelets immediately before surgery. B. Administer 50 mg of conjugated estrogen 1 hour before surgery. C. Transfuse 2 units of cryoprecipitate immediately before surgery. D. Administer DDAVP 0.3 mcg/kg immediately before surgery. E. Proceed with surgery as planned. Select the single best answer Correct Answer: D

253

There seems little doubt that the primary disorder of coagulation in CRF relates to platelet dysfunction, although the exact mechanism remains obscure. In this man, this is manifest as marked prolongation of his bleeding time (BT). Both the administration of a conjugated estrogen (such as 'Premarin') and DDAVP have been shown to reduce the likelihood of bleeding in the uremic patient, but the former requires hours (if not days) to achieve its maximal effect. The administration of a blood product such as platelets (or cryoprecipitate) - with its attendant risks, is probably not justified in this situation. Similarly, to proceed with surgery in the face of such a marked prolongation of the BT would be foolhardy - given the low morbidity associated with the use of DDAVP. An online review of coagulation has been recently publsihed by Triplett. With regard to the bleeding time, Triplett remarks "The BT has also been widely utilized as a means of accessing primary hemostatic response (platelet-injured vessel wall interaction). Unfortunately, the BT is relatively insensitive and, in many cases, nonspecific with respect to identifying abnormalities of primary hemostasis . The major variables are the inherent differences between individuals performing the BT and the various BT devices. The introduction of BT devices designed to decrease the variability of the depth of the induced wound was a major advance over the traditional Ivy BT test. Despite the introduction of the newer devices, there remains substantial variability between individuals performing BTs as well as the possible complication of scar formation at the test site (typically, the anteriorlateral aspect of the arm). There are several variables in the BT in addition to the technical aspects of performing the test. BTs tend to be longer in females and decrease with aging. One cosmetic complication frequently seen in elderly patients who have experienced extensive sun exposure is the formation of a somewhat symmetrical subepidermal hemorrhage, which is attributable to blood dissecting into the subepidermis as opposed to exiting onto the surface of the skin at the site of the BT incision. The BT is also affected by the hematocrit and platelet mass. Patients with chronic renal disease and decreased hematocrit often have a prolonged BT. Increasing the hematocrit to >30% often will correct a prolonged BT in a patient with chronic renal disease Number: 967 Which of the following is true of penicillin allergy? A. It occurs in 20% of hospitalized patients after exposure. B. It occurs more commonly following intravenous than intramuscular administration. C. Cross reactivity with cephalosporins is high. D. Premedication with antihistamines and steroids reduces the severity of an anaphylactic reaction. Select the single best answer

254

Correct Answer: B 10-20% of hospitalized patients report an allergy to penicillin. Between 1-10% of all patients treated with penicillins develop reaction to them. IgE mediated reactions are more common after intravenous administration. Initial reports suggested a cross reactivity with cephalosporins. This is felt to be overestimated. There are no figures to describe the actual incidence which is felt to be small, however, true anaphylactic cross reactivity has occurred. Premedication and 'test' doses have not been shown to reduce the severity of severe reactions The majority of reactions involve rashes. All four types of hypersensitivity reactions described by Gell & Coombs have been seen with penicillin. In addition, some reactions have an obscure pathogenesis and have been labelled as idiopathic. Among these are the common maculopapular rash, Stevens-Johnson's syndrome, exfoliative dermatitis, and toxic epidermal necrolysis. Ampicillin induce rashes with a greater frequency than penicillin. Pseudoanaphylactic reactions have been observed after IMI injections of procaine penicillin and are most likely caused by a combination of toxic and embolic phenomena from procaine Anaphylactic reactions occur in 0.004-0.015% of treatment cases. Anaphylactic reactions are mediated by IgE antibodies and include urticaria, laryngospasm, bronchospasm, and cardiovascular collapse. The most effective means of prevention is by identifying those who are destined to have a reaction and withhold the drug from them. Factors which are associated with increased risk are: (1) Second or more exposure. Allergic reactions can occur on initial exposure, however, it is more common to find a patient who has tolerated the penicillin on initial exposure and developed a reaction on subsequent exposure. Presumably sensitization has already occurred when the patient reacts to the initial exposure. Non-therapeutic exposure to environmental and occupational sources may account for this; for example, in utero exposure or occult sources of penicillin in foods. Cross sensitization with other drugs is another source; for example, cephalosporin sensitivity following previous penicillin allergy. (2) Age.Young and middle aged adults are most likely to develop penicillin allergy. (3) Route of administration. Adverse reactions are more common after IV than IMI or Oral administration. (4) History of allergy. A patient with a history of an earlier reaction to penicillin has a 6 fold risk of experiencing a reaction on subsequent exposure. However, the most serious and fatal reactions occur in individuals with no history of previous reaction. Occult sensitization as described above may have occurred. A history of allergy in general appears to have no bearing on a patients likelihood to react to skin tests with penicillin reagents. (5) Skin test reactions. 10-20% of hospitalized patients will disclose a history of allergic reaction to penicillin. The most useful single piece of information in assessing an individuals risk of experiencing an immediate IgE mediated reaction is the skin test. Greatest sensitivity is found with tests including " minor " determinants (penicilloyl-polylysine reagent or minor determinant mixtures) as compared with those testing " major " determinants (RAST). Approximately 1% of patients with negative skin tests will develop severe reactions compared with 50-70% who will develop them after positive skin tests.

255

References SHEFFER, A.;" Anaphylaxis, continuing medical education ", The Journal Of Allergy and Clinical Immunology, Nov 1986, pp 1051-2. FISHER, M.;" The Prevention of Second Anaphylactoid Reactions To Anaesthetic Drugs ", Anaesth Intens Care, 9, pp 242-6, 1981. ROGERS, M.C ET AL (EDS); Principles and Practice of Anesthesiology, Mosby, 1993, pp 2467-70. Number: 1003 An otherwise fit, 60kg, 25 year old, asthmatic patient requires an urgent appendicectomy. The patient uses a salbutamol inhaler frequently, but is otherwise on no regular medication. Following a rapid-sequence induction with 300 mg of thiopentone and l00 mg of suxamethonium, the patient is noted to have a sharply upward sloping phase 3 on the capnograph trace. Breath sounds are diminished bilaterally, but no wheezes are heard. Peak inspiratory pressures are 55 cm H2O and the pulse oximeter reads 90% with an FiO2 of 0.98. The most appropriate next step in management is: A. Hand-ventilation with a high concentration of volatile anaesthetic. B. The administration of a further 120 mg of thiopentone. C. The administration of a non-depolarising neuromuscular blockade to facilitate ventilation. D. The administration of aminophylline 300 mg intravenously and initiation of a maintenance infusion. E. The institution of 10 cmH2O positive end-expiratory pressure. Select the single best answer Correct Answer: A Hand-ventilation with a high concentration of volatile anaesthetic (?particularly sevoflurane) is the most appropriate initial response to a high airway resistance. See, for example: Rooke GA, Choi JH, Bishop MJ. The effect of isoflurane, halothane, sevoflurane, and thiopental / nitrous oxide on respiratory system resistance after tracheal intubation. Anesthesiology. 1997 Jun;86(6):1294-9. The administration of a further 120 mg of thiopentone is very unlikely to be effective. In view of the likely effect of 7 mgs/kg of thiopentone on the cardiac output, it could well make the hypoxia worse. Similarly, the administration of a non-depolarising neuromuscular blockade to facilitate ventilation in this situation is unlikely to be of any practical use, although relaxants may be indicated as part of a hypoventilation strategy in the therapy of acute, severe asthma. The role of theophylline derivatives in asthma is debatable, but they are certainly not first line therapy. See, for example: Beveridge RC, Grunfeld AF, Hodder RV, Verbeek PR. Guidelines for the emergency management of asthma in adults. CAEP/CTS Asthma Advisory

256

Committee. Canadian Association of Emergency Physicians and the Canadian Thoracic Society. The role of PEEP in asthma is controversial. Number: 1009 Which of the following may be associated with hyperthermia during anaesthesia. 1. Phaeochromocytoma. 2. Thyrotoxicosis. 3. Osteogenesis Imperfecta. 4. Neuroleptic Malignant Syndrome. A: 1,2,3 Correct B: 1,3 Correct C: 2,4 Correct D: 4 Correct E: All Correct

Correct Answer: E Hyperthermia in the anaesthetised patient may be secondary to : 1. Iatrogenic causes 2. Coexisting diseases - Phaeochromocytoma - Thyrotoxicosis - Osteogenesis imperfecta - Riley-Day syndrome - CNS pathology - Sepsis - Parkinson's Disease 3. Drugs - Malignant Hyperthermia - Neuroleptic Malignant Syndrome - Central Anticholinergic Syndrome - 'Tyramine-like' response to MAOI's 1. Iatrogenic-active warming, heated humidifiers, non-permeable drapes. Application of tourniquets for prolonged periods of time have been reported to induce hyperthermia. The mechanism is felt to be release of catecholamines. Injection of A-V malformation with sclerosing solutions may increase temperature for reasons not understood. 2. Secondary to diseases - Phaeochromocytoma secondary to catecholamine release. - Thyrotoxicosis and thyroid storm. Thyroid storm presents with hypertension, hyperthermia and tachycardia, but unlike MH, muscle rigidity and acidosis are unusual. Clinical observation indicates that thyroid gland surgery causes mild hyperthermia, perhaps 257

because of release of thyroid hormone with manipulation of the gland. The mechanism is Na/K ATPase mediated in contrast to MH in which intracellular Ca is elevated Alternatively, the increased temperature may occur independently, secondary to inadequate heat dissipation from a small operative site. - Osteogenesis Imperfecta-Hyperthermia occurs during anaesthesia in these patients. Although a few episodes of true MH have been reported in this population, in many cases clinical and laboratory testing reveal that MH was mistakenly diagnosed. - CNS dysfunction-Status epilepticus is associated with hyperthermia due to increased muscle activity. " Central " or " neurogenic " hyperthermia is associated with brainstem lesions. Patients who experience hypoxic encephalopathy characteristically develop hyperthermia. This may manifest with coma, seizures and abnormal posturing post resuscitation from cardiac arrest and differentiation from MH may be difficult. - Bacteremia/Sepsis- Body temperature characteristically falls when febrile patients are anaesthetized. However, hyperthermia recurrs after surgery, characterized by rigors and intense peripheral vasoconstriction. Bacteremia may be induced by surgical manipulation, leading to postoperative hyperthermia. Appendicectomy constitutes a common scenario for this occurrence , with fever engendered by the release of pyrogens consequent to handling of the septic organ. Surgery for head trauma especially when the oral cavity is disrupted is another example. - Riley-Day Syndrome-This syndrome involves a deficiency of dopamine B-hydroxylase. People with this exhibit profound instability of the autonomic nervous system with a wide variation in blood pressure, heart rate, and body temperature apparently unrelated to temperature. Parkinson's Disease. 3. Drug induced - Malignant Hyperthermia. - Neuroleptic Malignant Syndrome-This is characterized by hyperthermia, muscle rigidity, rhabdomyolysis, arrhythmia, acidosis and death. It is precipitated by haloperidol alone, or with phenothiazines, or occasionally with antidepressants. Despite the clinical similarity to MH, the mechanism is different. Most believe that NMS results from blockade of dopamine receptors in the CNS. The dopamine agonist, bromocriptine is one of the drugs effective in treatment. - Central anticholinergic syndrome - MAO inhibitors-Either alone or in association with pethidine or indirect acting sympathomimetic agents. BENUMOF, J.L & SAIDMAN,L.J; Anesthesia and Postoperative Complications, Mosby, 1991, pp 340-43. Number: 1010

258

Which of the following is / are specifically relevant in a patient with Ankylosing Spondylitis due to undergo anaesthesia ? 1. Aortic incompetence may exist. 2. Reduced lung compliance may make GA hazardous. 3. Maintaining anaesthesia with a face mask may be difficult. 4. Intubation should not pose any additional hazard. A: 1,2,3 Correct B: 1,3 Correct C: 2,4 Correct D: 4 Correct E: All Correct

Correct Answer: B Several of the articular and extra-articular manifestations of Ankylosing Spondylitis (AS) are of significance to the anaesthetist. Cardiovascular involvement occurs in 3.5% of patients over a 15 year period and usually manifests as aortic incompetence. Controversy exists with regard to pulmonary involvement; some sources state that lung function is well preserved, others describe abnormalities in pulmonary function testing as commonplace. Pulmonary dysfunction when it occurs is primarily due to decreased chest wall compliance, and even though upper lobe fibrosis is described, this is not generally associated with loss of lung compliance. Cervical spine, temporo-mandibular, and crico-arytenoid involvement can make airway management and intubation difficult. AS is a disease characterized by fibrosis and ossification of ligamentous insertions and joint capsules. Ossification occurs at the vertebral discs with progression to a degree of complete fusion of the cervicolumbar spine. A minority develop the full features of 'bamboo spine'. The fused spine may fracture even after trivial injury and necessitate vigilance during positioning , head manipulation and transport. In later stages, sensory and motor deficits associated with cauda equina lesions may appear constituting a greater risk for nerve damage. 10% have TMJ involvement. This together with an immobile cervical spine may make airway management difficult. Cervical spine involvement may range from a small degree of limitation of movement to complete ankylosis, usually in flexion with a rotational component. A small group of patients with advanced disease are at risk of sustaining a cervical fracture. Here, the anaesthetist will be confronted with a possibly difficult intubation, difficult airway and risk of fracture. Although rare, it would appear that crico-arytenoid arthritis may also occur in ankylosing spondylitis. In 1984, 5 patients had been reported with this. Three developed acute upper respiratory complications. One had cor-pulmonale felt to be due to narrowing of the glottis. Extra-articular involvement is common. Patients frequently suffer from fatigue, weight loss and low grade fever, associated with hypochromic anaemia and a raised ESR. 83% of males suffer prostatitis, and uveitis occurs in 35% during the course of the disease. Cardiovascular, pulmonary and neurological manifestations are relatively rare but correlate with long standing disease. CVS involvement is found in 3.5% of patients with a 15 year and 10% with a 30 year history and is frequently manifest as AI and cardiomegaly. Scarring of the adventitia and fibrous proliferation of the intima of the aorta and aortic valve cusps may give rise to aortitis 259

and aortic incompetence. Occasionally, the mitral valve is involved as well, and fibrosis may affect the purkinje fibres giving rise to persistent conduction defects. Involvement of the lungs as measured by reduced lung volumes is a known manifestation of the disease. This has suggested to be due to reduced mobility of the thoracic cage due to inflammation or ankylosis of the sternocostal or costoclavicular joints and adjacent syndondroses.This correlates with disease duration resulting in permanently reduced lung function. Asymptomatic patients with normal CXRs have undergone pulmonary function testing. The main abnormalities are reduced TLC and VC and increased ratio of closing capacity to VC. FRC and RV are spared (In this study the authors suggested this pattern is consistent with a restrictive defect. Note that WEST and other texts describe a restrictive pattern as one with reduction in all lung volumes and capacities, but with preservation of 'normal' proportions and flow ratios.) Flow patterns, static compliance and diffusion capacity are also normal. The observation that reduction in lung volumes is also related to the inflammatory intensity of the disease suggests also an inflammatory involvement of the lung parenchyma. Upper lobe fibrosis is known to be a late pulmonary manifestation and may mimic TB when associated with cyst formation. However, reduced chest wall compliance is felt to be the cause of significant reductions in total pulmonary compliance which creates hazard in these patients. Diaphragmatic activity should be normal. Neurological complications have been described in 22% of patients in one series of 45. Spinal cord compression, cauda equina lesions, focal epilepsy, vertebrobasilar insufficiency and peripheral nerve lesions have been described. Vertebral fractures are also more common. See: Feltelius N, Hedenstrom H, Hillerdal G, Hallgren R. Pulmonary involvement in ankylosing spondylitis. Ann Rheum Dis. 1986 Sep;45(9):736-40. Sinclair JR, Mason RA. Ankylosing spondylitis. The case for awake intubation. Anaesthesia. 1984 Jan;39(1):3-11. Salathe M, Johr M. Unsuspected cervical fractures: a common problem in ankylosing spondylitis. Anesthesiology. 1989 May;70(5):869-70.

OPIATES AND OTHER TESTS Number: 5 Clopidogrel: 1. Is a glycoprotein IIb/IIa inhibitor. 2. Is more likely to cause bleeding complications than aspirin. 3. Is an isomer of ticlopidine. 4. May rarely cause thrombocytopenia. A: 1,2,3 Correct

260

B: 1,3 Correct C: 2,4 Correct D: 4 Correct E: All Correct ABCDE Correct Answer: D Kam and Nethery have recently published an excellent review on the thienopyridine platelet antagonists (Clopidogrel and Ticlopidine). According to these authors: "ADP is released from activated platelets, erythrocytes and endothelial cells, and induces platelet adhesion and aggregation. ADP activates platelets by binding to membrane-bound nucleotide receptors (purinoceptors) on the platelet surface called P2 receptors [4]. Human platelets possess two major G protein-coupled ADP receptors, the P2Y1 and P2Y12 receptors, and a third ionotropic receptor, P2X1. The human P2Y1 receptor is a Gq protein-coupled receptor that activates phospholipase C to form inositol triphosphate (IP3) and causes calcium to be released from intracellular stores. The P2Y1 receptor is necessary to trigger a response and initiates the formation of platelet pseudopodia in response to low concentrations of thromboxane A2 or thrombin, and transient platelet aggregation occurs. However, activation of the P2Y1 receptor is insufficient for a full platelet response. The P2Y12, formerly known as P(2T), P2T (AC), P2Y (ADP) or P2Y (cyc), receptor is a Gi protein-coupled receptor that inhibits adenylyl cyclase. This results in a decreased platelet cyclic adenosine monophosphate (AMP) level in response to ADP, activating platelet glycoprotein IIb/IIIa (IIb3 integrin) receptors that bind fibrinogen, leading to stabilisation of platelet aggregation and enhanced platelet secretion. Platelets also possess a third ADP receptor, P2X1, which is a ligand-gated ion channel that mediates rapid transient calcium ion influx. However, the P2X1 receptor does not contribute to platelet aggregation." The incidence of bleeding complications with clopidogrel is very similar to that which occurs with aspirin when used for similar indications (~ 9%). The currently available glycoprotein IIb/IIa inhibitors are: abciximab, tirofiban, and eptifibatide. The drug is structurally very similar to ticlopidine, but is not an isomer (it contains an additional carboxymethyl side group). The drug may rarely cause thrombocytopenia, but far less commonly than ticlopidine. See: Kam PC, Nethery CM. The thienopyridine derivatives (platelet adenosine diphosphate receptor antagonists), pharmacology and clinical developments. Anaesthesia. 2003 Jan;58(1):28-35. Number: 16 Which of the following drugs does NOT possess anti-platelet activity?

261

A. Sevoflurane. B. Tirofiban. C. Hydroxy Ethyl Starch (Hetastarch). D. Remifentanil. E. Dipyridamole. Select the single best answer ABCDE Correct Answer: D Sevoflurane alters platelet aggregation - possibly by suppression of thromboxane A2 formation. Tirofiban is a platelet GP IIb/IIIa receptor antagonist. Others in this class of drug are the human-murine chimeric monoclonal antibody Fab fragment abciximab, the peptide antagonist eptifibatide and the peptidomimetic lamifiban. Hetastarch inhibits platelet function by reducing the availability of the functional receptor for fibrinogen on the platelet surface. (It may also impair coagulation by prolonging the partial thromboplastin times and decreasing factor VIII activity and fibrinogen levels.) There is no evidence of anti-platelet activity for any of the synthetic opioids. Dipyridamole is a platelet adhesion inhibitor, although the mechanism of action has not been fully elucidated. The mechanism may relate to inhibition of red blood cell uptake of adenosine, itself an inhibitor of platelet reactivity, phosphodiesterase inhibition leading to increased cyclic-3,5-adenosine monophosphate within platelets, and inhibition of thromboxane A2 formation which is a potent stimulator of platelet activation. Number: 54 With regard to delta-9 tetrahydrocannabinol (THC): 1. It is the primary psychoactive component of cannabis. 2. It readily crosses the placental barrier. 3. It has clinically useful anti-emetic effects. 4. It has an analgesic efficacy roughly equivalent to codeine. A: 1,2,3 Correct B: 1,3 Correct C: 2,4 Correct D: 4 Correct E: All Correct ABCDE

262

Correct Answer: E The pharmacological actions and therapeutic uses of cannabis and cannabinoids have recently been superbly reviewed by Kumar et al. According to these authors: 1. Although the term cannabis is used colloquially to describe a single entity, over 60 different compounds have been identified and collectively referred to as cannabinoids. These are largely derived from the female plant of Cannabis sativa. The most abundant cannabinoid and the primary psychoactive constituent is d-9 tetrahydrocannabinol (THC), which was isolated in 1964. Other natural cannabinoids are d-8THC, cannabinol and cannabidiol. The THC content is highest in the flowering tops, declining in the leaves, stem and seeds of the plant. Marijuana (THC content 0.55%) is prepared from the dried flowering tops and leaves; hashish (THC content 220%) consists of dried cannabis resin and ompressed flowers. 2. Cannabinoids also cross the placenta, enter the foetal circulation and penetrate into breast milk. Cannabinoids are highly lipid soluble and accumulate in fatty tissues from where they are released slowly back into the bloodstream. Because of this sequestration, elimination from the body is extremely slow and can take many days. With repeated dosage, cannabinoids accumulate and continue to reach the brain over a longer period. 3. Cannabinoids have been used in the prevention of nausea and vomiting caused by anticancer drugs. Nabilone and dronabinol (THC in sesame oil) have been shown to be as effective or more effective than phenothiazines, metoclopramide and domperidone for this indication, although they have not been tested against the 5-HT3 antagonist ondansetron. Nabilone is usually given in a dose of 48 mg per day in divided doses for a few days during cancer chemotherapy. There is a high incidence of adverse effects and 50100% of patients experience drowsiness, dizziness and lethargy. 4. The efficacy of THC appears to be approximately equivalent to codeine and an adjunctive role seems to be the most promising use of cannabinoids in the management of pain. See: Kumar RN, Chambers WA, Pertwee RG. Pharmacological actions and therapeutic uses of cannabis and cannabinoids. Anaesthesia. 2001 Nov;56(11):1059-68. Number: 92 Which of the following complications of amiodarone generally occur with acute therapy? 1 .Pulmonary fibrosis. 2. Thyrotoxic crisis. 3. Hepatitis. 4. Digoxin toxicity. A: 1,2,3 Correct B: 1,3 Correct C: 2,4 Correct D: 4 Correct E: All Correct

263

ABCDE Correct Answer: D The complications generally seen early after administration include rhythm disturbances and potentially, toxicity with other drugs whose plasma concentrations have been elevated as a result of interaction. Patients on long term oral therapy may also experience these side effects, but in addition, may develop pulmonary fibrosis, thyroid dysfunction and reversible abnormality of liver function. Amiodarone has a large side effect profile. Many of its side effects occur during chronic therapy. As a result, it is considered a useful short term agent for the treatment of refractory supraventricular tachyarrhythmias, and it has been replaced by sotalol as the first line agent for long term use. Amiodarone is a structural analog of thyroid hormone. Its electrophysiological effects include - prolonged action potential duration (Vaughan-Williams Class 3) in atria and ventricles. - increased effective refractory period in the AV node, His Purkinje system, and ventricular conducting system. - depression of SA node activity. Haemodynamic effects include powerful systemic and coronary vasodilation with moderate, non-competitive adrenergic receptor antagonism. It does not cause clinically significant myocardial depression in patients with normal ventricular function but may aggravate existing heart failure in patients dependent upon augmented sympathetic drive. Cardiac output should be increased or preserved as a result of afterload reduction. Toxicity is dose dependent, cumulative with time, and not predictable from serum assays. Moreover, due to a half life of 14-04 days, toxicity once manifest, may take months to resolve. Toxic effects which manifest early include: (1)Sinus bradycardia, sinus arrest, AV block, prolongation of the QT interval, and Torsades de Pointes. The effect is additive with other drugs which depress the sinus node including lignocaine and halothane. Two case reports of sinus arrest refractory to atropine under general anaesthesia exist and some sources advocate prophylactic temporary pacing in this scenario. (2)Drug interactions include an elevation of serum levels of digoxin and warfarin. Effects which tend to occur with chronic oral treatment include: (3)Thyroid dysfunction. Hypo- or hyperthyroidism may occur. Hyperthyroidism is sometimes fulminant and refractory to medical treatment, requiring thyroidectomy under " toxic " conditions. (4)Pulmonary fibrosis occurs in 1-13%, and has a mortality of 20-25%. (5)Hepatotoxicity- transient elevation of transaminases occurs. (6)Corneal deposits - Iodine containing microdeposits occur throughout the body. Whilst generally considered asymptomatic, they have led to blurring of vision when in the cornea. (7)Photosensitivity and a characteristic slate grey pigmentation of the face occurs. (8)Neurotoxicity-10-74% develop ataxia and tremor which is not always reversible. HOLT,A.W; " Supraventricular tachyarrhythmias in Critically Ill Patients ", Australasian Anaesthesia 1992, pp 78-85.

264

Number: 176 Regarding this compound: catechol ring and substitutions ( no figure) 1. It is the parent compound of the catechol amines. 2. Substitution on the alpha carbon blocks oxidation by MAO. 3. Substitution on the Beta carbon increases peripheral agonist activity. 4. Substitution on the Beta carbon generally increases central stimulant action. A: 1,2,3 Correct B: 1,3 Correct C: 2,4 Correct D: 4 Correct E: All Correct ABCDE Correct Answer: A Beta Phenylethylamine is the parent compound of the Catechol Amines. Substitutions can be made on the Benzene ring as well as the Alpha and Beta positions on the ethylamine side chain with retention of sympathomimetic activity. Adrenaline, Noradrenaline, dopamine and isoprenaline all have hydroxy groups subtituted onto positions 3 & 4 of the ring. Since o-dihydroxybenzene is also known as 'Catechol' these compounds are also known as catechol amines. Substitution on the beta carbon generally decreases central stimulant activity by lowering the lipid solubility of the compound, but generally greatly enhances alpha and beta potency. Potency of the compounds is reduced markedly when more than two carbons separate the aromatic and amine groups. Number: 177 The parent compound illustrated below: 1. Is Pethidine if R1 = B and R2 = A 2. Is Fentanyl if R1 = C and R2 = D 3. Will have Atropine like qualities 4. Is a Phenylpiperidine A: 1,2,3 Correct B: 1,3 Correct C: 2,4 Correct D: 4 Correct E: All Correct

265

ABCDE Correct Answer: E The phenylpiperidine analgesics (Pethidine and Fentanyl family) were first developed in 1939 by Eisleb and Schaumann who were looking for Atropine like agents. Morphine is a Phenanthrene derivative. Pethidine is partially metabolised by (30%) by N-demethylation to Norpethidine which is responsible for many of the toxic effects of chronic pethidine administration. Number: 211 Sodium nitroprusside A. Promotes fluid and sodium retention B. Decreases venous return C. Causes CNS depression D. Increases cardiac work E. None of the above Select the single best answer ABCDE Correct Answer: B Nitroprusside is the most potent 'mixed' vasodilator - acting on both the arterial and venous side of the circulation. This, results in reduced peripheral vascular resistance and venous return. Nitroprusside does not cause CNS depression and tends to reduce cardiac work. In the presence of ischaemic heart disease the drug can occasionally precipitate acute myocardial ischaemia. This may occur if: 1. Diastolic blood pressure is reduced in the presence of critically stenotic lesions, 2. Reflex tachycardia is precipitated or, 266

3. Coronary 'Steal' has occurred. Number: 212 The side-effects of Enalapril include: A. Drowsiness B. Excess fluid retention C. Hyperglycaemia D. Urinary retention E. Hyperkalemia Select the single best answer ABCDE Correct Answer: E The adverse effects that are common to ACE inhibitors include acute renal failure, hyperkalaemia, angioedema and dry cough, sometimes accompanied with wheezing. Plasma creatinine should be routinely monitored in patients undergoing treatment with ACE inhibitors - particularly at the start of therapy. Renal artery stenosis can come to light under these circumstances. Number: 264 A patient with a type 'B' aortic dissection being treated with sodium nitroprusside(SNP) for the control of arterial blood pressure. The patient develops a lactic acidosis and cyanide toxicity is suspected. Appropriate resuscitation includes the administration of: 1. Sodium thiosulfate. 2. Sodium bicarbonate. 3. Sodium nitrite. 4. Vitamin B12. A: 1,2,3 Correct B: 1,3 Correct C: 2,4 Correct D: 4 Correct E: All Correct ABCDE Correct Answer: A When life-threatening tissue hypoxia is present, ventilating with 100% oxygen, correcting metabolic acidosis with sodium bicarbonate, and administering 3% sodium nitrite (46 mg/kg

267

very slowly intravenously), and sodium thiosulfate (150200 mg/kg IV over 15 min) is the usual treatment for suspected cyanide toxiicty. Sodium nitrite converts haemoglobin to methaemoglobin which competes with cytochrome oxidase for cyanide radicals. Some clinicians will withhold sodium nitrite therapy in anaemic patients, particularly when oxygen delivery is already compromised while others consider its vasodilating effects to be problematic. Hydroxocobalamin (vitamin B12a) may also have a role in the treatment of cyanide toxicity. Hydroxocobalamin binds CN- forming cyanocobalamin which acts as a nontoxic reservoir and can be excreted in the urine. The vitamin can be infused at 25 mg/h to a total of 100 mg or more during and after SNP infusion. Infusion of vitamin B12 (cyanocobalamin) is considered ineffective in removing CN- due to poor binding, thus is not a substitute for hydroxocobalamin . The principal toxic effect of hydroxocobalamin is reddish discoloration of the skin and mucous membranes. The evidence that hydroxocobalamin administration is either needed or efficacious is controversial. See: Anesth Analg 1995 Jul;81(1):152-62: Sodium nitroprusside: twenty years and counting. Friederich JA, Butterworth JF 4th - for an excellent, comprehensive review of the problem. Number: 300 For acute short-term use, (eg hypotensive anaesthesia lasting a few hours) the total intraoperative dose of sodium nitroprusside (SNP) should be limited to a total dose of: A. 0.5 mg/kg. B. 1.5 mg/kg. C. 3.0 mg/kg. D. 5.0 mg/kg. E. 10.0 mg/kg. Select the single best answer ABCDE Correct Answer: B Possible cyanide poisoning has been reported over a wide range of SNP infusion rates and total doses; deaths clearly linked to cyanide toxicity appear to have occurred at infusion rates in excess of, 30120 mcg/kgmin. Blood cyanide concentrations required for clinical toxicity appear to exceed 40 microM/L ; deaths have been reported with blood concentrations exceeding 77, 100, or 1309 microM/L. There appears to be a discrepancy between the maximum 'safe-dosage' recommended by some textbooks of anaesthesia and that recommended in the anaesthetic literature. Conventional wisdom (See, for example 'Drugs and Anesthesia' Ed: Wood and Wood) appears to be that for acute short-term use, as during hypotensive surgery, the total intraoperative dose of SNP should be limited to 1.0 to 1.5 mg/kg administered over a 1 to 3

268

hour period. If the initial dose requirements are such that it seems likely this figure will be exceeded, alternate methods of inducing hypotension should be used. More recently, Friederich and Butterworth have suggested the use of alternative drugs to reduce the dose or shorten the duration of infusion when a dose of 2 micrograms/kg/min (0.12 mg/kg/hr) is exceeded. This is a much lower figure and corresponds to the rate of spontaneous detoxification of SNP in humans as reported by Schulz. I have arbitrarily chosen the 'textbook' answer as the correct answer because it appears to me that the lower dose can generally be quite safely exceeded. However, prolonged or high-dose infusions of nitroprusside should be minimised in critically ill patients, especially if hepatic and / or renal dysfunction is present. See: Anesth Analg 1995 Jul;81(1):152-62: Sodium nitroprusside: twenty years and counting. Friederich JA, Butterworth JF 4th - for an excellent, comprehensive review of the problem. Clin Pharmacokinet 1984 May-Jun;9(3):239-51: Clinical pharmacokinetics of nitroprusside, cyanide, thiosulphate and thiocyanate. Schulz V Number: 311 The risk of intoxication during digoxin treatment is increased by: 1, Hypercalcaemia 2. Hypokalaemia 3. Hypomagnesaemia 4. Hypoalbuminaemia A: 1,2,3 Correct B: 1,3 Correct C: 2,4 Correct D: 4 Correct E: All Correct ABCDE Correct Answer: A The most common precipitating cause of digitalis intoxication is depletion of potassium stores. Advanced age, acute myocardial infarction or ischemia, hypoxemia, magnesium depletion, renal insufficiency, hypercalcemia, electrical cardioversion, and hypothyroidism also reduce the tolerance of the patient to the digitalis glycosides and may provoke latent digitalis intoxication. The calcium channel antagonist verapamil and the antiarrhythmic agent amiodarone also appear to raise serum digoxin levels and can precipitate toxicity if administered to a patient who is already digitalised.

269

Number: 330 Adenosine: 1. Is an endogenous nucleotide. 2. Is a vasodilator. 3. Has a half-life of about 10 seconds. 4. Will successfully revert at least 80% of supraventricular tachycardias. A: 1,2,3 Correct B: 1,3 Correct C: 2,4 Correct D: 4 Correct E: All Correct ABCDE Correct Answer: E Adenosine is an endogenous nucleotide natural to all cells of the body. In pharmacologic doses, it slows conduction through the AV node and has proved highly efficacious as acute IV therapy for patients with paroxysmal supraventricular tachycardia in both reentry and accessory pathway (Wolff-Parkinson-White) dysrhythmias. After IV administration, adenosine undergoes rapid redistribution to erythrocytes and cells of the vascular endothelium, with a half-life estimated at less than 10 seconds. Subsequently, it is metabolized to inosine or adenosine monophosphate. After a bolus of 6 mg, about 60% of patients with paroxysmal supraventricular tachycardia will convert to sinus rhythm within 1 minute. If the initial bolus is unsuccessful, 12 mg given intravenously will convert most of the remaining patients, for a cumulative effectiveness of 92%. Transient high-grade blocks and even asystole may be seen following adenosine administration. These usually resolve rapidly and without therapy. Lerman BB, Belardinelli L Cardiac electrophysiology of adenosine, basic and clinical concepts. Circulation 83:1499, 1991 Number: 343 Phenytoin: 1. Is a Vaughan-Williams Class 1B anti-dysrhythmic. 2. Can precipitate Stevens-Johnson Syndrome. 3. Can be used for the treatment of digoxin toxicity. 4. Can precipitate lupus erythematosus. A: 1,2,3 Correct B: 1,3 Correct

270

C: 2,4 Correct D: 4 Correct E: All Correct ABCDE Correct Answer: E Although mainly used as an anticonvulsant, phenytoin has been used in patients with atrial and ventricular dysrhythmias, digitalis toxicity, and chronic ventricular dysrhythmias. It can be used alone or in combinations. Its action is similar to the other class IB agents. A usual loading dose is 1000 mg given at a rate that does not exceed 50 mg/min, and the ECG and blood pressure are monitored to prevent hypotension and cardiovascular collapse. Rarer but more serious reactions include severe dermatitis and Stevens-Johnson syndrome, a lupus-like syndrome, and possibly even hematologic malignancies. Number: 364 With regard to adverse reactions to the administration of protamine: 1. They are more likely to occur in diabetics. 2. Previous exposure to heparin increases the risk of a reaction 3. Previous exposure to protamine increases the risk of a reaction 4. Pulmonary vasodilation is usually a feature. A: 1,2,3 Correct B: 1,3 Correct C: 2,4 Correct D: 4 Correct E: All Correct ABCDE Correct Answer: B Adverse reactions to intravenous protamine administration include rash, urticaria, bronchospasm, pulmonary vasoconstriction, and/or systemic hypotension leading at times to cardiovascular collapse and death. Diabetic patients receiving daily subcutaneous injections of insulins containing protamine have a 40- to 50-fold increased risk for life-threatening reactions when given protamine intravenously. Another group putatively at increased risk for protamine reactions are men who have undergone vasectomies. With disruption of the blood-testis barrier, studies have shown that

271

20% to 33% of such men develop hemaglutinating autoantibodies against protamine-like compounds. Previous exposure to intravenous protamine given for reversal of heparin anticoagulation may increase the risk for a reaction on subsequent protamine administration. Sharath MD, Metzger WJ, Richerson HB et al: Protamine-induced fatal anaphylaxis, J Thorac Cardiovasc Surg 90:86, 1985. There is evidence that some protamine reactions are mediated through protamine-specific IgG antibody. Weiss ME, Nyhan D, Zhikang P et al: Association of protamine IgE and IgG antibodies with life-threatening reactions to intravenous protamine, N Engl J Med 320:886, 1989. Adverse reactions to intravenous protamine administration include rash, urticaria, bronchospasm, pulmonary vasoconstriction, and/or systemic hypotension leading at times to cardiovascular collapse and death. - It is the combination of pulmonary vasoconstriction with systemic vasodilation which can make the reactions so devastating. Pulmonary hypertension has been associated with the generation of thromboxane, a known pulmonary vasoconstrictor. Number: 364 With regard to adverse reactions to the administration of protamine: 1. They are more likely to occur in diabetics. 2. Previous exposure to heparin increases the risk of a reaction 3. Previous exposure to protamine increases the risk of a reaction 4. Pulmonary vasodilation is usually a feature. A: 1,2,3 Correct B: 1,3 Correct C: 2,4 Correct D: 4 Correct E: All Correct ABCDE Correct Answer: B Adverse reactions to intravenous protamine administration include rash, urticaria, bronchospasm, pulmonary vasoconstriction, and/or systemic hypotension leading at times to cardiovascular collapse and death. Diabetic patients receiving daily subcutaneous injections of insulins containing protamine have a 40- to 50-fold increased risk for life-threatening reactions when given protamine intravenously.

272

Another group putatively at increased risk for protamine reactions are men who have undergone vasectomies. With disruption of the blood-testis barrier, studies have shown that 20% to 33% of such men develop hemaglutinating autoantibodies against protamine-like compounds. Previous exposure to intravenous protamine given for reversal of heparin anticoagulation may increase the risk for a reaction on subsequent protamine administration. Sharath MD, Metzger WJ, Richerson HB et al: Protamine-induced fatal anaphylaxis, J Thorac Cardiovasc Surg 90:86, 1985. There is evidence that some protamine reactions are mediated through protamine-specific IgG antibody. Weiss ME, Nyhan D, Zhikang P et al: Association of protamine IgE and IgG antibodies with life-threatening reactions to intravenous protamine, N Engl J Med 320:886, 1989. Adverse reactions to intravenous protamine administration include rash, urticaria, bronchospasm, pulmonary vasoconstriction, and/or systemic hypotension leading at times to cardiovascular collapse and death. - It is the combination of pulmonary vasoconstriction with systemic vasodilation which can make the reactions so devastating. Pulmonary hypertension has been associated with the generation of thromboxane, a known pulmonary vasoconstrictor. Number: 372 Remifentanil: 1. Has a similar lipid solubility to alfentanil. 2. Relies on pseudocholinesterase for its metabolism. 3. Does not need dose modification in the presence of renal failure. 4. Can be used for induction of anaesthesia when spontaneous respiration is planned. A: 1,2,3 Correct B: 1,3 Correct C: 2,4 Correct D: 4 Correct E: All Correct ABCDE Correct Answer: B Remifentanil is a potent esterase-metabolised opioid (EMO) characterized by rapid clearance and lack of accumulation following repeated or prolonged dosing. This profile results in the rapid dissipation of respiratory depressant and analgesic effects (within 5 to 10 minutes) after the infusion of Remifentanil is discontinued.

273

Remifentanil has a short duration of action (t1/2 = 3 to 10 minutes) because of the rapid esterase metabolism. Steady-state conditions can be achieved in 5 to 10 minutes without a loading or bolus dose of Remifentanil. The esterase metabolism of remifentanil allows higher relative doses (ED90) of Remifentanil to be administered. This flexibility for higher doses of Remifentanil results in superior control of intraoperative stress responses, compared to alfentanil and fentanyl, without delaying recovery. The pharmacokinetic and pharmacodynamic profile of remifentanil allows rapid titration to attenuate signs of light anaesthesia and rapid, predictable recovery when its analgesic effects are no longer needed. The esterase metabolism and rapid blood-brain equilibration result in a rapid onset (approximately 1 minute) and offset of action. Rapid offset of effect requires that early postoperative analgesia be established as part of the anaesthesia plan when postoperative pain is anticipated. The drug does not need dose modification in the presence of renal impairment. No dosage adjustment is recommended in patients with hepatic impairment. The pharmacokinetics of remifentanil are not changed in patients with severe hepatic impairment awaiting liver transplant, or during the anhepatic phase of liver transplant surgery. Individuals with severe hepatic impairment demonstrated statistically significant, reduced sensitivity to carbon dioxide simulation of minute ventilation, which may indicate an increased sensitivity to the respiratory depressant effects of remifentanil. These patients should be closely monitored and the dose of remifentanil should be titrated to the individual patient's need. It must never be administered rapidly for induction of anaesthesia and is not indicated for use as an induction agent when spontaneous respiration is planned. Number: 403 Phenothiazines: 1. Lower the seizure threshold. 2. Possess anticholinergic activity. 3. Possess local anaesthetic activity. 4. Characteristically have small distribution volumes. A: 1,2,3 Correct B: 1,3 Correct C: 2,4 Correct D: 4 Correct E: All Correct ABCDE Correct Answer: A Phenothiazines differ structurally but have similar pharmacologic activity. These agents block postsynaptic dopamine receptors, exhibit anticholinergic activity, and inhibit reuptake of norepinephrine and 5HT in the CNS.

274

They also have peripheral alpha-adrenergic blocking and anticholinergic activity and lower the seizure threshold. Some phenothiazines have a quinidine-like effect on the heart. These agents are efficiently absorbed from the GIT, are highly protein bound, and have large apparent volumes of distribution. They are slowly eliminated by hepatic metabolism with half-lives of 20 to 40 h. Number: 426 Rapid bolus intravenous adenosine could be expected to terminate the following arrhythmias: 1. Atrial flutter with 2:1 atrioventricular block 2. Atrioventricular nodal re-entry tachycardia (AVNRT). 3. Paroxysmal atrial fibrillation 4. Orthodromic atrioventricular re-entry tachycardia A: 1,2,3 Correct B: 1,3 Correct C: 2,4 Correct D: 4 Correct E: All Correct ABCDE Correct Answer: C 1. It would increase the degree of AV block which might aid the diagnosis but would not usually stop the atrial flutter. 2. AVNRT is a re-entry circuit within the AV node which is terminated by adenosine. 3. It might slow the AV conduction but not stop the atrial fibrillation. 4. This is the commonest tachycardia associated with WPW syndrome. The circuit is from atria to ventricles via the AV node and back to the atria via the accessory pathway. Number: 445 Lithium therapy can result in: 1. Inappropriate ADH secretion. 2. Sinus node dysfunction. 3. Hyperglycaemia. 4. Hypothyroidism. A: 1,2,3 Correct B: 1,3 Correct C: 2,4 Correct D: 4 Correct E: All Correct

275

ABCDE Correct Answer: C Several medical complications can develop during lithium therapy. About 25 percent of patients develop some degree of vasopressin-resistant nephrogenic diabetes insipidus with polyuria and polydipsia. The lithium inhibition of adenylate cyclase activity is responsible for the disruption of renal tubular transport. These symptoms are usually completely reversible by lithium withdrawal and often can be ameliorated by a reduction in dosage. Lithium may induce the following ECG changes especially in older patients: T-wave depression, sinus node dysfunction, and, very rarely, sinoatrial block and ventricular irritability. The drug tends to have an 'insulin-like' effect and for this reason may lower blood sugar. Because of its effect on adenylate cyclase activity, lithium inhibits the thyroid gland's secretory function; non-toxic goitres and hypothyroidism can develop, which can be readily corrected during lithium therapy by thyroid supplement. Number: 446 Which statement(s) is/are true with regard to Dantrolene: 1. It has muscle relaxant properties. 2. It has a role in the treatment of neuroleptic malignant syndrome. 3. It lowers intracellular calcium levels. 4. Its solubility increases with increasing pH. A: 1,2,3 Correct B: 1,3 Correct C: 2,4 Correct D: 4 Correct E: All Correct ABCDE Correct Answer: E Dantrolene relaxes skeletal muscle by inhibiting the calcium influx induced by electrical stimulation or potassium-induced depolarisation and inhibtiing augmented caffeine-induced contracture in MH Susceptible muscle. The mechanism and site of action of dantrolene is not completely understood, but it is postulated that it depresses the rate and amount of calcium release from the sarcoplasmic reticulum or increases calcium uptake into the sarcoplasmic reticulum.

276

Dantrolene is packaged in lyophilised form at 20 mg/vial and is reconstructed with 50 ml of sterile water in which it is relatively insoluble. Sodium hydroxide and 3 g of mannitol are added to the vial to allow the dantrolene to dissolve in 2 to 3 minutes. The resulting pH of the solution is 9.5, so care must be taken to prevent extravasation and to monitor for thrombophlebitis. See also: Lancet 1998 Oct 3;352(9134):1131-6 Denborough M Malignant hyperthermia. Number: 473 The anticoagulant effect of warfarin may be enhanced by 1. Carbamazepine 2, Metronidazole 3. Vitamin K 4. Aspirin A: 1,2,3 Correct B: 1,3 Correct C: 2,4 Correct D: 4 Correct E: All Correct ABCDE Correct Answer: C Drugs may affect the activity of warfarin in at least three important ways. Drugs which alter the activity of the heaptic microsomal mixed-function oxidase system which is responsible for warfarin elimination in humans will alter plasma warfarin concentration and effect. Enzyme inducers increase the rate of metabolism and, hence, result in reduced warfarin effect whereas enzyme inhibitors result in increased warfarin effect. Carbamazepine is an enzyme inducer and will increase the metabolism of warfarin leading to a reduced anticoagulant effect. Metronidazole is an enzyme inhibitor and will reduce the metabolism of warfarin leading to an increased anticoagulant effect. Vitamin K will reduce the anticoagulant effect - Warfarin interferes with vitamin K metabolism and prevents the caroboxylation of glutamic residues in factors II, VII, IX and X. Aspirin competes for protein binding with warfarin.

277

- In vitro protein binding interactions have been frequently described with warfarin. However, the clinical significance of many of these reported interactions was not critically evaluated. Warfarin is an example of a low-clearance drug whose clearance is dependent on the free fraction of drug in plasma. Increasing the free fraction will result in a transient increase in the free concentration (and possibly increased effect); however, as the clearance is increased, the total level will fall, restoring the free concentration to pretreatment levels. Number: 475 Metabolic acidosis characteristically occurs in poisoning with 1. Aspirin 2. Ethylene glycol 3. Methanol 4. Phenobarbitone A: 1,2,3 Correct B: 1,3 Correct C: 2,4 Correct D: 4 Correct E: All Correct ABCDE Correct Answer: A Salicylate intoxication is common. It results in impaired generation of adenosine triphosphate and produces a primary respiratory alkalosis. In adults the clinical manifestations may closely simulate a cerebrovascular event or alcoholic ketoacidosis. Central nervous system dysfunction, fever, glycosuria, ketonuria, respiratory alkalosis with an elevated anion gap, tinnitus, dehydration, hypokalaemia and haemostatic defects are common. Ethylene glycol is rapidly absorbed. Peak levels occur approximately 2 h following ingestion. Ethylene glycol has a volume of distribution of 0.6 to 0.8 L/kg of body weight. It is oxidized by alcohol dehydrogenase to glycoaldehyde, which is then metabolized to glycolic acid, glyoxylic acid, and oxalic acid. Effects begin 30 min following ingestion and include nausea, vomiting, slurred speech, ataxia, nystagmus, and lethargy. A faint, sweet aromatic odor may be detected on the breath. Coma, seizures, respiratory depression, cardiovascular collapse, and death may occur. Effects caused by metabolites begin 4 to 12 h following ingestion. At this stage the patient appears more ill than intoxicated. Manifestations include tachypnea, hypotension, agitation, confusion, lethargy, coma, and seizures. Hypocalcemia occurs in a third of patients. Leukocytosis is present in the majority. In severe cases, adult respiratory distress syndrome, cyanosis, pulmonary edema, and cardiomegaly may be seen. In this stage the diagnosis is suggested by metabolic acidosis, an increased anion gap (low bicarbonate and chloride), and an abnormal urinalysis (crystalluria). In patients who survive the early stages, acute tubular necrosis manifested by proteinuria, oliguria, and anuria ensues 12 to 24 h following ingestion. Renal failure may be permanent but typically lasts days to weeks. In early intoxication, osmolality is elevated. Later, an elevated anion gap and decreased serum bicarbonate and chloride are

278

observed. Signs of alcohol-like intoxication suggest a serum ethylene glycol level greater than 8 to 16 mmol/L (50 to 100 mg/dL). Survival has been reported with levels as high as 100 mmol/L (650 mg/dL). Methanol intoxication, a rare and potentially lethal form of poisoning, usually results from ingestion and occasionally inhalation of methanol. Initial symptoms are of blurred vision, elongated anion gap and metabolic acidosis which are typically delayed in onset and may not at first be recognised as methanol-related complaints. Number: 477 Chlorpromazine: 1. Can precipitate cholestatic jaundice. 2. Has alpha-blocking activity. 3. Can produce a photosensitivity rash. 4. Lowers seizure threshold. A: 1,2,3 Correct B: 1,3 Correct C: 2,4 Correct D: 4 Correct E: All Correct ABCDE Correct Answer: E Azer SA, Stacey NH J Gastroenterol Hepatol 1996 Apr;11(4):396-407 Current concepts of hepatic uptake, intracellular transport and biliary secretion of bile acids: physiological basis and pathophysiological changes in cholestatic liver dysfunction. Eberlein-Konig B, Bindl A, Przybilla B Dermatology 1997;194(2):131-5 Phototoxic properties of neuroleptic drugs. - Most neuroleptics are strongly phototoxic in vitro indicating a potential risk for photoinduced reactions also to occur in patients treated with these drugs. Aguglia U, Gambardella A, Le Piane E, De Sarro GB, Zappia M, Quattrone A J Neurol 1994 Oct;241(10):605-10 Chlorpromazine versus sleep deprivation in activation of EEG in adult-onset partial epilepsy. - EEG activation by either sleep deprivation or Chlorpromazine is highly specific in the diagnosis of adult-onset partial epilepsy.

279

Number: 483 Amiodarone: 1. Enhances the peripheral transformation of T4 to T3 2. Inhibits TSH release 3. Rarely causes corneal microdeposits 4. Readily crosses the placenta A: 1,2,3 Correct B: 1,3 Correct C: 2,4 Correct D: 4 Correct E: All Correct ABCDE Correct Answer: C 1. Amiodarone inhibits the peripheral transformation of T4 to T3. 2. Amiodarone inhibits TSH release and decreases pituitary sensitivity to T4 and T3. 3. Most patients develop (reversible) corneal microdeposits. 4. Amiodarone is contraindicated in pregnancy and breast feeding. Number: 491 Remifentanil: 1. Will prevent the rise in intraocular pressure associated with the administration of suxamethonium. 2. Will diminish cerebral vasoreactivity to carbon dixoide. 3. Tends to cause a bradycardia. 4. Provokes histamine release. A: 1,2,3 Correct B: 1,3 Correct C: 2,4 Correct D: 4 Correct E: All Correct ABCDE Correct Answer: B 1. Br J Anaesth 1998 Oct;81(4):606-7 Alexander R, Hill R, Lipham WJ, Weatherwax KJ, elMoalem HE Remifentanil prevents an increase in intraocular pressure after succinylcholine and tracheal intubation.

280

2. Anesthesiology 1998 Aug;89(2):358-63 Ostapkovich ND, Baker KZ, Fogarty-Mack P, Sisti MB, Young WL Cerebral blood flow and CO2 reactivity is similar during remifentanil/N2O and fentanyl/N2O anesthesia. 3. Br J Anaesth 1998 Apr;80(4):467-9 Thompson JP, Hall AP, Russell J, Cagney B, Rowbotham DJ Effect of remifentanil on the haemodynamic response to orotracheal intubation. 4. Anesth Analg 1995 May;80(5):990-3 Sebel PS, Hoke JF, Westmoreland C, Hug CC Jr, Muir KT, Szlam F Histamine concentrations and hemodynamic responses after remifentanil. Number: 499 Acetazolamide: 1. Is used in the management of renal tubular acidosis 2. Causes hypokalaemia 3. Causes metabolic alkalosis 4. Inhibits the action of carbonic anhydrase A: 1,2,3 Correct B: 1,3 Correct C: 2,4 Correct D: 4 Correct E: All Correct ABCDE Correct Answer: C 1. Acetazolamide is a cause of renal tubular acidosis. 3. Acetazolamide enhances renal bicarbonate excretion thereby causing a metabolic acidosis. K Number: 508 Hepatic first-pass metabolism: 1. Is avoided by giving a drug intranasally. 2. Is avoided by giving a drug rectally. 3. Is avoided by giving an intramuscular injection of a drug. 4. Is seen when a drug has a low hepatic extraction ratio. A: 1,2,3 Correct B: 1,3 Correct C: 2,4 Correct

281

D: 4 Correct E: All Correct ABCDE Correct Answer: B Transdermal, sublingual, intranasal and parenteral (IV, IM, SC) routes avoid the portal circulation and hence first-pass metabolism. A variable proportion of a drug given rectally will be absorbed into the portal circulation, therefore first-pass metabolism is not totally avoided. Drugs with high hepatic extraction ratios undergo substantial first-pass metabolism. Number: 512 Epileptiform activity may be seen with: 1. Propofol 2. Neostigmine 3. Etomidate 4. Ketamine A: 1,2,3 Correct B: 1,3 Correct C: 2,4 Correct D: 4 Correct E: All Correct ABCDE Correct Answer: B There is now no doubt that propofol can cause epileptic fits in those who are susceptible. These fits may be considerably delayed in onset. Paradoxically, the drug may also be used in the treatment of status epilepticus that is resistant to other forms of therapy. Etomidate can induce convulsion-like EEG potentials in epileptic patients without the appearance of myoclonic or convulsion-like motor activity. Opitz A, Marschall M, Degen R et al: General anesthesia in patients with epilepsy and status epilepticus. In Delgado-Escueta AV, Wastedain CG, Treiman DM et al, eds: Status epilepticus: mechanisms of brain damage and treatment, New York, 1983, Raven Press. Ketamine is not epileptogenic but enhanced skeletal muscle tone may be manifested by tonic and clonic movements which may resemble fits. The drug may lower seizure threshold.

282

Number: 513 Hyperglycaemia may result from the administration of: 1. Adrenaline 2. Thyroid stimulating hormone 3. Thiazide diuretics 4. Beta blockers A: 1,2,3 Correct B: 1,3 Correct C: 2,4 Correct D: 4 Correct E: All Correct ABCDE Correct Answer: A Adrenaline increases glucagon and stimulates gluconeogenesis. The adverse metabolic effects of thiazide diuretics include hypokalaemia, hypomagnesaemia, hyperglycaemia, and hypercholesterolaemia. Increases in plasma insulin and glucose levels have been observed in thiazide-treated hypertensive patients and have been attributed to a diminished insulin sensitivity induced by diuretic therapy. Patients on beta blockers are at risk of hypoglycaemia under general anaesthesia. Number: 528 Which of the following drugs readily cross the placenta? 1. Physostigmine 2. Suxamethonium 3. Naloxone 4. Heparin A: 1,2,3 Correct B: 1,3 Correct C: 2,4 Correct D: 4 Correct E: All Correct ABCDE Correct Answer: B The transplacental passage of drugs is dependent on several important factors:

283

lipid solubility; degree of drug ionisation molecular weight; protein binding; drug concentration gradients; maternal and foetal blood pH; and placental and foetal drug metabolism. Drug transfer across the placenta occurs mostly by diffusion, which is governed by the Fick diffusion equation. Physostigmine, being a tertiary amine readily crosses the placenta. Naloxone, because of its high lipid solubility, is easily transferred to the foetus and can be administered to the mother to counteract anticipated respiratory depression in the infant. Suxamethonium and heparin are highly ionised and therefore do not readily cross the placenta. Number: 530 Droperidol: 1. Has alpha blocking activity. 2. Is extensively metabolised by the liver. 3. Causes extrapyramidal side effects. 4. Is a phenothiazine. A: 1,2,3 Correct B: 1,3 Correct C: 2,4 Correct D: 4 Correct E: All Correct ABCDE Correct Answer: A Droperidol acts by interfering with dopaminergic transmission in the brain. Because it blocks dopamine receptors, it may cause extrapyramidal side effects. The drug also has some (competitive) alpha-blocking activity. It is a butyrophenone and is extensively metabolised by the liver. Number: 558 With regard to tramadol: 1. Its effects can be largely reversed by naloxone. 2. It is a potent respiratory depressant. 3. It suppresses EEG activity at typically-used dosages. 4. It is supplied as a racemic mixture of D and L forms. A: 1,2,3 Correct B: 1,3 Correct C: 2,4 Correct D: 4 Correct

284

E: All Correct ABCDE Correct Answer: D In volunteer studies only 30% of the effect of tramadol could be antagonised by naloxone. The drug has been found both in vivo and in vitro to have a low activity at mu opioid receptors, and also to inhibit both noradrenaline and 5hydroxytryptamine (5HT) neuronal reuptake, and to facilitate 5HT release . The drug has little respiratory depressant effect, but post-operative nausea is a significant problem. EEG frequencies are significantly increased in patients treated with the drug. Tramadol is supplied as a racemic mixture of the dextro- and laevo-rotatory forms. The effects of tramadol on opioid receptors, noradrenaline and 5HT are now attributed to the separate activities of these enantiomers. (+)Tramadol has a greater affinity at mu receptors and inhibits 5HT uptake and enhances its release, whereas (-) tramadol inhibits noradrenaline uptake. Number: 569 With regard to benzodiazepines which statements are correct? 1. They inhibit release of gamma-aminobutyric acid. 2. When used in premedicant doses,cardiovascular depression is uncommon. 3. They possess analgesic properties. 4. Nausea and vomiting are uncommon side effects. A: 1,2,3 Correct B: 1,3 Correct C: 2,4 Correct D: 4 Correct E: All Correct ABCDE Correct Answer: C The sedative action of benzodiazepines is said to result from their facilitation of gammaaminobutyric acid-mediated inhibitory neurotransmission. The muscle-relaxant effects of the drugs may be due to glycine-mimetic effects in the spinal cord and brainstem. Benzodiazepines cause little depression of either the cardiovascular or respiratory systems.

285

Benzodiazepines are not analgesic agents. Nausea and vomiting are not associated with the administration of benzodiazepines. In fact, in some situations the drugs have useful anti-emetic effects. For example: Anaesth Intensive Care 1999 Feb;27(1):38-40 The effect of midazolam on persistent postoperative nausea and vomiting. Di Florio T, Goucke CR Number: 569 With regard to benzodiazepines which statements are correct? 1. They inhibit release of gamma-aminobutyric acid. 2. When used in premedicant doses,cardiovascular depression is uncommon. 3. They possess analgesic properties. 4. Nausea and vomiting are uncommon side effects. A: 1,2,3 Correct B: 1,3 Correct C: 2,4 Correct D: 4 Correct E: All Correct ABCDE Correct Answer: C The sedative action of benzodiazepines is said to result from their facilitation of gammaaminobutyric acid-mediated inhibitory neurotransmission. The muscle-relaxant effects of the drugs may be due to glycine-mimetic effects in the spinal cord and brainstem. Benzodiazepines cause little depression of either the cardiovascular or respiratory systems. Benzodiazepines are not analgesic agents. Nausea and vomiting are not associated with the administration of benzodiazepines. In fact, in some situations the drugs have useful anti-emetic effects. For example: Anaesth Intensive Care 1999 Feb;27(1):38-40 The effect of midazolam on persistent postoperative nausea and vomiting. Di Florio T, Goucke CR

286

Number: 597 During the administration of Sodium Nitroprusside (SNP), there will be a tendency for: 1. Intra-cranial pressure to rise. 2. Platelet aggregation to be inhibited. 3. Hypoxic pulmonary vasoconstriction to be inhibited. 4. Coronary 'steal' to occur. A: 1,2,3 Correct B: 1,3 Correct C: 2,4 Correct D: 4 Correct E: All Correct ABCDE Correct Answer: E SNP is a direct cerebral vasodilator, leading to increased cerebral blood flow and cerebral blood volume. These changes, when they occur in patients with reduced intracranial compliance, may cause an undesirable elevation of intracranial pressure. Infusion rates of SNP of more than 3 microgram / kg / minute may result in a dose-related decrease in platelet aggregation. The mechanism of this effect is uncertain. All peripheral vasodilators attenuate the hypoxic pulmonary vasoconstrictive response presumably as a result of inappropriate release of constriction to poorly ventilated areas of the lung. SNP has been demonstrated to cause coronary 'steal' in various situations. In the context of acute coronary ischaemia, it probably results from dilation of resistance vessels in nonischaemic myocardium, resulting in diversion of blood flow from ischaemic areas where vessels are already maximally dilated. Any reduction in diastolic blood pressure which occurs as a result of afterload reduction may also contribute to the occurrence of coronary ischaemia. Number: 598 Which of the following drugs increases gastrointestinal motility? A. Fentanyl. B. Atropine. C. Neostigmine. D. Rocuronium. E. Glycopyrrolate. Select the single best answer ABCDE

287

Correct Answer: C By enhancing cholinergic activity, neostigmine increases gastrointestinal motility. Fentanyl, like the endorphins, has several important effects on the enteric nervous system. Within myenteric neurones, the opioids exert inhibitory influences and reduce intestinal motility. Within submucosal neurones they act to decrease mucosal secretion. Atropine, and glycopyrrolate both decrease gut motility. Rocuronium has no effect. Number: 613 With regard to remifentanil: 1. It has a histamine-releasing effect equivalent to an equianalgesic dose of morphine. 2. It has an elimination half-life of approximately 10 minutes. 3. Its effect is prolonged in patients with pseudocholinesterase deficiency. 4. It has a comparable potency to fentanyl. A: 1,2,3 Correct B: 1,3 Correct C: 2,4 Correct D: 4 Correct E: All Correct ABCDE Correct Answer: C 1. It produces no detectable histamine release when administered in doses of up to 30 mcgs/kg over 60 seconds. 2. Using a 3 compartment model, the drug has an initial distribution half-life of about 1 minute, a slower distribution half-life of about 6 minutes and an elimination half-life of approximately 10 minutes. Less than 10% of the AUC is contained within the terminal elimination component 3. The drug is hydrolysed by non-specific esterases in blood and tissues. It is not a substrate for pseudo-cholinesterase and its action is therefore not prolonged in patients with pseudocholinesterase deficiency. 4. The drug has a comparable potency to fentanyl. The pharmacokinetic profile of the drug is such that it is ideally suited to use by continuous infusion. The typical dose range for the drug in an intubated, ventilated patient is 0.1 - 0.5 mcg/kg/min. Hypotension and bradycardia are the most troublesome side-effects in such patients.

288

Number: 620 Which of the following are features of digoxin toxicity? 1. Nausea. 2. Gynaecomastia. 3. Xanthopsia. 4. Hypokalaemia. A: 1,2,3 Correct B: 1,3 Correct C: 2,4 Correct D: 4 Correct E: All Correct ABCDE Correct Answer: A Anorexia, nausea, and vomiting, are among the earliest signs of digoxin intoxication, and are caused by direct stimulation of centres in the medulla. The most typical dysrhthmia is ventricular ectopy either bigeminy, ventricular tachycardia, or, rarely, ventricular fibrillation. Atrioventricular block of varying degrees of severity is also characteristic. Gynaecomastia is a well-recognised complication of digoxin therapy which presumably reflects the affinity (but low potency) of the drug at androgen receptors. Chronic digoxin intoxication may be insidious in onset and characterised by weight loss, cachexia, neuralgias, gynaecomastia, Xanthopsia (yellow vision), and delirium. Hypokalaemia predisposes to toxicity but is not a cause by digoxin overdosage. Number: 629 In the case of a drug which exhibits first-order kinetics: 1. A constant proportion of it is metabolised in a given period of time. 2. It undergoes the most metabolism when plasma concentration is greatest. 3. It has a constant elimination half-life - irrespective of plasma concentration. 4. There is a linear relationship between plasma concentration and time. A: 1,2,3 Correct B: 1,3 Correct C: 2,4 Correct D: 4 Correct E: All Correct

289

ABCDE Correct Answer: A Option 4 describes, in part, the behaviour of a drug which exhibits zero-order kinetics. Zero-order kinetics are observed when the enzyme system responsible for the elimination of the drug is saturated. Some compounds (such as alcohol) are always eliminated by zero order processes, while many drugs (such as phenytoin, salicylates, thiopentone and theophylline derivatives) switch from first-order to zero-order as dosage is increased and the enzyme systems responsible for the elimination of the drug become saturated. Number: 630 Magnesium: 1. Can be used to control hypertension during surgery for phaeochromocytoma. 2. Is an N-methyl-D-aspartate antagonist. 3. Potentiates neuromuscular blockade. 4. Is a pulmonary vasoconstrictor. A: 1,2,3 Correct B: 1,3 Correct C: 2,4 Correct D: 4 Correct E: All Correct ABCDE Correct Answer: A Magnesium is an underused and underrated therapeutic compound. In addition to its important anti-dysrhythmic properties, it is an effective pulmonary and systemic vasodilator, has a role in chronic pain management because of its NMDA receptor blocking properties and potentiates the neuromuscular blocking effects of non-depolarising relaxants. For its use in phaeochromocytoma see: James MF. Use of magnesium sulphate in the anaesthetic management of phaeochromocytoma: a review of 17 anaesthetics. Br J Anaesth. 1989 Jun;62(6):616-23. For NMDA receptor blockade see: Antonov SM, Johnson JW. Permeant ion regulation of N-methyl-D-aspartate receptor channel block by Mg(2+). Proc Natl Acad Sci U S A. 1999 Dec 7;96(25):14571-6. For pulmonary effects see: Fullerton DA, Hahn AR, Agrafojo J, Sheridan BC, McIntyre RC Jr. Magnesium is essential in mechanisms of pulmonary vasomotor control. J Surg Res. 1996 Jun;63(1):93-7.

290

Number: 640 Which of the following agents inhibit adenosine diphosphate (ADP) induced platelet aggregation when administered at typical clinical concentrations? 1. Halothane. 2. Sevoflurane 3. Nitric Oxide. 4. Isoflurane. A: 1,2,3 Correct B: 1,3 Correct C: 2,4 Correct D: 4 Correct E: All Correct ABCDE Correct Answer: A The effects of anaesthetic agents on platelet function are complicated. Basically, two aspects have been examined. First, researchers have looked at the effect of volatile agents on induction of platelet aggregation by triggers such as ADP, collagen or thromboxane A2. Halothane has been found to inhibit this at clinically-used MAC levels. Sevoflurane probably exerts a similar effect, but the effect of isoflurane is only demonstrable at very high MAC (> 20) levels (Can J Anaesth 1997 Nov;44(11):1157-6. Platelet aggregation is impaired during anaesthesia with sevoflurane but not with isoflurane. Hirakata H, et al.). However, it should be noted that some agents reportedly have different effects on different triggers - see, for example, Nozuchi S et al. Sevoflurane does not inhibit human platelet aggregation induced by thrombin. Anesthesiology. 2000 Jan;92(1):164-70. The platelet inhibitory effect of 0-40 ppm inhaled nitric oxide (NO) has been investigated in volunteers. ADP induced platelet aggregation is significantly inhibited at inspired concentrations of 5, 10, and 40 ppm. Moreover, the in vitro bleeding time is significantly prolonged during inhalation of 40 ppm (Thromb Haemost 2000 Feb;83(2):309-15. Randomized, placebo-controlled, blinded and cross-matched study on the antiplatelet effect of inhaled nitric oxide in healthy volunteers. Gries A et al.). The second aspect of platelet function which has been examined is the effect of volatile anaesthetics on the expression of the adhesion molecule P-selectin. This aspect is dealt with in another question. Number: 642 Overdosage of 'Ecstasy' (3,4-methylene dioxymethamphetamine) may be associated with:

291

1. Hyperthermia. 2. Coagulopathy. 3. Rhabdomyolysis. 4. Peripheral neuropathy. A: 1,2,3 Correct B: 1,3 Correct C: 2,4 Correct D: 4 Correct E: All Correct ABCDE Correct Answer: A Ingestion of 3,4-methylene dioxymethamphetamine (MDMA), commonly known as "Ecstasy", produces a pattern of toxicity that includes fulminant hyperthermia, convulsions, disseminated intravascular coagulation, rhabdomyolysis, and acute renal failure. Number: 669 A basic drug with a pKa of 5.0: 1. Can be expected to readily cross the blood-brain barrier. 2. Will be subject to increased excretion if the urine is alkalinised. 3. Will be 50% ionised at a pH of 5.0. 4. Will be almost totally ionised at normal blood pH (7.4). A: 1,2,3 Correct B: 1,3 Correct C: 2,4 Correct D: 4 Correct E: All Correct ABCDE Correct Answer: B A basic drug with a pKa of 5.0 will be present largely in the unionised, lipid-soluble form at blood pH, and so can be expected to readily cross the blood-brain barrier. An alkaline urine retards the excretion of a basic drug - by increasing the proportion of drug in the unionised, lipid-soluble form which can be re-absorbed back into the blood. This is the definition of pKa. Only 0.5% of the drug will be in the ionised form at blood pH.

292

Number: 678 The term "opioid" can be correctly applied to a compound which: 1. Is an agonist at the mu receptor. 2. Is a naturally occurring compound. 3. Is a partial agonist at the mu receptor. 4. Is an antagonist at the mu receptor. A: 1,2,3 Correct B: 1,3 Correct C: 2,4 Correct D: 4 Correct E: All Correct ABCDE Correct Answer: E The term "opioid" can be applied to agonists, partial agonists, mixed agonist-antagonists, and competitive antagonists at the opioid receptors. Opioids can be either naturally occurring or synthetic agents, and includes the endogenous peptides that act on opioid receptors. Number: 679 With regard to morphine: 1. It is approximately 60% protein-bound in the plasma. 2. Its blood-brain barrier penetration is more rapid than most other opiates. 3. Its bioavailability is between 60 and 80%. 4. It has a large steady-state volume of distribution. A: 1,2,3 Correct B: 1,3 Correct C: 2,4 Correct D: 4 Correct E: All Correct ABCDE Correct Answer: D 1. After intravenous administration, about 1/3rd of the administered dose becomes proteinbound, primarily to albumin. The remainder is redistributed rapidly, with a mean redistribution half-time of between 1.5 and 5.0 minutes.

293

2. Morphine is more water soluble than most other opiates and as a result it crosses the blood brain barrier quite slowly. Its peak effects may be delayed for 10 minutes or longer after intravenous administration as a result of this hydrophilicity. 3. Morphine has a high hepatic extraction ratio of 0.7, and thus a bioavailability after oral administration of only 20-30%. 4. The steady-state volume of distribution of morphine is large, approximately 3-4 L/kg. Number: 727 Dopamine antagonists: 1. Cause extrapyramidal side-effects. 2. May be used as antiemetics. 3. May increase gastric emptying. 4. Cause tachycardia. A: 1,2,3 Correct B: 1,3 Correct C: 2,4 Correct D: 4 Correct E: All Correct ABCDE Correct Answer: A The therapeutic effects of the antipsychotics are most closely related to their receptor binding action on dopamine receptors. The desired reversal of hallucinations and delusions is directly related to the relative degree of blockade of central dopamine receptors that inhibit adenylate cyclase. Thus, blockade of dopaminergic receptors is responsible for many of the side effects of the compounds. Dopamine receptors are distributed in five main areas in the central nervous system: 1. The retina; 2. The tuberoinfundibular area; 3. The mesocortical area; 4. The mesolimbic area; 5. The nigrostriatal area. Blockade of dopamine receptors in the nigrostriatal area accounts for the extrapyramidal sideeffects of the dopamine antagonists. Dopamine antagonists have a powerful action on the Chemoreceptor Trigger Zone.

294

Domperidone and metoclopramide are dopamine antagonists which increase the rate of gastric emptying and the tone of the lower oesophageal sphincter. Dopamine antagonists do not cause tachycardia per se. However, some dopamine antagonists (e.g. chlorpromazine) also induce alpha-receptor blockade and this may lead to a tachycardia secondary to vasodilatation. Number: 738 Nitroglycerine (GTN) has been administered for 24 hours through the lumen of a central venous catheter (CVC). The infusion is now changed to one of normal saline administered through the same lumen. Clinically significant amounts of GTN will be eluted into the saline for approximately: A. 20 seconds. B. 2 minutes. C. 20 minutes. D. 2 hours. E. 20 hours. Select the single best answer ABCDE Correct Answer: C A surprisingly large amount of GTN can be absorbed into a CVC. Akiyama et al have reported that clinically significant amounts of GTN may be delivered by elution for 20-45 minutes after cessation of a GTN infusion. Number: 740 Plasma pseudocholinesterase is: 1. Predominately responsible for the inactivation of cis-atracurium. 2. Inhibited by aminoglycoside antibiotics. 3. Found at an increased level in pregnancy. 4. Inhibited by organophosphorous compounds. A: 1,2,3 Correct B: 1,3 Correct C: 2,4 Correct D: 4 Correct E: All Correct ABCDE Correct Answer: D

295

Plasma pseudocholinesterase is found in the plasma but not in the red cell. It is also present in many other tissues, including the liver, brain, kidney, intestine, and pancreas. The physiological function of this enzyme is unclear, but it hydrolyses a large number of choline and other esters and, thus, is less specific than true acetylcholinesterase. It is synthesized in the liver. A number of drugs of importance to the anaesthestist are metabolised by pseudocholinesterase. These include suxamethonium, mivacurium and some of the local anaesthetics, such as procaine, 2-chloroprocaine, and tetracaine, which are esters of benzoic acid derivatives. Cis-atracurium is predominately converted to laudanosine and the monoquaternary acrylate metabolite via Hofmann elimination. Aminoglycoside antibiotics have no effect on plasma pseudocholinesterase, but may prolong the action of non-depolarising neuromuscular blockers. The plasma level of pseudocholinesterase is reduced in pregnancy. Number: 746 The plasma concentration of a drug declines in a manner which is compatible with 'first-order' kinetics. Which of the following statements is correct? A. The drug is likely to be eliminated via a single metabolic pathway. B. The rate of elimination of the drug is independent of plasma concentration. C. The elimination half-life is constant whatever the plasma concentration of the drug. D. The plasma concentration of the drug plotted against time will yield a straight line. E. Aspirin is an example of such a drug. Select the single best answer ABCDE Correct Answer: C A. First-order kinetics do not imply the presence of a single metabolic pathway.- Provided that each elimination pathway follows first-order kinetics, the plasma level of the drug will also follow this kinetic model. B. The rate of elimination of the drug is directly proportional to the plasma concentration. C. The elimination half-life is constant whatever the plasma concentration of the drug. D. A logarithmic plot of the plasma concentration of the drug against time will yield a straight line. E. Aspirin is not an example of such a drug. The half-life of aspirin is determined by the plasma concentration of the drug.

296

Number: 756 Which of the following effects are NOT provoked by histamine? A. Tachycardia B. Enhancement of myocardial contractility. C. Coronary artery vasodilatation D. Prolongation of the PR interval E. Prolongation of the QT interval. Select the single best answer ABCDE Correct Answer: E Histamine is a positive chronotrope - both directly via H2 receptors and indirectly by stimulation of adrenal catecholamine release. Enhancement of myocardial contractility occurs via stimulation of H2 receptors. Both vasodilation (via H2 receptors) and vasoconstriction (via H1 receptors) can occur. Histamine markedly increases the PR interval. Blockade of H1 receptors causes prolongation of the QT interval. Number: 757 Dantrolene Sodium: 1. Depresses neuromuscular transmission. 2. Lowers the Ca++ level in the sarcoplasmic reticulum (SPR). 3. Is more soluble in an acid solution. 4. Is active at the ryanodine receptor. A: 1,2,3 Correct B: 1,3 Correct C: 2,4 Correct D: 4 Correct E: All Correct

ABCDE Correct Answer: D

297

Dantrolene is a skeletal muscle relaxant which acts by inhibiting intracellular Ca(2+) release from sarcoplasmic reticulum (SR). The skeletal muscle ryanodine receptor is the principal molecular target for the drug. It has no direct effects on neuromuscular transmission. Dantrolene is poorly soluble in water. It is supplied in ampoules containing 20 mg of lyophilised dantrolene sodium (powder) together with mannitol (3G, to improve solubility) and sodium hydroxide to yield a solution of pH 9.5, when the contents are dissolved in 60 mI of water. The final concentration of the parenteral preparation is 0.33 mg/ml. Number: 760 The risk of a severe reaction to protamine used for heparin reversal following cardiopulmonary bypass is considerably more likely in: 1. A diabetic treated with neutral protamine Hagedorn (NPH) insulin. 2. A man who has had a vasectomy. 3. A patient who gives a history of allergy to fish. 4. A non-insulin dependent diabetic A: 1,2,3 Correct B: 1,3 Correct C: 2,4 Correct D: 4 Correct E: All Correct ABCDE Correct Answer: B In a recent study by Kimmel et al three risk factors which were independently associated with an adverse reaction to protamine were identified. These were neutral protamine Hagedorn insulin use, fish allergy and a history of non-protamine medication allergy. Surprisingly previous exposure to protamine (for heparin reversal) did not increase the risk of a severe reaction, but did increase the risk of development of transient pulmonary hypertension. The situation with regard to previous vasectomy is less clear. - The majority of vasectomised men develop antibodies against different sperm antigens, including protamine. Because salmon protamine is used for heparin reversal and a cross-reactivity has been observed between human and salmon protamine, vasectomised men are theorectically be at risk of adverse reactions. In practice this does not appear to be a major issue and (for example) Vezina et al have concluded that "Vasectomized men are not at increased risk of adverse reactions following the injection of protamine sulfate." Number: 775 Midazolam 1. Can be given intranasally.

298

2. May have a prolonged effect if given with erythromycin. 3. Has active metabolites. 4. Is soluble in water at a pH of 4.0 A: 1,2,3 Correct B: 1,3 Correct C: 2,4 Correct D: 4 Correct E: All Correct ABCDE Correct Answer: E Midazolam can be given intranasally - it is particularly useful in children when used by this route. Both midazolam and erythromycin are substrates for the cytochrome P450 (CYP) 3A system. Midazolam is metabolised in the liver by hydroxylation to the major metabolite, 1-hydroxy midazolam, and a secondary metabolite, 4-hydroxy midazolam. Both 1- and 4-hydroxy midazolam metabolites are conjugated and then excreted in the urine as glucuronides. Although the metabolites have pharmacological activity, they are probably of little clinical importance. Midazolam is water soluble (pK, of midazolam is 6.5) and the parenteral formulation therefore does not contain organic solvents.

Number: 783 Esmolol: 1. Is metabolised by plasma pseudocholinesterase. 2. Has an elimination half-life of about 10 minutes. 3. Rarely causes hypotension. 4. Is a cardio-selective beta blocker. A: 1,2,3 Correct B: 1,3 Correct C: 2,4 Correct D: 4 Correct E: All Correct ABCDE Correct Answer: C

299

Esmolol is an ultra short-acting intravenous cardioselective beta-blocker. It has an extremely short elimination half-life (mean: 9 minutes; range: 4 to 16 minutes) and a total body clearance approaching 3 times cardiac output and 15 times hepatic blood flow. The elimination of esmolol is independent of renal or hepatic function as it is metabolised by red blood cell cytosol esterases to an acid metabolite and methanol. The acid metabolite, which is renally eliminated, has 1500-fold less activity than esmolol. The principal adverse effect of esmolol is hypotension. The incidence of hypotension appears to increase with doses exceeding 150 micrograms/kg/min and in patients with low baseline blood pressure. Hypotension infrequently requires any intervention other than decreasing the dose or discontinuing the infusion. The drug has been comprehensively reviewed by Wiest. See: Clin Pharmacokinet 1995 Mar;28(3):190-202: Esmolol. A review of its therapeutic efficacy and pharmacokinetic characteristics. Wiest D Number: 802 The use of sodium nitroprusside is relatively contraindicated in a patient with: 1. Tobacco Amblyopia. 2. Protein - Calorie Malnutrition (PCM). 3. Leber's Optic Atrophy. 4. Uncomplicated type I diabetes mellitus. A: 1,2,3 Correct B: 1,3 Correct C: 2,4 Correct D: 4 Correct E: All Correct ABCDE Correct Answer: A Tobacco amblyopia and Leber's optic atrophy have both been attributed to the effects of chronic cyanide intoxication. For this reason, the use of sodium nitroprusside is relatively contraindicated. Tobacco amblyopia is now so rare that some observers have questioned whether or not the condition exists!! - However, two cases were reported in the United States as recently as 1993. (See: Rizzo JF, Lessell S. Tobacco amblyopia. Am J Ophthalmol. 1993 Jul 15;116(1):84-7.)

300

Leber's Optic Atrophy is believed to be caused by a defect in cyanide metabolism. See: Berninger TA, von Meyer L, Siess E, Schon O, Goebel FD. Leber's hereditary optic atrophy: further evidence for a defect of cyanide metabolism? Br J Ophthalmol. 1989 Apr;73(4):314-6. The use of sodium nitroprusside is relatively contraindicated in a patient with protein - calorie malnutrition as thiosulfate stores are depleted in PCM. This is particularly true if the diet of the starving person includes cassava roots (as is the case in some parts of Africa) which are an important natural source of cyanide. See Osuntokun BO. Nutritional problems in the African region. Bull Schweiz Akad Med Wiss. 1976 Mar;31(4-6):353-76. The use of SNP does not appear to be contraindicated in a patient with uncomplicated diabetes mellitus. Number: 825 Clonidine: 1. Increases the minimal alveolar concentration of volatile agents. 2. Is an alpha 1 adrenoceptor agonist. 3. Is contraindicated via the epidural route. 4. Reduces plasma catecholamine levels. A: 1,2,3 Correct B: 1,3 Correct C: 2,4 Correct D: 4 Correct E: All Correct ABCDE Correct Answer: D Numerous studies have shown that the drug reduces the minimal alveolar concentration of volatile agents. Clonidine is a relatively selective alpha 2 agonist. By stimulating alpha 2 receptors in the central nervous system, sympathetic outflow from the central nervous system is reduced, catecholamine levels fall, and a reduction in blood pressure occurs. In addition, plasma renin activity falls, probably as a result of reduced sympathetic stimulation at the renal adrenergic receptors. Stimulation of the presynaptic receptors on the adrenergic neurons, which inhibit catecholamine release, may also contribute to the observed reduction in catecholamines following clonidine administration. Similarly, the drug has an important adjunctive role in the field of epidural analgesia. See, for example, Kizilarslan S, Kuvaki B, Onat U, Sagiroglu E. Epidural fentanyl-bupivacaine compared with clonidine-bupivacaine for analgesia in labour. Eur J Anaesthesiol. 2000 Nov;17(11):692-7.

301

Number: 838 Which of the following compounds is NOT a precursor of adrenaline? A. Tyrosine. B. Phenylalanine C. Dopamine. D. Phenylephrine. E. Noradrenaline. Select the single best answer ABCDE Correct Answer: D The biosynthetic pathway of adrenaline is: Phenylalanine - Tyrosine - DOPA - Dopamine - Noradrenaline - Adrenaline. Number: 838 Which of the following compounds is NOT a precursor of adrenaline? A. Tyrosine. B. Phenylalanine C. Dopamine. D. Phenylephrine. E. Noradrenaline. Select the single best answer ABCDE Correct Answer: D The biosynthetic pathway of adrenaline is: Phenylalanine - Tyrosine - DOPA - Dopamine - Noradrenaline - Adrenaline. Number: 839 Which statement is INCORRECT with regard to hydralazine? A. It is partly metabolised by acetylation. B. It is partly metabolised by plasma cholinesterase. C. It can cause a lupus-like syndrome. D. It stimulates the baroreceptor reflex. E. It is a direct-acting arteriolar vasodilator.

302

Select the single best answer ABCDE Correct Answer: B Hydralazine remains a useful and effective antihypertensive agent. It acts directly to produce arterial muscle relaxation with a consequent fall in peripheral resistance and blood pressure. Reflex tachycardia (due to stimulation of the baroreceptors) can be a troublesome side-effect - which is well treated by simultaneous beta blockade. A drug-induced lupus syndrome is produced by prolonged high-dose hydralazine therapy. This side effect occurs more commonly at high hydralazine concentrations, being seen in 10 to 20% of patients receiving 400 mg/day. The drug is partially metabolised by acetylation, the rate of which is bimodally distributed in the population. Slow acetylators, because they eliminate the drug more slowly, develop druginduced lupus at lower hydralazine doses than do fast acetylators. Number: 905 Which of the following drugs possess anti-platelet activity? 1. Sevoflurane. 2. Abciximab. 3. Hydroxy Ethyl Starch (Hetastarch). 4. Clopidogrel. A: 1,2,3 Correct B: 1,3 Correct C: 2,4 Correct D: 4 Correct E: All Correct ABCDE Correct Answer: E Sevoflurane (in common with many other volatile agents) alters platelet aggregation possibly by suppression of thromboxane A2 formation. Abciximab (human-murine chimeric monoclonal antibody Fab fragment) is a platelet GP IIb/IIIa receptor antagonist. Others in this class of drug are the peptide antagonist eptifibatide and the peptidomimetics tirofiban and lamifiban. Hetastarch inhibits platelet function by reducing the availability of the functional receptor for fibrinogen on the platelet surface. (It may also impair coagulation by prolonging the partial thromboplastin times and decreasing factor VIII activity and fibrinogen levels.)

303

Clopidogrel is an inhibitor of A.P. induced platelet aggregation acting by direct inhibition of adenosine diphosphate (ADP) binding to its receptor and of the subsequent A.P. mediated activation of the glycoprotein GPIIb/ IIIa complex. Chemically it is methyl (+)-( S)-a -(2chlorophenyl)-6,7-dihydrothieno[3,2-c] pyridine-5(4H)-acetate sulfate (1:1). Number: 938 The plasma half-life of protamine is approximately: A. 10 minutes. B. 30 minutes. C. 1 hour. D. 2 hours. E. 4 hours. Select the single best answer ABCDE Correct Answer: A This topic has been the subject of recent research by Buttwerworth et al. They found the halflife to be about 7.4 minutes. According to these authors: "Despite its long use in clinical medicine, protamine concentrations and pharmacokinetics in humans have not been reported. The occasional reoccurrence of anticoagulation after protamine reversal of heparin led us to hypothesize that protamine plasma concentrations decrease rapidly. We developed a method for the measurement of protamine in plasma. Eighteen fit volunteers gave their consent to receive 0.5 mg/kg protamine sulfate administered IV by an infusion pump over 10 min. Heart rate, mean arterial blood pressure, and cardiac output, all measured noninvasively, were recorded and blood samples obtained during and after protamine infusion. Blood plasma was subjected to solid-phase extraction and highperformance liquid chromatography. The administration of protamine was associated with no significant changes in heart rate, mean arterial blood pressure, or cardiac output. Plasma protamine concentrations decreased rapidly, becoming nondetectable within approximately 20 min. Protamine elimination differed significantly between men and women: men had significantly larger areas under the concentration versus time curve. Model-independent pharmacokinetic analysis revealed median (range) values as follows: volume of distribution at steady state, 12.3 (6.963.1) L; clearance, 2.2 (1.112.1) L/min; and t1/2, 7.4 (5.99.3) min. Concentration versus time plots revealed an atypical pattern inconsistent with usual exponential models. The Schwartz-Bayesian criterion identified a one-compartment Michaelis-Menten model and a two-compartment exponential model with irreversible binding as performing better than conventional one- or two-compartmental exponential models; however, performance errors were large with both Michaelis-Menten and exponential models. All models described rapid decreases in protamine blood concentrations. IMPLICATIONS: We developed a method for measurement of protamine in human blood. In volunteers, protamine concentrations decreased rapidly after administration. The rapid

304

disappearance of protamine from the circulation, as defined by a median half-life of 7.4 min, could contribute to cases of "heparin rebound" after initial adequate reversal of heparin. " See: Butterworth J, Lin YA, Prielipp R, Bennett J, James R. The pharmacokinetics and cardiovascular effects of a single intravenous dose of protamine in normal volunteers. Anesth Analg. 2002 Mar;94(3):514-22. Number: 939 The effects of cannabis include: 1. Anti-emesis. 2. A reduction in intra-ocular pressure. 3. Bronchodilatation. 4. Hypertension. A: 1,2,3 Correct B: 1,3 Correct C: 2,4 Correct D: 4 Correct E: All Correct ABCDE Correct Answer: A The pharmacological actions and therapeutic uses of cannabis and cannabinoids have recently been superbly reviewed by Kumar et al. According to these authors: 1. Cannabinoids have been used in the prevention of nausea and vomiting caused by anticancer drugs. Nabilone and dronabinol (THC in sesame oil) have been shown to be as effective or more effective than phenothiazines, metoclopramide and domperidone for this indication, although they have not been tested against the 5-HT3 antagonist ondansetron. Nabilone is usually given in a dose of 48 mg per day in divided doses for a few days during cancer chemotherapy. There is a high incidence of adverse effects and 50 100% of patients experience drowsiness, dizziness and lethargy. 2. Several studies have shown that smoked or orally administered cannabis and intravenous infusions of THC can decrease intraocular pressure (IOP) in normal subjects. Only two double-blind controlled trials of THC in patients with glaucoma have been reported. Merritt et al. studied 18 patients who inhaled THC 2%. They observed a significant decrease in IOP but also noted hypotension, palpitations and psychotropic effects. These effects occurred with such frequency as to militate against the routine use of cannabis in this way. Tolerance to the IOP-decreasing effect develops rapidly and the place of cannabinoids in the treatment of glaucoma remains to be established. 3. Acute administration of cannabis and THC exert a definite bronchodilator effect on the small airways of the lungs. Tashkin et al. studied 14 asthmatic volunteers and compared

305

smoked cannabis (THC 2%), oral THC (15 mg) and a standard bronchodilator (isoprenaline 0.5%). They found that smoked cannabis and oral THC produced significant bronchodilation for at least 2 h. However, smoking cannabis is not a therapeutic option because of the other smoke constituents. 4. The drug tends to cause hypotension - at least in part due to a peripheral vasodilatory effect. See: Kumar RN, Chambers WA, Pertwee RG. Pharmacological actions and therapeutic uses of cannabis and cannabinoids. Anaesthesia. 2001 Nov;56(11):1059-68. Number: 984 Clopidogrel: 1. May prolong the bleeding time for at least a week. 2. Is metabolised to a more active metabolite. 3. May cause thrombocytopenia. 4. Is a reversible platelet ADP receptor antagonist. A: 1,2,3 Correct B: 1,3 Correct C: 2,4 Correct D: 4 Correct E: All Correct ABCDE Correct Answer: A Kam and Nethery have recently published an excellent review on the thienopyridine platelet antagonists (Clopidogrel and Ticlopidine). According to these authors: "The ADP receptor antagonists produce irreversible inhibition of platelet aggregation, and therefore the effect is present for the life of the platelet. Bleeding time returns to normal 10 days after ceasing clopidogrel administration. A platelet count should be performed to exclude neutropenia or thrombocytopenia. Platelet aggregation tests may be useful in patients presenting for elective surgery. Further studies are required to evaluate the value of the pointof-care platelet function monitor (PFA-100) for assessing residual antiplatelet effects in patients receiving the thienopyridines and presenting for urgent or emergency surgery. For elective surgical procedures, these agents should be stopped 7-10 days before surgery, except if the benefit of the antiplatelet effect outweighs the risk of peri-operative bleeding. Clopidogrel treatment in addition to aspirin before percutaneous coronary intervention has been shown to decrease the incidence of coronary occlusive events. Pre-operative clopidogrel treatment may be useful in decreasing the incidence of thrombosis in patients with peripheral arterial disease undergoing vascular interventions, e.g. arterial bypass, endarterectomy or percutaneous transluminal angioplasty."

306

Clopidogrel undergoes extensive and rapid metabolism by the hepatic cytochrome P450-1A enzymes to produce active and inactive metabolites. The active metabolite has been identified only recently - being a thiol derivative of the parent molecule. The drug may cause thrombocytopenia, but far less commonly than ticlopidine. See: Kam PC, Nethery CM. The thienopyridine derivatives (platelet adenosine diphosphate receptor antagonists), pharmacology and clinical developments. Anaesthesia. 2003 Jan;58(1):28-35. Number: 989 Protamine administered after termination of cardio-pulmonary bypass may cause: 1. An anaphylactic reaction. 2. Intense pulmonary vasoconstriction. 3. Histamine release. 4. Profound myocardial depression. A: 1,2,3 Correct B: 1,3 Correct C: 2,4 Correct D: 4 Correct E: All Correct ABCDE Correct Answer: A The subject of protamine reactions is superbly addressed by Gravlee in his book on cardiopulmonary bypass. According to this author: "Horrow's classification sorts protamine reactions into type 1, type II, and type Ill. Type 1 reactions result in transient systemic hypotension secondary to rapid administration. Type II reactions consist of anaphylactic and anaphylactoid reactions, which are further divided into types Ha, Ilb, and Ilc. Type lla comprises true anaphylactic reactions. Immediate anaphylactoid reactions characterize type llb, and delayed anaphylactoid (e.g., noncardiogenic pulmonary edema) are considered type IIc reactions. Type Ill reactions consist of catastrophic pulmonary vasoconstriction. We propose an alternative classification as follows: (a) pharmacologic histamine release, (b) true anaphylaxis mediated by a specific antiprotamine immunoglobulin (Ig) IE antibody, and (c) thromboxane release leading to pulmonary vasoconstriction and bronchoconstriction. Thus, Horrow's type I reactions, which are characterized by systemic hypotension secondary to rapid administration, correspond in our classification to pharmacologic histamine release. Whereas Horrow includes true anaphylactic and anaphylactoid reaction in his type 11, we consider them as two different types because true anaphylaxis is mediated by a specific

307

antiprotamine lgE antibody that can be produced by protamine in the absence of heparin. The role of antiprotamine IgG antibodies is not clear. The anaphylactoid reaction is independent of a specific lgE antibody, is associated with the heparin-protamine complex, and does not occur in the absence of heparin. Catastrophic pulmonary vasoconstriction is the most common example of the anaphylactoid reaction in our categorization, whereas Horrow classifies it separately. The rare occurrence of delayed pulmonary edema and adult respiratory distress syndrome appear to represent different manifestations of anaphylactoid responses, thus we classify them that way." Ford et al have recently analysed the records of 23 patients who suffered anaphylaxis during cardiac surgery. In their study, 7 of 23 (30%) patients reacted to the antibiotics (6 to cephalosporins, 1 to vancomycin); 6 of 23 (26%), to gelatin solution (Hemaccel, Hoechst, Australia); 4 of 23 (17%), to nondepolarizing muscle relaxants; 3 of 23 (13%), to protamine; 2 of 23 (9%), to blood products; and 1 of 23 (4%), to morphine. The matter is slightly contentious, but the drug is almost certainly NOT a profound myocardial depressant. See: Cardiopulmonary Bypass: Principles and Practice 2/ed. Glenn P. Gravlee. Lippincott Williams & Wilkins ISBN: 0683304763 7 LCCN: 99057043 Ford SA, Kam PC, Baldo BA, Fisher MM. Anaphylactic or anaphylactoid reactions in patients undergoing cardiac surgery. J Cardiothorac Vasc Anesth. 2001 Dec;15(6):684-8. Number: 1021 Dexmedetomidine: 1. Is an alpha2-adrenoreceptor antagonist. 2. Has analgesic properties. 3. Has an elimination phase half-life of about 6 minutes. 4. Has a tendency to cause bradycardia. A: 1,2,3 Correct B: 1,3 Correct C: 2,4 Correct D: 4 Correct E: All Correct ABCDE Correct Answer: C

308

MEDICINE TESTS Number: 2 A 27 year old, otherwise fit man gives a history suggestive of recurrent tachyarrhythmias. During a previous anaesthetic he apparently arrested and required DC cardioversion. He now presents for an arthroscopy. His ECG is shown below. The most likely diagnosis is: A. Lown Ganong Levine syndrome. B. Prolonged QT syndrome. C. Wolff Parkinson White syndrome. D. Sick Sinus syndrome. E. Atrio-Ventricular (AV) nodal re-entrant tachycardia. Select the single best answer

ABCDE Correct Answer: C This is a classic picture of the Wolff Parkinson White syndrome. - Note the short PR interval (less than 120 ms), slurred upstroke to the QRS indicating pre-excitation ('delta' wave), broad QRS and secondary ST and T wave changes. The typical clinical presentation of prolonged QT syndrome is the occurrence of syncope or cardiac arrest, precipitated by emotional or physical stress, in a young individual with a prolonged Q-T interval on the ECG. The syncopal episodes are due to torsade de pointes (not tachycardia) - which often degenerates into ventricular fibrillation. Traditionally, a Q-T interval in excess of 440msec is considered prolonged. Lown Ganong Levine syndrome is a form of pre-excitation characterised by a short PR interval associated with a normal QRS complex.

309

The diagnosis of sick sinus syndrome is best made by Holter monitoring (at the time of symptoms). AV nodal re-entrant tachycardia is the most common type of re-entrant paroxysmal supraventricular tachycardia (SVT), occurring in about 60% of patients with SVT. Symptoms most commonly appear in the late teens or early twenties. Patients with AV nodal re-entrant tachycardia have no greater incidence of heart disease than the general population. The AV nodal re-entrant circuit involves the AV node and tissue in close proximity to it. Dual conduction pathways within the node allow electrical impulses to recycle within the AV nodal region at rate of 150-250 beats per minute after a premature beat encounters one of the pathways during its refractory period. The ECG is normal when the re-entry circuit is not active. For those with an interest in eponymous conditions: Louis Wolff qualified in medicine at Harvard in 1922. He spent his internship at the Massachusetts General Hospital and subsequently specialised in cardiology. He was a visiting physician and chief of the electrocardiographic laboratory at the Beth Israel Hospital. John Parkinson attended University College, London, and subsequently trained in medicine at the University of Freiburg and the London Hospital. He qualified 1907 and obtained a doctorate in 1910. He became the consultant in charge of the cardiology department at the London Hospital and was also appointed to the consulting staff of the National Heart hospital, London. From 1931 to 1956 he was a civilian cardiologist to the Royal Air Force. Paul Dudley White was the son of a family doctor in Roxbury, Massachusetts. He was educated at the Roxbury Latin Grammar School, proceeding to Harvard University and graduating in 1908. He studied medicine at Harvard, graduating in 1911 and interned at the newly established department of paediatrics at Massachusetts General Hospital. For the next two years he was on the medical service with Dr. R. I. Lee and together they developed a technique for measuring blood coagulation, which is still commonly used, called the Lee and White method. This was his first medical publication. Financed by a Sheldon travelling scholarship he then spent a year with Thomas Lewis (1881-1945) studying the electrocardiogram (ECG) at the University College Hospital in London. Number: 50 A fifty year old man suffers an acute myocardial infarction (MI). In which of the following conditions is temporary transvenous pacing most clearly indicated? A. First-degree heart block. B. Mobitz type I second-degree AV block with normal haemodynamics. C. Mobitz type II second-degree AV block. D. Accelerated idioventricular rhythm. E. Bundle branch block known to exist before the acute MI. Select the single best answer

310

ABCDE Correct Answer: C A Committee of the American College of Cardiology and the American Heart Association has published an authoritative series of recommendations for the management of patients suffering acute myocardial infarction. Their recommendations for the institution of temporary transvenous pacing (from 'Class 1' (most pursuasive) to 'Class 3' (least pursuasive)) are: Class 1: 1. Asystole. 2. Symptomatic bradycardia (includes sinus bradycardia with hypotension and type I seconddegree AV block with hypotension not responsive to atropine). 3. Bilateral BBB (alternating BBB or RBBB with alternating LAFB/LPFB) (any age). 4. New or indeterminate age bifascicular block (RBBB with LAFB or LPFB, or LBBB) with first-degree AV block. 5. Mobitz type II second-degree AV block. Class 2a: 1. RBBB and LAFB or LPFB (new or indeterminate). 2. RBBB with first-degree AV block. 3. LBBB, new or indeterminate. 4. Incessant VT, for atrial or ventricular overdrive pacing. 5. Recurrent sinus pauses (greater than 3 seconds) not responsive to atropine. Class 2b: 1. Bifascicular block of indeterminate age. 2. New or age-indeterminate isolated RBBB. Class 3: 1. First-degree heart block. 2. Type I second-degree AV block with normal hemodynamics. 3. Accelerated idioventricular rhythm. 4. Bundle branch block or fascicular block known to exist before acute MI. Number: 60 In the case of a fifty year old man with a phaeochromocytoma: 1. The administration of glucagon will elevate his plasma catecholamine levels. 2. The administration of clonidine will suppress his plasma catecholamine levels. 3. I-131 - metaiodobenzyl guanidine can be used to localise the site of his tumour. 4. There is a less than 1% chance that the tumour is malignant. A: 1,2,3 Correct B: 1,3 Correct C: 2,4 Correct D: 4 Correct

311

E: All Correct ABCDE Correct Answer: B The diagnosis of phaeochromocytoma requires the demonstration of excessive catecholamine secretion. The most widely used procedure includes measurement of urinary catecholamines or their metabolites, vanillomandelic acid and total metanephrines. Of these, the urinary metanephrines provide a more sensitive clue to the presence of phaeochromocytoma. Furthermore, plasma catecholamine measurements may be as reliable as the measurement of urinary metabolites. Determination of plasma catecholamines requires that the patient be in a fasting state and should rest comfortably in the supine position for at least 30 minutes before testing. The amounts of excreted free catecholamines and their metabolites vary depending on the levels of synthesising and metabolising enzymes within the tumour. Additional provocative testing may be done with a glucagon stimulation test. Glucagon is given as an IV bolus of 1.02.0mg. A positive glucagon test requires a clear increase of at least three fold or over 2,000 picogram/ml in plasma catecholamines one to three minutes after drug administration. A simultaneous increase in blood pressure of at least 20mmHg should be present. A suppression test uses the ability of clonidine, a centrally acting alpha- adrenergic agonist, to suppress the release of neurogenically mediated catecholamines. Plasma catecholamines in patients with essential hypertension are suppressed by clonidine, whereas they are unaltered in patients with phaeochromocytoma. A normal clonidine suppression tests consists of a fall of noradrenaline and adrenaline to a level below 500 picogram/ml 2- 3 hours after the administration of 0.3mg of clonidine. Bravo et al have devised the following approach to patients with possible pheochromocytoma. Concentrations of total plasma catecholamines are measured after the patient is rested in a supine position for at least 30 minutes. Values over 2,000 picogram/ml are considered pathognomonic of phaeochromocytoma. Patients with values between 1,000 and 2,000 picogram/ml receive a clonidine suppression test. An abdominal CT scan is then performed on patients with clinical and biochemical features suggestive of phaeochromocytoma. A recently developed technique for localisation of neoplastic chromaffin tissues uses I-131 metaiodobenzyl guanidine (MIBG), a radioactive compound selectively taken up by adrenergic cells. 8 - 10% of the tumours are malignant. See: Sjoberg RJ, Simcic KJ, Kidd GS. The clonidine suppression test for pheochromocytoma. A review of its utility and pitfalls. Arch Intern Med. 1992 Jun;152(6):1193-7. Grossman E, Goldstein DS, Hoffman A, Keiser HR. Glucagon and clonidine testing in the diagnosis of pheochromocytoma. Hypertension. 1991 Jun;17(6 Pt 1):733-41

312

Number: 222 With regard to the Dandy-Walker syndrome: 1. The cerebellar vermis is hypoplastic. 2. Obstructive hydrocephalus is the commonest mode of presentation. 3. Presentation usually occurs within the 1st two years of life. 4. A cyst is always present in the posterior fossa A: 1,2,3 Correct B: 1,3 Correct C: 2,4 Correct D: 4 Correct E: All Correct ABCDE Correct Answer: E The syndrome comprises dilatation of the 4th ventricle, a posterior fossa cyst and cerebellar vermis agenesis. Presentation usually occurs in early childhood as obstructive hydrocephalus. Cone AM. Dandy-Walker syndrome. Anaesth. Intens. Care 1995; 23:613-615. Number: 251 In transfusion practice: 1. Fever is usually due to anti-leucocyte antibodies 2. Desmopressin (DDAVP) will raise levels of factor VIII in patients with mild haemophilia A 3. Haemolytic reactions may be delayed for up to 1 week 4. Severe anaphylaxis may be seen in IgA deficient individuals A: 1,2,3 Correct B: 1,3 Correct C: 2,4 Correct D: 4 Correct E: All Correct ABCDE Correct Answer: E In IgA deficient individuals IgA is a foreign body. Number: 252

313

Recognised causes of macrocytosis in the peripheral blood with normoblastic erythropoiesis in the bone marrow include: 1. Hypothyroidism 2. Chronic alcohol abuse 3. Cryptogenic cirrhosis 4. The administration of phenytoin A: 1,2,3 Correct B: 1,3 Correct C: 2,4 Correct D: 4 Correct E: All Correct ABCDE Correct Answer: A A number of drugs antagonize folate by mechanisms which are poorly understood but are thought to involve an effect on absorption of the vitamin by the intestine. In this category are the anticonvulsants phenytoin (Dilantin) and primidone (Mysoline) and phenobarbital (Luminal). Megaloblastic anemia induced by these agents is mild. Number: 253 Which are true of Hepatitis E virus? 1, It is a 34nm single stranded RNA virus. 2. It is more common in IV drug abusers 3. Can produce epidemic waterborne infections 4. Will cause 10% of patients to develop chronic hepatitis A: 1,2,3 Correct B: 1,3 Correct C: 2,4 Correct D: 4 Correct E: All Correct ABCDE Correct Answer: B A distinct type of waterborne "non-A, non-B" hepatitis has been identified in India, Asia, Africa, and Central America (previously labeled epidemic or enterically transmitted non-A, non-B hepatitis). This agent, with epidemiologic features resembling those of hepatitis A, has been classified as hepatitis E virus (HEV). Hepatitis E virus is a 32- to 34-nm, nonenveloped, HAV-like virus with a 7600-nucleotide, single-stranded RNA genome.

314

Number: 254 Which of the following are correctly paired? 1. Conn's syndrome and metabolic alkalosis 2. Uretero-colic anastomosis and hyperchloraemic acidosis 3. Shock and metabolic acidosis 4. Pancreatic fistula and metabolic acidosis A: 1,2,3 Correct B: 1,3 Correct C: 2,4 Correct D: 4 Correct E: All Correct ABCDE Correct Answer: E Conn's syndrome The majority of cases of Conn's syndrome involve a unilateral adenoma, usually small and occurring with equal frequency on either side. Rarely, primary aldosteronism occurs in association with adrenal carcinoma. It is twice as common in women as in men, occurs between the ages of 30 and 50, and is present in approximately 1 percent of unselected hypertensive patients. Many cases have clinical and biochemical features characteristic of primary aldosteronism, but a solitary adenoma is not found at surgery. Instead, these patients have bilateral cortical nodular hyperplasia. In the literature this disease has been alternatively termed "pseudo" primary aldosteronism, idiopathic hyperaldosteronism, or nodular hyperplasia. The cause is unknown. The continual hypersecretion of aldosterone increases the renal distal tubular exchange of intratubular sodium for secreted potassium and hydrogen ions, with progressive depletion of body potassium and development of hypokalemia. Most patients have diastolic hypertension, usually not of marked severity, and complain of headaches. The hypertension is probably due to the increased sodium reabsorption and extracellular volume expansion. Potassium depletion is responsible for the muscle weakness and fatigue and is related to the effect of potassium depletion on muscle membrane. The polyuria results from impairment of concentrating ability and is often associated with polydipsia. Number: 255 The following are recognised causes of pulmonary fibrosis: 1. External raditaion for carcinoma of the breast 2. Bleomycin 3. Sarcoidosis 4. Myelofibrosis A: 1,2,3 Correct B: 1,3 Correct

315

C: 2,4 Correct D: 4 Correct E: All Correct ABCDE Correct Answer: A Idiopathic Pulmonary Fibrosis (IPF) is a chronic inflammatory fibrotic disorder localized to the lower respiratory tract and characterized by an alveolitis dominated by alveolar macrophages and neutrophils and, to a lesser extent, lymphocytes and eosinophils. The disease usually presents as dyspnea on exertion, The three classic x-ray patterns of pulmonary sarcoidosis are type I--bilateral hilar adenopathy with no parenchymal abnormalities; type II--bilateral hilar adenopathy with diffuse parenchymal changes; and type III--diffuse parenchymal changes without hilar adenopathy. The type III pattern is sometimes split into two categories, with films that show fibrosis and upper lobe retraction classified separately. Although patients with type I x-rays tend to have the acute or subacute, reversible form of the disease while those with types II and III often have the chronic, progressive disease, these patterns do not represent consecutive "stages" of sarcoidosis. Number: 256 Recognised causes of finger clubbing include: 1. Carcinoid tumour 2. Bronchial carcinoma 3. Iron deficiency anaemia 4. Coeliac disease A: 1,2,3 Correct B: 1,3 Correct C: 2,4 Correct D: 4 Correct E: All Correct ABCDE Correct Answer: C The selective bullous enlargement of the distal segments of the fingers and toes due to proliferation of connective tissue, particularly on the dorsal surface, is termed clubbing; an increase occurs in the sponginess of the soft tissue at the base of the nail. Clubbing may be hereditary, idiopathic, or acquired and associated with a variety of disorders, including cyanotic congenital heart disease, infective endocarditis, and a variety of pulmonary conditions (among them primary and metastatic lung cancer, bronchiectasis, lung abscess, cystic fibrosis, and mesothelioma), as well as with some gastrointestinal diseases (including regional enteritis, chronic ulcerative colitis, and hepatic cirrhosis).

316

Clubbing in patients with primary and metastatic lung cancer, mesothelioma, bronchiectasis, and hepatic cirrhosis may be associated with hypertrophic osteoarthropathy. In this condition, the subperiosteal formation of new bone in the distal diaphyses of the long bones of the extremities causes pain and symmetric arthritis-like changes in the shoulders, knees, ankles, wrists, and elbows. The diagnosis of hypertrophic osteoarthropathy may be confirmed by bone radiographs and scans. Although the mechanism of clubbing is unclear, it appears to be secondary to a (presumably humoral) substance which causes dilation of the vessels of the fingertip. Number: 257 Dupuytren's contracture has a recognised association with: 1. Diabetes Mellitus 2. Peyronie's disease 3. Epilepsy 4. Alcoholic liver cirrhosis A: 1,2,3 Correct B: 1,3 Correct C: 2,4 Correct D: 4 Correct E: All Correct ABCDE Correct Answer: E Palmar erythema, facial telangiectasia, and Dupuytren's contractures are associated with cirrhosis, particularly as a result of chronic ethanol ingestion. An interesting accompaniment of insulin-dependent diabetes is the presence of joint contractures (Dupuytren's contracture) coupled with tight, waxy skin over the dorsum of the hands. The hands resemble those in patients with scleroderma. The cause of the tendon contractures is unknown, although alterations of cross-linking in collagen has been proposed. Number: 259 In the treatment of chronic hepatitis B virus: 1. Corticosteroids are always beneficial 2. 40% of patients can be expected to respond to interferon 3. Combined treatment with acyclovir and interferon is more effective than interferon alone 4. Interferon can be associated with a rise in transaminases towards the end of therapy A: 1,2,3 Correct B: 1,3 Correct

317

C: 2,4 Correct D: 4 Correct E: All Correct ABCDE Correct Answer: C Interferon a2b administered to patients with stable chronic hepatitis B infection resulted in loss of markers of HBV replication, such as HBeAg and HBV DNA, in 40 to 50 percent of patients who received daily doses of 5 million units for 16 weeks; 10 to 20 percent of patients also became HBsAg negative. More than 80 percent of patients who lose HBeAg and HBV DNA markers will have return of serum aminotransferases to normal levels, and both shortand long-term improvement in liver histopathology has been described. Predictors of a favorable response to therapy include low pretherapy HBV DNA levels, high pretherapy serum ALT levels, short duration of chronic hepatitis B infection, and active liver histopathology. Adverse effects of the preceding dose of interferon are common and include fever, chills, myalgia, and fatigue. Approximately 25 percent of patients receiving a daily dose of 5 million units will require dose reduction, but fewer than 5 percent will require discontinuation of therapy. Number: 260 Which of the following statements is / are true of streptococcus pyogenes infections: 1. They account for less than 5% of upper respiratory infections in children under the age of 2 years 2. Rheumatic chorea is a recognised sequela 3. It is a cause of erysipelas 4. The treatment of choice is ampicillin A: 1,2,3 Correct B: 1,3 Correct C: 2,4 Correct D: 4 Correct E: All Correct ABCDE Correct Answer: A Lancefield's group A consists of a single species, S. pyogenes. As its species name implies, this organism is associated with a variety of suppurative infections. In addition, group A streptococci are unique among known bacterial pathogens in their capacity to trigger the postinfectious syndromes of acute rheumatic fever and poststreptococcal glomerulonephritis.

318

Erysipelas is due to S. pyogenes and is characterized by an abrupt onset of fiery, red swelling of the face or extremities. Distinctive features are its well-defined margins, particularly along the nasolabial fold, rapid progression, and intense pain. Flaccid bullae may develop during the second or third day of illness, but extension to deeper soft tissues is rare. Treatment with penicillin is effective; swelling may progress despite appropriate treatment, though fever, pain, and the intense red color diminish. Desquamation of the involved skin occurs 5 to 10 days into the illness. Infants and elderly adults are most commonly afflicted, and the severity of systemic toxicity may vary. Chorea is a disorder of the central nervous system characterized by sudden, aimless, irregular movements, often accompanied by muscle weakness and emotional instability. Chorea is a delayed manifestation of rheumatic fever, and other manifestations may or may not still be present at the time it appears. Polyarthritis, when part of the same attack, almost always subsides before chorea appears. Carditis is often discovered for the first time when the presenting feature of rheumatic fever is chorea. Chorea usually appears after a long latent period (up to several months) from the antecedent streptococcal infection and at a time when all other manifestations of rheumatic fever have abated. Number: 262 The following may occur in uncomplicated haemolytic jaundice: 1. Bilirubinuria 2. High conjugated serum bilirubin 3. High serum alkaline phosphatase 4. Reticulocytosis A: 1,2,3 Correct B: 1,3 Correct C: 2,4 Correct D: 4 Correct E: All Correct ABCDE Correct Answer: D In many patients with haemolytic anemia, the diagnosis can be deduced from a careful history and physical examination. The patient often complains of fatigue and other symptoms of anemia. Less commonly, jaundice and even red-brown urine (haemoglobinuria) are reported. A complete drug history is often critical. The family history is especially helpful in the diagnosis of an inherited haemolytic anemia. Elevation of the reticulocyte count in the anemic patient is the single most useful indicator of haemolysis, reflecting erythroid hyperplasia of the bone marrow. Haemolysis produces unconjugated bilirubinaemia.

319

Number: 265 Nephrotoxicity is described with the following drugs: 1. Gentamicin 2. Acetazolamide 3. Ampicillin 4. Rifampicin A: 1,2,3 Correct B: 1,3 Correct C: 2,4 Correct D: 4 Correct E: All Correct ABCDE Correct Answer: E Nephrotoxic ARF may complicate administration of diverse pharmacologic agents. The kidney is particularly susceptible to nephrotoxic injury by virtue of its rich blood supply (25 percent of cardiac output) and ability to concentrate toxins in the medullary interstitium (via the renal countercurrent mechanism) and renal epithelial cells (via specific transporters). ARF complicates 10 to 30 percent of courses of aminoglycoside antibiotics. Aminoglycosides are filtered across the glomerular filtration barrier and accumulated by proximal tubule cells after interaction with phospholipid residues on brush border membranes. Aminoglycosides appear to disrupt normal processing of membrane phospholipids by lysosomes. Patients with nephrotoxicity due to ampicillin or rifampicin have interstitial inflammation and a glomerular lesion almost identical to minimal change glomerulonephritis. Number: 266 1. Hyperacute rejection is mediated by T-cells 2. Chronic rejection is immunoglobulin mediated 3. Early acute rejection is mediated by B-lymphocytes 4. Steroids are effective in reversing acute rejection of a donor organ A: 1,2,3 Correct B: 1,3 Correct C: 2,4 Correct D: 4 Correct E: All Correct ABCDE Correct Answer: C

320

Hyperacute rejection is mediated by preformed antibodies, Early acute rejection is mediated by T -lymphocytes. Number: 267 The following are recognised associations: 1. Ulcerative colitis and HLA B8 2. Primary sclerosing cholangitis and HLA B8 3. Haemochromatosis and HLA A3 4. Primary biliary cirrhosis and HLA DR3 A: 1,2,3 Correct B: 1,3 Correct C: 2,4 Correct D: 4 Correct E: All Correct ABCDE Correct Answer: C Primary or idiopathic sclerosing cholangitis is a disorder characterized by a progressive, inflammatory, sclerosing, and obliterative process affecting the extrahepatic and, often, the intrahepatic bile ducts. The lesion may appear as an isolated entity or may occur in association with inflammatory bowel disease, especially ulcerative colitis, or with multifocal fibrosclerosis syndromes such as retroperitoneal, mediastinal, and/or periureteral fibrosis, The cause of primary biliary cirrhosis remains unknown. Several observations suggest that a disordered immune response may be involved. Primary biliary cirrhosis (PBC) is frequently associated with a variety of disorders presumed to be autoimmune in nature, such as the CRST syndrome (calcinosis, Raynaud's phenomenon, sclerodactyly, telangiectasia), the sicca syndrome (dry eyes and dry mouth), autoimmune thyroiditis, and renal tubular acidosis. Number: 268 Pleural calcification is a recognised result of: 1. Tuberculosis 2. Chronic empyema 3. Asbestosis 4. Bagassosis A: 1,2,3 Correct B: 1,3 Correct C: 2,4 Correct D: 4 Correct E: All Correct

321

ABCDE Correct Answer: A Bagassosis is a hypersensitivity pneumonitis due to exposure to moldy sugar cane! Number: 269 The early manifestations of chronic inorganic lead poisoning include: 1. Absent knee reflexes 2, Punctate basophilia 3. Constipation 4. Abdominal colic A: 1,2,3 Correct B: 1,3 Correct C: 2,4 Correct D: 4 Correct E: All Correct ABCDE Correct Answer: C The toxic effects of lead differ between children and adults. The adult form is generally characterized by abdominal pain, anemia, renal disease, headache, peripheral neuropathy with demyelination of long neurons, ataxia, and memory loss. Symptoms are usually associated with prolonged elevation of lead levels above 4 to 5 umol/L (80 to 100 ug/dL) of whole blood. A subclinical form in adults affects primarily the peripheral nervous system and the kidneys. A linear association between hypertension and elevated lead levels (i.e., greater than 1.5 umol/L (30 ug/dL)) has been reported. Encephalopathy is rare in adults. Childhood lead poisoning is manifested by abdominal pain and anaemia, but the central nervous system effects are most important. As an enzymatic poison, lead affects developing tissues more than tissues with slow turnover. Hence subclinical lead poisoning is most dangerous to children because its effects emerge without associated symptoms that bring the victim to medical attention. In the acute clinical form, signs and symptoms reflect both the direct effect of high concentrations of lead (i.e., blood lead greater than 4 umol/L (80 ug/dL)) and consequent severe alterations in porphyrin synthesis. Signs and symptoms include abdominal pain and irritability, followed by lethargy, anorexia, pallor (anaemia), ataxia, and slurred speech. In severe cases, convulsions, coma, and death are usually due to severe generalized cerebral oedema and renal failure. A history of "high-dose" exposure to lead (usually paint chips), pica (the ingestion of nonfood substances), and malnutrition (iron, calcium, and zinc deficiency) almost always is associated with this syndrome.

322

Number: 270 Temporal lobe epilepsy is particularly associated with: 1. Dreamy states 2. Euphoria 3. An olfactory aura 4. Repetitive conjugate movements of the eyes A: 1,2,3 Correct B: 1,3 Correct C: 2,4 Correct D: 4 Correct E: All Correct ABCDE Correct Answer: B Temporal lobe epilepsy leads to changes in behavior in which an individual loses conscious contact with the environment. The onset of these seizures may consist of any of a variety of auras: an unusual smell (as of burning rubber), a feeling that the current experience has happened before (deja vu), a sudden intense emotional feeling, a sensory illusion such as that of objects growing smaller (micropsia) or larger (macropsia), or a specific formed sensory hallucination. Patients may come to recognize these as heralding their seizures, or the memory of the aura may be lost in the postictal amnesia that often occurs if the seizure becomes generalized. During complex partial seizures, there may be a cessation of activity with some minor motor activity, such as lip smacking, swallowing, walking aimlessly, or picking at one's clothes (automatisms). Complex partial seizures also may be accompanied by the unconscious performance of highly skilled activities such as driving a car or playing complicated musical pieces. When the seizure ends, the individual is amnesic for events which took place during the seizure and may take minutes or hours to recover full consciousness. Repetitive conjugate movements of the eyes are a feature of frontal lobe epilepsy. Number: 271 Cannon waves may be seen in the juglar veins in: 1. First degree AV heart block 2. Ventricular pacing 3. Tricuspid stenosis 4. Nodal tachycardia A: 1,2,3 Correct B: 1,3 Correct C: 2,4 Correct D: 4 Correct

323

E: All Correct ABCDE Correct Answer: C In the case of tricuspid stenosis there are tall 'a' waves but they are not true cannon waves. Number: 274 Infective endocarditis rarely occurs with: A. Congenital bicuspid aortic valve B. Atrial septal defect C. Patent ductus arteriosus D. Coarctation of the aorta E. Mitral valve prolapse Select the single best answer ABCDE Correct Answer: B In native valve endocarditis, the proportion of males is higher than females, and most patients are over age 50. Endocarditis is uncommon in children. Between 60 and 80 percent of patients have an identifiable predisposing cardiac lesion. Rheumatic valvular disease accounts for about 30 percent of cases. The mitral valve is most commonly involved, followed by the aortic. Right-sided endocarditis usually affects the tricuspid valve but is rare on rheumatic valves. Congenital heart disease other than mitral valve prolapse is the underlying lesion in about 10 to 20 percent of patients with endocarditis. Predisposing lesions include patent ductus arteriosus, ventricular septal defect, tetralogy of Fallot, coarctation of the aorta, pulmonary stenosis, and bicuspid aortic valve but not uncomplicated atrial septal defect. Mitral valve prolapse is the underlying lesion in about 10 to 33 percent of cases. Number: 276 Cerebral abscess in the absence of endocarditis is a recognised complication of: 1. Tetralogy of Fallot 2. Eisenmenger's Syndrome 3. Tricuspid artresia 4. Transposition of the great arteries A: 1,2,3 Correct B: 1,3 Correct

324

C: 2,4 Correct D: 4 Correct E: All Correct ABCDE Correct Answer: E Cerebral abscess can complicate any condition which is characterised by a right to left shunt. Number: 277 Nephrogenic diabetes insipidus: 1. Has an autosomal recessive inheritance 2. Can be associated with sickle cell anaemia 3. Is characterised by normal or low serum vasopressin levels 4. Can be caused by hypercalcaemia A: 1,2,3 Correct B: 1,3 Correct C: 2,4 Correct D: 4 Correct E: All Correct

ABCDE Correct Answer: C Diabetes insipidus must be distinguished from other types of hypotonic polyuria (primary polydipsia and nephrogenic diabetes insipidus) and from states of osmotic diuresis. Several are recognizable by the history (e.g., following lithium or mannitol administration, surgery under methoxyflurane anesthesia, or renal transplantation). In others the physical examination or simple laboratory procedures will indicate the diagnosis (evidence of glycosuria, renal disease, sickle cell anemia, hypercalcemia, or potassium depletion, including primary aldosteronism). Congenital nephrogenic diabetes insipidus is usually inherited as an X-linked recessive trait. Affected males are totally resistant to vasopressin, while heterozygote females are asymptomatic or have mild polyuria. In several families the abnormal gene is localized to the Xq28 region of the long arm of the X chromosome. Almost all these patients have a V2 receptor abnormality. Nephrogenic diabetes insipidus also may be inherited as an autosomal recessive trait or may occur sporadically. Females with sporadic disease appear to have a defect in the pathway of AVP action distal the V2 receptor, and some of them respond to

325

large doses of desmopressin. V1 receptor-mediated functions are normal in patients with congenital nephrogenic diabetes insipidus. When patients with nephrogenic and central diabetes insipidus cannot be differentiated by simpler means, documentation of elevated plasma or urinary AVP concentration in relation to plasma osmolality or of a high AVP concentration in relation to urine osmolality will allow the diagnosis of nephrogenic diabetes insipidus. Number: 278 Extrinsic allergic alveolitis: 1. Occurs only in atopic individuals. 2. Is associated with precipitin production to relevant antigens. 3. Characteristically causes an eosinophilia. 4. Results in a restrictive pattern of ventilatory abnormality with marked reduction of transfer factor. A: 1,2,3 Correct B: 1,3 Correct C: 2,4 Correct D: 4 Correct E: All Correct ABCDE Correct Answer: C Hypersensitivity pneumonitis, or extrinsic allergic alveolitis, is an immunologically induced inflammation of the lung parenchyma, involving alveolar walls and terminal airways, secondary to repeated inhalation of a variety of organic dusts and other agents by a susceptible host. In contrast to many of the other interstitial lung diseases, the cause of this interstitial and alveolar filling disease is known. Number: 279 Which of the following are associated with dissecting aneurysm of the aorta? 1. Coarctation of the aorta 2. Hypertension 3. Marfan's syndrome 4. Syphilitic aortitis A: 1,2,3 Correct B: 1,3 Correct C: 2,4 Correct D: 4 Correct E: All Correct

326

ABCDE Correct Answer: A The factors that predispose to aortic dissection include systemic hypertension, a coexisting condition in 70 percent of patients, and cystic medial necrosis. Aortic dissection is the major cause of morbidity and mortality in patients with the Marfan syndrome. The incidence is also increased in patients with congenital aortic valve anomalies (e.g., bicuspid valve), in those with coarctation of the aorta, and in otherwise normal women during the third trimester of pregnancy. Patients with syphilitic aortitis are prone to the development of aortic aneurysm, but not dissection. Number: 281 Lung complications may result from the ingestion of: 1. Paraquat 2. Busulphan 3. Bleomycin 4. Aspirin A: 1,2,3 Correct B: 1,3 Correct C: 2,4 Correct D: 4 Correct E: All Correct ABCDE Correct Answer: E Paraquat causes pulmonary oedema, Busulphan - pulmonary fibrosis, Bleomycin pneumonitis and fibrosis, Aspirin - bronchospasm, Number: 283 In coarctation of the aorta: 1. Rupture of the aorta is a recognised complication 2. Congestive failure is usually due to hypertension 3. Cerebrovascular haemorrhage is a known hazard 4. Infective endocarditis of the bicuspid valves is a very rare occurrence A: 1,2,3 Correct

327

B: 1,3 Correct C: 2,4 Correct D: 4 Correct E: All Correct ABCDE Correct Answer: B Most children and young adults with isolated, discrete coarctation are asymptomatic. Headache, epistaxis, cold extremities, and claudication with exercise may occur, and attention is usually directed to the cardiovascular system when a heart murmur or hypertension in the upper extremities and absence, marked diminution, or delayed pulsations in the femoral arteries are detected on physical examination. Enlarged and pulsatile collateral vessels may be palpated in the intercostal spaces anteriorly, in the axillae, or posteriorly in the interscapular area. The upper extremities and thorax may be more developed than the lower extremities. A midsystolic murmur over the anterior part of the chest, back, and spinous processes may become continuous if the lumen is narrowed sufficiently to result in a high-velocity jet across the lesion throughout the cardiac cycle. Additional systolic and continuous murmurs over the lateral thoracic wall may reflect increased flow through dilated and tortuous collateral vessels. The electrocardiogram reveals left ventricular hypertrophy of varying degree. Roentgenograms may show a dilated left subclavian artery high on the left mediastinal border and a dilated ascending aorta. Indentation of the aorta at the site of coarctation and pre- and poststenotic dilatation (the "3" sign) along the left paramediastinal shadow are almost pathognomonic. Notching of the ribs, an important radiographic sign, is due to erosion by dilated collateral vessels. Two-dimensional echocardiography from para- or suprasternal windows identifies the site and length of coarctation, while Doppler studies record and quantify the pressure gradient. Transesophageal echocardiography and magnetic resonance imaging or digital angiography allow visualization of the length and severity of the obstruction and the associated collateral arteries. In adults, cardiac catheterization is indicated primarily to evaluate the coronary arteries. The chief hazards result from severe hypertension and include the development of cerebral aneurysms and hemorrhage, rupture of the aorta, left ventricular failure, and infective endocarditis. Number: 285 Hypothyroidism: 1. Is more common than hyperthyroidism 2. Is a cause of pericardial effusion 3. Is associated with low serum carotene 4. May present with cerebellar ataxia A: 1,2,3 Correct B: 1,3 Correct C: 2,4 Correct

328

D: 4 Correct E: All Correct ABCDE Correct Answer: C Hyperthyroidism is about 8 times more common than Hypothyroidism. Serum carotene levels are elevated in hypothyroidism. Hypothyroidism may be primary or secondary. Low thyroid hormone and elevated serum TSH concentrations are characteristic of primary hypothyroidism. Low thyroid hormone concentrations with a "normal" or "low" TSH concentration are found in secondary hypothyroidism. Number: 286 A patient suffering from dystrophia myotonica is likely to suffer from: A. Progressive external ophthalmoplegia. B. Cataracts. C. Symptoms that begin in childhood. D. Fasiculations presenting in childhood. E. Tendon reflexes that are retained despite muscle wasting. Select the single best answer ABCDE Correct Answer: B This condition was originally described by Steinert in 1909 and represents the most common adult muscular dystrophy. This disorder has an incidence of 13.5 per 100,000 live births and involves an equal proportion of males and females. Cardiac disturbances occur in the majority of patients with myotonic dystrophy. Electrocardiographic abnormalities are common, including first degree heart block or more extensive conduction system involvement. Complete heart block and sudden death can occur. Congestive heart failure occurs infrequently, but may result from cor pulmonale secondary to respiratory failure. Mitral valve prolapse also occurs commonly in myotonic dystrophy patients. Other features associated with myotonic dystrophy include intellectual impairment, hypersomnia, posterior subcapsular cataracts, gonadal atrophy, insulin resistance and decreased esophageal and colonic motility. Number: 286 A patient suffering from dystrophia myotonica is likely to suffer from: A. Progressive external ophthalmoplegia.

329

B. Cataracts. C. Symptoms that begin in childhood. D. Fasiculations presenting in childhood. E. Tendon reflexes that are retained despite muscle wasting. Select the single best answer ABCDE Correct Answer: B This condition was originally described by Steinert in 1909 and represents the most common adult muscular dystrophy. This disorder has an incidence of 13.5 per 100,000 live births and involves an equal proportion of males and females. Cardiac disturbances occur in the majority of patients with myotonic dystrophy. Electrocardiographic abnormalities are common, including first degree heart block or more extensive conduction system involvement. Complete heart block and sudden death can occur. Congestive heart failure occurs infrequently, but may result from cor pulmonale secondary to respiratory failure. Mitral valve prolapse also occurs commonly in myotonic dystrophy patients. Other features associated with myotonic dystrophy include intellectual impairment, hypersomnia, posterior subcapsular cataracts, gonadal atrophy, insulin resistance and decreased esophageal and colonic motility. Number: 290 Proliferative glomerulonephritis: 1. May present as a nephrotic syndrome 2. May present with haematuria + convulsions 3. Is the lesion which can occur in Henoch-Schnlein syndrome 4. In childhood can be expected to respond well to steroids A: 1,2,3 Correct B: 1,3 Correct C: 2,4 Correct D: 4 Correct E: All Correct ABCDE Correct Answer: A Proliferative glomerulonephritis is a heterogeneous group of glomerular diseases. Some patients with this morphologic lesion may in fact represent resolving postinfectious glomerulonephritis, hereditary nephritis, or other multisystem diseases such as HenochSchonlein purpura, vasculitis, or systemic lupus erythematosus.

330

This lesion accounts for approximately 5 percent of idiopathic nephrotic syndrome in adults and 5 to 10 percent in children. It is more common in older children and young adults. Males are affected slightly more often than females. Hematuria, either gross or microscopic, is common. The childhood form of the disease tends to be unresponsive to steroids. Number: 291 Pneumococcal meningitis: 1. Has its peak age incidence in childhood 2. Is a recognised late sequel to splenectomy in children 3. Should be treated with 100 000 units benzyl penicillin intrathecally daily for seven days 4. Can be effectively treated with chloramphenicol in patients hypersensitive to penicillin A: 1,2,3 Correct B: 1,3 Correct C: 2,4 Correct D: 4 Correct E: All Correct ABCDE Correct Answer: C Pneumococcal meningitis is the most frequently observed agent in adults over the age of 30 years and accounts for approximately 15 percent of the total cases of meningitis in the United States. Mortality rates remain high, in the 19 to 30 percent range. Pneumococcal infection of the meninges is often associated with distant foci such as pneumonia, otitis media, mastoiditis, sinusitis, or endocarditis. Serious pneumococcal infections may be observed in patients with predisposing conditions, including splenectomy or asplenic states, multiple myeloma, hypogammaglobulinemia, alcoholism, cirrhosis, and the Wiskott-Aldrich syndrome. S. pneumoniae is the most common meningeal isolate in head trauma patients who have suffered a basilar skull fracture with subsequent CSF leak. Number: 292 Legionnaire's disease: 1. May present with diarrhoea 2. Is associated with leucopenia 3. Should be treated with erythromycin 4. Causes microscopic haematuria in 10% patients A: 1,2,3 Correct B: 1,3 Correct

331

C: 2,4 Correct D: 4 Correct E: All Correct ABCDE Correct Answer: B Legionnaires' disease refers to an epidemic of pneumonia that affected 221 people and caused 34 deaths during the American Legion Convention at the Bellevue-Stratford Hotel in Philadelphia during July and August 1976. Initially referred to as the Legionnaires' disease agent, the organism was shown to be a new species of bacterium and subsequently designated L. pneumophila. Pneumonic illness typically begins with an abrupt prodrome of malaise, headache, myalgia, and weakness. Fever and intermittent rigors appear 24 h later, with temperatures exceeding 40 degC in more than half of patients. Nonproductive cough is common. About half of patients eventually produce thin or minimally purulent sputum, and one-third may have scant hemoptysis. Pleuritic chest pain and dyspnea can raise the suspicion of pulmonary embolism. Gastrointestinal symptoms include diarrhea, nausea, vomiting, and abdominal pain. Altered mental status suggesting toxic encephalopathy may include confusion, disorientation, lethargy, hallucinations, depression, delirium, obtundation, or coma. Seizures are rare, but cranial or peripheral neuropathy and cerebellar dysfunction are not uncommon. Physical examination usually shows a toxic appearance and high fever. Relative bradycardia is common. Lung examination reveals rales and consolidation, but the physical findings are mild when compared to radiographic findings. Complications and systemic manifestations include lung abscess, empyema, respiratory failure, hypotension, shock, rhabdomyolysis, disseminated intravascular coagulation (DIC), thrombotic thrombocytopenic purpura (TTP), and renal failure. Pontiac fever is an acute, self-limited illness lasting 2 to 5 days. A prodrome of malaise, myalgia, and headache is followed rapidly by fever, chills, and, variably, cough, coryza, and sore throat. Diarrhea, nausea, and mild neurologic symptoms such as dizziness or photophobia may be present. Number: 293 Which of the following are true of Hepatitis C virus: 1. It accounts for >90% of post transfusional hepatitis 2. Less than 10% of patients will progress to chronic liver disease 3. 50% of patients treated with interferon will relapse on stopping treatment 4. It has an incubation period of less than 6 weeks A: 1,2,3 Correct B: 1,3 Correct C: 2,4 Correct D: 4 Correct E: All Correct

332

ABCDE Correct Answer: B Routine screening of blood donors for HBsAg and the elimination of commercial blood sources in the early 1970s reduced the frequency of, but did not eliminate, transfusionassociated hepatitis. During the 1970s, the likelihood of acquiring hepatitis after transfusion of voluntarily donated, HBsAg-screened blood was approximately 10 percent per patient (up to 0.9 percent per unit transfused). Although hepatitis B accounted for up to 5 to 10 percent of these cases, the remaining 90 to 95 percent were classified, based on serologic exclusion, as "non-A, non-B" hepatitis. Although many of these patients have no symptoms and a nonprogressive course, ultimately, cirrhosis develops in as many as 20 percent of those with chronic posttransfusion hepatitis C within 10 years of acute illness. The likelihood of chronic hepatitis is also approximately 50 percent after sporadic hepatitis C occurring in the absence of identifiable percutaneous inoculation with blood products or contaminated needles. Number: 294 Pulsus paradoxus is found with: 1. Severe asthmatic attack 2. Severe left ventricular failure 3. Constrictive pericarditis 4. Cardiac amyloidosis A: 1,2,3 Correct B: 1,3 Correct C: 2,4 Correct D: 4 Correct E: All Correct ABCDE Correct Answer: E Arterial 'paradox' is, in fact, an exaggeration of the normal blood pressure response to respiration - in contrast to venous paradox which is truly paradoxical. Number: 295 Renal papillary necrosis may be due to: 1. Obstructive uropathy

333

2. Blackwater fever 3. Phenacitin abuse 4. Polycystic kidneys A: 1,2,3 Correct B: 1,3 Correct C: 2,4 Correct D: 4 Correct E: All Correct ABCDE Correct Answer: B When infection of the renal pyramids develops in association with vascular diseases of the kidney or with urinary tract obstruction, renal papillary necrosis is likely to result. Patients with diabetes, sickle cell disease, chronic alcoholism, and vascular disease seem peculiarly susceptible to this complication. Hematuria, pain in the flank or abdomen, and chills and fever are the most common presenting symptoms. Individuals who ingest large quantities of analgesic drugs are prone to develop tubulointerstitial damage and papillary necrosis. Indeed, in Australia, Switzerland, and Sweden, analgesic abuse is one of the most common causes of chronic renal failure, and it is an important cause in the United States as well. While aspirin, phenacetin, and acetaminophen (the metabolite of phenacetin) may alone or in combination induce chronic renal disease, epidemiologic studies incriminate phenacetin and acetaminophen as the more injurious combination. Chronic ingestion of aspirin without these other compounds seems to be an uncommon cause of serious renal damage. Number: 298 The following are features of ulnar nerve entrapment at the elbow: 1. It occurs more commonly in women 2. Weakness of grip 3. Symptoms and signs are more obvious when the elbow is flexed 4. Wasting of the thenar eminence A: 1,2,3 Correct B: 1,3 Correct C: 2,4 Correct D: 4 Correct E: All Correct ABCDE Correct Answer: B

334

Complete ulnar paralysis results in a characteristic claw-hand deformity owing to wasting and weakness of many of the small hand muscles and hyperextension of the fingers at the metacarpophalangeal joints and flexion at the interphalangeal joints. The flexion deformity is most pronounced in the fourth and fifth fingers. Sensory loss occurs over the fifth finger, the ulnar aspect of the fourth finger, and the ulnar border of the palm. The superficial location of the nerve at the elbow makes it a common site of pressure palsy. The ulnar nerve may also become entrapped just distal to the elbow in the cubital tunnel formed by the aponeurotic arch linking the two heads of the flexor carpi ulnaris. Also, prolonged pressure on the base of the palm, as occurs with use of hand tools or bicycle riding, may result in damage to the deep palmar branch of the ulnar nerve, causing weakness of the small hand muscles but no sensory loss. The most common upper extremity injuries that result from malpositioning are ulnar damage, caused by direct pressure on the nerve at the elbow, and brachial plexus injury, caused by stretching of the plexus. Number: 306 The following disorders are recognised sequelae of long-continued abuse of alcohol: 1. Macrocytosis 2. Hypertriglyceridiaemia 3. Atrial fibrillation 4. Pancreatic carcinoma A: 1,2,3 Correct B: 1,3 Correct C: 2,4 Correct D: 4 Correct E: All Correct ABCDE Correct Answer: A Macrocytosis may be encountered in patients with alcoholism. However, the red cell precursors in the bone marrow do not appear megaloblastic. In any individual, the daily consumption of large amounts of ethanol can produce a mild, asymptomatic elevation in the plasma triglyceride level due to an elevation of VLDL. However, in a subgroup ethanol ingestion regularly produces massive and clinically significant hyperlipidemia with elevations in both VLDL and chylomicrons (type 5 lipoprotein pattern). In most of this group, the VLDL level remains mildly elevated (type 4 lipoprotein pattern), even in the basal state after recovery from the severe alcoholic hyperlipidemia. This suggests that these individuals have a form of familial hypertriglyceridemia or multiple lipoprotein-type hyperlipidemia that is exacerbated and converted to a type 5 pattern by the ethanol ingestion.

335

Alcoholic cardiotoxicity may present in individuals without overt heart failure and consists of recurrent supraventricular or ventricular tachyarrhythmias. Termed the "holiday heart syndrome," it typically appears after a drinking binge; atrial fibrillation is seen most frequently, followed by atrial flutter and ventricular premature depolarizations. A large case control study has correlated chronic pancreatitis with an increased risk of pancreatic cancer, but there are no convincing data to link epidemiologic factors such as alcohol abuse. Number: 307 Carcinoma of the lung may present with: 1. Pain in the ulnar distribution of the arm. 2. Horner's Syndrome. 3. Hypercalcaemia. 4. Thrombophlebitis A: 1,2,3 Correct B: 1,3 Correct C: 2,4 Correct D: 4 Correct E: All Correct ABCDE Correct Answer: E Pancoast's (or superior sulcus tumor) syndrome results from local extension of a tumor (usually epidermoid) growing in the apex of the lung with involvement of the eighth cervical and first and second thoracic nerves, with shoulder pain which characteristically radiates in the ulnar distribution of the arm, often with radiologic destruction of the first and second ribs. Sympathetic nerve paralysis with Horner's syndrome (enophthalmus, ptosis, miosis, and ipsilateral loss of sweat) occurs as a result of regional spread of tumor in the thorax. Endocrine syndromes are seen in 12 percent of patients with carcinoma of the lung and include hypercalcemia and hypophosphatemia resulting from ectopic parathyroid hormone or PTH-related peptide production by epidermoid cancer. Thrombotic manifestations occur in 1 to 8 percent of patients and include migratory venous thrombophlebitis (Trousseau's syndrome), Number: 314 Hypercalcaemia is associated with:

336

1. Sarcoidosis 2. Diabetes Insipidus 3. Zollinger-Ellison Syndrome. 4. Myxoedema A: 1,2,3 Correct B: 1,3 Correct C: 2,4 Correct D: 4 Correct E: All Correct ABCDE Correct Answer: A In sarcoidosis hypercalcemia is directly related to an increased intestinal calcium absorption. This occurs because normal relations between 25(OH)D and the active metabolite 1,25(OH)2D, are not maintained. Hypercalcemia and hypokalemic nephropathy are reversible causes of nephrogenic diabetes insipidus. Thyrotoxicosis is associated with hypercalcaemia - The hypercalcemia seems due to increased bone turnover, with bone resorption exceeding bone formation. In 1955, Zollinger and Ellison described the syndrome that bears their names, which consists of ulcer disease of the upper gastrointestinal tract, marked increases in gastric acid secretion, and non-beta islet cell tumors of the pancreas. Between one-fourth and one-half of gastrinomas occur in association with the MEN 1 syndrome. Hyperparathyroidism is the most common component of MEN 1 and occurs in about 80 percent of patients with this form of Zollinger-Ellison syndrome. Number: 318 Antimicrobial agents effective against pseudomonas aeruginosa infections include 1. Metronidazole 2. Gentamicin 3. Cephalexin 4. Carbenicillin A: 1,2,3 Correct B: 1,3 Correct C: 2,4 Correct D: 4 Correct E: All Correct

337

ABCDE Correct Answer: C Pseudomonas species are ubiquitous, free-living, opportunistic gram-negative pathogens. P. aeruginosa, is the most common human pathogen in this group. In general, the choice of antibiotics with antipseudomonal activity includes the aminoglycosides (e.g., gentamicin, tobramycin, netilmicin, amikacin), selected thirdgeneration cephalosporins (e.g., ceftazidime, cefoperazone), selected extended-spectrum penicillins (e.g., carbenicillin, ticarcillin, ticarcillin/clavulanate, piperacillin, mezlocillin, azlocillin), carbapenem (imipenem), monobactams (e.g., aztreonam), and fluoroquinolones (e.g., ciprofloxacin, ofloxacin). Number: 321 A broad complex tachycardia is more likely to be supraventricular tachycardia with aberrant conduction than ventricular tachycardia if: A. Cannon waves are seen in the neck waves. B. Fusion beats are seen on the ECG. C. The tachycardia is abolished by carotid massage. D. There is a concordant pattern across the precordial leads. E. The QRS duration is > 160 msec. Select the single best answer ABCDE Correct Answer: C Characteristics of the ECG during the tachycardia that suggest a ventricular origin for the arrhythmia are: (1) A QRS complex > 140 msec in the absence of antiarrhythmic therapy, (2) AV dissociation (with or without fusion or captured beats) or variable retrograde conduction, (3) A superior QRS axis in the presence of a right bundle branch block pattern, (4) Concordance of the QRS pattern in all precordial leads (i.e., all positive or all negative deflections), and (5) Other QRS patterns with prolonged duration that are inconsistent with typical right or left bundle branch block patterns. A wide, complex, bizarre tachycardia that is very irregular suggests AF with conduction over an AV bypass tract. Number: 327 Mycoplasma pneumoniae:

338

1. Infection is associated with the development of agglutinins to a non-haemolytic streptococcus 2. Predominantly causes infection in the elderly 3. Infection is associated with the Stevens-Johnson syndrome 4. Infection is associated with a polymorphonuclear leucocytosis A: 1,2,3 Correct B: 1,3 Correct C: 2,4 Correct D: 4 Correct E: All Correct ABCDE Correct Answer: B Mycoplasmas are ubiquitous in nature and cause a wide variety of diseases among animals, birds, plants, and insects. In humans the most important pathogen is Mycoplasma pneumoniae, a common cause of respiratory tract infections. M. pneumoniae produces an influenza-like respiratory illness of gradual onset with headache, malaise, fever, and cough. When pneumonia is present, physical findings may be minimal despite extensive changes seen in chest x-rays. Synonyms include Eaton agent pneumonia, cold haemagglutinin-positive pneumonia, atypical or primary atypical pneumonia, and "walking" pneumonia. The organism has been implicated in up to 50 percent of pneumonia episodes in college students and in 20 to 30 percent of cases occurring in military recruits. Nondescript maculopapular skin rashes are frequent in children, and M. pneumoniae infections have been associated with erythema multiforme and the Stevens-Johnson syndrome. Number: 328 In Polycythaemia Rubra Vera (PRV): 1. Serum erythropoietin is low. 2. Serum iron is characteristically raised. 3. Hepatic vein thrombosis may occur. 4. The platelet count is typically low. A: 1,2,3 Correct B: 1,3 Correct C: 2,4 Correct D: 4 Correct E: All Correct ABCDE

339

Correct Answer: B Typically, patients present with an elevated haemoglobin concentration and haematocrit associated with thrombocytosis, leukocytosis, and splenomegaly. Determination of the serum erythropoietin concentration is important in distinguishing primary from secondary causes of polycythaemia. An elevated erythropoietin concentration suggests secondary erythrocytosis, while a low level is compatible with PRV. Polycythaemia rubra vera is a cause of thrombosis of the hepatic veins (Budd-Chiari syndrome). In PRV, the neutrophil alkaline phosphatase score is frequently increased, as is the serum vitamin B12 level and serum vitamin B12-binding capacity. Number: 329 The Bain Circuit: 1. Requires a fresh gas flow of about 3 times the minute ventilation to prevent rebreathing during SV. 2. Requires a lower fresh gas flow than the Mapleson A circuit to prevent rebreathing during IPPV. 3. Will maintain a constant PCO2 in the presence of a changing MV during IPPV. 4. Is more effective than than the Mapleson A circuit at conserving moisture during SV. Where SV= Spontaneous Ventilation, IPPV = Intermittent Positive Pressure Ventilation and MV = Minute ventilation. A: 1,2,3 Correct B: 1,3 Correct C: 2,4 Correct D: 4 Correct E: All Correct ABCDE Correct Answer: A For the Bain circuit, the patient's PCO2 during controlled positive-pressure ventilation depends mainly on the fresh gas flow. During spontaneous breathing the Bain circuit requires higher fresh gas flow than the Mapleson A circuit to ensure no rebreathing; however, it is the most efficient circuit for CO2 clearance during positive-pressure ventilation. The Mapleson A circuit is the more efficient circuit for conserving moisture because the anatomical deadspace from the previous breath (fully saturated) is entirely rebreathed.

340

Bain JA, Spoerel WE: A streamlined anaesthetic system, Can Anaesth Soc J 19:426, 1972. Bain JA, Spoerel WE: Flow requirements for a modified Mapleson D system during controlled ventilation, Can Anaesth Soc J 20:629, 1973. Number: 331 A carotid bruit in an asymptomatic thirty year old is best managed by: A. Telling the patient to 'Forget about it' and recommending no further follow up. B. Carotid Angiography. C. Carotid Doppler studies. D. Annual review of the patient. E. Transthoracic Echocardiography. Select the single best answer ABCDE Correct Answer: A A cervical bruit is not necessarily indicative of carotid stenosis. Hammond and Eisenger examined 1000 normal patients for the presence of bruit. Patients under age 5 years had a bruit in 87% of cases, and patients aged 30 to 34 had bruit in 22% of cases. Jones reported that a cervical venous hum could be readily perceived in 27% of subjects with an average age of 32 years. The differential diagnosis also includes radiated cardiac murmurs. Hammond JH, Eisinger RP: Carotid bruits in 1,000 normal subjects, Arch Intern Med 109:109, 1962. Jones FL: Frequency characteristics and importance of the cervical venous hum in adults, N Engl J Med 267(13):658, 1962. Number: 333 Signs of posterior inferior cerebellar artery thrombosis include: 1. Ipsilateral 5th nerve sensory loss 2. Nystagmus to the side of the lesion 3. Contralateral loss of pain in limbs and trunk 4. Bulbar palsy A: 1,2,3 Correct B: 1,3 Correct C: 2,4 Correct D: 4 Correct

341

E: All Correct ABCDE Correct Answer: E The posterior inferior cerebellar artery in its proximal segments supplies the lateral medulla and, in its distal branches, the inferior surface of the cerebellum. The 'lateral medullary syndrome' may include: On side of lesion: Pain, numbness, impaired sensation over half the face. Ataxia of limbs, falling to side of lesion. Nystagmus, diplopia, oscillopsia, vertigo, nausea, vomiting. Horner's syndrome Dysphagia, hoarseness, paralysis of palate, paralysis of vocal cord, diminished gag reflex. Loss of taste. Numbness of ipsilateral arm, trunk, or leg On side opposite lesion: Impaired pain and thermal sense over half the body, sometimes face Number: 340 Haemochromatosis is associated with: 1. Cirrhosis 2. Diabetes mellitus 3. Cardiomyopathy 4. Hypogonadotrophic hypogonadism A: 1,2,3 Correct B: 1,3 Correct C: 2,4 Correct D: 4 Correct E: All Correct ABCDE Correct Answer: E This relatively common genetically determined disorder involves accumulation of abnormal amounts of iron due to inappropriate absorption in the intestine. The liver, as a primary site of iron storage, is affected most directly. There is diffuse deposition of excess iron in hepatocytes, in contrast to the characteristic accumulation of iron in the reticuloendothelial compartment typical of secondary iron overload and hemosiderosis. Cirrhosis, diabetes mellitus, arthritis, cardiomyopathy, and hypogonadotrophic hypogonadism are the usual manifestations of haemochromatosis.

342

Number: 344 Eaton-Lambert syndrome can be distinguished from Myasthesia Gravis by: 1. The response to exercise. 2. The response to succinyl choline. 3. The response to anticholinesterases. 4. The response to calcium channel blockers. A: 1,2,3 Correct B: 1,3 Correct C: 2,4 Correct D: 4 Correct E: All Correct ABCDE Correct Answer: E Eaton-Lambert syndrome (ELS) is a condition that results from an association between carcinomatous conditions, especially small cell carcinoma of the lungs, and motor neuropathy. Clinically, it resembles Myasthesia Gravis (MG). Patients complain of weakness of the proximal muscles that, unlike in patients with MG, improves with exercise. The sera of these patients contain IgG antibodies that are directed against voltage-dependent calcium channels. The cells of small cell carcinoma of the lung and other malignancies possess calcium channels that may serve as the antigenic stimulus for the production of these antibodies. The destruction of the presynaptic calcium channels at the nerve terminal by immune-mediated complement activation results in decreased release of acetylcholine. Patients with ELS have been reported to be excessively sensitive to calcium channel blockers. Patients with ELS symptoms do not respond well to anticholinesterases. The symptoms are improved by agents such as 4-aminopyridine, guanidine, and germine that increase repetitive nerve firing and therefore increase the synaptic release of acetylcholine. Patients with ELS are extremely sensitive to the effects of all muscle relaxants, both depolarisers and nondepolarisers. In this respect, they are different from patients with MG, who are usually resistant to the effect of succinylcholine. Number: 362 Which of the following is the LEAST likely cause of massive haemoptysis: A. Tuberculosis B. Bronchiectasis C. Emphysema D. Lung abscess E. Carcinoma

343

Select the single best answer ABCDE Correct Answer: C The common causes of massive haemoptysis include cavitary tuberculosis, anaerobic lung abscess, lung cancer and bronchiectasis. The risk of mortality is far greater than when the cause of the massive haemoptysis is bronchitis or bronchiectasis. Increasingly, the efficacy of bronchial arterial embolisation in the treatment of this condition is being recognised. See: Witt Ch, Schmidt B, Geisler A, Borges AC, John M, Fietze I, Romaniuk P. Value of bronchial artery embolisation with platinum coils in tumorous pulmonary bleeding. Eur J Cancer. 2000 Oct;36(15):1949-54. Number: 375 Recognised complications of chronic renal failure include: 1. Sensory-motor neuropathy 2. Pruritis 3. Metastatic calcification 4. Proximal myopathy A: 1,2,3 Correct B: 1,3 Correct C: 2,4 Correct D: 4 Correct E: All Correct ABCDE Correct Answer: E Peripheral neuropathy is common in advanced CRF. Initially, sensory nerve involvement exceeds motor, lower extremities are involved more than the upper, and the distal portions of the extremities more than proximal. The "restless legs syndrome" is characterized by illdefined sensations of discomfort in the feet and lower legs and frequent leg movement. In CRF there is a tendency to extraosseous, or metastatic, calcification when the calciumphosphate product is very high.

344

The clinical picture of the myopathy of chronic renal failure is identical to that of primary hyperparathyroidism and osteomalacia. There is proximal limb weakness with bone pain. Number: 377 Complications of dialysis-dependent renal failure can include: 1. Arthropathy 2. Dementia 3. Accelerated artherosclerosis 4. Raised intracranial pressure A: 1,2,3 Correct B: 1,3 Correct C: 2,4 Correct D: 4 Correct E: All Correct ABCDE Correct Answer: E 1 - If renal failure is due to amyloidosis, an arthropathy can complicate the clinical picture. Amyloid can directly involve articular structures by its presence in the synovial membrane and synovial fluid or in the articular cartilage. Amyloid arthritis can mimic a number of rheumatic diseases because it can present as a symmetric arthritis of small joints with nodules, morning stiffness, and fatigue. 2 - Characterised by speech dyspraxia, myoclonus, dementia and siezures. Thought to be due to aluminium overload. 3 - There are many reasons such as hypertension, hyperlipidaemia, glucose intolerance, chronic high cardiac output and metastatic calcification. 4. - Dialysis dysequilibrium - from lowering serum urea too quickly. Mannitol can be given during the first few haemodialysis sessions when the urea is particularly high. Number: 378 Circulating anticoagulants have been described in: 1. Systemic Lupus Erythematosus. 2. Factor VIII deficient patients who have received plasma transfusions. 3. Otherwise normal post-partum females. 4. Elderly individuals.

345

A: 1,2,3 Correct B: 1,3 Correct C: 2,4 Correct D: 4 Correct E: All Correct ABCDE Correct Answer: E Circulating anticoagulants, or inhibitors, are usually IgG antibodies which interfere with coagulation reactions. Specific inhibitors inactivate individual coagulation proteins and may cause severe hemorrhage. As discussed above, they arise in 15 to 20 percent of patients with factor VIII or factor IX deficiency who have received plasma infusions. Specific inhibitors also occur in previously normal individuals. Although the most common target protein is factor VIII, inhibitors have been described with specificity for each of the coagulation proteins. In addition to hemophiliacs, anti-factor VIII antibodies are seen in postpartum females, in patients on various drugs, as part of the spectrum of autoantibodies in systemic lupus erythematosus patients, and in normal elderly individuals. Circulating anticoagulants also have been reported in patients with AIDS. Number: 380 The manifestations of lithium toxicity include: 1. Nephrogenic diabetes insipidus. 2. Seizures. 3. Hyperthermia. 4. Prolonged QT interval. A: 1,2,3 Correct B: 1,3 Correct C: 2,4 Correct D: 4 Correct E: All Correct ABCDE Correct Answer: E The manifestations of lithium toxicity include gastrointestinal effects such as nausea, vomiting, and diarrhoea; CNS effects including ataxia, tremor, myoclonus, seizures, confusion, and coma; and cardiovascular effects including hypotension, atrioventricular block, and prolonged QT interval. Hyperthermia and nephrogenic diabetes insipidus may also occur.

346

Number: 384 A 70 year old male is found to have a WBC count of 32,000/microliter with differential count of 8 segmented polymorphs, 1 band, 88 lymphocytes, and 4 monocytes. His haemoglobin is 122 g/l, hct 37.1%, and platelet count 178,000/microliter. His peripheral blood smear is shown here. The most likely diagnosis is: A. Infectious mononucleosis B. Chronic lymphocytic leukemia C. Acute myelogenous leukemia D. Hodgkin's disease E. Multiple myeloma Select the single best answer Number: 384 A 70 year old male is found to have a WBC count of 32,000/microliter with differential count of 8 segmented polymorphs, 1 band, 88 lymphocytes, and 4 monocytes. His haemoglobin is 122 g/l, hct 37.1%, and platelet count 178,000/microliter. His peripheral blood smear is shown here. The most likely diagnosis is: A. Infectious mononucleosis B. Chronic lymphocytic leukemia C. Acute myelogenous leukemia D. Hodgkin's disease E. Multiple myeloma Select the single best answer ABCDE Correct Answer: B Chronic lymphocytic leukemia (CLL) is a neoplasm of activated B lymphocytes. The CLL cells, which morphologically resemble mature, small lymphocytes of the peripheral blood, accumulate in the bone marrow, blood, lymph nodes, and spleen in large numbers. The disease is usually seen in patients over 50 years of age, but improved diagnosis has identified many younger patients. CLL is the most common form of leukemia in the Australia and is more frequent in males than females. The CLL cells commonly have trisomy 12 alone or with additional chromosomal abnormalities.

347

ABCDE Correct Answer: B Chronic lymphocytic leukemia (CLL) is a neoplasm of activated B lymphocytes. The CLL cells, which morphologically resemble mature, small lymphocytes of the peripheral blood, accumulate in the bone marrow, blood, lymph nodes, and spleen in large numbers. The disease is usually seen in patients over 50 years of age, but improved diagnosis has identified many younger patients. CLL is the most common form of leukemia in the Australia and is more frequent in males than females. The CLL cells commonly have trisomy 12 alone or with additional chromosomal abnormalities. Number: 385 A 17 year old male experiences easy fatiguability and cramping pain of muscles with exercise during physical education classes. This condition does not improve with additional exercise, or with anti-inflammatory medications. He does not have problems with activities of daily living. The best explanation is: A. Duchenne muscular dystrophy B. Myasthenia gravis C. McArdle's disease D. Amyotrophic lateral sclerosis E. Trichinosis Select the single best answer ABCDE Correct Answer: C

348

Myophosphorylase deficiency, or McArdle's disease, is an uncommon muscle energy disease. Symptoms of pain and cramps after exercise usually develop during the second or third decade. A history of myoglobinuria is present in most, and on occasion myoglobinuria can cause renal failure. Affected individuals are otherwise healthy, without evidence of hepatic, cardiac, or metabolic disturbance. Performance of an ischemic exercise test usually causes painful cramping, which is helpful diagnostically. In addition, blood lactate does not rise, whereas serum creatine phosphokinase usually is elevated at rest but rises substantially after strenuous exercise. Number: 386 A 42 year old female complains of recent onset of easy bruising. The PT and PTT are normal, but her platelet count is only 10,000/microliter. A bone marrow biopsy reveals a normocellular marrow with increased numbers of megakaryocytes. These findings most strongly suggest a diagnosis of: A. Myeloproliferative disorder B. Drug reaction to recent antibiotic therapy C. Wiskott-Aldrich syndrome D. Epstein-Barr virus infection E. Idiopathic thrombocytopenic purpura Select the single best answer ABCDE Correct Answer: E In contrast to children, Most adults present with an indolent form of idiopathic thrombocytopenic purpura which may persist for many years and is referred to as chronic ITP. Women aged 20 to 40 are afflicted most commonly and outnumber men by a ratio of 3:1. They may present with an abrupt fall in platelet count and bleeding similar to patients with acute ITP. More often they have a prior history of easy bruising or menoorrhagia. These patients have an autoimmune disorder with antibodies directed against target antigens on the glycoprotein IIb-IIIa or glycoprotein Ib-IX complex. Wiskott-Aldrich syndrome is is an X-linked genetic disease characterized by eczema, thrombocytopenia, and repeated infections. The platelets are small and have a shortened halflife. Affected male infants often present with bleeding and most do not survive childhood, dying of complications of bleeding, infection, or lymphoreticular malignancy.

349

Number: 401 A prolonged QT interval may: 1. Be caused by hypocalcaemia. 2. Predispose to 'Torsade de pointes'. 3. Be caused by amiodarone. 4. Be caused by cardiac glycosides. A: 1,2,3 Correct B: 1,3 Correct C: 2,4 Correct D: 4 Correct E: All Correct ABCDE Correct Answer: A Prolongation of the QT interval is seen with drugs that increase the duration of the ventricular action potential: - ie Vaughan-Williams type 1A antiarrhythmic agents and related drugs (e.g., quinidine, disopyramide, procainamide) and type III agents (amiodarone, sotalol). Marked QT prolongation, sometimes with deep, wide T-wave inversions, may occur with intracranial bleeds, particularly subarachnoid haemorrhage. Hypocalcaemia typically prolongs the QT interval (ST portion), while hypercalcaemia shortens it. Digitalis glycosides shorten the QT interval, often with a characteristic "scooping" of the STT-wave complex. Torsades de pointes ("twisting of the points") refers to VT characterised by polymorphic QRS complexes that change in amplitude and cycle length, giving the appearance of oscillations around the baseline. This rhythm is, by definition, associated with QT prolongation. Number: 401 A prolonged QT interval may: 1. Be caused by hypocalcaemia. 2. Predispose to 'Torsade de pointes'. 3. Be caused by amiodarone. 4. Be caused by cardiac glycosides.

350

A: 1,2,3 Correct B: 1,3 Correct C: 2,4 Correct D: 4 Correct E: All Correct ABCDE Correct Answer: A Prolongation of the QT interval is seen with drugs that increase the duration of the ventricular action potential: - ie Vaughan-Williams type 1A antiarrhythmic agents and related drugs (e.g., quinidine, disopyramide, procainamide) and type III agents (amiodarone, sotalol). Marked QT prolongation, sometimes with deep, wide T-wave inversions, may occur with intracranial bleeds, particularly subarachnoid haemorrhage. Hypocalcaemia typically prolongs the QT interval (ST portion), while hypercalcaemia shortens it. Digitalis glycosides shorten the QT interval, often with a characteristic "scooping" of the STT-wave complex. Torsades de pointes ("twisting of the points") refers to VT characterised by polymorphic QRS complexes that change in amplitude and cycle length, giving the appearance of oscillations around the baseline. This rhythm is, by definition, associated with QT prolongation. Number: 405 Insulinoma: 1. Usually presents with symptoms of hypoglycaemia. 2. Has about a 1:10 chance of being malignant. 3. May be associated with peptic ulceration. 4. Is most common in the second and third decade of life. A: 1,2,3 Correct B: 1,3 Correct C: 2,4 Correct D: 4 Correct E: All Correct ABCDE Correct Answer: A The hallmark of pancreatic beta cell tumours is the development of symptomatic hypoglycemia from unregulated insulin hypersecretion. Symptoms related to the

351

hypoglycemia include headache, slurred speech, psychological alterations, visual disturbances and confusion. About 10% of tumours are malignant. As with gastrinomas, insulinomas are frequently associated with Multiple Endocrine Neoplasia type 1. Multiple endocrine neoplasia type 1, or Wermer's syndrome, is the association of neoplastic transformation of parathyroid, pituitary, and pancreatic islet cells. The syndrome is inherited as an autosomal dominant trait; each child born to an affected parent has a 50 percent chance of inheriting the predisposing gene. About 60% of patients with MEN 1 will develop a gastrinoma and about 35% an insulinoma. Insulinomas arise most frequently in the fifth to seventh decades, although cases have been reported at all ages. Number: 407 Hyposplenism: 1. Markedly increases the risk of overwhelming Neisseria meningitidis infection. 2. Is compatible with the blood film shown below. 3. Is an indication for vaccination against pneumococcal infection. 4. Is a cause of thrombocytopenia. A: 1,2,3 Correct B: 1,3 Correct C: 2,4 Correct D: 4 Correct E: All Correct

352

ABCDE Correct Answer: A The blood film shows Howell-Jolly Bodies. Thrombocytopenia is a feature of hypersplenism. The usual causes of hyposplenism are splenectomy, congenital asplenia, sickle cell anaemia in patients older than 5 years (with autosplenectomy due to repeated infarcts), and splenic irradiation Findings in the peripheral blood that indicate diminished splenic function include the presence of nucleated red cells, Howell-Jolly bodies, Heinz bodies, basophilic stippling and rarely, circulating nucleated red blood cells. Splenectomised patients or patients with functional asplenia (such as in sickle cell disease) are prone to bacterial infections, which are frequently overwhelming and life-threatening, particularly with encapsulated organisms such as Streptococcus pneumoniae, Neisseria meningitidis, Escherichia coli, and Haemophilus influenzae. The overall risk of bacterial sepsis in splenectomized patients has been estimated to be approximately 7 percent over a 10-year period. The role of pneumococcal vaccination has been the subject of a recent mata-analysis.

353

Number: 408 Hyperosmolar non-ketotic diabetic coma: 1. Is unusual in the elderly 2. Is rarely associated with a blood sugar level > 30 mmol/l 3. Typically causes hyperventilation. 4. May cause focal neurological signs A: 1,2,3 Correct B: 1,3 Correct C: 2,4 Correct D: 4 Correct E: All Correct ABCDE Correct Answer: D Hyperosmolar, nonketotic diabetic coma is usually a complication of non-insulin-dependent diabetes. It is a syndrome of profound dehydration resulting from a sustained hyperglycemic diuresis under circumstances in which the patient is unable to drink sufficient water to keep up with urinary fluid losses. Commonly an elderly diabetic patient--often living alone or in a nursing home--develops a stroke or infection which worsens hyperglycemia and prevents adequate water intake. The absence of ketoacidosis is important in the pathophysiology. Clinically, patients present with extreme hyperglycemia, hyperosmolality, and volume depletion, coupled with central nervous system signs ranging from clouded sensorium to coma. Seizure activity--sometimes Jacksonian in type--is not unusual, and transient hemiplegia may be seen. Infections, particularly pneumonia and gram-negative sepsis, are common and indicate a grave prognosis. Pneumonia is often due to gram-negative organisms. Number: 410 Hypomagnesaemia: 1. Is likely to be found in a patient with alcoholic liver disease. 2. Is likely to be found in a poorly controlled diabetic. 3. Can cause ventricular arrhythmias. 4. Can occur as a complication of primary hyperaldosteronism. A: 1,2,3 Correct B: 1,3 Correct C: 2,4 Correct D: 4 Correct E: All Correct

354

ABCDE Correct Answer: E Ethanol causes a transient loss of magnesium into the urine. Alcoholics with a reasonably normal nutrient intake and normal intestinal function usually have normal or only slightly depressed magnesium levels in blood. The total-body deficit of magnesium in chronic alcoholics is modest, amounting to 100 to 150 mmol (2.4 to 3.6 g). Mild hypomagnesemia occurs in poorly controlled diabetes mellitus. Moderate hypomagnesemia may occur in hyperparathyroidism, hypoparathyroidism, hyperthyroidism, and primary hyperaldosteronism, as well as during recovery from diabetic ketoacidosis. In primary hyperaldosteronism, aldosterone enhances magnesium excretion directly and acts via volume expansion to cause net losses of magnesium. These effects can be reversed by spironolactone. Cardiac arrhythmias, disturbances of conduction, and even ventricular fibrillation and cardiac arrest can occur in patients with hypomagnesemia. Number: 412 A 56-year-old male with hypertension (blood pressure 155/95 mm Hg) smokes 2 packs of cigarettes per day and has an HDL cholesterol that is greater than normal. Which of the following factors in his case is NOT associated with an increased risk for his developing atherosclerosis: A Hypertension B Male sex C Age D Increased HDL cholesterol E Smoking Select the single best answer ABCDE Correct Answer: D High-density lipoproteins (HDL) The level of HDL, a complex family of particles that carry about 20 percent of the total plasma cholesterol, is inversely associated with the development of premature atherosclerosis and therefore can be considered an "antirisk factor." HDL levels can be assessed simply by measurement of cholesterol in the supernatant fluid after the other lipoproteins in plasma have been precipitated. Thus individuals whose HDL cholesterol is elevated may be less likely to

355

develop IHD; conversely, low HDL cholesterol is associated with increased risk of IHD. In the Framingham Study, low HDL cholesterol was a more potent lipid risk factor than was high total cholesterol or LDL. At least five diverse population studies have confirmed a close correlation between IHD and low HDL, independent of other factors. Consistent with differences in risk between the sexes, HDL cholesterol averages about 25 percent higher in women than in men. Estrogens tend to raise and androgens tend to lower HDL levels. In women, low HDL, particularly when associated with diabetes and obesity, markedly raises the risk of IHD. Octogenarians tend to have high HDL, which may be partly familial. Of interest for preventive measures, cigarette smoking decreases and regular strenuous exercise increases HDL cholesterol. Regular exercise increases HDL even in individuals after myocardial infarction. A small daily intake of alcohol has been associated with both reduced risk of IHD and high HDL levels. Mechanisms for these effects remain unknown. Number: 415 A 17 year old girl presents with breathlessness, ankle oedema and evidence of right heart failure. She is known to have had a heart murmur since birth and appears to be slightly cyanosed. Her ECG is shown below. This picture is compatible with: 1. Ebstein's anomaly. 2. Primary Pulmonary Hypertension. 3. Eisenmenger Syndrome. 4. Hypertrophic Obstructive Cardiomyopathy. A: 1,2,3 Correct B: 1,3 Correct C: 2,4 Correct D: 4 Correct E: All Correct

356

ABCDE Correct Answer: B The ECG shows: Sinus rhythm Markedly peaked P waves (best seen in leads 11, V,) Normal axis Right bundle branch block Right atrial hypertrophy is seen with pulmonary hypertension of any cause, tricuspid stenosis, and Ebstein's anomaly. Right bundle branch block is characteristic of atrial septal defects. These conditions can all be diagnosed by echocardiography. This patient had Ebstein's anomaly and an atrial septal defect. Ebstein's anomaly This malformation is characterized by a downward displacement of the tricuspid valve into the right ventricle, due to anomalous attachment of the tricuspid leaflets. Tricuspid valve tissue is dysplastic; a variable portion of the septal and inferior cusps adhere to the right ventricular wall some distance away from the atrioventricular junction. The abnormally situated tricuspid orifice produces a portion of the right ventricle lying between the atrioventricular ring and the origin of the valve, which is continuous with the right atrial chamber. This proximal segment is "atrialized," and the distal ventricular chamber is small. The degree of impairment of right ventricular function depends primarily on the extent to which the right ventricular inflow portion is atrialized and on the magnitude of tricuspid valve regurgitation. Most patients survive at least to the third decade. Although the clinical manifestations are variable, some patients come to initial attention because of progressive cyanosis from right-to-left atrial shunting, or symptoms due to tricuspid regurgitation and right ventricular dysfunction, or paroxysmal atrial tachyarrhythmias with or without bypass tracts (type B Wolff-Parkinson-White syndrome is common). Eisenmenger's syndrome In adults who were born with a large left-to-right shunt, pulmonary vascular obstruction with pulmonary hypertension, right-to-left shunting (shunt reversal), and cyanosis can develop. This is known as Eisenmenger's syndrome. Primary pulmonary hypertension Primary pulmonary hypertension is an uncommon disease characterized by increased pulmonary artery pressure and pulmonary vascular resistance without an obvious cause. The diagnosis can be made only after all causes of pulmonary hypertension have been excluded. There is a female-to-male preponderance (1.7:1), with patients most commonly presenting in the third and fourth decades, although the age range is from infancy to greater than 60 years. Because the predominant symptom of primary pulmonary hypertension is dyspnea, which can have an insidious onset in an otherwise healthy person, the disease is typically diagnosed late in its course. By that time, the clinical and laboratory findings of severe pulmonary hypertension are usually present.

357

Number: 416 Platelet activation: 1. Is caused by Von Willebrand factor 2. Is inhibited by prostacyclin (PGI2) 3. Is inhibited by endothelium derived nitric oxide 4. Is inhibited by thromboxane A2 A: 1,2,3 Correct B: 1,3 Correct C: 2,4 Correct D: 4 Correct E: All Correct ABCDE Correct Answer: A 1 - binding to glycoprotein Ib receptors and exposed subendothelial components 2 - by increasing intraplatelet cAMP 3 - by increasing intraplatelet cGMP 4 - ADP and thromboxane A2 are released by platelet degranulation and lead to further platelet activation (Am J Med 1996;101:199-209) Number: 417 Which are true? 1. Wilcoxon's rank test needs equal sample sizes 2. 'r' is the symbol denoting coefficent of correlation 3. Student's t-test is a non-parametric test 4. y=a+bx is a regression equation A: 1,2,3 Correct B: 1,3 Correct C: 2,4 Correct D: 4 Correct E: All Correct ABCDE Correct Answer: C Student's t-test is a parametric test. y=a+bx is a regression equation and is the equation of a straight line.

358

Number: 418 In acute tubular necrosis: A The urinary sodium concentration is classically less than 30 mmol/l B The urine plasma osmolality ratio is more than 1:1 C Red cell casts are usually present in the urine D Proteinuria is an expected finding E The creatinine clearance is uaually normal after 1 year Select the single best answer ABCDE Correct Answer: D A > 30. B < 1:1. C Suggests nephritis. E Only 40% of cases. Number: 423 Hypomagnesemia: 1. Is commonly present in patients on admission to surgical Intensive Care Units. 2. Can occur as a result of treatment with aminoglycoside antibiotics. 3. Will potentiate the effect of non-depolarising muscle relaxants. 4. Can prolong the QT interval. A: 1,2,3 Correct B: 1,3 Correct C: 2,4 Correct D: 4 Correct E: All Correct ABCDE Correct Answer: E Abnormalities of serum magnesium (Mg++) concentrations may rank as the most common electrolyte disturbance among hospitalised patients. Disorders of Mg++ balance appear to be especially common among patients in intensive care. The clinical signs of hypomagnesemia are similar to those seen in patients with hypocalcemia and are characterized by neuronal excitability and tetany. Trousseau's sign, Chvostek's sign, muscle fasciculations, muscle spasticity, hyporeflexia, and seizures occur with progressive hypomagnesemia. Hypomagnesemia can cause subclinical respiratory muscle weakness in patients that improves after Mg++ supplementation. Both hypomagnesemia and hypocalcemia prolong the QT interval on the ECG.

359

Laboratory evidence of hypomagnesemia has been found in over 60% of surgical patients being admitted for postoperative intensive care in at least two studies. Chernow B, Bamberger S, Stoiko M, et al: Hypomagnesemia in postoperative intensive care, Chest 95:391-397, 1989. Ryzen E, Wagers PW, Singer FR, and Rude RK: Magnesium deficiency in a medical ICU population, Crit Care Med 13:19-21, 1985. Number: 424 Low T waves on an ECG are seen in: 1. Hypokalaemia. 2. Hypercalcaemia. 3. Pericardial effusion. 4. Athletes. A: 1,2,3 Correct B: 1,3 Correct C: 2,4 Correct D: 4 Correct E: All Correct ABCDE Correct Answer: B 1. In association with prominent U waves. 2. Hypercalcaemia causes a shortened ST segment and steeper distal limb of the T wave. 3. In association with generally small P waves and QRS complexes. 4. Athletic or vagotonic hearts tend to have tall T waves. Number: 425 Ostium secundum ASD is associated with 1. Left bundle branch block. 2. Fixed splitting of the second heart sound. 3. Onset of atrial fibrillation in the second decade. 4. Mitral stenosis. A: 1,2,3 Correct B: 1,3 Correct C: 2,4 Correct D: 4 Correct E: All Correct

360

ABCDE Correct Answer: C 1. Right bundle branch block. 2. A feature of all ASDs 3. Usually AF occurs later in life 4. This is Lutembacher's syndrome. Secundum ASD is also associated with mitral valve prolapse. Number: 427 Which of the following are features of Fallot's tetralogy? 1. A prominent systolic murmur from the ventricular septal defect. 2. Squatting. 3. Pulmonary plethora. 4. Paradoxical embolism. A: 1,2,3 Correct B: 1,3 Correct C: 2,4 Correct D: 4 Correct E: All Correct ABCDE Correct Answer: C 1. The systolic murmur comes from the pulmonary infundibular stenosis. 2. Squatting relieves the dyspnoea by increasing SVR. 3. There is usually pulmonary oligaemia as there is a right to left shunt. 4. Because of the right to left shunt at ventricular level. Tetralogy accounts for about 10 percent of all forms of congenital heart disease and is the most common cause of cyanotic forms. The four components of the tetralogy of Fallot are ventricular septal defect, obstruction to right ventricular outflow, aortic override of the ventricular septal defect, and right ventricular hypertrophy. The basic anomaly results from an anterior and superior deviation of the infundibular ventricular septum away from its usual location in the heart between the limbs of the trabecular septum. This displacement causes subpulmonary obstruction, aortic "override," and a large, nonrestrictive, malalignment-type ventricular septal defect.

361

Number: 430 Autosomal dominant inheritance: 1. Has surviving affected individuals, in a heterozygous state. 2. Is the mode of inheritance in multiple neurofibromatosis. 3. Affects both sexes equally. 4. Shows a pattern of vertical inheritance in a pedigree. A: 1,2,3 Correct B: 1,3 Correct C: 2,4 Correct D: 4 Correct E: All Correct ABCDE Correct Answer: E Dominant diseases are those manifest in the heterozygous state, i.e., when only one abnormal gene (mutant allele) is present and the corresponding partner allele on the homologous chromosome is normal. The gene responsible for an autosomal dominant disorder is located on one of the 22 autosomes, and both males and females can be equally affected. Since alleles segregate independently at meiosis, there is a 1 in 2 chance that the offspring of an affected heterozygote will inherit the mutant allele and, similarly, a 1 in 2 chance of the offspring inheriting the normal allele. The characteristic features of an autosomal dominant trait are that: (1) Each affected individual has an affected parent (unless the condition arose by a new mutation or is mildly expressed in the affected parent); (2) an affected individual will bear, on average, both normal and affected offspring in equal proportions; (3) normal children of an affected individual will have only normal offspring; (4) males and females are affected in equal proportions; (5) each sex is equally likely to transmit the condition to male and female offspring, with male-to-male transmission occurring; and (6) vertical transmission of the condition through successive generations occurs, especially when the trait does not impair reproductive capacity. Number: 447 Which statement is true with regard to Campylobacter jejuni: A. It most commonly attack the elderly B. It usually responds to treatment with ciprofloxacin C. Is a recognised pathogen in domestic animals D. It is readily isolated in stool culture E It is a rare cause of enteritis. Select the single best answer

362

ABCDE Correct Answer: C Campylobacter infections are not rare. Several studies indicate that in the United States diarrhoeal disease due to campylobacters is more common than that due to Salmonella and Shigella combined. Infections occur throughout the year, but their incidence peaks during summer and early autumn. Persons of all ages are affected; however, attack rates are highest among young children and young adults. If antibiotic therapy is indicated, erythromycin is the drug of choice. Campylobacters are found within the gastrointestinal tract of many animals used for food production (including poultry, cattle, sheep, and swine) and of household pets (including birds, dogs, and cats). However, these microorganisms usually do not cause illness in their animal hosts. In most cases, campylobacters are transmitted to humans in raw or undercooked food products or through direct contact with infected animals. In the United States and other developed countries, ingestion of contaminated poultry that has not been sufficiently cooked is the most common means of acquiring infection (50 to 70 percent of cases). The organism is difficult to culture. Number: 448 Which of the following is NOT a recognised complication of chronic renal failure? A. Sensory-motor neuropathy B. Pruritis C. Metastatic calcification D. Menorrhagia E. Proximal myopathy Select the single best answer ABCDE Correct Answer: D A. Common, sensory preceeds motor and dialysis helps prevent progression. B. Often resistant to treatment. C. Particularly when the serum phosphate and calcium levels are high. D. Amenorrhoea is a common and early symptom of uraemia - Infertility in men also occurs. E. The myopathy of chronic renal failure is distinct from the better known uraemic polyneuropathy.

363

Number: 449 Which of the following is NOT a well-recognised complication of long-term haemodialysis? A. Arthropathy B. Dementia C. Accelerated artherosclerosis D. Raised intracranial pressure E. Leucocytosis Select the single best answer ABCDE Correct Answer: E A. Amyloid can directly involve articular structures by its presence in the synovial membrane and synovial fluid or in the articular cartilage. Amyloid arthritis can mimic a number of rheumatic diseases because it can present as a symmetric arthritis of small joints with nodules, morning stiffness, and fatigue. B. Characterised by speech dyspraxia, myoclonus, dementia and siezures. Though to be due to aluminium overload. C. Caused by hypertension, hyperlipidaemia, glucose intolerance, chronic high cardiac output and metastatic calcification. D. Dialysis dysequilibrium - from lowering serum urea too quickly. Mannitol is often given during the first few haemodialysis sessions when the urea is particularly high. E. Leucopenia is common and transient in those exposed to cellophane-derived filters Number: 450 Which of the following is/are found in a patient with jaundice due to uncomplicated haemolytic anaemia? 1. Bilirubinuria 2. High conjugated serum bilirubin 3. High serum alkaline phosphatase 4. Reticulocytosis A: 1,2,3 Correct B: 1,3 Correct C: 2,4 Correct D: 4 Correct E: All Correct ABCDE

364

Correct Answer: D In the haemolytic anaemias, the rate of bilirubin production is increased and may exceed the amount that can be removed by a normal liver. The resulting jaundice is primarily an unconjugated hyperbilirubinaemia. There may also be a small increase in the serum conjugated bilirubin. Bilirubin appears in the urine only after it is converted to a water-soluble form; generally this involves conjugation with polar glucuronide groups which enhance water solubility. In the absence of bone disease or pregnancy, elevated levels of alkaline phosphatase activity usually reflect impaired biliary tract function. The increased levels reflect increased synthesis of the enzyme by hepatocytes and biliary tract epithelium rather than regurgitation of enzyme due to obstruction. Number: 460 Which of the following statements are true with regard to hereditary haemorrhagic telangiectasia (Osler-Weber-Rendu disease)? 1. Cerebral abscess is associated with the condition. 2. Cyanosis is associated with the condition. 3. Nose bleeding is associated with condition. 4. It is inherited as an autosomal dominant disorder. A: 1,2,3 Correct B: 1,3 Correct C: 2,4 Correct D: 4 Correct E: All Correct ABCDE Correct Answer: E Hereditary haemorrhagic telangiectasia is an important risk factor for brain abscess, especially in affected patients with clubbing, cyanosis, and/or polycythaemia. The condition is inherited as an autosomal dominant disorder and patients have frequent episodes of nasal and gastrointestinal bleeding from abnormal telangiectatic capillaries. A significant degree of right-to-left shunting (through pulmonary telangiectases) can occur.

365

Number: 476 Which of the following statements are true regarding Ankylosing Spondylitis? 1. It is associated with aortic incompetence. 2. It is associated with upper lobe fibrosis. 3. The earliest radiologically apparent changes are in the sacro-iliac joints. 4. About 10% of sufferers have the HLA B27 gene. A: 1,2,3 Correct B: 1,3 Correct C: 2,4 Correct D: 4 Correct E: All Correct ABCDE Correct Answer: A Aortic insufficiency develops in a small percentage of cases. There is thickening of the aortic valve cusps and the aorta near the sinuses of Valsalva, with dense adventitial scar tissue and intimal fibrous proliferation, the scar tissue often extending into the ventricular septum with resultant heart block. Pulmonary involvement, characterized by slowly progressive upper lobe fibrosis, is a rare complication of long-standing AS. Radiological abnormalities generally appear in the sacroiliac joints before appearing elsewhere in the spine. In most ethnic groups, the HLA-B27 gene is present in approximately 90 percent of patients with AS. Number: 479 The signs of hypertrophic cardiomyopathy with ventricular outflow obstruction can include: 1. A pansystolic murmur 2. The murmur becoming louder with Valsalva manoeuvre 3. A presystolic lift at the apex. 4. A slow rising pulse A: 1,2,3 Correct B: 1,3 Correct C: 2,4 Correct D: 4 Correct E: All Correct

366

ABCDE Correct Answer: A 1. A pansystolic murmur of associated mitral regurgitation is common. 2. It is also diminished by the Mueller manoeuvre (creating a negative intrathoracic pressure). 3. This is characteristic. Most patients with gradients demonstrate a double or triple apical impulse, a rapidly rising carotid arterial pulse, and a fourth heart sound. 4. A jerky pulse due to shortened systolic ejection is seen in HOCM - a slow rising pulse is seen in aortic stenosis. Number: 480 Giant 'a' waves in the JVP occur in: 1. Pulmonary hypertension 2. Aortic regurgitation 3. Tricuspid stenosis 4. Constrictive pericarditis A: 1,2,3 Correct B: 1,3 Correct C: 2,4 Correct D: 4 Correct E: All Correct ABCDE Correct Answer: B Giant 'a' waves occur when there is a poorly compliant right ventricle (or tricuspid stenosis) increasing the impedence against which the right atrium has to eject blood. A giant a wave in the jugular venous pulse without palpatory evidence of pulmonary hypertension or right ventricular enlargement is particularly suggestive of tricuspid stenosis. In constrictive pericarditis the JVP is high with an abrupt fall in systole (x descent) and may rise with inspiration (Kussmaul's sign).

367

Number: 482 Contraindications to streptokinase include: 1. Age over 75 years 2. Recent stroke. 3. Atrial fibrillation 4. Pregnancy. A: 1,2,3 Correct B: 1,3 Correct C: 2,4 Correct D: 4 Correct E: All Correct ABCDE Correct Answer: C The role of thrombolysis in pregnancy has been reviewed by Turrentine et al. Obstet Gynecol Surv 1995 Jul;50(7):534-41 Turrentine MA, Braems G, Ramirez MM Use of thrombolytics for the treatment of thromboembolic disease during pregnancy. Number: 496 Diabetic nephropathy: 1. Occurs more commonly in type II (as opposed to type I) diabetes mellitus. 2. Is usually associated with retinopathy. 3. Is more likely in hypertensive diabetics. 4. Can be reversed by meticulous diabetic control. A: 1,2,3 Correct B: 1,3 Correct C: 2,4 Correct D: 4 Correct E: All Correct ABCDE Correct Answer: A 1. It occurs in about 30% of those who have had diabetes for 20 years and is therefore more likely in IDDM.

368

2. It is almost always associated with diabetic retinopathy. If it is not then another cause of renal failure in a diabetic should be considered. 3. Hypertension must be treated agressively in diabetics to help prevent diabetic nephropathy. 4. Slowed but not reversed. Number: 501 Score: 14 Attempted: 84 Category: Medicine Hyponatraemia is a recognised feature of: 1. Primary hyperaldosteronism. 2. Acute renal failure. 3. Hyperthyroidism. 4. Prolonged oxytocin infusion. A: 1,2,3 Correct B: 1,3 Correct C: 2,4 Correct D: 4 Correct E: All Correct ABCDE Correct Answer: C 1. Primary hyperaldosteronism is characterised by hypokalaemia (less than 3 mmol/L) and hypernatraemia (due to both sodium retention and a concomitant water loss from polyuria). 2. Due to volume overload. 3. Hyponatraemia is a feature of severe hypothyroidism. Decreased delivery of tubular fluid to diluting segments and persistent release of ADH both limit water excretion in this condition. 4. Oxytocin has mild ADH activity. Water intoxication and hyponatraemia are particularly likely if the drug has been administered in 5% dextrose for a prolonged period. See: Eur J Obstet Gynecol Reprod Biol 1996 Sep;68(1-2):47-8 Higgins J, Gleeson R, Holohan M, Cooney C, Darling M Maternal and neonatal hyponatraemia: a comparison of Hartmanns solution with 5% dextrose for the delivery of oxytocin in labour. Number: 515 Legionella pneumophila: 1. Causes Pontiac fever. 2. Infection is characterised by encephalopathy and renal failure. 3. Pneumonia is associated with hyponatraemia and hypophosphataemia.

369

4. Is best treated with carbenicillin. A: 1,2,3 Correct B: 1,3 Correct C: 2,4 Correct D: 4 Correct E: All Correct ABCDE Correct Answer: A Pontiac fever is a self-limiting tracheobronchitis caused by legionella pneumophila. Clinical features include high fever, gastrointestinal upset, headache and encephalopathy. Serious infections are characterised by the development of hyponatraemia, hypophosphataemia and renal failure. Treatment options include erythromycin +/- rifampicin or the newer macrolide antibiotics such as azithromycin or clarithromycin. Number: 517 The cough associated with angiotensin-converting enzyme (ACE) inhibitors is: A. Dose dependent B. More common in women than men C. More common in patients who smoke cigarettes D. More likely to occur when treating hypertension E. Likely to improve on changing to a different ACE inhibitor Select the single best answer ABCDE Correct Answer: B A dry, tickly and often bothersome cough is the most common adverse effect of ACE inhibitors. Recent studies indicate that cough may develop in around 10% of the patients treated with ACE inhibitors. In half of these patients, the ACE inhibitor has to be discontinued. Cough has emerged as a class effect occurring with all ACE inhibitors with no clear difference between the single substances. While ACE inhibition is safe in the vast majority of patients with obstructive airways disease, asthmatic symptoms or exacerbation of asthma as well as a rise in bronchial reactivity have been occasionally reported. ACE inhibition increases the cough reflex.

370

The mechanisms underlying ACE inhibitor-induced cough are probably linked to suppression of kininase II activity, which may be followed by an accumulation of kinins, substance P and prostaglandins. Physicians should be aware that a dry cough is the most common adverse effect of ACE inhibitors and that this symptom may occur not necessarily shortly after institution of therapy but months or even a year later. Replacement by another ACE inhibitor should not be tried, since the cough will almost always recur on rechallenge with the same or another ACE inhibitor. After withdrawal of the ACE inhibitor, which is the treatment of choice, cough will resolve usually within a few days. Cough occurs more commonly in women than men. Number: 518 Guillain-Barre syndrome: 1. Is usually associated with a raised CSF protein. 2. Is commonly preceded by a viral-like illness. 3. Can involve the cranial nerves. 4. Can involve sensory nerves. A: 1,2,3 Correct B: 1,3 Correct C: 2,4 Correct D: 4 Correct E: All Correct ABCDE Correct Answer: E Guillain-Barre syndrome (GBS) is an acute, frequently severe and fulminant polyneuropathy. In over two-thirds of cases, an infection, usually viral, either clinically overt or evidenced by serum titre rise, precedes the onset of neuropathy by 1 to 3 weeks. GBS also occurs against a background of lymphoma, including Hodgkin's disease, and in lupus erythematosus. The clinical features of GBS typically include areflexic motor paralysis with mild sensory disturbance coupled with an acellular rise of total protein in the cerebrospinal fluid by the end of the first week of symptoms. The paralysis is typically symmetrical and can include bulbar cranial nerves. Most patients with GBS require hospitalisation, and about 30 percent will need ventilatory assistance at some point during the illness. The prognosis is good; approximately 85 percent of patients will make a complete or nearly complete recovery. The mortality rate is 3 to 4 percent. Management is generally supportive care, but plasmapheresis also has a role. Large, multicentre, controlled trials in North America and Europe have demonstrated a beneficial effect of plasmapheresis, if initiated in the first 2 weeks of illness. Intravenous administration of high-dose immunoglobulin (2 g/kg body weight given over 5 days) is probably as effective as plasmapheresis. In contrast, glucocorticoid treatment has not been shown to be effective.

371

Number: 532 Pneumocystis carinii pneumonia: 1. Occurs almost exclusively in the immunocompromised patient. 2. May respond to treatment with co-trimoxazole. 3. Is frequently accompanied by cytomegalovirus infection. 4. Can usually be isolated from the sputum. A: 1,2,3 Correct B: 1,3 Correct C: 2,4 Correct D: 4 Correct E: All Correct ABCDE Correct Answer: A Pneumocystis carinii causes severe pneumonia in immunocompromised hosts. Although this most commonly occurs in patients with the acquired immunodeficiency syndrome (AIDS), other groups of immunocompromised patients without AIDS are also at risk for P. carinii pneumonia. These patients have solid or haematologic malignancies, organ transplantation, or inflammatory conditions requiring chronic immunosuppressive drugs, particularly corticosteroids. There are important differences in the clinical presentation of P. carinii pneumonia in patients with and without AIDS. P. carinii causes an acute fulminate pneumonia in patients without AIDS while patients with AIDS have more insidious involvement. The two major drugs used in the treatment of Pneumocystis carinii pneumonia have been trimethoprim-sulfamethoxazole and pentamidine isethionate. These agents are equally effective, with an overall success rate of 70 to 80 percent. Trimethoprim-sulfamethoxazole acts by inhibiting folic acid synthesis, but the mode of action of pentamidine against Pneumocystis is unclear. The organism is not usually isolated in the sputum. Fibreoptic bronchoscopy with bronchoalveolar lavage (BAL), is the mainstay of P. carinii diagnosis. This procedure provides information about the organism burden, host inflammatory response, and the presence of other opportunistic infections. Transbronchial biopsy and open lung biopsy, which are the most invasive procedures, are reserved for situations when a diagnosis cannot be made by lavage. CMV and pneumocystis are frequently found together in immunosuppressed patients with severe interstitial pneumonia. CMV may function as a cofactor to activate latent HIV infection.

372

Number: 547 Clopidogrel: A. Is a platelet glycoprotein IIb / IIIa receptor blocker. B. Is a platelet cyclooxygenase inhibitor. C. Is a platelet ADP receptor inhibitor. D. Is a factor Xa inhibitor. E. Is an anti-thrombin III activator. Select the single best answer ABCDE Correct Answer: C Clopidogrel inhibits ADP-induced platelet activation. It is generally believed that clopidogrel is biotransformed and activated in the liver. However, the precise active metabolite(s) of ticlopidine and clopidogrel have not been precisely identified. It has been proposed that this metabolite is highly reactive and labile, and binds quickly and irreversibly to platelets, altering certain receptors, interfering with the binding of ADP, and blocking further ADPinduced signaling. Recent data have also suggested that both ticlopidine and clopidogrel can interfere with invitro ADP-induced aggregation of washed human platelets; thus biotransformation may not be a necessary step. This effect was not noted when either plasma or albumin was present. Regardless of the exact mechanism, thienopyridines produce a permanent inhibition of the low-affinity ADP receptor, and platelets exposed to clopidogrel are irreversibly inhibited for their lifetime. Clopidogrel has superceded ticlopidine because of its better safety profile. Number: 548 Platelet activators include: 1. Thrombin. 2. Calcium. 3. Thomboxane A2 4. Prostacyclin. A: 1,2,3 Correct B: 1,3 Correct C: 2,4 Correct D: 4 Correct E: All Correct

373

ABCDE Correct Answer: A The important platelet activators include: Collagen, ADP, Thrombin, TXA2 and Calcium. Prostacyclin is the 'natural' platelet inhibitor which acts by elevating the level of intraplatelet cyclic AMP. Elevation of cAMP facilitates entry of calcium into the sarcoplasmic reticulum, lowers cytoplasmic calcium and thus inhibits TXA2 production.

Number: 549 Type II Heparin-Induced Thrombocytopenia: 1. Is usually associated with the presence of Heparin:Platelet Factor 4 complex antibodies. 2. Typically occurs within 2-3 days of iniotiation of heparin therapy. 3. Can complicate therapy with low molecular weight heparin. 4. Can occur as a result of the use of hirudin. A: 1,2,3 Correct B: 1,3 Correct C: 2,4 Correct D: 4 Correct E: All Correct ABCDE Correct Answer: B Type I Heparin-Induced Thrombocytopenia occurs early in the course of treatment, is not immunologically mediated and does not usually depress the platelet count severely. Heparin-induced thrombocytopenia (HIT) type II is an immune-mediated reaction that generally occurs 5 to 14 days after initiation of heparin therapy. It is characterized by a severe decline in platelet count (by >50%) in association with a new thromboembolic complication. Type II HIT is mediated by antibodies, usually of the IgG class. The antibodies cause platelet activation in the presence of heparin or other polysulfated saccharides. The antigen in type II HIT frequently is a complex of platelet factor 4 (PF4) and heparin. Hirudin is structurally unrelated to heparin and does not cause Heparin-Induced Thrombocytopenia. LMWH may rarely cause the syndrome.

374

Nuttall et al have recently reviewed the management of 12 patients with a history of HIT type II who required cardiac surgery on cardiopulmonary bypass. See: Nuttall GA, Oliver WC Jr, Santrach PJ, McBane RD, Erpelding DB, Marver CL, Zehr KJ. Patients with a History of Type II Heparin-Induced Thrombocytopenia with Thrombosis Requiring Cardiac Surgery with Cardiopulmonary Bypass: A Prospective Observational Case Series. Anesth Analg. 2003 Feb;96(2):344-50. Number: 552 Which of the following forms of sickle cell disease represent a significant risk to a patient undergoing general anaesthesia? 1. S - S disease. 2. Sickle - Beta thalassaemia. 3. S - C disease. 4. Sickle cell trait. A: 1,2,3 Correct B: 1,3 Correct C: 2,4 Correct D: 4 Correct E: All Correct ABCDE Correct Answer: A Different genotypes can give rise to sickle cell disease. The classic form of the disease may be considered to be the homozygous state for haemoglobin S, i.e., SS disease or sickle cell anaemia. However, the manifestations of SC disease, and sickle cell - Beta thalassemia are very similar. Typically, sickle cell anaemia has the most severe clinical course whereas extreme splenomegaly may be more likely to occur in haemoglobin SC disease and S - Beta thalassaemia. However, there is a great deal of overlap between the severity and clinical manifestations of these disorders. The principal importance of the distinctions between different sickle disease genotypes is in their diagnosis and in genetic counseling. A number of factors influence the course of sickle cell disease and may be important in precipitating crises. These include infection and fever, dehydration, and exposure to low oxygen tension. Even apart from these factors, the severity of the clinical manifestations of sickle cell disease varies greatly from patient to patient, not only in the degree to which they are affected but also with respect to which of the morbid manifestations predominate. Thus, some may suffer chiefly from anaemia with few, if any, painful episodes, while others experience many painful crises but have haemoglobin levels that are nearly normal.

375

The trait does not constitute a significant risk to the patient who is to undergo otherwise uncomplicated general anaesthesia and more recently it has been confirmed that patients with sickle cell trait can be safely anaesthetised for coronary artery grafting. Djaiani GN, Cheng DC, Carroll JA, Yudin M, Karski JM Anesth Analg 1999 Sep;89(3):598603 Fast-track cardiac anesthesia in patients with sickle cell abnormalities. Number: 561 With regard to an X-linked recessive disease: 1. Mothers are always carriers 2. Fathers never transmit to their sons 3. Variable expression in females is due to random inactivation 4. 50% of daughters of carrier females will be carriers A: 1,2,3 Correct B: 1,3 Correct C: 2,4 Correct D: 4 Correct E: All Correct ABCDE Correct Answer: E The genes responsible for X-linked disorders are located on the X chromosome, and the clinical risks are different for the two sexes. Since a female has two X chromosomes, she may be either heterozygous or homozygous for a mutant gene, and the mutant allele may demonstrate either recessive or dominant expression. Expression in females is often variable and influenced by random X-chromosome inactivation. Males, on the other hand, have only one X chromosome, so they are more likely to display the full phenotype, regardless of whether the mutation produces a recessive or dominant allele in the female. Thus, the terms X-linked dominant or X-linked recessiverefer only to the expression of the mutation in women. An important feature of all X-linked inheritance is the absence of male-to-male (i.e., father-toson) transmission of the trait. This follows because a male contributes his Y chromosome to his son and does not contribute an X chromosome. On the other hand, since a male contributes his sole X chromosome to each daughter, all daughters of a male with an X-linked disorder will inherit the mutant allele. Examples of X-linked recessive disorders in humans include the Lesch-Nyhan syndrome, glucose-6-phosphate dehydrogenase deficiency, testicular feminization, and Hunter's mucopolysaccharidosis

376

Number: 571 Which of the following cytotoxic agents is / are quite likely to cause pericarditis? 1. Doxorubicin 2. Busulfan 3. Bleomycin 4. Cyclophosphamide A: 1,2,3 Correct B: 1,3 Correct C: 2,4 Correct D: 4 Correct E: All Correct ABCDE Correct Answer: D Busulphan and bleomycin rarely cause pericarditis. Subacute doxorubicin toxicity occurs uncommonly. It develops early in the course of therapy and is characterised by myocarditis and pericarditis. Chronic toxicity is the most common form of doxorubicin-induced cardiac toxicity. It is manifest by chronic dilated cardiomyopathy, which develops late in the course of therapy or shortly after its termination. The incidence of pericarditis in association with the use of cyclophosphamide may be as high as 33%. Number: 572 The risk of acquiring hepatitis C (HCV) from a needle stick injury where the needle has been contaminated with the blood of an HCV +ve patient is approximately: A. 0.003% B. 0.1% C. 3% D. 10% E. 30% Select the single best answer ABCDE Correct Answer: C The risk of hepatitis C from a HCV-infected needle stick is approximately 3%. Number: 573 Which of the following forms of hepatitis is / are primarily transmitted by blood?

377

1. B 2. D 3. C 4. A A: 1,2,3 Correct B: 1,3 Correct C: 2,4 Correct D: 4 Correct E: All Correct ABCDE Correct Answer: A Hepatitis B, D, and C are most commonly transmitted by parenteral inoculation with the blood of an infected person. Hepatitis A is primarily transmitted by the faeco-oral route Number: 575 A patient with osteogenesis imperfecta is at increased risk of: 1. Kyphoscoliosis. 2. Mitral valve prolapse. 3. Hyperthermia. 4. Joint laxity. A: 1,2,3 Correct B: 1,3 Correct C: 2,4 Correct D: 4 Correct E: All Correct ABCDE Correct Answer: E 1. In types III and IV osteogenesis imperfecta, multiple fractures from minor physical stress can produce severe deformities. Kyphoscoliosis can result. 2. Mitral valve prolapse is a frequent finding in patients with heritable disorders of connective tissue, including Marfan's syndrome, osteogenesis imperfecta, and Ehler-Danlos syndrome. 3. For unknown reasons, some patients are at risk of a hypermetabolic state with elevated serum thyroxine levels, hyperthermia, and excessive sweating. They are also at increased of classical malignant hyperthermia.

378

4. About 50% of patients have a molecular defect in type I procollagen that produces lax joints similar to those in type VII Ehler-Danlos syndrome. Number: 580 Which of the following is an example of discrete interval data? A. ASA physical status B. Mallampati score C. Nasopharyngeal temperature D. Pain score E. None of the above. Select the single best answer ABCDE Correct Answer: E ASA physical status, Mallampati score and Pain score are examples of ordinal data. (Data which can be ranked or measured without a constant scale interval.) Nasopharyngeal temperature is an example of continuous data. (Data which are measured on a continuum with a consistent scale interval.) Number: 581 In the case of a sample population which is normally distributed, the proportion who are within +/- 1 standard deviation of the mean is approximately: A. 33% B. 50% C. 68% D. 95% E. 99% Select the single best answer ABCDE Correct Answer: C One standard deviation encompasses roughly 68% of the sample; two standard deviations roughly 95% and three standard deviations roughly 99%.

379

Number: 583 Which of the following factors increase the risk of bleomycin-induced pulmonary toxicity? 1. Age. 2. Concurrent radiotherapy. 3. Concurrent cytotoxic therapy. 4. Cumulative dose. A: 1,2,3 Correct B: 1,3 Correct C: 2,4 Correct D: 4 Correct E: All Correct ABCDE Correct Answer: E Risk factors predisposing to the development of pulmonary toxicity by bleomycin include: total dose administered, age, concurrent radiotherapy, exposure to a high inspired oxygen and concomitant use of other cytotoxic drugs. Of most significance to anaesthetists is the association between the development of pulmonary toxicity and transient exposure to a high FiO2. The risk of toxicity is strongly dose-related, and above a cumulative dose of ~ 500 mg, the incidence increases exponentially. Fatal pulmonary toxicity has been reported with doses as low as 100 mg. Number: 584 The most important complcation of cisplatin therapy is: A. Cardiotoxicity. B. Nephrotoxicity. C. Pulmonary toxicity. D. Hepatotoxicity. E. Neurotoxicity. Select the single best answer ABCDE Correct Answer: B Cisplatin is a common component of combination chemotherapy for ovarian, testicular and bladder tumours. Nephrotoxicity is by far the most important complication of cisplatin therapy.

380

As early as 35 days after administration, there may be a progressive decrease in glomerular filtration rate followed by evidence of acute tubular necrosis. In addition to the usual biochemical pattern of a tubular injury, a magnesium-wasting defect may be evident in up to 50% of patients with renal impairment. Number: 588 With regard to renal transplantation: 1. The one year graft survival rate for cadaveric grafts is > 85%. 2. The maximum acceptable pre-implantation ischaemic time is 24 hours. 3. A cadaveric graft is likely to function for more than 10 years. 4. The one year graft survival rate for living donor grafts is > 98%. A: 1,2,3 Correct B: 1,3 Correct C: 2,4 Correct D: 4 Correct E: All Correct ABCDE Correct Answer: B The one year graft survival rate for cadaveric grafts is about 87% and that for living donors 93.9%. The maximum acceptable pre-implantation ischaemic time is about 48 hours. The half-life graft survival for cadaveric grafts is 19.5 years. N Engl J Med 2000 Mar 2;342(9):605-12. Hariharan S et al Improved graft survival after renal transplantation in the United States, 1988 to 1996. Hariharan S et al. have recently reported the results of ~94000 renal transplants in the US (See abstract). Number: 593 Chronic alcohol abuse is associated with: 1. Korsakoff's syndrome. 2. Peipheral neuropathy. 3. Wernicke's syndrome. 4. Cerebellar atrophy.

381

A: 1,2,3 Correct B: 1,3 Correct C: 2,4 Correct D: 4 Correct E: All Correct ABCDE Correct Answer: E Wernicke's and Korsakoff's syndromes are the result of thiamine deficiency in vulnerable individuals (possibly in association with a genetic transketolase deficiency). Korsakoff's syndrome presents with profound anterograde amnesia and a milder retrograde amnesia, in the presence of a normal intelligence quotient. Chronic intake of high doses of ethanol causes peripheral neuropathy in 5 to 15 percent of alcoholics - probably related to thiamine deficiency. Thiamine (vitamin B1) deficiency causes Wernicke's encephalopathy. The clinical presentation is of a malnourished individual with confusion, ataxia, and diplopia (Charcot's triad). About 1 percent of malnourished alcoholics develop cerebellar degeneration. Number: 599 Nephrogenic diabetes insipidus may be caused by: 1. Lithium. 2. Hypokalaemia. 3. Hypercalcaemia. 4. Cerebral death. A: 1,2,3 Correct B: 1,3 Correct C: 2,4 Correct D: 4 Correct E: All Correct ABCDE Correct Answer: A 20-70% of patients receiving long term lithium therapy have some degree of nephrogenic diabetes insipidus. The condition can also be caused by other drug or electrolyte disturbances - particularly hypercalcaemia and hypokalaemia.

382

Central diabetes insipidus may complicate head injury or brain death. When patients with nephrogenic or central diabetes insipidus cannot be differentiated by simpler means, documentation of elevated plasma or urinary vasopressin (AVP) concentration in relation to plasma osmolality or of a high AVP concentration in relation to urine osmolality will allow the diagnosis of nephrogenic diabetes insipidus. The causes of acquired nephrogenic diabetes insipidus include: (1) Various forms of chronic renal disease. (2) The recovery phases of obstructive uropathy and acute tubular necrosis. (3) Potassium deficiencies, including primary aldosteronism. (4) Acute and chronic hypercalcaemia, including hyperparathyroidism. (5) Drug-induced: lithium, methoxyflurane, Amphotericin B. (6) Systemic disorders including: multiple myeloma, amyloidosis, sickle cell anaemia, Sjogren's syndrome. Number: 606 With regard to systemic hypertension: A. 1% is caused by primary renal disease. B. Benign nephrosclerosis is the usual cause of renal impairment in patients with essential hypertension. C. Medial hypertrophy is a common reno-vascular abnormality . D. Cerebrovascular disease is loosely correlated with it. E. All of the above. Select the single best answer ABCDE Correct Answer: B 95% of hypertension is considered idiopathic or "essential"; 5% is considered secondary to identifiable causes. Renal disease represents the majority of these. Renal dysfunction resulting from chronic essential hypertension is generally due to benign nephrosclerosis which is associated with intimal degeneration. Accelerated renal dysfunction associated with malignant hypertension involves the classic "onion skinning" hypertrophy of the media. There is a strong correlation between hypertension and cerebrovascular disease and similarly, control of blood pressure is associated with delayed progression of cerebrovascular disease. The Joint National Committee on the Detection , Evaluation and Treatment of Hypertension defines it as a systolic BP > 140 or diastolic BP > 90. This is obtained from an average of three readings reproducible on three different days. A pathophysiological classification describes primary or essential hypertension which accounts for 95% and secondary, accounting for 5%. Of the latter, renal disease accounts for the majority. This includes renovascular disease (RAS), disorders in Na/H2O excretion, and disorders of the renin-angiotensin system. Endocrine disorders constitute less than 1% of causes, and include hyperaldosteronism, Cushings syndrome, and phaeochromocytoma.

383

Globally, the common histopathological changes are intimal proliferation but further changes may be seen as end organ disease develops. End organ sequelae may affect any circulatory bed but heart, brain and kidneys are of greatest significance. Renal dysfunction associated with hypertension may manifest in a number of ways. Long standing hypertension may cause chronic renal injury characterized by intimal hyperplasia, sclerosis of afferent arterioles and parenchymal fibrosis causing gradual deterioration in function: this process is called benign nephrosclerosis. Conversely, a more fulminant decline culminating rapidly in irreversible renal failure may occur less commonly in a process called malignant nephrosclerosis: it is commonly associated with hypertensive crisis. Histologically, fibrinoid necrosis of the medial layer of the vessel walls is the most prominent feature. The course of malignant nephrosclerosis is such that renal failure will ensue within days to weeks unless blood pressure is controlled. There is an established role of hypertension in the genesis of cerebrovascular disease and lowering blood pressure has been shown to offer protection from cerebrovascular accidents. The pathogenesis of these may be thrombotic, embolic, hypotensive (haemorrhagic, dysrhythmic). References The Washington Manual- Manual of Medical Therapeutics, Little Brown & Co.,27th Ed., 1992, pp 62-64. ROGERS, M.C ET AL (EDS); Principles and Practice of Anesthesiology, Mosby, 1993, pp 155-65. Number: 623 A patient newly diagnosed with pulmonary tuberculosis (TB): 1. Should be nursed by personnel wearing masks capable of filtering particles in the submicrometre range. 2. Can undergo elective surgery provided that appropriate therapy has been initiated. 3. Is not a risk to others once 2 negative acid-fast bacillus sputum examinations have been performed. 4. Should be screened for Human Immuno-deficiency Virus (HIV) infection. A: 1,2,3 Correct B: 1,3 Correct C: 2,4 Correct D: 4 Correct E: All Correct ABCDE Correct Answer: D TB is transmitted by aerolisation of sputum and all those involved in the care of such patients should wear masks designed to filter particles 1-5 microns in diameter.

384

Elective surgery should be postponed until an infected individual has had an adequate course of therapy. In a recent study, it was found that of a total of 166 acid-fast bacilli positive suspects who had three sputum smears examined sequentially, 128 (77.1%) were found on the first smear, a further 25 (15%) on the second smear and 13 (7.9%) additional cases were identified on the third smear. Alcoholics, drug users, individuals who have been in close contact with people with tuberculosis, and patients with AIDS are all at increased risk of the disease. Number: 625 The proportion of patients infected with hepatitis C (HCV) who will develop chronic hepatitis is about: A. 10% B. 30% C. 50% D. 80% E. 100% Select the single best answer ABCDE Correct Answer: D Infection with HCV proceeds to chronicity in more than 80% of cases, and even recovery does not protect against subsequent re-exposure to the virus. Number: 626 The risk of acquiring hepatitis B from a needle stick injury where the needle has been contaminated with the blood of an hepatitis B +ve patient is approximately: A. 0.3% B. 0.1% C. 3% D. 10% E. 30% Select the single best answer ABCDE Correct Answer: E

385

The risk of acquiring hepatitis B is about 30%. For comparison, the risk of hepatitis C from an HCV-infected needle stick is approximately 3% and the risk of acquiring HIV from an HIVinfected needle stick is about 0.3%. Number: 644 A 20 year old asthmatic is admitted to the Emergency Department with acute asthma. He is breathing 100% oxygen via a non-rebreathing system. He has used his salbutamol puffer 'several times' since the attack began. His blood pressure is 140/55 mm Hg and his pulse rate is 110 bpm. A blood gas analysis is performed which shows: PaO2: 55 mm Hg. PaCO2: 86 mm Hg. pH: 7.08. BXS: -3 Given the scenario above, what should the next step in management be? A. Immediate intubation. B. Reduction of FiO2 from 1.0 to 0.28. C. Administration of more aerosolised salbutamol. D. Administration of intravenous adrenaline. E. Administration of intravenous aminophylline. Select the single best answer ABCDE Correct Answer: D The PaCO2 of 86 mm Hg is very worrying, but is not an indication for immediate intubation in the present context. Neither is it an indication for a reduction in FiO2! - Young asthmatics do not rely on hypoxic drive (and if more evidence is needed, the lack of a base XS attests to this) and any reduction in FiO2 will only worsen the hypoxaemia. The patient needs adrenaline -although it is debatable whether it needs to be given intravenously or not. It should be remembered that adrenaline is probably more effective than salbutamol in acute asthma - possibly because it has vasoconstrictive effects on the bronchial mucosa which further enhances its bronchodilator activity and / or the fact that it is less likely to cause inappropriate release of hypoxic pulmonary vasoconstriction (See for example: Coupe MO, Guly U, Brown E, Barnes PJ. Nebulised adrenaline in acute severe asthma: comparison with salbutamol. Eur J Respir Dis. 1987). Aminophylline has practically no role in the therapy of acute asthma (See: Littenberg B. Aminophylline treatment in severe, acute asthma. A meta-analysis. JAMA. 1988 Mar 18;259(11):1678-84.). Number: 650 Hirschprung's Disease: 1. Is usually not diagnosed until several weeks after birth.

386

2. Is more common in females. 3. Is incurable. 4. Can be diagnosed by biopsy of the anorectal wall. A: 1,2,3 Correct B: 1,3 Correct C: 2,4 Correct D: 4 Correct E: All Correct ABCDE Correct Answer: D 90% of cases are diagnosed within 3 days of life. It is much more common in males than females. It is curable through surgical intervention. This often involves an initial colostomy to relieve intestinal obstruction. The definitive operation is designed to bypass the aganglionic segment and bring normal bowel down to the anus. Prognosis is associated with the amount of viable bowel left after resection.The diagnosis may be made by barium enema, rectal biopsy or ano-rectal manometry.The presence of Auerbach's and Meissner's plexuses are diagnostic; excluding Hirschsprung's. Hirschsprung's Disease is results from the failure of migration of ganglion cells to the submucosal (Meissner's) and myenteric (Auerbach's) nerve plexuses of the large gut. The aganglionic segment, which remains tonically contracted and aperistaltic, invariably involves the internal sphincter. Sometimes it involves a very short segment, but 80% involve the rectosigmoid colon and 15% extend to the more proximal colon. The incidence is 1/4500 births. It accounts for 10% of neonatal intestinal obstruction, and must be considered if constipation presents in infancy. It may also present in the older child. The typical infant fails to pass meconium within the first 24 hours of life, develops abdominal distension, refuses to feed and finally, develops bilious vomiting. A severe form of enterocolitis with perforation and septicaemia may complicate this and carries a high mortality rate. The older child suffers intermittent bouts of intestinal obstruction from faecal impaction, failure to thrive, hypochromic anaemia and hypoproteinaemia. Soiling is extremely unusual, but not unknown. George Meissner was the Professor of Physiology at Gottingen University and described the plexus in 1853. Auerbach described his plexus in 1862. References HULL, D. JOHNSTON, D.I; Essential Paediatrics, 2nd Ed., pp 162. Number: 653 Which of the following pituitary hormones are well known to be secreted by a malignant lung tumour? 1. ACTH.

387

2. MSH. 3. Prolactin (in men). 4. Gonadotrophin. A: 1,2,3 Correct B: 1,3 Correct C: 2,4 Correct D: 4 Correct E: All Correct ABCDE Correct Answer: A Although other hormones are secreted, these are the most likely. By far the most common are ACTH and MSH. This phenomena is part of a constellation of complications of malignancy not directly caused by the tumour mass effect, or paraneoplastic syndrome. The effects are presumed to be mediated by either secreted tumour products or the development of autoantibodies and include hypercalcemia, SIADH, polymyositis, dermatomyositis, Eaton-Lambert myasthenic syndrome, clubbing and Hypertrophic Pulmonary Osteoarthropathy (HPOA). A useful classification of paraneoplastic syndromes is: (1) Metabolic/endocrine (2) Neuromuscular (3) Haematologic/vascular (4) Dermatologic (5) Skeletal/connective tissue The Washington Manual- Manual of Medical Therapeutics, Little Brown & Co., 27th Ed., 1992, pp 363-5. Number: 654 Which of the following are signs of Subacute Bacterial Endocarditis? A. Osler's Nodes. B. Splenomegaly. C. Microscopic haematuria. D. Splinter haemorrhages. E. All of the above. Select the single best answer ABCDE Correct Answer: E The clinical features of infective endocarditis depend upon the causative organism. Streptococcus viridans causes the classical picture of subacute bacterial endocarditis (SBE).

388

Parenteral drug abusers and patients with catheter associated sepsis have an increased risk of staphylococcal disease which typically causes acute bacterial endocarditis (ABE). Either organism may cause either syndrome, however. Gram negative and fungal endocarditis occur infrequently and are usually seen in patients with prosthetic valves and parenteral drug users. Patients with SBE are chronically ill, with symptoms of fatigue, weight loss, low grade fever, immune complex disease (nephritis, arthralgia, petechiae, Osler's nodes on fingertips, Janeway lesions on ophthalmoscopic examination) and embolic phenomena (renal, splenic and cerebral infarcts). The Washington Manual- Manual of Medical Therapeutics, Little Brown & Co, 27th Ed., 1992, pp 259-60. Number: 655 A 38 year old male presents with increasing dyspnoea and peripheral oedema. A chest X-Ray reveals a grossly enlarged heart. A CT scan demonstrates a 15 cm mass involving the right ventricle that appears to contain areas of haemorrhage and necrosis. Which of the following neoplasms is most probable, given these findings: A. B. C. D. E. Rhabdomyosarcoma Mesothelioma Myxoma Angiosarcoma Papillary fibroelastoma

Select the single best answer ABCDE Correct Answer: D The clinical picture is suggestive of a high grade malignancy - thus a myxoma is not likely. Primary cardiac malignancies are very rare, but by far the most common are the angiosarcomas. Papillary fibroelastomas are benign, usually valvular tumours, which are being increasingly noted as echocardiographic imaging of the heart becomes more widespread. Number: 656 A 17 year old previously healthy female dies suddenly and unexpectedly. She had complained only of a slight headache in the days before her demise. At autopsy, the heart is noted to be slightly dilated and there is a small pericardial effusion. The coronary arteries and heart valves are normal. Microscopically, the myocardium is infiltrated by lymphocytes and there are areas of focal necrosis. Which of the following infectious agents is most likely to have caused these findings? A Coxsackie B virus.

389

B Cytomegalovirus. C Hepatitis C virus. D Streptococcus viridans. E Corynebacterium Diphtheriae. Select the single best answer ABCDE Correct Answer: A Coxsackie infection is by far the most common cause of primary myocarditis in young people. Cytomegalovirus also has a predilection for cardiac muscle but is usually more problematic when reactivated in patients under immunosuppression. Hepatitis C is associated with dilated cardiomyopathy, but not with a picture of fulminant, acute myocarditis. Diphtheric myocarditis is becoming more common, but is associated with more marked symptomatology. Number: 662 A patient presents with ventricular tachycardia associated with a BP of 100/60. A diagnosis of digoxin toxicity is made. Appropriate management might include the use of: 1. Lignocaine. 2. Digoxin Fab fragments (Digibind). 3. Phenytoin. 4. Immediate cardioversion. A: 1,2,3 Correct B: 1,3 Correct C: 2,4 Correct D: 4 Correct E: All Correct ABCDE Correct Answer: B The management of VT associated with digoxin toxicity depends upon the clinical condition of the patient. If the patient is not markedly haemodynamically compromised, then treatment includes lignocaine and or phenytoin. If the patient is hypotensive such that they will not tolerate these agents, immediate cardioversion is indicated. Caution is required as this may precipitate asystole and 0.25 joules/ Kg is the recommended current. Digibind is indicated when absolute ingested dose exceeds 10 mg in adults or 4 mg in children; plasma levels of

390

digoxin are > 10ng/ml; serum potassium is > 5 mmol/L or when life threatening rhythm disturbances occur which do not respond to conventional therapy. Digoxin toxicity occurs in 5-15% of patients at some time during therapy. The therapeutic range is narrow and toxicity may develop despite serum levels within the normal range. Factors which commonly contribute to toxicity include hypokalemia, hypoxaemia, hypomagnesaemia, hypothyroidism, renal insufficiency and hypovolemia. Manifestations include cardiac arrhythmias of virtually all types including VEBs, bigeminy, and junctional tachycardia with varying degrees of block. More specifically, bidirectional VT, PAT with AV block and regularization of atrial fibrillation occur almost exclusively as a result of digoxin toxicity. Non-cardiac manifestations include anorexia, nausea, diarrhoea, agitation, lethargy and visual disturbances (xanthopsia). On the basis of this, management involves discontinuation of the drug, correction of precipitating factors and monitoring of cardiac rhythm and blood pressure. K+ should be maintained in the high normal range hence, it is often indicated, however if haemodynamic comprise is marked a metabolic acidosis may cause hyperkalemia. Symptomatic bradycardias should be treated with atropine or temporary pacing. Sympathomimetic agents should be avoided as they may precipitate a malignant ventricular arrhythmia. Ventricular arrhythmias or accelerated supraventricular arrhythmias should be treated with lignocaine. Phenytoin has a proven role in the management of VT unresponsive to lignocaine. Quinidine should not be used as it may elevate digoxin levels further. Cardioversion in the presence of digoxin toxicity is generally contraindicated as it may precipate malignant arrhythmias, specifically asystole. However, it may be necessary when all other measures have been exhausted or in haemodynamic compromise. In this situation the current required is generally smaller than for other arrhythmias; ie 0.25 J/kg. Digibind is digoxin specific Fab Antibody Fragments. It works by reversibly binding with digoxin whereupon the complexes are cleared from the circulation via renal excretion. It is useful in acute intoxication. Literature to date suggests its use be reserved for specific clinical situations. These include a serum K+ > 5.0 ,ingested dose > 10 mgs , serum level > 10 ng/ml , or when death appears imminent. The suggested dose is determined by the estimated amount of drug in the circulation: Dose = ingested dose (mg) x 0.8 / 0.6 Dose = serum level (ng/ml) x wt(kg) /100 One vial (40mg) inactivates 0.6mg digoxin. In acute intoxication this may require many vials making it a very expensive therapy. References The Washington Manual- Manual of Medical Therapeutics, Little Brown & Co., 27th Ed., 1992, pp 109-10.

391

Number: 667 Which of the following is true of the second heart sound: A. Normal splitting is present throughout life. B. Normal splitting is heard maximally at end expiration. C. Fixed splitting can occur in aortic stenosis. D. Reversed splitting can be a normal variant. E. None of the above. Select the single best answer ABCDE Correct Answer: E Normal splitting of the second heart sound can generally be heard up until the fourth decade of life, and is heard maximally at end inspiration. Fixed splitting may occur when the pulmonary valve closure is delayed as in pulmonary stenosis, hence accentuation on inspiration (normal splitting) is lost. Reversed splitting may occur when closure of the aortic valve is so delayed that it occurs even after the pulmonary valve. It is never a normal variant and occurs with, for example, severe aortic stenosis. The second heart sound is caused by closure of the aortic and pulmonary valves. Normally, the aortic component (A2) precedes the pulmonary (P2). This can be heard as " normal splitting " which is heard maximally in the pulmonary area at end inspiration and minimally at end expiration. During inspiration, negative intrathoracic pressure enhances filling of the right ventricle and hence right ventricular filling pressure. Closure of the valve is delayed. The converse is true during expiration. Normal splitting is invariably present in healthy individuals < 30 years old. After this age , splitting has an increasing chance of being abnormal. Fixed splitting occurs when there is failure of splitting to close during expiration and appears split throughout the respiratory cycle. It is caused by either delayed closure of the pulmonary valve (acute right heart strain, pulmonary embolus, right bundle branch block, atrial septal defect, or pulmonary stenosis) or early closure of the aortic valve. Reversed or Paradoxical splitting is when splitting increases on expiration and decreases during inspiration. It may be caused by delayed closure of the aortic valve and never occurs in the absence of cardiac disease. Aortic stenosis and left bundle branch block constitute 25% of the conditions; Coronary artery disease and hypertension are others. References ALPERT,J.S &RIPPE,J.M; Manual of Cardiovascular Diagnosis and Therapy, Little Brown & Co., 3rd Ed., 1988, pp 5-8.

392

Number: 671 Which of the following is/are indicative of SEVERE Aortic Stenosis (AS)? 1. A long ejection systolic murmur. 2. A peak gradient of 100 mm Hg. 3. A history of Stokes Adams attacks. 4. Cardiomegaly on CXR. A: 1,2,3 Correct B: 1,3 Correct C: 2,4 Correct D: 4 Correct E: All Correct ABCDE Correct Answer: A Cardiomegaly is not necessarily present unless there is also volume overload of the LV. A gradient at rest of 100 mmHg would be considered as severe, although it should be remembered that this measurement is CO dependent. An absolute valve area of 0.75 cm2 is usually taken as severe AS. Stokes Adams attacks, angina, and pulmonary oedema are always taken seriously. Aortic stenosis is classified as valvular, subvalvular, or supravalvular based on the anatomic location of the stenotic lesion. Pure valvular aortic stenosis is the most common, accounting for more than 75% of cases. In the past, rheumatic valvular degeneration was the primary cause. More recently, calcific degeneration of a congenitally bicuspid aortic valve has emerged as the most common aetiology. This anomaly is present in 1-2% of the population. Senile degeneration of a normal valve also occurs. 30% of patients >85 years are found to have significant degeneration of the aortic valve at autopsy. Patients with rheumatic aortic valve disease may be asymptomatic for 40 years or more. Patients with congenitally bicuspid valves may develop symptomatic stenosis anywhere between the ages of 15-65, but calcification of the valve more often occurs after 30 and usually in the 7th and 8th decades. The onset of the triad of symptoms of angina, syncope and exertional dyspnoea is ominous and indicates a life expectancy of < 5 years. (1) Angina pectoris is the initial symptom in 50-70% of patients with severe aortic stenosis. It develops due to a progressive imbalance between myocardial oxygen supply and demand in a hypertrophied ventricle. Concurrent coronary disease may also exist. Angina secondary to the stenotic valve is commonly exertional. Angina occurring at rest commonly indicates coronary disease. These patients are at high risk of subendocardial ischaemia, ventricular dysrhythmias, and sudden death. (2)Syncope is the first symptom in 15-30 %, and is associated with a life expectancy of 3-4 years.

393

(3)Exertional dyspnoea and other stigmata of congestive cardiac failure carry a life expectancy of 2 years. Quantitative criteria for severity include peak systolic pressure gradient and effective aortic valve orifice size: values greater than 50 mm Hg and less than 0.7 cm2 respectively are associated with a 72% 5 year mortality. References: The Washington Manual - Manual of Medical Therapeutics, Little Brown & Co., 27th Ed., 1992, p 117. HENSLEY,F.A; The Practice Of Cardiac Anaesthesia, Little Brown & Co.,1990, pp 351-3. Number: 673 A 40 year old man with a long history of alcohol abuse presents to an emergency department. He is emaciated and difficult to rouse. On closer examination you note that he has an impairment of his lateral gaze. Given this scenario, which vitamin deficiency is the most likely cause of his signs? A. Folic Acid. B. Thiamine. C. Pyridoxine. D. Cyanocobalamin. E. Niacin. Select the single best answer ABCDE Correct Answer: B In developed nations, thiamine deficiency occurs principally in alcoholics or food faddists. In chronic alcoholics it is mainly due to low thiamine intake and impaired thiamine absorption. The two major manifestations of thiamine deficiency are 'wet' and 'dry' beriberi - involving the cardiovascular and nervous systems respectively. 'Dry' beriberi is manifest as either Wernicke's encephalopathy or Korsakoff's syndrome. Wernicke's encephalopathy is a syndrome of vomiting, nystagmus (horizontal more commonly than vertical), rectus muscle palsy progressing to ophthalmoplegia, fever, ataxia and a global confusional state. It may progress to coma and death. Korsakoff's syndrome consists of retrograde amnesia, impaired ability to learn, and confabulation.

394

Number: 674 A patient presents with a pulse of 150 / min. Carotid sinus massage causes it to slow initially, then return to its original rate. The underlying rhythm may be: 1. Sinus tachycardia. 2. Paroxysmal atrial tachycardia. 3. Atrial fibrillation. 4. Orthodromic SVT associated with Wolff-Parkinson-White syndrome. A: 1,2,3 Correct B: 1,3 Correct C: 2,4 Correct D: 4 Correct E: All Correct ABCDE Correct Answer: B The role of manoeuvres which stimulate vagal tone is to differentiate dysrhythmias which are generated from sites above the AV node as opposed to those from the AV node, and to terminate the latter. Dysrhythmias generated from the AV node often involve a re-entrant pathway. Increased vagal tone may delay conduction through this site and terminate the reentrant behaviour. A return to sinus rhythm may occur. In contrast, when vagal tone is increased during other tachyarrhythmias, ventricular rate will be slowed transiently. The effect of carotid sinus massage and other vagal manoeuvres (ocular pressure, valsalva manoeuvres) is to accentuate vagal tone in the heart. The effect of this is: -Rate-spontaneous discharge of the sinus node is markedly slowed. -Conduction-is delayed throughout the entire conducting system. -Contractility is mildly reduced. In the presence of tachyarrhythmias this effect may intervene transiently, or until the manoeuvre is completed. The exception is rhythms caused by re-entrant or circus rhythms in the atrioventricular node, where transient slowing of conduction by increased vagal tone may arrest the re-entrant cycle, restoring the normal rhythm or underlying rhythm. Hence, sinus tachycardia may transiently slow then speed up again as will AF. PAT may be arrested completely causing a return to SR. In orthodromic SVT in WPW the anterograde path of the re-entrant pathway is the AVN so it will behave like PAT. Antidromic SVT is again similar . In AF or Flutter the degree of block may be increased transiently causing slowing which reveals the underlying flutter waves. References The Washington Manual- Manual of Medical Therapeutics, Little Brown & Co., 27th Ed., 1992, pp 128-44.

395

ECHO TESTS Number: 4 The structure labelled 'A' in this 2D, mid-oesophageal echo image is: A. The left main coronary artery. B. The right main coronary artery. C. The left anterior descending artery. D. The circumflex artery. E. The first diagonal artery Select the single best answer

ABCDE Correct Answer: D

396

The division of the left main coronary artery into the left anterior descending artery and the circumflex artery is not usually visualised. However, in this patient, the left main trunk was very short, and the division into the anterior LAD and more posterior Cx is well seen. New techniques for transoesophageal imaging of coronary arteries are becoming available. See: Wild PS, Zotz RJ. Fragment reconstruction of coronary arteries by transesophageal echocardiography: a method for visualizing coronary arteries with ultrasound. Circulation. 2002 Apr 2;105(13):1579-84. Number: 6 This pulsed wave Doppler image of the mitral inflow of a 50 year old man is most compatible with a pattern of: A. Impaired relaxation. B. Restrictive inflow. C. Normality. D. Moderate mitral stenosis (MS). E. Severe mitral incompetence. Select the single best answer

397

ABCDE Correct Answer: A This is a fairly typical picture of impaired ventricular relaxation. The deceleration time is prolonged at 290 msecs (Normal 160 - 240 msecs) and the 'E' veolocity is less than the 'A'. In a fifty year old, the peak 'E' velocity should still exceed the peak 'A' velocity. The normal pattern is one where the 'E' wave velocity is greater than the 'A' wave velocity and the deceleration time is in the range 160-240 msecs. The restrictive pattern is one where the 'E' wave velocity is considerably greater than the 'A' wave velocity and the deceleration time is less than 160 msecs. The pressure half-time is incompatible with mitral stenosis (as it corresponds to a mitral valve area of about 2.5 sq cms). (Note also the virtual absence of acoustic broadening and the presence of an 'A' wave - both of which are against the diagnosis of MS) 398

The maximum 'E' velocity of 50 cm/sec is incompatible with mitral incompetence - where values of more than 1.5 m/sec are often observed. See: Rakowski H, Appleton C, Chan KL et al. Canadian consensus recommendations for the measurement and reporting of diastolic dysfunction by echocardiography: from the Investigators of Consensus on Diastolic Dysfunction by Echocardiography. J Am Soc Echocardiogr. 1996 Sep-Oct;9(5):736-60. Number: 79 This descending thoracic aortic pulsed wave doppler trace is consistent with: 1. A patient with severe aortic incompetence. 2. A patient on full cardio-pulmonary bypass. 3. A patient with a patent ductus arteriosus. 4. A normal patient undergoing general anaesthesia. A: 1,2,3 Correct B: 1,3 Correct C: 2,4 Correct D: 4 Correct E: All Correct

399

ABCDE Correct Answer: D The trace is consistent with a normal patient undergoing general anaesthesia. Note the absence of diastolic flow reversal (which would suggest either a patient with severe aortic incompetence or a patient with a patent ductus arteriosus) and the fact that pulsatile rather than continuous flow is occurring - which would suggest that the patient is on full cardio-pulmonary bypass.

400

Number: 85 The structure labeled 'A' in this 2D mid-oesophageal view is: A. The main pulmonary artery. B. The coronary sinus. C. The left atrial appendage. D. The left pulmonary artery. E. The right pulmonary artery. Select the single best answer

ABCDE Correct Answer: E In this typical long-axis view through the aortic valve, the aorta is seen arching over the right pulmonary artery.

401

OBSTETRICS TESTS Number: 20 The plasma volume of a normal primiparous patient in the 32nd week of pregnancy exceeds plasma volume in the non-pregnant state by: A. 0 - 10%. B. 10 -20%. C. 20 -30%. D. 30 - 40%. E. 40 - 50%. Select the single best answer ABCDE Correct Answer: E Cilberto and Marx have written a comprehensive, on-line review of the physiological changes associated with pregnancy. According to these authors, "Blood volume increases progressively from 6-8 weeks gestation (pregnancy) and reaches a maximum at approximately 32-34 weeks with little change thereafter. Most of the added volume of blood is accounted for by an increased capacity of the uterine, breast, renal, striated muscle and cutaneous vascular systems, with no evidence of circulatory overload in the healthy pregnant woman. The increase in plasma volume (40-50%) is relatively greater than that of red cell mass (20-30%) resulting in hemodilution and a decrease in haemoglobin concentration. Intake of supplemental iron and folic acid is necessary to restore hemoglobin levels to normal (12 g/dl). The increased blood volume serves two purposes. First, it facilitates maternal and fetal exchanges of respiratory gases, nutrients and metabolites. Second, it reduces the impact of maternal blood loss at delivery. Typical losses of 300-500 ml for vaginal births and 750-1000 ml for Caesarean sections are thus compensated with the so-called "autotransfusion" of blood from the contracting uterus." Number: 26 An otherwise fit, primiparous patient is undergoing emergency caesarean section under general anaesthesia. The extraction of the infant is proving difficult and the obstetrician requests that the patient be given glyceryl trinitrate (GTN) intravenously in order to facilitate uterine relaxation. The most appropriate initial dose of GTN is:

402

A. 25 micrograms. B. 50 micrograms. C. 250 micrograms. D. 1 mg. E. 2.5 mg. Select the single best answer ABCDE Correct Answer: C Caponas has recently written a comprehensive review on the role of glyceryl trinitrate as a uterine relaxant. He noted that "The dosage of GTN used in published reports varies from 50 micrograms to 1850 micrograms. Generally intravenous GTN 100 micrograms to 500 micrograms and sublingual GTN 400 micrograms to 800 micrograms (1-2 metered sprays) was administered." However, it should also be noted that the author also concluded that "The evidence supporting the acute administration of GTN in producing uterine relaxation to assist in the resolution of obstetric emergencies is inconsistent. The traditionally held view that GTN releases NO, with subsequent effects of NO on uterine smooth muscle acting via a cGMP mediated process in effecting uterine relaxation, is not borne out by all laboratory work." and that "A randomized controlled trial conducted in the nonemergency setting would require sufficient numbers of labouring women with intrauterine pressure transducers to monitor the potential effects of GTN and placebo. Until such trials are performed, clinicians seeking potentially life-saving management options are faced with reliance on clinical reports, in the presence of conflicting clinical and experimental data." Thus, although the dosage of the drug seems to have been established, the efficacy of this form of treatment remains uncertain. See: Caponas G. Glyceryl trinitrate and acute uterine relaxation: a literature review. Anaesth Intensive Care. 2001 Apr;29(2):163-77. Number: 33 The 'D' antigen can be found on the erythrocytes of Rh-positive foetuses by the: A. 6th week of gestation. B. 10th week of gestation. C. 14th week of gestation. D. 18th week of gestation. E. 20th week of gestation. Select the single best answer ABCDE

403

Correct Answer: A The 'D' antigen can be found on the erythrocytes of Rh-positive foetuses by the 6th week of gestation. As foeto-maternal transfusions occur in the majority of pregnancies, from the 6th week of pregnancy, blood containing Rhesus antigens may be infused into the maternal circulation and cause sensitisation in Rh negative women. In case of miscarriage, abortion, ectopic pregnancy and cystic mole the chance of foetomaternal transfusion followed by sensitisation of the mother is significantly increased. Therefore, in such cases, immunoprophylaxis with anti-D-immunoglobins should be performed in all Rhesus negative women.. Rh haemolytic disease is the prototype of maternal alloimmunization and fetal haemolytic disease although there are other antigens capable of causing alloimmunisation and haemolytic disease such as 'C', Kell, and Fya. Rh immunisation is usually caused by a prior Rh positive foetal maternal transplacental haemorrhage, which occurs in at least 75% of pregnancies. See: Maas DH. Anti-D prophylaxis after abortions and interruptions. Fortschr Med. 1979 Jan 25;97(4):148-52. Bowman J. The management of hemolytic disease in the fetus and newborn. Semin Perinatol. 1997 Feb;21(1):39-44. Chalmers I, Enkin M, Keirse MJNC. Effective care in pregnancy and childbirth. Oxford: Oxford University Press, 1989 pp. 569-571, 605 Number: 39 The typical arterial PCO2 (PaCO2) of a normal, non-labouring woman at term is: A. 44 mm Hg. B. 40 mm Hg. C. 36 mm Hg. D. 32 mm Hg. E. 28 mm Hg. Select the single best answer ABCDE Correct Answer: D Minute ventilation at term is increased by about 50% - primarily due to an increased tidal volume with little change, or at most a slight increase, in respiratory rate. As a result of this increased alveolar ventilation at term, maternal PaCO2 is usually decreased to about 32 mm

404

Hg, but little maternal alkalosis occurs because of a compensatory decrease in serum bicarbonate of about 4 mmol/L(from 26 to 22 mmol/L). See: Templeton A, Kelman GR. Maternal blood-gases, (PAo2--Pao2), physiological shunt and VD/VT in normal pregnancy. Br J Anaesth. 1976 Oct;48(10):1001-4. Number: 39 The typical arterial PCO2 (PaCO2) of a normal, non-labouring woman at term is: A. 44 mm Hg. B. 40 mm Hg. C. 36 mm Hg. D. 32 mm Hg. E. 28 mm Hg. Select the single best answer ABCDE Correct Answer: D Minute ventilation at term is increased by about 50% - primarily due to an increased tidal volume with little change, or at most a slight increase, in respiratory rate. As a result of this increased alveolar ventilation at term, maternal PaCO2 is usually decreased to about 32 mm Hg, but little maternal alkalosis occurs because of a compensatory decrease in serum bicarbonate of about 4 mmol/L(from 26 to 22 mmol/L). See: Templeton A, Kelman GR. Maternal blood-gases, (PAo2--Pao2), physiological shunt and VD/VT in normal pregnancy. Br J Anaesth. 1976 Oct;48(10):1001-4. Number: 41 Which of the following parameters support improved neonatal outcome from epidural/spinal anaesthesia as compared with general anaesthesia for caesarean section ? A. Apgar score. B. Acid-base status. C. Neurobehavioural score (ENNS or ABS). D. All of the above. E. None of the above. Select the single best answer ABCDE Correct Answer: E

405

Historically, it has been believed that regional anaesthetic techniques were associated with less neonatal depression than general anaesthesia for delivery by LSCS. Those parameters studied have included: (1) Apgar scores (2) Acid-base status (3) Time to sustained respiration (4) Neurobehavioural scores More recently, these parameters have been challenged and no significant correlation has been found to favour any one technique. The literature of the 60's and 70's suggested that the 1 minute Apgar scores were higher with regional than with general anaesthetic techniques, whereas 5 minute scores tended not to be statistically significant. Studies done in the late 70's and early 80's looked at Apgar scores with halothane, methoxyflurane, trilene, and N2O compared with epidural anaesthesia. There were no statistically significant differences in the 1-minute and 5-minute scores when any general and regional anaesthetic technique was compared in elective caesarian section. Similarly, improved acid-base status has been a proposed advantage of regional anaesthesia. Recent studies have shown differences between maternal artery and uterine vein pH to be essentially negligible. The neonate should have a sustained respiratory pattern in less than 60 seconds. All studies in the 70's showed that although time to sustained respirations may be a few seconds faster with regional than general anaesthesia, all occurred within 60 seconds. A recently developed index of neonatal depression is the neurobehavioural scoring scale. The early neonatal neurobehavioural score (ENNS) was adapted for use in the operating theatre by a paediatrician called Scanlon from the Brazelton paediatric scoring scale. This has recently been modified by Amiel-Tielsen, Barrier and Schnider (ABS) to a less invasive but still complete scoring scale. As with the other parameters, no differences in foetal outcome could be ascribed to either technique using neonatal neurobehavioural scores. See: Joyce,T.H; " Regional versus General Anaesthesia- Any Advantage? ", Seminars in Anesthesia, vol 1, no 2, June 1982. Anesth Analg 1995 Jul;81(1):90-5 Gambling DR, Sharma SK, White PF, Van Beveren T et al Use of sevoflurane during elective cesarean birth: a comparison with isoflurane and spinal anesthesia. Number: 43 The administration of 0.5% halothane as a supplement to nitrous oxide during Lower Segment Caesarean Section is associated with: A. Increased post-partum blood loss. B. Depressed neonatal neurobehavioural scores for several hours. C. Decreased sensitivity to oxytocin.

406

D. Awareness. E. None of the above. Select the single best answer ABCDE Correct Answer: E The use of halogenated agents-low dose halothane (0.5%); isoflurane (0.75%); or enflurane (1%) as supplements to N2O is very common. The N2O concentration is usually reduced to 50%. The halogenated agents decrease the incidence of maternal awareness, permit higher maternal inspired oxygen tension; may improve uterine blood flow; and do not depress the neonate. Whilst they decrease uterine muscle tone, the uterus remains immediately responsive to oxytocin and postpartum blood loss does not increase. Concern has been raised that the halogenated agents may decrease uterine muscle tone resulting in increased postpartum blood loss. They produce a dose dependent decrease in uterine contractility and tone, however several studies have failed to reveal any increased blood loss with low dose halothane (0.1-0.8%); enflurane (0.5-1.5%),or isoflurane (0.75%), during caesarian section. At these low concentrations, the uterus is immediately responsive to oxytocin. When higher concentrations are used, blood loss is still not clinically significantly raised , and this may be utilized when higher concentrations of oxygen are desired. Moreover, clinical experience indicates that the slight increase in maternal depth is not reflected in the neonate at birth. Several surveys have reported a high incidence of maternal awareness and postoperative recall of intraoperative events with subsequent unpleasant experiences, such as nightmares, associated with the use of N2O/O2 and muscle relaxant techniques. The incidence of awareness appears to vary inversely with the concentration of N2O. In one study, approximately 9% of parturients who received 67% of N2O in O2 were aware compared with 26% in the 50% N2O group. No awareness has ben reported if halothane 0.1-0.65%, enflurane 0.5-1.5%, methoxyflurane 0.1%, or isoflurane 0.75% is added to 50% N2O before delivery. References Schnider, S.M & Levinson, G; Anesthesia for Obstetrics, 3rd Ed., 1993, pp 231-235. Number: 44 A pulmonary embolus is suspected in a 36 week gestation woman: 1. Hypoxaemia is predominantly caused by an increase in dead space ventilation. 2. Pulmonary angiography is considered safer for the foetus than V/Q scanning. 3. Warfarin is contraindicated because it is teratogenic. 4. Thrombolytic agents are contraindicated. A: 1,2,3 Correct B: 1,3 Correct C: 2,4 Correct D: 4 Correct

407

E: All Correct ABCDE Correct Answer: D The initial pathophysiological effect of a pulmonary thromboembolus is an increase in dead space ventilation. Regional microatelectasis and pulmonary oedema supervene resulting in intrapulmonary shunting which is exacerbated in the presence of a low cardiac output state. This is reflected by hypoxaemia which is poorly responsive to oxygen supplementation. Ventilation/perfusion scans using technetium as the marker, are considered safer than pulmonary angiography and its associated radiation exposure. Warfarin is highly teratogenic in the first trimester. It is contraindicated in the second and third trimesters because of the risk of foetal bleeding. Thrombolytic agents are contraindicated in the treatment of thromboembolism during pregnancy. Epidemiological studies indicate that thromboembolic disease is 5 times more common during pregnancy or in the postpartum period than in non-pregnant women. The most recent large review of maternal death in the USA (1985) identify it as the leading cause of death followed by, in decreasing order, hypertensive disease, ectopic pregnancy, hemorrhage, cerebrovascular accidents and anaesthetic complications. The last Report on Confidential Enquiries into Maternal Deaths in England and Wales for the period 1982-4 also shows that it remains the leading cause of maternal death in the U.K. The aetiology is best described in terms of Virchow's triad: vessel wall trauma, venous stasis, and alterations in the coagulation mechanism. Vessel wall trauma may contribute to some forms of thrombosis, ie. pelvic thrombosis following caesarian section. Venous stasis is a risk factor during pregnancy. Venous distensibility increases during the first trimester of pregnancy. Varicose veins, hormonal changes, anaemia, toxaemia and the hypercoaguable state have also been implicated. Mechanical compression by the gravid uterus on the inferior vena cava causes venous stasis and is uniformly considered to be a major factor contributing to deep venous thrombosis. This begins in the early part of the second trimester. Leg vein obstruction is found to be almost universal in the standing position in the third trimester. Pregnancy causes a number of alterations in the coagulation mechanism. Factors VII, VIII, IX, X, and fibrinogen are increased. Antithrombin III activity falls in patients with hereditary antithrombin III deficiency. Fibrinolytic activity is decreased. Neither the platelet count or adhesiveness is increased. The decreased number of platelets observed after delivery is probably due to normal thrombus formation at the placental site. Pathophysiological events depend upon the size and nature of the embolus, the site of impaction and preexisting cardiovascular disease. A single large embolus may cause sudden fatal obstruction of the pulmonary circulation, or may break up showering the lungs (predominantly the lower lobes) resulting in emboli, infarction, secondary infection, and or adult respiratory distress syndrome (ARDS). Very small emboli may pass to the microcirculation, undergo lysis and cause no respiratory dysfunction. Recurrent emboli may result in pulmonary hypertension and cor pulmonale. Respiratory changes may include hypoxaemia secondary to an increased A-a gradient; and a decreased ETCO2 secondary to an increased dead space gradient. The latter may be the first sign of the event during anaesthesia. Hypoxaemia may initially be caused by an increase in

408

dead space ventilation. Release of humoral factors including histamine and serotonin, and decreased CO2 tensions have been attributed to regional bronchial alveolar constriction and loss of surfactant. This may then cause foci of atelectasis and pulmonary oedema resulting in intrapulmonary shunting. This explains why hypoxaemia may not improve despite oxygen supplementation. Falls in cardiac output consequent to right heart strain will exacerbate intrapulmonary shunting. If the patient is awake this may manifest as dyspnoea, tachycardia, anxiety, confusion, , syncope, fever, or sweating. During anaesthesia, changes in monitored parameters may occur; decreased ETCO2, desaturation, elevation of CVP or PAP/PCWP. The ECG may show signs of right ventricular strain; right bundle branch block, right axis deviation, or S1 Q3 T3 configurations, however the most common changes are sinus tachycardia or other supraventricular tachyarrhythmias consequent to increased right atrial pressures, ie: atrial fibrillation or supraventricular tachycardia. The CXR may show diminished vascular markings, enlargement of the proximal pulmonary vessels, elevation of the hemidiaphragm, pleural effusion, patchy opacities, or wedged shaped opacities. It may alternatively be normal. More accurate is a combined ventilation/perfusion scan. If highly suggestive, therapy may be commenced on the basis of this without a pulmonary angiogram. Nuclear medicine scans can be performed safely however, technetium should be used rather than iodine and uterine shielding is necessary. Pulmonary angiography is usually avoided because of the risk of radiation exposure to the foetus. With respect to abnormalities of skeletal growth, this risk is maximal in the first trimester. Latent bony malignancy can occur in latter life subsequent to significant exposure to radiation in utero or early childhood. It should in principle be minimized. The cornerstone of treatment after supportive measures is anticoagulation. It is now recommended that warfarin not be used at any point during the pregnancy, however, it has been used for long term therapy during the middle stages of the pregnancy. Warfarin is a small molecule that readily crosses the placenta. Exposure during the first trimester has significant teratogenic potential. Used in the third trimester, it can cause pre or intrapartum foetal bleeding. Because of immature liver enzymes, it appears to affect the foetus more profoundly than the mother. An overall foetal mortality of 10-15% has been reported in women taking oral anticoagulants. If warfarin has been used, it is recommended that it is converted to heparin several weeks before delivery is expected. In this group, 3-14 days will be required before the effects of the warfarin subside. No benefit will be gained from administration on maternal IV or intraamniotic vitamin K, or FFP, however, parenteral vitamin K administered to the foetus will normalize clotting factors within 30-48 hours. Heparin is the agent of choice. It does not cross the placenta hence, does not affect the foetus. It has a short half life and can be discontinued within 4-6 hours of the expected delivery. Its specific antagonist protamine may be used in more urgent cases. Thrombolytic agents are contraindicated during pregnancy. References SKERMAN, J.H. & BLASS, N.H;" Management of the Obstetric Patient with Thromboembolic Disease ", Chapter 30, Clinics in Anaesthesiology, vol 4, no 2, April 1986.

409

Number: 45 Which of the following are seen with therapeutic maternal plasma concentrations of MgSO4 during the treatment of pre-eclampsia? 1. Decreased neonatal muscle tone. 2. Widening of QRS complexes on ECG. 3. Loss of maternal deep tendon reflexes. 4. Delayed recovery from non-depolarizing muscle relaxants. A: 1,2,3 Correct B: 1,3 Correct C: 2,4 Correct D: 4 Correct E: All Correct ABCDE Correct Answer: C In many countries, magnesium is the first line treatment of pre-eclampsia/eclampsia. It is usually initiated by IVI administration of 4-6 g over 15 minutes. 2-4 g given more rapidly during eclamptic convulsions frequently terminate them. Therapeutic blood levels of 4-8 mmol/l are then maintained by continuous infusion at a rate of 1-3 g/hour. Magnesium therapy is associated with maternal and neonatal side effects. Serious toxicity may be the result of absolute overdosage or , more frequently, elevation of blood levels, following repeated doses or continuous infusions in the presence of decreased renal function. Most of these develop at plasma levels greater than 10 mmol/l at which loss of deep tendon reflexes occurs. This should be monitored clinically and used as a marker at which the infusion is decreased or ceased. In therapeutic dosages it is associated with abnormal neuromuscular transmissions which correlates with increased serum magnesium levels and decreased serum calcium levels. It increases the sensitivity of the mother to both depolarizing and non-depolarizing muscle relaxants by decreasing release of acetyl choline from the nerve terminal, decreasing the depolarizing action of acetyl choline at the motor endplate, and decreasing the excitability of the muscle fibre membrane. It does not affect the pseudocholinesterase activity. Magnesium readily crosses the placenta, however significant detrimental effects are not seen in the neonate at therapeutic maternal levels. When high maternal levels are reached, the neonate may develop decreased muscle tone, respiratory depression, and apnoea. Below is a guide to the clinical monitoring of serum magnesium levels (their similarity to bupivacaine toxicity may make the numbers easier to remember !). Values are quoted in mEq/L. 1.5 - 2.0 -- normal plasma level 4.0 - 8.0 -- therapeutic plasma level 5.0 - 10.0 -- electrocardiographic changes (P-R interval prolonged, QRS complexes widened) 10 -- loss of deep tendon reflexes

410

15 -- sino-atrial and atrio-ventricular block 15 -- respiratory paralysis 25 -- cardiac arrest References SCHNIDER, S.M. & LEVINSON, G; Anesthesia for Obstetrics, 3rd Ed., 1993, pp 315-6. Number: 99 Which of the following changes to the coagulation system are considered normal with pregnancy? 1. Increased factor VII. 2. Decreased factor XI. 3. Decreased fibrinolysis. 4. Thrombocytopenia. A: 1,2,3 Correct B: 1,3 Correct C: 2,4 Correct D: 4 Correct E: All Correct ABCDE Correct Answer: A Pregnancy has various effects on the coagulation system and has been described as a state of chronic compensated disseminated intravascular coagulopathy. Clotting factors which increase include fibrinogen, factor VII, VIII, VIII antigen, VIII von-Willebrand, X, and XII. Fibrinogen is elevated approximately twice normal levels at 20 weeks and remains elevated throughout pregnancy. Factor VII appears to increase 200% in the second trimester and also remains elevated. All components of VII peak and plateau in the third trimester. X increases 200% in the third trimester, and XII may increase modestly. Factors that decrease include XI, and XIII. These decrease to approximately 70% of normal by the third trimester. There is a slight increase in FDPs that reflects increased fibrin formation. Although controversial, it appears that the platelet count and functional capacity are unchanged, however, small falls have been observed. It has been suggested that the normal range is lower in pregnancy. Regardless, thrombocytopenia is a term used to describe an abnormally low platelet count. Antithrombin III is unchanged in normal pregnancy. John Bonnar has written extensively on the subject of haemostasis in pregnancy. JANES, S.L;" Thrombocytopenia in Pregnancy ", Postgrad Med J, 68, 1992, pp 321-6.

411

PERRY,G.P & MARTIN,J.N;" Abnormal hemostasis and coagulopathy in pre-eclampsia and eclampsia ", Clinics of Obstetrics and Gynaecology, vol 35, no 2, June 1992. Number: 111 A woman presents at 28 weeks gestation for emergency appendicectomy. Which of the following agents should be avoided? 1. Halothane. 2. Neostigmine. 3. Atropine. 4. Ketorolac (Toradol). A: 1,2,3 Correct B: 1,3 Correct C: 2,4 Correct D: 4 Correct E: All Correct ABCDE Correct Answer: D The basic objectives of anaesthetic management of pregnant women undergoing general surgery are: (1) Maternal safety. (2)Avoidance of teratogenic drugs. (3)Avoidance of intrauterine foetal asphyxia. (4)Prevention of preterm labour. No anaesthetic agent- premedicant, induction, inhalation, or local anaesthetic has been proven to be teratogenic in humans, however recommendations are to minimize foetal exposure in first trimester. In no study has any anaesthetic agent or technique been found to be associated with a higher or lower risk of preterm labour. Some agents carry a theoretical risk and should be avoided when possible. These include ketamine > 1.1 mg/Kg, some vasopressors, and rapid injections of neostigmine and edrophonium (increase uterine tone). With regard to pretrerm labour, the incidence rises from 5.13% without surgery to 7.4% with surgery. It is largely related to the type of surgery performed. For instance, the incidence is very high following insertion of a Shirodker suture (30%). Apart from this, intra-abdominal procedures involving retraction of the uterus carry the greatest risk, whilst neurosurgical, orthopedic, and plastic procedures carry a smaller risk. Prostaglandin inhibitors may cause early closure of the ductus arteriosus and should be avoided.

412

SCHNIDER, S.M. & LEVINSON, G; Anesthesis for Obstetrics, 3rd Ed., Williams and Wilkins, 1993, pp 275-6. Number: 411 Placenta praevia: 1. Is more common in multiparous patients. 2. Can be an indication for classical Caesarean section. 3. Increases the risk of postpartum haemorrhage. 4. Is graded I - V. A: 1,2,3 Correct B: 1,3 Correct C: 2,4 Correct D: 4 Correct E: All Correct ABCDE Correct Answer: A Placental implantation in the lower uterine segment occurs in 1 of every 200 pregnancies. The multiparous patient is at greater risk than is the primigravida. The incidence of recurrence in a subsequent pregnancy is approximately 5%. Placenta praevia varies in degree and may be complete (37%), partial (27%), or marginal (low implantation; 46%) - defined by the degree of encroachment onto the internal os. The main symptom is painless vaginal bleeding in the third trimester of pregnancy. Placenta praevia can be an indication for classical caesarean section and significantly increases the risk of postpartum haemorrhage. Anticipate placenta accreta which often accompanies placenta praevia (Clin. Obs. Gyn 33:414, 1990) Number: 616 A nulliparous woman in early labour with no other risk factors asks your advice on the possible impact of epidural analgesia on maternal outcome. Your advice is that the technique: A. Has no adverse effect. B. May prolong labour, but otherwise has no adverse effect. C. May prolong labour and increase the instrumental delivery rate. D. May prolong labour and increase the Caesarean section rate. E. None of the above. Select the single best answer

413

ABCDE Correct Answer: C (WMD=Weighted Mean Difference, OR=Odds Ratio, CI=Confidence Interval). An important meta-analysis by Halpern has concluded that: The risk of caesarean delivery does not differ between patients receiving epidural (8.2%) vs parenteral opioid (5.6%) analgesia (OR, 1.5; 95% CI, 0.81-2.76). Epidural patients have longer first (WMD, 42 minutes; 95% CI, 17-68 minutes) and second (WMD, 14 minutes; 95% CI, 5-23 minutes) labour stages. Epidural patients are more likely to have instrumented delivery (OR, 2.19; 95% CI, 1.327.78), but they are not more likely to have instrumented delivery for dystocia (OR, 0.68; 95% CI, 0.31-1.49). Thus it seems likely that epidural analgesia prolongs labour and increases the instrumental delivery rate. Number: 666 Hyperemesis in the first trimester of pregnancy may be associated with: 1. Multiple pregnancy. 2. Helicobacter pylori infection. 3. Hydatiform mole. 4. Diabetes mellitus. A: 1,2,3 Correct B: 1,3 Correct C: 2,4 Correct D: 4 Correct E: All Correct ABCDE Correct Answer: A Hyperemesis gravidarum, is usually associated with a marked increase Human Chorionic Gonadotrophin (HCG) level - such as occurs in cases of multiple pregnancy or hydatiform mole. More recently, an association between helicobacter pylori infection and hyperemesis has been recognised. (See, for example: Jacoby EB, Porter KB. Helicobacter pylori infection and persistent hyperemesis gravidarum. Am J Perinatol. 1999;16(2):85-8.)

414

Number: 680 The properties of a drug that would tend to favour trans-placental diffusion from the mother to the foetus include: 1. A high degree of ionisation 2. A molecular weight < 50 daltons 3. Extensive maternal protein binding 4. High lipid solubility A: 1,2,3 Correct B: 1,3 Correct C: 2,4 Correct D: 4 Correct E: All Correct ABCDE Correct Answer: C Drugs cross the placenta by simple diffusion, with the rate of transfer being principally determined by: the concentration gradient, lipid solubility, degree of ionisation, and molecular size. Lipophilic compounds cross much more easily than hydrophilic compounds and, because unionised drugs are considerably more lipophilic than those that exist in the ionised form, they also cross much more readily. Smaller compounds also cross the placenta with greater ease, and drugs with a molecular weight of less than 50 daltons cross the placenta virtually unimpeded. A high degree of maternal protein binding tends to impede the placental transfer of a substance. Number: 713 With regard to placental abruption ('Abruptio Placentae'): 1. The perinatal mortality rate is more than 50%. 2. Abruption is typically accompanied by bright vaginal bleeding. 3. The incidence is increased in patients with hypertensive disease. 4. Augmentation of labour (in order to expedite vaginal delivery) is usually indicated. A: 1,2,3 Correct B: 1,3 Correct C: 2,4 Correct D: 4 Correct E: All Correct

415

ABCDE Correct Answer: B Abruptio placenta carries a maternal mortality of 2 to 3% and a perinatal mortality of over 50%. It usually occurs in the final 10 weeks of pregnancy in association with hypertensive disease of pregnancy. The diagnosis is suggested by uterine tenderness and hypertonus in the presence of dark vaginal bleeding. Haemorrhage may be severe, and is quite often concealed. Artificial rupture of the membranes and augmentation of labour may be acceptable in mild cases, but emergency caesarean section is indicated in the presence of foetal distress or severe haemorrhage. A coagulopathic state due to disseminated intravascular coagulation and / or fibrinolysis frequently complicates severe cases of abruption. Number: 798 The most common early sign of amniotic fluid embolism (AFE) following a normal vaginal delivery is: A. Skin rash. B. Hypertension. C. Fitting. D. Abnormal bleeding. E. Cyanosis. Select the single best answer ABCDE Correct Answer: E The overall incidence of amniotic fluid embolus ranges from 1 in 8,000 to 1 in 80, 000 pregnancies, with a maternal mortality of up to 86%. Cyanosis and respiratory distress are the commonest early presenting signs of AFE. These are shortly followed by cardiovascular collapse +/- coma. In addition, prodromal chills, sweating, coughing, hyperreflexia, and convulsions occasionally occur. Although the classic opinion holds that a haemorrhagic tendency is likely only if the patient survives beyond the first hour, abnormal bleeding was the presenting feature in 12% of cases reviewed by Morgan in 1979. For a more recent review, see: Am J Obstet Gynecol 1995 Apr;172(4 Pt 1):1158-67; discussion 1167-9 Amniotic fluid embolism: analysis of the national registry. Clark SL, Hankins GD, Dudley DA, Dildy GA, Porter TF

416

Number: 863 Intrapulmonary shunt(Qs/Qt) in a pregnant, normotensive woman in the lateral position at term is approximately: A. 3% B. 5% C. 10% D. 15% E. 20% Select the single best answer ABCDE Correct Answer: D Hankins et al measured Qs/Qt in a group of pregnant volunteers at term. They found that directly measured Qs/Qt averaged 15.3% in the left lateral, 15.2% in the right lateral, 13.9% in the supine, 12.8% in the knee-chest, 13.8% in the sitting, and 13.0% in the standing position. This represented a marked increase in venous admixture in comparison with the non-pregnant state. See: Hankins GD, Clark SL, Uckan E, Van Hook JW. Maternal oxygen transport variables during the third trimester of normal pregnancy. Am J Obstet Gynecol. 1999 Feb;180(2 Pt 1):406-9. Number: 871 Ritodrine infusion (for tocolysis) may be associated with: 1. Hypokalaemia. 2. ECG changes. 3. Hyperglycemia. 4. Hypomagnesaemia A: 1,2,3 Correct B: 1,3 Correct C: 2,4 Correct D: 4 Correct E: All Correct ABCDE Correct Answer: A

417

Maternal side effects associated with the USE of ritodrine include: hyperglycemia, hypokalaemia and various ECG changes (ST segment depression and 'peaking' of the 'T' wave). Importantly, there are also some 'rebound' effects associated with the CESSATION of ritodrine - of these, extreme hyperkalaemia is probably the most significant, but hypoglycemia has also been reported. In a series of cases reported by Kotani et al, these authors remarked that "Ritodrine administration frequently causes hypokalemia because of a marked increase in plasma insulin, which results in stimulation of cellular uptake of potassium. The decrease in insulin after cessation of ritodrine leads to a considerable change in the equilibration between intracellular and extracellular potassium, and might produce a rebound release of intracellular potassium to the extracellular space. Similarly, an intraoperative glucose-insulin-potassium infusion for inotropic support can cause a life-threatening hyperkalemia postoperatively. We found that hyperkalemia was maximal 90150 minutes after cessation of ritodrine, which is compatible with the 60120 minutes of plasma half-life of ritodrine. According to Braden et al., intraoperative factors including changes in acid-base balance and plasma aldosterone do not modulate plasma potassium levels. " See: Anesth Analg 2001 Sep;93(3):709-711 Rebound perioperative hyperkalemia in six patients after cessation of ritodrine for premature labor. Kotani N, Kushikata T, Hashimoto H, Muraoka M, Tonosaki M, Matsuki A. Number: 878 The most appropriate anticonvulsant for the treatment of an eclamptic fit is: A. Thiopentone. B. Phenytoin. C. Diazepam. D. Magnesium sulphate. E. Lorazepam. Select the single best answer ABCDE Correct Answer: D Refer to the important conclusions of the Collaborative Eclampsia Trial and the Cochrane Review by Duley and Gulmezoglu. See: Which anticonvulsant for women with eclampsia? Evidence from the Collaborative Eclampsia Trial. Lancet. 1995 Jun 10;345(8963):1455-63. Duley L, Gulmezoglu AM. Magnesium sulphate versus lytic cocktail for eclampsia (Cochrane Review).

418

Cochrane Database Syst Rev. 2001;1:CD002960. The authors of the collaborative review noted that: "Eclampsia, the occurrence of a seizure in association with pre-eclampsia, remains an important cause of maternal mortality. Although it is standard practice to use an anticonvulsant for management of eclampsia, the choice of agent is controversial and there has been little properly controlled evidence to support any of the options. 1687 women with eclampsia were recruited into an international multicentre randomised trial comparing standard anticonvulsant regimens. Primary measures of outcome were recurrence of convulsions and maternal death. Data are available for 1680 (99.6%) women: 453 allocated magnesium sulphate versus 452 allocated diazepam, and 388 allocated magnesium sulphate versus 387 allocated phenytoin. Most women (99%) received the anticonvulsant that they had been allocated. Women allocated magnesium sulphate had a 52% lower risk of recurrent convulsions (95% CI 64% to 37% reduction) than those allocated diazepam (60 [13.2%] vs 126 [27.9%]; ie, 14.7 [SD 2.6] fewer women with recurrent convulsions per 100 women; 2p < 0.00001). Maternal mortality was non-significantly lower among women allocated magnesium sulphate. There were no significant differences in other measures of serious maternal morbidity, or in perinatal morbidity or mortality. Women allocated magnesium sulphate had a 67% lower risk of recurrent convulsions (95% CI 79% to 47% reduction) than those allocated phenytoin (22 [5.7%] vs 66 [17.1%] ie, 11.4 [SD 2.2] fewer women with recurrent convulsions per 100 women; 2p < 0.00001). Maternal mortality was nonsignificantly lower among women allocated magnesium sulphate. Women allocated magnesium sulphate were also less likely to be ventilated, to develop pneumonia, and to be admitted to intensive care facilities than those allocated phenytoin. The babies of women who had been allocated magnesium sulphate before delivery were significantly less likely to be intubated at the place of delivery, and to be admitted to a special care nursery, than the babies of mothers who had been allocated phenytoin. There is now compelling evidence in favour of magnesium sulphate, rather than diazepam or phenytoin, for the treatment of eclampsia."

Number: 902 The glomerular filtration rate (GFR) of a normal primiparous patient at term exceeds GFR in the non-pregnant state by: A. 10 -20%. B. 20 -30%. C. 30 - 40%. D. 40 - 50%. E. 50 - 60%. Select the single best answer ABCDE Correct Answer: E Cilberto and Marx have written a comprehensive, on-line review of the physiological changes associated with pregnancy.

419

According to these authors, "Renal plasma flow and glomerular filtration rate begin to increase progressively during the first trimester. At term, both are 50-60% higher than in the non-pregnant state. This parallels the increases in blood volume and cardiac output. The elevations in plasma flow and glomerular filtration result in an elevation in creatinine clearance. Blood urea and serum creatinine are reduced by 40%. The increase in glomerular filtration may overwhelm the ability of the renal tubules to reabsorb leading to glucose and protein losses in the urine. Thus, mild glycosuria (1-10 gm/day) and/or proteinuria (to 300 mg/day) can occur in normal pregnancy. There is also an increase in filtered sodium, but tubular absorption is increased by an increase in aldosterone secretion, via the reninangiotensin mechanism (see Physiology of the Kidney). There is also a decrease in plasma osmolality. This is a measure of the osmotic activity of a substance in solution and is defined as the number of osmoles in a kilogram of solvent. In practice it indicates that the plasma concentrations of electrolytes, glucose and urea, fall if more water than sodium, for example, is retained. Over the whole period of gestation there is retention of 7.5L of water and 900 mmol of sodium." Number: 904 The Functional Residual Capacity (FRC) of a normal primiparous patient at term is reduced (in comparison with the non-pregnant state) by: A. 10%. B. 20%. C. 30%. D. 40%. E. 50%. Select the single best answer ABCDE Correct Answer: B Cilberto and Marx have written a comprehensive, on-line review of the physiological changes associated with pregnancy. According to these authors, "Upward displacement by the gravid uterus causes a 4 cm elevation of the diaphragm, but total lung capacity decreases only slightly because of compensatory increases in the transverse and antero-posterior diameters of the chest, as well as flaring of the ribs. These changes are brought about by hormonal effects that loosen ligaments. Despite the upward displacement, the diaphragm moves with greater excursions during breathing in the pregnant than in the non-pregnant state. In fact, breathing is more diaphragmatic than thoracic during gestation, an advantage during supine positioning and high regional blockade. From the middle of the second trimester, expiratory reserve volume, residual volume and functional residual capacity are progressively decreased, by approximately 20% at term. Lung compliance is relatively unaffected, but chest wall compliance is reduced, especially in the lithotomy position"

420

Number: 904 The Functional Residual Capacity (FRC) of a normal primiparous patient at term is reduced (in comparison with the non-pregnant state) by: A. 10%. B. 20%. C. 30%. D. 40%. E. 50%. Select the single best answer ABCDE Correct Answer: B Cilberto and Marx have written a comprehensive, on-line review of the physiological changes associated with pregnancy. According to these authors, "Upward displacement by the gravid uterus causes a 4 cm elevation of the diaphragm, but total lung capacity decreases only slightly because of compensatory increases in the transverse and antero-posterior diameters of the chest, as well as flaring of the ribs. These changes are brought about by hormonal effects that loosen ligaments. Despite the upward displacement, the diaphragm moves with greater excursions during breathing in the pregnant than in the non-pregnant state. In fact, breathing is more diaphragmatic than thoracic during gestation, an advantage during supine positioning and high regional blockade. From the middle of the second trimester, expiratory reserve volume, residual volume and functional residual capacity are progressively decreased, by approximately 20% at term. Lung compliance is relatively unaffected, but chest wall compliance is reduced, especially in the lithotomy position" Number: 945 In relation to foetal outcome following Caesarean section, which is the most important? A. Uterine incision-delivery time. B. Displacement of the uterus. C. Minimal thiopentone on induction. D. The use of 100% oxygen. E. Minimal use of inhalational agent. Select the single best answer

421

ABCDE Correct Answer: A Much literature exists examining this issue. Many of the controversies in results have arisen secondary to criticisms of the selection of parameters which have been correlated with neonatal morbidity. It has perhaps been better clarified more recently as a result of a strengthening correlation between neonatal depression and acid-base status The relationship between induction-to-delivery and uterine incision-to-delivery times to neonatal outcome in 105 parturients undergoing caesarian section was studied by Datta. During general anaesthesia, induction-to-delivery times >8 minutes, and uterine incision-todelivery times > 3 minutes were associated with significantly more instances of neonatal acidosis (umbilical artery pH 7.31 vs 7.22), and a greater incidence of low 1-minute Apgars (4% vs 73%). In groups receiving regional anaesthesia, prolongation of the latter > 3 minutes was the only important factor influencing foetal outcome, as determined by increased acidosis (7.3 vs 7.18), and by depressed 1-minute Apgars (0% vs 62%). In another study, no neonatal morbidity was associated with an induction-delivery time of up to 30 minutes. Crawford and Datta showed that a time in excess of 90-100 sec from the beginning of the surgical incision into the uterus to complete expulsion of the neonate will tend to produce a higher incidence of depressed 1-minute Apgar scores, but normal acid-base scores. In those neonates where uterine incision to delivery time exceeds 300 seconds both 1-minute and 5minute scores tend to be depressed. There is also an increased possibility of lower acid-base scores. The uterine incision-delivery time does appear to be significant. In 1972, Crawford in England reported on the effects of lateral tilt. This paper demonstrated clearly the significant impact of aortic caval compression upon the foetus. Since this time left uterine displacement have been universally employed. Failure to apply this to earlier caesarian sections has been one of the criticisms directed at early studies on induction time. Thiopentone crosses the placenta and it is not possible to deliver the baby before the drug is transferred to the foetus. After a single maternal intravenous dose, the drug can be detected in the umbilical venous blood within 30 seconds.Peak concentrations are found in umbilical venous blood in 1 minute and umbilical arterial blood in 2-3 minutes. All the pharmacokinetic studies have shown that mother and foetus are in equilibrium in 2 minutes. An early report from the MAYO Clinic suggested that the neonate should not be delivered within 10 minutes of induction to allow redistribution. .There is no advantage to delaying delivery until the thiopentone has redistributed in the mother or foetus. There is an observed resistance of the foetal brain to maternally administered barbiturate following a single intravenous injection . Reasons for lack of neonatal depression after a sleep dose of thiopentone are the rapid falls in maternal plasma concentration due to redistribution, dilution within the central compartment, non-homogeneity of blood in the intervillous spaces, and progressive dilution in the circulation due to shunting. Moreover,because of the foetal circulatory pattern, the foetal liver will extract a large percentage of the drug on its first pass before it reaches the brain The foetal brain will not be exposed to high concentrations if the induction dose is < 4mg/Kg. With this dose, umbilical arterial levels are much lower than

422

umbilical vein. It should be stressed however, that after large doses of thiopentone (8 mg/Kg), babies are depressed. Maternal hyperoxia has been shown to improve foetal oxygenation and neonatal clinical condition at birth. A summary of literature follows: (1)A study of 75 women undergoing elective LSCS under general anaesthesia demonstrated that foetal oxygen tension, saturation and content improved significantly with increase in maternal FIO2 until maternal PaO2 reached 300 torr. The clinical condition of the newborn was also better in the higher oxygen groups. It was noted that no additional benefit was gained from maternal PaO2 > 300 torr. (2)Another study compared general anaesthesia between 50% oxygen/N2O/halogenated agent and 100% oxygen/halogenated agent. The use of 100% oxygen significantly improved foetal oxygenation with particular benefit in emergency cases. Babies born to the 100% group required less resuscitation. This issue has been contested on the grounds that foetal acid-base status is the best indicator of neonatal depression, and it was not measured in this study. Proponents of 30%-50% O2 during caesarian section, argue that the parameters that have been shown to be depressed as a result of lower FIO2 do not constitute clinically significant neonatal depression, and that as long as maternal arterial oxygen saturation is well maintained, neonatal outcome will not be influenced by FIO2. This group also argues that the use of a high FIO2 is associated with an increased incidence of maternal awareness. The use of halogenated agents-low dose halothane (0.5%); isoflurane (0.75%); or enflurane (1%) as supplements to N2O is very common. The N2O concentration is usually reduced to 50%. The use of these concentrations do not result in neonatal depression. References Schnider, S.M. and Levinson, G; Anesthesia For Obstetrics, 3rd Ed., 1993, p 231-2. Datta, S;" Neonatal effects of prolonged anaesthetic induction for caesarian section ", Obstetrics and Gynaecology, vol 58, no 3, Sept 1981. Crawford, J;" Time and Lateral Tilt at Caesarian Section ", BJA, 48, 1976, p 661. Joyce, T.H; " Regional versus General Anaesthesia- Any Advantage? ", Seminars In Anaesthesia, Vol 1,no 2, June 1982. Number: 951 Which of the following cardiovascular abnormalities could be consistent with normal changes of pregnancy? 1. 2. 3. 4. A grade II systolic ejection murmur at the left sternal edge. A grade I diastolic murmur at the left sternal edge. Left axis deviation on ECG. Right axis deviation on ECG.

423

A: 1,2,3 Correct B: 1,3 Correct C: 2,4 Correct D: 4 Correct E: All Correct ABCDE Correct Answer: B The diaphragm rises and shifts the position of the heart leftward. This may cause equivocal cardiomegaly on CXR. Echocardiography at 38 weeks indicate an increased end-diastolic chamber size and an increase in total left ventricular wall thickness. Asymptomatic pericardial effusion has been demonstrated in some women. An innocent grade I-II systolic murmur caused by increased blood flow may occur. The ECG may show an increase in benign dysrhythmia, reversible ST, T, and Q wave changes, and left axis deviation. These normal findings must be differentiated from those indicating heart disease. These include a systolic murmur > grade III, any diastolic murmur, severe arrhythmias, and unequivocal cardiomegaly on CXR. SCHNIDER,S.M. & LEVINSON, G; Anesthesia for Obstetrics, 3rd Ed., 1993, pp 6-7. Number: 961 In comparison with a normal patient at the same stage of pregnancy, a patient with severe pregnancy-induced hypertension (PIH) will have: 1. An increased plasma volume. 2. A higher blood viscosity. 3. A similar serum albumin concentration. 4. A lower cardiac output. A: 1,2,3 Correct B: 1,3 Correct C: 2,4 Correct D: 4 Correct E: All Correct ABCDE Correct Answer: C Pregnancy-induced hypertension (PIH) is estimated to affect 7% to 10% of all pregnancies in the United States. Despite being the leading cause of maternal death and a major contributor of maternal and perinatal morbidity, the mechanisms responsible for the pathogenesis of PIH have not yet been fully elucidated.

424

The initiating event in PIH appears to be reduced uteroplacental perfusion as a result of abnormal cytotrophoblast invasion of spiral arterioles. Placental ischemia is thought to lead to widespread activation / dysfunction of the maternal vascular endothelium that results in enhanced formation of endothelin and thromboxane, increased vascular sensitivity to angiotensin II, and decreased formation of vasodilators such as nitric oxide and prostacyclin. The quantitative importance of the various endothelial and humoral factors in mediating the reduction in renal haemodynamic and excretory function and elevation in arterial pressure during PIH is still unclear. In established severe disease there is volume contraction, reduced cardiac output, enhanced vascular reactivity, increased vascular permeability and platelet consumption. Serum albumin falls, both due to loss from the intravascular space because of the increased vascular permeability and renal loss in the form of proteinuria. Blood viscosity increases, thereby aggravating the problem of decreased perfusion. Despite a frequently increased haemoglobin value and hematocrit, these patients have relative anaemia and tolerate blood loss poorly. Number: 1012 The drug of choice for the treatment of Chlamydia trachomatis infection during pregnancy is: A. Metronidazole. B. Cephazolin. C. Amoxycillin. D. Tetracycline. E. Clindamycin. Select the single best answer ABCDE Correct Answer: C Chlamydia infection in the non-pregnant state is usually treated with a tetracycline, or with erythromycin - although more recently, amoxycillin has been found to be as effective as the latter. During pregnancy, tetracycline therapy is contraindicated because of incorporation into fetal bones and teeth. Thus, for the options listed, amoxycillin is the drug of choice. See the systematic review by Brocklehurst and Rooney who concluded that: "Amoxycillin appears to be an acceptable alternative therapy for the treatment of genital chlamydial infections in pregnancy when compared with erythromycin. Clindamycin and azithromycin may be considered if erythromycin and amoxycillin are contra-indicated or not tolerated."

425

Brocklehurst P, Rooney G. Interventions for treating genital chlamydia trachomatis infection in pregnancy (Cochrane Review). In: The Cochrane Library, Issue 4 2002. Oxford: Update Software. Number: 1013 Which of the following conditions found in pregnancy is NOT associated with an increased risk of pre-eclampsia? A. Diabetes mellitus. B. Systemic lupus erythematosus. C. Multiple pregnancy. D. Hydatidiform mole. E. Placenta praevia. Select the single best answer ABCDE Correct Answer: E Diabetes mellitus, systemic lupus erythematosus, multiple pregnancy and hydatidiform mole all place the patient at a significantly increased risk of pre-eclampsia. Placenta praevia is not associated with any such increase in risk.

PEDIATRIC

ANESTHESIA

1, A child with dystrophic epidermolysis bullosa is also likely to suffer from: 1. Iron deficiency anaemia. 2. Malnutrition. 3. Oesophageal reflux. 4. Cardiomyopathy A: 1,2,3 Correct B: 1,3 Correct C: 2,4 Correct D: 4 Correct E: All Correct Correct Answer: A Epidermolysis bullosa is the name given to a group of rare, genetically determined disorders characterised by mucocutaneous blistering and subsequent scarring. Several variants have

426

been established, which can be grouped into three broad categories: epidermolysis bullosa simplex, junctional epidermolysis bullosa and dystrophic epidermolysis bullosa. Iohom and Lyons have recently published an excellent, comprehensive review of the subject. According to these authors: "Anaemia is present in most patients with severe dystrophic epidermolysis bullosa. Investigations demonstrate haematological features both of iron deficiency and of decreased red cell iron utilization (anaemia of chronic disease)." "The malnutrition, which occurs in severe dystrophic epidermolysis bullosa, is a consequence of a combination of decreased nutritional intake and increased requirements as a result of blood and plasma loss from denuded epithelium, skin infection and continuous wound healing." "Gastro-oesophageal reflux is extremely common in this population group." There is no association between any variants of the syndrome and cardiomyopathy. See: Iohom G, Lyons B. Anaesthesia for children with epidermolysis bullosa: a review of 20 years' experience. Eur J Anaesthesiol. 2001 Nov;18(11):745-54.

2, A woman is treated for imminent eclampsia with therapy which includes large doses of magnesium sulphate. A live infant is subsequently delivered by Caesarean section A: 1,2,3 Correct B: 1,3 Correct C: 2,4 Correct D: 4 Correct E: All Signs in the neonate which are suggestive of hypermagnesaemia include: 1. Hypereflexia. 2. Lethargy. 3. Convulsions. 4. Paralytic ileus Correct Answer: C The classic signs of neonatal hypermagnesaemia include hyporeflexia, hypotonia, lethargy and respiratory depression. Paralytic ileus and 'meconium plug' syndrome have also been reported.

427

See: Sullivan JE, Berman BW. Hypermagnesemia with lethargy and hypotonia due to administration of magnesium hydroxide to a 4-week old infant. Arch Pediatr Adolesc Med. 2000;154:1272-1274.

3, With regard to an infant with a tracheo-oesophageal fistula: 1. The diagnosis is suggested by the inability to pass a feeding tube into the stomach. 2. Other congenital abnormalities commonly occur. 3. Polyhydramnios may well have been a feature of the antenatal period. 4. The child should be intubated soon after birth to reduce aspiration risk. A: 1,2,3 Correct B: 1,3 Correct C: 2,4 Correct D: 4 Correct E: All Correct Correct Answer: A Tracheo-oesophageal fistula is generally diagnosed by the triad of reflux of secretions, failure to pass a feeding tube, and CXR confirmation of this. These infants experience problems relating to aspiration/dehydration; prematurity/pulmonary insufficiency, and congenital anomalies. The severity of their illness is highly variable and management should be guided by this. Oesophageal atresia and TOF represent one of the commonest congenital malformations, occurring in 1/4000 births. The prenatal history often features polyhydramnios. The infants are often premature and the condition clusters with other congenital problems (50%), including oesophageal atresia, imperforate anus and cardiovascular disease.30-50% have the associated anomalies of the VATER syndrome. 90% of these infants have a blind oesophageal pouch and a fistula connecting the distal oesophagus and the distal trachea, usually within 1-2 cm of the carina. It may be suspected in the delivery room if the neonate produces excessive secretions which require frequent suctioning. Failed attempts to pass a catheter through the nose or mouth are highly suggestive. A CXR appearance of the feeding tube coiled in the distal oesophagus is diagnostic. Early intervention is mandatory to prevent pulmonary aspiration. A large sump catheter is inserted into the pouch, and suction is maintained to clear secretions. If significant aspiration occurs, corrective surgery is deferred and a decompressing gastrostomy is placed under local or caudal anaesthesia. This condition rarely presents as life threatening although associated conditions may be problematic. Prematurity may be associated with pulmonary immaturity requiring ventilatory assistance. These infants may present an enormous challenge for the management team. The

428

stomach may have a much higher compliance than the lungs, with the result that positive pressure ventilation may be quite ineffective in ventilating the lungs, however, insufflates the stomach with gas. This is further aided by the air leak through the fistula, which increases regurgitation and aspiration, and elevates the diaphragm, further reducing pulmonary compliance. Gastric perforation is also possible. For these reasons, these infants are often managed with an emergency ligation of the fistula, or passage of a Fogarty catheter down the trachea and into the fistula, with ventilation until pulmonary function is optimized whereupon definitive surgical repair can be carried out. Improved survival from 68% to 94% has been described in Seattle as a result of delayed repair. Prior to 1939, all children with TOF died. The procedure of oesophageal anastomosis was introduced in 1941. In 1962, a large review was conducted by Waterston et al who devised a classification of factors influencing survival: Class A - Birth weight >2.5 Kg; healthy- survival 1962, 100%. Class B1 - 1.8 - 2.5 Kg; healthy. Class B2 - >2.5 Kg; moderate pneumonia; moderate congenital anomaly (limb, cleft lip/palate, ASD, or small PDA)- survival all group B 1962, 68%. Class C1 - <1.8 Kg; severe pneumonia; severe congenital anomaly (additional intestinal atresia, cyanotic congenital cardiac disease, severe renal abnormalities, multiple moderate anomalies)- survival 1962, 6%. Survival in groups B and C has been improving due to earlier diagnosis, improved ventilatory management, improved anaesthetic and surgical management, and improved multidisciplinary management of multiple congenital anomalies. Pneumonia has been replaced by congenital anomalies as the main cause of death. Anaesthetic issues relating to improved outcome include management of the problems of persistent air leak through the fistula, and gastric distension. Both of these result in hypoventilation, particularly of the left lung. Studies existing describe guidelines to placement of the ETT in the trachea below the fistula, occlusion of the fistula, and decompressive gastrostomy as measures to deal with these problems. More recently the classification proposed by Spitz has been adopted. References ROGERS, M.C. ET AL (EDS); Principles and Practice of Anesthesiology, Mosby, 1993 pp 2149-50. BENUMOF, J.L & SAIDMAN, L.J; Anesthesia and Perioperative Complications, Mosby, 1992, pp 553-4. BROWN, D.L; Risk and Outcome in Anesthesia, Lippincott, 2nd Ed., 1992, pp 434-5.

4, With regard to fluid resuscitation in an infant awaiting pyloromyotomy for pyloric stenosis, which of the following are considered reliable indicators of volume status? 1. Serial body weight.

429

2. Urinary sodium. 3. Serum sodium. 4. Urinary chloride A: 1,2,3 Correct B: 1,3 Correct C: 2,4 Correct D: 4 Correct E: All Correct Correct Answer: D Patients with pyloric stenosis may show a wide range of metabolic disturbances. Typically, it is characterised by a chloride (or saline) responsive, hypochloraemic, hypokalaemic, hypovolaemic, and sometimes hyponatraemic metabolic alkalosis. The pathophysiology of these is as follows: Hypochloraemia- results from Cl- losses in the vomitus. Conservation of Cl- at a renal level is maximal and relatively efficient. This is manifest as a falling urinary Cl concentration. Urinary Cl is felt to flux in a manner which reliably indicates volume status. < 20mmol/l is indicative of volume depletion. > 20mmol/l suggests volume status has been corrected. Note that this parameter will remain low if adequate volumes (manifest as return to baseline weight) of inappropriately hypotonic solutions are used for resuscitation. Urine Cl results are more relevant than serum electrolytes when assessing volume status in these infants. Hypokalaemia - whilst some K is lost in the vomitus, its concentration here is < 15mmol/l. Of greater importance are renal losses followed by intracellular uptake. To recall normal events, H+ secreted into gastric juice is accompanied by equimolar amounts of HCO3- which are secreted across the basal membrane of the parietal cell to enter the blood stream. Eventually, this is secreted with pancreatic juices into the duodenum to neutralize acidic chyme as it arrives. As HCL is lost in the vomitus, an excess of HCO3- develops causing a metabolic alkalosis. At a renal level, the capacity of the proximal convoluted tubule (PCT) to reabsorb HCO3- is rapidly saturated. Normally, 80% of HCO3- is secreted here with subsequent reabsorption of most as NaHCO3 to maintain electroneutrality. The concentration of Na HCO3 in the ultrafiltrate entering the distal convoluted tubule (DCT) rises. NaHCO3 cannot be reabsorbed in the DCT. An attempt to conserve sodium is made via aldosterone with the subsequent loss of large amounts of K as the cation. K is also lost in the urine as it is exchanged for H+ in an attempt to maintain serum electroneutrality. K+ also moves intracellularly in exchange for H+. Hypovolaemia results both from vomiting and a relatively inefficient renal concentrating mechanism. Hyponatraemia - Despite attempts to conserve sodium, excess may be lost in the urine as a cation to HCO3-. In contrast to Cl-, urinary sodium may not move in a predictable fashion. In hypovolemic states not caused by vomiting, the urinary Cl- and sodium are usually reduced the same degree as they are conserved by reabsorption together. Similarly, they will rise together as resuscitation proceeds. Here, the finding of a reduced or absent urinary sodium is virtually pathognomonic of reduced tissue perfusion and is diagnostic of hypovolemia. In

430

vomiting however, urinary sodium may remain normal as described in (2) and is not a useful indirect marker of volume status See the excellent review by: Bissonnette B, Sullivan PJ. Pyloric stenosis. Can J Anaesth. 1991 Jul;38(5):668-76.

5, Which of the following conditions typically complicate the syndrome of tetralogy of Fallot? 1. Congestive cardiac failure. 2. Polycythaemia. 3. Eisenmenger's Syndrome. 4. Neonatal hypoxaemia. A: 1,2,3 Correct B: 1,3 Correct C: 2,4 Correct D: 4 Correct E: All Correct Correct Answer: C Typically, this condition presents with cyanosis developing progressively after the neonatal period. The sequelae are related the right to left shunt and include cyanosis, secondary polycythaemia and embolic phenomena - septic, thrombotic, gaseous, etc. As Eisenmenger's Syndrome complicates left to right shunts, it is not seen. CCF is not typical. Tetralogy of Fallot and Transposition of the Great Vessels comprise the most common causes of cyanotic congenital heart disease. It comprises stenosis or atresia of the pulmonary valve or infundibulum; right ventricular hypertrophy consequent to this; a VSD high up in the membranous part of the septum beneath the aortic valve and, secondary to this, an overriding aorta (the septal defect lies just inferior to the valve and blood is shunted directly from the right ventricle to the aorta which appears to override it.) The right to left shunt is responsible for a number of associated problems. The child is typically pink in the newborn period and cyanosis develops over a period of weeks to months as influenced by the magnitude of stenosis and shunt. The baby may be pink at rest with cyanosis developing during feeding or crying. Alternatively, the baby may be well most of the time with cyanotic spells occurring periodically which are thought to be due to spasm of the infundibular septum, increasing the right to left shunt As the child ages the severity of the disease progresses. The child may learn to modify symptoms. For example, during episodes of cyanosis the child may assume a squatting position. This acts to decrease venous return and hence mitigate the degree of right to left shunt temporarily. Congestive cardiac failure is not a feature of the disease

431

Polycythaemia may develop secondary to hypoxaemia and may be complicated by thromboembolic phenomena including cerebral abscesses. The Eisenmenger syndrome does not occur because the direction of the shunt is essentially always right to left. The Eisenmenger complex is a term applied to those cases of atrial septal defect (ASD), ventricular septal defect (VSD), or patent ductus arteriosus (PDA), in which there is pulmonary hypertension . It occurs in a fair proportion of VSDs and PDAs but rarely in ASDs (ostium primum defects). The changes in the pulmonary vessels in many cases are present at birth and possibly result from a persistence of foetal circulation, the pulmonary arteries preserving their thick walls and narrow lumen after birth. Pulmonary hypertension results and in its turn causes more severe changes in the vessels. In other cases the changes in the pulmonary vessels are acquired; this is possibly more often the case in VSD. The pressure in the pulmonary artery is either equal to , or more commonly, greater than that in the aorta so that there is a reversal of the shunt through the defect.The prognosis is poor and surgery is contraindicated References HULL, D. & JOHNSTON, D.I; Essentials of Paediatrics, 2nd Ed., pp 138-9. JOLLY, H. & LEVENE, M; Diseases of Children, 5th Ed., 1985, p 171.

6, The most likely cause of persistent hypertension in a six year old child with a normal body mass index is: A. Reno-vascular disease. B. Coarctation of the aorta. C. Phaeochromocytoma. D. Renal disease. E. 'Essential' hypertension. Select the single best answer Correct Answer: D

7, A woman is treated for imminent eclampsia with therapy which includes large doses of magnesium sulphate. A live infant is subsequently delivered by Caesarean section. A: 1,2,3 Correct B: 1,3 Correct C: 2,4 Correct D: 4 Correct E: All Correct

432

Correct Answer: C The classic signs of neonatal hypermagnesaemia include hyporeflexia, hypotonia, lethargy and respiratory depression. Paralytic ileus and 'meconium plug' syndrome have also been reported. See: Sullivan JE, Berman BW. Hypermagnesemia with lethargy and hypotonia due to administration of magnesium hydroxide to a 4-week old infant. Arch Pediatr Adolesc Med. 2000;154:1272-1274. and: Narchi H. The pediatric forum: neonatal hypermagnesemia: more causes and more symptoms. Arch Pediatr Adolesc Med. 2001 Sep;155(9):1074. 8, The smallest 'Univent' tube available for paediatric use has an internal diameter of: A. 3.5 mm. B. 4.0 mm. C. 4.5 mm. D. 5.0 mm. E. 5.5 mm Select the single best answer Correct Answer: A

9, A neonate awaiting repair of a congenital diaphragmatic hernia will typically have which of the following associated problems: 1. Pulmonary hypertension. 2. Aspiration. 3. Other congenital abnormalities. 4. Prematurity. A: 1,2,3 Correct B: 1,3 Correct C: 2,4 Correct D: 4 Correct E: All Correct Correct Answer: E

433

The condition typically clusters with other congenital abnormalities (16%).These may include cardiac abnormalities. All are associated with a degree of gut malrotation. 75% have polyhydramnios. The high incidence of polyhydramnios carries with it a risk of premature labour. As a result neonates with a diaphragmatic hernia, like those with trache-ooesophageal fistula and related conditions, may be premature. In this event, problems related to prematurity may also exist. Gut malformations may be associated with gastric insufflation and aspiration, particularly if the stomach occupies an intrathoracic site and resuscitation is attempted without tracheal intubation. The pathogenesis is unresolved; one theory maintains that the presence of the gut in the pleural cavity restricts development of the lungs. Conversely, the other proposes that primary pulmonary hypoplasia occurs, and that the gut migrates into the thoracic cavity by default. The net result in Severe CDH is respiratory insufficiency secondary to pulmonary hypoplasia. Hypoxaemia ensues and this results in increased pulmonary vascular resistance, pulmonary hypertension and a persistence of the right to left shunt present in utero. This is called persistent foetal circulation. Left untreated, this will result in a self sustaining cycle of progressive hypoxaemia and acidosis, pulmonary hypertension, right heart failure, systemic hypotension and death. Management consists of early intubation, ventilation with 100% oxygen, neuromuscular paralysis, nasogastric suctioning, pulmonary vasodilators, and emergency surgical decompression of intrathoracic intestinal contents. Typically, neonates with diaphragmatic hernia may be divided into two groups based on the age at presentation. They are considered to have CDH if they develop respiratory symptoms at less than 24 hours of age. Most often this group will develop symptoms at birth or within the first 6 hours of life. The mortality rate remains between 40%-80% despite initial expectations that in-utero diagnosis and transfer to obstetric centres with neonatal intensive care facilities would lead to improved outcome. Neonates presenting after 24 hours generally have limited defects and a reduced incidence of pulmonary complications. They have a survival of 100% (excluding other congenital abnormalities). It should be noted that the severity of the defect often correlates with early detection on ultrasound, and that if it is missed such that diagnosis occurs in the delivery room, then the neonate is likely to fall into this group (ie severe pulmonary hypoplasia is unlikely). References BROWN, D.L.; Risk and Outcome in Anesthesia, 2nd Ed., Lippincott, 1993, pp 428-34.

10, Which of the following are true when comparing general anesthesia with halothane in a three year old compared with an adult? 1. Induction is faster. 2. Higher concentrations will be required for maintenance of anaesthesia. 3. Myocardial depression is greater. 4. The risk of aspiration is higher.

434

A: 1,2,3 Correct B: 1,3 Correct C: 2,4 Correct D: 4 Correct E: All Correct Correct Answer: E Several differences in the response to anaesthetic agents in the paediatric population exist. Induction of anaesthesia proceeds more rapidly. Studies demonstrate a steady decline in MAC with age; ie: 1.2% at 0-6 months, 1.16% at 6-24 months, and 1.07% at 24-48 months. The younger the child is, the more halothane is required achieve adequate anaesthesia. At the same time, increasing doses are more likely to cause myocardial depression. The therapeutic index is low, and it may be difficult to achieve anaesthesia and normotension simultaneously. In one study, a dose dependent decrease in blood pressure, pulse rate and cardiac output occured which was generally responsive to atropine. This was felt to be of particular relevance in infants whose cardiac output is largely rate dependant due to low myocardial compliance. Depression of baroreceptor function is also more pronounced. Children under the age of 12 years are at greater risk of aspiration. One study showed 74% to be at risk. This is associated with a lower gastric acid pH and higher residual gastric volume compared with adults. References BROWN, D.L.; Risk and Outcome in Anesthesia, 2nd Ed., Lippincott, 1993, pp 419-22.

11, Which of the following statements are true of bronchiolitis: 1. Up to 50% of patients continue to wheeze after recovery 2. Tachypnoea is invariable 3. Air-trapping is normally present 4. The typical pathogen is para influenza virus A: 1,2,3 Correct B: 1,3 Correct C: 2,4 Correct D: 4 Correct E: All Correct Correct Answer: A

435

Bronchiolitis is usually caused by Respiratory Syncitial Virus (RSV). RSV infection leads to a wide spectrum of respiratory illnesses. In infants, 25 to 40 percent of infections result in lower respiratory tract involvement, including pneumonia, bronchiolitis, and tracheobronchitis. In infants, illness begins most frequently with rhinorrhea, low-grade fever, and mild systemic symptoms, often accompanied by cough and wheezing. Most patients gradually recover in 1 to 2 weeks. In more severe illness, tachypnea and dyspnea develop, and eventually frank hypoxia, cyanosis, and apnea can ensue. Physical examination may reveal diffuse wheezing, rhonchi, and rales. Chest x-ray shows hyperexpansion, peribronchial thickening, and variable infiltrates ranging from diffuse interstitial infiltrates to segmental or lobar consolidation.

12, A ventricular septal defect in children: 1. Is the commonest congenital heart lesion 2. If of Maladie de Roger type is the most severe form 3. Closes spontaneously in up to 50% of cases 4. Is associated with pulmonary oligaemia A: 1,2,3 Correct B: 1,3 Correct C: 2,4 Correct D: 4 Correct E: All Correct Correct Answer: B Defects of the ventricular septum are common as isolated defects and as one component of a combination of anomalies. The opening is usually single and situated in the membranous portion of the septum. The functional disturbance is dependent primarily on its size and on the status of the pulmonary vascular bed, rather than on the location of the defect. The 'Maladie de Roger' is a small VSD.

13, The distance from the incisors to the mid-tracheal point in a five year old child is most likely to be: A. 10 cms. B. 12 cms. C. 15 cms. D. 17 cms. E. 19 cms. Select the single best answer

436

Correct Answer: C Premature 8-9 cms. 0 - 6 mo 10 -11cms. 6 - 12 mo 12 cms. 1 - 2 yr 13 cms. 2 - 4 yr 14 cms. 4 - 6 yr 15 cms. 6 - 8 yr 16 cms. 8 - 10 yr 17 cms. 10 - 12 yr 18 cms. 14, A 3400g neonate is examined at 5 minutes of life with the following findings: temperature: 37.2C ; heart rate: 120 bpm; respiration rate: 25 bpm (regular); blood pressure: 80/60 mm Hg. His eyes are closed and his pupils are constricted. All four extremities are actively moving with good muscle tone. When you suction the nares there is no response. Skin colour is pink on both the body and the extremities. What is the correct 5 minute Apgar score for this neonate? A. 5. B. 7. C. 8. D. 9. E. 10. Select the single best answer Correct Answer: C The Apgar scoring system is: Colour: 0: Pale or cyanosed. 1: Pink trunk / blue limbs. 2: Pink. Pulse rate (normally 120-160 bpm): 0: Absent. 1: < 100 bpm. 2. > 100 bpm. Reflex response to airway suction: 0: Absent. 1. Grimace. 2. Cough / sneeze.

437

Muscle tone: 0: Flaccid. 1. Present. 2: Actively moving. Respiration (Normal: 30 - 60 bpm): 0: Absent. 1: Irregular. 2. Regular. 15, A 5-month-old female infant presents with persistent tachypnoea of a few weeks duration following an upper respiratory tract infection. The chest X-Ray is shown below. This scenario is compatible with: 1. Congenital lobar emphysema. 2. Aspiration of a foreign body. 3. Congenital cystic adenomatoid malformation. 4. Lobar pneumonia A: 1,2,3 Correct B: 1,3 Correct C: 2,4 Correct D: 4 Correct E: All Correct Correct Answer: B Aspiration of a foreign body - typically occurs in children aged 1-2 years and is therefore unlikely although the Xray is compatible with the diagnosis. Tracheobronchial foreign body aspiration in children. Burton EM; Brick WG; Hall JD; Riggs W Jr; Houston CS South Med J, 89(2):195-8 1996 Feb. In this retrospective study, we reviewed the demographic and radiographic findings of 155 children with bronchoscopy-proven tracheobronchial foreign body aspiration (FBA). Two thirds of the patients were male, and most were children between 1 and 2 years of age. An aspirated peanut accounted for one third of all cases. Foreign body location was distributed nearly evenly to the right and left primary bronchi; tracheal foreign body was noted in 16 patients. The most frequent symptoms of FBA were cough (85 patients) and wheezing (60 patients). Although most patients were seen within 1 day of aspiration, 30 patients had symptoms that lasted at least 1 week before diagnosis. The most common radiographic findings were unilateral or segmental hyperlucency (59) or atelectasis (38). The trachea was the site of the foreign body in one half of children with a normal chest radiograph and FBA. The Xray is compatible with both congenital lobar emphysema and congenital cystic adenomatoid malformation. - It is, in fact, a case of the latter.

438

Congenital cystic adenomatoid malformation is a relatively uncommon developmental abnormality of the lung thought to be caused by an arrest of bronchial maturation and, at the same time, overgrowth of mesenchymal elements. It occurs sporadically as a defect in the embryogenesis of the lung without identifiable causes and can be identified in utero as early as 20 weeks of gestation, either incidentally or because it often causes hydramnios. The usual mode of presentation is respiratory distress in the neonatal period, the degree being proportional to the degree of emphysema and mediastinal shift. Histologically, congenital cystic adenomatoid malformation has been subdivided into three types. Type I consists of relatively large cysts of uneven size, and has a better prognosis compared to types II and III, which are often associated with other congenital malformations. Type II congenital cystic adenomatoid malformation consists of uniform small cysts throughout. It often coexists with chromosomal abnormalities and other defects. Type III is predominantly solid. Surgery is therapeutic, the ultimate prognosis dependent on associated anomalies. 16, In a 3 year old child with congenital heart disease, which of the following murmurs are best heard at the apex? 1. 2. 3. 4. Hypertrophic Obstructive Cardiomyopathy Tetralogy of Fallot Mitral Regurgitation Patent Ductus Arteriosus

A: 1,2,3 Correct B: 1,3 Correct C: 2,4 Correct D: 4 Correct E: All Correct

Correct Answer: B 1. In hypertrophic obstructive cardiomyopathy, the abnormal interventricular septum is often associated with mitral regurgitation. 2. The murmur in Tetralogy of Fallot is usually pulmonary and best heard at the upper left sternal margin. 3. The apex is the best place to hear a mitral valvular murmur. 4. The murmur of a PDA is best heard at the upper left sternal margin.

439

17, Which of the following may predispose to the development of respiratory distress syndrome (RDS) in the neonate? 1. Low gestational age. 2. Low weight for gestational age. 3. Maternal diabetes. 4. Congenital heart disease in the foetus. A: 1,2,3 Correct B: 1,3 Correct C: 2,4 Correct D: 4 Correct E: All Correct Correct Answer: B The incidence of RDS is higher in infants of low gestational age. However, there is no significant difference in the incidence between infants whose birth weight is above or below the mean for their gestational age. Maternal diabetes is a powerful determinant of the incidence of RDS - the risk being increased about twentyfold if no specific prophylactic therapy is used. Congenital heart disease in the foetus has no effect on the incidence of RDS. 18, One minute after birth, a 2.2 kg term infant has a pink trunk but cyanotic fingers and toes and an irregular respiratory pattern. The child grimaces with nasal suctioning, is immobile, but has flexion of the elbows and knees and a heart rate of 85 bpm. Which of the following statements are true? 1. The Apgar score is consistent with mild intrauterine asphyxia. 2. Atropine 40 mcg via the umbilical vein is indicated. 3. Bag-mask ventilation with pure oxygen should be initiated. 4. Intubation is indicated. A: 1,2,3 Correct B: 1,3 Correct C: 2,4 Correct D: 4 Correct E: All Correct Answer: B The child has an Apgar score of 5.

440

At 1 minute, the appropriate guidelines for the management infants with various Apgar scores are: 7-10: Warm the baby, nasal / oral suction only. 4-6: Stimulate baby, suction airway. If respiration remains irregular or heart rate (HR) persists at < 100 bag-mask ventilation with FiO2 1.0. 0-3: Bag-mask ventilate with FiO2 1.0; if HR stays < 60, intubate and begin chest compressions; commence a formal resuscitation protocol. Bradycardia in this context is a symptom of hypoxia and will not be helped by atropine. The Apgar scoring system is: Colour: 0: Pale or cyanosed. 1: Pink trunk / blue limbs. 2: Pink. Pulse rate (normally 120-160 bpm): 0: Absent. 1: < 100 bpm. 2. > 100 bpm. Reflex response to airway suction: 0: Absent. 1. Grimace. 2. Cough / sneeze. Muscle tone: 0: Flaccid. 1. Present. 2: Actively moving. Respiration (Normal: 30 - 60 bpm): 0: Absent. 1: Irregular. 2. Regular.

19, The parents of a 10 year old boy are worried that his school performance has decreased over the past 2 months. The boy complains of increasing headache and double vision and on examination you note that he has gait ataxia. Which of the following is the most appropriate immediate investigation?

441

A. B. C. D. E.

Computed tomography (CT) scan of the brain. Electroencephalogram. Auditory evoked potentials. Single Photon Emission Computed Tomography (SPECT) scan of the brain. Magnetic resonance imaging (MRI) scan of the brain

Select the single best answer Correct Answer: E An MRI scan is the most appropriate initial investigation for a suspected posterior fossa lesion - CT resolution in this region being less good. See, for example: Kent DL, Haynor DR, Longstreth WT Jr, Larson EB. The clinical efficacy of magnetic resonance imaging in neuroimaging. Ann Intern Med. 1994 May 15;120(10):856-71. SPECT scanning is used in the assesment of regional cerebral blood flow and for the diagnosis of functional brain abnormalities - in particular epilepsy. Number: 723 Shortly after birth, a full-term infant weighing 3.1 kgs, is noted to be cyanosed during feeding. The cyanosis is relieved by crying. The infant was delivered normally following an uncomplicated pregnancy. Which of the following diagnoses is most likely? A. Tracheo-oesophageal fistula. B. Tetralogy of Fallot. C. Respiratory distress syndrome. D. Patent ductus arteriosus (PDA). E. Choanal atresia. Select the single best answer Correct Answer: E Neonates are obligate nose-breathers. For this reason, choanal atresia classically presents with cyanosis during feeding or at rest which is relieved by crying. The diagnosis is confirmed by the inability to pass a suction catheter through either nostril. Some cases of choanal atresia are associated with the CHARGE syndrome or other congenital abnormalities. In contrast, an infant with a right-to-left shunt will become more cyanosed during crying because crying increases pulmonary vascular resistance. Tracheo-oesophageal fistula is characterised by episodes of choking and / or cyanosis during feeding - but it will not improve if the infant cries. A PDA constitutes a left-to-right shunt and does not cause cyanosis.

442

Respiratory Distress Syndrome is unlikely given the scenario and does not present in this manner anyway.

20, The retinopathy of prematurity (ROP): 1. Is a cause of myopia in later life. 2. Is more likely to occur in an infant exposed to supplemental oxygen. 3. Is associated with the development of strabismus in later life. 4. Is unlikely to occur in an infant born at a gestational age of more than 32 weeks. A: 1,2,3 Correct B: 1,3 Correct C: 2,4 Correct D: 4 Correct E: All Correct Correct Answer: E ROP is associated with the development of both myopia and strabismus in later life. The development of the condition is most clearly related to the use of supplemental oxygen in low birth weight infants (<1200G) of low gestational age (<28 weeks). The condition is very unusual in infants born at a gestational age of more than 32 weeks. Avoidance of arterial hyperoxia is almost certainly important in reducing the incidence of the disease. See: Pediatrics 1999 Sep;104(3): Current incidence of retinopathy of prematurity, 1989-1997. Hussain N, Clive J, Bhandari V. Curr Opin Ophthalmol 1999 Jun;10(3):155-63: Results of screening low-birth-weight infants for retinopathy of prematurity. Clemett R, Darlow B. 21, The typical biochemical profile of a neonate with severe, newly diagnosed, pyloric stenosis includes: 1. Hypochloraemia. 2. Alkalosis. 3. Hypokalaemia. 4. Unconjugated hyperbilirubinaemia

443

A: 1,2,3 Correct B: 1,3 Correct C: 2,4 Correct D: 4 Correct E: All Correct Correct Answer: A 22, The smallest size of double lumen tube (DLT) which is commercially available for paediatric use is: A. 24 F B. 26 F C. 28 F D. 30 F E. 32 F Correct Answer: B Conventional plastic DLTs, once available only in adult sizes (35F, 37F, 39F, and 41F), are now available in smaller sizes. The smallest cuffed DLT is a 26F (Rusch, Duluth, GA), which may be used in children as young as 8 yr old. DLTs are also available in sizes 28F and 32F (Mallinckrodt Medical, Inc., St. Louis, MO), suitable for children 10 yr and older. See the comprehensive review by Hammer entitled "Pediatric thoracic anesthesia." (Hammer GB. Pediatric thoracic anesthesia. Anesth Analg. 2001 Jun;92(6):1449-64.)

23, A 4-day-old, 1500-gram, premature infant recovering from a hyaline membrane disease is noted to have bounding peripheral pulsations and a hyperactive precordium. A continuous machinery murmur is most audible in the left infraclavicular area. Left ventricle hypertrophy is present on the electrocardiogram. The chest x-ray shows slight enlargement of the heart and increased pulmonary venous markings. Which of the following is the most likely diagnosis? A. Ventricular septal defect. B. Atrial septal defect. C. Patent ductus arteriosus. D. Pulmonary stenosis. E. Tetralogy of Fallot. Select the single best answer

444

Correct Answer: C A patient with patent ductus arteriosus has a continuous machinery murmur and bounding pulses with a hyperactive precordium. Although the electrocardiogram is usually not helpful, it may show left ventricular hypertrophy. Patients with atrial septal defect have a widely split and fixed second heart sound. Patients with ventricular septal defect have a pan-systolic murmur. The tetralogy of Fallot is constituted by the presence of a VSD, right ventricular hypertrophy, pulmonary stenosis, and an overriding aorta. The murmur of tetralogy of Fallot is a long, systolic ejection murmur most audible at the mid-left sternal border. In patients with pulmonary stenosis, there is a high-pitched systolic ejection murmur, most audible at the upper left sternal border, that transmits fairly well to the back. A systolic thrill may be present at the upper left sternal border and, rarely, in the suprasternal notch.

24, The American Society of Anesthesiologists recommends that a breast fed infant scheduled for elective surgery be fasted for: A. 30 minutes. B. 1 hour. C. 2 hours. D. 4 hours. E. 6 hours. Select the single best answer Correct Answer: D The Consultants and Task Force support a fasting period for breast milk of 4 hours for both neonates and infants. 25, The most appropriate drug for the treatment of an autonomic crisis in a patient with Riley-Day syndrome is: A. Midazolam. B. Droperidol. C. Metoprolol. D. Sodium Nitroprusside. E. Morphine.

445

Select the single best answer Correct Answer: A Familial dysautonomia (Riley-Day syndrome) is a rare genetic disorder that is transmitted via an autosomal recessive gene. The disease, typically involving Jewish children, affects the central nervous system and can be characterised by pathological deficits in peripheral autonomic and sensory neurones. The signs, which begin in early childhood, include poor perception of pain and temperature, poor co-ordination of muscles, emotional crises with hypertension and profound sweating, postural hypotension, and excessive vagal reflexes. Benzodiazepines are extremely effective in the treatment of crises. 26, Electrocardiographic evidence of left ventricular hypertrophy (LVH) is found in children with: 1. Coarctation of the aorta. 2. Tetralogy of Fallot. 3. Ventricular septal defect. 4. Atrial septal defect. A: 1,2,3 Correct B: 1,3 Correct C: 2,4 Correct D: 4 Correct E: All Correct Correct Answer: B LVH occurs when the left ventricle is either pressure or volume overloaded. This is the case in the presence of coarctation of the aorta and ventricular septal defect (with a left-to-right shunt). In tetralogy of Fallot and atrial septal defect there is right ventricular overload, and consequently there may be electrocardiographic evidence of right ventricular hypertrophy. 27, Which of the following childhood conditions may present with acute stridor? 1. Viral croup. 2. Acute bacterial tracheitis. 3. Epiglottis. 4. Tetany. A: 1,2,3 Correct B: 1,3 Correct C: 2,4 Correct D: 4 Correct E: All Correct

446

Correct Answer: E 28, Cleft lip and Palate: A. Are usually associated with other congenital anomalies. B. Need to be repaired urgently because the infant will be unable to feed. C. Are usually a midline defect. D. Constitute a major intubation problem. E. Are usually repaired separately. Select the single best answer Correct Answer: E There is not an association with other anomalies. There is no urgency regarding the repairs which are usually undertaken as separate procedures (lip first then palate). It is rarely a midline defect. The defects are generally repaired as elective procedures and the greatest anaesthetic concerns relate to the nutrition of the child A cleft lip rarely occurs in the midline. The cleft generally occurs to one side of the midline, may be single or bilateral and may exist by itself or in association with a cleft palate. A midline cleft lip and palate is very rare and if present is often associated with a cerebral malformation, particularly abscence of the corpus callosum with a single ventricle; these features may occur in trisomy 13-15. The disorder of cleft lip may be familial. In its mildest form there is only a lip scar (submucous cleft), indicating delayed intrauterine fusion of the maxillary process, but when severe the cleft extends to the nose. No major intubation difficulties are encountered. It is usually repaired at 3 months and surgeons commonly insist that the baby's weight has reached 5 kg. In its mildest form this condition exists as a fish-tail notching of the uvula. In increasing order of severity it includes bifid uvula, cleft soft palate, cleft soft and hard palate. Cleft palate may be lateral or midline.A lateral cleft is usually associated with a cleft lip; a midline is rarely associated with a cleft lip.The maxilla on the side of the cleft is underdeveloped. Surgical repair is usually left until 12 months , treatment being complete by the time speech is developed. The orthodontic fitting of an oral appliance early is used to stimulate development of the underdeveloped maxilla, making primary surgical repair more successful. Cleft lip and palate do not present a problem in intubation. The immediate problem is difficulty feeding and a variety of modified feeding teats of tubes can be used if breast feeding is unsuccessful. Attention to this is crucial; surgical repair may be postponed in babies that have become poorly nourished and anaemic whilst on the waiting list. Children with cleft palate are particularly prone to otitis media from reflux up the eustachian tube. Recurrent infections and deafness may result. References JOLLY, H. & LEVENE, M.I; Diseases of Children, 5th Ed., 1985, pp 136-9. HULL, D. & JOHNSTON, D; Essential Paediatrics, Churchill Livingstone, 2nd Ed, p 162.

447

29, Congenital Diaphragmatic Hernia (CDH): 1. Complicates about 1 in 5000 live births. 2. Most often occurs on the left side. 3. Is associated with other congenital abnormalities in at least 50% of cases. 4. May be associated with severe pulmonary hypertension. A: 1,2,3 Correct B: 1,3 Correct C: 2,4 Correct D: 4 Correct E: All Correct Correct Answer: C See the comprehensive review by Hammer. According to this author: "CDH is a life-threatening condition occurring in approximately 1 in 2000 live births. Failure of a portion of the fetal diaphragm to develop allows abdominal contents to enter the thorax, interfering with normal lung growth. Most often (70%80% of diaphragmatic defects), a portion of the left posterior diaphragm fails to close, forming a triangular defect known as the foramen of Bochdalek. Hernias through the foramen of Bochdalek occurring early in fetal life usually cause respiratory failure immediately after birth because of pulmonary hypoplasia. Distension of the gut postnatally, as with bag-and-mask ventilation, exacerbates the ventilatory compromise by further compressing the lungs. The diagnosis is often made before birth, and fetal surgical repair has been described. Neonates present with tachypnea, a scaphoid abdomen, and absent breath sounds over the affected side. Chest radiography (CXR) typically shows bowel in the left hemithorax, with deviation of the heart and mediastinum to the right and compression of the right lung. Right-sided hernias may occur late and present with milder signs. In the presence of significant respiratory distress, bag-and-mask ventilation should be avoided and immediate tracheal intubation performed. Because pulmonary hypertension with right-to-left shunting contributes to severe hypoxemia in neonates with CDH, a variety of vasodilators have been used. These include tolazoline, prostacyclin, dipyridamole, and nitric oxide. High-frequency oscillatory ventilation has been used in conjunction with pulmonary vasodilator therapy to improve oxygenation before surgery. In cases of severe lung hypoplasia and pulmonary hypertension refractory to these therapies (e.g., PaO2 <50 mm Hg with a fraction of inspired oxygen of 1.0), extracorporeal membrane oxygenation (ECMO) should be initiated early to avoid progressive lung injury. Improved outcomes have been associated with early use of ECMO followed by delayed surgical repair." About 15% of neonates with CDH have other congenital abnormalities. See: Hammer GB. Pediatric thoracic anesthesia. Anesth Analg. 2001 Jun;92(6):1449-64.

448

30, A 5 year old child diagnosed with a patent ductus arteriosus: 1. Is inoperable if there is a loud 2nd sound and right ventricular hypertrophy on ECG. 2. Should have surgical closure delayed until adult life if asymptomatic. 3. Should have a trial of closure with Indomethacin. 4. Will have pulmonary plethora on CXR. A: 1,2,3 Correct B: 1,3 Correct C: 2,4 Correct D: 4 Correct E: All Correct Correct Answer: D A patent duct may present as failure to thrive. Pulmonary hypertension is usually reversible at this stage and is not a contraindication to surgery. Duct closure is recommended at this age because the risk of subacute bacterial endocarditis is high if left untreated. A simple duct will show pulmonary plethora on X-ray because of the large left to right shunt. Indomethacin is only of use in neonates. This condition is after VSD one of the commonest types of non-cyanotic congenital cardiac disease. Others include coarctation of the aorta, aortic stenosis and hypoplastic left heart; all of these tend to present as CCF in infancy. PDA is twice as common in girls. It is usually detected by hearing a typical machinery murmur on routine examination. The murmur, which is accompanied by a thrill, is best heard in the pulmonary area and is continuous throught systole and diastole, though louder in systole. A venous hum may sound similar (continuous murmur heard in normal individuals just below the clavicles, especially on the right). It is caused by flow of blood in the great veins. It is a softer murmur and can be abolished by occlusion of the jugular vein in the neck and varies with extension and flexion of the head) ; apart from this it is distinct. Diagnosis is more difficult in infancy as at this age the ductus produces only a systolic murmur. Absence of the diastolic murmur is due to the pulmonary artery pressure in infancy being nearly equal to that in the aorta, thereby preventing flow through the ductus during diastole. The diastolic component is usually present by the end of the first year when the aortic pressure has become higher than the pulmonary. If a PDA causes CCF, it will manifest in infancy, hence diagnosis is essential in this age group. Diagnostic signs include a high pulse pressure and collapsing pulse indicative of a high cardiac output state. CXR shows some degree of cardiac enlargement and dilatation of the pulmonary artery; and plethora of the lung fields. The ECG is normal or shows either left ventricular hypertrophy or a combination of right and left ventricular hypertrophy. A PDA should be ligated in all cases to prevent later complications. If this is not carried out adult life will almost certainly be shortened by the onset of CCF and/or bacterial endocarditis. The best time for surgical repair is 3-7 years. Surgery is contraindicated in some situations. If pulmonary hypertension has progressed to an Eisenmenger Complex closure of the ductus is likely to be fatal because of the need for a right to left shunt through the open duct. It should

449

not be undertaken if the patient is cyanosed or if right axis deviation is present on the ECG. Pulmonary hypertension is generally reversible in a 5 year old and is not a contraindication to surgery. Medical manipulation is possible in the neonate with an inhibitor of prostaglandin synthesis; ie: indomethacin. This may be necessary as a PDA may precipitate intractable heart failure in infancy especially in preterms. Conversely, if a coexisting anomaly is present such as transposition, in which the only communication between the left and right sides of the heart is the patent ductus, spontaneous closure will cause death. As a protective measure, until surgical repair is performed, a continuous infusion of prostaglandin should be given to delay closure.

PERFUSION ( CARDIAC ANESTHESIA) TESTS Number: 38 An adult is undergoing cardio-pulmonary bypass at 26 degrees celsius. The perfusionist is attempting to follow an 'Alpha Stat' management strategy. The target PaCO2 (measured at 26 degrees celsius) which the perfusionist should aim for is approximately: A. 15 mm Hg. B. 25 mm Hg. C. 40 mm Hg. D. 50 mm Hg. E. 65 mm Hg. Select the single best answer ABCDE Correct Answer: B The aim of 'alpha-stat' management is to maintain normocarbia (and a pH of 7.40) in the arterial blood when the blood gas tensions are measured at 37 degrees celsius. Blood with a PCO2 of 25 mm Hg at 26 degrees celsius will have a PCO2 of about 40 mm Hg when warmed to 37 celsius (depending on the temperature correction algorithm used). The 'alpha-stat' technique is based on the proposition that the constancy of the 'internal milieu' is accomplished predominantly by the buffering capacity of the imidazole group of histidine. As temperature changes, this imidazole protein buffer changes its pK, in parallel with the pN of water. The fraction of unprotonated histidine imidazole groups, known as alpha, remains constant; total CO2 remains constant; and pH changes as temperature changes. The term "alpha-stat" refers to maintenance of this constant net charge on proteins with temperature changes by keeping total CO2 stores constant. The alternative method of acid-base strategy is termed 'pH-stat'. Under these circumstances, the strategy is to maintain a pH of 7.40 and normocarbia at the actual patient temperature.

450

Several formulae are available for adjusting a blood gas pCO2 determined at 37 C for a patient's body temperature. For elevated body temperatures the pCO2 is increased; for reduced body temperatures the pCO2 is decreased. corrected pCO2 = (measured pCO2) * (10^ ( X * ((patient temperature in C) - 37))) where X = 0.021 for the Radiometer ABL-3 X = 0.019 for the Corning and Instrumentation Laboratory (IL) instruments References: Ashwood ER, Kost G, Kenny M. Temperature correction of blood-gas and pH measurements. Clin Chem. 1983;29:1877-1885. Ciba-Corning 288 Blood Gas System Operator's Manual. 1988, page 2-15 Kelman GR, Nunn JF. Nomograms for correction of blood pO2, pCO2, pH and base excess for time and temperature. J Appl Physio. 1966; 21: 1484-1490. Pruden EL, Siggaard-Andersen O, Tietz NW. Chapter 30: Blood gases and pH. pages 13751410 (1406-1409). IN: Burtis C, Ashwood E. Tietz Textbook of Clinical Chemistry, Second edition. W.B. Saunders Company. 1994. Number: 48 A 50 year old man presents for urgent on-pump, coronary artery grafting after suffering an acute coronary syndrome. He has been treated with intravenous heparin for one week. During this time his platelet count has fallen from 250,000 / microlitre to 100,000 / microlitre, but Heparin-PF4 antibodies are not present. Given this scenario, the most appropriate anticoagulant management for cardiopulmonary bypass is: A. The use of systemic heparin in conventional dosages. B. Systemic anticoagulation with ancrod. C. Systemic anticoagulation with danaparoid (Orgaran). D. Systemic anticoagulation with hirudin. E. The use of a Carmeda coated circuit without systemic heparinisation Select the single best answer ABCDE Correct Answer: A From the description that has been given, there is little doubt that the patient is either suffering from HIT type I or has some other cause for thrombocytopaenia. This being so, it is entirely reasonable to use systemic heparinisation in a routine manner. Thrombocytopaenia associated with the use of heparin occurs in two general forms. The first (HIT Type I) is a benign type seen in many patients 1-3 days after starting standard heparin

451

and felt to be caused by the non-immune enhancement of platelet aggregation followed by sequestration and removal by the spleen. The platelet count usually remains above 100,000, resolves spontaneously, and requires neither cessation of heparin nor other treatment. It is not associated with the development of thrombosis. Unlike HIT Type I, HIT Type II is a more severe, delayed-onset thrombocytopaenia that is immunologically mediated. Consistent with the generation of an immune response, HIT Type II typically occurs approximately 5 to 14 days following initiation of heparin therapy but may occur earlier if the patient has been previously exposed to heparin. The thrombocytopaenia is usually more severe than in HIT Type I, with platelet counts often dropping to 60,000/ microlitre or below and remaining low until heparin is withdrawn. After cessation of heparin therapy, platelet counts usually take several days to rebound, but in rare cases may take up to 1 month to normalise. HIT Type II is much more dangerous than Type I because a proportion of those affected will go on to develop intravascular thrombosis. The diagnosis of HIT Type II is based on immunological testing and platelet function analysis. The immunological test consists of an enzyme-linked immunosorbent assays (ELISA) which measures the presence of antibodies capable of binding the heparin-PF4 complex. Although this test may be more sensitive than the functional tests, it is also less. Sensitivities and specificities also vary depending on the specific ELISA used. Various functional analyses have been used for establishing the diagnosis, but the most sensitive and specific appears to be the serotonin release assay (SRA). Donor platelets are isolated from platelet-rich plasma and incubated with radiolabeled (14C) serotonin. The platelets are then washed to remove free serotonin and incubated with heat-inactivated patient serum in the presence of therapeutic (0.1 U/mL) and high (100 U/mL) concentrations of heparin. A positive result is defined as more than 20% release at low heparin concentrations and less than 20% release at high concentrations of heparin. The test is generally thought to be the most sensitive test for confirmation of HIT Type II. Nuttall et al have recently reviewed the management of 12 patients with a history of HIT type II who required cardiac surgery on cardiopulmonary bypass. See: Nuttall GA, Oliver WC Jr, Santrach PJ, McBane RD, Erpelding DB, Marver CL, Zehr KJ. Patients with a History of Type II Heparin-Induced Thrombocytopenia with Thrombosis Requiring Cardiac Surgery with Cardiopulmonary Bypass: A Prospective Observational Case Series. Anesth Analg. 2003 Feb;96(2):344-50. Number: 70 A patient is undergoing hypothermic cardio-pulmonary bypass at 25 degrees Celsius. 'Alphastat' (as opposed to 'pH-stat') management is being used. Which of the following statements is / are true? 1. pH will be higher if an alpha stat technique is being used. 2. Cerebral blood flow is greater if pH stat management is used. 3. Alpha stat management is associated with a better neurological outcome.

452

4. Cerebral autoregulation is better maintained under pH stat conditions. A: 1,2,3 Correct B: 1,3 Correct C: 2,4 Correct D: 4 Correct E: All Correct ABCDE Correct Answer: A Alpha stat management is now commonly used throughout Australia. It is characterised by lower cerebral blood flows with maintenance of autoregulation and no uncoupling of oxygen demand and supply. At this temperature the PCO2 difference between alpha stat and pH stat management would be about 12-15 mm Hg. Studies contrasting alpha stat and pH stat indicate that myocardial function during hypothermic CPB is better preserved when the alpha stat method is used. When pH is held constant at 7.4 by the addition of CO2, myocardial lactate extraction decreases, indicating the presence of either myocardial ischaemia or, more likely, depression of cellular function due to relatively acidotic conditions. Also, during pH stat perfusions, contractility, myocardial oxygen consumption, and coronary blood flow have been shown to decrease. These observations are believed to indicate depressed cellular function in the acidotic environment produced by maintaining the actual body temperature pH at 7.4 during hypothermia. Similar depression of cellular function may occur in other organs as well. Maintenance of cerebral blood flow autoregulation appears to remain intact with alpha stat management, whereas flow becomes pressure dependent with pH stat. Alpha stat management is probably associated with better neurological outcomes. See: Patel et al. References Murkin JM, Farrar JK, Tweed WA, McKenzie FN, Guiraudon G. Cerebral autoregulation and flow / metabolism coupling during cardiopulmonary bypass: the influence of PaCO2. Anesth Analg. 1987 Sep;66(9):825-32. Rogers AT, Stump DA, Gravlee GP, Prough DS, Angert KC, Wallenhaupt SL, Roy RC, Phipps J. Response of cerebral blood flow to phenylephrine infusion during hypothermic cardiopulmonary bypass: influence of PaCO2 management. Anesthesiology. 1988 Oct;69(4):547-51. Patel RL, Turtle MR, Chambers DJ, James DN, Newman S, Venn GE. Alpha-stat acid-base regulation during cardiopulmonary bypass improves neuropsychologic outcome in patients undergoing coronary artery bypass grafting. J Thorac Cardiovasc Surg. 1996 Jun;111(6):1267-79.

453

Number: 204 Which of the following may be a cause of micro-embolism during Cardio-pulmonary bypass (CPB)? 1. Rapid re-warming. 2. Rapid cooling. 3. PVC from the roller pump insert of the circuit. 4. pH stat management A: 1,2,3 Correct B: 1,3 Correct C: 2,4 Correct D: 4 Correct E: All Correct ABCDE Correct Answer: A There are numerous causes of microembolism during CPB. Gaseous microembolism can occur during both heating and cooling of the patient. At the initiation of CPB (when rapid cooling may be occuring) microemboli may be created in the patient as the cold blood reaches the tissues and is rapidly re-warmed. Conversely, during rewarming, if blood reaches the oxygenator relatively well saturated then raising the blood temperature in the heat exchanger will produce showers of microemboli in the extra-corporeal circuit. In both cases, the mechanism of embolisation is the same. - As the temperature of the blood is raised, so the solubility of oxygen in the blood is reduced and the gas comes out of solution. In addition, as temperature rises, so does PCO2. This has the secondary effect of decreasing the affinity of haemoglobin for oxygen and as a result, further raising the PO2. Roller pumps tend to microfracture the interior surface of the PVC (or Silicone Rubber) 'boot'. This phenomenon is known as 'spallation'. The result is particulate micro-embolism of either PVC or Silicon into the circulation. This effect is particularly marked with Silicone Rubber. In addition, silicate particles from the defoamer used in reservoirs has also been found to embolise. pH stat management is not a cause of microembolism, but may make the sequelae of embolism worse. - As cerebral blood flows during pH stat are often inappropriately high, the brain is exposed to a larger number of emboli when compared to alpha stat management. References: Analysis of' Microembolic Particles Originating in Extracorporeal Circuits. Perfusion, Vol. 2, No. 1, 1987 Gideon Uretzky Particulate microembolism resulting from extracorporeal circuits (ECC) are regarded as the cause of many complications and organ dysfunction after cardiopulmonary bypass (CPB). This investigation demonstrated that large numbers of particles are released from that part of the tubing which is in contact with the roller pumphead during pumping. The present study identifies and quantitates by number, size and time the nonbiological microembolic particles

454

spalled off polyvinyl chloride (PVC) and silicon tubing during recirculation without an oxygenator in the circuit. These nonbiological particles carry a great hazard since they are liable to be retained in the patients organs following CPB. The mock circulation set-up in this study managed to show substantial production of these nonbiological microemboli by a roller pump in conjunction with the use of silicone rubber and PVC tubing. The fact that even during a short period of pumping, silicone rubber spalls off particles precludes its use in CPB. The Silicone tubing continuously generated a marked quantity of particulate matter at a steadily increasing rate. Electron microscopic examination of the silicone tubing revealed multiple particles emanating from the inner surface of the tubing. When using PVC tubing, the amount of particles of all sizes rose above acceptable standards shortly after activating the roller pump. Following the early upsurge, the number of particles continued to increase for the remainder of the pump run, albeit at a more moderate rate. The authors advocate replacing roller pump heads with a recently developed, less traumatic, vortex pump, for routine usage during cardiopulmonary bypass. Number: 233 In a patient on bypass with a hollow-fibre membrane lung, an abnormally high inlet (premembrane) pressure is more likely to be observed if: 1. The pre-operative platelet count is high. 2. The circuit prime contains albumen. 3. The patient has demonstrated relative heparin resistance. 4. The patient is normothermic. A: 1,2,3 Correct B: 1,3 Correct C: 2,4 Correct D: 4 Correct E: All Correct ABCDE Correct Answer: B Abnormally high inlet pressures are occasionally observed with modern, hollow-fibre membranes. The aetiology is uncertain, but probably relates to cryoprecipitation of fibrinogen. The incidence can be eliminated or greatly reduced by including in the circuit prime a small amount (~2 grams) of albumen. Number: 241 Which of the following manoeuvres may improve outcome in a patient who is to be subjected to a deep hypothermic arrest at 17 degrees centigrade? 1. Haemodilution to a haematocrit of 18 - 20%. 2. Maintenance of a blood sugar level of at least 15 mmol/L.

455

3. Administration of thiopentone prior to the arrest. 4. Induction of hypocarbia prior to arrest. A: 1,2,3 Correct B: 1,3 Correct C: 2,4 Correct D: 4 Correct E: All Correct ABCDE Correct Answer: B Hypothermia is associated with marked increases in blood viscosity. As a result, microcirculatory flow is far better maintained under hypothermic conditions if haemodilution is used. The presence of hyperglycaemia at the time of a neurological insult is associated with poorer outcomes. The administration of barbiturates prior to an ischaemic insult probably improves neurological outcome. Hypocarbia prior to arrest is almost certainly of no benefit to the patient Number: 242 With regard to the following balloon pump trace: 1. Inflation is too early. 2. The assistance ratio is 1:1. 3. Deflation is too early 4. Deflation is too late. A: 1,2,3 Correct B: 1,3 Correct C: 2,4 Correct D: 4 Correct E: All Correct

456

ABCDE Correct Answer: D Inflation is correctly timed to a point just before the dicrotic notch and the assistance ratio is 1:2. Deflation is set too late, as evidenced by a 'Pump End-Diastolic Pressure' which exceeds the patient's own diastolic pressure. Of all the 'mistimings' which are possible with an IABP, the two most harmful are early inflation and late deflation. Number: 316 With reference to hypothermic blood cardioplegia: 1. Bicarbonate will potentially reduce the availability of the delivered oxygen in the solution. 2. Hypothermia will potentially reduce the availability of the delivered oxygen in the solution. 3. Myocardial acidosis during cross-clamping will facilitate the extraction of oxygen from the solution. 4. The partial pressure of oxygen in the delivered solution has little effect on myocardial preservation. A: 1,2,3 Correct B: 1,3 Correct C: 2,4 Correct D: 4 Correct E: All Correct ABCDE Correct Answer: A Vinten-Johansen J, Julian JS, Yokoyama H, et al. Efficacy of myocardial protection with hypothermic blood cardioplegia depends on oxygen. Ann Thorac Surg 1991;52:939-948.

457

Vinten-Johansen et al. studied whether oxygen was an important factor in the myoprotection afforded by hypothermic blood cardioplegia. The oxygen content was adjusted by controlled gas mixture in the cardioplegia delivery line to fully saturated (10.2 +/- 0.6 ml O2 per deciliter), intermediate (4.3 +/- 0.5 ml O2 per deciliter) and low (1.1 +/- 0.2 ml O2 per deciliter) levels, while other compositional attributes were held constant. The heart was subjected to 30 minutes of normothermic ischemia, and hypothermic (4C) blood cardioplegia was delivered every 20 minutes for 1 hour of cardioplegia arrest. Postischaemic left ventricular function, evaluated by pressure-volume relations, generally reflected the level of oxygen content in the cardioplegia. Postischaemic left ventricular performance was wellpreserved in the saturated blood cardioplegia group, moderately depressed in the intermediate oxygenated group, and severely depressed in the relatively desaturated group (Fig. 7.11). It was concluded that oxygen is extracted in significant quantities from hypothermic blood cardioplegia, and that oxygen is important to the myocardial protection provided by hypothermic blood cardioplegia. Therefore, despite the unfavorable shift in the oxyhaemoglobin dissociation curve with profound hypothermia, the ischemic myocardium apparently creates an environment that counteracts these effects of hypothermia and facilitates extraction of oxygen. These enviromental forces include: * the increased tissue PCO2 * tissue acidosis (Bohr effect) * the facilitated extraction due to low tissue PO2 and * the greater affinity of hypothermic tissue for oxygen. Number: 317 Which of the following will reduce the likelihood of myocardial reperfusion injury: 1. The use of a leucocyte depletion filter in the cardioplegia system . 2. The addition of mannitol to the cardioplegia solution. 3. The addition of a calcium channel blocker to the cardioplegia solution. 4. The addition of glucose to the cardioplegia solution. A: 1,2,3 Correct B: 1,3 Correct C: 2,4 Correct D: 4 Correct E: All Correct ABCDE Correct Answer: B The majority of damage resulting from suboptimal myocardial protection is realised during reperfusion. Experimental evidence gained from research on long-term myocardial preservation for cardiac transplantation supports the concept that reperfusate modification can lead to subsequent improvements in myocardial function. In particular, leucocyte depletion

458

with blood filters has been demonstrated to significantly enhance the functional recovery of donor hearts preserved for 12 hours with Stanford solution. Calcium plays a central role in the establishment of a reperfusion injury. For this and other reasons calcium channel blockers have been added to cardioplegia solutions. Mannitol will reduce the likelihood of myocardial oedema and improve the distribution of blood flow once perfusion is re-established. It will not modify the reperfusion injury process itself. Glucose will provide a substrate source, but may, in fact, worsen the reperfusion injury: Number: 322 Which statements are true of the 'Delphin' centrifugal pump? 1. Its output will change in response to a change in the vascular resistance of the patient. 2. It works on the principle of 'viscous drag'. 3. It can be used to kinetically assist venous drainage. 4. It can be interfaced to the 'Biomedicus' pump console. A: 1,2,3 Correct B: 1,3 Correct C: 2,4 Correct D: 4 Correct E: All Correct ABCDE Correct Answer: B Like all centrifugal pumps, output is afterload dependent. Unlike the 'Biomedicus' pumphead, it is an impeller device. As with all centrifugal pumps, it can be used in the venous line to kinetically assist venous drainage through a small venous cannula. It can only be interfaced to the Sarns pump console. Number: 451 A 'Biomedicus' centrifugal pump is hand-cranked in the wrong direction. In comparison to a pump being hand-cranked in the right direction at the same rpm, this will result in: A. Blood flow at the same rate in an antegrade direction. B. Blood flow at the same rate in a retrograde direction. C. Blood flow at a reduced rate in an antegrade direction. D. Blood flow at a reduced rate in a retrograde direction. E. None of the above. Select the single best answer

459

ABCDE Correct Answer: C Perhaps surprisingly, the 'Biomedicus' centrifugal pump is almost as efficient going 'backwards' as compared to going 'forwards. Note that the direction of blood flow is NOT reversed when the direction of rotation of the pump head is reversed (unlike the case of a roller pump). Number: 452 A capnograph applied to the effluent port of a co-current hollow fibre membrane oxygenator wll provide: A. An accurate measure of arterial PCO2 at the current nasopharyngeal temperature. B. An accurate measure of arterial PCO2 at the current arterial temperature. C. An accurate measure of venous PCO2 at the current nasopharyngeal temperature. D. An accurate measure of venous PCO2 at the current arterial temperature. E. None of the above. Select the single best answer ABCDE Correct Answer: B The relationship between effluent PCO2 and arterial CO2 is not well-recognised by perfusionists. A capnograph applied to the effluent port of a co-current hollow fibre membrane gives a very accurate indication of the arterial PCO2 corrected to the current arterial temperature. This is in contrast to a bubble oxygenator which has a relatively high deadspace - and, as a result, there is no close relationship between gas and blood phase PCO2. Number: 459 Blood at 27 centigrade with a haematocrit of 24% is equilibrated with oxygen at a partial pressure of 100 mm Hg. It is then warmed under hermetic (sealed) conditions to 37 centigrade. The PO2 at 37 will be approximately: A. 73 mm Hg. B. 100 mm Hg. C. 118 mm Hg. D. 143 mm Hg. E. 171 mm Hg. Select the single best answer

460

ABCDE Correct Answer: D 143 mm Hg. - Remember that two relevant phenomena occur when blood is warmed. Firstly, the solubility of oxygen in plasma decreases and secondly the haemoglobin dissociation curve shifts to the right. Both these effects tend to raise the partial pressure of oxygen as it both comes out of solution and dissociates from haemoglobin. The basis of the calculation is quite complex, but a working model of the dissociation curve can be obtained from the Manbit website if you want to see it in action. Number: 541 Which of the following drugs will reverse the anti-coagulant effect of unfractionated heparin? 1. Protamine. 2. Heparinase. 3. Hexadimethrine. 4. Recombinant platelet factor IV (rPF4). A: 1,2,3 Correct B: 1,3 Correct C: 2,4 Correct D: 4 Correct E: All Correct ABCDE Correct Answer: E A. Cooney, T. J. Mann: Anaesth Intensive Care 1999; 27: 298-300 Recent Experiences with Hexadimethrine for Neutralizing Heparin After Cardiopulmonary Bypass. Hexadimethrine bromide (Polybrene) is a poly-cation that was introduced into clinical practice in 1954 and was used extensively for neutralization of heparin after CPB. In 1962, it was reported to cause proteinuric renal failure if given in excessive amounts and its use was largely curtailed. It was recommended for use in protamine-allergic patients by Doolan in 1981. There have been two recent in vitro studies comparing it favourably with other agents for the reversal of heparin. Protamine is usually used for neutralization of heparin after cardiopulmonary bypass (CPB). It is not suitable for use in some patients, such as those who may have had significant previous allergic reactions to either intravenous protamine or protamine-containing insulin preparations. The overall incidence of adverse reactions to protamine has been quoted as 10.7%, with 1.6% having severe cardiovascular reactions'.

461

Alternatives to protamine include heparinase and recombinant platelet factor IV (rPF4), neither of which is widely available. Number: 544 Which of the following drugs may improve neurological outcome when used in the setting of open-heart surgery? 1. Thiopentone 2. Lignocaine 3. Aprotinin 4. Ketamine. A: 1,2,3 Correct B: 1,3 Correct C: 2,4 Correct D: 4 Correct E: All Correct ABCDE Correct Answer: A The answer is debatable!! Thiopentone, Lignocaine and Aprotinin have all been associated with improved neurological outcomes in various studies. Ketamine, an NMDA anatagonist will theoretically reduce reperfusion injury, but has never been clinically trialled as far as I am aware. Number: 546 A patient scheduled for mitral valve replacement who is found to have cold agglutinins: 1. Requires pre-operative exchange transfusion and steroid therapy. 2. Is likely to have antibodies to red-cell 'I' or 'i' antigens. 3. Has antibodies which are likely to be active at 37 centigrade. 4. May require pre-operative plasmapheresis. A: 1,2,3 Correct B: 1,3 Correct C: 2,4 Correct D: 4 Correct E: All Correct ABCDE Correct Answer: C

462

Cold agglutinaemia is a condition characterised by the presence of agglutinating or haemolysing serum antibodies that become active at a decreased body temperature. The antibodies are classically directed at the 'I' or 'i' antigens on the red blood cells and can be characterised in the laboratory by their 'Thermal Amplitude' - the range in which the antibody is active and by their 'Activity Titre' - the dilutional concentration below which the activity of the antibody becomes undetectable. Cold agglutinaemia can present as either a primary or secondary form of the disease. The former occurs late in life and is usually asymptomatic. The secondary form usually occurs as a sequel to mycoplasma or cytomegalovirus infection or as a manifestation of an underlying lymphoproliferative disorder. The overall incidence of the condition is in the order of 1:75,000 and is sufficiently uncommon to make routine pre-operative screening unwarranted. The condition is usually of little clinical importance. However, in the setting of cardiac surgery, the condition is of greater relevance because of the use of hypothermia. Thus an antibody which is active at 4 centigrade at a dilution of 1:8 may present a problem if the administration of cold cardioplegia is required, whereas an antibody active at 32 centigrade at a dilution of 1:256 constitutes a major contraindication to bypass unless treated appropriately. The first-described management strategy of the condition included the use of exchange transfusion and steroid therapy. This is now recognised as being generally unnecessary. The use of normothermic ischaemic arrest, warm crystalloid cardioplegia and possibly preoperative plasmapheresis - together with meticulous attention to the avoidance of systemic cooling, are usually all that is required for the safe management of the disorder. Number: 743 Which of the following therapeutic techniques are unequivocally indicated in the treatment of Massive Air Embolism (MAE) occurring during cardio-pulmonary bypass? 1. Retrograde Cerebral Perfusion. 2. Administration of thiopentone. 3. Hyperbaric Oxygen Therapy (HBT). 4. Administration of dexamethasone. A: 1,2,3 Correct B: 1,3 Correct C: 2,4 Correct D: 4 Correct E: All Correct ABCDE Correct Answer: B The use of Retrograde Cerebral Perfusion and post-operative Hyperbaric Oxygen Therapy are clearly indicated following an episode of MAE complicating CPB. Unfortunately, HBT is

463

often not available as a treatment option (and is most effective if administered within 6 hours of the incident). Readers are referred to the classic paper by Mills and Ochsner on the causes, prevention, and management of MAE during CPB. The role of drugs such as thiopentone or dexamethasone administered after the event is unclear and not of proven benefit. For those readers who have access to the publication 'Australasian Anesthesia', there is an excellent article by Stephen Same in the 2000 issue (pp 147-158). Number: 766 A 70 kg adult is undergoing cardiopulmonary bypass at 30 degrees centigrade. The following measurements are noted: Arterial Oxygen Content 100 mls/L. Mixed Venous Oxygen Content 85 mls/L. Arterial pump output 5.0 L/min. The patient's oxygen consumption (VO2) is: A. 15 mls/min. B. 30 mls/min. C. 45 mls/min. D. 75 mls/min. E. 105 mls/min. Select the single best answer ABCDE Correct Answer: D VO2 is simply the product of the AV content difference for oxygen multiplied by the cardiac output. In essence, this is the Fick principle. Adolf Fick never in fact used his principle to determine cardiac output! - He was also the inventor of the contact lens. (See Fick AE. A contact lens. 1888. Arch Ophthalmol. 1997 Jan;115(1):120-1. For a reprint of his original report) Number: 820 A 27 year old, hepatitis C positive, intravenous drug-user (IVDU) with infective endocarditis is undergoing an emergency aortic valve replacement. A hollow fibre oxygenator is in use. The circuit has been primed with 2000 mls of Hartmann's solution, 500 mls of 4% albumen and 10000 units of heparin. The patient has been heparinised before bypass and the measured activated clotting time is 666 seconds. Twenty minutes into the procedure, after the patient

464

has been cooled to 28 degrees centigrade, the oxygenator inlet pressure is noted to be 850 mm Hg despite a reduction in main pump flow to 1.5 litres/minute. The most likely cause of the oxygenator failure is: A. High inlet pressure syndrome. B. Inadequate heparinisation. C. A manufacturing fault in the oxygenator. D. The presence of cold-agglutinins in the patients blood. E. The presence of cryoglobulins in the patients blood. Select the single best answer ABCDE Correct Answer: E There is a strong relationship between infection with hepatitis C and the syndrome of 'Mixed cryoglobulinemia'. See, for example: Minerva Med 2001 Feb;92(1):35-42 Mixed cryoglobulinemia and hepatitis C virus infection. Lunel F, Musset L. It is also quite conceivable that the patient may have cold-agglutinins in his blood - both because of the association of hepatitis C with lymphoreticular tumours and because of the fact that IVDUs are at greater risk of acquiring atypical infections (such as mycoplasma) than the normal population. The 'High inlet pressure syndrome' can affect most hollow-fibre oxygenators, but is extremely unusual in cases where the membrane has been primed with solutions containing albumen. Number: 870 The most specific indicator of brain injury following cardio-pulmonary bypass is an increased serum level of: A. Lactate Dehydrogenase (LDH). B. Troponin T. C. Creatine Phosphokinase (CPK). D. Protein S100B. E. C-reactive protein. Select the single best answer ABCDE Correct Answer: D The level of serum protein S100B is a sensitive and specific biochemical marker of cerebral injury following cardio-pulmonary bypass. Protein S100 is an acidic calcium binding protein with a molecular weight of 21 kDa. The S100 family comprises 17 monomers, each of which exhibits a unique pattern of tissue-

465

specific expression. Two of the monomers, S100A1 and S100B, are found in high concentrations in the nervous system of vertebrates. These two monomers form either homodimers or heterodimers. S100BB is present in high concentrations in glial and Schwann cells and S100A1B is present in glial cells. Structural damage to the glial cells causes leakage of S100BB protein into the extracellular compartment and finally into the patient's serum, in which a monoclonal two-sided immunoradiometric assay can measure the B-subunit of protein S100. Several studies suggest that S100B level might be a useful prognostic marker for detection of brain injury in cardiac surgery. See, for example: Westaby S et al. Ann Thorac Surg 1996 Jan;61(1):88-92 Serum S100 protein: a potential marker for cerebral events during cardiopulmonary bypass. Number: 927 Which of the following forms of therapy is / are likely to be clinically effective in reversing the anti-coagulant effects of unfractionated heparin given to a patient before or during cardiopulmonary bypass (CPB)? 1. Protamine. 2. Fresh Frozen Plasma. 3. Heparinase - I 4. Heparinase - III A: 1,2,3 Correct B: 1,3 Correct C: 2,4 Correct D: 4 Correct E: All Correct ABCDE Correct Answer: B Heparinase - I is emerging as an important drug for the reversal of the anti-coagulant effects of unfractionated heparin - because it is practically as effective as protamine, but virtually devoid of any significant side effects. Protamine use is associated with significant adverse responses, including systemic hypotension, pulmonary vasoconstriction and anaphylactic reactions. It also inhibits platelet function, and in excess may exert an anticoagulant effect itself. Transfusion of platelets and / or fresh frozen plasma instead of protamine administration after CPB does not prevent substantial haemorrhage.

466

The Gram-negative soil bacterium, Flavobacterium heparinum, synthesises a family of enzymes that degrade glycosaminoglycans. Heparinase - I lyses heparin at its glycosidic linkages, whereas heparinase - III degrades heparan. Refer to the recent paper by Heres et al. According to these authors "Protamine returns heparin-induced anti-factor IIa and anti-factor Xa activities to undetectable levels. Heparinase-I, however, neutralizes only approximately 70% of the anti-factor Xa activity while also returning anti-factor IIa activity to zero. The clinical effects of the residual anti-factor Xa activity after heparinase-I administration remain unknown, because this trial was not designed to probe the possible beneficial (e.g., antithrombotic) or deleterious (e.g., hemorrhagic) effects of residual anti-factor Xa activity. " The dose of Heparinase - I is 7-10 micrograms / kg. Number: 928 An adult is undergoing cardio-pulmonary bypass at 26 degrees celsius. The perfusionist is attempting to follow a 'pH Stat' management strategy. The target PaCO2 (measured at 37 degrees celsius) which the perfusionist should aim for is approximately: A. 15 mm Hg. B. 28 mm Hg. C. 40 mm Hg. D. 48 mm Hg. E. 65 mm Hg. Select the single best answer ABCDE Correct Answer: E The aim of 'pH Stat' management is to maintain normocarbia (and a pH of 7.40) in the arterial blood at the actual temperature of the patient (in this case 26 celsius) whereas an 'alpha-stat' management strategy aims for normocarbia of the arterial blood at 37 celsius. Blood with a PCO2 of 40 mm Hg at 26 degrees celsius will have a PCO2 of about 65 mm Hg when warmed to 37 celsius (depending on the temperature correction algorithm used). The alternative method of acid-base strategy is termed 'alpha-stat'. This technique is based on the proposition that the constancy of the 'internal milieu' is accomplished predominantly by the buffering capacity of the imidazole group of histidine. As temperature changes, this imidazole protein buffer changes its pK, in parallel with the pN of water. The fraction of unprotonated histidine imidazole groups, known as alpha, remains constant; total CO2 remains constant; and pH changes as temperature changes. The term "alpha-stat" refers to maintenance of this constant net charge on proteins with temperature changes by keeping total CO2 stores constant.

467

Several formulae are available for adjusting a blood gas pCO2 determined at 37 C for a patient's body temperature. For elevated body temperatures the pCO2 is increased; for reduced body temperatures the pCO2 is decreased. corrected pCO2 = (measured pCO2) * (10^ ( X * ((patient temperature in C) - 37))) where X = 0.021 for the Radiometer ABL-3 X = 0.019 for the Corning and Instrumentation Laboratory (IL) instruments References: Ashwood ER, Kost G, Kenny M. Temperature correction of blood-gas and pH measurements. Clin Chem. 1983;29:1877-1885. Ciba-Corning 288 Blood Gas System Operator's Manual. 1988, page 2-15 Kelman GR, Nunn JF. Nomograms for correction of blood pO2, pCO2, pH and base excess for time and temperature. J Appl Physio. 1966; 21: 1484-1490. Pruden EL, Siggaard-Andersen O, Tietz NW. Chapter 30: Blood gases and pH. pages 13751410 (1406-1409). IN: Burtis C, Ashwood E. Tietz Textbook of Clinical Chemistry, Second edition. W.B. Saunders Company. 1994. Number: 929 An adult patient is undergoing cardiopulmonary bypass at a temperature of 26 degrees celsius using an 'alpha-stat' management strategy. Which of the following manouevres is / are likely to abolish cerebral autoregulation? 1. Conversion to a 'pH-stat' management strategy. 2. Cooling to a core temperature of 18 degrees celsius. 3. The administration of 1000mgs of sodium thiopentone. 4. The administration of 1% isoflurane. A: 1,2,3 Correct B: 1,3 Correct C: 2,4 Correct D: 4 Correct E: All Correct ABCDE Correct Answer: A Cerebral autoregulation is not well preserved during pH stat management. - See, for example, Murkin et al.

468

Similarly, profound hypothermia and deep anaesthesia completely abolish cerebral vasoreactivity. Under these circumstances, cerebral blood flow becomes completely pressure dependent. The administration of 1% isoflurane will have little impact on cerebral vasoreactivity if an 'alpha-stat' technique is being used. See: Aladj et al. Murkin JM, Farrar JK, Tweed WA, McKenzie FN, Guiraudon G. Cerebral autoregulation and flow / metabolism coupling during cardiopulmonary bypass: the influence of PaCO2. Anesth Analg. 1987 Sep;66(9):825-32. Rogers AT, Stump DA, Gravlee GP, Prough DS, Angert KC, Wallenhaupt SL, Roy RC, Phipps J. Response of cerebral blood flow to phenylephrine infusion during hypothermic cardiopulmonary bypass: influence of PaCO2 management. Anesthesiology. 1988 Oct;69(4):547-51. Aladj LJ, Croughwell N, Smith LR, Reves JG. Cerebral blood flow autoregulation is preserved during cardiopulmonary bypass in isoflurane-anesthetized patients. Anesth Analg. 1991 Jan;72(1):48-52. Number: 1011 A 50 year old man presents for urgent on-pump, coronary artery grafting after suffering an acute coronary syndrome. He has been treated with intravenous heparin for 10 days. In the last two days his platelet count has fallen from 250,000 / microlitre to 80,000 / microlitre, and Heparin-PF4 antibodies have been detected. He has no clinical evidence of intravascular thrombosis and the heparin infusion has now been stopped. Given this scenario, acceptable anticoagulant strategies for cardiopulmonary bypass include: 1. The use of systemic heparin in conventional dosages. 2. The use of systemic heparin together with a Tirofiban infusion. 3. The use of a Carmeda coated circuit without systemic heparinisation. 4. Systemic anticoagulation with hirudin. A: 1,2,3 Correct B: 1,3 Correct C: 2,4 Correct D: 4 Correct E: All Correct ABCDE Correct Answer: C Given this scenario, one is more or less obliged to take specific measures to avoid a catastrophic HIT Type II thrombotic response to heparin. Two broad strategies are available.

469

1. The use of heparin in conventional doses while at the same time preventing platelet aggregation with potent platelet inhibitors such as tirofiban or epoprostenol. 2. The avoidance of heparin completely while anticoagulating the patient with drugs such as hirudin, ancrod, danaparoid etcetera. It should be noted that option 3 (The use of a Carmeda coated circuit without systemic heparinisation.) is completely unacceptable as the patient is not protected against platelet aggregation and / or thrombosis induced by other mechanisms, neither does it eliminate the risk of a catastrophic HIT type II response. Thrombocytopaenia associated with the use of heparin occurs in two general forms. The first (HIT Type I) is a benign type seen in many patients 1-3 days after starting standard heparin and felt to be caused by the non-immune enhancement of platelet aggregation followed by sequestration and removal by the spleen. The platelet count usually remains above 100,000, resolves spontaneously, and requires neither cessation of heparin nor other treatment. It is not associated with the development of thrombosis. Unlike HIT Type I, HIT Type II is a more severe, delayed-onset thrombocytopaenia that is immunologically mediated. Consistent with the generation of an immune response, HIT Type II typically occurs approximately 5 to 14 days following initiation of heparin therapy but may occur earlier if the patient has been previously exposed to heparin. The thrombocytopaenia is usually more severe than in HIT Type I, with platelet counts often dropping to 60,000/ microlitre or below and remaining low until heparin is withdrawn. After cessation of heparin therapy, platelet counts usually take several days to rebound, but in rare cases may take up to 1 month to normalise. HIT Type II is much more dangerous than Type I because a proportion of those affected will go on to develop intravascular thrombosis. The diagnosis of HIT Type II is based on immunological testing and platelet function analysis. The immunological test consists of an enzyme-linked immunosorbent assays (ELISA) which measures the presence of antibodies capable of binding the heparin-PF4 complex. Although this test may be more sensitive than the functional tests, it is also less. Sensitivities and specificities also vary depending on the specific ELISA used. Various functional analyses have been used for establishing the diagnosis, but the most sensitive and specific appears to be the serotonin release assay (SRA). Donor platelets are isolated from platelet-rich plasma and incubated with radiolabeled (14C) serotonin. The platelets are then washed to remove free serotonin and incubated with heat-inactivated patient serum in the presence of therapeutic (0.1 U/mL) and high (100 U/mL) concentrations of heparin. A positive result is defined as more than 20% release at low heparin concentrations and less than 20% release at high concentrations of heparin. The test is generally thought to be the most sensitive test for confirmation of HIT Type II. See: von Segesser LK, Mueller X, Marty B, Horisberger J, Corno A. Alternatives to unfractionated heparin for anticoagulation in cardiopulmonary bypass. Perfusion. 2001 Sep;16(5):411-6.

470

Aouifi A, Blanc P, Piriou V, Bastien OH, Ffrench P, Hanss M, Lehot JJ. Cardiac surgery with cardiopulmonary bypass in patients with type II heparin-induced thrombocytopenia. Ann Thorac Surg. 2001 Feb;71(2):678-83. Nuttall GA, Oliver WC Jr, Santrach PJ, McBane RD, Erpelding DB, Marver CL, Zehr KJ. Patients with a History of Type II Heparin-Induced Thrombocytopenia with Thrombosis Requiring Cardiac Surgery with Cardiopulmonary Bypass: A Prospective Observational Case Series. Anesth Analg. 2003 Feb;96(2):344-50.

PHARMACOLOGY TESTS Number: 11 When used under hyperbaric conditions, nitrous oxide (N2O): 1. Tends to cause hypotension. 2. May cause clonus. 3. Has a Minimum Alveolar Concentration (MAC) value of about 180%. 4. Tends to cause tachypnoea. A: 1,2,3 Correct B: 1,3 Correct C: 2,4 Correct D: 4 Correct E: All Correct ABCDE Correct Answer: C Although N2O has been used to produce analgesia and anaesthesia for more than 100 yr, general anaesthesia with N2O can only be attained in a hyperbaric environment because the MAC value of the drug is 1.04 (104%). In a study in volunteers, Russell et al found that the anaesthetic state was associated with tachypnoea, tachycardia, increases in systemic blood pressure, mydriasis, diaphoresis, and at times, clonus and opisthotonus. See: Russell GB, Snider MT, Richard RB, Loomis JL. Hyperbaric nitrous oxide as a sole anesthetic agent in humans. Anesth Analg. 1990 Mar;70(3):289-95. Number: 87 Enflurane Anaesthesia: 1. May cause Spike and Wave activity at 2 MAC. 2. May sensitise the myocardium to the dysrhythmic effects of catechol amines at 1 MAC. 3. May impair the ventilatory response to hypoxaemia at subanaesthetic concentrations. 4. May produce a renal concentrating defect after 4 hours of 1 MAC.

471

A: 1,2,3 Correct B: 1,3 Correct C: 2,4 Correct D: 4 Correct E: All Correct ABCDE Correct Answer: E Enflurane causes epileptiform activity on the EEG that increases with depth of anaesthesia and hypocapnia. In healthy, normocapnic people.The arrhythmogenic threshold of the myocardium in the presence of adrenaline is twice as high for enflurane and five times as high for isoprenaline than halothane. Enflurane causes an increase in cerebral blood flow. It causes ventilatory depression at anaesthetic concentrations and will impair a patients response to hypoxaemia at subanaesthetic concentrations. It can impair the concentrating ability of the kidney but is does not cause renal impairment. EEG spiking is maximal at end-tidal concentrations of 2-3%, and grand mal patterns occur at 3-6%. At a given end tidal concentration, hyperventilation increases seizure activity, and hypoventilation decreases it such that the minimum epileptogenic concentration is approximately 1% lower at a PaCO2 of 20 mmHg, and 1% higher at a PaCO2 of 60 mmHg. Even though seizures have occurred days after enflurane anaesthesia in non-epileptic patients, EEG documentation of postoperative seizure activity is rare. One study exists involving 11 patients which shows that enflurane causes a mild impairment of renal concentrating ability. After 9.6 MAC hours maximal urinary osmolality following ADH was reduced from 1050 to 800 mOsm/kg. This was associated with a serum Fconcentration of 33 uM/L. Note that this was not associated with hypernatremia, hyperosmolality, or increased serum urea or creatinine and was therefore not considered clinically significant. It is generally accepted that enflurane does not cause renal dysfunction because the serum F- concentration never reaches threshold levels as it does with methoxyflurane. Some clinicians feel that enflurane may cause additional renal dysfunction when administered to patients with significant preexisting renal impairment, however this has not been borne out in rat studies. BENUMOF, J.L. & SAIDMAN, L.J; Anesthesia and perioperative complications, Mosby, 1992,p 360. MILLER, R(ED); Anesthesiology, Churchill-Livingstone, 1990, pp 123-5, p 135. The Medical Journal of Australia,'Modern Inhalational Agents- a review of halothane, isoflurane, and enflurane.'vol 150, Jan 16th, 1989. Number: 88 The following is true of Isoflurane: A. 25% of patients with coronary artery disease are prone to coronary steal. B. It decreases the rate of production of CSF and increases reabsorption. C. Hypercapnic cerebral vasodilation is reduced.

472

D. It is 0.4% metabolized. E. All of the above. Select the single best answer ABCDE Correct Answer: D 15% of patients with coronary artery disease are felt to be prone to coronary "steal". Despite this it's avoidance in CABG's is widely questioned. Isoflurane decreases resistance to reabsorption of CSF however production is unchanged. Enflurane increases production and increases resistance to reabsorption and Halothane increases production but decreases resistance to reabsorption. Hence, Isoflurane decreases CSF volume and limits rises in ICP due to CSF. This is particularly relevant in long procedures. Isoflurane allows most vascular responsiveness to CO2, hence hyperventilation induced lowering of ICP is most effective. Potentially ICP will be higher during hypercarbia. SHAW,J; " Modern inhalational anaesthetic agents:a review of halothane, isoflurane, and enflurane ", MJA, vol. 150, Jan 16, 1989, pp 95-100. Number: 89 Nitrous Oxide: 1. May lead to megaloblastic changes after 5 hours exposure in critically ill patients. 2. Is not teratogenic when administered to women during the first or second trimesters. 3. Lowers Pulmonary Artery Pressure. 4. Impairs the " hypoxic ventilatory response ". A: 1,2,3 Correct B: 1,3 Correct C: 2,4 Correct D: 4 Correct E: All Correct ABCDE Correct Answer: C Nitrous Oxide has never been demonstrated to be teratogenic to humans although it has been shown to be in animals. The drug raises pulmonary artery pressure - particularly in those with pre-existing pulmonary hypertension. The hypoxic ventilatory response is significantly impaired although CO2 sensitivity is little affected. KNILL, R.L & CLEMENT, J.L;" Variable effects of anesthetics on the ventilatory response to hypoxaemia in man ". Can Anaesth. Soc. J, 29, 1982, pp93-99.

473

SCHULTE-SASSE, U. HESS, W. & TARNOW, J;" Pulmonary vascular responses to N2O in patients with normal & high pulmonary vascular resistance ". Anesthesiology, 57,1982, pp 913. Number: 90 Methoxyflurane nephrotoxicity is more likely: 1. With preexisting liver disease. 2. After 1.5 MAC hours has been administered. 3. When Methoxyflurane has been administered within 3 months. 4. When other nephrotoxic agents are present. A: 1,2,3 Correct B: 1,3 Correct C: 2,4 Correct D: 4 Correct E: All Correct ABCDE Correct Answer: D Methoxyflurane nephrotoxicity is considered likely when plasma concentrations of 50 micromol/l are present. This correlates with a duration of 3.5 MAC hours. If enzyme induction has occured, it is possible that liberation of free fluoride ions will be augmented, but liver disease in general will not increase the likelihood of toxicity. Recurrent exposure, unless it is very recent is not associated with toxicity (unlike Halothane Hepatitis). In general, renal dysfunction is more likely as the number of nephrotoxic insults increases. Vasopressin resistant polyuric renal failure was first reported in 1966. Subsequently, evidence was gathered indicating that the causative agent was free F-, a metabolic byproduct. The extent of renal dysfunction has been correlated with the dosage in MAC hours, and peak serum concentrations. After 2.5-3 MAC hours, which correlates with a serum concentration of 50-80 micromol//l, patients had subclinical toxicity characterized by elevated serum urate, decreased urate clearance, and delayed return to preoperative serum osmolality. After 5 MAC hours, serum concentrations were 90-120 micromol//L, and patients had well established but mild nephrotoxicity, characterized by hyperosmolality, hypernatremia, polyuria, and urinary hypoosmolality. After 9 MAC hours,serum concentrations were 175 micromol//L and were associated with marked nephrotoxicity. Marked patient variability occurs. The multifactorial nature of renal dysfunction is held responsible for this as well as genetic predisposition. For example, an additive effect is seen with methoxyflurane and aminoglycosides, which are also nephrotoxic. BERMAN, M.L & HOLADAY, D.A;" Inhalation anesthetic metabolism & toxicity ", Clinical Anesthesia. Eds:Barash, Cullen & Stoelting, JB Lippincott & Co, 1989.

474

Number: 91 Which of the following agents can react with soda lime: A. Trichlorethylene. B. Ethyl Chloride. C. Halothane. D. Sevoflurane. E. All of the above. Select the single best answer ABCDE Correct Answer: E Halothane and Sevoflurane both interact with soda lime at 65 degrees C. Surprisingly, the breakdown products of halothane are much more toxic than those of sevoflurane. Trichlorethylene and ethyl chloride both react with soda lime at clinically significant temperatures BROWN, B; Can J Anaesth, 39, 1992, pp 207-209. Number: 180 A 20 ml ampoule of 'Diprivan' (propofol): 1. Contains egg lecithin. 2. Contains soybean oil. 3. Contains glycerol. 4. Has a pH of 3.7. A: 1,2,3 Correct B: 1,3 Correct C: 2,4 Correct D: 4 Correct E: All Correct ABCDE Correct Answer: A The formulation contains soybean oil (100 mg/mL), glycerol (22.5 mg/mL), egg lecithin (12 mg/mL), and disodium edetate (0.005%); with sodium hydroxide to adjust pH. Diprivan Injectable Emulsion is isotonic and has a pH of 7-8.5. The formulation of the drug - particularly the fact that it is an emulsion - has important pharmcaodynamic effects. See, for example: Dutta S, Ebling WF. Emulsion formulation reduces propofol's dose requirements and enhances safety. Anesthesiology. 1997 Dec;87(6):1394-405. A generic form of the drug has recently become available.

475

Number: 213 The boiling point of nitrous oxide (at atmospheric pressure) is: A. + 36.4 C B. - 90.9 C C. - 88.5 C D. + 34.6 C E. - 12.5 C Select the single best answer ABCDE Correct Answer: C Some physical constants of nitrous oxide are: Freezing point: - 90.9 C Boiling point: - 88.5 C (at atmospheric pressure) Critical temperature 309.6 K (+ 36.4 C) Critical pressure 72.4 bar abs Critical volume 0.097 m/kmol Number: 229 The minimum alveolar concentration ( MAC) of Sevoflurane: 1. Is approximately 2.0% in a 40 year old man. 2. Is approximately 1.4% in an 80 year old man. 3. Is approximately 3 times the MAC of Halothane. 4. Is approximately 1/3rd the MAC of Desflurane. A: 1,2,3 Correct B: 1,3 Correct C: 2,4 Correct D: 4 Correct E: All Correct ABCDE Correct Answer: E The MAC of sevoflurane falls progressively with age from about 3% at age 3 years to about 1.4% at age 80 years. In 100% oxygen corresponding values for other volatile agents are: Halothane 0.7% Enflurane 1.6% Isoflurane 1.2% Sevoflurane 2.0% Desflurane 6.0%

476

Number: 230 Production of 'Compound A' from sevoflurane in a CO2 absorption system is increased when: 1. Canister temperatures are high. 2. Sevoflurane concentrations are high. 3. Fresh gas flow rates are low. 4. Baralyme is used instead of soda lime. A: 1,2,3 Correct B: 1,3 Correct C: 2,4 Correct D: 4 Correct E: All Correct ABCDE Correct Answer: E Sevoflurane produces low levels of 'Compound A' and trace levels of 'Compound B' when in direct contact with CO2 absorbants. Number: 231 With regard to the biotransformation of Sevoflurane: 1. Approximately 5% is metabolised. 2. Plasma fluoride levels peak 2-3 hours after administration of the agent. 3. The principal metabolite is hexafluoroisopropranolol. 4. A secondary metabolite is pentafluoro isopropenyl fluoromethyl ether. A: 1,2,3 Correct B: 1,3 Correct C: 2,4 Correct D: 4 Correct E: All Correct ABCDE Correct Answer: A Pentafluoro isopropenyl fluoromethyl ether is 'Compound A'. In contrast to methoxyflurane anaesthesia, - where potentially nephrotoxic levels of fluoride can be detected for 2-3 days after administration, the elevation of plasma fluoride which follows sevoflurane anaesthesia is transient and of a far lesser degree.

477

Sevoflurane is unique among the fluorinated inhalational agents in that its metabolites do not include Tri-fluoroacetic acid (TFA). TFA has been associated with the hepatotoxicity of the other inhalational agents - as a result of hapten formation with the TFA metabolite. Number: 232 Reported unwanted effects of sevoflurane include: 1. Impaired renal concentrating ability. 2. Epileptiform EEG activity. 3. Post-operative shivering. 4. Malignant hyperpyrexia. A: 1,2,3 Correct B: 1,3 Correct C: 2,4 Correct D: 4 Correct E: All Correct ABCDE Correct Answer: E Impaired renal concentrating ability has been occasionally reported following administration of sevoflurane. There have been single reports of both Epileptiform EEG activity (in the absence of convulsions) and Malignant hyperpyrexia in association with sevoflurane anaesthesia. Post-operative shivering is reported to follow about 6% of sevoflurane anaesthetics. Number: 308 With regard to Desflurane: 1. It has a MAC in 100% oxygen of about 6%. 2. It can be administered using a conventional variable bypass vapouriser. 3. It has a molecular weight of 168. 4. It has a boiling point of 53.5 degrees centigrade. A: 1,2,3 Correct B: 1,3 Correct C: 2,4 Correct D: 4 Correct E: All Correct

478

ABCDE Correct Answer: B In 100% oxygen the MAC values for other volatile agents are: Halothane 0.7% Enflurane 1.6% Isoflurane 1.2% Sevoflurane 2.0% Desflurane 6.0% *MAC of desflurane is age dependent: 18 to 35 years = 7.25%; 31 to 65 years = 6.0%. It has a boiling point of 23.5 C and for this reason cannot be administered using a conventional variable bypass (Tec-type) vaporiser. Rampil IJ, Lockhart SH, Eger EI II et al: The electroencephalographic effects of desflurane in humans, Anesthesiology 74:429, 1991. Number: 379 With regard to ketamine: 1. It is a direct myocardial depressant. 2. Emergence phenomena are more likely if anticholinergic premedication is used. 3. It may induce cardiac dysrhythmias in patients receiving tricyclic antidepressants. 4. Has little effect on intracranial pressure. A: 1,2,3 Correct B: 1,3 Correct C: 2,4 Correct D: 4 Correct E: All Correct ABCDE Correct Answer: A Ketamine is a direct myocardial depressant in vitro but increases sympathetic tone in vivo causing an increase in heart rate and blood pressure. Ketamine is a potent cerebral vasodilator and can increase intracranial pressure significantly. Emergence delirium occurs in 5% to 30% of patients. This phenomenon is more likely to occur with anticholinergic or droperidol premedication, in patients under 16 years of age, in females, and in patients with a psychiatric history. The incidence is decreased with benzodiazepine premedication. In patients receiving tricyclic antidepressants, ketamine may induce cardiac dysrhythmias.

479

Takeshita H, Okuda Y, and Sari A: The effects of ketamine on cerebral circulation and metabolism in man, Anesthesiology 36:69-75, 1972. Schwartz DA and Horwitz LD: Effects of ketamine on left ventricular performance, J Pharmacol Exp Ther 194:410-414, 1975. Number: 521 Etomidate: 1. Causes more nausea and vomiting than propofol. 2. Is rarely associated with anaphylaxis. 3. Causes adreno-cortical suppression. 4. Is contra-indicated in acute intermittent porphyria. A: 1,2,3 Correct B: 1,3 Correct C: 2,4 Correct D: 4 Correct E: All Correct ABCDE Correct Answer: E Etomidate produces a high incidence of postoperative nausea and emesis, especially when used in combination with opioid analgetics for short outpatient procedures. In contrast, propofol may have antiemetic properties, since the incidence of emetic sequelae after outpatient anaesthesia with propofol appears to be extremely low. (McCollum JSC, Milligan KR, and Dundee JW: The antiemetic action of propofol, Anaesthesia 43:239, 1988.) Etomidate does not increase plasma histamine levels and, although anaphylactic reactions to etomidate have been reported, it is probably the intravenous anesthetic least likely to induce a hypersensitivity reaction. (Watkins J: Etomidate: an "immunologically safe" anaesthetic agent? Anaesthesia 40:1015, 1985.) Etomidate inhibits the activity of the 17 alpha-hydroxylase and 11 beta-hydroxylase enzymes necessary for the synthesis of cortisol, aldosterone, 17-hydroxyprogesterone, and corticosterone. Even after an induction dose of etomidate, adrenal suppression persists for 5 to 8 hours. (Wagner RL, White PF: Etomidate inhibits adrenocortical function in surgical patients, Anesthesiology 61:647, 1984. ) The clinical importance of short-term blockade of cortisol synthesis is not known. It is said that etomidate is contra-indicated in acute intermittent porphyria although there is at least one case report of its safe use in this condition. (Famewo CE. Induction of anaesthesia with etomidate in a patient with acute intermittent porphyria. Can Anaesth Soc J. 1985 Mar;32(2):171-3.)

480

Number: 545 A propofol infusion: 1. Is typically used in the range 20 - 50 mcg/kg/min as a sedative in adult intensive care. 2. Is appropriate for use as an anticonvulsant in the treatment of status epilepticus. 3. Will provide surgical anaesthesia if used in the range 100 - 200 mcg/kg/min together with N2O. 4. Is appropriate for use as a sedative in paediatric intensive care. A: 1,2,3 Correct B: 1,3 Correct C: 2,4 Correct D: 4 Correct E: All Correct ABCDE Correct Answer: A 1. 20 - 50 mcg/kg/min is a typical rate of infusion for patients undergoing sedation and IPPV in an intensive care unit. 2. Notwithstanding the possibility that propofol may occasionally be associated with epileptiform activity, the drug has been found to be effective in the treatment of otherwise resistant status epilepticus. 3. A propofol infusion will provide surgical anaesthesia if used in the range 100 - 200 mcg/kg/min together with N2O. 4. The drug is contraindicated for administration by prolonged infusion in children. In this context it has been associated with the development of severe metabolic acidosis. BMJ 1992 Sep 12;305(6854):613-6 Metabolic acidosis and fatal myocardial failure after propofol infusion in children: five case reports. Parke TJ, Stevens JE, Rice AS, Greenaway CL, Bray RJ, Smith PJ, Waldmann CS, Verghese C Number: 560 Which statements are true with regard to the 'context-sensitive' half-time of an intravenous anaesthetic agent? 1. It is the time required for the plasma concentration to decrease by 50% on cessation of administration as a function of the duration of drug administration. 2. It is a better predictor of recovery than the terminal half-life. 3. It is useful for comparing the effect of a drug in different patient populations. 4. It is a better predictor of recovery than the 'effect-site decrement' time.

481

A: 1,2,3 Correct B: 1,3 Correct C: 2,4 Correct D: 4 Correct E: All Correct ABCDE Correct Answer: A The complex pharmacokinetics and pharmacodynamics of intravenous anaesthetics precludes easy prediction of the duration of anaesthetic effect after drug administration. The terminal elimination half-life often is discussed as if it predicts recovery - but in fact can be quite misleading. Hughes et al. introduced the 'context sensitive half-time,' the time required for a 50% decrease in plasma concentration, as a function of infusion duration - as an alternative to the concept of 'effect-site decrement time' which had been introduced by Shafer and Varvel in 1991. For a full discussion see the following papers: Evolving Clinically Useful Predictors of Recovery from Intravenous Anesthetics. Anesthesiology 83:902-905, 1995: Schnider, T W, Shafer,SL. Context-sensitive half-time in multicompartment pharmacokinetic models for intravenous anesthetic drugs. Anesthesiology 76:334-341, 1992: Hughes MA, Glass PS, Jacobs JR. Pharmacokinetics, pharmacodynamics, and rational opioid selection. Anesthesiology 74:5363, 1991 Shafer SL, Varvel JR. Number: 563 After exposure to 2 MAC-Hour of anaesthesia with the following inhalational anaesthetics, which agent would be associated with the lowest plasma inorganic fluoride levels? A. Methoxyflurane. B. Isoflurane. C. Sevoflurane. D. Enflurane. E. Desflurane. Select the single best answer ABCDE Correct Answer: E MOF > SEV > ENF > ISO > DES

482

The rate of sevoflurane metabolism is approximately one-third that of methoxyflurane, and 1.5-2 times that of enflurane, while that of desflurane is minimal. The extent of metabolism of sevoflurane and desflurane is 2-5% and 0.02-0.2% of the dose taken up, respectively. Peak serum fluoride concentrations occur within one hour after sevoflurane anesthesia, are usually in the range of 20-40 microM, and decline rapidly. Fluoride concentrations have exceeded 50 microM in approximately 7% of sevoflurane patients. Number: 570 Which of the following drugs is an NMDA receptor anatgonist? A. Thiopentone. B. Midazolam. C. Ketamine. D. Etomidate. E. Sevoflurane. Select the single best answer ABCDE Correct Answer: C NMDA receptors are unaffected by most anaesthetic drugs including the volatile agents and most induction agents. The one notable exception is ketamine, which has been shown to be a potent and selective inhibitor of NMDA resulting from binding to the phencyclidine site on the NMDA receptor protein Number: 589 The Meyer-Overton rule: 1. Is incompatible with the finding of 'pressure reversal' of general anaesthetics. 2. Correlates the potency of anaesthetic gases with their oil:gas partition coefficients. 3. Suggests that the site of action of volatile anaesthetic agents may be hydrophilic in nature. 4. Applies only to gases and volatile liquids.

A: 1,2,3 Correct B: 1,3 Correct C: 2,4 Correct D: 4 Correct E: All Correct ABCDE Correct Answer: C

483

The Meyer-Overton rule correlates the potency of the volatile agents with their solubility in olive oil. Because a wide variety of structurally unrelated anaesthetics obey this rule, it has been suggested that all anaesthetics act at the same molecular site. Pressure reversal of volatile general anaesthetic agents is thought to be entirely consistent with the Meyer-Overton observation. Number: 652 With regard to nitrous oxide: 1. It does not support combustion. 2. Its critical temperature is 36.5 centigrade. 3. It has a Minimum Alveolar Concentration (MAC) value of about 180%. 4. Its boiling point is -88.5 centigrade. A: 1,2,3 Correct B: 1,3 Correct C: 2,4 Correct D: 4 Correct E: All Correct ABCDE Correct Answer: C Nitrous oxide supports combustion, but in itself is neither flammable nor explosive. The critical temperature is defined as the temperature above which a substance cannot be liquefied by compression. Therefore, a substance is a gas when at a temperature above its critical value. Molecules are able to leave the surface of a liquid by evaporation at temperatures below the critical value and thus constitute a vapour above the liquid phase - that is, a vapour is the gaseous phase above the liquid phase at a temperature below the critical value. The MAC value of nitrous oxide is about 104% - See: Russell GB, Snider MT, Richard RB, Loomis JL. Hyperbaric nitrous oxide as a sole anesthetic agent in humans. Anesth Analg. 1990 Mar;70(3):289-95. Number: 683 The graph below expresses the relationship between the ratio of alveolar to inspired fractional concentration of several volatile agents plotted against time. Assume that inspired concentration, alveolar ventilation and cardiac ouput (CO) remain constant. Which of the following statements are correct? 1. Agent 'A' is more soluble than agent 'C'. 2. Agent 'A' is more potent than agent 'C'. 3. Agent 'A' has a higher blood:gas partition coefficient than agent 'C'.

484

4. The rate of induction with agent 'A' will be less affected by a change in CO than agent 'C'. A: 1,2,3 Correct B: 1,3 Correct C: 2,4 Correct D: 4 Correct E: All Correct

ABCDE Correct Answer: D Readers are referred to the classic work by Edmond Eger entitled "Anesthetic Uptake and Action." (Baltimore: Williams and Wilkins, 1974). Blood:gas solubility determines the rate of anaesthetic induction, the ease with which maintenance concentrations are changed, and the speed of recovery. Inhalational anesthetic agents with a high blood/gas partition coefficient, such as methoxyflurane, are highly soluble in blood and, thus, are removed in large quantities from the lungs by the pulmonary circulation. As a result Fa/Fi rises more slowly. Thus in this case, curve 'A' must represent an agent which has a lower blood:gas partition coefficient and therefore is less soluble than the agent represented in curve 'C'.

485

Strictly speaking, it is not possible to make any deductions about the relative potencies of agents 'A' and 'C' from the data supplied, although it can be said that as oil:gas solubility parallels anaesthetic potency it is highly likely that 'A' is less potent than 'C'. Cardiac output influences uptake by carrying away more or less inhalational anaesthetic from the lungs. The greater the cardiac output, the greater the uptake of anaesthetic agent from the alveoli by the blood and thus the slower the rate of rise of alveolar concentration. When the cardiac output is reduced, less anaesthetic is removed from the alveoli, and therefore the alveolar concentration approaches the inspired concentration more rapidly. This effect is greater with the more soluble agents. Number: 691 The graph below expresses the relationship between the ratio of alveolar to inspired fractional concentration of two volatile agents plotted against time. The curves labelled 'A' and 'B' represent the same agent, obtained under different patient and/or circuit conditions and the curve labelled 'C' represents a different agent. What is the most likely explanation for the difference between these curves? A. In case 'A' alveolar ventilation is less than in case 'B'. B. In case 'A' cardiac output is less than in case 'B'. C. In case 'A' inspired concentration is greater than in case 'B'. D. In case 'A' inspired concentration is less than in case 'B'. E. Curve 'A' was obtained under hyperbaric conditions. Select the single best answer

486

ABCDE Correct Answer: B Readers are referred to the classic work by Edmond Eger entitled "Anesthetic Uptake and Action." (Baltimore: Williams and Wilkins, 1974). Cardiac output influences uptake by carrying away more or less inhalational anaesthetic from the lungs. The greater the cardiac output, the greater the uptake of anaesthetic agent from the alveoli by the blood and thus the slower the rate of rise of alveolar concentration. When the cardiac output is reduced, less anaesthetic is removed from the alveoli, and therefore the alveolar concentration approaches the inspired concentration more rapidly. This effect is greater with the more soluble agents. Similarly, changes in alveolar ventilation deliver greater or lesser amounts of agent to the lungs. Thus curve 'A' is compatible with either a greater alveolar ventilation or lesser cardiac output than is the case in 'B'. Changes in inspired concentration have no significant effect on the Fa/Fi ratio. Number: 695 The impurities which may be produced during the manufacture of nitrous oxide include: 1. Ammonia. 2. Nitric Oxide. 3. Nitrogen dioxide. 4. Nitrogen. A: 1,2,3 Correct B: 1,3 Correct C: 2,4 Correct D: 4 Correct E: All Correct ABCDE Correct Answer: E All of these compounds may occur as contaminants during the production of nitrous oxide. Number: 701 Which of the following induction agents are contraindicated in acute intermittent prophyria (AIP)? 1. Methohexitone. 2. Ketamine. 3. Thiopentone.

487

4. Propofol. A: 1,2,3 Correct B: 1,3 Correct C: 2,4 Correct D: 4 Correct E: All Correct ABCDE Correct Answer: B Acute intermittent porphyria is a autosomal dominant disorder caused by a defect in porphobilinogen deaminase activity. Many cases exist in latent form, but in manifest cases it is more frequently seen in women. The estimated prevalence of the disorder is 5-10 case per 100,000 population. The latent form of the disease may exist indefinitely, but certain drugs, infections, and excessive dieting (starvation) can precipitate attacks. All barbiturates are contraindicated in patients with AIP, whereas ketamine and propofol appear to be safe for use. It is also said that etomidate is contraindicated in AIP although there is at least one case report of its safe use in this condition. (Famewo CE. Induction of anaesthesia with etomidate in a patient with acute intermittent porphyria. Can Anaesth Soc J. 1985 Mar;32(2):171-3.) Number: 742 Which of the following factors will enhance induction with an inhalational agent? 1. A high blood / gas partition coefficient. 2. Low alveolar ventilation. 3. Low saturated vapour pressure (SVP) at room temperature. 4. The second gas effect. A: 1,2,3 Correct B: 1,3 Correct C: 2,4 Correct D: 4 Correct E: All Correct ABCDE Correct Answer: D 1. A low blood / gas partition coefficient is necessary for rapid induction with an inhalational agent as the more 'insoluble' an agent is, the more rapidly the partial pressure of the agent (in the blood phase) rises. 2. Increasing alveolar ventilation increases the rate of induction.

488

3. The SVP at room temperature effectively dictates the maximum concentration of agent that may be administered to a patient. If SVP is low, this may limit the use of 'over pressure' to increase the speed of induction. 4. Rapid uptake of a second gas (eg nitrous oxide) from the alveoli allows the alveolar concentration of a volatile agent to increase more rapidly, thus favouring rapid induction of anaesthesia. Number: 780 (S+) Ketamine in comparison to (R-) Ketamine: 1. Is about 4 times more potent. 2. Is associated with a higher incidence of emergence reactions. 3. Is associated with a more rapid recovery of psychomotor skills. 4. Has a significantly longer elimination half-life. A: 1,2,3 Correct B: 1,3 Correct C: 2,4 Correct D: 4 Correct E: All Correct

ABCDE Correct Answer: B The pharmacokinetic profiles for the individual isomers of ketamine do not differ significantly. (S+) Ketamine is associated with a lower incidence of emergence reactions. See: Br J Anaesth 1985 Feb;57(2):197-203: Comparative pharmacology of the ketamine isomers. Studies in volunteers. White PF, Schuttler J, Shafer A, Stanski DR, Horai Y, Trevor AJ Number: 795 The critical temperature of nitrous oxide is: A. + 36.4 C B. - 90.9 C C. - 88.5 C D. + 34.6 C E. - 12.5 C Select the single best answer

489

ABCDE Correct Answer: A The critical temperature of nitrous oxide is the temperature above which nitrous oxide cannot be liquified by pressure alone. Some physical constants of the gas are: Freezing point: - 90.9 C Boiling point: - 88.5 C (at atmospheric pressure) Critical temperature 309.6 K (+ 36.4 C) Critical pressure 72.4 bar abs Critical volume 0.097 m/kmol Number: 837 Which of the following volatile agents are ethers? 1. Sevoflurane 2. Desflurane 3. Enflurane 4. Halothane A: 1,2,3 Correct B: 1,3 Correct C: 2,4 Correct D: 4 Correct E: All Correct ABCDE Correct Answer: A Volatile inhalational anaesthetic agents can be divided into two groups: halogenated hydrocarbons and halogenated ethers. Examples of halogenated hydrocarbons include halothane, trichloroethylene, and chloroform. An ether is an organic compound which contain the divalent functional group ' O ' . Ethers are denoted as 'ROR' where 'R' may be any alkyl group. The alkyl groups of an ether may be similar or different. The newer halogenated ethers include enflurane, desflurane and isoflurane. Although the older members of the ether family are flammable, the new members of the series are not. Unlike the halogenated hydrocarbons, the ethers do not sensitise the myocardium to circulating catecholamines and tend to be chemically more stable. Number: 844 The effect of ischaemic pre-conditioning on the myocardium is enhanced by: 1. Opiates. 2. Sevoflurane.

490

3. Isoflurane. 4. Ketamine. A: 1,2,3 Correct B: 1,3 Correct C: 2,4 Correct D: 4 Correct E: All Correct ABCDE Correct Answer: A Brief sub-lethal periods of myocardial ischaemia enhance the tolerance of the myocardium to a period of sustained ischaemia. This phenomenon was first described by Murry et al. and is known as "ischaemic preconditioning". Opiates, sevoflurane and isoflurane have all been shown to enhance the efficacy of preconditioning, whereas ketamine has been shown to diminish the effect. See: Murry CE, Jennings RB, Reimer KA. Preconditioning with ischemia: a delay of lethal cell injury in ischemic myocardium. Circulation 1986; 74: 112436. Kersten JR, Schmeling TJ, Pagel PS, et al. Isoflurane mimics ischemic preconditioning via activation of KATP-channels: reduction of myocardial infarct size with an acute memory phase. Anesthesiology 1997; 87: 36170. Novalija E, Stowe DF. Prior preconditioning by ischemia or sevoflurane improves cardiac work per oxygen use in isolated guinea pig hearts after global ischemia. Adv Exp Med Biol 1998; 454: 53342. Miki T, Cohen MV, Downey JM. Opioid receptor contributes to ischemic preconditioning through protein kinase C activation in rabbits. Mol Cell Biochem 1998; 186: 312. Number: 932 Which of the following agents administered at 1 MAC to a healthy, ventilated adult is / are unlikely to impair cerebral autoregulation? 1. Halothane. 2. Desflurane. 3. Ethrane. 4. Sevoflurane. A: 1,2,3 Correct B: 1,3 Correct C: 2,4 Correct D: 4 Correct

491

E: All Correct ABCDE Correct Answer: D It seems to be well-established that halothane, desflurane and ethrane significantly impair cerebral autoregulation when administered at 1 MAC or greater. Sevoflurane, on the other hand does not have this effect. The evidence with regard to the effect of 1 MAC isoflurane is conflicting. References: Miletich DJ, Ivankovich AD, Albrecht RF, Reimann CR, Rosenberg R, McKissic ED. Absence of autoregulation of cerebral blood flow during halothane and enflurane anesthesia.Anesth Analg. 1976 Jan-Feb;55(1):100-9. Larsen R, Maurer I, Khambatta H. The effects of isoflurane and enflurane on cerebral hemodynamics and cerebral oxygen consumption in humans. Anaesthesist. 1988 Mar;37(3):173-81. Gupta S, Heath K, Matta BF. Effect of incremental doses of sevoflurane on cerebral pressure autoregulation in humans. Br J Anaesth. 1997 Oct;79(4):469-72. Bedforth NM, Girling KJ, Skinner HJ, Mahajan RP. Effects of desflurane on cerebral autoregulation. Br J Anaesth. 2001 Aug;87(2):193-7. Summors AC, Gupta AK, Matta BF. Dynamic cerebral autoregulation during sevoflurane anesthesia: a comparison with isoflurane. Anesth Analg. 1999 Feb;88(2):341-5.

OTHER PHYSIOLOGY TESTS Number: 98 Which of the following are generally associated with increases in intra-ocular pressure? A. Normal blinking. B. The head down position. C. Pupillary dilation. D. Hypercarbia. E. All of the above. Select the single best answer ABCDE Correct Answer: E

492

Normal IOP is 12-20 mmHg. The most important influences are movement of aqueous humor, changes in choroidal blood volume, central venous pressure (CVP), and extraocular muscle tone. The main physiological determinant is the balance between production and removal of aqueous humor. Aqueous humor is secreted via a Na/K dependant ATPase pump from the ciliary processes of the posterior chamber (2/3), and passive ultrafiltration through blood vessels on the anterior surface of the iris (1/3) at an overall rate of 2ul/min. It flows between the lens and iris, through the pupil, into the anterior chamber. It is eliminated via passage through the trabeculae of the episcleral venous system (spaces of Fontana) and eventually through the canal of Schlemm, which lies at the iridocorneal angle. The factors determining rate of absorption are the IOP, the resistance to flow through the spaces of Fontana and canal of Schlemm, and the CVP. Increased CVP will decrease absorption of humor into the venous system. This elevation in pressure will be transmitted to the IOP, which will equilibrate, whereupon flow will normalize. The increases in CVP associated with coughing, straining, and the Valsalva manoeuvre can elevate IOP by 30-40 mm Hg. Similar increases can be seen at intubation. Because they are transient, they are generally innocuous in a closed eye. In an open eye, after trauma or during cataract or corneal transplant surgery, however, this may lead to loss of intraocular contents, hemorrhage, or permanent loss of vision. A slight head up tilt during intraocular surgery helps to counteract the effects of CVP. Resistance to outflow will increase if the iridocorneal angle is reduced. This occurs physiologically with relaxation of the sphincter pupillary muscle during mydriasis, or associated with pathological states, as seen in closed angle glaucoma, where swelling of the peripheral iris closes the angle. In open angle glaucoma, sclerosis of the venous trabecular network causes resistance to outflow in the presence of a normal angle. Changes in choroidal blood volume (CBV) also affect IOP significantly. The choroid is a vascular meshwork of arterial anastomoses located in the posterior chamber. Choroidal blood flow is usually autoregulated over the normal range of perfusion pressures to keep IOP stable. Sudden increases in systemic blood pressure may cause the perfusion pressure to rise above its autoregulatory threshold as with cerebral blood flow, resulting in transient swelling of CBV and elevation of IOP. Similarly, hypotension may cause decreases in CBV and IOP. Choroidal blood volume will also increase in response to acidosis, hypercarbia, and hypoxaemia. As with cerebral blood flow, there is a linear relationship between PaCO2 and choroidal blood flow, and IOP. Hypocarbia decreases IOP through a decrease in CBV and formation of aqueous humor secondary to decreased activity of carbonic anhydrase. The control of IOP may be compared with intracranial pressure in general. Recalling the Monroe- Kelly Doctrine, the brain is a poorly compliant organ, encased within the bony vault of the cranium. Total ICP is contributed to by the individual pressure and compliance profiles of the three compartments: blood volume, CSF, and brain. Changes in any one will affect the others, and the pressure response to changing volume in any one is influenced by the capacity of the others to accommodate, ie:their compliances. IOP can be increased by extrinsic compression of the globe. The force of the eyelid during a normal blink may cause an increase of 10 mmHg; a forceful squeeze can increase IOP to over 50 mmHg. A poorly positioned facemask could increase IOP to the point of zero blood flow.

493

ROGERS,M.C ET AL (EDS); Principles and Practice of Anesthesiology, Mosby, 1993, pp 2243-4. MILLER,R. (ED); Anesthesiology, Churchill-Livingstone, 1991, pp 2004-5.

Number: 131 Which intra-abdominal pressure (IAP) will result in oliguria? A. 5 mm Hg. B. 15 mm Hg. C. 25 mm Hg. D. 35 mm Hg. E. None of the above- there is no direct relationship . Select the single best answer ABCDE Correct Answer: C The adverse relationship between elevated IAP and renal function is firmly established. Oliguria occurs predictably as IAP rises above 25 mm Hg, and is related to direct renal parenchymal compression associated with raised renal vascular resistance and reduced renal blood flow (RBF). It is not caused by ureteric obstruction or renal vein occlusion. Elevated IAP reduces cardiac output and RBF, and filtration fraction falls in parallel. The fall in urine output occurs more readily in the presence of hypovolemia, however, RBF remains depressed with elevated IAP despite volume loading. Urine output is deceased to a greater extent than the decrease in RBF. Other effects resulting from elevated IAP include: (1) Venous return and cardiac output- This depends upon the magnitude of the IAP. Increasing IAP to up to 10 mm Hg causes an augmentation of venous return, probably by reducing the blood volume sequestered in the splanchnic vessels. This increases cardiac output and arterial pressure. This response is absent in the presence of hypovolaemia. When IAP exceeds 20 mm Hg, the inferior vena is compressed. Venous return from the lower half of the body is impeded resulting in a fall in cardiac output. Systemic vascular resistance is also increased and this further impairs ventricular function and cardiac output. The sequelae of IAP varies between different patients and procedures.The above would be typical of events during gynaecological laparoscopy. During cholecystectomy, cardiac output could be expected to fall above 15 mm Hg, probably due to differences in surgical positioning and cardiac status of the patients. (2)Pulmonary compliance-Pneumoperitoneum causes cephalad displacement of the diaphragm, resulting in the reduction in lung volumes including functional residual capacity. Pulmonary compliance is reduced, and airway resistance is increased. This is manifest as increasing airway pressures for any given tidal volume with an increased risk of haemodynamic changes and barotrauma during positive pressure ventilation. Airway closure may occur, and ventilation-perfusion mismatch may result in hypoxia.

494

(3)Regurgitation of gastric contents and pulmonary aspiration are more likely. ULYATT, D.B; " Elevated Intra-abdominal Pressure ", Australian Anaesthesia 1992, pp 10813. CHUI, P.T. GIN,T. & OH, T.E; "Anaesthesia for Laparoscopic General Surgery ", Anaesth Intens Care, 21, 1993, pp 163-71 (REVIEW). Number: 132 The elimination half life of carbon monoxide when breathing 100% oxygen at 2.5 ATM atmospheric pressure is: A. 250 minutes. B. 180 minutes. C. 120 minutes. D. 75 minutes. E. 20 minutes. Select the single best answer ABCDE Correct Answer: E Carboxyhaemoglobin readily dissociates when the partial pressure of carbon monoxide in alveolar gas falls below that of mixed venous: the elimination half life is reduced from 250 minutes when breathing air at sea level, to 75 minutes when breathing 100% O2 at sea level. When breathing 100% O2 at 2.5 ATM, it is further reduced to 22 minutes. See: Meredith T, Vale A. Carbon monoxide poisoning. Br Med J (Clin Res Ed). 1988 Jan 9;296(6615):77-9. Number: 261 Bile acids: 1. Are about 50% reabsorbed from the intestinal lumen 2. Are formed in the liver from cholesterol 3, Cannot be metabolised by intestinal bacteria 4. Are conjugated with taurine and glycine before excretion into bile A: 1,2,3 Correct B: 1,3 Correct C: 2,4 Correct D: 4 Correct E: All Correct ABCDE

495

Correct Answer: C The primary bile acids, cholic and chenodeoxycholic acids, are synthesized from cholesterol in the liver, conjugated with glycine or taurine, and excreted into the bile. Secondary bile acids, including deoxycholate and lithocholate, are formed in the colon as bacterial metabolites of the primary bile acids. However, lithocholic acid is much less efficiently absorbed from the colon than deoxycholic acid. Other secondary bile acids, found in trace amounts, which include ursodeoxycholic acid (a stereoisomer of chenodeoxycholate) and a variety of other unusual or "aberrant" bile acids, may be produced in increased amounts in patients with chronic cholestatic syndromes. In normal bile, the ratio of glycine to taurine conjugates is about 3:1, while in patients with cholestasis, increased concentrations of sulfate and glucuronide conjugates of bile acids are often found. Number: 289 Normal bile: 1. Is hypertonic 2. Contains phospholipid 3. Output is 125ml/24hrs 4. Contains unesterified cholesterol A: 1,2,3 Correct B: 1,3 Correct C: 2,4 Correct D: 4 Correct E: All Correct ABCDE Correct Answer: C Major components of bile by weight include water (82 percent), bile acids (12 percent), lecithin and other phospholipids (4 percent), and unesterified cholesterol (0.7 percent). Other constituents include conjugated bilirubin, proteins (IgA, by-products of hormones, and other proteins metabolized in the liver), electrolytes, mucus, and, often, drugs and their metabolic by-products. The total daily basal secretion of hepatic bile is approximately 500 to 600 mL. Number: 442 With regard to the oesophagus: 1. Striated muscle ends at the junction of the upper and middle thirds. 2. The lower end is anchored by the phreno-oesophageal ligament. 3. Stratified squamous epithelium is usually found until the level of the diaphragm. 4. Pressure in the lower oesophagus may reach 500 mmHg.

496

A: 1,2,3 Correct B: 1,3 Correct C: 2,4 Correct D: 4 Correct E: All Correct ABCDE Correct Answer: E The musculature of the pharynx and the upper quarter of the oesophagus is striated muscle, and, therefore, the peristaltic waves in these regions are controlled only by skeletal nerve impulses in the glossopharyngeal and vagus nerves. In the lower two thirds of the oesophagus, the musculature is smooth, but this portion is also strongly controlled by the vagus nerves acting through their connections with the enteric nervous system. Number: 799 Metabolic toxicity of sodium nitroprusside may be caused by accumulation of: 1. Cyanide radicals. 2. Cyanomethaemoglobin. 3. Thiocyanate radicals. 4. Ferricyanide radicals. A: 1,2,3 Correct B: 1,3 Correct C: 2,4 Correct D: 4 Correct E: All Correct ABCDE Correct Answer: B Metabolic toxicity of nitroprusside occurs because the drug contains five cyanide groups and breaks down in the body to yield cyanide ions which, when reacted with sulphur, yield thiocyanate. Toxicity may occur from excessive accumulation of either cyanide or thiocyanate. During high infusion rates of nitroprusside, cyanide toxicity may occur. This appears to be a particular problem when doses greater than 10 mcg/kg/min (0.6 mg/kg/hour) are used. Cyanide ion combines with cytochrome C, an enzyme required for aerobic metabolism. The impairment of aerobic metabolism results in a shift to anaerobic metabolism which is manifested as metabolic acidosis, with elevated plasma lactate concentrations.

497

Nonspecific symptoms of thiocyanate toxicity include fatigue, tinnitus, nausea and vomiting. Clinical signs of thiocyanate neurotoxicity include hyperreflexia, confusion, psychosis and miosis. Toxicity may progress to seizures and coma when thiocyanate concentrations exceed 60 mg/L. Life-threatening thiocyanate toxicity is of concern when blood thiocyanate concentrations approach 200 mg/mL. Thiocyanate is renally excreted and has a half-life of 4 to 7 days in patients with normal renal function and even longer in patients with impaired renal function. Nonspecific symptoms of thiocyanate toxicity include fatigue, tinnitus, nausea and vomiting. Clinical signs of thiocyanate neurotoxicity include hyperreflexia, confusion, psychosis and miosis. Toxicity may progress to seizures and coma when thiocyanate concentrations exceed 60 mg/L. Lifethreatening thiocyanate toxicity is of concern when blood thiocyanate concentrations approach 200 mg/mL. Normal adult methaemoglobin concentrations in blood (approximately 0.5% of all hemoglobin species) are capable of binding the cyanide released from 18 mg of SNP. Cyanomethaemoglobin remains in a dynamic equilibrium with free cyanide and is considered nontoxic. See: Clin Pharmacokinet 1984 May-Jun;9(3):239-51: Clinical pharmacokinetics of nitroprusside, cyanide, thiosulphate and thiocyanate. Schulz V Number: 855 The principal factor controlling the day-to-day production of albumin in a healthy adult is: A. Thyroid hormone. B. Growth hormone. C. Colloid osmotic pressure. D. Cortisol. E. Insulin. Select the single best answer ABCDE Correct Answer: C According to the comprehensive review by Margarson and Soni: "Synthesis occurs in the polysomes bound to the endoplasmic reticulum of hepatocytes at a rate of 912g.day-1 in a healthy adult. Only 2030% of hepatocytes seem to produce albumin at any one time; it is not stored in the liver and there is, therefore, no reserve for release on demand. In states of maximal stimulus the synthesis of albumin can only be increased two to three fold. The primary factor controlling the rate of production is a change in the colloid osmotic pressure and the osmolality of the extravascular liver space. Synthesis is also stimulated by raised concentrations of insulin, thyroxine and cortisol. Growth hormone, despite its effects on reducing total urinary nitrogen loss, has no measurable effect on albumin synthesis in patients."

498

The research supporting these statements appears to have been largely performed on assorted rodents! However, there is evidence in humans that growth hormone is not useful for promoting albumin synthesis. (See: Barle H, Gamrin L, Essen P, McNurlan MA, Garlick PJ, Wernerman J. Growth hormone does not affect albumin synthesis in the critically ill. Intensive Care Med. 2001 May;27(5):836-43.) Number: 990 The highest concentration (mg/ml) of fibrinogen is found in: A. Fresh frozen plasma. B. Cryoprecipitate. C. Cryosupernate. D. Platelet concentrate. E. Salt-poor albumen. Select the single best answer ABCDE Correct Answer: B The highest concentration (mg/ml) of fibrinogen is found in cryoprecipitate (Each 10 to 15 ml unit contains about 20% of the fibrinogen present in one 200 to 250 mI unit of fresh frozen plasma or one 400 to 450 mI unit of whole blood.) See: Rossi's Principles of Transfusion Medicine. Eds: Simon, Dzik, Snyder, Stowell and Strauss. ISBN: 0-7817-3024-4

REGIONAL AND SPECIALTY TESTS Number: 13 When performing a femoral nerve block, the needle entry point which is most likely to result in contact with the femoral nerve is one performed at the level of: A. The inguinal ligament immediately adjacent to the lateral border of the femoral artery. B. The inguinal ligament immediately adjacent to the medial border of the femoral artery. C. The inguinal crease immediately adjacent to the lateral border of the femoral artery. D. The inguinal crease 2 cm lateral to the femoral artery. E. The inguinal crease immediately adjacent to the medial border of the femoral artery. Select the single best answer ABCDE

499

Correct Answer: C The femoral nerve is LATERAL to the artery!! The site for needle insertion in femoral nerve block varies significantly among various descriptions of the technique. Vloka et al performed a cadaveric study designed to establish the needle entry point which was most likely to result in contact with the femoral nerve. They found that insertion of the needle at the level of the inguinal crease, next to the lateral border of the femoral artery resulted in the highest frequency of needle-femoral nerve contacts (71%). Of note, the femoral nerve was significantly wider (14.0 vs 9.8 mm) and closer to the fascia lata (6.8 vs 26.4 mm) at the inguinal crease than at the inguinal ligament level. They concluded that needle insertion at the inguinal crease level immediately adjacent to the femoral artery produced the highest rate of needle-femoral nerve contacts. The main factors influencing this result include the greater width of the femoral nerve and the more predictable femoral artery-femoral nerve relationship at the inguinal crease level, compared with the inguinal ligament level. See: Vloka JD, Hadzic A, Drobnik L, Ernest A, Reiss W, Thys DM. Anatomical landmarks for femoral nerve block: a comparison of four needle insertion sites. Anesth Analg. 1999 Dec;89(6):1467-70. Number: 19 A tourniquet applied to the limb of a conscious, unsedated patient will typically become intolerable after: A. 10 minutes. B. 30 minutes. C. 45 minutes. D. 60 minutes. E. 90 minutes. Select the single best answer ABCDE Correct Answer: B The topic of arterial tourniquets has been recently reviewed by Kam, Kavanaugh and Yoong. See: Kam PC, Kavanaugh R, Yoong FF. The arterial tourniquet: pathophysiological consequences and anaesthetic implications.Anaesthesia. 2001 Jun;56(6):534-45. And: Hagenouw RRPM, Bridenbaugh PO, Van Egmond J, Stuebing R. Tourniquet pain: volunteer study. Anesthesia and Analgesia 1986; 65: 117580.

500

Number: 24 A patient scheduled for minor urological surgery is given intrathecal pethidine in the sitting position. 1. The effects of the pethidine will be completely reversed by naloxone. 2. The appropriate dose range is 2 mg/kg. 3. The duration of the sensory block will be in the range 30-60 minutes. 4. The drug will behave as a hyperbaric solution. A: 1,2,3 Correct B: 1,3 Correct C: 2,4 Correct D: 4 Correct E: All Correct ABCDE Correct Answer: D 1. Naloxone reversal: According to Kee:"A number of in vitro studies have confirmed the ability of phenylpiperidine opioids to block conduction in peripheral nerve. Gissen et al found that fentanyl and sufentanil in high concentrations decreased the amplitude of action potentials after nerve stimulation. This effect was not prevented by pretreatment with naloxone and is evidence that the effect of the phenylpiperidine derivatives on nerve conduction is independent of opioid receptors." 2. The appropriate dose range is 0.5 - 1.0 mg/kg. 3. Duration of the sensory block: When used in the dose range 0.5 - 1.0 mg/kg, the duration of the sensory block is in the range 60-120 minutes. 4. Baricity: According to Kee: "Clinical reports of spinal anaesthesia using pethidine have used commercially available preparations which have a concentration of 50 mg/ml and are preservative free. This solution has a specific gravity of 1.009 which is close to the upper limit of specific gravity of cerebrospinal fluid (CSF) and when injected intrathecally, pethidine has the properties of a hyperbaric agent." See: Ngan Kee WD. Intrathecal pethidine: pharmacology and clinical applications. Anaesth Intensive Care. 1998 Apr;26(2):137-46. Mircea N, Constantinescu C, Jianu E et al Subarachnoid anesthesia with pethidine. Ann Fr Anesth Reanim. 1982;1(2):167-71. Number: 67 In the case of a brachial plexus block using Bupivacaine:

501

A. Paraesthesia extending beyond 24 hours is most likely to represent neuropraxia. B. The addition of adrenaline does not increase the duration of the block. C. Accidental IV injection is safe as long as the plasma concentration does not exceed 10 ug/ml. D. There is significant separation of sensory and motor blockade. E. All of the above. Select the single best answer ABCDE Correct Answer: A The average duration of Bupivacaine anaesthesia is 3-10 hours. Its longest duration of action occurs in major peripheral nerve blocks. Paraesthesia may last 5-16 hours with extension up to 24 hours with the addition of adrenaline, however, this may represent neuropraxia. The vascular absorption is influenced to a variable extent by adrenaline but less so than for Lignocaine. Recent studies have demonstrated a prolonged duration of action, however. The plasma concentration associated with cardiotoxicity is generally quoted as 10 ug/ml, however, the relationship between malignant arrhythmias and plasma concentration shows greater interindividual variation than Lignocaine; parturients are especially vulnerable and animal studies have shown that VT occurs at lower plasma concentrations when the rate of rise is rapid; ie accidental intravascular injection. Whilst separation of sensory anaesthesia and motor blockade is an accepted property of Bupivacaine, this effect is obscured by doses generally used to obtain surgical anaesthesia. References: (1)COUSINS,M & BRIDENBAUGH; Neural Blockade, p 138 (2) " " pp 121-8 Number: 68 Which of the following is true of post-dural puncture headache? A. 90% commence within 18 hours of dural puncture. B. The mechanism is due to traction on nerves supplying the meninges. C. The majority cause severe pain that is referred to the frontal and occipital areas. D. The technique of blood patch is 90% successful. E. All of the above. Select the single best answer ABCDE Correct Answer: E Post-dural puncture headache may occur as a result of a planned dural puncture associated with a subarachnoid block or it may be an unplanned complication of an epidural procedure. 90% commence within the first 3 days and resolve spontaneously in a further 2-3 days. The mechanism is due to decreased pressure in the neuraxis caused by leakage of CSF through the

502

puncture site. More specifically, it is thought to be due to traction on trigeminal, glossopharyngeal and vagal innervations of the meninges. Those associated with epidural procedures are generally more severe due to the bigger gauge needles. Overall however, 50% are mild and do not interfere greatly with normal activities; 35% require the patient to assume a recumbent position for relief, and 15% are so severe that the patient cannot sit up. The pain is usually referred to the frontal and occipital regions however diffuse neck pain may occur which is mediated by upper cervical nerves. Epidural blood patches have a 90% success rate.

Number: 69 Which of the following is true of tourniquet pain experienced during spinal anaesthesia? A. It is often associated with a " patchy " block. B. It is less likely to occur with blockade at a high level. C. It occurs more frequently with bupivacaine. D. Intravenous opioids have been shown to be of little effect. E. All of the above. Select the single best answer ABCDE Correct Answer: D Tourniquet pain is an ill defined pain occurring 45-60 minutes after inflation of a tourniquet. It is frequently described as a dull, ill-defined ache, the intensity of which increases until it becomes unbearable. It has been reported in a large number of patients under regional anaesthesia despite an otherwise adequate block. Several theories have been postulated in an attempt to explain tourniquet pain. (1) Transmitted by nerve fibres larger than those carrying types of pain, running along the classical anatomic segmental distribution. This may be due to ischaemia or compression of the sciatic nerve of sufficient intensity to " penetrate " the spinal block. Alternatively, it may involve activation of nociceptive fibres larger than those transmitting other types of pain. Whilst initially adequate, the concentration of local anaesthetic rapidly falls until it is not sufficient to block the stimulus. The proponents of this theory were able to decrease the incidence of tourniquet pain by increasing the dose of tetracaine used for the block. (2) Transmitted by small fibres (A and c). These unmyelinated fibres would travel along sympathetic trunks before entering the spinal cord at a level higher than the sensory block level. However, the incidence does not appear to be related to the sensory level of anaesthesia. Tourniquet pain has been reported in the presence of thoracic and cervical levels of sensory anaesthesia in both spinals and epidurals. (3) Differential nerve fibres. The nature of tourniquet pain seems consistent with pain arising from "c" type nerves. Some authors suggest that these are more resistant to blockade by local anaesthetic agents than A delta fibres. Again, an initially adequate block may become

503

inadequate as the concentration of local anaesthetic falls below that required to block "c" fibres. This is supported by the greater differential activity of tetracaine as compared with bupivacaine and the greater incidence of tourniquet pain with tetracaine than bupivacaine which has been described many times. (4) Frequency dependent block. Differences exist between the local anaesthetic agents with regard to frequency dependent block. For example, the potency of bupivacaine is markedly enhanced by an increase in the rate of nerve stimulation. This could explain the reduced frequency of tourniquet pain with bupivacaine. The constant pressure of the tourniquet may result in an increased rate of firing nociceptive fibres. Once tourniquet pain develops, it is necessary to treat it early. The use of intravenous narcotics is disappointing. The induction of general anaesthesia is often necessary. The only efficacious treatment is deflation of the tourniquet. It is now accepted that the addition of narcotics to local anaesthetics to subarachnoid or epidural blockade improves the quality of anaesthesia. In one study, the incidence of tourniquet pain was reduced from 30% to 8% with the addition of 200ug of Fentanyl to the epidural injection. The topic of arterial tourniquets has been recently reviewed by Kam, Kavanaugh and Yoong. See: Kam PC, Kavanaugh R, Yoong FF. The arterial tourniquet: pathophysiological consequences and anaesthetic implications.Anaesthesia. 2001 Jun;56(6):534-45. Number: 71 When positioning endobronchial tubes: A. Auscultation is a reliable sign of correct positioning of the tube. B. The incidence of mal-positioning is far higher with right sided tubes. C. At least 60% of either right or left sided tubes can be shown to be mal-positioned by bronchoscopy. D. Insertion too far is the commonest category of mal-position. E. The left upper lobe bronchus is unlikely to be obstructed by a left sided tube. Select the single best answer ABCDE Correct Answer: C The incidence of mal-position as judged by fibreoptic bronchoscopy is at least 70% for both right and left sided tubes. Auscultation has been shown to be an unreliable sign of correct positioning and it is arguable that all patients should be bronchoscoped after tube placement. The left upper lobe bronchus is not immune to obstruction - particularly if PVC (as opposed to red rubber) tubes are used. References: Alliaume B, Coddens J, Deloof T. Reliability of auscultation in positioning of double-lumen endobronchial tubes. Can J Anaesth. 1992 Sep;39(7):687-90.

504

Klein U, Karzai W, Bloos F et al. Role of fiberoptic bronchoscopy in conjunction with the use of double-lumen tubes for thoracic anesthesia: a prospective study. Anesthesiology. 1998 Feb;88(2):346-50. Cheong KF, Koh KF. Placement of left-sided double-lumen endobronchial tubes: comparison of clinical and fibreoptic-guided placement. Br J Anaesth. 1999 Jun;82(6):920-1. Number: 72 Oxygenation during one lung ventilation (OLV) may be improved by: 1. Increasing tidal volume (TV) to 15mls/kg. 2. Insufflating oxygen to the unventilated (non-dependant) lung (NDL). 3. Applying PEEP (5cmH2O) to the ventilated (dependant) lung (DL). 4. Maintaining CPAP (5cm H2O) in the unventilated lung. A: 1,2,3 Correct B: 1,3 Correct C: 2,4 Correct D: 4 Correct E: All Correct ABCDE Correct Answer: E The basic principles for the initial management of the patient on OLV include: (1) maintain two lung ventilation for as long as possible. (2 ) use 100% O2. (3) start OLV with large tidal volumes ~ 10-12 ml/Kg. (4) adjust respiratory rate to keep ETCO2 approximately 40 mmHg. Hypocarbia should be avoided because it can directly dilate the pulmonary vessels in the NDL, thus interfering with hypoxic pulmonary vasoconstriction. (5 ) continuously monitor oxygenation using pulse oximetry. (6) continuously monitor ventilation by listening to breath sounds and monitoring ETCO2. (7) produce pulmonary vasodilation in the DL, facilitating diversion of blood away from the NDL. (8) absorption atelectasis caused by a high FIO2 can be decreased by using large TVs and a low level of PEEP to the DL.The use of low concentrations of N2 (20% with 80% O2) has been suggested as N2 splints the alveoli in areas of low V/Q. (9) Adjust TV to patient response, airway pressures, ETCO2 and oxygenation. Excessively high TVs to the DL can cause high airway pressures, increased PVR, and diversion of blood back to the NDL. An excessively low TV is associated with decreases in FRC, airway closure, and atelectasis. Further adjustments include:

505

(10) Dependant lung PEEP.This results in recruitment of closed airways, increased FRC, and increased compliance. The disadvantage would be an increase in PVR in that lung. Thus it may have a variable effect particularly if the NDL is totally collapsed and non-ventilated. (11) CPAP to the NDL.The application of CPAP to the non-ventilated, NDL is the most important measure to improve oxygenation during OLV and should be the first step to improve oxygenation on a background of (1)-(9). Even with a maximal HPV response, 20% of the CO still flows through the NDL. With the use of inhalational agents this may be increased to 24% with < 1 MAC of isoflurane. Application of CPAP leads to oxygenation of that portion of the CO and a significant improvement in PaO2. No significant haemodynamic effects are seen with 5-10cmH2O of CPAP, and it does not interfere with surgical exposure. (12) Optimize venous return. A low CO state is associated with a decreased PvO2 and this may magnify intrapulmonary shunting (13) If these measures fail it is suggested that CPAP to the NDL be increased as this may redirect blood back to the DL secondary to intraalveolar collapse of blood vessels. References ROGERS,M.C ET AL (EDS); Principles and Practice of Anesthesiology, Mosby, 1993, pp 1779-85. Number: 73 Mortality associated with surgery for a fractured hips is increased: 1. In males as opposed to females. 2. With increasing age 3. In trochanteric as opposed to neck fractures. 4. When general as opposed to regional anaesthesia is used for surgery. A: 1,2,3 Correct B: 1,3 Correct C: 2,4 Correct D: 4 Correct E: All Correct ABCDE Correct Answer: B Mortality associated with fractured hips is related to the elderly patient population, the presence of multisystem disease and the nature of surgery. Age itself has not been shown to be an independent predictor of mortality. Many studies show that perioperative morbidity and mortality is higher in older patients, however this has also correlated with other preexisting risk factors. For example, age >65, poor medical condition, and emergency surgery appear in Goldman's cardiac index score. Operation specific data are limited but exist for hip fracture, prostatectomy and CABG. For patients >65, mortality is 4.3%, 3.5% and 5.2% within 3 months respectively. In another series, hip, pelvis and gastrointestinal operations accounted for 50% of procedures performed

506

on patients >90 years of age. The mortality was 11% and 29% respectively. The high incidence of hip fracture in the elderly make it of interest to the anaesthetist. The mean age of patients sustaining fractured hips is 75-80. 70-80% are female. Preexisting medical problems are common; approximately 50% are designated ASA class III, 10% class IV. 50% have a history of ischaemic heart disease and 20% have hypertension. Significant arrhythmias occur in 40% and may contribute to the fall. Several studies have been carried out to compare the mortality following surgical correction between general and spinal anaesthesia. In general, mortality within the first two postoperative months is lower following spinal anaesthesia. After this time the two equalize. The high short term mortality associated with fractured hip is related to age as described above, sex (male > female), and site of fracture( trochanteric > femoral neck) Hypoxaemia plays an important role in the mortality from fractured hips and whilst it has been demonstrated globally, it has been found to be more severe and persist longer after general anaesthesia. In one study, 23% of patients had a PaO2 < 60mmHg. In general, PaO2 is less than predicted than for age alone. Amongst other factors, fat embolism appears to be significant. References COVERT,C;" 'Anaesthesia for hip surgery in the elderly ", Can J Anaesth, vol 36, no 3, pp 311-9, 1989. ROGERS,M.C. ET AL (EDS); Principles and Practice of Anesthesiology, Mosby, 1993, pp 2205-6. Number: 74 The absorption of large volumes of isotonic glycine during a TURP may cause: 1. Hypotension. 2. Haemolysis. 3. Hypokalemia. 4. Hypertension. A: 1,2,3 Correct B: 1,3 Correct C: 2,4 Correct D: 4 Correct E: All Correct ABCDE Correct Answer: E The major complications of TURP surgery may be classified as: 1. Haemorrhage. 2. Irrigation solution related - overhydration/hyponatremia/haemolysis/glycine toxicity.

507

3. Sepsis. 4. Accidental hypothermia. 5. Surgical complications. Complications related to the use of glycine as an irrigant may be secondary to absorption of large volumes of hypotonic solution and direct effects of glycine: (1)WATER INTOXICATION The syndrome of overhydration has been well described. It consists of bradycardia, elevated systolic and diastolic blood pressures with an increased pulse pressure; cerebral agitation and depression. Cerebral signs may be variable but commence with restlessness, agitation, and confusion, graduating to progressive obtundation, stupor and coma. Muscle cramps, twitching and focal or generalized convulsions may occur. Later, dyspnoea, cyanosis, refractory hypotension and cardiac arrest may follow. In less florid cases, hypothermia, weight gain and postoperative diuresis may be seen. The absorption of large volumes of electrolyte free solutions result in increased intravascular volume, haemodilution, and hyponatremia, leading to the development of left heart failure, pulmonary and cerebral oedema, and ultimately cardiovascular collapse. It has been suggested that the more rapid the fall in serum sodium, the more likely are toxic symptoms to develop. A drop of 20-30 mmol/l or an absolute value of 120 mmol/l indicates the likelihood that the reaction will be severe. Serum osmolality probably changes less often than sodium concentration, and if significant changes are found, serious sequelae are more likely. Hyponatremia per se may produce ECG signs such as widened QRS complexes and Tinversion, later progressing to VT and standstill. (2)GLYCINE TOXICITY Recently, authors have suggested that major changes in serum sodium and glycine occur independently, and that each anomaly may affect different organ systems. The overhydration syndrome has been ascribed to a combination of dilutional hyponatremia due to intravascular absorption of the solution, to water intoxication resulting from absorption of the irrigant into the abdominal cavity, and a direct neurotoxic effect of the glycine, functioning as an inhibitory neurotransmitter - whether or not gross aberrations in serum sodium or osmolality have occurred. Factors affecting the volume of irrigant absorbed and guidelines for prevention of complications: (1) Duration of resection < 40 min (2) Expertise of surgeon - avoidance of extracapsular vessels (3) Hydrostatic pressure < 45 cm, and avoidance of overfilling the bladder. (4) A commonly accepted value for rate of irrigation is 20 ml/min. References HATCH,P; " Surgical and anaesthetic considerations in transurethral resection of the prostate ", Anaesth Intens Care, vol 15, p 203-11, 1987. (review) Number: 75 In which of the following situations associated with cardiac arrest is the use of sodium bicarbonate acceptable.

508

1. Hyperkalemia. 2. Tricyclic antidepressant overdose. 3. Pre-existing metabolic acidosis. 4. Hypoxic lactic acidosis subsequent to prolonged period of circulatory arrest. A: 1,2,3 Correct B: 1,3 Correct C: 2,4 Correct D: 4 Correct E: All Correct ABCDE Correct Answer: A Revisions to the American Heart Association guidelines have followed recommendations of the 1992 Conference on Cardiopulmonary Resuscitation and Emergency Cardiac Care. The system used to classify recommendations for treatment is as follows: Class 1- Treatment that is usually indicated, always acceptable, and considered useful and effective. Class 2- A treatment that is acceptable, but of uncertain efficacy and may be controversial. Class 2a- A treatment with the weight of evidence in its favour of its usefulness and efficacy. Class 2b-A treatment not well supported by evidence, but may be helpful, and is probably not harmful. Class 3-An inappropriate treatment, without scientific evidence of efficacy which may be harmful. Sodium bicarbonate is given a class 3 status for the most common problem for which its use was thought to be indicated during a cardiac arrest, that is, hypoxic lactic acidosis. However, it is given a class 1 ranking for the treatment of hyperkalemia, class 2a for treatment of preexisting bicarbonate responsive metabolic acidosis, and class 2b for the management of acidosis after the return of spontaneous circulation when there has been a prolonged cardiac arrest. Adequate alveolar ventilation is seen as the mainstay of treatment of acidosis during cardiac arrest. References DODD, G.J." New guidelines for cardiopulmonary resuscitation and emergency cardiac care ", Intensive Care World ,vol 9, no 4, Dec. 1992, pp 169-9. Number: 76 Which of the following agents are considered to have an antifibrillatory role in the treatment of VF:

509

1. Lignocaine. 2. Adrenaline. 3. Bretylium. 4. Sodium bicarbonate. A: 1,2,3 Correct B: 1,3 Correct C: 2,4 Correct D: 4 Correct E: All Correct ABCDE Correct Answer: B Antifibrillatory agents increase the threshold to fibrillation in ventricular muscle. They do not defibrillate, but increase the likelihood of successful electrical defibrillation and decrease the likelihood of degeneration back into a malignant dysrhythmia following successful defibrillation. They are generally recommended after initial attempts at defibrillation have failed. Adrenaline is not an antifibrillatory agent. Its role in VF is that of a vasopressor where it has been shown to improve cerebral perfusion. Specific guidelines now exist for the use of sodium bicarbonate during cardiac arrest. It is not an antifibrillatory agent. References DODD, G.J." New guidelines for cardiopulmonary resuscitation and emergency cardiac care ", Intensive Care World ,vol 9, no 4, Dec. 1992, pp 169-9. Number: 77 Cerebral perfusion during cardiopulmonary massage is best preserved with: A. Open cardiac massage. B. Simultaneous compression and ventilation. C. Increased compression rate from 60-80. D. Trendelenberg posturing. E. Compression ratio < 0.4. Select the single best answer ABCDE Correct Answer: A Open cardiac massage is a special technique that may provide near normal perfusion of the brain and heart during circulatory arrest. Coronary and cerebral perfusion during chest compression following circulatory arrest are determined by aortic diastolic and mean arterial blood pressure respectively. Properly performed chest compression can produce systolic arterial pressure peaks of 60-80 mmHg, but diastolic blood pressure is low. Mean pressure in

510

the carotid artery seldom exceeds 40 mmHg. Cardiac output is likely to be only 1/4 normal even when performed optimally. Vascular pressure can be maximized by extending the length of compressions to 50% of the cycle and increasing the rate to 80-100 / minute. Simultaneous compression-ventilation in animals has been shown to increase carotid blood flow. This is consistent with the " thoracic pump " theory of blood flow during chest compression which postulates that increasing intrathoracic pressure generates an extrathoracic arterio-venous pressure gradient which generates blood flow. This differs from the original " cardiac pump " theory which postulates that blood is squeezed from the cardiac chambers as they are compressed between the sternum and vertebral spine. Experimental studies have shown that direct cardiac massage applied early in the cardiac arrest or after a short period of ineffective closed chest CPR can improve survival from a cardiac arrest. It is not efficacious when applied late (after > 25 minutes of total arrest time). It has no role as a last effort at the end of a failed resuscitation sequence. It requires a multidisciplinary approach, necessitating a thoracotomy. It may be indicated in: (1)Cardiac arrest caused by hypothermia, pulmonary embolism, pericardial tamponade, or intraabdominal hemorrhage. (2)Chest deformity where closed massage is ineffective. (3)Penetrating abdominal trauma with deterioration to cardiac arrest. (4) Blunt trauma with cardiac arrest. References Basic Life Support (recommendations of the AHA guidelines for CPR), JAMA, vol 268, Oct.28th, 1992, pp 2189-90. MILLER,R (ED); Anesthesiology, Churchill- Livingstone, 3rd Ed., 1992, pp 2309.

Number: 107 With regard to an interscalene block: 1. Eliciting paraesthesia is vital to success. 2. The most commonly missed component is the superior trunk. 3. Blockade of the ipsilateral phrenic nerve occurs uncommonly. 4. The vertebral artery may be punctured with a correctly placed needle. A: 1,2,3 Correct B: 1,3 Correct C: 2,4 Correct D: 4 Correct E: All Correct ABCDE Correct Answer: D The brachial plexus sheath can be entered via the interscalene space between the anterior and middle scalene muscles at the level of the sixth cervical vertebra. Identifying the plexus by

511

eliciting paraesthesia is not vital to success as the local anaesthetic solution tracks along the sheath to a more distal level. The most commonly missed segment is the inferior trunk. Blockade of the ipsilateral phrenic nerve occurs commonly and figures of at least 60% are quoted. Puncture of the ipsilateral vertebral artery may well occur with a correctly placed needle due to the proximity of the structure to the area of infiltration. References ATKINSON, R. S. ET AL; Lee's Synopsis of Anaesthesia, 11th Ed., p 646. KNOBLANCHE, G.;" The incidence and aetiology of phrenic nerve blockade associated with supraclavicular brachial plexus block. ", Anaesth Intens Care, vol 7, 1979, p 346. Number: 108 To operate on the ear, one would have to block which of the following nerves: 1. Auriculotemporal. 2. Great auricular nerve. 3. Auricular branch of the vagus. 4. Greater occipital. A: 1,2,3 Correct B: 1,3 Correct C: 2,4 Correct D: 4 Correct E: All Correct ABCDE Correct Answer: A Three nerves provide the sensory innervation to the external part of the ear. These are the auriculotemporal (anterosuperior aspect); greater auricular (inferior aspect); and lesser occipital (a small area superoposteriorly). Part of the external auditory canal and tympanium are innervated by the auricular branch of the vagus nerve. The greater occipital nerve supplies the posterior aspect of the scalp from the trapezius to the vortex. It does not innervate the ear. References SIR ROBERT MACINTOSH; Local Anaesthesia for the Head and Neck. Number: 109 The most appropriate level for a transurethral resection of the prostate (TURP) under spinal anaesthesia is:

512

A. T8. B. T10. C. L4. D. S1. E. None of the above. Select the single best answer ABCDE Correct Answer: B The most appropriate level for spinal anaesthesia for a TURP is T10. The urethra, prostate, and bladder are innervated by both sympathetic and parasympathetic components of the autonomic nervous system. Sympathetic nerves provide motor innervation to the most of the bladder including the internal sphincter and trigone. These travel via the superior hypogastric plexus to enter the spinal cord at segments from T10 to L3. Motor innervation to the sphincter and urethra, and sensory innervation to the urethra, perineum, and glans penis are provided by the parasympathetic, pelvic splanchnic nerves which are generated from sacral segments S2-S4. Afferents from the prostatic capsule enter the spinal cord between T11-L2, as do those of the bladder. Anaesthesia should be achieved to the level of T10, excepting that appreciation of perforation of the prostatic capsule is desired, then the upper level of the block should be reduced to achieve this. HATCH, P.D; " Surgical and Anaesthetic Considerations in Transurethral Resection of the Prostate ", Anesth Intens Care, 15, 1987, pp 203-211.

Number: 112 Nerves commonly left unblocked in an axillary approach to the brachial plexus are: 1. Musculocutaneous. 2. Intercostobrachial. 3. Axillary. 4. Medial brachial radial. A: 1,2,3 Correct B: 1,3 Correct C: 2,4 Correct D: 4 Correct E: All Correct ABCDE

513

Correct Answer: E The intercostobrachial nerve does not originate in the brachial plexus and the medial brachial radial nerve may not lie within that part of the sheath blocked by this approach. These nerves provide sensory innervation to the medial aspect of the arm and intolerance of a pneumatic tourniquet may occur if they are not blocked. This can be achieved by infiltrating 2-5 mls of local anaesthetic agent in the subcutaneous tissue superficial to the brachial artery. The extent of blockade of the proximal plexus (and the nerves leaving at this level) is influenced by the volume of local anaesthetic agent used. When lower volumes (approx. 20 mls) are injected into the axillary sheath without digital pressure, it will not consistently reach the level of the cords of the plexus. In this event, the axillary and musculocutaneous nerves may not be blocked resulting in residual sensation over the lateral aspect of the forearm and arm and power in the forearm flexors. When 40 mls is used the solution will reach the proximal part of the plexus, ensuring complete sensory and motor blockade. WINNIE, A.P.; Plexus Anesthesia, Schultz/Churchill Livingstone, Vol 1, 1983, pp 132-4. Number: 113 Which of the following is / are true of the technique of epidural blood patch in the treatment of a postdural puncture headache? 1. The success rate is maximal when delayed to after the first 24 hours. 2. It should be performed at the interspinous level at which the leak is assumed to occur. 3. Generally 15 -20 mls of blood will be required. 4. Prophylactic blood patches done immediately after the dural puncture is detected are of no benefit to the patient. A: 1,2,3 Correct B: 1,3 Correct C: 2,4 Correct D: 4 Correct E: All Correct ABCDE Correct Answer: B The success is generally of the order of 90% when performed after 24 hours in established cases. Magnetic resonance imaging has elucidated much of the behaviour of the blood patch in recent times. It has been observed that blood clots faster in the presence of cerebrospinal fluid. The clot has a mass effect initially, compressing the dural sac and displacing the conus medullaris and cauda equina. There is also compression or displacement of nerve roots nearby. The main bulk of the clot occupies four or five vertebral levels with thinner spread in cephalad and caudad directions. Some blood enters the subarachnoid space, but within a few hours this resolves to a focal clot within the dural sac adherent to the dura. The mass effect diminishes after about 7 hours, with a thinning layer of blood adherent to the dural sac, but

514

extending much further cephalad than caudad. This confirms the recommendations that an interspace lower be used . Temporary displacement of nerve roots corresponds to radicular pain which can accompany the technique. Additionally, extensive spread into the subcutaneous fat occurs and it is suggested that this may cause backache which sometimes accompanies the procedure. 15 mls of blood is generally required . The success rate is generally considered to be reduced if the patch is performed within the first 24 hours. For instance, in one study prophylactic patching reduced the incidence of postdural puncture headache from 76.5% to 17.6%. This represents a considerable reduction in the incidence of the headache, but is not as spectacular a result as a 90% cure rate which occurs when the procedure is performed late in established cases. Moreover, prophylactic patching exposures a group of patients to the hazards of the procedure who would otherwise not develop a headache (25-30%). For these reasons it is not generally recommemded. References CARRIE, L.E.S.; " Postdural puncture headache and extradural blood patch ", British Journal of Anaesthesia, vol 71, no 2, August 1993, pp179-81. Number: 114 Which of the following are likely to increase the level of blockade after an intrathecal injection of local anaesthetic solution? 1. The bevel directed upwards. 2. Obesity. 3. Isobaric solution (compared with hyperbaric). 4. Barbotage. A: 1,2,3 Correct B: 1,3 Correct C: 2,4 Correct D: 4 Correct E: All Correct ABCDE Correct Answer: A Density, volume, and dose combined with posture and physical characteristics of the patient are the major factors that determine the spread of intrathecal anaesthesia. Consistent results have been obtained with respect to the effects of barbotage and it has been found to have no effect at all on the spread of analgesia. The direction of the needle and the speed of injection have also been studied; it has been shown that a rapid injection (1 ml/sec) with the bevel up produces a significantly higher block than a slow injection (0.2 ml/sec) with the bevel down. Adrenaline will increase the extent of subarachnoid blockade with 0.5%

515

bupivacaine by 1 or 2 segments and the duration by about 1 hour (usually 3-4 hours with 3 mls in an adult). Increasing the volume to 4 mls will generally increase the spread of the block by only 2 segments. The main effect is to increase the duration of the block by about 1 hour. In pregnancy, obesity, short stature and advanced age, less than 3 mls may be adequate due to more extensive spread in these groups, but the spread is unreliable, with lower volumes. Hyperbaric solutions are claimed by some to provide better control of the spread of the block by exploiting postural changes. This is evident in the sitting position when 0.5% bupivacaine is used, however, in the lateral position, this solution spreads the same or even more than isobaric solutions. The volume used ranges from 2-4 mls with the most common being 3 mls. The duration of action of this preparation is only 2-3 hours. Studies on the spread of solution with the patient in the sitting position indicate that the density of the solution has no effect on cephalad spread if the sitting time is less than 2 minutes. However, with longer sitting times, differences appear. For example, after a sitting time of 5 minutes, and the same volume of tetracaine, a block to T10 and L2 will be achieved with isobaric and hyperbaric tetracaine respectively Another study showed in the lateral position, hyperbaric solutions spread 3-5 segments higher on the dependent side. References DEACON, G.; " Factors Influencing the Spread of Spinal Anaesthesia ", Australian Anaesthesia 1990, pp31-34. Number: 115 With respect to performance of a peripheral nerve block with a nerve stimulator, which of the following will increase chances of success? 1. An insulated needle. 2. Placing the earth electrode in close proximity to the injection site. 3. Decreasing the current until a positive twitch is obtained at less than 0.5 mA. 4. Injecting the solution in a series of steps with relocation of position in-between. A: 1,2,3 Correct B: 1,3 Correct C: 2,4 Correct D: 4 Correct E: All Correct ABCDE Correct Answer: B Insulated needles are associated with a higher success rate. With insulated needles, only the tip is left bare, which means that all the current passing through the needle will pass through the tip of the needle. In uninsulated needles, current will pass out the tip and from the sides, making it less accurate. With insulated needles, the tip of the needle is always in close proximity to the nerve at the point at which the current stimulating muscle twitch is minimal.

516

With uninsulated needles, when this point is reached, the tip of the needle has advanced past the nerve. The low inactive output of the machine is connected to the patient electrode. This should be placed some distance away from the injection site as placement too close may result in direct muscle stimulation. During location of the nerve, the needle should be advanced with a 2Hz frequency and constant current output reading. Muscle twitch should be elicited at no more than 2-2.5 mA.When this occurs the current should be decreased. Proximity to the nerve compatible with a successful block should be anticipated as a maximal muscle twitch from a stimulating current of less than 0.5 mA. Note that at higher currents (2.5mA), muscle twitch can be obtained from a needle which is sitting outside the nerve sheath in the case of a brachial plexus block. At this point, the solution is injected as a single bolus, as muscle twitch will disappear immediately as the fluid pushes the nerve away from the needle, and blocks the nerve. Identification of the nerve after this event will be difficult. References MACINTYRE, P; " Are Nerve Stimulators Useful for Regional Anaesthesia? ", Australian Anaesthesia 1990, pp39-41. Number: 116 With respect to intravenous regional anaesthesia: 1. The minimum time recommended before deflation of the cuff is 30 minutes. 2. Exsanguination of the limb provides better analgesia. 3. Onset of anaesthesia typically occurs after about 10 minutes. 4. Leakage of local anaesthetic agent into the venous circulation is more likely with a double cuff . A: 1,2,3 Correct B: 1,3 Correct C: 2,4 Correct D: 4 Correct E: All Correct ABCDE Correct Answer: C The tourniquet should not be released until a minimum of 15 minutes has elapsed, and it should follow a pattern of release/reinflate, with a release interval of not more than 30 seconds. This should occur 2 or 3 times. It is generally felt that exsanguination of the limb is important in achieving adequate analgesia. This should be achieved with an Esmarch bandage, but if this is not available, elevation of the limb for 3 minutes is acceptable.

517

Onset of analgesia is rapid. If it is not adequate within 5 minutes a further bolus may be required. The procedure can usually commence within 10 minutes of injection. Unintentional leakage of local anaesthetic agent has been implicated in many of the serious complications of intravenous regional anaesthesia which have included cardiac arrest and seizures. Investigations have shown that local anaesthetic agent can leak past a correctly inflated tourniquet. Leakage occurs into the venous circulation indicating that venous pressures in the anaesthetized limb may equal or exceed the tourniquet pressure. The effective tourniquet pressure is that transmitted to the underlying arteries and veins. This may be less than the pressure registered on the manometer. The main factor determining this difference is the width of the tourniquet in relation to the size of the limb. For effective pressure to equal the registered pressure, the tourniquet cuff width should be approximately 20% wider than the diameter of the limb. For most adults, this is 12-14cm. Double cuff tourniquets used to minimize tourniquet pain, are each 5-6cm wide. Consequently, the effective pressure may be 80-100 mmHg less than the registered pressure. Recommendations generally advise that the cuff be inflated to a pressure 50 mmHg greater than systolic pressure in the upper limb, however this may allow leakage. References MITCHELL, T.J;" Intravenous Limb Anaesthesia ", Patient Management, Dec., 1990, pp 157. GRACE, S.C. ET AL; " Intravenous Regional Anaesthesia: Evaluation and Prevention of Leakage under the Tourniquet ", Anesthesiology, vol 65, no 3, Sep. 1986, pp 316-20. Number: 118 Which of the following are true of a stellate ganglion block performed for Raynaud's Disease? 1. A Horner's Syndrome always signifies a successful block. 2. The block may be performed bilaterally. 3. It should be repeated if evidence of sympathetic block is not apparent in the arm within 15 minutes. 4. Pain between the scapulae usually signifies placement of the needle too far laterally or posteriorly. A: 1,2,3 Correct B: 1,3 Correct C: 2,4 Correct D: 4 Correct E: All Correct ABCDE Correct Answer: D A Horner's syndrome typically appears shortly after the administration of local anaesthetic agent in the proximity of the stellate ganglion. The onset and duration time vary with the agent used; both generally precede the onset and diminishment of the block by some time. For

518

example, with lignocaine, the Horner's syndrome occurs in 2-15 minutes and lasts 1-3 hours, increasing further with high concentrations and with the use of vasoconstrictors. Signs of vasodilation may not occur for 30 minutes, and may not recede for 24-48 hours. Thus, a successful block is generally heralded by a Horner's syndrome and signs of an ipsilateral sympathetic block in the arm. However, a Horner's syndrome will occur if the middle or superior cervical sympathetic ganglion, or any part of the sympathetic chain in proximity are blocked. As this is common, this sign may be present in the event of an unsuccessful block. This makes no difference if the block was performed as a cerebral sympathetic block, but no benefit will be gained if a block of the arm was intended. Signs of sympathetic blockade in the arm will signify success. These include an increase in skin temperature, dryness of the skin, , and vasodilation of the peripheral veins. If these are not present, the procedure should be repeated. 15-30 minutes commonly elapse before these signs evolve, as this time can be required for the local anaesthetic agent injected at C7 to diffuse to T4, even when injected into the correct fascial plane. The block should not be repeated within this time. Many important structures lie within the vicinity of the stellate ganglion, and may be encountered during the procedure. The gravity of injury to any of these will be compounded if it occurs bilaterally, hence simultaneous bilateral blockade is not recommended. Examples include: -pneumothorax <1% -subarachnoid injection -Paralysis of recurrent laryngeal nerve 5-8% -injury to brachial plexus 5% -vagal cardiac arrest may follow bilateral blockade of cardioaccelerator fibres. This is seldom noted in unilateral blocks. Pretreatment with atropine should be carried out in the event of bilateral blockade. -bilateral blockade of phrenic, vagus, inferior laryngeal nerves Therefore, if a bilateral block is desired, the initial block should be allowed to wear off to exclude complications which will be poorly tolerated if they occur bilaterally. If the needle is placed too far laterally or advanced too far posteriorly, when contact with the seventh cervical transverse process is being attempted, the following may be noted. The needle appears to advance into fibrinous tissue with bone encountered at a greater depth. Normally this bony landmark is encountered after little or no resistance.Therefore when partial resistance is met , it can be assumed that the needle is malpositioned in fibrous tissue. This is reinforced by engaging bone at a deeper level. When this occurs the following may be noted: the solution will not easily be discharged through the needle; a Horner's syndrome will not appear; and the patient may complain of pain between the scapulae. References MOORE, D.C.; Regional Block, 4th Ed., Charles C Thomas (publisher), Illinois, 1965, pp 123-137. Number: 119 Which of the following have been reported to have a role in the treatment of postdural puncture headache?

519

1. Intravenous saline. 2. Epidural saline. 3. Intravenous caffeine. 4. Abdominal binders. A: 1,2,3 Correct B: 1,3 Correct C: 2,4 Correct D: 4 Correct E: All Correct ABCDE Correct Answer: A Extradural blood patch remains the definitive treatment for post-dural puncture headache. Other methods with variable success include intravenous saline, extradural saline boluses or infusion, and IV or oral caffeine benzoate. Methods which have not been shown to be of any benefit include abdominal binding. CARRIE, L.E.S.; " Postdural puncture headache and extradural blood patch ", British Journal of Anaesthesia, vol 71, no2, August 1993, pp 179-81. Number: 120 The disadvantages of a peribulbar block as compared with a retrobulbar block include: 1. Larger volume of solution required. 2. Higher incidence of incomplete block. 3. Longer onset time. 4. Blinking, which is more prevalent. A: 1,2,3 Correct B: 1,3 Correct C: 2,4 Correct D: 4 Correct E: All Correct ABCDE Correct Answer: A The advantages of a peribulbar approach as compared with a retrobulbar block are essentially a reduced incidence of injury to the optic nerve, retrobulbar hemorrhage, and intradural spread of solution.

520

The disadvantages are that blockade of branches of cranial nerves 3-7, and ciliary ganglion occur by diffusion of the local anaesthetic agent resulting in the need for larger volumes (causing greater swelling and conjunctival oedema), longer onset time, and a greater incidence of inadequate block. In contrast to the retrobulbar block, akinesia of the extraocular muscles is usually adequate without a separate block of the facial nerve. ATKINSON, R.S. ET AL; Lee's Synopsis of Anaesthesia, 11th Ed., pp 544-6. Number: 122 In the presence of a suspected unstable cervical spine injury: A. Stabilization is best achieved with the head taped between sand bags. B. Airway manoeuvres of chin lift and jaw thrust are safe as long as the neck is immobilized in a hard collar. C. In-line axial traction greatly reduces movement during laryngoscopy. D. No difference in movement occurs between naso- and oro-tracheal intubation, hence either technique may be employed. E. All are correct. Select the single best answer ABCDE Correct Answer: A Stabilization of the cervical spine is best achieved by immobilizing the head and neck with tape between two sandbags. Most manoeuvres performed in securing the airway have been shown to cause some instability of the cervical spine despite employing approved methods of stabilization. Nasotracheal intubation is a safer method in this respect, however, it is associated with a greater incidence of airway complications, and is a relative contraindication in the presence of a basal skull fracture. Controversy still surrounds the best method of managing these patients. Successful orotracheal intubation depends upon flexion of the neck and extension at the atlantoaxial joint. Considerable neck movement may be required to create satisfactory alignment of the oral-pharyngeal-laryngeal axes. The disposition of the neck has been studied radiologically during laryngoscopy. It has been noted that the standard intubating position is associated with relatively horizontal lower cervical vertebrae, but with increasing extension from C4 to C1, becoming maximal at the atlanto-axial joint. As most injuries occur between C3 to C7, the relative stability found here offers a degree of protection during airway manoeuvres. The best method of stabilization of the cervical spine during intubation remains controversial. The efficacy of bidirectional traction during laryngoscopy has been studied in patients with variable results. One study performed postmortem on patients with unstable injuries found it to permit unacceptable distraction at sites of instability. Another study on healthy volunteers compared straight and curved laryngoscope blades, the hard collar, and in-line axial traction. None of these methods prevented movement during orotracheal intubation, but it occurred least with in-line traction. In another study, the effects of chin lift, jaw thrust, head tilt, airway insertion, and oro- or naso- intubation were studied in cadavers, where C5-C6 ligamentous

521

instability was created surgically. They found that any manoeuvre which produced extension of the head on the neck produced anterior subluxation and widening of the joint space at the level of the injury. This occurred least with an oro- or naso- pharyngeal airway (2mm), but to a greater degree with the chin lift and jaw thrust manoeuvres (5mm). Orotracheal intubation produced intermediate widening in disc space (3-4 mm), while blind nasopharyngeal intubation produced up to 2mm of posterior subluxation, but no increase in disc space. These manoeuvres were repeated after application of either a soft or hard collar, however no consistent effect was seen and in some cases they produced a worsening of the instability. Despite these experimental findings, extensive clinical experience with rapid sequence inductions and oro-tracheal intubation exists. Two centres in the USA published their results which when combined describe more than 3000 cases of suspected cervical spine injury. 37 were subsequently shown to have actual unstable injuries. Either in-line axial traction or neutral immobilization were employed. Neurological deterioration occurred in none of these cases as a result of airway management. The use of blind naso-tracheal intubation to control the airway of spontaneously breathing patients is a popular technique in emergency rooms in the U.S.A. As explained, experimental studies suggest it causes less instability in the presence of unstable injuries, and in this group of patients, it is included in the protocol of the American College of Surgeons Committee of Trauma. It has been compared with oro-tracheal intubation in a study involving 113 patients with unstable injuries requiring surgical stabilization. Oral intubation was found to be superior in terms of success rate (100% versus 65%), time to accomplish (64 seconds versus 276 seconds), and mean number of attempts (1.3 versus 3.7). The nasal route had a complication rate of 86% (69% epistaxis, 17% vomiting, and 10% aspiration). Multiple attempts at nasal intubation were not unusual. Oro-tracheal intubation has not been associated with more adverse neurological outcome than naso-tracheal intubation in actual patients. In addition, there are also objections to the use of this route in the presence of a basal skull fracture. The successful use of fibreoptic intubation is well described but may be confounded by lack of operator experience and extensive airway trauma. WOOD, P.R. & LAWLER, P.G.P.; " Managing the airway in cervical spine injury ", Anaesthesia, vol 47, 1992, pp 792-7. Number: 123 Which period related to elective abdominal aortic resection is associated with the greatest mortality? A. Induction. B. Aortic cross-clamping. C. Emergence. D. Postoperative. E. None of the above. Select the single best answer ABCDE

522

Correct Answer: D The 30 day operative mortality from elective abdominal aortic aneurysm resection is between 1% and 6%. Myocardial infarction is responsible for 40%-70% of cases. Whilst a PMI may occur at any time during the perioperative period, they typically occur on the third postoperative day. Whether intraoperative or postoperative events are responsible for these is not resolved. Moreover, up to 30% of intraoperative infarctions will not be accompanied by ECG changes but may manifest in the postoperative period. CUNNINGHAM, A.J; " Anaesthesia for abdominal aortic surgery- a review (part 1) ", Can J Anaesth, vol 36, no 4, 1989, pp 426-44. Number: 124 With regards to the risks and benefits of carotid endarterectomy, which of the following are true? 1. The overall mortality is 10%. 2. Outcome is clearly poorer in the presence of untreated hypertension. 3. Operability should be based on degree of occlusion as evidenced by duplex scanning, and not on symptoms. 4. The risk of neurological deficit from an untreated and asymptomatic carotid bruit is less than that from this procedure. A: 1,2,3 Correct B: 1,3 Correct C: 2,4 Correct D: 4 Correct E: All Correct ABCDE Correct Answer: C The mortality from carotid endarterectomy (CEA) is approximately 2%, with a 9% complication rate. These figures are highly variable. Certain high risk groups have been identified in whom the incidence of morbidity and mortality from cerebrovascular accidents (CVA) and perioperative myocardial infarction (PMI) is high. Untreated hypertension is clearly associated with a higher incidence of these. These risks must be balanced against the likelihood of neurological deficit associated with the natural course of the disease left untreated. Transient ischaemic attacks, especially occurring within one month can herald sudden deterioration. Crescendo attacks and amaurosis fugax are considered indications for surgery. These are usually supported by significant stenosis as described by duplex scanning. Alternatively, severe stenosis unaccompanied by symptoms is also an indication for surgery. The incidence of CVA occurring in the presence of an asymptomatic bruit is 4% and in many centres is not considered an indication for surgery. The risk/benefit ratio associated with carotid endarterectomy has been examined recently. Overall, the mortality approaches 2%, however there is considerable variation in outcome between centres and the type of lesion being operated on. The main causes of serious

523

morbidity and mortality associated with CEA are PMI and CVA. PMI is primarily responsible for death after CEA (Hertzer & Less reported 60% versus 17% neurological causes in 1981).With respect to myocardial ischaemia, the risks are similar to those described for other forms of noncardiac surgery. With respect to neurologic complications, recent work has compared the risk of stroke from the unoperated lesion with that associated with the operation. Current tentative indications include: (1)SYMPTOMATIC - Transient ischaemic attacks (TIA) or amaurosis fugax in a setting of >50% occlusion of the ipsilateral common or internal carotid artery on angiography or duplex scanning. - Persistent focal TIAs despite anticoagulation and <50% occlusion. - Crescendo TIAs or stroke-in-evolution. In symptomatic patients, untreated lesions produce an annual incidence of CVA of 5% ,approaching a maximum of 15% in the first year. It is generally accepted that an overall morbidity and mortality rate (MM) of <5% is required to justify surgery in these patients.With respect to isolated TIAs, the indication for surgery is less clear.The risk of stroke is 4-6% per year and 24-36% overall. In another study, the calculated surgical risk in this group approached 9.1% and it was considered a reasonable indication for surgery. (2)ASYMPTOMATIC - Asymptomatic bruits. In 1988, Winslow conducted a study of indications, complications and outcome in CEAs, and determined that 50% were done for this group. This was felt to be inappropriate since the complication rate of 9% greatly exceeded a risk of stroke from the unoperated lesion of 4%. The argument is not entirely resolved; proponents of surgery argue that a major stroke will ensue within 5 years in 15-20%, whereas those supporting medical treatment argue that the annual stroke rate in untreated cases is 2.5%. -Significant occlusion of common or internal carotid arteries as evidenced by duplex scanning or angiography. Most surgeons agree that an occlusion of >75% is associated with a greater risk of stroke untreated. A consideration of the factors affecting outcome is required when evaluating anaesthetic risk in individual cases. These include: - Degree of carotid disease-stroke in evolution (21% MM) - Previous TIAs or asymptomatic bruit (6.5% combined MM) - acute carotid occlusion (20% MM versus 55% untreated) - Age-Mortality is directly related to age - Hypertension-Preoperative hypertension has a greater incidence of uncontrolled, postoperative hypertension and neurological complications (3.4% normotensive versus 10.2% hypertensive). The mechanism may be that previously protected vessels are now exposed to higher perfusion pressures following removal of occlusive plaques - Diabetes- no correlation. ROGERS, M.C. ET AL (EDS); Principles and Practice of Anesthesiology, Mosby, 1993, pp 201-4, 209-12.

524

Number: 125 Which of the following are true of abdominal laparoscopic procedures? 1. Initial insufflation is applied to an intra-abdominal pressure of 30 mm Hg. 2. A volume of 6L of CO2 is generally required to enable satisfactory visualization. 3. CO2 emboli always require treatment. 4. Referred pain to the scapulae is common. A: 1,2,3 Correct B: 1,3 Correct C: 2,4 Correct D: 4 Correct E: All Correct ABCDE Correct Answer: D An intra-abdominal pressure (IAP) of 10-15 mm Hg is generally adequate to achieve insufflation; exceeding this may result in a variety of cardiopulmonary and haemodynamic derangements. The total volume of gas required will depend upon the duration of the procedure, as continuous leaks must be replaced. Flows of 4-6 L/min are generally employed during initial insufflation, and are reduced when the pneumoperitoneum is established. CO2 emboli are felt to occur commonly as small, multiple emboli which dissolve spontaneously and do not cause any cardiopulmonary disturbance. Symptomatic emboli occur uncommonly and require treatment. Referred pain to the scapulae is common despite the early anticipation that the high solubility of CO2 would cause rapid resolution of the pneumoperitoneum. In abdominal and pelvic laparoscopic surgery, a pneumoperitoneum is created by the insufflation of gas through a puncture created in the abdominal wall. The pneumoperitoneum separates the abdominal wall from the viscera, improving visual and physical access. For initial insufflation and to replenish the pneumoperitoneum in the event of gas leak, high gas flow insufflators are used. Modern models can usually deliver gas at flows of up to 4-6 L/min. These flows are either reduced or ceased when an adequate pneumoperitoneum is achieved. Total volumes of gas may be of the order of 20-40 L. These insufflators generally contain digital displays of the intraabdominal pressure (IAP) which can be used to guide insufflation and troubleshoot. An IAP of 10-15 mm Hg is generally adequate. Note that older models designed for gynaecological surgery lack controls to limit the maximum insufflation pressure and can reach IAPs of 40 mm Hg. These features are of relevance to the anaesthetist for many reasons. (1)CO2 emboli are felt to occur commonly, however most of these are small and multiple and are accommodated by the cardiovascular system, where they dissolve and are eliminated by the lungs. The sequelae of CO2 emboli depend upon their size, ability to dissolve spontaneously, site of embolization, and the preexisting condition of the patient. In dogs a large intravenous bolus of air (3-8 ml/Kg) will produce an " air lock " with complete mechanical obstruction of the right atrium and ventricle. In contrast,a slow infusion (0.3 ml/Kg/min) is absorbed across the pulmonary capillary-alveolar membranes without

525

producing any clinical effect. CO2 is the preferred gas because it occurs " naturally ", and a physiological system exists for its elimination. Moreover, it is 5-6 times more soluble than air. Hence, the incidence of postoperative pain from a residual pneumoperitoneum is reduced and the " lethal " volume of an embolus is 5-6 times that of air. Despite its greater margin of safety, these volumes can be exceeded. If the Verres needle is malpositioned in a vessel in the abdominal wall or peritoneum during high flow insufflation, large volumes may enter the venous system. Moreover, as these structures lack the compliance of the peritoneal cavity, insufflating pressures will increase rapidly, encouraging this event. Analysis of case reports describing this complication and others like surgical emphysema and pneumothorax often carry a history of difficulty with insertion or repositioning of the Verres cannula, which shortly precedes clinical deterioration, although late deterioration has been described, presumably due to occult pooling of the gas in the portal system. (2) The Intraabdominal pressure (IAP) changes which occur subsequent to gas insufflation may cause disturbances in pulmonary compliance, myocardial function (secondary to decreased venous return and increased afterload), perfusion to intra-abdominal organs, and renal function. These changes are pressure dependant and are seen in other conditions where IAP is elevated. These derangements may be exacerbated by postural requirements of the surgery, and large total volumes of CO2 which are likely to accompany them. References CHUI, P.T. GIN,T. & OH, T.E; " Anaesthesia for Laparoscopic General Surgery ", Anaesth Intens Care, vol 21, no 2, April 1993, pp 163-71, (REVIEW). LEW, J.K.L. GIN, T. & OH, T.E; " Anaesthetic Problems During Laparoscopic Cholecystectomy ", Anaesth Intens Care, vol 20, no 1, Feb. 1992, pp 91-2. (CASE REPORT). CLARK, C.C ET AL ;" Venous Carbon Dioxide Embolism During Laparoscopy ", Anesth Analg, 56, 1977, pp 650-2. (CASE REPORT). GABBOTT, D.A. ET AL ;" Carbon dioxide pneumothorax occurring during laparoscopic cholecystectomy ", Anaesthesia, vol 47, 1992, pp 587-8. (CASE REPORT). ROOT, B ET AL; " Gas Embolism Death After Laparoscopy Delayed By "Trapping" in Portal Circulation ", Anesth Analg, vol 57, 1978, pp 232-7. Number: 126 A patient is undergoing a minor gynaecologic laparoscopic procedure. CO2 embolus is suspected. Which of the following would support this diagnosis? 1. Third heart sound. 2. Millwheel murmur. 3. Abdominal distension. 4. Decreased ETCO2. A: 1,2,3 Correct B: 1,3 Correct C: 2,4 Correct D: 4 Correct E: All Correct

526

ABCDE Correct Answer: C Disturbances in myocardial function associated with a gas embolism generally take the form of right ventricular strain or failure (jugular venous distension, parasternal heave, loud second heart sound with failure of splitting to close at expiration, right axis or bundle branch block on ECG). A millwheel murmur betrays the presence of gas "locked" in the cardiac chambers. Systemic hypotension results from a fall in cardiac output subsequent to diminished output from the right ventricle. A third heart sound occurs in congestive cardiac failure and is felt to result from the rapid filling of a dysfunctional left ventricle. CO2 emboli have generally occurred in the context of a malpositioned verres needle whose tip may be sitting in the abdominal wall rather than the peritoneal cavity. Possibly a failure to achieve abdominal distension may be more typical, however it has been reported as normal in most case reports. A rapid fall in the ETCO2 is highly suggestive of pulmonary embolism of any description. The presentation of CO2 embolus during laparoscopic surgery depends upon the volume of the embolus, site of embolization, and physical condition of the patient. It is felt that it occurs commonly in the form of multiple, small emboli which dissolve spontaneously and do not produce any cardiopulmonary disturbances. In contrast, a massive embolus may present as sudden cardiovascular collapse. Cardiovascular collapse from gas embolism has been demonstrated experimentally. Immediately after a bolus of air enters the right cardiac chamber, contractions of the right ventricle against blood gas interfaces produce the loud, churning sound known as the millwheel murmur. There is a sudden marked dilation of the right side of the heart, especially in the right ventricular outflow tract. When gas lodges in the peripheral pulmonary vasculature, bubbles provoke neutrophil clumping, activation of the coagulation cascade, and platelet aggregation at blood-gas interfaces. The release of chemical mediators produces pulmonary vasoconstriction, bronchospasm, pulmonary oedema and delayed pulmonary haemorrhage. Pulmonary vascular resistance increases. The increase in right ventricular afterload may cause arrhythmias. Paradoxical emboli may occur through a pressure patent foramen ovale. Increased right ventricular pressure restricts the right ventricular thebesian flow and decreases the aortic-right ventricular pressure gradient, which is reflected in myocardial ischaemia. The pulmonary artery pressure rises and the systemic blood pressure falls; this is termed " equalization ". References CLARK, C.C ET AL; " Venous Carbon Dioxide Embolism During Laparoscopy. ", Anesth Analg, vol 56, 1977, pp 650-2. CHUI, P.T. GIN, T. & OH, T.E; " Anaesthesia for Laparoscopic General Surgery ", Anaesth Intens Care, 21, 1993, pp 163-71.

527

Number: 127 The oculo-cardiac reflex occurs: 1. Commonly during traction on the medial rectus muscle. 2. Rarely during the performance of a retrobulbar block. 3. Commonly during pressure on the eyeball. 4. More commonly in people with blue eyes. A: 1,2,3 Correct B: 1,3 Correct C: 2,4 Correct D: 4 Correct E: All Correct ABCDE Correct Answer: A The oculocardiac reflex (Aschner's reflex) was first described in 1908 and refers to abnormalities of the rate or rhythm of the heart ensuing from stimuli arising in or around the eye. The reflex is trigeminovagal. The afferent limb is from orbital contents to ciliary ganglion, to ophthlalmic division of the trigeminal nerve, to sensory nucleus of the trigeminal nerve near the fourth ventricle. The efferent limb is via the vagus nerve. Anaesthesia appears to make the heart vulnerable to increased vagal tone, especially in the young. It is suggested that people with blue or grey eyes are less likely to develop an oculocardiac reflex dysrhythmia than patients with brown or hazel eyes. Dysrhythmias include sinus bradycardia, atrio-ventricular block, ventricular ectopy, and sinus arrest with ventricular standstill. A variety of stimuli may cause the reflex, however, traction on the medial rectus or eyeball pressure are notorious. It can occur during operations on an empty orbit. Hypercapnia and hypoxia exacerbate the reflex. Gallamine, atropine, glycopyrrolate, and hyoscine butylbromide ('buscopan') given intravenously have all been recommended as prophylactic agents. Intramuscular atropine has no preventative role. Retrobulbar injections of local anaesthetic agents are not uniformly effective in preventing the reflex, and may actually cause it. It generally resolves with cessation of the stimulus. If it does not, 7 ug/Kg atropine given intravenously is the recommended treatment. See: Complications of local anaesthesia for ophthalmic surgery. Rubin AP; Br. J. Anaesth. 1995; 75:93-96.

528

Number: 128 Which of the following may be beneficial in the event of cardiovascular collapse from a gas embolism? 1. Durant's manoeuvre. 2. Central venous access. 3 .Oesophageal stethoscope. 4. Positive Trendelenberg positioning. A: 1,2,3 Correct B: 1,3 Correct C: 2,4 Correct D: 4 Correct E: All Correct ABCDE Correct Answer: E Cardiovascular collapse subsequent to a massive gas embolism may result from "locking" of gas in the right cardiac chambers or outflow tract and pulmonary artery.The Durant manoeuvre involves turning the patient into the left lateral positioning such that the outflow tract occupies a position inferior to the rest of the right ventricle. Effective ventricular output may ensue. In patients in the sitting position in whom this manoeuvre is not possible, aspiration of gas from the cardiac chambers should be attempted.This necessitates central venous access. Recovery of intracardiac gas is difficult. Precordial and oesophageal stethoscopes have been shown to be of use as they facilitate early detection of small emboli. A positive trendelenberg may be of benefit. If the gas is entrained during a neurosurgical procedure, this position will cause the cerebral venous pressure to rise which will reduce the rate of entrainment. CLARK, C.C ET AL; " Venous Carbon Dioxide Embolism During Laparoscopy ", Anesth Analg, vol 56, 1978, pp 650-2. Number: 201 With regard to Intra Vascular Regional Anaesthesia (IVRA): 1. Bupivacaine is a suitable agent for this technique. 2. Intra-arterial regional anaesthesia (IARA) requires only half the dose of agent as compared to IVRA.

529

3. The dose required for 'top-up' after cuff release and re-inflation during IVRA is the same as the initial dose. 4. Prilocaine is a safer agent than lignocaine. A: 1,2,3 Correct B: 1,3 Correct C: 2,4 Correct D: 4 Correct E: All Correct ABCDE Correct Answer: C Bupivacaine is specifically contraindicated for IVRA. It has been associated with several deaths and offers no advantages compared to prilocaine or lignocaine. Intra-arterial regional anaesthesia was first described in 1908 but has only been used sporadically since that time. It approximately halves the amount of agent which has to be administered, but has the obvious drawback of the potentially greater morbidity associated with intra-arterial puncture. A technique of prolonged IVRA involving intermittent cuff release every hour has been described. Under these circumstances only half the initial dose is required for the 'top-up'. Although there have been no large studies comparing the safety of prilocaine and lignocaine, it seems likely that prilocaine is intrinsically the safest agent. References: Bupivacaine toxicity and Bier Blocks. Heath ML:Anesthesiology 59:481, 1983 Heath has reported seven deaths in the United Kingdom associated with the use of bupivacaine during Biers Blockade. As there is absolutely no advantage to the patient resulting from the use of this drug for this technique, it is absolutely contraindicated in this context. In Australia, the drug is no longer approved for use with this technique. Intra-arterial versus intravenous regional anaesthesia for hand surgery. Koscielniak ZJ and Horn A. Anaesthesia 1993, 48: pp 769-792 The authors compared the efficacy of IARA vs IVRA in 26 patients undergoing hand surgery. IA injection was found to be more painful than IV injection, but otherwise the techniques produced similar results in terms of quality of analgesia, incidence of tourniquet pain, motor blockade and bruising. IA injection only requires half the dose of lignocaine as compared to IV injection. Continuous Intravenous Regional Anaesthesia: Glickman LT, Mackinnon SE, Vasudera P, Rao T. J Hand Surg 1992; 17A: 82-86. This study evaluates the effectiveness of continuous intravenous regional anesthesia for prolonged operations on the upper extremity. The factors evaluated include patient's sex and age, number of procedures performed, tourniquet on and off times, anesthetic doses, adjunctive drugs used, technical complications, and side effects. Seventy-two procedures were

530

done on 34 patients. The first tourniquet time averaged 58 minutes. Off time averaged 10 minutes. Second tourniquet time averaged 33 minutes, and the mean total tourniquet time was 91 minutes. The mean first anesthetic dose was 275 mg. Mean second anesthetic dose was 128 mg. Mean total anesthetic dose was 402 mg. There were two (6%) technical complications and two (6%) patients had side effects. Continuous intravenous regional anesthesia offers the prolonged anesthesia of brachial plexus block or general anesthesia and the safety, reliability, and ease of intravenous regional anesthesia. Continuous intravenous regional anesthesia should be considered an alternative choice of anesthetic method in upper extremity surgery. Prilocaine for Bier's block: how safe is safe? Bartholomew K; Sloan JP.Arch Emerg Med. 1990 Sep; 7(3): 189-95 Prilocaine has become the agent of choice for Bier's block (or intravenous regional anaesthesia--IVRA), since 1983 when the product licence of bupivacaine was withdrawn for this purpose owing to fatal or serious complications. No serious complications have been documented in the literature relating to prilocaine in IVRA and we have conducted a survey within the U.K. which indicates that about 45,000 Bier's blocks have been carried out with prilocaine without convulsion, arrhythmia or fatality. This includes cases of accidental cuff deflation or even failure to inflate the cuff, resulting in bolus doses to the circulation. Prilocaine has now been in use since 1964 and the Committee for Safety of Medicines has no deaths on record over a 25-year period. (This includes other types of regional anaesthesia). We suggest that intravenous regional anaesthesia using prilocaine is a safe technique. It is highly unlikely that fatalities will occur, provided present guidelines are adhered to. We can find no reason to limit its use to trained anaesthetists only, but would recommend that a strict protocol is adhered to. Number: 226 In relation to spinal anaesthesia and "Transient Radicular Irritation" (TRI): 1. It is more common with lignocaine than bupivacaine. 2. It is more common when hyperbaric solutions are used. 3. It usually resolves within two weeks. 4. It is commoner when more concentrated local anaesthetic solutions are used. A: 1,2,3 Correct B: 1,3 Correct C: 2,4 Correct D: 4 Correct E: All Correct ABCDE Correct Answer: B Transient Radicular Irritation (back pain with radiation to one or both buttocks which occurs within 24 hours of spinal anaesthesia and which resolves within two weeks) is particularly associated with the use of hyperbaric lignocaine for spinal anaesthesia.

531

However, the main risk factors for TRI are still in dispute. Baricity itself does not appear to be a risk factor in that the incidence of TRI is low if hyperbaric bupivacaine (rather than lignocaine) is used. - See: Br J Anaesth 1995 Mar;74(3):328-9 Transient radicular irritation after spinal anaesthesia with hyperbaric 5% lignocaine.Tarkkila P, Huhtala J, Tuominen M. As far as I can tell, the syndrome is not more common when more concentrated local anaesthetic solutions are used. Number: 296 An epidural anaesthetic administered for obstetric analgesia is less likely to be successful if: 1. Inadvertent dural puncture (treated conservatively) occurred during a previous labour. 2. An uncomplicated epidural anaesthetic was administered during a previous labour. 3. Inadvertent dural puncture (treated by blood patching) occurred during a previous labour. 4. The patient has had a previous spinal anaesthetic administered with a 27 gauge needle. A: 1,2,3 Correct B: 1,3 Correct C: 2,4 Correct D: 4 Correct E: All Correct ABCDE Correct Answer: B Accidental dural punctures can occur during attempted epidural anesthesia and, in addition to complicating the anesthetic administration at the time, can compromise subsequent epidural anesthetic attempts. In reviewing the records of 200 patients who had accidental dural punctures during insertion of epidural catheters, Ong reported on the subsequent administration of epidural anesthesia in those who were treated with and without epidural blood patches. He showed that a previous dural puncture significantly reduced success of a subsequent epidural anesthetic. If a blood patch had been administered, only 59% of the patients had an uncomplicated successful second epidural anesthetic. In the event of a dural puncture but with no blood patch, the success rate was 65%. This compared unfavorably with the 90% success rate in parturients who had previous epidural anesthesia without dural puncture. Ong BY, Graham CR, Ringaert KRA, et al: Impaired epidural analgesia after dural puncture with and without subsequent blood patch, Anesth Analg 70:76-79, 1990. Number: 305 Complications of stellate ganglion blockade include:

532

1. Recurrent laryngeal nerve block. 2. Cerebral air embolism. 3. Phrenic nerve block. 4. Partial brachial plexus block. A: 1,2,3 Correct B: 1,3 Correct C: 2,4 Correct D: 4 Correct E: All Correct ABCDE Correct Answer: E Side effects from a stellate ganglion block should be distinguished from complications. Most unpleasant side effects result from the Horner's syndrome and include ptosis, myosis, and nasal congestion. Common complications from a stellate ganglion block occur from the diffusion of local anesthetic onto nearby nervous structures. These include the recurrent laryngeal nerve with complaints of hoarseness, feeling of a lump in the throat, and sometimes a subjective shortness of breath. Bilateral stellate blocks are rarely advised, since bilateral blocking of the recurrent laryngeal nerve can result in respiratory compromise and loss of laryngeal reflexes. Block of the phrenic nerve results in temporary paralysis of the diaphragm and can result in respiratory embarrassment in patients whose respiratory reserve is already severely compromised. Partial brachial plexus block can also result secondary to spread along the prevertebral fascia or if the needle location is too posterior. The patient should be discharged with a sling and careful instructions on how to care for a partially blocked arm should this occur. The two most feared complications from a stellate ganglion block include an intraspinal injection and seizures from an intravascular injection. Respiratory embarrassment and the need for mechanical ventilation can result from either injection into the epidural (if high concentrations of local anesthetic are used) or intrathecal space. Should this occur, patients need continual reassurance that everything is being appropriately managed, and they will recover without sequelae. Some sedation is required while the local anesthetic wears off. No drugs are necessary for endotracheal cannulation since profound anesthesia of the larynx can be expected. Intravascular injection most commonly occurs in the vertebral artery. Small amounts of local anesthetic result in unconsciousness, respiratory paralysis, seizures, and sometimes severe arterial hypotension. Increased IV fluids, vasopressors if indicated, oxygen, and endotracheal intubation may be necessary. If the amount of drug injected into the artery is small (less than 2 ml), the above sequelae will be short-lived and self-limiting, with oxygen and increased fluid administration often being the only needed therapy. One must be careful when performing a stellate ganglion block that no air is injected from the syringe. Cerebral air embolism has been reported from this procedure and is preventable.

533

Number: 310 Carotid endarterectomy under regional anaesthesia may be associated with: 1. A lower incidence of perioperative stroke. 2. A lower incidence of perioperative MI. 3. Ipsilateral phrenic nerve block in at least 50% of patients. 4. A lower incidence of perioperative death. A: 1,2,3 Correct B: 1,3 Correct C: 2,4 Correct D: 4 Correct E: All Correct ABCDE Correct Answer: B There is evidence that the perioperative stroke rate is lower if CEA is performed under regional anaesthesia. Perioperative MI and death rates seem to be unaffected by the anaesthetic technique. Zvara DA J. Cardiothor and Vasc Anaesth. 111-114, 1998. 55-60% of patients undergoing CEA under cervical plexus block will have evidence of an ipsilateral phrenic nerve block. G. EMERY, G. HANDLEY, M. J. DAVIES:, P. H. MOONEY Anaesth Intensive Care 1998; 26: 377-381 Incidenceof Phrenic Nerve Block and Hypercapnia in Patients Undergoing Carotid Endarterectomy Under Cervical Plexus Block. Number: 313 Pethidine administered intrathecally: 1. Behaves as a hyperbaric solution. 2. Has a shorter half-life in the CSF than morphine. 3. Is likely to cause pruritus. 4. Can be used as a sole agent for lower limb surgery. A: 1,2,3 Correct B: 1,3 Correct C: 2,4 Correct D: 4 Correct E: All Correct ABCDE

534

Correct Answer: E Pethidine was the first synthetic opioid to be used to provide analgesia in humans. Shortly after its analgesic properties were discovered by Eisleb and Schaumann in 1939, pethidine was shown also to have local anaesthetic activity comparable with that of cocaine. Subsequently, pethidine has been investigated for a number of clinical applications, including peripheral nerve block, intravenous regional anaesthesia, and intrathecally for spinal anaesthesia and analgesia. Following initial reports in animals, Cousins et al reported the use of intrathecal pethidine in doses of 10 to 30 mg to treat cancer pain in humans. The first report of the use of pethidine as sole agent for spinal anaesthesia was published in the French literature by Mircea in 1982 and has been followed by a number of reports that have used pethidine alone and in combination with conventional local anaesthetics for spinal anaesthesia for a variety of different surgical procedures. The standard preparation has a baricity of 1009. It disappears more rapidly from the CSF than morphine because it is more lipid soluble. In a dose of 0.5 - 1.0 mgs/kg it can be used as a sole agent for lower limb surgery. Anaesth Intens Care 1998; 26:137-146 Intrathecal Pethidine: Pharmacology and Clinical Applications W. D. NGAN KEE Number: 315 With regard to spinal anaesthesia with pethidine: 1. The addition of clonidine will prolong the duration of surgical anaesthesia. 2. The addition of adrenaline will prolong the duration of surgical anaesthesia. 3. The addition of clonidine will increase the incidence of bradycardia and hypotension. 4. An appropriate dose of pethidine is 1mg/kg. A: 1,2,3 Correct B: 1,3 Correct C: 2,4 Correct D: 4 Correct E: All Correct ABCDE Correct Answer: E Spinal anesthesia with meperidine. Effects of added alpha-adrenergic agonists: epinephrine versus clonidine. Acalovschi I, Bodolea C, Manoiu C. Anesth Analg 1997; 84: 1333-1339. Both epinephrine (adrenaline) and clonidine enhance the duration and intensity of spinal blockade produced by a local anaesthetic. This study addresses the question of whether this would also be the case when pethidine is used instead of a local anaesthetic agent. Intrathecal pethidine has distinct local anaesthetic properties, resulting in surgical analgesia. In this

535

double-blind, randomized study, 45 patients were assigned to three groups to receive 1 mg/kg pethidine alone, or 1 mg/kg pethidine plus either 200 micrograms epinephrine or 2 micrograms / kg clonidine. All treatments were administered intrathecally. When compared with pethidine alone, both pethidine plus epinephrine and pethidine plus clonidine produced longer lasting and more intense spinal blockade as measured by the occurrence of spinal motor block at Bromage grade 3. The duration of sensory blockade was significantly longer in the groups receiving either epinephrine or clonidine: pethidine alone 70.8 +/- 13.6 minutes; pethidine and epinephrine 96.1 +/- 23.0 minutes; pethidine and clonidine 109.0 +/-19.9 minutes. The addition of clonidine to the pethidine solution significantly increased the time to the first postoperative analgesic demand when compared with pethidine alone and pethidine and epinephrine (555.0 +/- 100.0 minutes vs 202.5 +/-156.0 minutes and 201.0 +/-71.0 minutes respectively). There was no difference, however in the need for further analgesia during the first postoperative 24 hours in the three groups. The level of sedation, as measured by the Ramsay sedation scale, was more pronounced in patients receiving the pethidine and clonidine combination. The favourable effect of clonidine was further limited by the occurrence of bradycardia and hypotension. Number: 396 With regard to a patient who reports a previous allergic reaction to lignocaine during a dental procedure: 1, The most likely diagnosis is that the patient suffered a vasovagal attack. 2. The patient can be safely challenged by administration of 1% lignocaine diluted 1:100. 3. Skin challenge with normal saline is likely to reproduce symptoms, 4. An ester linked local anaesthetic can be safely administered. A: 1,2,3 Correct B: 1,3 Correct C: 2,4 Correct D: 4 Correct E: All Correct ABCDE Correct Answer: E The likelihood of true allergy to a local anaesthetic solution is less than 5%. Far more likely is that the patient suffered a vasovagal attack or a reaction to some other drug. 1% lignocaine diluted 1:100 is an appropriate dilution for performing the initial skin tests in such a patient. Skin challenge with normal saline has at least a 50% chance of reproducing the symptoms.

536

Anaesth IntensCare 1997; 25: 611-614 Alleged Allergy to Local Anaesthetics M. MCD. FISHER*, C. J. BOWEY The aim of this study was to determine the incidence of true local anaesthetic allergy in patients with an alleged history of local anaesthetic allergy and whether subsequent exposure to local anaesthetics is safe. Two hundred and eight patients with a history of allergy to local anaesthesia were referred over a twenty year penod to our Anaesthetic Allergy Clinic. In this open study, intradermal testing was performed in three patients and progressive challenge in 202 patients. Four patients had immediate allergy and four patients delayed allergic reactions. One hundred and ninety seven patients were not allergic to local anaesthetics. In 39 patients an adverse response to additives in local anaesthetic solutions could not be excluded. In all but one patient local anaesthesia has been given uneventfully subsequently. A history of allergy to local anaesthesia is unlikely to be genuine and local anaesthetic allergy is rare. In most instances LA allergy can be excluded from the history and the safes of LA verified by progressive challenge. Number: 507 For an operation on the palm of the hand which of the following nerves must be blocked? 1. Musculocutaneous 2. Ulnar 3. Radial 4. Median A: 1,2,3 Correct B: 1,3 Correct C: 2,4 Correct D: 4 Correct E: All Correct ABCDE Correct Answer: C The ulnar and median nerves supply the medial and lateral aspects of the palm respectively. The radial nerve supplies sensation over the radial side of the dorsum of the hand. The musculocutaneous nerve continues in the forearm as the lateral cutaneous nerve and supplies the radial side of the forearm as far as the thenar eminence.

537

Number: 543 In the first trimester of pregnancy, the MAC of isoflurane: A. Is increased by about 50%. B. Is increased by about 30%. C. Is unchanged from the non-pregnant state. D. Is reduced by about 30%. E. Is reduced by about 50%. Select the single best answer ABCDE Correct Answer: D In the first trimester of pregnancy, the MAC of isoflurane is reduced by about 30%. It returns to normal in the early post-partum period. The same is probably true of all other volatile agents. The effect is in some way associated with high progesterone levels. Chan MT, Gin T: Anesthesiology 1995 Jun;82(6):1360-3 Postpartum changes in the minimum alveolar concentration of isoflurane. Gin T, Chan MT: Anesthesiology 1994 Oct;81(4):829-32 Decreased minimum alveolar concentration of isoflurane in pregnant humans. Number: 576 The principal site of action of an epidurally-administered local anaesthetic injected in the midthoracic region is: A. The dorsal root ganglia. B. The substantia gelatinosa. C. The spinal nerves. D. The spinal roots. E. The spinal cord. Select the single best answer ABCDE Correct Answer: D The dorsal and ventral spinal roots appear to be the main site of action of epidurallyadministered local anaesthetics, although they also may act on the spinal nerves within the epidural space. In dogs, it has been shown that rapid diffusion of local anaesthetic into the CSF at the dural cuff region is the most important determinant of onset of epidural block.

538

Local anaesthetics can penetrate the dura and act directly on the spinal cord, but if this occurs, their concentration in the spinal cord is much lower than in the spinal roots. Number: 585 With regard to the differences between phenol and alcohol when used for neurolytic blockade: 1. The onset of blockade is more rapid with alcohol. 2. Alcohol causes pain on injection. 3. Alcohol is a hypobaric solution in comparison to cerebro-spinal fluid (CSF). 4. Phenol is a hypobaric solution in comparison to CSF. A: 1,2,3 Correct B: 1,3 Correct C: 2,4 Correct D: 4 Correct E: All Correct ABCDE Correct Answer: A Alcohol and phenol are both effective neurolytic agents. The most important differences between the drugs are that alcohol has a much more rapid onset than phenol, but produces pain on injection. In contrast, phenol is painless on injection, but takes 15-20 minutes to exert its neurolytic effects. In comparison to the CSF, pure alcohol is hypobaric, whereas 10% phenol is hyperbaric. Number: 618 Which of the following statements regarding spinal anaesthesia is / are true? 1. Whitacre needles cause less trauma to the dura than Sprotte needles. 2. Failure of anaesthesia is more common using a pencil-point needle than using a Quincke needle. 3. Postdural puncture headache increases with increasing age. 4. The incidence of backache increases if more than two attempts at dural puncture are made. A: 1,2,3 Correct B: 1,3 Correct C: 2,4 Correct D: 4 Correct E: All Correct ABCDE

539

Correct Answer: C There is no evidence that any of the pencil-point needles are more or less traumatic than any of the others. However, Quincke needles cause more trauma to the dura than do pencil-point needles. Failure of anesthesia is higher using a pencil-point needle than using a Quincke needle; the need to insert the pencil-point needle a little farther once CSF has been obtained may be a factor. Postdural puncture headache becomes less likely with increasing age. It is most likely in young women. Backache can be a problem after more than two attempts at dural puncture are required especially in women. Number: 639 During left-sided, one lung ventilation with isoflurane in 100% oxygen, a 60 year old patient develops a nodal rhythm with no change in pulse rate, but a 20% fall in mean blood pressure. The arterial saturation which had been steady at 91% falls to 85%. Given the scenario above, the likeliest cause of desaturation is: A. Malposition of the endobronchial tube. B. Absorption atelectasis due to ventilation with 100% oxygen. C. Inappropriate release of hypoxic pulmonary vasoconstriction due to ventilation with isoflurane. D. A reduction in cardiac output. E. None of the above. Select the single best answer ABCDE Correct Answer: D Malposition of the endobronchial tube must always be excluded in every case of unexplained desaturation during one lung anaesthesia (OLA). However, in this case, the effect of reduction in cardiac output is the likeliest cause of the problem. - The patient already has a significant right-to-left shunt and under these circumstances, the effect of a sudden reduction in cardiac output on the mixed venous saturation is more than enough to explain the fall in arterial saturation. Inappropriate release of hypoxic pulmonary vasoconstriction occurs when patients are exposed to most volatile agents, but the magnitude of the effect is usually of little clinical significance.

540

Similarly, absorption atelectasis due to ventilation with 100% oxygen does occur (The 'Critical V/Q' hypothesis of West), but the effect is usually minor. Number: 645 A 78 year old patient with a long-history of Chronic Airflow Limitation (CAL) presents to an Emergency Department with an exacerbation of his chronic bronchitis. He is placed on supplementary oxygen delivered through a 'Hudson' type mask at a flow rate of 10 lpm and an intravenous infusion is commenced. A blood gas analysis taken 30 minutes after admission is reported as: PaO2: 90 mm Hg. PaCO2: 87 mm Hg. pH 7.28. BXS: 14. Hb: 190 G/L SaO2: 95% Given the scenario above what is the next step which should be taken in this patient's management? A. Endotracheal intubation and controlled ventilation. B. Inititiation of antibiotic therapy. C. Initiation of bronchodilator therapy. D. Initiation of controlled oxygen therapy. E. Initiation of aggressive physiotherapy. Select the single best answer ABCDE Correct Answer: D This gentleman has clear evidence of a reliance on his hypoxic ventilatory drive and should be commenced on controlled oxygen therapy (probably 24 or 28% in the first instance.) - The tell-tale signs are: A positive base excess, a high haemoglobin and the history. Number: 647 With regard to the complications of spinal anaesthesia: 1. Post-Dural Puncture Headache (PDPH) is more common in women than men. 2. PDPH is more common if lignocaine as opposed to bupivacaine is used. 3. PDPH is more common if a Quincke as opposed to pencil-point needle is used. 4. Backache is more common if a Quincke as opposed to pencil-point needle is used. A: 1,2,3 Correct B: 1,3 Correct C: 2,4 Correct

541

D: 4 Correct E: All Correct ABCDE Correct Answer: A Surprisingly, the agent used for spinal anaesthesia appears to affect the PDPH rate - see: Naulty JS, Hertwig L, Hunt CO, Datta S, Ostheimer GW, Weiss JB. Influence of local anesthetic solution on postdural puncture headache. Anesthesiology. 1990 Mar;72(3):450-4. The incidence of backache is unaffected by needle design, but is more common if a cutting needle is used and if the patient is female (even non-pregnant). See: Halpern S, Preston R. Postdural puncture headache and spinal needle design. Meta-analyses. Anesthesiology 81:1376-1383, 1994 Number: 658 A fit 20 year old is undergoing reconstruction of the anterior cruciate ligament under spinal anaesthesia. Spinal anaesthesia is initiated with 2.8 mls of hyperbaric 0.5% bupivacaine and an infusion of 1000 mls of lactated Ringers solution is commenced. Five minutes after the spinal injection, the sensory block to pin-prick is noted to be at T8. The blood pressure has fallen from 100/60 mm Hg before the spinal injection to 85/50 mm Hg and the pulse rate has risen from 60 bpm to 75 bpm. 2 minutes later the patient becomes suddenly asystolic. The mechanism(s) which may be implicated in this clinical picture is / are: 1. Hypoperfusion of the vasomotor centre. 2. The von Bezold-Jarisch reflex. 3. Blockade of the cardiac-accelerator nerves. 4. The Bainbridge reflex. A: 1,2,3 Correct B: 1,3 Correct C: 2,4 Correct D: 4 Correct E: All Correct ABCDE Correct Answer: C Although all 4 mechanisms have been implicated in cases of cardiac arrest during spinal anaesthesia, there is a subset of cases where arrest has occurred suddenly and unexpectedly in the presence of a relatively low block. There have been numerous reports of this phenomenon and both the von Bezold-Jarisch and the Bainbridge reflexes have been implicated as mechanisms. It should also be noted that the

542

presence of a prolonged PR interval has been identified as an independent risk factor for this complication. The von Bezold-Jarisch reflex is a paradoxical reflex which is believed to be provoked by vigorous cardiac contraction around a relatively empty ventricular chamber. - The concept of depressor reflexes originating in the heart was introduced by von Bezold in 1867 and was later revived by Jarisch in 1940. The von Bezold-Jarisch reflex originates in cardiac sensory receptors with nonmyelinated vagal afferent pathways. The left ventricle, particularly the inferoposterior wall, is a principal location for these sensory receptors. Stimulation of these inhibitory cardiac receptors by stretch, chemical substances or drugs increases parasympathetic activity and inhibits sympathetic activity. These effects promote reflex bradycardia, vasodilatation and hypotension (von Bezold-Jarisch reflex) and also modulate renin release and vasopressin secretion. Conversely, decreases in the activity of these inhibitory sensory receptors reflexly increase sympathetic activity, vascular resistance, plasma renin activity and vasopressin. It is believed that the reflex may be precipitated by spinal anaesthesia if the ventricle is allowed to operate at a very low end-diastolic volume. The Bainbridge reflex: The stretch receptors of the atria that elicit the Bainbridge reflex transmit their afferent signals through the vagus nerves to the medulla. Efferent signals are transmitted back through both the vagal and sympathetic nerves to increase the heart's rate and presumably also strength of contraction. Thus atrial filling tends to increase pulse rate and contractility and the atrial emptying - such as occurs with vasodilatation induced by spinal anaesthesia - tends to reduce pulse rate and contractility and permit vagal tone to predominate. An excellent review of the problem has recently been published by Pollard. (See: Anesth Analg 2001;92:252-256; Cardiac Arrest During Spinal Anesthesia: Common Mechanisms and Strategies for Prevention. Pollard JB) Number: 688 During removal of an epidural catheter, the tip breaks off, remaining in the patient. After informing the patient, the most appropriate advice to the patient is that: A. Apart from a course of antibiotics, no further treatment is needed unless symptoms occur. B. No further treatment is needed unless symptoms occur. C. Urgent surgical removal of the catheter tip is required. D. Flouroscopically guided removal under local anaesthesia should be attempted. E. Elective surgical removal should be undertaken in 4-6 weeks. Select the single best answer ABCDE Correct Answer: B

543

The consensus view seems to be that no further treatment is needed unless symptoms occur. Surgery certainly has no role unless symptoms occur that are unequivocally related to the presence of the catheter tip. Number: 709 Systemic absorption of local anaesthetic occurs most rapidly following which of the following techniques? A. Femoral block. B. Caudal block. C. Lumbar epidural block. D. Axillary block. E. Intercostal block. Select the single best answer ABCDE Correct Answer: E The plasma level of local anaesthetic peaks most rapidly following intercostal blockade. The order in which plasma levels peak (from most rapid to most delayed) is probably: Intercostal > Caudal > Epidural > Axillary > Femoral. See, for example: Tucker GT, Moore DC, Bridenbaugh PO, Bridenbaugh LD, Thompson GE. Systemic absorption of mepivacaine in commonly used regional block procedures. Anesthesiology. 1972 Sep;37(3):277-87. Number: 711 The intravenous injection of 15 micrograms of adrenalin (the amount contained in a typical 'test dose' for an epidural) to an awake patient will result in an increase in pulse rate of: A. 0 beats per minute. B. 5 beats per minute. C. 10 beats per minute. D. 30 beats per minute. E. 50 beats per minute. Select the single best answer ABCDE Correct Answer: D The purpose of epidural test dosing is to exclude inadvertent subarachnoid or intravascular injection.

544

3 mL of a local anaesthetic solution with adrenalin (1:200,000) contains 15 micrograms of adrenalin. This amount given intravenously produces, on average, a 30 beats per minute increase in heart rate 20 to 40 seconds after injection. It must be understood that this test is not reliable. Patients who are beta-blocked or pacemaker-dependent may not show any increase in heart rate at all and the test may also be unreliable in the active stages of labour. Similarly, general anaesthesia significantly modifies the response to the intravenous injection of small amounts of adrenalin. See also : Mackie K, Lam A: Epinephrine-containing test dose during beta-blockade. J Clin Monit 7:213, 1991 Moore D, Batra M: The components of an effective test dose prior to epidural block. Anesthesiology 55:693, 1981 Number: 718 With regard to a retrobulbar block: 1. Retrobulbar haemorrhage occurs in about 5% of cases. 2. Globe perforation occurs in about 0.1% of cases. 3. Temporary loss of vision in the eye is unlikely to occur. 4. Institution of the block can provoke the oculo-cardiac reflex. A: 1,2,3 Correct B: 1,3 Correct C: 2,4 Correct D: 4 Correct E: All Correct ABCDE Correct Answer: D Retrobulbar haemorrhage is a serious complication which occurs in 0.1-1.7% of patients. Vascular or haematological disease may be a predisposing factor. It is also more likely in the elderly, and in those receiving steroids, aspirin or other non-steroidal anti-inflammatory drugs, and anticoagulants. See, for example: Edge KR, Nicoll JM. Retrobulbar hemorrhage after 12,500 retrobulbar blocks. Anesth Analg. 1993 May;76(5):1019-22.

545

There is a body of opinion that supports the use of eye blocks in patients who are receiving anticoagulants within the therapeutic range and who consider that the risk of haemorrhagic complications is less than the risk of systemic problems if the anticoagulants are stopped. The insertion of a needle into (penetration) or through the globe (perforation) are rare complications which are more likely to occur in myopic eyes which are longer and thinner. The length of the globe should be known as part of the preparation for lens implant surgery, and globes longer than 26 mm are particularly at risk. Patients who have had or are presenting for retinal detachment surgery or for surgery of short-sightedness are likely to have long eyes. Perforation may occur with both peribulbar and retrobulbar blocks. In one study it was noted that 12 of 23 referred perforations followed retrobulbar injections and the other 11 followed peribulbar injections. It is more likely when there is poor patient co-operation or difficult access to the conjunctival fornix, and may occur whether the block is performed by an ophthalmologist or non-ophthalmologist. It is essential that anyone performing regional anaesthesia for the eye should have extensive knowledge of orbital anatomy and be fully aware of the risks of globe perforation. The incidence described in the literature varies from 0 in 2000 peribulbar blocks, to 1 in 12000 of a series of peribulbar and retrobulbar blocks, 3 in 4000 retrobulbar blocks, and 1 in 16224 peribulbar blocks. Peribulbar injections of local anaesthetic do not lead to temporary loss of vision, whereas retrobulbar injections cause this as a consequence of the optic nerve becoming blocked. Patients should be warned of the possibility of loss of light perception, especially if a block is being performed on the only functioning eye, and reassured that even if light perception is retained, it does not imply that the block is not effective and that they should not be worried by the microscope light or movements that they might see. It is well-recognised that institution of a retrobulbar block can occasionally provoke the oculo-cardiac reflex. See: Complications of local anaesthesia for ophthalmic surgery. Rubin AP; Br. J. Anaesth. 1995; 75:93-96. For a complete review of the complications of peribulbar block see: Davis DB 2nd, Mandel MR. Efficacy and complication rate of 16,224 consecutive peribulbar blocks. A prospective multicenter study. J Cataract Refract Surg. 1994 May;20(3):327-37. Number: 719 Shortly after decannulation following cardiopulmonary bypass (CPB) and coronary artery grafting, a 65 year old man develops a supraventricular tachycardia (SVT) at 180 bpm. His blood pressure falls from 120/70 mm Hg to 65/40 mm Hg and his arterial saturation (on 100% oxygen) falls from 98% to 93%. The most appropriate immediate response is: A. Re-cannulation and return to CPB. B. Esmolol 20 mg IV. C. Verapamil 5 mg IV.

546

D. Adenosine 6 mg IV. E. Cardioversion. Select the single best answer ABCDE Correct Answer: E The most appropriate immediate response is cardioversion. This should be immediately followed by an assessment of the cause of the SVT. - In particular conditions such as hypokalaemia, hypomagnesaemia, acid-base disturbance, hypoxia or hypercarbia and ongoing myocardial ischaemia must be excluded. Re-cannulation and return to CPB can often take longer than you might think and is not without risk - particularly if the CPB circuit has been completely or partially de-primed. In other clinical situations adenosine and verapamil could reasonably be used in the treatment of SVT, but they are inappropriate when immediate cardioversion facilities are available. Beta blockers should be regarded as 'second line' therapy for SVT. Number: 722 Which of the following complications of a deep cervical plexus block is LEAST likely to occur? A. Recurrent laryngeal nerve block. B. Phrenic nerve block. C. Accessory nerve block. D. Facial nerve block. E. Horner's syndrome. Select the single best answer ABCDE Correct Answer: D Recognised complications of cervical plexus block include failure, intravertebral artery injection, recurrent laryngeal nerve block, Horner's syndrome, subarachnoid injection, accessory nerve block, phrenic nerve block and facial nerve palsy. Of these complications, facial nerve palsy is the most uncommon. If such a block occurs during the course of carotid endarterectomy, it may be necessary to distinguish between peripheral facial nerve blockade and a central (embolic) cause of facial weakness. If the patient is able to wrinkle his or her forehead a central cause for facial paralysis is much more likely.

547

See: Temporary Facial Nerve Palsy During Carotid Endarterectomy Under Local Anesthesia. Szocik JF, Kellogg W, Wakefield T W, Anesth Analg 1995; 81:1106-7 Number: 739 A 67-kg diabetic patient undergoes a 3-hr craniotomy during which time he receives 350mls of a 20% mannitol solution and 2000 ml of Hartmann's solution. His intraoperative urine output is 800 ml and he passes a further 1000 ml during the first 30 minutes in the recovery room. Which of the following causes of polyuria may be relevant in this case? 1. Hyperglycaemia. 2. Nephrogenic diabetes insipidus. 3. Mannitol-induced osmotic diuresis. 4. Excessive fluid administration intraoperatively. A: 1,2,3 Correct B: 1,3 Correct C: 2,4 Correct D: 4 Correct E: All Correct ABCDE Correct Answer: B Drugs commonly used during anaesthesia do not cause nephrogenic diabetes insipidus. (By far the most common cause is lithium). Fluid administration does not appear to have been excessive during this case. Number: 758 The continuous application of -10 cms H2O pressure via an underwater seal drain is ABSOLUTELY contraindicated: A. Following a lobectomy. B. Following a pneumonectomy. C. In a patient with mesothelioma. D. In a patient with a broncho-pleural fistula. E. Following the removal of a pulmonary hydatid cyst. Select the single best answer ABCDE Correct Answer: B

548

The role of the chest drain following pneumonectomy is NOT the same as in any other form of thoracic surgery. In this situation, the main role of the drain is to allow the physician to centralise the mediastinum if necessary. The drain should be continuously clamped following pneumonectomy and only unclamped for a few seconds during quiet respiration while connected to an underwater drainage system which is not pressurised in any way. The application of a subatmospheric pressure to the drain will swing the mediastinum to the side of the pneumonectomy and may cause severe cardiovascular disturbance. Number: 759 An otherwise fit 20 year old is undergoing thoracoscopic pleurodesis for the treatment of a recurrent right-sided pneumothorax. The patient is anaesthetised and intubated with a leftsided double lumen tube (DLT). The patient is paralysed with rocuronium and ventilated with sevoflurane in oxygen. 5 minutes after one lung ventilation is initiated, the saturation is noted to have fallen from 100 to 92%. The most appropriate IMMEDIATE management of this situation is: A. Insufflation of oxygen into the non-dependent lung. B. Application of positive end-expiratory pressure to the non-dependent lung. C. Performance of arterial blood gas analysis. D. Confirmation of the correct positioning of the DLT. E. Re-ventilation of the non-dependent lung. Select the single best answer ABCDE Correct Answer: D The message is that mal-positioning of the DLT should always be suspected if oxygenation during one-lung anaesthesia is not as good as you expected. Bronchoscopy is the definitive method for confirming DLT positioning, but in my experience this is certainly not always necessary. Modern PVC tubes tend to advance in the bronchus as they warm and soften and ventilation of the left upper lobe bronchus may become impaired as this occurs. Under these circumstances simply withdrawing the tube a centimetre or two and gently manually ventilating for a few breaths may restore oxygenation. Once the correct position of the tube has been confirmed other forms of therapy and / or investigation for a relative failure of oxygenation can be undertaken. Number: 771 The incidence of cardiac arrest during spinal anaesthesia is:

549

A. More than 1 in 500. B. Between 1 in 500 and 1 in 2000. C. Between 1 in 2000 and 1 in 5000. D. Between 1 in 5000 and 1 in 10000. E. Less than 1 in 10000. Select the single best answer ABCDE Correct Answer: B Cardiac arrests during spinal anesthesia are thought to be rare but are, in fact, quite common. The two largest prospective studies designed to evaluate the incidence of complications during spinal anaesthesia reported two arrests in 1881 patients (Tarkkila PJ, Kaukinen S) and 26 arrests in 40,640 patients (Chopra et al) for an overall incidence of seven arrests for every 10,000 (0.07%) spinal anaesthetics. Reg Anesth 1991 Mar-Apr;16(2):101-6. Complications during spinal anesthesia: a prospective study. Tarkkila PJ, Kaukinen S. Anaesthesia 1990 Jan;45(1):3-6. Accidents, near accidents and complications during anaesthesia. A retrospective analysis of a 10-year period in a teaching hospital. Chopra V, Bovill JG, Spierdijk J An excellent review of the problem has recently been published by Pollard. (See: Anesth Analg 2001;92:252-256; Cardiac Arrest During Spinal Anesthesia: Common Mechanisms and Strategies for Prevention. Pollard JB). Although it is possible to implicate all sorts of mechanisms in the aetiology of such arrests, Pollard is of the view that the combination of high vagal tone and reduced preload is of particular importance. Number: 774 The oculo-cardiac reflex: 1. Is mediated by the trigeminal and vagus nerves. 2. Is more common if propofol as compared to sevoflurane is used for anaesthesia 3. Can be abolished by intravenous atropine. 4. Is more likely to occur in the presence of hypercarbia. A: 1,2,3 Correct B: 1,3 Correct C: 2,4 Correct D: 4 Correct E: All Correct ABCDE

550

Correct Answer: A The reflex is trigeminovagal. The afferent limb is from orbital contents to ciliary ganglion, to ophthlalmic division of the trigeminal nerve, to sensory nucleus of the trigeminal nerve near the fourth ventricle. Numerous studies have shown that propofol appears to be a particular risk factor for facilitating the reflex. (See, for example, Gurkan et al). The reflex can be abolished by intravenous atropine. Blanc et al originally suggested that hypercarbia pre-disposed to the condition, but more recently Mirakhur found that hypercarbia had no such effect. References: Gurkan Y, Kilickan L, Toker K. Propofol-nitrous oxide versus sevoflurane-nitrous oxide for strabismus surgery in children. Paediatr Anaesth. 1999;9(6):495-9. Blanc VF, Hardy JF, Milot J, Jacob JL. The oculocardiac reflex: a graphic and statistical analysis in infants and children. Can Anaesth Soc J 1983; 30: 3609. Mirakhur RK, Shepherd WFI, Jones CJ. Ventilation and the oculocardiac reflex: prevention of oculocardiac reflex during surgery for squints: the role of controlled ventilation and anticholinergic drugs. Anaesthesia 1986; 41: 8258. Number: 823 A 65 year old man presents for a right upper lobectomy. He has signs and symptoms consistent with moderate obstructive lung disease. 5 years ago he suffered a myocardial infarction which was complicated by transient atrial fibrillation. He is now asymptomatic from a cardiological viewpoint. As part of your anaesthetic you insert a thoracic epidural catheter at T4-5 with the intention of using it for post-operative pain relief by the continuous infusion of 0.125% bupivacaine containing 5 micrograms/ml of fentanyl. The use of thoracic epidural analgesia (TEA) in this manner is likely to: 1. Reduce the incidence of post-operative ventricular arrhythmias. 2. Provide better post-operative pain relief than a continuous paravertebral infusion of bupivacaine. 3. Significantly impair the patient's ventilatory mechanics. 4. Reduce the incidence of post-operative supraventricular arrhythmias. A: 1,2,3 Correct B: 1,3 Correct C: 2,4 Correct D: 4 Correct E: All Correct ABCDE

551

Correct Answer: D 1. There is no good evidence that the incidence of post-operative ventricular arrhythmias is reduced by the use of peri-operative TEA. In contrast, the incidence of supraventricular arrhythmias is definitely reduced. See, for example, Oka T, Ozawa Y,Ohkubo Y: Anesth Analg 2001;93:253-259. Thoracic Epidural Bupivacaine Attenuates Supraventricular Tachyarrhythmias After Pulmonary Resection. 2. Perhaps surprisingly, continuous paravertebral analgesia (CPA) is a remarkably effective form of post-operative analgesia following thoracotomy. Richardson et al recently compared CPA with TEA and found "Significantly lower visual analogue pain scores at rest and on coughing in the paravertebral group" and that "Pulmonary function was significantly better preserved in the paravertebral group who had higher oxygen saturations and less postoperative respiratory morbidity". See: Richardson J, Sabanathan S, Jones J et al: Br J Anaesth 1999 Sep;83(3):387-92. A prospective, randomized comparison of preoperative and continuous balanced epidural or paravertebral bupivacaine on post-thoracotomy pain, pulmonary function and stress responses. 3. There is good evidence that epidural infusions of dilute bupivacaine do not significantly impair ventilatory function. See, for example, Gruber EM, Tschernko EM, Kritzinger M et al: Anesth Analg 2001 Apr;92(4):1015-9. The effects of thoracic epidural analgesia with bupivacaine 0.25% on ventilatory mechanics in patients with severe chronic obstructive pulmonary disease.

Number: 824 A 65 year old man presents for a right upper lobectomy. He has signs and symptoms consistent with moderate obstructive lung disease. You have anaesthetised him with propofol, paralysed him with pancuronium and passed a 39 F left-sided endobronchial tube. You have confirmed the correct position of the tube bronchoscopically. Anaesthesia is maintained with enflurane in oxygen. After 5 minutes of one-lung ventilation, you note that his arterial saturation has fallen from 97% to 91%. You confirm the integrity of the circuit, confirm that the FiO2 is 1.0 and ventilate him by hand. The most appropriate next response to this situation is to: A. Apply 3 cms of Positive End-Expiratory Pressure to the unventilated lung. B. Apply 5 cms of Positive End-Expiratory Pressure to the ventilated lung. C. Change the volatile agent from enflurane to isoflurane. D. Check the position of the endobronchial tube. E. Re-inflate the unventilated lung. Select the single best answer ABCDE Correct Answer: D

552

You must confirm the correct placement of the tube. - Remember that PVC softens considerably as it warms up and the tip of the tube may migrate a considerable distance (several cms). - In my experience, usually inwards. Once you have confirmed tube position, you can try some of the other options. Refer to the excellent web article entitled "The patient intolerant of one-lung ventilation." by Peter Slinger for more information on the subject. Number: 828 Of the complications listed below, that most likely to complicate mediastinscopy is: A. Phrenic nerve palsy. B. Haemorrhage. C. Recurrent laryngeal nerve palsy. D. Air embolism. E. Pneumothorax. Select the single best answer ABCDE Correct Answer: B According to Cybulsky and Bennett the most common complication is the occurrence of a supraventricular arrhythmia. See: Cybulsky IJ, Bennett WF. Mediastinoscopy as a routine outpatient procedure. Ann Thorac Surg. 1994 Jul;58(1):176-8. However, of those listed, the most common (and important) is probably haemorrhage because if it occurs, it is often difficult to treat and may develop into a major catastrophe (the pulmonary artery is the vessel most commonly damaged). The incidence appears to be around 0.75%. In his series of 324 cases, Urschel stated that "Major hemorrhage was an uncommon complication of mediastinoscopy. Although there is a paucity of published data on the management of this problem, sternotomy and arterial repair is usually considered the treatment of choice. Two major hemorrhagic complications (1 definite and 1 possible pulmonary artery tear) occurred. Mediastinoscopic gauze packing successfully controlled the hemorrhage." In my experience as an anaesthetist (~300 cases), the three cases of major haemorrhage with which I have been involved have all proceeded to median sternotomy. The incidence of recurrent laryngeal nerve palsy is variously quoted at between 0.3 and 0.7%, while pneumothorax complicates between 0.2 and 0.6% of cases. Air embolism and phrenic nerve palsy have also been reported but are extremely uncommon. Footnote: In a recent report by Hujala, the incidence of recurrent laryngeal nerve palsy was about 2%.

553

Number: 840 Which of the following nerves will NOT be blocked by a correctly placed retrobulbar block? 1. Lacrimal nerve. 2. Frontal nerve. 3. Trochlear nerve. 4. Ciliary ganglion. A: 1,2,3 Correct B: 1,3 Correct C: 2,4 Correct D: 4 Correct E: All Correct ABCDE Correct Answer: A

A correctly placed retrobulbar injection of local anaesthetic is intended to block the 'Intraconal' nerves. The 'extra-conal' nerves are the lacrimal, frontal and trochlear nerves. Number: 841 A correctly placed retrobulbar block will block all of the following nerves EXCEPT the: A. Nasociliary nerve. B. Lacrimal nerve. C. Optic nerve. D. Abducent nerve. E. Ciliary nerves. A, C, & E are correct. (Ref: BJA 1995; 75:88-92 & Cousins 1980 p.457-458) Nerves within the cone are blocked Select the single best answer ABCDE Correct Answer: B A correctly placed retrobulbar injection of local anaesthetic is intended to block the 'Intraconal' structures. ie the ciliary ganglion, nasociliary and ciliary nerves, and cranial nerves II, III and VI.

554

Cranial nerve IV is not affected since it lies outside the muscle cone. When the block is performed, the local anaesthetic is delivered within the muscle cone itself. The ciliary ganglion, a parasympathetic ganglion, lies approximately 1 cm from the posterior boundary of the orbit between the lateral surface of the optic nerve and the ophthalmic artery. Parasympathetic fibers originating in the oculomotor nerve and postganglionic fibers supply the ciliary body and pupillary sphincter muscles. The nasociliary nerve, a branch of the ophthalmic nerve, supplies sensory innervation of the cornea, iris, and ciliary body by way of the short ciliary nerves (these short ciliary nerves are 6-10 small filaments that run with the ciliary arteries). Number: 852 A patient with a clinically 'normal' cardiac output undergoing coronary artery grafting is heparinised with 350 U/kg of bovine lung heparin administered via a central venous line. The patient will be effectively heparinised within: A. 60 secs. B. 90 secs. C. 120 secs. D. 300 secs. E. 600 secs. Select the single best answer ABCDE Correct Answer: A A recent, well-designed study suggest that heparin effectively anticoagulates patients within a single circulation time. This is in contrast to earlier work. See: Heres EK, Speight K, Benckart D, Marquez J, Gravlee GP. The clinical onset of heparin is rapid. Anesth Analg. 2001 Jun;92(6):1391-5. These authors concluded that "In summary, we have shown that, when a large bolus of heparin is administered into the central venous circulation, the onset of peak ACT prolongation in systemic arteries is almost immediate (<30 seconds), and that it is nearly that rapid in systemic veins. Consequently, in the presence of circulatory stability, it appears acceptable to initiate such procedures as vascular cross-clamping and cannulation of the central circulation within 60 seconds of heparin administration." Number: 859 A 70 year old man has recently undergone a left pneumonectomy which has been complicated by the development of a broncho-pleural fistula. At the time of the pneumonectomy he was in otherwise good health and had no significant co-morbidities. You note that he is now

555

tachypnoeic at rest and coughing up copious amounts of watery, blood stained sputum. He has a left sided chest drain in position which is bubbling constantly. With regard to induction of anaesthesia, you are contemplating either a gaseous induction with maintenance of spontaneous ventilation until the airway is secured ('G') or an intravenous induction with a small dose of thiopentone and suxamethonium ('IV'). You have both right ('R') and left ('L') sided double lumen ('DL') and 'Univent' ('U') tubes available. Given this scenario, the most appropriate induction technique is: A. G-L-DL B. G-R-DL C. IV-L-DL D. IV-R-DL E. G-U Select the single best answer ABCDE Correct Answer: B The airway issues here include: 1. The requirement to protect the right lung from soiling. 2. The fact that surgery may involve the L main bronchial stump. 3. The presence of a large air leak. 4. The possibility that insertion of an airway device into the bronchial stump may make the fistula even larger. A left-sided DLT is contraindicated because of the proximity of the fistula to the bronchial lumen. Similarly, a Univent tube is also contraindicated. The use of muscle relaxants before successful splitting of the lungs is also relatively contraindicated in the presence of a large air leak. (Having said this, many current thoracic anaesthetists would be prepared to use suxamethonium in this circumstance.) Number: 872 Factors which tend to ENHANCE the spread of a local anaesthetic (LA) administered epidurally by the lumbar route include: 1. Administration to a patient with a large body-mass index. 2. Administration to an elderly patient. 3. Increasing the dose of the local anaesthetic. 4. The addition of fentanyl to the LA. A: 1,2,3 Correct B: 1,3 Correct C: 2,4 Correct D: 4 Correct

556

E: All Correct ABCDE Correct Answer: E See: Acta Anaesthesiol Scand 1994 Oct;38(7):646-52. A multifactorial analysis of the spread of epidural analgesia. Curatolo M, Orlando A, Zbinden AM, Scaramozzino P, Venuti FS. These authors studied a group of over 800 (non-pregnant) patients undergoing lumbar epidural anaesthesia and concluded that "Spread significantly increases with increasing age, weight, body-mass index, dose of local anaesthetic, addition of fentanyl, higher site of injection, and decreasing body height.". Number: 873 With regard to epidural blood-patching (EBP) for post-dural puncture headache (PDPH), complete symptomatic relief is likely to be achieved in: A. Over 90% of cases. B. 80 - 90% of cases. C. 70 - 80% of cases. D. 60 - 70% of cases. E. Less than 60% of cases. Select the single best answer ABCDE Correct Answer: C Safa-Tisseront et al studied a series of 504 patients with PDPH, and found that EBP was a completely effective treatment for symptoms of CSF leak after dura mater puncture in 75% of cases. There was a 7% (complete) failure rate. See: Anesthesiology 2001 Aug;95(2):334-9. Effectiveness of epidural blood patch in the management of post-dural puncture headache. Safa-Tisseront V, Thormann F, Malassine P, Henry M, Riou B, Coriat P, Seebacher J. Number: 874 Factors which are predictive of FAILURE of epidural blood-patching (EBP) for post-dural puncture headache (PDPH) include: 1. A female patient. 2. Delay in the administration of the EBP. 3. The presence of neck pain.

557

4. Dural puncture with a large (<20G) needle. A: 1,2,3 Correct B: 1,3 Correct C: 2,4 Correct D: 4 Correct E: All Correct ABCDE Correct Answer: D See: Safa-Tisseront V, Thormann F, Malassine P, Henry M, Riou B, Coriat P, Seebacher J. Effectiveness of epidural blood patch in the management of post-dural puncture headache. Anesthesiology. 2001 Aug;95(2):334-9. These authors studied a series of 504 patients with PDPH. Females were more likely to suffer PDPH than males, but EBP was no less effective in the treatment of the headache. The presence of neck pain was predictive of only a partial cure of symptoms, but not of failure. Dural puncture with a large (<20G) needle significantly increased the likelihood of failure of EBP. Surprisingly, delay in the administration of the EBP did NOT increase the likelihood of failure. - In fact the reverse was true, in that the percentage of failure of EBP was significantly increased when EBP was performed within 3 days after dural puncture. - They surmised that "Symptoms are more severe as the CSF leak is greater, and it is likely that patients experiencing more pain were treated earlier because of the severity of their symptoms. This suggests that the diminished effectiveness of early EBP was more related to the size of the dural puncture and the severity of the CSF leak than with the fact of not delaying EBP. " Number: 892 The use of regional anaesthesia for major thoracic, abdominal, or lower limb surgery will significantly reduce the incidence of: 1. Peri-operative mortality. 2. Transfusion requirement. 3. Deep venous thrombosis. 4. Peri-operative stroke. A: 1,2,3 Correct B: 1,3 Correct C: 2,4 Correct D: 4 Correct

558

E: All Correct ABCDE Correct Answer: A See the important meta-analysis by Rodgers et al. These authors found that "Overall mortality was reduced by about a third in patients allocated to neuraxial blockade. Neuraxial blockade also reduced the odds of deep vein thrombosis by 44%, pulmonary embolism by 55%, transfusion requirements by 50%, pneumonia by 39%, and respiratory depression by 59%." Rodgers A, Walker N, Schug S, McKee A, Kehlet H, van Zundert A, Sage D, Futter M, Saville G, Clark T, MacMahon S. Reduction of postoperative mortality and morbidity with epidural or spinal anaesthesia: results from overview of randomised trials. BMJ. 2000 Dec 16;321(7275):1493. See, also the meta-analysis by Beattie, Badner and Choi on the impact of epidural analgesia on the peri-opertaive myocardial infarction rate. Beattie WS, Badner NH, Choi P. Epidural analgesia reduces postoperative myocardial infarction: a meta-analysis. Anesth Analg. 2001 Oct;93(4):853-8. Number: 915 The adverse effects of intrathecal pethidine include: 1. Pruritus. 2. Hypotension. 3. Sedation. 4. Bradycardia. A: 1,2,3 Correct B: 1,3 Correct C: 2,4 Correct D: 4 Correct E: All Correct ABCDE Correct Answer: E The most recent review of the use of intrathecal pethidine is that by Kee. According to this author: "Adverse effects that are associated with intrathecal pethidine include nausea and vomiting, pruritus, sedation and respiratory depression. Several comparative studies have shown that the incidence of these side-effects is greater with pethidine compared with conventional local anaesthetics although this may be dose-dependent."

559

"Intrathecal pethidine 1 mg/kg caused reductions in mean arterial pressure, right atrial pressure, mean pulmonary artery pressure and pulmonary capillary wedge pressure." "Bradycardia after intrathecal pethidine has been observed in a number of reports. Conway et al reported that bradycardia requiring atropine occurred in six of 14 (43%) elderly patients who received pethidine 0.8 mg/kg for urological surgery and four of 14 (29%) who received a mixture of pethidine 0.4 mg/kg plus bupivacaine 7.5 mg. Patients in that study did not receive intravenous fluid preload and bradycardia only occurred when the upper level of the block was T7 or higher. The incidence of bradycardia in other reports has ranged from 0 to 20%. " See: Ngan Kee WD. Intrathecal pethidine: pharmacology and clinical applications. Anaesth Intensive Care. 1998 Apr;26(2):137-46. Conway F, Critchley LA, Stuart JC, Freebairn RC. A comparison of the haemodynamic effects of intrathecal meperidine, meperidine-bupivacaine mixture and hyperbaric bupivacaine. Can J Anaesth. 1996 Jan;43(1):23-9. Number: 937 An adult has undergone an aortic valve replacement and coronary artery grafting. Immediately before separation from bypass, his ECG is as shown. The surgeon has implanted epicardial, dual chamber pacing leads. The most appropriate mode setting for the temporary pacemaker at this time is: A. DDD B. DOO C. VOO D. VVI E. AOO Select the single best answer

ABCDE Correct Answer: B DOO - At this stage of the procedure, further diathermy is inevitable, for this reason, an obligatory mode must be selected (otherwise the pacemaker will be inhibited). The patient is in complete heart block - therefore AOO is completely inappropriate.

560

VOO is acceptable, but the patient will be deprived of the benefit of the atrial systolic contribution to cardiac output. Number: 948 A left-sided double lumen tube (DLT) is absolutely contra-indicated in a patient scheduled for: A. Left lower lobectomy. B. Left pneumonectomy. C. Repair of broncho-pleural fistula following left pneumonectomy. D. Repair of broncho-pleural fistula following right pneumonectomy. E. None of the above. Select the single best answer ABCDE Correct Answer: C A left-sided double lumen tube is absolutely contra-indicated in the presence of significant endoluminal disease of the left main bronchus. Left pneumonectomy is a relative contraindication to the use of a left-sided tube - because the tube will have to be withdrawn before the bronchus is transected. Very little appears to have been written about contra-indications to the DLT, and the opinion expressed above is my own. Campos has recently reviewed lung isolation techniques. See: Campos JH. Lung isolation techniques. Anesthesiol Clin North America. 2001 Sep;19(3):45574. Number: 953 You have anaesthetised a 65 year old non-insulin dependent diabetic for emergency coronary artery grafting. The patient is ventilated with sevoflurane in oxygen. The registar has heparinised the patient with 400 IU/kg of heparin. Immediately before cannulation, arterial blood samples are drawn for measurment of Activated Clotting Time, blood gas analysis and blood sugar measurement. Which of the following results would demand your most urgent attention? A. Blood Sugar: 13.7 mmol/L (250 mg/dL). B. Arterial PO2: 100 mm Hg. C. Base Excess: - 10. D. Activated Clotting Time: 130 seconds. E. Haemoglobin 80 G/L.

561

Select the single best answer ABCDE Correct Answer: D The Activated Clotting Time of 130 seconds is incompatible with heparinisation. The surgeon must not be allowed to cannulate and bypass must not be initiated until the situation has been clarified. - Either the wrong drug was given, the drug did not reach the circulation or a measurement error has occurred. Although the other results are clearly abnormal (in particular, early diabetic ketoacidosis should be excluded) these can wait until the issue of effective anti-coagulation is resolved. Number: 969 The percentage of patients who will report chronic, post-thoracotomy pain one year after surgery is: A. Less than in 1%. B. 1 - 9% C. 10 - 29%. D. 30 - 49%. E. 50% or more. Select the single best answer ABCDE Correct Answer: E Perttunen et al have recently remarked that "The incidence of long-term post-thoracotomy pain is reported to be up to 67%. A relationship between the severity of acute postoperative pain and the development of chronic post-thoracotomy pain has been suggested." and that "A significant proportion of patients undergoing thoracotomies will suffer from chronic pain. Surgeons and anaesthetists should be aware of this fact and they should look for effective means of preventing and treating this pain syndrome." In their own study of one hundred and ten patients, they found an incidence of 61% and in 35% of cases pain was described as severe. See: Perttunen K, Tasmuth T, Kalso E. Chronic pain after thoracic surgery: a follow-up study. Acta Anaesthesiol Scand. 1999 May;43(5):563-7. Number: 982 With regard to a supraclavicular block:

562

A. An interscalene block is more suitable for a patient with severe respiratory disease. B. The volume of solution required for effective blockade is more than for a block by the axillary route. C. The incidence of pneumothorax is of the order of 8 %. D. Cervical Sympathetic blockade is likely to occur. E. All of the above. Select the single best answer ABCDE Correct Answer: D Interscalene block in fact carries an almost universal incidence of diaphragmatic paralysis on the same side. For this reason, it is contraindicated in those patients who would not be able to tolerate unilateral loss of diaphragmatic function. The risk of pneumothorax with the supraclavicular approach makes it debatable whether either technique is appropriate for the patient with severe respiratory disease. Supraclavicular blockade is performed at the most compact point in the plexus and is therefore the most efficient approach to brachial plexus blockade with regard to local anaesthetic usage. The published incidence of pneumothorax is in the region of 0.5 - 6.0 %, but cervical sympathetic blockade (as evidenced by a Horner's Syndrome) occurs in at least 50 % of cases. The following is the abstract from a recent article dealing with the issue of respiratory function following supraclavicular blocks: " We studied the effects of unilateral hemidiaphragmatic paresis caused by interscalene brachial plexus block on routine pulmonary function in eight patients. In an additional four patients, we studied changes in chest wall motion during interscalene block anesthesia by chest wall magnetometry. Ipsilateral hemidiaphragmatic paresis, as diagnosed by ultrasonography, developed in all patients within 5 min of interscalene injection of 45 mL of 1.5% mepivacaine with added epinephrine and bicarbonate. Large decreases in all pulmonary function variables were measured in every patient. Forced vital capacity and forced expiratory volume at 1 s decreased 27% +/- 4.3% and 26.4% +/- 6.8%, respectively (P = 0.0001). Peak expiratory and maximum midexpiratory flow rates were also significantly reduced. Interscalene block caused changes in pulmonary function and chest wall mechanical motion that were similar to those published in previous studies on patients with hemidiaphragmatic paresis of pathological or surgical etiology. Interscalene block probably should not be performed in patients who are dependent on intact diaphragmatic function and in those patients unable to tolerate a 25% reduction in pulmonary function......" References: URMEY. W.F. & MCDONALD, M; " Hemidiaphragmatic paresis during interscalene brachial plexus block: effects on pulmonary function and chest wall mechanics. ", Anesth Analg, vol 74, no 3, 1992 Mar; pp 352-7.

563

Number: 994 A correctly placed, left-sided double lumen tube in an adult is most likely to show which of the following centimetre depth markers at the incisors? A. 23 B. 25. C. 29. D. 33. E. 35 Select the single best answer ABCDE Correct Answer: C In a group of 240 patients undergoing left-sided endobronchial intubation, Chow et al found that in about 65% of cases the DLT was inserted to between 27 and 30 cms when correctly positioned. The modal distance was 29 cms and the mean 28 cms. See: Chow MY, Goh MH, Ti LK. Predicting the depth of insertion of left-sided double-lumen endobronchial tubes. J Cardiothorac Vasc Anesth. 2002 Aug;16(4):456-8.

RELAXANT AND LOCAL ANESTHETICS PHARMACOLOGY TESTS Number: 10 Which of the following relaxants are likely to be potentiated in a patient on cyclophosphamide? 1. Suxamethonium. 2. Cisatracurium. 3. Mivacurium. 4. Rapacuronium. A: 1,2,3 Correct B: 1,3 Correct C: 2,4 Correct D: 4 Correct E: All Correct ABCDE Correct Answer: B

564

Cyclophosphamide inhibits the activity of numerous cholinesterases and has been reported to potentiate the effects of both suxamethonium and mivacurium. Cisatracurium is largely hydrolysed by non-enzymatic (Hofmann) degradation. Rapacuronium is an aminosteroid, nondepolarising neuromuscular blocking agent and is not hydrolysed by cholinesterases. Number: 86 Which of the following statements are true with regard to local anaesthetic toxicity? 1.A predictable relationship between plasma concentration and toxic effect exists for both lignocaine and bupivacaine. 2. Bretylium is a useful antiarrhythmic in bupivacaine-induced ventricular tachycardia. 3. Lignocaine potentiates the cardiotoxicity of bupivacaine. 4. Convulsions can be expected with lignocaine at 10-12 ug/ml. A: 1,2,3 Correct B: 1,3 Correct C: 2,4 Correct D: 4 Correct E: All Correct ABCDE Correct Answer: C Toxicity associated with the systemic uptake of LA's predominantly involves CNS and CVS effects. For Lignocaine a reasonably predictable relationship exists between signs and symptoms of toxicity and plasma concentration (ug/ml) (perioral numbness-4; convulsions10-12; coma-16; respiratory arrest-20; CVS collapse-25).The ratio between CVS collapse and convulsions (CC/CNS ratio) is a useful index of the safety of the agent. Bupivacaine is considered a more toxic agent because it has a lower CC/CNS ratio as compared with Lignocaine (3.6 v 1.6 ) and it is less predictable. Moreover, toxicity parallels anaesthetic potency which is related to lipid solubility. Cardiotoxicity is a greater threat with Bupivacaine than Lignocaine. VT/VF is a real possibility with the former and is notoriously difficult to treat especially in the parturient. Bretylium has a role because of this, but is in itself not reliable. Lignocaine induced VT/VF has not been described in humans. Hypercarbia decreases protein binding, hence more drug is available to cross the BBB. WOOD, A. & WOOD, M; Drugs and Anesthesia, 2nd Ed.,1990. COUSINS ,M & BRIDENBOUGH; Neural Blockade, pp 121-8.

565

Number: 174 With regard to prilocaine: 1. It is a tertiary amine. 2. It has a shorter duration of action than lignocaine. 3. Its duration of action is increased with adrenaline. 4. Clinically detectable cyanosis can be expected in doses greater than 600 mg. A: 1,2,3 Correct B: 1,3 Correct C: 2,4 Correct D: 4 Correct E: All Correct ABCDE Correct Answer: D Prilocaine is an amide type local anaesthetic agent. Hence, it has structural similarities to lignocaine and mepivacaine; however, it is a secondary amine. It has a significantly decreased potential for producing systemic toxicity which is partly attributed to this. It requires no preliminary dealkylation, undergoing immediate enzymatic hydrolysis to ortho-toluidine and N-propylalamine. Recall that lignocaine undergoes N-dealkylation to yield monoethylglycinexylidide (MEGX) which, in turn, is followed by secondary N-dealkylation to give glycinexylidide (GX) before hydrolysis. Prilocaine also has larger volume of distribution. Its rapid metabolism and redistribution result in lower serum levels after intravenous anaesthesia or accidental intravascular injection. Prilocaine has a clinical profile which is similar to lignocaine. It has a relatively rapid onset of action, a moderate duration of action, and causes a profound depth of conduction blockade. In contrast, it causes significantly less vasodilation than lignocaine. As a result its duration of action is not increased by adrenaline. The duration of action of prilocaine without adrenaline is similar to that of lignocaine with adrenaline. The major problem associated with the use of prilocaine is its tendency to cause methaemoglobinaemia. Minor degrees of methaemoglobinaemia occasionally follow lignocaine, however, prilocaine is unique amongst the local anaesthetic agents for its ability to reduce the blood's oxygen carrying capacity to cause clinically detectable cyanosis. Methaemoglobin (metHb) exists physiologically in a concentration of about 1% of total haemoglobin. This is because the ferrous porphyrin component of haem is continuously being oxidized to the ferric form. However, a specific reductase continuously reduces it back to the functional ferrous form. Cyanosis does not become clinically detectable until 5-6% of the patient's haemoglobin is in the ferric state. A total dose of 300 mg of prilocaine will produce a metHb of 1.9%, while a dose of 600 mg will produce a level of 5.3%. Thus, by limiting the total dose to 600 mg, cyanosis can be avoided. Note that in healthy individuals, enough to cause any compromise in oxygenation, but this may occur in patients whose oxygen carrying capacity is already reduced. It is not

566

recommended in obstetrics, as the cyanosis caused by prilocaine may make exclusion of neonatal cyanosis from other causes difficult. The following is the abstract ( Bartholomew) from an editorial entitled: Prilocaine for Bier's block: how safe is safe? " Prilocaine has become the agent of choice for Bier's block (or intravenous regional anaesthesia--IVRA), since 1983 when the product licence of bupivacaine was withdrawn for this purpose owing to fatal or serious complications. No serious complications have been documented in the literature relating to prilocaine in IVRA and we have conducted a survey within the U.K. which indicates that about 45,000 Bier's blocks have been carried out with prilocaine without convulsion, arrhythmia or fatality. This includes cases of accidental cuff deflation or even failure to inflate the cuff, resulting in bolus doses to the circulation. Prilocaine has now been in use since 1964 and the Committee for Safety of Medicines has no deaths on record over a 25-year period. (This includes other types of regional anaesthesia). We suggest that intravenous regional anaesthesia using prilocaine is a safe technique. It is highly unlikely that fatalities will occur, provided present guidelines are adhered to. We can find no reason to limit its use to trained anaesthetists only, but would recommend that a strict protocol is adhered to. " References WINNIE, A.P.; Plexus Anesthesia, vol 1, Churchill Livingstone, 1984, p 224. COUSINS & BRIDENBAUGH; Neural Blockade, p138. WOOD, M. & WOOD, A.J.J.; Drugs and Anesthesia, 2nd Ed.,1990, p 332 BARTHOLOMEW, K. & SLOAN, J.P; Arch. Emerg. Med., vol7, no 3, 1990 Sep, pp 18995. Number: 175 With regard to EMLA* (Eutectic Mixture of Local Anaesthetics): 1. The melting point of a eutectic mixture is greater than that of the individual components 2. The components are lignocaine and prilocaine 3. The components are lignocaine and bupivacaine 4. It requires a minimum of 60 minutes to work A: 1,2,3 Correct B: 1,3 Correct C: 2,4 Correct D: 4 Correct E: All Correct ABCDE Correct Answer: C EMLA* is a 5% cream consisting of a eutectic mixture of 2.5% lignocaine and 2.5% prilocaine. A eutectic mixture is one which has a melting point lower than that of the individual components. When lignocaine and prilocaine are mixed together in base form, the crystalline powders interact to form a liquid at room temperature. This is important in the

567

preparation of oil-in-water emulsions as it obviates the need for dissolving the local anaesthetic base in an oil before the addition of an emulsifier. Thus, it is possible to increase the concentration of local anaesthetic in the emulsion droplets from approximately 20% (lignocaine) to 80% (lignocaine-prilocaine) in eutectic formulations, while still keeping the total concentration of local anaesthetic low (5%). EMLA* requires a minimum of 60 minutes of application under an occlusive bandage for anaesthetic effect. References POWELL, D.M. ET AL; "Damage to Tissue Defenses by EMLA* Cream", The Journal of Emergency Medicine, vol 9, 1991, pp 205-9. Number: 182 The constant infusion of Vecuronium for a period of more than 48 hours has been implicated in: 1. The persistence of residual weakness which may last 3-6 months. 2. An acute myopathy which becomes apparent as the neuromuscular junction (NMJ) returns to normal. 3. A cardiomyopathy in asthmatics who have received high dose steroids. 4. Prolonged NMJ failure in patients with renal failure. A: 1,2,3 Correct B: 1,3 Correct C: 2,4 Correct D: 4 Correct E: All Correct ABCDE Correct Answer: E Myopathy complicating the therapy of severe asthma has been recently described in several case reports. Twenty-five consecutive patients admitted to the intensive care unit (ICU) at the hospital cited in the reference below, for mechanical ventilation for severe asthma were studied for the incidence of creatine kinase (CK) enzyme rise and for the development of clinical myopathy. Pharmacologic therapy was standardized, every patient receiving corticosteroids and aminophylline intravenously and salbutamol both nebulized and intravenously. Twenty-two patients received muscle relaxant therapy with vecuronium. In 19 of 25 (76%) of patients there was elevation of CK levels to a median of 1,575 U/L (range, 66 to 7,430) occurring 3.6 +/- 1.5 days after admission. In nine patients there was clinically detectable myopathy. The presence of either myopathy or CK enzyme rise was associated with a significant prolongation of ventilation time. Arterial blood gas measurements on admission to the ICU revealed a pH (mean +/- SD) of 7.07 +/- 0.21, a PaCO2 of 87.2 +/- 32.7, and a PaO2 (with a high FIO2) of 129 +/- 97 mm Hg; however, no correlation was found

568

between the severity of initial metabolic disturbance and the subsequent development of myopathy. There was no association between the type of corticosteroid administered and the subsequent development of myopathy. Patients with myopathy had received a significantly higher total dose of vecuronium when compared with those who did not develop myopathy (p < 0.001, Kruskal Wallis test). The authors have therefore found a surprisingly high incidence of CK enzyme rise and myopathy in this group of mechanically ventilated patients with severe asthma. References: DOUGLASS, J.A. ET AL; Am Rev Respir Dis., vol 146, no 2, Aug 1992 , pp 517-9 .

Number: 184 Suxamethonium: 1. Is unlikely to cause muscle pains in a young, ambulant woman. 2. Causes a rise in serum potassium typically of the order of 0.5-1.0 mmol/L. 3. Causes an atypically large rise in serum potassium in patients with renal failure. 4. Causes a rise in serum potassium which is not prevented by pretreatment with a nondepolarizing muscle relaxant. A: 1,2,3 Correct B: 1,3 Correct C: 2,4 Correct D: 4 Correct E: All Correct ABCDE Correct Answer: C Suxamethonium typically causes an increase in the serum potassium of 0.5-1.0 mmol/L ( and sometimes 2 mmol/L). An atypically large rise in serum potassium occurs in patients who have developed " extra-junctional" acetylcholine receptors at the muscle motor end-plate. Patients with renal failure have disorders of potassium regulation, but do not have " extrajunctional " receptors. A typical rise in serum potassium should be expected in these patients, however, if they already have hyperkalemia, a further rise of 1-2 mmol/L may be enough to cause serious complications. Muscle pain associated with suxamethonium typically occur in young women following minor procedures. This is partly explained on the basis of a greater muscle mass in the young; and earlier ambulation following minor procedures, which " betrays " the condition. Longer 569

periods of bed rest associated with major procedures are felt to conceal the potential for pain. It is interesting to note that it is not commonly reported in patients undergoing electroconvulsive therapy. The pathogenesis is unclear. Theories include: -muscle damage caused by the asynchronous contraction of muscle during fasciculation. -release of lactic acid subsequent to muscle fasciculation. -fascial damage caused by synchronous muscle fasciculation, and -the rise in serum potassium. Muscle pain may be prevented or reduced by " pre-treatment " with 10% of the standard intubating dose of a non-depolarizing muscle relaxant. Greatest protection is afforded when the interval between administration of the intravenous induction agent and suxamethonium is small. The " normal " rise in serum potassium associated with suxamethonium is postulated to occur secondary to opening of outward potassium channels on depolarization of the motor endplate. Muscle fibre damage associated with fasciculations may also contribute. It typically lasts for a period of 15 minutes, but may not return to normal values for 1 hour after the administration of suxamethonium. Anaesthetic agents have been implicated in modifying this effect. This evoked release may be greater after N2O and halothane, for example. In contrast, the evoked release may be attenuated by " pre-treatment " with gallamine. d-tubo-curarine, pancuronium, Mg, dantrolene, diazapam, lignocaine, calcium gluconate, and salbutamol. " Self-taming " by pretreatment with suxamethonium may also occur. These are experimental findings and apply to the " normal " response only. Certain groups of patients experience proliferation of " extra-junctional" acetylcholine receptors which occur outside the margins of the motor end-plate. Suxamethonium causes a larger than usual rise in serum potassium. This rise is of the order of 2-3 mmol/L, but has been reported as higher. It varies with the nature of the condition involved, its age, and severity. The conditions in which it has been described include: (1)BURNS. A 10-60 day rule has been described subsequent to reports of the phenomenon between days 14 -66 after burns. As in other conditions, a latent phase occurs; " extra-junctional " receptors do not appear immediately after injury. (2)ACUTE UPPER MOTOR NEURON LESIONS & STRUCTURAL BRAIN DAMAGE. These include spinal cord injury, anoxic brain damage, encephalitis, subarachnoid haemorrhage, and cerebrovascular accidents. (3)PROGRESSIVE NEUROLOGICAL CONDITIONS. Demyelinating diseases, and brain tumours may be associated with the response. Again this will be greater if the disease is rapidly progressive, and in general, the potential for evoked release of potassium is of a lower magnitude and occurs over a longer period of time. The relationship in Parkinson's disease is not clear. (4)CHRONIC STABLE NEUROLOGICAL CONDITIONS. These are generally associated with a " normal " response only, including cerebral palsy. (5)LOWER MOTOR NEURON DISEASE. The phenomenom has been described betwen days 42-192 following acute peripheral nerve damage, and cardiac arrest has been reported follwing suxamethonium with diffuse neuropathies.

570

(6)MUSCULAR DYSTROPHIES. Hyperkalemia has been reported in patients with Duchenne's muscular dystrophy, however, this was ascribed to Malignant Hyperpyrexia. (7)TRAUMA. As with burns, this occurs in manner influenced by the time from injury, and the extent of nerve and muscle damage. (8)INFECTION. Again, this response has been seen in severe infection including osteomyelitis and gangrene. References GIBB, D.B; Anaesth Intens Care, 1, 109, 1973 & 1, 183, 1974. (The last word on suxamethonium in two parts!) Number: 200 When used in conjunction with amide local anaesthetics (LA's): 1. Lipid carrier solutions prolong the duration of effect. 2. IV Clonidine reduces the incidence of epidural shivering. 3. Clonidine potentiates membrane stabilisation produced by LA's 4. Alkalinisation of solutions increases the rate of onset and efficacy of LA's. A: 1,2,3 Correct B: 1,3 Correct C: 2,4 Correct D: 4 Correct E: All Correct ABCDE Correct Answer: A Both lipids and clonidine have potentially useful interactions with local anaesthetic compounds. A lipid carrier (such as iophendylate) can effectively double the duration of effect of most local anaesthetics. Clonidine exerts complex effects at various sites - These include:anti-nociception at a spinal cord level, (as do other Alpha 2 agonists), membrane stabilisation and abolition of shivering. . Alkalinisation of LA's consistently increases their rate of onset of the local anaesthetics (whatever their route of administration), but usually has not been found to affect their efficacy. References: Prolongation of epidural anesthesia using a lipid drug carrier with procaine, lidocaine, and tetracaine. Langerman L; Grant GJ; Zakowski M; Golomb E; Ramanathan S; Turndorf H Anesthesia Analgesia. 1992 Dec; 75(6): 900-5 This study evaluated the effect of a lipid drug carrier (iophendylate) on epidural anesthesia. The intensity and duration of motor blockade produced by aqueous and lipid preparations of local anesthetics were assessed in rabbits with long-term indwelling catheters in the epidural space. Motor blockades produced by procaine (1%, 2%, and 4%), lidocaine (1%, 2%, and 4%), and tetracaine (0.5%, 1%, and 2%) in normal saline solution were compared with the effects produced by equimolar amounts of the drug solutions in iophendylate. Procaine (4%) in aqueous solution produced motor blockade lasting 30 +/- 3.54 min (mean +/- SD) versus 84

571

+/- 4.18 min in lipid solution. Lidocaine (2% and 4%) in aqueous solution produced motor blockade lasting 41 +/- 4.18 and 65 +/- 6.12 min versus 39 +/- 4.18 and 118 +/- 10.1 min, respectively, in lipid solution. Aqueous tetracaine (0.5%, 1%, and 2%) produced motor blockade of 106 +/- 9.62, 189 +/- 6.52, and 273 +/- 26.8 min versus 284 +/- 14.7, 335 +/15.8, and 365 +/- 26.9 min, respectively, in their lipid counterparts. A control group of animals that received normal saline solution or iophendylate alone did not exhibit motor blockade. These results may be attributed to sustained release of local anesthetics from the lipid vehicle. Hence, lipid drug carriers may be effective in prolonging epidural anesthesia. Low dose i.v. clonidine reduces shivering after epidural anaesthesia Tagariello V, Bertini L, Alcione AM, Molino FM, Rossignoli L. ALR 1991; 1: 38-43. Forty-one male patients who had received epidural anaesthesia with 2 % mepivacaine and who had experienced shivering following epidural block were enrolled in this randomized, double-blind, crossover study. Patients were divided into two groups to receive either i.v. clonidine (group A, n : 21) or saline as placebo (group B, n: 20) to control postepidural shivering. i.v. fluids were warmed to body temperature (37C) and local anaesthetic solution was administered at room temperature. ECG, blood pressure and oxygen saturation body temperature were recorded. Hypotension was promptly treated by i.v. ephedrine. Shivering was noted and graded according to a fourpoint scale (0-- no shivering; 1 = moderate shivering, patient comfortable; 2 = mild shivering, subjective cold sensation; 3= generalized shivering, patient uncomfortable, intolerable subjective cold sensation). According to the study protocol, patients with grade-2 shivering were given 50 p.g i.v. clonidine every 4 minutes up to 150 pg, or saline at the same time intervals and bolus dosages. After 12 minutes, if no effects on shivering were noted, the protocol was repeated changing the study solutions. In the group A patients (n= 19, 90%) a dramatic reduction in shivering was observed (P <0.001), while in the placebo group 90% of patients worsened. Group B patients improved when they received clonidine (P <0.005). No significant differences were noted in sensory level of analgesia, hypotension and ephedrine administration, oxygen saturation and body temperature between the groups. Hypotension was related to the epidura| block, which was not worsened by the administration of clonidine. Comment Although the mechanism of action of clonidine on shivering is unknown, its clinical effects seem surprising. The rapidity of action and the lack of hypotensive effects at such low doses suggest that further larger studies are needed to investigate the possibility of its routine use in controlling postepidural shivering. Clonidine enhances the effects of lidocaine on C-fiber action potential. Gaumann DM; Brunet PC; Jirounek P Anesthesia Analgesia. 1992 May; 74(5): 719-25 We examined local anesthetic effects of clonidine and its interaction with lidocaine with regard to tonic inhibition of the C-fiber action potential (AP) on the isolated, desheathed rabbit vagus nerve by the sucrose gap method. Clonidine and lidocaine at 500 microM concentrations caused a comparable degree of C-fiber inhibition, corresponding to an AP area under the curve of 75.8% +/- 9.4% (mean +/- SE) and 82.2% +/- 5.9% of control, respectively. Concentrations of clonidine less than 500 microM did not inhibit C-fiber AP. Clonidine, added in concentrations of 500 nM, 500 microM, and 5 mM to a 500 microM

572

lidocaine perfusion, caused a significant decrease in fiber blockade of 18%, 20%, and 54%, respectively, as compared with clonidine added to Locke perfusion (P less than 0.05). The sodium channel blocker tetrodotoxin (3 microM) decreased the AP area to 9.3% +/- 1.3% of control. The remaining tetrodotoxin-resistant AP was almost completely blocked by clonidine (500 microM) and lidocaine (500 microM), indicating a higher susceptibility of tetrodotoxinresistant fibers to the two drugs than the C-fiber population as a whole. The enhancing effect of a low dose of clonidine (500 nM) on lidocaine-induced (500 microM) inhibition of C-fiber AP might explain the clinical observation that clonidine, at approximately 1000-fold lower concentrations than lidocaine, prolongs the action of lidocaine in peripheral nerve block. Comparison of plain and alkalinized local anaesthetic mixtures of lignocaine and bupivacaine for elective extradural caesarean section. Fernando R; Jones HM Br. J. Anaesth. 1991 Dec; 67(6): 699-703 We have examined a local anaesthetic mixture of 0.5% bupivacaine 10 ml and 2% lignocaine 10 ml with adrenaline 1 in 200,000, to which 8.4% sodium bicarbonate 2 ml was added, for extradural Caesarean section. The alkalinized mixture of local anaesthetics produced a block of more rapid onset and density than a mixture of bupivacaine and lignocaine alone (P less than 0.001). Comparison between clonidine and epinephrine admixture to lidocaine in brachial plexus block. Gaumann D; Forster A; Griessen M; Habre W; Poinsot O; Della-Santa D Anesthesia Analgesia. 1992 Jul; 75(1): 69-74 The admixture of clonidine or epinephrine to lidocaine for brachial plexus block was studied with regard to duration of block, postoperative analgesia, and plasma concentrations of lidocaine. Thirty-three patients of ASA physical status I and II received an admixture of either clonidine (150 micrograms; n = 15) or epinephrine (200 micrograms; n = 18) to 40 mL of 1% lidocaine in a randomized, double-blind fashion. Bone surgery predominated in those patients receiving clonidine and soft-tissue surgery in those receiving epinephrine (P less than 0.05). Onset and duration of block were not different between the groups. With the admixture of clonidine, fewer patients were completely pain free for greater than 12 h (13.3%) and pain scores (visual analogue scale 0-10) were higher 6 h after the block (median 4; range 0-6) than with epinephrine (61.1%; median 2; range 0-7, respectively; P less than 0.05). In patients who had received clonidine, peak plasma concentrations of lidocaine were higher (10.29 +/- 2.96 mumol/L) and occurred earlier (23.7 +/- 9.3 min; mean +/- SD) than in those treated with epinephrine (6.9 +/- 1.71 mumol/L; 72.5 +/- 56.2 min; P less than 0.05). This indicates the absence of a local vasoconstrictor effect of clonidine and implies a reduced margin of safety with regard to local anesthetic toxicity. Although clonidine does not offer advantages compared with epinephrine, it may be a useful adjunct to local anesthetics in those patients in whom the administration of epinephrine is contraindicated. Number: 227 With regard to Ropivacaine: 1. It is a pure S-stereoisomer. 2. It is slightly less potent than bupivacaine (1:1.3 to 1:1.5).

573

3. Reversal of myocardial Na+ channel blockade is considerably faster with ropivacaine than with bupivacaine. 4. It has a butyl group on the piperidine ring. A: 1,2,3 Correct B: 1,3 Correct C: 2,4 Correct D: 4 Correct E: All Correct ABCDE Correct Answer: A The butyl group on the piperidine ring of bupivacaine has been replaced by a propyl group in ropivacaine. Commercial bupivacaine is a racemic mixture of the R and S stereoisomers, whereas ropivacaine is a pure S isomer. Na+ channel blockade by bupivacaine is notoriously slow to reverse and is much more rapid in the case of ropivacaine. Slow Na+ channel reversal has been associated with persistent conduction defects, re-entry disorders and the development of major ventricular arrhythmias. The combination of easy reversal of Na+ channel blockade together with a relative lack of negative inotropic effect leads to a significant reduction in cardiotoxic effects of ropivacaine as compared to bupivacaine. Number: 228 With regard to cardiotoxicity of bupivacaine and ropivacaine: 1. The cardiotoxic profile of these drugs is similar. 2. QRS prolongation parallels the local anaesthetic potency of these drugs on the isolated nerve. 3. Myocardial Na+ channel blockade is less marked with R-isomers 4. Myocardial depression parallels the local anaesthetic potency of these drugs on the isolated nerve. A: 1,2,3 Correct B: 1,3 Correct C: 2,4 Correct D: 4 Correct E: All Correct ABCDE Correct Answer: D

574

Ropivacaine is generally less cardiotoxic than bupivacaine. Myocardial depression of these (and other) local anaesthetics parallels the local anaesthetic potency of the drugs on the isolated nerve. However, conduction disorders are far less marked with ropivacaine than with bupivacaine. - This may be a reflection of the fact that R-stereoisomers are far more potent at blocking myocardial Na+ channels than the corresponding S-isomers. Number: 235 With regard to the placental transfer of ropivacaine: 1. It is more likely to reach the fetus than chloroprocaine. 2. It is facilitated by fetal acidosis. 3. It is less transferable than mepivacaine. 4. All of the above. A: 1,2,3 Correct B: 1,3 Correct C: 2,4 Correct D: 4 Correct E: All Correct ABCDE Correct Answer: D Ester-linked local anaesthetic agents undergo such rapid hydrolysis in the maternal plasma that minimal amounts are transferred to the fetus. The plasma half-life of chloroprocaine in the presence of a normal pseudocholinesterase is about 20 seconds. In contrast, the amide-linked agents have relatively long half-lives. - Thus, in combination with their low molecular weights and high lipid solubilities, they can all transverse the placenta to a greater or lesser degree. The ease of transfer is determined, in part, by the degree of ionisation at physiological pH. This, in turn, is determined by the pKa of the agent. Drugs with a low pKa have a greater proportion of the un-ionised species and are therefore more likely to be transferred. The five amide agents can be ranked in order of PKa and diffusibility as follows: Mepivacaine (pKa 7.65) Etidocaine (pKa 7.76) Lidocaine (pKa 7.85) Ropivacaine (pKa 8.1) Bupivacaine (pKa 8.16). On this basis, mepivacaine is likely to show the greatest degree of placental transfer and bupivacaine the least. The phenomenon of ion trapping may facilitate transfer of the amide-linked agents to the acidotic fetus. Reference: Shnider, S, Anesthesia for Obstetrics, 3rd Edition, pp. 86-90.

575

Number: 235 With regard to the placental transfer of ropivacaine: 1. It is more likely to reach the fetus than chloroprocaine. 2. It is facilitated by fetal acidosis. 3. It is less transferable than mepivacaine. 4. All of the above. A: 1,2,3 Correct B: 1,3 Correct C: 2,4 Correct D: 4 Correct E: All Correct ABCDE Correct Answer: D Ester-linked local anaesthetic agents undergo such rapid hydrolysis in the maternal plasma that minimal amounts are transferred to the fetus. The plasma half-life of chloroprocaine in the presence of a normal pseudocholinesterase is about 20 seconds. In contrast, the amide-linked agents have relatively long half-lives. - Thus, in combination with their low molecular weights and high lipid solubilities, they can all transverse the placenta to a greater or lesser degree. The ease of transfer is determined, in part, by the degree of ionisation at physiological pH. This, in turn, is determined by the pKa of the agent. Drugs with a low pKa have a greater proportion of the un-ionised species and are therefore more likely to be transferred. The five amide agents can be ranked in order of PKa and diffusibility as follows: Mepivacaine (pKa 7.65) Etidocaine (pKa 7.76) Lidocaine (pKa 7.85) Ropivacaine (pKa 8.1) Bupivacaine (pKa 8.16). On this basis, mepivacaine is likely to show the greatest degree of placental transfer and bupivacaine the least. The phenomenon of ion trapping may facilitate transfer of the amide-linked agents to the acidotic fetus. Reference: Shnider, S, Anesthesia for Obstetrics, 3rd Edition, pp. 86-90. Number: 236 A 2 mg/ml solution of ropivacaine: 1. Is vasoconstrictive when given intradermally. 2. May reasonably be used for brachial plexus anaesthesia. 3. May reasonably be used for obstetric analgesia. 4. Is likely to prove cardiotoxic if 15ml (30mgs) are given intravenously to an adult.

576

A: 1,2,3 Correct B: 1,3 Correct C: 2,4 Correct D: 4 Correct E: All Correct ABCDE Correct Answer: B In common with all amide-linked local anaesthetics, vasoconstriction occurs at low concentrations and vasodilatation at high concentrations. Plexus anaesthesia should be performed using at least 5 mg/ml ropivacaine and is unlikely to be successful if a 2 mg/ml solution is used. Sensory-motor splitting is a prominent feature of the drug. This may be used to advantage in the area of obstetric analgesia when ropivacaine 2 mg/ml can provide satisfactory analgesia with little motor impairment. Symptoms of neurotoxicity generally appear before those of cardiotoxicity. In adult human volunteers, symptoms consistent with mild CNS toxicity appeared after an average (intravenous) total dose of ~ 125 mg. It should be noted that ropivacaine is not approved for intravenous use. Number: 237 A 3mg/ml solution of ropivacaine infused by the mid-thoracic epidural route: 1. Is unlikely to produce motor blockade if infused at 10ml (30mg) /hr. 2. Is roughly equivalent to 0.5% bupivacaine in terms of its potency. 3. Is unlikely to cause CNS toxicity if infused at 10ml/hr for 24 hours. 4. Will produce less effective postoperative analgesia than a 5mg/ml solution given at 6 ml / hr. A: 1,2,3 Correct B: 1,3 Correct C: 2,4 Correct D: 4 Correct E: All Correct ABCDE Correct Answer: B

577

A 3mg/ml solution of ropivacaine infused at this rate is very unlikely to produce significant motor blockade, neither will it produce the plasma concentrations which are associated with CNS toxicity if epidurally administered to a 70kg adult. 5mg/ml ropivacaine is an unnecessarily concentrated solution to use for postoperative analgesia. Solutions in the range 2-3mg/ml (0.2 0.3%) having been generally found to be appropriate for this application. A recent study by Schug SA et al in the British Journal of Anaesthesia is typical of the results of research into the use of ropivacaine for postoperative analgesia. These results (reproduced from The International Monitor) are summarised below. Postoperative analgesia by continuous extradural infusion of ropivacaine after upper abdominal surgery Schug SA, Scott DA, Payne J, Mooney PH, Haeggloef B. Br J Anaesth 1996; 76 (4): 487-491. Ropivacaine is a new local anaesthetic with advantages that suggest an important role in the provision of postoperative analgesia. The main aim of this study was to investigate the doseresponse relationship of extradural infusion of ropiva-caine. We studied 36 ASA I-III patients undergoing upper abdominal surgery during general anaesthesia and extradural block (catheter insertion at T6-9) using 0.5% ropivacaine in a randomized, double-blind study. After surgery nine patients each received an extradural infusion of either ropivacaine 0.1%, 0.2%, 0.3% or saline at a rate of 10 ml.h-l for 21 h. All patients had access to i.v. morphine via a PCA device. The ropivacaine groups consumed significantly less morphine over the 21h infusion period than the saline group (medians: saline 75 mg; 0.1% ropivacaine 32 mg; 0.2% ropivacaine 39 mg; 0.3% ropivacaine 13 mg) (P < 0.05). Pain (VAS scores) at rest was significantly lower in all ropivacaine groups than in the saline group after 4 h of infusion (medians: saline 45 mm; 0.1% ropivacaine 15 mm; 0.2% ropivacaine 12 mm; 0.3% ropivacaine 0 mm). Pain on coughing was significantly less in all ropivacaine groups than in the saline group after 4 h infusion (medians: saline 67 mm; 0.1% ropivacaine 44 mm; 0.2% ropivacaine 33 mm; 0.3% ropivacaine 0 mm) and for 0.2% and 0.3% ropivacaine at later times. Motor block was negligible throughout the infusion. Patient satisfaction was higher in the 0.2% and 0.3% ropivacaine groups than the two other groups. Number: 238 Ropivacaine is appropriate for use in: 1. A patient with proven allergy to procaine. 2. Intravenous Regional Anaesthesia (Biers blocks). 3. A patient with dialysis-dependent renal failure. 4. Spinal anaesthesia. A: 1,2,3 Correct B: 1,3 Correct

578

C: 2,4 Correct D: 4 Correct E: All Correct ABCDE Correct Answer: B Ropivacaine is not approved for use in either intravenous regional anaesthesia (IVRA) or spinal anaesthesia. Cross-reactivity between the ester-linked (eg procaine) and amide-linked (eg ropivacaine) local anaesthetics probably does not occur. Although not yet approved for IVRA the use of the drug in this manner has recently been described by Hartmannsgruber et al. Number: 422 Mivacurium: 1. Is metabolised by plasma cholinesterase. 2. Has an elimination half-life of about 20 minutes. 3. Will lower systemic vascular resistance by about 1/3 when given at a dose of 0.2 mg/kg. 4. Does not cause significant histamine release. A: 1,2,3 Correct B: 1,3 Correct C: 2,4 Correct D: 4 Correct E: All Correct ABCDE Correct Answer: B Mivacurium is a relatively new nondepolarising diester neuromuscular blocking agent. It is metabolised in vitro by human plasma cholinesterase at 70% the rate of succinylcholine and undergoes ester hydrolysis in the liver. Its duration of action is twice that of succinylcholine but only one third to one half the duration of action of atracurium and vecuronium. It has an elimination half-life of 8 to 10 minutes. Thus it shows significant clinical potential for maintenance of blockade by continuous infusion. Mivacurium 0.15 mg mg/kg produces changes of only small changes in mean arterial pressure (12%) and systemic vascular resistance index(16%). However, mivacurium 0.2 mg/kg produces a 25% reduction in mean arterial pressure, a 14% increase in cardiac index and a 35% decrease in systemic vascular resistance index. Comparison of the haemodynamic effects of mivacurium and atracurium during fentanyl anaesthesia. Loan PB; Elliott P; Mirakhur RK; Bunting HF; Bhanumurthy S; McMurray TJ.

579

The drug causes significant histamine release which can be prevented with H1 antagonists. Effect of oral antihistamine premedication on mivacurium-induced histamine release and side effects. Doenicke A; Moss J; Lorenz W; Mayer M; Rau J; Jedrzejewski A; Ostwald P Number: 428 Cisatracurium: 1. Releases histamine when given at a dose of 0.1 mg/kg. 2. Is metabolised primarily by Hofmann degradation. 3. Is an aminosteroid compound. 4. Is metabolised to largely inactive compounds. A: 1,2,3 Correct B: 1,3 Correct C: 2,4 Correct D: 4 Correct E: All Correct ABCDE Correct Answer: C Even at 8 times the ED95 dose, Cisatracurium is not a histamine releasing drug. It is a benzylisoquinolinium compound unlike, for example, vecuronium which is an aminosteroid. Cisatracurium is an intermediate-duration, non-depolarizing neuromuscular-blocking agent, the purified form of one of the 10 stereoisomers of atracurium which is cleared primarily by Hofmann elimination. It produces minimal cardiovascular changes and does not release histamine even at up to eight times the dose that produced 95% of the maximum response (ED95) for neuromuscular bloclc At twice the ED95, the pharrnacodynamic profile of cisatracurium is similar to that of an equipotent dose of atracurium apart from a slightly slower onset of action. A more rapid onset is produced when the dose is increased. The clinically effective duration of action, as assessed by the twitch response of the abductor policis muscle to a train of four stimulations, increases with increasing doses; however, doubling the dose only adds approximately 23 min to the duration of clinically effective block The clinically effective duration of action ranges from 45 min after twice the ED95 (0.1 mg/kg) to approximately 90 min after eight times the ED95 (0.4 mg/kg). Neuromuscular block can be maintained with incremental or repeat bolus doses or continuous infusions of cisatracurium without any cumulative neuromuscular~blocking effect. Neuromuscular block induced with cisatracurium can be readily reversed using anticholinesterase agents. Cisatracurium has a predictable and rapid rate of spontaneous recovery (5-95% recovery about 30 min) irrespective of the initial dose administered or the duration of maintenance dosing.

580

The pharmacodynamics of cisatracurium in healthy adults. Savarese JJ et al. Current Opinion in Anaesthesiology; Vol 9, Suppl 1 April 1996. Number: 429 Pancuronium: 1. Is excreted largely unchanged in the urine. 2. Has an active metabolite that retains 50% of the activity of pancuronium. 3. Can cause a myopathy if used for prolonged periods. 4. Is a benzylquinolinium compound. A: 1,2,3 Correct B: 1,3 Correct C: 2,4 Correct D: 4 Correct E: All Correct ABCDE Correct Answer: A Pancuronium is excreted largely unchanged in the urine, and a small percentage is metabolized, most significantly to 3-hydroxypancuronium. The metabolite has 50% of the neuromuscular-blocking activity of pancuronium, is excreted in the urine and bile, and may reach significant concentrations when large doses of Pancuronium are administered. In common with vecuronium, which is also steroid-based, there have been reports of myopathy following prolonged use of pancuronium. Pancuronium may be associated with undesirable vagolytic effects. Structurally pancuronium is an aminosteroid. Current Opinion in Anaesthesiology; Vol 9, Suppl 1 April 1996 p S48. Number: 431 Which of the following statements are true regarding atracurium: 1. It theoretically has 16 stereo-isomers. 2. More than 50% of its neuromuscular blocking activity is due to cisatracurium in the preparation. 3. It does not need to be given in modified dosage to a patient in renal failure. 4. It is a benzylisoquinolinium. A: 1,2,3 Correct B: 1,3 Correct C: 2,4 Correct D: 4 Correct E: All Correct

581

ABCDE Correct Answer: E Atracurium has four chiral centres, but because of molecular symmetry, the 16 theoretically possible isomers are reduced to 10. Six of the 10 isomers have been prepared and evaluated for neuromuscular, autonomic and cardiovascular effects. All of the isomers exhibit neuromuscular-blocking activity with a 10fold range of potency, and with the exception of cisatracurium, produce autonomic and cardiovascular effects at doses similar to those of atracurium. Cisatracurium is more potent than atracurium and, unlike atracurium and all the other isomers, bolus injections of high multiples of the 95% effective neuromuscular-blocking dose does not produce histamine-like cardiovascular effects or increase plasma histamine concentration. Number: 470 Lignocaine: 1. Is a Vaughan-Williams Class 1B anti-arrhythmic. 2. Is given in the dose range 1-1.5 mg.kg as an initial bolus dose. 3. Increases the fibrillation threshhold of the heart. 4. Increases the defibrillation threshhold of the heart. A: 1,2,3 Correct B: 1,3 Correct C: 2,4 Correct D: 4 Correct E: All Correct ABCDE Correct Answer: E All of the are correct. An interesting study has recently been published which questions the role of such drugs in 'in-hospital cardiac arrest' - see below. Do advanced cardiac life support drugs increase resuscitation rates from in-hospital cardiac arrest? The OTAC Study Group. Ann Emerg Med 1998 Nov;32(5):544-53 van Walraven C, Stiell IG, Wells GA, Hebert PC, Vandemheen K STUDY OBJECTIVE: The benefit of Advanced Cardiac Life Support (ACLS) medications during cardiac resuscitation is uncertain. The objective of this study was to determine whether the use of these medications increased resuscitation from in-hospital cardiac arrest.

582

METHODS: A prospective cohort of patients undergoing cardiac arrest in 1 of 5 academic hospitals was studied. Patient and arrest factors related to resuscitation outcome were recorded. We determined the association of the administration of ACLS drugs (epinephrine, atropine, bicarbonate, calcium, lidocaine, and bretylium) with survival at 1 hour after resuscitation. RESULTS: Seven hundred seventy-three patients underwent cardiac resuscitation, with 269 (34. 8%) surviving for 1 hour. Use of epinephrine, atropine, bicarbonate, calcium, and lidocaine was associated with a decreased chance of successful resuscitation (P <.001 for all except lidocaine, P <.01). While controlling for significant patient factors (age, gender, and previous cardiac or respiratory disease) and arrest factors (initial cardiac rhythm, and cause of arrest), multivariate logistic regression demonstrated a significant association between unsuccessful resuscitation and the use of epinephrine (odds ratio . 08 [95% confidence interval .04-.14]), atropine (.24 [.17-.35]), bicarbonate (.31 [.21-.44]), calcium (.32 [.18-.55]), and lidocaine (.48 [.33-.71]). Drug effects did not improve when patients were grouped by their initial cardiac rhythm. Cox proportional hazards models that controlled for significant confounders demonstrated that survivors were significantly less likely to receive epinephrine (P <. 001) or atropine (P <.001) throughout the arrest. CONCLUSION: We found no association between standard ACLS medications and improved resuscitation from in-hospital cardiac arrest. Randomized clinical trials are needed to determine whether other therapies can improve resuscitation from cardiac arrest when compared with the presently used ACLS drugs. Number: 472 Which of the following concentrations of ropivacaine will provide effective woundinfiltration analgesia after hernia repair? 1. 0.75% 2. 0.125% 3. 0.5% 4. 0.0625% A: 1,2,3 Correct B: 1,3 Correct C: 2,4 Correct D: 4 Correct E: All Correct ABCDE Correct Answer: B Mulroy et al found that ropivacaine 0.25% and 0.5% was adequate for pain relief after outpatient hernia repair, whereas the 0.125% solution was no more effective than saline. Reg Anesth Pain Med 1999 Mar-Apr;24(2):136-41 Mulroy MF, Burgess FW, Emanuelsson BM

583

Ropivacaine 0.25% and 0.5%, but not 0.125%, provide effective wound infiltration analgesia after outpatient hernia repair, but with sustained plasma drug levels. Number: 489 Histamine release is a feature of administration of which of the following relaxants? 1. Rocuronium. 2. Mivacurium. 3. Vecuronium. 4. Atracurium. A: 1,2,3 Correct B: 1,3 Correct C: 2,4 Correct D: 4 Correct E: All Correct ABCDE Correct Answer: C Br J Anaesth 1995 Nov;75(5):588-92 Naguib M, Samarkandi AH, Bakhamees HS, Magboul MA, el-Bakry AK Histamine-release haemodynamic changes produced by rocuronium, vecuronium, mivacurium, atracurium and tubocurarine. We have examined the effects of different benzyl-isoquinolinium and steroidal neuromuscular blocking compounds on plasma concentrations of histamine, heart rate and arterial pressure in surgical patients. A single, rapid (5-s) bolus of mivacurium 0.2 mg kg-1, atracurium 0.6 mg kg-1, tubocurarine 0.5 mg kg-1, vecuronium 0.1 mg kg-1 or rocuronium 0.6 mg kg-1 was administered to 75 patients (n = 15 in each group). Anaesthesia was induced with thiopentone 6 mg kg-1 i.v. and maintained with isoflurane and 70% nitrous oxide in oxygen. Venous blood samples were obtained before induction, 1 min after thiopentone and 1, 3 and 5 min after administration of the neuromuscular blocking drug. Mivacurium, atracurium and tubocurarine caused 370%, 234% and 252% increases in plasma histamine concentrations at 1 min, respectively. Corresponding values at 3 min were 223%, 148% and 157%, respectively. These changes were significant (P < 0.01) at 1 and 3 min. In contrast, the rocuronium and vecuronium groups had no significant changes in either plasma histamine concentrations or haemodynamic variables.

584

Number: 489 Histamine release is a feature of administration of which of the following relaxants? 1. Rocuronium. 2. Mivacurium. 3. Vecuronium. 4. Atracurium. A: 1,2,3 Correct B: 1,3 Correct C: 2,4 Correct D: 4 Correct E: All Correct ABCDE Correct Answer: C Br J Anaesth 1995 Nov;75(5):588-92 Naguib M, Samarkandi AH, Bakhamees HS, Magboul MA, el-Bakry AK Histamine-release haemodynamic changes produced by rocuronium, vecuronium, mivacurium, atracurium and tubocurarine. We have examined the effects of different benzyl-isoquinolinium and steroidal neuromuscular blocking compounds on plasma concentrations of histamine, heart rate and arterial pressure in surgical patients. A single, rapid (5-s) bolus of mivacurium 0.2 mg kg-1, atracurium 0.6 mg kg-1, tubocurarine 0.5 mg kg-1, vecuronium 0.1 mg kg-1 or rocuronium 0.6 mg kg-1 was administered to 75 patients (n = 15 in each group). Anaesthesia was induced with thiopentone 6 mg kg-1 i.v. and maintained with isoflurane and 70% nitrous oxide in oxygen. Venous blood samples were obtained before induction, 1 min after thiopentone and 1, 3 and 5 min after administration of the neuromuscular blocking drug. Mivacurium, atracurium and tubocurarine caused 370%, 234% and 252% increases in plasma histamine concentrations at 1 min, respectively. Corresponding values at 3 min were 223%, 148% and 157%, respectively. These changes were significant (P < 0.01) at 1 and 3 min. In contrast, the rocuronium and vecuronium groups had no significant changes in either plasma histamine concentrations or haemodynamic variables. Number: 489 Histamine release is a feature of administration of which of the following relaxants? 1. Rocuronium. 2. Mivacurium. 3. Vecuronium. 4. Atracurium.

585

A: 1,2,3 Correct B: 1,3 Correct C: 2,4 Correct D: 4 Correct E: All Correct ABCDE Correct Answer: C Br J Anaesth 1995 Nov;75(5):588-92 Naguib M, Samarkandi AH, Bakhamees HS, Magboul MA, el-Bakry AK Histamine-release haemodynamic changes produced by rocuronium, vecuronium, mivacurium, atracurium and tubocurarine. We have examined the effects of different benzyl-isoquinolinium and steroidal neuromuscular blocking compounds on plasma concentrations of histamine, heart rate and arterial pressure in surgical patients. A single, rapid (5-s) bolus of mivacurium 0.2 mg kg-1, atracurium 0.6 mg kg-1, tubocurarine 0.5 mg kg-1, vecuronium 0.1 mg kg-1 or rocuronium 0.6 mg kg-1 was administered to 75 patients (n = 15 in each group). Anaesthesia was induced with thiopentone 6 mg kg-1 i.v. and maintained with isoflurane and 70% nitrous oxide in oxygen. Venous blood samples were obtained before induction, 1 min after thiopentone and 1, 3 and 5 min after administration of the neuromuscular blocking drug. Mivacurium, atracurium and tubocurarine caused 370%, 234% and 252% increases in plasma histamine concentrations at 1 min, respectively. Corresponding values at 3 min were 223%, 148% and 157%, respectively. These changes were significant (P < 0.01) at 1 and 3 min. In contrast, the rocuronium and vecuronium groups had no significant changes in either plasma histamine concentrations or haemodynamic variables. Number: 542 The relative potency of ropivacaine in comparison to bupivacaine for analgesia in labour is: A. 1.2 B. 1.0 C. 0.8 D. 0.6 E. 0.4 Select the single best answer ABCDE Correct Answer: D Ropivacaine has about 60% of the potency of bupivacaine when used for analgesia in labour.

586

The studies by Capogna et al and Polley et al use an elegant methodology to explore this relationship. G. Capogna, D. Celleno, P. Fusco, G. Lyons and M. Columb: BJA SA Edition Volume 5 May 1999 152-155 Relative potencies of bupivacaine and ropivacaine for analgesia in labour. Polley LS, Columb MO, Naughton NN, Wagner DS, van de Ven CJ: Anesthesiology 1999 Apr;90(4):944-50 Relative analgesic potencies of ropivacaine and bupivacaine for epidural analgesia in labor: Implications for therapeutic indexes. Number: 542 The relative potency of ropivacaine in comparison to bupivacaine for analgesia in labour is: A. 1.2 B. 1.0 C. 0.8 D. 0.6 E. 0.4 Select the single best answer ABCDE Correct Answer: D Ropivacaine has about 60% of the potency of bupivacaine when used for analgesia in labour. The studies by Capogna et al and Polley et al use an elegant methodology to explore this relationship. G. Capogna, D. Celleno, P. Fusco, G. Lyons and M. Columb: BJA SA Edition Volume 5 May 1999 152-155 Relative potencies of bupivacaine and ropivacaine for analgesia in labour. Polley LS, Columb MO, Naughton NN, Wagner DS, van de Ven CJ: Anesthesiology 1999 Apr;90(4):944-50 Relative analgesic potencies of ropivacaine and bupivacaine for epidural analgesia in labor: Implications for therapeutic indexes. Number: 579 With regard to Cisatracurium and atracurium: 1. Cisatracurium is the major constituent by weight of atracurium. 2. Cisatracurium is three times more potent than atracurium. 3. Cisatracurium has the same onset time as atracurium when given in equipotent dosage. 4. Cisatracurium has a similar duration of effect to atracurium.

587

A: 1,2,3 Correct B: 1,3 Correct C: 2,4 Correct D: 4 Correct E: All Correct ABCDE Correct Answer: C Cisatracurium besylate is one of 10 isomers of atracurium besylate and constitutes approximately 15% of that mixture. The neuromuscular blocking potency of Cisatracurium is approximately threefold that of atracurium besylate. The time to maximum block is up to 2 minutes longer for equipotent doses of Cisatracurium compared to atracurium besylate. The clinically effective duration of action and rate of spontaneous recovery from equipotent doses of Cisatracurium and atracurium besylate are similar. Number: 594 Mivacurium: 1. Has a duration of effect about twice that of suxamethonium. 2. Undergoes Hofmann degradation. 3. Provokes histamine release. 4. Triggers malignant hyperthermia in susceptible swine. A: 1,2,3 Correct B: 1,3 Correct C: 2,4 Correct D: 4 Correct E: All Correct ABCDE Correct Answer: B The duration of action of mivacurium is approximately twice that of suxamethonium and 30% to 40% that of atracurium. The drug is hydrolysed by plasma cholinesterase, but does not undergo spontaneous (Hofmann) hydrolysis.

588

Administration of 3 times the ED 95 of mivacurium over 10 to 15 seconds evokes sufficient histamine release to lower mean arterial pressure by about 15%. Mivacurium does not trigger malignant hyperthermia in susceptible swine. Number: 595 Rapacuronium: 1. Is a depolarising muscle relaxant. 2. Is typically given in a dose of 1.5 mgs/kg. 3. Produces intubating conditions within 1 circulation time. 4. Has a duration of effect of 15 - 20 minutes if given in a dose of 1.5 mgs/kg. A: 1,2,3 Correct B: 1,3 Correct C: 2,4 Correct D: 4 Correct E: All Correct ABCDE Correct Answer: C Rapacuronium is a nondepolarising,aminosteroid neuromuscular blocking agent. The recommended dose range is 1.5-2.5 mgs/kg. The onset time of the drug when given at a dose of 2.5 mg/kg is more than 1 circulation time, but comparable to suxamethonium. It lasts for 15 - 20 minutes if given in a dose of 1.5 mgs/kg. Number: 675 When used within the therapeutic range, which of the following drugs is most likely to potentiate neuromuscular blockade induced by vecuronium? A. Magnesium. B. Cyclophosphamide. C. Lithium. D. Phenytoin. E. Bambuterol Select the single best answer ABCDE

589

Correct Answer: A Magnesium is a physiological calcium antagonist, and there is a correlation between the depression of neuromuscular transmission and serum magnesium concentration. Although overdosage of lithium causes muscle weakness, there is no real evidence that lithium potentiates neuromuscular blockade by non-depolarising relaxants if the level is within the therapeutic range. Resistance to neuromuscular blockade in patients on long-term phenytoin has been universally observed with the aminosteroid relaxants (pancuronium, vecuronium, pipecuronium, rocuronium). Bambuterol is an inactive prodrug that is slowly converted in the body to its active form terbutaline. The carbonate groups that are split off can selectively inhibit plasma cholinesterase. Thus blockade by either suxamethonium or mivacurium may be potentiated. Cyclophosphamide inhibits the activity of numerous cholinesterases and has been reported to potentiate the effects of mivacurium. The subject has recently been reviewed by Haywood et al. (Haywood PT, Divekar N, Karalliedde LD. Concurrent medication and the neuromuscular junction. Eur J Anaesthesiol. 1999 Feb;16(2):77-91.) Number: 720 Ion-trapping may be a clinically relevant mechanism in enhancing the placental transfer of: 1. Amide local anaesthetics. 2. Salicylates. 3. Opiates. 4. Non-depolarising muscle relaxants.

A: 1,2,3 Correct B: 1,3 Correct C: 2,4 Correct D: 4 Correct E: All Correct ABCDE Correct Answer: B If there is a pH gradient across a membrane, drug will be trapped on the side that has the higher ionised fraction, because only the un-ionised drug is diffusible. This phenomenon is known as 'ion trapping'.

590

Foetal pH is slightly lower than maternal pH. This pH gradient causes the ionised fraction of weak bases, such as opioids and local anaesthetics, to be higher in the foetus. In contrast, weak acids (such as salicylates) are preferentially retained on the maternal side of the placenta. Non-depolarising muscle relaxants are very highly ionised. As a result, very small amounts are transferred to the foetus and mechanisms such as ion-trapping are not clinically relevant. Number: 720 Ion-trapping may be a clinically relevant mechanism in enhancing the placental transfer of: 1. Amide local anaesthetics. 2. Salicylates. 3. Opiates. 4. Non-depolarising muscle relaxants. A: 1,2,3 Correct B: 1,3 Correct C: 2,4 Correct D: 4 Correct E: All Correct ABCDE Correct Answer: B If there is a pH gradient across a membrane, drug will be trapped on the side that has the higher ionised fraction, because only the un-ionised drug is diffusible. This phenomenon is known as 'ion trapping'. Foetal pH is slightly lower than maternal pH. This pH gradient causes the ionised fraction of weak bases, such as opioids and local anaesthetics, to be higher in the foetus. In contrast, weak acids (such as salicylates) are preferentially retained on the maternal side of the placenta. Non-depolarising muscle relaxants are very highly ionised. As a result, very small amounts are transferred to the foetus and mechanisms such as ion-trapping are not clinically relevant. Number: 720 Ion-trapping may be a clinically relevant mechanism in enhancing the placental transfer of: 1. Amide local anaesthetics.

591

2. Salicylates. 3. Opiates. 4. Non-depolarising muscle relaxants. A: 1,2,3 Correct B: 1,3 Correct C: 2,4 Correct D: 4 Correct E: All Correct ABCDE Correct Answer: B If there is a pH gradient across a membrane, drug will be trapped on the side that has the higher ionised fraction, because only the un-ionised drug is diffusible. This phenomenon is known as 'ion trapping'. Foetal pH is slightly lower than maternal pH. This pH gradient causes the ionised fraction of weak bases, such as opioids and local anaesthetics, to be higher in the foetus. In contrast, weak acids (such as salicylates) are preferentially retained on the maternal side of the placenta. Non-depolarising muscle relaxants are very highly ionised. As a result, very small amounts are transferred to the foetus and mechanisms such as ion-trapping are not clinically relevant. Number: 943 The most likely severe adverse effect of rapacuronium given at a dose of 1.5 mg / kg is: A. Bradycardia. B. Myocardial depression. C. Transient hyperkalaemia. D. Bronchospasm. E. Raised intra-ocular pressure. Select the single best answer ABCDE Correct Answer: D Bartkowski and Witkowski have recently reviewed this non-depolarising relaxant. These authors have commented that "Adverse effects reported following rapacuronium are similar to those seen with several other non-depolarizing neuromuscular blocking drugs. These include hypotension, tachycardia, bradycardia and bronchospasm. In general, these effects appear to be dose-related and are probably related to the low potency of rapacuronium, which necessitates a higher-milligram dose. This low potency is also the probable reason for its rapid onset. Studies employing doses greater than 1.5 mg/kg show a greater incidence of side-

592

effects than those found with lower doses. The recommended dose in the United States, i.e. 1.5 mg/kg, represents a compromise between rate of onset of action, reasonable duration and minimal side-effects. In fact, the US package insert lists 1.5 mg/ kg as the only recommended dose for routine circumstances. Overall, there has been a low reported incidence of side-effects in the United States since the approval of rapacuronium in August 1999. Since that time at least one million patients have been treated (data on file with Organon Inc.). The side-effect that has generated the most concern and discussion appears to be bronchospasm. The package insert lists its incidence as 3.2% in the clinical trials prior to approval. Since that time, after the official release of the drug, the reported incidence has been just 0.005% (data on file with Organon Inc.). As is typical of postrelease reporting, the incidence is likely to be underreported. We suspect that most of the short-lived and mild respiratory effects detected as wheezing or increased airway pressure are probably not reported. In spite of the low reported incidence of bronchospasm, its impact has been profound. Cases have been reported where the bronchospasm has been severe and life-threatening, mainly in children. These prompted editorial comment about the place of rapacuronium, given these adverse events. Further review by the regulatory agencies and manufacturer led to the voluntary withdrawal of rapacuronium from the US marketplace in March 2001, pending revision of the package insert." The drug is a low-potency, non-depolarising relaxant and is the 16-N-allyl-17-propionate analogue of vecuronium. It does not increase either serum potassium or intra-ocular pressure. See: Bartkowski RR, Witkowski TA. Rapacuronium: first experience in clinical practice. Eur J Anaesthesiol Suppl. 2001;23:90-3. Number: 976 A patient on long-standing anti-convulsant therapy is UNLIKELY to exhibit resistance to which of the following muscle relaxants? A. Pipecuronium. B. Pancuronium. C. Vecuronium. D. Rocuronium. E. Mivacurium. Select the single best answer ABCDE Correct Answer: E Resistance to neuromuscular blockade in patients on long-term anticonvulsant therapy has been universally observed with the aminosteroid relaxants (pancuronium, vecuronium, pipecuronium, rocuronium). Much less commonly it has been reported with benzoylisoquinolones (d-tubocurarine and atracurium).

593

This effect has been shown to be absent in the case of mivacurium, which is spontaneously hydrolysed by plasma cholinesterase. Phenytoin and carbamazepine are potent inducers of hepatic microsomal enzymes (particularly cytochrome P450 isoenzyme III A4), which results in the enhanced elimination of many drugs. It may be that this is the mechanism aminosteroid relaxant resistance, but this is by no means certain. It has also been suggested that an increase in protein binding results in a decreased free fraction of the drug and that upregulation of the ACh receptor has been induced. The subject has recently been reviewed by Haywood et al. (Haywood PT, Divekar N, Karalliedde LD. Concurrent medication and the neuromuscular junction. Eur J Anaesthesiol. 1999 Feb;16(2):77-91.) Number: 995 Which of the following local anaesthetic drugs is NOT a chiral molecule? A. Bupivacaine B. Mepivacaine C. Ropivacaine D. Lignocaine E. Prilocaine Select the single best answer ABCDE Correct Answer: D A chiral molecule is one that is not superimposable on its mirror image; it has the property of rotating the plane of polarisation of plane-polarised monochromatic light that is passed through it. This phenomenon is called optical activity. Chiral molecules all have at least one asymmetric carbon atom, i.e. an atom surrounded by four different functional groups. Three-dimensionally, these optical isomers are characterised by different configurations in space and cannot be merged into one another by simple rotation. This allows the compounds to exist in at least two forms as mirror images of each other, which are not superimposable. The isomeric forms are totally identical regarding physiochemical properties with the exception of the rotation of plane light.

Number: 93 For every increase in PaCO2 of 1 mmHg, the subsequent rise in cerebral blood flow (CBF) is: A. 0.5 ml/100 g/min. B. 1.5 ml/100 g/min.

594

C. 5 ml/100 g/min. D. 10 ml/100 g/min. E. The elevation in CBF in response to hypercarbia is non-linear in the range 30 - 50 mm Hg.. Select the single best answer ABCDE Correct Answer: B Over a range of PaCO2 between 20-80 mmHg, a linear relationship between PaCO2 and CBF exists such that CBF increases by about 1.5 ml/100 g/min for every increase in PaCO2 of 1 mmHg. Outside of these limits the relationship is non-linear, with the overall relationship describing a sigmoid curve. The lowest PaCO2 achievable by hyperventilation is 10 mmHg, which correlates with a CBF of 18-20 ml/ 100 g/ min. PaCO2 > 80 mmHg are not typically seen thus, in practice, the relationship is described as linear. ROGERS, M.C ET AL (EDS); Principles and Practice of Anesthesiology, Mosby, 1993, p 1608. RENAL AND NEURO PHYSIOLOGY TESTS Number: 93 For every increase in PaCO2 of 1 mmHg, the subsequent rise in cerebral blood flow (CBF) is: A. 0.5 ml/100 g/min. B. 1.5 ml/100 g/min. C. 5 ml/100 g/min. D. 10 ml/100 g/min. E. The elevation in CBF in response to hypercarbia is non-linear in the range 30 - 50 mm Hg.. Select the single best answer ABCDE Correct Answer: B Over a range of PaCO2 between 20-80 mmHg, a linear relationship between PaCO2 and CBF exists such that CBF increases by about 1.5 ml/100 g/min for every increase in PaCO2 of 1 mmHg. Outside of these limits the relationship is non-linear, with the overall relationship describing a sigmoid curve. The lowest PaCO2 achievable by hyperventilation is 10 mmHg, which correlates with a CBF of 18-20 ml/ 100 g/ min. PaCO2 > 80 mmHg are not typically seen thus, in practice, the relationship is described as linear. ROGERS, M.C ET AL (EDS); Principles and Practice of Anesthesiology, Mosby, 1993, p 1608.

595

Number: 95 Which of the following organs does autoregulation predominate in the control of blood flow? 1. Renal. 2. Coronary. 3. Eye. 4. Uterine. A: 1,2,3 Correct B: 1,3 Correct C: 2,4 Correct D: 4 Correct E: All Correct ABCDE Correct Answer: B Autoregulation is seen under most conditions in the control of renal, cerebral, and choroidal blood flow. If basal metabolic conditions are maintained constant, it is also demonstrable in the coronary circulation. Here however, large fluctuations in blood flow, predominantly under the control of metabolic byproducts, is the norm. Likewise, in other organs, autoregulatory mechanisms exist, but blood flow is influenced to a greater degree by extrinsic factors, eg sympathetic nervous system stimulation in the uterine bed. The blood flow of an organ is said to be autoregulated when mechanisms exist which are intrinsic to the vasculature of that organ which act to maintain blood flow constant in the face of changes in perfusion pressure. The blood flow of any vasculature bed may be defined by the following equation, which is an analog of Ohm's law for electrical resistance: Blood flow = perfusion pressure / vascular resistance Perfusion pressure = systemic pressure - venous pressure (or tissue pressure if this is greater; eg. ICP or IOP) Autoregulatory mechanisms often involve locally initiated changes in resistance in response to changes in perfusion pressure. The three classic mechanisms postulated were (1)Myogenic (2)Metabolic and (3)Tissue- pressure, however, mechanisms specific to certain beds have also been identified. These include, glomerulo-tubular feedback of the renal bed, and the neurogenic mechanism postulated for cerebral blood flow. ROGERS,M.C. ET AL (EDS); Principles and Practice of Anesthesiology, Mosby, 1993, p 2244.

596

Number: 345 With regard to body fluids: 1. In a 70 kg man total body water (TBW) in litres represents approximately 60% of the body weight. 2. 2/3rd of the TBW is located in the extracellular fluid (ECF) and 1/3rd in the intracellular fluid (ICF). 3. The major ion determining the osmolarity of the ECF is sodium. 4. Most of the calcium ions in the body fluids are located intracellularly. A: 1,2,3 Correct B: 1,3 Correct C: 2,4 Correct D: 4 Correct E: All Correct ABCDE Correct Answer: B 1. Thus the TBW is approximately 42L in a 70kg man. 2. 2/3rd (28L) is located in the ICF and 1/3rd (14L) in the ECF. 3. The normal plasma osmolarity is about 290 mOsm/L, approximately 280 mOsm being contributed by sodium chloride and sodium bicarbonate. 4. The calcium ion concentration in the ICF is very low. Number: 346 Concerning the GFR (glomerular filtration rate): 1. Usually the GFR is equal to the RPF (renal plasma flow). 2. The GFR can be measured using the clearance of PAH (para amino hippuric acid). 3. A low value always indicates renal disease. 4. The GFR will fall if there is obstruction to flow of urine through the ureter. A: 1,2,3 Correct B: 1,3 Correct C: 2,4 Correct D: 4 Correct E: All Correct ABCDE Correct Answer: D 1. Normally only about 20% of the RPF is filtered (the rest goes on to perfuse the peritubular capillaries). This ratio (GFR/RPF) is called the filtration fraction.

597

2. The GFR is measured using a substance such as inulin which is freely filtered but not reabsorbed or secreted by the tubules. PAH is secreted by the proximal tubule such that its clearance is higher than the GFR. 3. Obstruction to urine flow and under perfusion of the kidney also cause the GFR to fall. 4. Obstruction to the flow of urine through the urinary tract raises the hydrostatic pressure in the Bowman's capsule. This force opposes the filtration of fluid through the glomerular capillaries. Number: 347 With regard to the glomerular capillaries: 1. Constriction of the afferent arteriole causes the capillary blood pressure to fall. 2. The plasma glucose concentration is the same as that in the glomerular filtrate. 3. As blood flows through the capillaries the haematocrit increases. 4. The oncotic pressure of the plasma leaving the capillaries is lower than the general systemic oncotic pressure. A: 1,2,3 Correct B: 1,3 Correct C: 2,4 Correct D: 4 Correct E: All Correct ABCDE Correct Answer: A 1. The afferent arteriole is located before the glomerular capillaries so that vasoconstriction causes less blood to enter the capillaries. 2. All substances with a molecular weight lower than about 5,500 dalton are freely filtered at the glomerulus such that their concentration in the plasma is the same as that in the filtrate. 3. Only the blood plasma is filtered, not the cells. Thus as water is filtered the red cells remaining behind become more concentrated. 4. As blood flows through the glomerular capillaries water is filtered into the Bowman's capsule so that the proteins that are left behind in the blood become more concentrated causing the oncotic pressure to increase. The oncotic pressure of the plasma entering the peritubular capillaries is high (about 35 mmHg) and this helps in the reabsorption of water from the proximal tubule.

598

Number: 348 Which statement is FALSE regarding glycosuria: A. It occurs when the glucose transport mechanism in the proximal tubule becomes saturated. B. It may occur even at normal plasma glucose levels in conditions in which the TmG is low. C. It may not occur in an old diabetic patient. D.It may be accompanied by natriuresis and polyuria. E. It is accompanied by potassium ion retention. Select the single best answer ABCDE Correct Answer: E A. When the transport mechanism becomes saturated not all of the glucose that was filtered at the glomerulus can be reabsorbed and some will be excreted in the urine. At this point the filtered load exceeds the maximum rate of reabsorption. B. In conditions in which there is a lack of glucose carriers (low TmG), the transport mechanism will become saturated even at low plasma glucose levels. C. With ageing the GFR decreases such that the glucose threshold increases (the filtered load represents the GFR multiplied by the plasma glucose concentration). D. Glucosuria results in an osmotic diuresis in which there is decreased sodium and water reabsorption from the proximal tubule. E. Decreased proximal reabsorption of sodium and water leads to increased rate of delivery of sodium and water to the distal segment of the nephron leading to increased excretion of potassium ions and potassium depletion. Number: 349 The actions of angiotensin II include: 1. Peripheral vasodilatation. 2. Stimulation of thirst. 3. Inhibition of aldosterone secretion. 4. Stimulation of ADH secretion. A: 1,2,3 Correct B: 1,3 Correct C: 2,4 Correct D: 4 Correct E: All Correct ABCDE

599

Correct Answer: C 1. Angiotensin II is a potent vasoconstrictor agent and at high blood concentration increases the peripheral resistance. 2. Angiotensin II acts on the thirst centre to stimulate thirst and drinking. 3. Angiotensin II acts on the adrenal gland to stimulate the secretion of aldosterone and thereby bring about retention of salt and water. Number: 350 The collecting duct of the renal tubule: 1. Passively secretes ammonia. 2. Becomes more permeable to water in the presence of ADH. 3. Passively secretes potassium ions. 4. Is the primary site for reabsorption of water. A: 1,2,3 Correct B: 1,3 Correct C: 2,4 Correct D: 4 Correct E: All Correct ABCDE Correct Answer: A 1. Ammonia is the main urinary buffer and is synthesised in the kidney. 2. ADH stimulates the activity of the enzyme adenyl cyclase in the epithelium making it more permeable to water. 3. In the cortical collecting duct potassium is passively secreted out of the tubular cell into the tubular lumen down its concentration gradient. 4. The primary site for reabsorption of water is the proximal tubule - about 70% of the GFR being reabsorbed proximally. Number: 351 The action of aldosterone on the kidney causes: 1. Renal retention of sodium. 2. Increased excretion of hydrogen ions in the urine. 3. A rise in the plasma bicarbonate concentration. 4. Renal retention of potassium.

600

A: 1,2,3 Correct B: 1,3 Correct C: 2,4 Correct D: 4 Correct E: All Correct ABCDE Correct Answer: A 1. Aldosterone stimulates distal reabsorption of sodium. 2. Aldosterone stimulates distal secretion of hydrogen ions. 3. For every hydrogen ion secreted into the tubular lumen, one bicarbonate ion is added to the blood. 4. Aldosterone stimulates distal secretion and excretion of potassium. Number: 352 Secretion of aldosterone is stimulated by: 1. A fall in plasma potassium concentration. 2. A rise in pressure in the afferent arteriole. 3. A rise in plasma osmolality. 4. A fall in blood volume. A: 1,2,3 Correct B: 1,3 Correct C: 2,4 Correct D: 4 Correct E: All Correct ABCDE Correct Answer: D 1. A rise in plasma potassium ion concentration acts directly on the adrenal gland to stimulate aldosterone secretion. Aldosterone in turn increases the excretion of potassium thereby bringing the plasma potassium ion level back down towards normal. 2. A fall in pressure in the afferent arteriole causes renin release from the JG cells. 3. Aldosterone secretion is not affected by changes in plasma osmolality. A rise in plasma osmolality stimulates ADH secretion. 4. A fall in blood volume results in decreased RBF and a fall in pressure in the afferent arteriole.

601

Number: 353 Hyperkalaemia may occur as a result of: 1. Uraemia (end-stage renal disease when the GFR is very low). 2. Use of diuretics such as spironolactone which antagonise the action of aldosterone. 3. Metabolic acidosis. 4. Use of loop diuretics such as frusemide and ethacrinic acid. A: 1,2,3 Correct B: 1,3 Correct C: 2,4 Correct D: 4 Correct E: All Correct ABCDE Correct Answer: A 1. The urine is the main route for excretion of potassium. When a large number of nephrons have been destroyed the kidney can no longer maintain potassium balance. 2. Spironolactone inhibits distal sodium/potassium exchange and thereby causes retention of potassium. 3. In metabolic acidosis hydrogen ions enter the cell in exchange for potassium ions which move out into the ECF. However, hyperkalaemia occurs mostly in the setting of inorganic acidoses like renal failure (retention of sulphuric and phosphoric acids) or hyperchloraemic acidosis. 4. Loop diuretics inhibit the reabsorption of NaCl from the loop of Henle thereby increasing the delivery of sodium and water to the distal segment of the nephron. The larger the distal sodium load the more sodium is reabsorbed in exchange for potassium in the collecting duct. Number: 354 As the tubular fluid flows through the proximal tubule: 1. The urea concentration falls. 2. The concentration of glucose falls. 3. There is a large fall in pH to about pH 5.0. 4. The osmolality of the fluid does not change appreciably. A: 1,2,3 Correct B: 1,3 Correct C: 2,4 Correct D: 4 Correct

602

E: All Correct ABCDE Correct Answer: C 1. Only about 50% of the urea that is filtered at the glomerulus is reabsorbed from the proximal tubule. As water is reabsorbed the urea that is left behind in the lumen becomes more concentrated (as a result urea is passively reabsorbed down its concentration gradient). 2. All of the glucose filtered at the glomerulus is normally reabsorbed in the early part of the proximal tubule. 3. A large quantity of hydrogen ions are secreted by the proximal tubule into the tubular fluid, however they are buffered by the bicarbonate present in the lumen so that there is little change in pH of the fluid as it flows through the proximal tubule (the process is isohydric). 4. The reabsorption of solute and water from the proximal tubule occur at the same rate such that the osmolality of the tubular fluid does not change. The concentration of substances such as glucose and amino acids decrease while the concentration of other substances such as urea increase such that the fluid leaving the proximal tubule is isosmotic with plasma. Number: 373 The following are true of tubular function: 1. 50% of sodium has been reabsorbed by the start of the distal convoluted tubule 2. Creatinine is secreted into the proximal tubule 3. Aldosterone acts predominantly on cells in the collecting duct 4. Phosphate resorption is controlled by parathyroid hormone A: 1,2,3 Correct B: 1,3 Correct C: 2,4 Correct D: 4 Correct E: All Correct ABCDE Correct Answer: C By the time the tubular fluid reaches the distal tubules, all but about 8 per cent of the sodium has already been reabsorbed. About 65 per cent of this is reabsorbed in the proximal tubules because of the active transport of sodium by the proximal tubular epithelial cells. Administration of parathyroid hormone causes immediate and rapid loss of phosphate in the urine. This effect is caused by diminished proximal tubular reabsorption of phosphate ions. Parathyroid hormone also increases the reabsorption of calcium at the same time that it diminishes phosphate reabsorption. Moreover, it also increases the rate of reabsorption of magnesium ions and hydrogen ions, while it decreases the reabsorption of sodium, potassium,

603

and amino acid ions in much the same way that it affects phosphate. However, the increased calcium absorption occurs in the distal tubules and collecting ducts instead of the proximal tubules. Number: 481 Atrial Natriuretic Peptide: 1. Inhibits Na reabsorption in the loop of Henle. 2. Inhibits renin release. 3. Levels are low in severe congestive cardiac failure. 4. Is a vasodilator. A: 1,2,3 Correct B: 1,3 Correct C: 2,4 Correct D: 4 Correct E: All Correct ABCDE Correct Answer: C 1. Inhibits Na reabsorption in the distal convoluted tubule. 2. Atrial natriuretic peptide inhibits renin release. 3. Levels are high in severe congestive cardiac failure - triggered by stretching of the atrial wall. 4. It therefore acts as a vasodilator by preventing angiotensin II mediated vasoconstriction. Number: 531 Concerning osmolarity: 1. It is defined as mmol of solute per kg of solvent. 2. It may be measured by depression of the boiling point of the solvent. 3. Normal serum osmolarity is 250 mmol per litre. 4. The total osmotic pressure of plasma is about 7 atmospheres. A: 1,2,3 Correct B: 1,3 Correct C: 2,4 Correct D: 4 Correct E: All Correct ABCDE Correct Answer: D

604

1. Osmolarity is defined as mmol of solute per litre of solvent. - Osmolality is defined as mmol of solute per kg of solvent. 2. It may be measured by the depression of the freezing point. 3. The normal serum osmolarity is about 290 mmol per litre. 4. The total osmotic pressure of plasma is 7 atmospheres. Number: 590 By 80 years of age, creatinine clearance expressed as a percentage of the maximum clearance attained during life is: A. 10% B. 20% C. 50% D. 70% E. 90% Select the single best answer ABCDE Correct Answer: C Creatinine clearance progressively declines with increasing age so that, by 80 years of age, it is approximately 50% of its value in the young adult. However, serum creatinine is not elevated in healthy elderly patients because muscle mass, the source of creatinine, also decrease with age. About 40% of the cases of iatrogenic drug intoxication after drug administration occur in the elderly. A significant proportion of these cases can be attributed to the fact that impairment of renal function has gone unrecognised. Number: 867 An increase in arterial PCO2 from 40 to 60 mm Hg will increase cerebral blood flow (CBF) by approximately: A. 20% B. 40% C. 60% D. 80% E. 100% Select the single best answer ABCDE

605

Correct Answer: D Cerebral blood flow increases by about 4% for each increment of 1 mm Hg in the range 30 60 mm Hg. Thus, increasing PaCO2 from 40 to 60 mm Hg will increase CBF by about 80%. Number: 891 In the case of a normal patient, in the supine position, the intracranial pressure (referred to the level of the interventricular foramen) is: A. Less than 5 cm H20. B. 5 -15 cm H20. C. 15 - 20 cm H20. D. 20 - 25 cm H20. E. More than 25 cm H20. Select the single best answer ABCDE Correct Answer: B According to Guyton: "The normal pressure in the cerebrospinal fluid system when one is lying in a horizontal position averages 130 mm water (10 mm Hg), though this may be as low as 70 mm water or as high as 180 mm water even in the normal person. These values are considerably more positive than the - 5.3 mm Hg pressure in the interstitial spaces of the subcutaneous tissue." "Textbook Of Medical Physiology". A Guyton. W.B.Saunders Company -1996 ISBN:0721667732 Number: 893 The partial pressure of carbon dioxide (PCO2) in the cerebro-spinal fluid (CSF) of a normal person during quiet respiration is approximately: A. 34 mm Hg. B. 38 mm Hg. C. 42 mm Hg. D. 46 mm Hg. E. 50 mm Hg. Select the single best answer ABCDE Correct Answer: E

606

According to Nunn "The CSF, cerebral tissue, and jugular venous PCO2 are all approximately equal and about 10 mm Hg higher than the arterial PCO2." "Nunn's Applied Respiratory Physiology." Lumb, A. Butterworth-Heinemann. ISBN 0-75061336-X

STATISTICS TESTS Number: 258 In a clinical trial of a new drug: 1. Randomisation ensures that the allocation of treatment to each patient is a matter of chance. 2. Baseline differences in patients allocated to different treatment groups are eliminated by randomised allocation. 3. Increasing the number of patients allocated to each treatment group will reduce the effect of baseline differences. 4. The null hypothesis is true if there are significant differences in response in the randomly allocated treatment group. A: 1,2,3 Correct B: 1,3 Correct C: 2,4 Correct D: 4 Correct E: All Correct ABCDE Correct Answer: B Number: 326 With regard to data describing a population, the following are true: 1. The mode is the most commonly occurring value 2. The mode is usually different in value from the mean 3. The standard deviation is also referred to as the root mean square deviation 4. In distributions which are markedly skewed, the arithmetic mean is a more appropriate measure than the geometric mean A: 1,2,3 Correct B: 1,3 Correct C: 2,4 Correct D: 4 Correct E: All Correct ABCDE

607

Correct Answer: A M Bland. An Introduction to Medical Statistics, 2nd ed. Oxford University Press, 1995. Number: 355 A study of the intellectually handicapped was performed. 112 subjects were put through program A, and showed an increase in their mean IQ score of 6 points. 115 subjects were put through program B, and showed an increase in their mean IQ score of 4. The p value for this difference was > 0.05 . Which of the following is true? A. The numbers are too large for a Student t-test. B. The study demonstrates the usefulness of program A. C. The distribution of individual values is not important. D. Even though the difference between the means is not significant it would be appropriate to calculate confidence intervals. E. The above results would be found by chance in less than 1:20. Select the single best answer ABCDE Correct Answer: D A. The t-test is designed for small samples but as n increases it approximates the normal distribution. However the t-test should be used in cases where the population standard deviation is not known. B. A is no more useful than B or even simply repeating an IQ test? E. More Number: 374 In a trial of a new drug the following results were obtained:- treatment group 44 improved 16 not improved, placebo group 36 improved 26 not improved. A.The results so obviously show the benefit of treatment that statistical analysis is not required B. The data could be evaluated using the chi-squared test C. Pearson's coefficient of linear regression would be an appropriate significance test D. The numbers are too small to draw any conclusions E. A Student t-test could be used Select the single best answer ABCDE Correct Answer: B

608

A. Nothing is ever that obvious. B. These data are ideal for a chi-squared test. C. There is no linear regression to plot. D. No they're not. E. We are comparing proportions not means. M Bland. An Introduction to Medical Statistics, 2nd ed. Oxford University Press, 1995. Number: 509 For the data series: 2, 1, 6, 4, 2 1. The mode is 4 2. The median is 3 3. The standard deviation is 4 4. The mean is 3 A: 1,2,3 Correct B: 1,3 Correct C: 2,4 Correct D: 4 Correct E: All Correct ABCDE Correct Answer: D Mean = 3: ((2+1+6+4+2)/5) Mode = 2: (The commonest value) Median = 2: (The middle value) SD = 2: (Square root of the sum of (1-3)squared + (1-3)squared + (6-3)squared + (4-3)squared + (2-3)squared / (5-1) M Bland. An Introduction to Medical Statistics, 2nd ed. Oxford University Press, 1995. Number: 516 A clinical trial of 'cross-over' design: 1. Requires fewer patients than does a design comparing independent groups. 2. Uses each patient as his / her own control. 3. Requires that each patient receives all of the treatments under examination. 4. Is most useful for comparing treatments intended to alleviate chronic symptoms. A: 1,2,3 Correct B: 1,3 Correct C: 2,4 Correct

609

D: 4 Correct E: All Correct

ABCDE Correct Answer: E M Bland. An Introduction to Medical Statistics, 2nd ed. Oxford University Press, 1995. Number: 610 In order to calculate the 'Number Needed to Treat' (NNT) the following values are required: 1. The number who had the 'control' therapy. 2. The number who had the experimental intervention. 3. The number who responded to the 'control' therapy. 3. The number who responded to the experimental intervention. A: 1,2,3 Correct B: 1,3 Correct C: 2,4 Correct D: 4 Correct E: All Correct ABCDE Correct Answer: E The 'Bandolier' website has been a staunch advocate of NNTs for many years. The following quotations are taken from this site. Statistical ways to express outcomes of clinical trials include P values, odds ratios, relative risk and relative risk reduction or increase. All may have their place, but they are difficult outputs for the non-specialist to interpret. In order to overcome this, the NNT is increasingly being used. The NNT, as the name implies, is an estimate of the number of patients that would need to be given a treatment for one of them to achieve a desired outcome who would not have achieved it with control. The NNT should specify the characteristics of patients being treated, the intervention and its duration, and the outcome being measured. Taking a hypothetical example from a randomised trial: 50 patients were given ibuprofen, and 27 of them had pain relief for at least 6 hours. 50 patients were given placebo, and 10 of them had pain relief for at least 6 hours.

610

The NNT is therefore 1/((27/50) - (10/50)) = 1/(0.54 - 0.20) = 1/0.34 = 2.9 Number: 686 A recently discovered treatment for HIV AIDS significantly extends the lifespan of the patient, but does not prevent the disease or lead to its cure. Given this scenario, which of the following statements about HIV AIDS is true? A. Its incidence will increase. B. Its prevalence will increase. C. Its incidence will decrease. D. Its prevalence will decrease. E. Both the incidence and prevalence will increase. Select the single best answer ABCDE Correct Answer: B As the length of survival from HIV AIDS increases because of the new treatment (assuming the rate of new cases diagnosed remains unchanged), the number of cases present at any point in time (prevalence) increases. Incidence, based on the number of new cases only, remains unchanged. Number: 724 A random selection of 100 female medical students agree to participate in a study of the possible effects of the contraceptive pill. They are followed prospectively for a period of six years to see if there is an association between the incidence of ovarian cysts (demonstrated by ultrasound) and the use of the pill. How can this study be best described? A. Case-control study. B. Randomised controlled clinical trial. C. Cross-sectional study. D. Cohort study. E. Cross-over study. Select the single best answer ABCDE Correct Answer: D

611

In a case-control study cases (those with the problem) are compared with controls (those without the problem) with regard to the incidence of a specific risk factor (such as use of the pill). In a randomised controlled clinical trial some members of a cohort with a particular condition are given one treatment while others receive a different (often 'standard') treatment or placebo. A cross-sectional study is a 'snapshot' or 'point in time' study. In a cross-over study patients are exposed to at least two treatment modalities (+/- placebo) in sequence. The order of exposure to the different treatments is determined by the experimental design and is often random. Number: 725 You have performed a study designed to compare the effects of general (GA) as opposed to epidural anaesthesia (EA) on neonatal outcome in a group of patients undergoing elective Caesarean section. 50 patients are randomly allocated to either the GA or EA group. Which of the following tests would be most appropriate for assessing whether there are any differences in the neonates Apgar scores at 1 minute? A. Analysis of variance (ANOVA). B. Correlation coefficient. C. Student's 'T' test. D. Chi squared test. E. Logistic regression. Select the single best answer ABCDE Correct Answer: C The question relates to assessing group differences for ordinal data (the Apgar score). The Student's 'T' test is the appropriate test. If the data being compared between the two group were nominal (categorical) then a Chi squared test would be appropriate. If there was a third anaesthetic technique (eg spinal anaesthesia) comparisons should be made using analysis of variance. A correlation coefficient defines the strength of association between two variables, not the differences between groups. Logistic regression describes the relationship between a dichotomous response variable and a set of explanatory variables.

612

Number: 731 A biochemical assay for a newly-discovered hormone is stated to have a reference interval of between 0 and 7 picog/dl. Suppose now that this reference interval is (arbitrarily) reduced to between 0 and 3 picog/dl. The effect of reducing the reference interval will be to: A. Increase the test's specificity. B. Decrease the number of false-positive results. C. Increase the number of false-negative results. D. Increase the sensitivity of the test. E. Not alter the test's sensitivity or specificity. Select the single best answer ABCDE Correct Answer: D Narrowing the reference interval of a test increases the test's sensitivity, because there will be fewer false negatives in this range. However, increasing sensitivity automatically decreases specificity by increasing the number of false positives. This relationship underscores the principle that there is always a trade-off when increasing either sensitivity or specificity; increasing one always decreases the other, so they would never remain unaltered. Number: 744 The most appropriate statement applied to the statistical term 'validity' is that it: A. Expresses the degree to which two things are related. B. Implies that the results of a test can be reproduced. C. Describes how well a study measures what it purports to measure. D. Measures the strength of relationship between cause and effect. E. Indicates the probability of obtaining a given result by chance alone. Select the single best answer

ABCDE Correct Answer: C A. 'Association' expresses the degree to which two things are related. B. Reliability implies that the results of a test can be reproduced. C. Validity describes how well a study measured what it was designed to measure. D. Relative risk measures the strength of a cause-and-effect relationship. E. The P-value indicates the probability of obtaining a given result by chance.

613

Number: 785 The standard chi-squared test for a 2 by 2 contingency table is valid only if: 1. Both variables are continuous. 2. At least one variable is from a Normal distribution. 3. All the observed frequencies are greater than five. 4. All the expected frequencies are greater than five. A: 1,2,3 Correct B: 1,3 Correct C: 2,4 Correct D: 4 Correct E: All Correct ABCDE Correct Answer: D This question is a modified version of one which appears in Martin Bland's excellent book on medical statistics. (Follow the web link for publication details.) 80% of 4 is greater than 3, so all expected frequencies must exceed 5. The sample size can be as small as 20, if all row and column totals are 10. Number: 786 In a chi-squared test for a 5 by 3 contingency table: 1. Observed frequencies are compared to expected frequencies 2. There are 15 degrees of freedom. 3. At least 12 cells must have expected values greater than 5. 4. All the observed values must be greater than 1. A: 1,2,3 Correct B: 1,3 Correct C: 2,4 Correct D: 4 Correct E: All Correct ABCDE Correct Answer: B This question is a modified version of one which appears in Martin Bland's excellent book on medical statistics. (Follow the web link for publication details.)

614

(5-1) * (3-1) = 8 degrees of freedom, 80% * 15 = 12 cells must have expected frequencies greater than 5. It is acceptable. for an observed frequency to be zero. Number: 787 Fisher's exact test for a contingency table: 1. Applies to 2 by 2 tables. 2. Usually gives a larger probability than the ordinary chi-squared test. 3. Is suitable when expected frequencies are small. 4. Is difficult to calculate when the expected frequencies are large. A: 1,2,3 Correct B: 1,3 Correct C: 2,4 Correct D: 4 Correct E: All Correct ABCDE Correct Answer: E This question is a modified version of one which appears in Martin Bland's excellent book on medical statistics. (Follow the web link for publication details.) The factorials of large numbers can be difficult to calculate. Number: 788 When an odds ratio is calculated from a two by two table: 1. The odds ratio is a measure of the strength of the relationship between the row and column variables. 2. If the order of the rows and the order of the columns is reversed, the odds ratio will be unchanged. 3. The ratio may take any positive value. 4. The odds ratio will be changed to its reciprocal if the order of the columns is changed. A: 1,2,3 Correct B: 1,3 Correct C: 2,4 Correct D: 4 Correct E: All Correct ABCDE Correct Answer: E

615

This question is a modified version of one which appears in Martin Bland's excellent book on medical statistics. (Follow the web link for publication details.) The more closely related the variables are, the bigger the odds ratio will be. Reversing the order of both rows and columns turns the ad/bc formula to da/cb, the same, but reversing only the columns gives us bc/ad. The ratio of the proportions, or relative risk, would be (a/(a+c))/(b/(b+d)). Number: 789 McNemar's test could be used to: 1. Look at the relationship between cigarette smoking and respiratory symptoms in a group of asthmatics. 2. Examine the change in PEFR in a group of asthmatics from winter to summer. 3. Compare the number of cigarette smokers among a group of cancer cases and a random sample of the general population. 4. Compare the numbers of cigarette smokers among cancer cases and age and sex matched healthy controls. A: 1,2,3 Correct B: 1,3 Correct C: 2,4 Correct D: 4 Correct E: All Correct ABCDE Correct Answer: D This question is a modified version of one which appears in Martin Bland's excellent book on medical statistics. (Follow the web link for publication details.) The test compares proportions in matched samples. For a relationship, we use the chi-squared test. PEFR is a continuous variable, we use the paired t method. For two independent samples we use the chi-squared test. Number: 851 A new test ('T') is being evaluated for its sensititivity in diagnosing a disease state 'D'. A group of 200 patients, whose disease-state (D+ or D-) is correctly known are surveyed with the new test. The results of the survey (T+ or T-) are: D+ DT+ 40 60 T80 20 The sensitivity of the test expressed as a percentage is:

616

A. 20% B. 33% C. 40% D. 67% E. 90% Select the single best answer ABCDE Correct Answer: B Sensitivity is the ability of a test to include those who have a diagnosis. In the table, 120 people who were screened have the disease. Only 40 of these 120 people were identified by a positive screening test; therefore, sensitivity is 33%.

SURGERY TESTS A normal adult has a splenectomy following a road accident. The sequelae would include: 1. Thrombocytosis. 2. Decreased destruction of RBCs. 3. Increased susceptibility to infection. 4. Decreased serum transport of iron. A: 1,2,3 Correct B: 1,3 Correct C: 2,4 Correct D: 4 Correct E: All Correct ABCDE Correct Answer: A Absence of a spleen in healthy adults usually has few clinical consequences. RBC count and indices do not change, but RBCs with cytoplasmic inclusions may appear, eg: Heinz bodies, Howell-Jolly bodies, and siderocytes. Granulocytosis occurs immediately but is replaced after several weeks by lymphocytosis and monocytosis. Platelets are usually increased, and may stay at levels of 400,000-500,000/ul for over a year. Even more striking thrombocytosis may occur following splenectomy for haemolytic anaemia. A platelet count >1,000,000 is not an indication for anticoagulants, but antiplatelet drugs such as aspirin may help to prevent thrombosis.

617

In adults a temporary reduction in antibody production may occur, but children are very susceptible to infections by encapsulated organisms such as pneumococci and meningococci. The proposed mechanism is defective opsonization and clearance. References WAY, L.W; Current Surgical Diagnosis and Treatment, 9th Ed., Lange, 1991, pp 598-9. Number: 27 Which of the following is true of traumatic dislocation of the hip? A. Most occur in the elderly. B. Posterior dislocations are more common than anterior. C. The obturator nerve is most commonly injured. D. Recurrent dislocation is common. E. All of the above. Select the single best answer ABCDE Correct Answer: B Traumatic dislocation of the hip occurs in the active years of life and is usually the result of severe trauma unless pre-existing disease is present. They may be classified according to the direction of displacement of the femoral head. Posterior dislocation is most common and classically occurs in the context of an MVA; impact from ahead to the sitting victim giving the classic signs of a leg which is shortened , adducted , and medially rotated. Sciatic nerve injury is the most common nerve injury in posterior dislocations. The obturator nerve may be injured in anterior dislocations. Fracture of the head or shaft of the femur is common. Fracture of the acetabulum occurs less commonly; nerve injury is more common in this situation. In the abscence of acetabular or capital femoral fracture, recurrence is uncommon. References WAY,L.W; Current Surgical Diagnosis and Treatment, 9th Ed., Lange, 1991, pp 1022-24. Number: 28 Which of the following are true of a properly functioning intra-aortic balloon pump (IABP)? 1. It improves coronary blood flow and decreases afterload. 2. The tip should be positioned just distal to the left subclavian artery. 3. Helium rather than CO2 should be used for balloon inflation. 4. It can only be used in a heparinised patient. A: 1,2,3 Correct B: 1,3 Correct C: 2,4 Correct D: 4 Correct E: All Correct

618

ABCDE Correct Answer: B The IABP is a long narrow balloon placed in the thoracic aorta, usually percutaneously from the groin. The balloon is inflated during diastole, displacing blood from the thoracic aorta and increasing the aortic pressure. Thus it increases coronary blood flow by increasing diastolic coronary perfusion pressure. During systole the balloon is deflated, giving the left ventricle a lower afterload into which to eject. This reduces myocardial wall tension, decreasing myocardial oxygen consumption. The IABP drive console consists of a pressured gas reservoir that is connected to the balloon supply line through a solenoid valve that is electronically controlled. The gas used to inflate the balloon is either CO2 or helium. The advantage of CO2 is its increased solubility, thereby reducing the consequences of an accidental balloon rupture with gas embolisation. However, the advantage of helium is its decreased density, which decreases Reynold's Number. and allows the same flow through a smaller drive line. A tube with a smaller diameter decreases the injury to the artery from the puncture and also decreases the risk of infection and arterial thrombosis. During extended use , anticoagulation is advised. In the immediate post CPB period, anticoagulants are not used until drainage from the chest drains is acceptable (100-150 ml/hr). Heparin is the agent of choice. The balloon is ideally positioned so that its tip is at the junction of the descending aorta and the aortic arch. This promotes the removal of blood from the proximal aorta and minimizes the risk of renal artery occlusion by the balloon. References HENSLEY,F.A & MARTIN,D.E. (EDS); The Practice of Cardiac Anesthesia, Little Brown & Co., 1990, pp 647-8. Number: 32 Carcinoma of the oesophagus : 1. Is usually adenocarcinoma if it occurs at the lower end of the oesophagus 2. Metastasises early to the liver and the lungs. 3. May be resected if it occurs above the aortic arch. 4. May be associated with the Plummer-Vinson Syndrome. A: 1,2,3 Correct B: 1,3 Correct C: 2,4 Correct D: 4 Correct E: All Correct ABCDE

619

Correct Answer: D Most malignant tumours of the oesophagus are carcinomas.Those arising from segments proximal to the cardia are almost always SCCs. Those of the lower oesophagus, in direct continuity with the cardia of the stomach , are usually adenocarcinomas. They are considered to be gastric tumours that have extended proximally. Primary adenocarcinomas of the oesophagus are rare (3-4%) and may arise anywhere in the oesophagus eg: Barrett's oesophagus. By the time of presentation, the tumour is usually a fungating lesion of approximately 5cm. Ulceration of its base is common and obstruction is a relatively early event. They disseminate by direct invasion of mediastinal structures, via bloodstream, and via lymphatics. Metastases to lymph nodes in neck, mediastinum or coeliac areas are present in 80% at time of diagnosis. 80% extend extramurally to involve directly other structures, principally the trachea, left main bronchus or aorta. A TOF is present in 10% at time of diagnosis. Metastases to lung, liver, bone and adrenals occur as a late event. Aetiology includes the seven " S "s: smoking, sex, sun, spices, spirits, scarring,and sundry. Other premalignant syndromes are chronic Fe deficiency, oesophageal stasis (eg:achalasia), Barrett's oesophagus, reflux oesophagitis and congenital tylosis of the oesophagus. The Plummer-Vinson Syndrome typically afflicts fair haired, blue eyed women. It is associated with oesophageal webs and pernicious anaemia, and has a definite association. Carcinoma of the oesophagus may be treated by surgery, radiotherapy or chemotherapy. Nonresectability is suggested by direct spread to the tracheobronchial tree or aorta; TOF or hoarseness, or tumours > 10cm. Overall, about 50% are resectable at the time of presentation and about 75% after radiation. If life expectancy is greater than a few months, surgical resection is recommended because it provides the best palliation Usually Squamous Ca occurs in the upper 2/3 of the oesophagus and Adeno carcinoma in the lower third. Spread to the liver occurs late in the course of the disease but local infiltration occurs early. Occurrence above the level of the aortic arch is usually irresectable. References: THOMPSON,A.D,COTTON,R.E: Lecture Notes On Pathology,3rd Ed.,pp 184. Number: 110 A 30 year old man is stable but has evidence of thoraco-abdominal trauma following a motor vehicle accident. Which is the best method of excluding diaphragmatic rupture? A. CXR. B. Thoracoabdominal CAT scan. C. Peritoneal lavage. D. Immediate laparotomy. E. None of the above. Select the single best answer

620

ABCDE Correct Answer: A The diagnosis is best made at laparotomy. In many cases, multiple thoraco-abdominal injuries associated with this injury will necessitate an early laparotomy, however, in a stable patient, immediate laparotomy is not indicated. Rather, laparotomy and repair as the definitive treatment should follow thorough assessment and workup for this procedure. A CXR is the most likely of these investigations to indicate the injury. CAT scanning is insensitive; peritoneal lavage is non-specific. Traumatic injury to the diaphram results from both blunt and penentrating thoraco-abdominal trauma. It is said to occur in 5% of cases of the former. The most common mechanism of injury is blunt thoraco-abdominal trauma following motor vehicle accidents. A 5 year review of all cases handled at the Sunnybrook Trauma Centre in Toronto, Canada, generated the following results. Left sided impact generally results in only left sided injury, whereas frontal or right sided, results in left sided or bilateral injury. Isolated diaphragmatic injury is uncommon; it is usually associated with multiple injuries, which include fractured ribs, haemothorax, pneumothorax, splenic rupture, liver contusion, ruptured viscus, and/or retroperitoneal haemorrhage. In the group of 43 studied in the Sunnybrook review, no patient died directly as a result of diaphragmatic injury, or respiratory insufficiency resulting from it, nor was the requirement for postoperative ventilation concluded to on the basis of the diaphragmatic injury in isolation. The CXR remains the most valuable tool for the preoperative diagnosis of diaphragmatic rupture. Signs include visualization of the nasogastric tube in the left chest cavity, or visualization of a gastric bubble in the chest. In many cases, the CXR is abnormal but not diagnostic. A CXR demonstrating obliteration of the diaphragmatic border in a patient suffering thoraco-abdominal trauma should arouse suspicion. Other diagnostic tests add little to this diagnosis. CAT scans are not reliable. Peritoneal lavage when abnormal are nonspecific. Most of the patients in this series did not require immediate surgery, allowing time for full radiologic evaluation and preparation for surgery. References PAGLIARELLO, G; " Traumatic Injury to the Diaphram: Timely Diagnosis and Treatment ", The Journal of Trauma, vol 33, no2, 1992, pp 194-7. Number: 280 Complications of subtotal thyroidectomy include: 1. Aphonia 2. Recurrence of thyrotoxicosis 3. Myxoedema 4. Hypercalcaemia A: 1,2,3 Correct B: 1,3 Correct C: 2,4 Correct D: 4 Correct E: All Correct

621

ABCDE Correct Answer: A Subtotal thyroidectomy is a surgical treatment of hyperthyroidism. Aphonia may result from cutting the recurrent laryngeal nerve Recurrence of thyrotoxicosis may occur if not enough thyroid is removed Myxoedema may result if too much thyroid is removed Hypocalcaemia is a possibility when the parathyroids have been damaged / removed. Number: 288 In postgastrectomy syndromes: 1. Early dumping is due to rebound hypoglycaemia 2. Anaemia is seen in about half of the cases 3. Folate deficiency is common 4. Backaches are causatively linked A: 1,2,3 Correct B: 1,3 Correct C: 2,4 Correct D: 4 Correct E: All Correct ABCDE Correct Answer: C The presence of a malabsorption syndrome has been documented frequently in patients after subtotal gastrectomy. Steatorrhea is more common with a Billroth II than a Billroth I type of anastomosis. Folate deficiency is rare. Because the duodenum is the principal site of absorption of iron and calcium, in patients with a Billroth II anastomosis, impaired absorption of calcium and iron also may develop. Occult metabolic bone disease occurs frequently in this setting. Number: 361 With regard to the thoracic outlet syndrome: 1. Treatment can include 1st rib resection. 2. Compression of the lower trunk of the brachial plexus is the most common cause of symptoms. 3. Compression of the neurovascular bundle between scalenus anterior and medius can be the cause. 4. It can be caused by the presence of a cervical rib.

622

A: 1,2,3 Correct B: 1,3 Correct C: 2,4 Correct D: 4 Correct E: All Correct ABCDE Correct Answer: E Thoracic outlet syndrome results from compression of the neurovascular bundle at the thoracic outlet between the first rib and the clavicle or between the scalenus anterior muscle and the scalenus medius muscle. Aetiology includes presence of a cervical rib, anomalous first rib, anomalous fibrous bands, anomalous muscle insertions, hypertrophy of the scalene muscles, hypertrophy of the subclavius muscle, anomalous vessels, malunited clavicular fracture, and other osseous lesions. Clinical manifestations are frequently vague and obscure, including shoulder pain; pain along the medial aspect of the arm and forearm in the ulnar distribution; more proximally sited pain in the shoulder girdle, neck, or face; chest pain that can be mistaken for angina; weakness, numbness, and parasthesias; arterial insufficiency from subclavian artery compression, which results in coolness and numbness at rest or with use, exertion, and fatigue of the upper extremity; venous insufficiency from subclavian vein obstruction, which may cause oedema, cyanotic discoloration, and venous collateralisation; and mixed symptom patterns of neural as well as vascular compression. The most common symptom complex occurs as a result of compression of the lower trunk of the brachial plexus. Patients may complain of more proximal pain in the anterior and posterior aspects of the shoulder girdle similar to the symptomatology of C5-C6 disc herniation. Symptoms may also develop due to compression of the subclavian vessels. Number: 665 Colo-vesical fistula is a well-recognised complication of: 1. Crohn's disease. 2. Diverticular disease. 3. Colonic carcinoma. 4. Ischaemic colitis A: 1,2,3 Correct B: 1,3 Correct C: 2,4 Correct D: 4 Correct E: All Correct ABCDE

623

Correct Answer: A The common causes of colo-vesical fistula are: diverticular disease, carcinoma and inflammatory bowel disease. Of the inflammatory bowel conditions, Crohn's disease is much likely to cause a fistula than ulcerative colitis. The pathological process in Crohn's disease is essentially trans-mural - in contrast to the case of ulcerative colitis where the process is predominantly mucosal and submucosal. As a result, fistula formation is much more frequent in those with Crohn's disease. Number: 705 Known risk factors for the development of pancreatic carcinoma include: 1. Cigarette smoking. 2. Diabetes mellitus. 3. Chronic pancreatitis. 4. Cholelithiasis. A: 1,2,3 Correct B: 1,3 Correct C: 2,4 Correct D: 4 Correct E: All Correct ABCDE Correct Answer: A More than 90 percent of pancreatic cancers are ductal adenocarcinomas, with islet cell tumours constituting the remaining 5 to 10 percent. The distribution of the tumours is head:70%; body: 20%; tail: 10%, Cigarette smoking is the most consistently observed risk factor for the development of pancreatic cancer, with the disease being two to three times more common in heavy smokers than in non-smokers. A meta-analysis of 20 case-control and cohort studies has also shown a higher frequency of pancreatic cancer in persons with long-standing diabetes mellitus. Similarly, chronic pancreatitis been correlated with an increased risk of pancreatic cancer, but there are no convincing data to link cholelithiasis with the development of the tumour. See, for example Warshaw AL, Fernandez-del Castillo C. Pancreatic carcinoma. N Engl J Med. 1992 Feb 13;326(7):455-65

624

Number: 728 Which of the following is the most common cause of obstruction in the superior vena cava (SVC) of adults? A. Lymphoma. B. Goitre. C. Thoracic aortic aneurysm. D. Fibrosing mediastinitis. E. Bronchogenic carcinoma. Select the single best answer ABCDE Correct Answer: E Malignancy is the most common cause of obstruction in the superior vena cava, accounting for more than 95% of cases. In adults, bronchogenic carcinoma accounts for more than 80% of these malignancies, whereas lymphoma accounts for only 10%. (It should be noted that in a recent review of patients with SVC obstruction who were referred for a surgical opinion, lymphoma featured much more commonly than bronchogenic carcinoma as a cause. See: Porte H, Metois D, Finzi L, Lebuffe G, Guidat A, Conti M, Wurtz A. Superior vena cava syndrome of malignant origin. Which surgical procedure for which diagnosis? Eur J Cardiothorac Surg. 2000 Apr;17(4):384-8. Goitre, thoracic aortic aneurysms, and fibrosing mediastinitis are rare causes of superior vena caval obstruction. Superior vena caval syndrome presents with facial swelling, shortness of breath, cough, and orthopnoea. Physical signs include facial and glossal swelling, neck and upper extremity vein distension, and changes in mentation. Venography is often helpful in determining the extent of the superior vena caval obstruction and the presence of collateral circulation. However, only patients with benign disease benefit from bypass with autogenous vein graft. In the remainder, treatment usually consists of radiation therapy to the upper mediastinum. Most patients with lymphoma respond favorably to such treatment, whereas the response is variable at best in those with bronchogenic carcinoma. Although the syndrome may be alleviated quickly, long-term prognosis is based on the histology of the underlying malignancy.

625

Number: 733 A 17-year-old boy sees his physician complaining of a painful lump which has recently developed above his right knee. The pain is worse at night. Physical examination reveals a hard, warm, tender mass. There is a scar in his right groin which he attributes to an inguinal hernia repair he had as a child. An X-ray of the right femur shows an "onionskin" appearance at the location of the mass. Which of the following is the most likely diagnosis? A. Osteomyelitis. B. Ewing's sarcoma. C. Osteosarcoma. D. Chondrosarcoma. E. Eosinophilic granuloma Select the single best answer ABCDE Correct Answer: B Ewing's sarcoma most commonly presents during the teenage years. The radiographic features of Ewing's sarcoma are of a diffuse, mottled, lytic lesion affecting the medullary cavity and cortical bone. An "onionskin" appearance on radiographic examination is seen when the tumour penetrates the cortex and extends into the periosteum producing elevations represented by multiple layers of reactive new bone formation. Recently, an association between inguinal hernia and Ewing's sarcoma has been recognised. See: Cope JU, Tsokos M, Helman LJ, Gridley G, Tucker MA. Inguinal hernia in patients with Ewing sarcoma: a clue to etiology. Med Pediatr Oncol. 2000 Mar;34(3):195-9. Osteosarcoma is the most common malignant bone tumour in children. The diagnosis of osteosarcoma may be suspected from good-quality radiographs. In advanced cases of osteosarcoma, possible findings include cortical destruction; sclerosis; a "sunburst" pattern of new periosteal bone formation; and calcified, soft tissue extensions. Chondrosarcoma is rare in children. It occurs most commonly in the pelvis. Metastasis is usually by local extension. Eosinophilic granuloma presents as a painless or mildly painful swelling in the skull, long bones, ribs, pelvis, or vertebra. Radiographs show a lytic lesion with well-defined borders. Osteomyelitis most commonly affects the femur and tibia. Radiographs are normal early in the course of the disease.

626

Number: 749 A patient presents with symptoms suggestive of renal colic and on plain abdominal X-ray there is a radio-opaque shadow in the line of the appropriate ureter. If this is the culprit stone, it might contain: 1. Xanthine. 2. Calcium Oxalate / Phosphate. 3. Uric acid. 4. Cystine. A: 1,2,3 Correct B: 1,3 Correct C: 2,4 Correct D: 4 Correct E: All Correct ABCDE Correct Answer: C About 75% of renal calculi are radio-opaque and these stones contain calcium salts of oxalate, phosphate and carbonate. Stones composed of cystine are rare, usually redio-opaque, and are associated with metabolic disorders. Stones that contain uric acid and xanthine are radiolucent. Number: 772 A tumour arising in a burn scar is most likely to be: A. Squamous cell carcinoma. B. Malignant Melanoma. C. Fibrosarcoma. D. Basal cell carcinoma. E. None of the above. Select the single best answer ABCDE Correct Answer: A John Marjolini described malignant ulceration developing in burns scars in 1828. Very typically there is a long latent period between the burn and the development of the malignancy (~ 30 years). Histologically the lesion is a squamous cell ('epidermoid') carcinoma. Malignant melanoma, basal cell carcinoma and fibrosarcoma developing in burns scars are all extremely rare.

627

Number: 816 With regard to laparoscopic appendicectomy: 1. The chance of converting to an open procedure is about 10%. 2. The duration of hospitalisation is significantly less than for an open procedure. 3. The incidence of intra-abdominal abscess formation is higher than for an open procedure. 4. The procedure typically takes an hour longer than an open procedure. A: 1,2,3 Correct B: 1,3 Correct C: 2,4 Correct D: 4 Correct E: All Correct ABCDE Correct Answer: A 1. The chance of converting to an open procedure is about 8%. 2. The length of hospital stay following laparoscopic appendicectomy averages of 15 hours less and return to full activity occurs 6.5 days earlier than with conventional appendectomy. 3. The incidence of intra-abdominal abscess formation is about twice that which occurs following an open procedure (3 vs 1.5%). 4. The procedure typically takes an hour longer than an open procedure. - No! it just seems this way. Typically the procedure takes about 15 minutes more. See: Sauerland S, Lefering R, Holthausen U, Neugebauer EA. Laparoscopic vs conventional appendectomy--a meta-analysis of randomised controlled trials. Langenbecks Arch Surg. 1998 Aug;383(3-4):289-95. Number: 848 A 22 year old college student presents to her general practitioner with a "lump in the neck," which she noticed recently. She does not have a history of difficulty swallowing, breathing, or talking. There is no history of change in voice or irradiation to the neck. Clinical examination reveals a hard, solitary nodule within the thyroid gland. Cervical lymph nodes are not palpable. She has no symptoms or signs suggestive of hyperthyroidism. Which of the following should be used to establish a diagnosis in this patient? A. Ultrasound. B. Fine needle biopsy. C. Wedge biopsy. D. Level of thyroid stimulating hormone (TSH). E. Magnetic resonance imaging (MRI). Select the single best answer

628

ABCDE Correct Answer: B Needle biopsy is the most accurate diagnostic screening procedure available to distinguish between a benign and malignant thyroid nodule. It is the preliminary diagnostic step in the investigation of a thyroid nodule. Needle biopsy is contraindicated in patients with a history of neck irradiation because these tumours are usually multifocal and the lesion may be missed. Needle biopsy requires the availability of an experienced cytologist. An ultrasound is not the initial diagnostic test of choice. It will help to distinguish between a cystic and solid lesion, but no more. It is also useful when needle biopsy requires ultrasonic guidance. A wedge biopsy is inappropriate in light of the extremely high sensitivity of needle biopsy. Furthermore, it is an open and more invasive procedure. Determining the level of thyroid stimulating hormone (TSH) will not establish the presence of malignancy. Measuring the TSH level is a very sensitive diagnostic test to confirm an abnormal thyroxine level, as would occur in hyper- or hypo-thyroidism. While magnetic resonance imaging (MRI) is helpful to exclude substernal extension of the tumour, it is unable to establish the diagnosis. Number: 849 A 22 year old college student presents to her general practitioner with a "lump in the neck," which she noticed recently. She does not have a history of difficulty swallowing, breathing, or talking. There is no history of change in voice or irradiation to the neck. Clinical examination reveals a hard, solitary nodule within the thyroid gland. Cervical lymph nodes are not palpable. She has no symptoms or signs suggestive of hyperthyroidism. A fine-needle biopsy has been performed in order to establish the diagnosis. If the diagnosis is one of malignancy, which of the following would be most likely? A. Medullary carcinoma. B. Undifferentiated carcinoma. C. Papillary adenocarcinoma. D. Follicular adenocarcinoma. E. Primary malignant lymphoma. Select the single best answer ABCDE Correct Answer: C Thyroid carcinoma represents approximately 1% of all malignancies. Females are twice as likely as males to develop the disease. It is rare in children, and frequency increases with age. Carcinoma of the thyroid is rarely clinically detectable. Papillary adenocarcinoma is the most common malignant tumour of the thyroid gland (85% of all thyroid malignancies). It usually affects young individuals. It presents as a firm to hard, nontender, solitary nodule. TSH tends

629

to promote its growth. Spread is via the lymphatics to the regional nodes. The majority of patients have palpable nodes at the time of initial presentation. Distant metastasis usually occur in the lungs or bone. Number: 850 A 22 year old college student presents to her general practitioner with a "lump in the neck," which she noticed recently. She does not have a history of difficulty swallowing, breathing, or talking. There is no history of change in voice or irradiation to the neck. Clinical examination reveals a hard, solitary nodule within the thyroid gland. Cervical lymph nodes are not palpable. She has no symptoms or signs suggestive of hyperthyroidism. A fine-needle biopsy has confirmed the diagnosis of papillary adenocarcinoma. At surgery, the nodule is found to be greater than 1.5 cm in diameter but the lymph nodes are free of metastasis. Which of the following is the most appropriate next step in management? A. Subtotal lobectomy. B. Partial lobectomy. C. Total thyroidectomy with modified radical neck dissection. D. Total thyroidectomy followed by ablation with radio-iodine. E. Total thyroidectomy, modified radical neck dissection, and ablation with radio-iodine. Select the single best answer ABCDE Correct Answer: D The surgical treatment for papillary adenocarcinoma, which is greater than 1.5 cm in size, is total thyroidectomy followed by ablation with radioactive iodine. Use of radioactive iodine after surgery permits identification and treatment of occult or apparent local or distant metastasis. It is important to perform a total thyroidectomy because residual normal thyroid tissue has a greater avidity for radioactive iodine than do metastatic deposits. The recurrence rate after total thyroidectomy is half that of subtotal thyroidectomy (10% as opposed to 20%). For tumours that are less than 1.5 cm, lobectomy or near-total thyroidectomy (ipsilateral total lobectomy and contralateral subtotal lobectomy) are available options. However, after lobectomy there is a recurrence rate of 7%, which is unacceptable because 50% of patients with recurrent thyroid carcinoma die as a result of the illness. Partial lobectomy and subtotal lobectomy are incorrect because of the high recurrence rate. Total thyroidectomy with modified radical neck dissection is indicated in the presence of gross lymphatic metastasis, which is absent in this patient. Total thyroidectomy, modified radical neck dissection, and ablation with radioactive iodine is also incorrect because the lymph nodes are not involved. Number: 900 Testicular Neoplasms: 1. Are > 98% malignant. 2. Are most commonly teratomas. 3. Are the commonest form of cancer in young men.

630

4. Are preceded by trauma in 10% of cases. A: 1,2,3 Correct B: 1,3 Correct C: 2,4 Correct D: 4 Correct E: All Correct ABCDE Correct Answer: B Testicular tumours may be classified as seminoma, embryonal carcinoma, teratocarcinoma, teratoma, and choriocarcinoma. Testiculat cancer is one of the commonest forms of cancer in young men. It is usually a Seminoma (40%) followed by a Teratoma (32%). Mal-descent is a pre-disposing factor, but trauma is not - although it sometimes serves to draw attention to the condition. Lumbar pain due to metastatic disease is quite a common form of presentation. See: Postgrad Med 1981 Dec;70(6):125-7, 130: Testicular tumors of germ cell origin. Kaplan JH, Kudish HG, Sacks SA Number: 901 The sign causing the most concern in a patient with fat embolism is: A. Dyspnoea. B. Fat globules in the urine. C. Confusion. D. Fat in the sputum. E. None of the above. Select the single best answer ABCDE Correct Answer: A The syndrome resulting from fat embolism consists of neurologic dysfunction, respiratory insufficiency, and petechiae of the axilla, chest and proximal arms. Decreased haematocrit, thrombocytopenia and other changes in coagulation are usually seen. The finding of fat droplets in the sputum and urine is a non-specific event seen after trauma. The prognosis is related to the severity of the pulmonary insufficiency. It was originally described in trauma victims- especially those with long bone fractures - and was thought to be the result of bone marrow embolization. However the principal clinical manifestations of fat embolism are seen in other post-traumatic conditions, and the existence of fat embolism as an entity distinct from posttraumatic pulmonary insufficiency has been questioned.

631

Following trauma, the concentration of fat macroglobules (20 um diam.) in blood increases to reach a peak at 12 hours and then returns to normal within a few days.The theory of fat embolism is that fat originating from the bone marrow enters venules at the site of injury. This is supported by the finding of high concentrations of fat in the blood from the ipsilateral femoral vein. The lipid profile of fat globules is however different from that of bone marrow. An alternative theory attributes the origin of fat globules to chylomicron aggregates in response to stress or prolonged hypotension. Elevation of plasma lipase and free fatty acid concentrations following trauma supports this. Fat globules > 10 um are trapped in the lung capillaries. Lipolysis of the trapped fat releases free fatty acids, which may produce acute vasculitis, release kinins, and destroy pulmonary surfactant. The result is pulmonary oedema, increased A-a gradient, hypoxia and a syndrome which may be indistinguishable from ARDS. WAY ,L.W; Current Surgical Diagnosis and Treatment, 9th Ed., Lange, 1991, p 30. BORDOW ,R.A MOSER, K.M (EDS); Manual of Clinical Problems In Pulmonary Medicine, 3rd Ed.,1991, pp 305-6. Number: 903 Which of the following are true of limb tourniquets? 1. The cuff should be inflated 50 mmHg > SBP in upper limb. 2. The cuff should be inflated 100 mmHg > SBP in lower limb. 3. Ischaemic changes are observed after 1 hour. 4. Haemodynamic responses are more extreme when used in conjunction with regional anaesthesia A: 1,2,3 Correct B: 1,3 Correct C: 2,4 Correct D: 4 Correct E: All Correct ABCDE Correct Answer: A Tourniquet cuffs are usually inflated to a pressure 50 mmHg greater than systolic pressure in the arm , and 100 mmHg in the leg. This is because of the larger mass of the leg. Also, femoral systolic pressure is greater than radial. The duration of the tourniquet inflation has been the subject of controversy. Following 1 hour of inflation, depletion of glycogen granules in the sarcoplasm. After 2 hours, mitochondrial swelling, myelin degeneration, and Z-line lysis occur. These changes are reversible for tourniquet times of 1-2 hours. The haemodynamic responses to tourniquet inflation include elevations in systemic blood pressure and CVP. These may be marked if general anaesthesia is used. In one study, increases in blood pressure of 30% greater than the control occurred in 53% of subjects. It is much less marked with regional anaesthesia. Similarly, elevations in CVP may occur. It has

632

been reported that as much as 15% of the blood volume is shifted into the central compartment after exsanguination of the lower limb. Following tourniquet deflation, hypotension may occur, requiring marked compensatory alterations on the part of the cardiovascular system. ROGERS,M.C ET AL (EDS); Principles and Practice of Anesthesiology, Mosby, 1993, pp 2203. Number: 907 Mucous diarrhoea with a low serum K is most likely due to: A. Villous adenoma of the rectum. B. Carcinoid of the ileum. C. Whipples Disease. D. Crohn's Disease. E. Salmonella infection. Select the single best answer ABCDE Correct Answer: A Villous adenomas are premalignant adenomatous polyps which occur in the colon and rectum. 40% show malignant transformation.Their secretions are typical of colorectal secretions in general: mucoid with semi-formed stool. Particular to this lesion is their high rate of secretion of K+. Carcinoid of the ileum may secrete secretin, gastrin inhibitory polypeptide, enteroglucagon, vasoactive intestinal peptide, and somatostatin. Whipple's Disease is a rare disorder. It occurs usually in middle-aged males and is characterised by the deposition of glycoproteins and fat in the intestinal wall and mesenteric lymphatics. It causes steatorrhoea. It is now considered to be an infection and responds to antibiotics. Crohn's Disease is a granulomatous type, chronic inflammatory disorder of the gut which may involve the small intestine alone (30%), small intestine and colon (50%), or colon alone (20%). Diarrhoea from any cause may cause hypokalemia through depletion. However, overall, other causes of hypokalemia are more common and these may be present concomitantly. These include: (1) Depletion through inadequate intake - NBM and administration of K deficient IV fluids. (2) Depletion through upper GIT losses - vomiting, biliary and other fistulae. (3) Depletion through renal losses - diuretics, Na diuresis(eg :high dose Penicillins), alkalosis, and increased protein catabolism following trauma and sepsis.

633

References WAY,L.W; Current Surgical Diagnosis and Treatment, 9th Ed., Lange, 1991, Number: 911 With regard to acute mesenteric vascular occlusion: 1. It is commonly caused by emboli from the heart. 2. Symptoms occur early. 3. It may be complicated by Disseminated Intravascular Coagulation (DIC). 4. Laparotomy must be performed within 12 hours of occlusion. A: 1,2,3 Correct B: 1,3 Correct C: 2,4 Correct D: 4 Correct E: All Correct ABCDE Correct Answer: E Acute mesenteric vascular occlusion is generally the result of infarction of the mesenteric vessel watershed from an embolus originating from a mural thrombus; either from the atrium in AF or ventricle following an AMI. Symptoms of non-specific abdominal pain appear early, but signs are notoriously late in emerging. It may be complicated by DIC. The prognosis is poor if surgery is delayed > 12 hours. Acute mesenteric vascular occlusion, colloquially known as " dead gut "occurs as a result of acute occlusion of artery or vein. It is predominantly a disease of the elderly. Causes include emboli originating in an infarcted left ventricle or in a fibrillating atrium. Thrombosis of a mesenteric artery is the end result of an atherosclerotic stenosis and may present in the context of a history of intestinal angina. Dissecting aortic aneurysm and fusiform aneurysm are rare causes. Thrombosis of mesenteric veins is associated with portal hypertension, abdominal sepsis, hypercoaguable states or trauma, or there may be no underlying pathology identified. The pathology is caused by hemorrhagic necrosis of the mucosa with proliferation of bacteria in the ischaemic areas. Absorption of toxic products frequently causes systemic signs. Ischaemia progresses to full thickness infarction in as little as 6 hours or up to several days following occlusion. In the early stages there is a paucity of abdominal findings; this is a classic presentation. This is in the presence of severe, poorly localised pain, and variably, nausea, vomiting, diarrhoea, and constipation. Later in the course abdominal distension and tenderness, shock and peritonitis occur. Stool or gastric aspirate contain blood in 75-95% of cases. Striking leukocytosis and some elevation of amylase may be present. A metabolic acidosis may be present and DIC may evolve. If bleeding into the gut is large, the Hb may fall. The Hct may rise if third space losses are large.

634

WAY, L.W; Current Surgical Diagnosis and Treatment, 9th Ed., Lange, 1991, pp 494-5. Number: 912 The following is / are true of agents used in Deep Venous Thrombosis (DVT) prophylaxis: 1. Greater protection is afforded by heparin given 8 hourly than 12 hourly. 2. Low molecular weight heparin causes less bleeding than heparin. 3. Dipyridamole is useful in DVT prophylaxis. 4. Epidural techniques may be performed during low dose heparin regimens. A: 1,2,3 Correct B: 1,3 Correct C: 2,4 Correct D: 4 Correct E: All Correct ABCDE Correct Answer: D Low dose heparin administered subcutaneously every 8-12 hours started preoperatively reduces the rate of DVT by 60% and PE by 50% in general surgery, urology, moderate to high risk gynaecological surgery and neurosurgery, but is less beneficial in orthopedic patients. Low dose heparin has been used in two regimens; 5000 units every 8 or 12 hours. There does not seem to be a difference between the two regimens with respect to a DVT prophylactic effect, but there are more bleeding complications if the drug is given every eight hours. Low molecular weight heparin consists of fragments of heparin produced by depolymerization of the parent compound. It has an equipotent antithrombotic effect, but has less effect on platelets than conventional heparin. Theoretically, it should cause less bleeding but this has not been shown Dipyridamole is a vasodilator, and acts to inhibit ADP- induced platelet aggregation. Clinically, reductions in the embolus rate from prosthetic heart valves and transplanted kidneys have been seen, but no effect with respect to thrombosis prophylaxis has been documented. Several studies have reported no bleeding problems when low dose heparin is used in patients undergoing epidural anaesthesia. ROGERS, M.C. ET AL (EDS); Principles and Practice of Anesthesiology, Mosby, 1993, pp 123-6.

635

Number: 913 A 65 year old man is vomiting from a small bowel obstruction. He has lost about 5 L of fluid. It is likely that: 1. He will pass urine with a high osmolality. 2. His urine output will be decreased. 3. His plasma protein concentration will be increased. 4. His plasma sodium will be increased. A: 1,2,3 Correct B: 1,3 Correct C: 2,4 Correct D: 4 Correct E: All Correct ABCDE Correct Answer: A Loss of fluid associated with vomiting and bowel obstruction causes " desalting water loss ".This is the loss of water and salt from the body at concentrations close to isotonic. Extracellular fluid deficit refers to the result. Third space effect refers to one of the causes. It may be caused by renal or extrarenal factors. Extrarenal factors are most common in surgical patients and include gastrointestinal disease, fistulae, hyperfunction, obstruction, vomiting, short bowel syndrome, enteritis, colitis, typhoid fever, salmonellosis, cholera, and pseudomembranous colitis. In almost all cases the fluid is close to isotonic. Desalting water loss initially produces an isotonic volume reduction, the characteristic renal responses being decreased urine output with increased urine osmolality and decreased urine Na+. Sodium free water enters the circulation, largely from cells; this is a weak compensation toward volume restoration, but at the expense of sodium concentration, which falls. Any addition of Na+ free fluid further adds to the tendency to hyponatremia WAY, L.W; Current Surgical Diagnosis and Treatment, 9th Ed., Lange, 1991, pp 141-2. Number: 916 A front seat passenger in a motor vehicle accident (MVA) suffers blunt chest trauma. Which of the following should arouse suspicion of a ruptured aorta? 1. Left haemothorax. 2. Depressed left main bronchus. 3. Fractured 1st rib or apical haematoma. 4. Displacement of the mid oesophagus to the left.

636

A: 1,2,3 Correct B: 1,3 Correct C: 2,4 Correct D: 4 Correct E: All Correct ABCDE Correct Answer: A Ruptured aorta should be suspected when any of the following signs are present in association with blunt chest trauma: shock, widened mediastinum (> 8 cm), left haemothorax, depressed left main bronchus, blurred outline of the arch or descending aorta, fractured 1st rib or left apical haematoma, displacement of the mid oesophagus to the right (easily detectable in the presence of a nasogastric tube ), or evidence of disrupted perfusion to the upper or lower limbs. The classic site of rupture is at the junction of the mobile arch and fixed descending aorta which is immediately beyond the origin of the left subclavian artery. Rupture at this site is attributed to forward movement of the mobile arch against the tethered descending aorta in a deceleration situation. In about 10%, the tear is in the ascending aorta near the origin of the other great vessels. These tears are usually due to direct trauma. T.E.OH; Intensive Care Manual, 3rd Ed.,1990, p 440. Number: 921 12 months after a partial gastrectomy, the commonest cause of anaemia is: A. Fe deficiency. B. Intrinsic factor deficiency. C. Afferent loop syndrome. D. Bleeding from a (recurrent) stomach ulcer. E. None of the above. Select the single best answer ABCDE Correct Answer: A Fe deficiency anaemia develops in about 30% of patients within 5 years after partial gastrectomy. It is caused by failure to absorb dietary iron bound in an organic molecule. Is treated by supplemental inorganic Fe, Ferrous sulfate or gluconate which is absorbed normally. Sufficient parietal cells should remain after a partial gastrectomy to produce IF. Megaloblastic anaemia appear in only a few cases. AFFERENT LOOP SYNDROME: occurs in patients after a Billroth II gastrectomy (subtotal gastrectomy where 2/3-3/4 of the distal stomach are resected and the proximal stump is

637

anastomosed to the side wall of the jejunum leaving a blind loop.)The syndrome consists of abdominal pain occurring 15-30 minutes after a meal that is relieved suddenly by vomiting bilious fluid free of food. This is thought to be caused by partial obstruction of the afferent loop at the anastomosis and presumably is due to unimpeded flow of food from the stomach into the efferent lop, release of secretin and cholecystokinin, and stimulation of pancreatic secretion and gallbladder contraction. Bile accumulates in the afferent loop until the obstruction is suddenly overcome, and the empty stomach is flooded with bile, causing vomiting. Apparently this classic presentation occurs rarely. Nonetheless, it is not a cause of anaemia. Slow bleeding from a recurrent , marginal ulcer or tumour is a possibility and should be excluded before the diagnosis is made. References WAY,L.W; Current Surgical Diagnosis and Treatment, 9th Ed., Lange, 1991, pp 470-2. Number: 924 An adult patient suffers burns to the upper left arm, left leg and lower half of the anterior abdomen. The % area of burns is: A. 21% B. 27%. C. 32%. D. 35%. E. 40%. Select the single best answer ABCDE Correct Answer: C Morbidity and mortality from burns are related to the size and depth of the burn, the age and prior condition of the patient, the location of the burn, and the severity of the associated injuries. The total body surface area involved in the burn is most accurately determined by using the age related charts designed by Lund and Browder. A simplified algorithm is given by the rule of nines. A careful calculation of the size of the burn is important because there is a general tendency to underestimate the size and thus its severity; prognosis is directly related to severity; and it is an admission criterion into specialised burns units. The American Burn Association has adopted a severity index. References WAY,L.W; Current Surgical Diagnosis and Treatment, 6th Ed., 1983, pp 233-5.

638

Number: 926 In the carcinoid syndrome vasoactive substances which may be released include: A. Serotonin. B. Bradykinin. C. Histamine. D. Prostaglandin. E. All of the above Select the single best answer ABCDE Correct Answer: E No single substance is believed to be responsible for the entire spectrum of symptoms. Substances implicated are serotonin, catecholamines, histamine, bradykinin, kallikrein, prostaglandin, ACTH, and calcitonin. Carcinoid tumours are apudomas that arise from enterochromaffin cells throughout the gut. They may be associated with neoplasms of other enterochromaffin tissues (medullary carcinoma of the thyroid and phaeochromocytoma - MEAS). They occur in patients aged 2445 years. The appendix is the most common site followed by the small intestine; the ileum is about 10x more common than jejunum. Multiple tumours are present in 50% of cases.Carcinoids of the small intestine are premalignant; at time of diagnosis 40% have invaded the muscularis and 33% have spread to lymph nodes or liver. Small tumours are asymptomatic; larger ones cause symptoms of obstruction, bleeding, pain, or the carcinoid syndrome. The carcinoid syndrome occurs in about 10% of small bowel tumours. It consists of cutaneous flushing, diarrohoea, bronchoconstriction, and cor pulmonale due to collagen deposition on cardiac valves. The syndrome is more prevalent in metastases, primary ovarian and bronchial tumours as the substances are not cleared by the liver in the usual manner. References WAY,L.W; Current Surgical Diagnosis and Treatment, 9th Ed., Lange, 1991. Number: 958 Surgical resection of the terminal ileum will SIGNIFICANTLY reduce the absorption of which of the following? A. Bile salts. B. Iron. C. Folic acid. D. Vitamin C. E. Calcium. Select the single best answer

639

ABCDE Correct Answer: A Bile salts aid fat absorption in the duodenum and jejunum and are reabsorbed in the distal ileum as part of the enterohepatic circulation, so that usually only approximately 0.5 g bile salts are lost in the faeces daily. Surgical resection of the distal ileum will significantly reduce the absorption of bile salts. Iron absorption occurs predominantly in the duodenum and upper jejunum. Folic acid is absorbed in proximal small intestine and is reduced and methylated during absorption. Vitamin C and Calcium absorption also occurs primarily in the proximal small intestine. Number: 959 The symptoms of meralgia paresthetica will be temporarily relieved by blockade of the: A. Ilio-inguinal nerve. B. Ilio-hypogastric nerve. C. Lateral femoral cutaneous nerve of the thigh. D. Genito-femoral nerve. E. Accessory obturator nerve. Select the single best answer ABCDE Correct Answer: C Meralgia paresthetica is a symptom complex that includes numbness, paresthesias, and pain in the anterolateral thigh, which may result from either an entrapment neuropathy or a neuroma of the lateral femoral cutaneous nerve (LFCN). The condition can be differentiated from other neurologic disorders by the typical exacerbating factors and the characteristic distribution of symptoms. The disease process can be either spontaneous or iatrogenic. The spontaneous form is usually mechanical in origin. The LFCN is subject to compression throughout its entire course. Injuries most commonly occur as the nerve exits the pelvis. The regional anatomy of the LFCN is highly varied and may account for its susceptibility to local trauma. Relief of pain and paresthesias after injection of a local anesthetic agent is helpful in establishing the diagnosis. If no improvement is found, proximal LFCN irritation should be sought. Idiopathic meralgia paresthetica usually improves with nonoperative modalities, such as removal of compressive agents, nonsteroidal anti-inflammatory drugs, and, if necessary, local corticosteroid injections. If intractable pain persists despite such measures, surgery can be considered, although whether neurolysis or transection is the procedure of choice is still controversial. Iatrogenic meralgia paresthetica has been found to occur after a number of orthopaedic procedures, such as anterior iliac-crest bone-graft harvesting and anterior pelvic procedures. Prone positioning for spine surgery has also been implicated. Variations in the anatomy of the LFCN about the anterior superior iliac spine may place the nerve at higher risk

640

for damage. Although nonoperative management usually results in satisfactory results, efforts should be made to avoid injury at the time of surgery. See: Grossman MG, Ducey SA, Nadler SS, Levy AS. Meralgia paresthetica: diagnosis and treatment. J Am Acad Orthop Surg. 2001 Sep-Oct;9(5):336-44. Number: 960 With regard to carcinoma of the prostate: A. Multifocal carcinoma in situ is rarely associated with metastases to regional lymph nodes. B. The majority of patients present with early stage disease. C. The serum prostatic acid phosphatase is always elevated in the presence of bony metastases. D. Lymphangiography is the preferred technique for detection of lymphatic spread. E. Paget's disease may confound the interpretation of a positive bone scan. Select the single best answer ABCDE Correct Answer: E Carcinoma of the prostate is one of the most common cancers of men and is the second most common cause of cancer deaths in Australian men. There appears to have been a rise in the incidence of the disease over the past decade, although this may be due to increased awareness of the condition, screening and more frequent diagnosis rather than any real change in the incidence. Screening can be undertaken by the measurement of serum prostate-specific antigen, although the exact place of this screening tool is still debated. Prostate-specific antigen is a more reliable indicator of bony disease than serum acid phosphatase, which is frequently normal in patients with established metastatic disease. Lymphangiography has been replaced by CT scanning as the imaging procedure of choice to assess lymph node involvement. Radioisotope examination is used to look for skeletal metastatic disease, although areas of Paget disease can be mistaken for deposits of prostatic cancer. Although evidence of the disease only comes to light in many patients through histological analysis of prostatic tissue resected for urinary obstruction, almost 50% of patients with carcinoma of the prostate have established metastatic disease at the time of presentation. Metastatic disease occurs to pelvic nodes, the lumbar spine, pelvis and adjacent tissues. Number: 993 The technique of intra-aortic balloon pumping is absolutely or relatively contraindicated in a patient with: 1. A postinfarction ventricular septal defect. 2. Dynamic left ventricular outflow tract obstruction.

641

3. A flail posterior mitral leaflet. 4. Severe aortic incompetence. A: 1,2,3 Correct B: 1,3 Correct C: 2,4 Correct D: 4 Correct E: All Correct ABCDE Correct Answer: C The presence of severe aortic incompetence is the clearest contraindication to the technique. However, it should also be borne in mind that it is also relatively contraindicated in cases where profound afterload reduction should not be induced - for example in patients with dynamic left ventricular outflow tract obstruction. The use of IABP has been well described in the therapy of postinfarction VSD. IABP generally achieves its beneficial effects by two mechanisms: Diastolic augmentation increases coronary perfusion pressure, and systolic afterload reduction lowers myocardial oxygen consumption. In the context of a VSD, the afterload reduction also helps the left ventricle to eject preferentially in the aorta, consequently reducing left-to-right shunting. See: Coddens J, Van Alphen J, Deloof T, Hendrickx J. Dynamic left ventricular outflow tract obstruction caused by afterload reduction induced by intra-aortic balloon counterpulsation. J Cardiothorac Vasc Anesth. 2002 Dec;16(6):749-51.

TRAUMA TESTS Number: 15 The recommended dose of vasopressin for the treatment of refractory pulseless ventricular tachycardia (VT) or ventricular fibrillation (VF) is: A. 10 IU. B. 20 IU. C. 40 IU. D. 80 IU. E. 160 IU. Select the single best answer ABCDE Correct Answer: C Vasopressin is emerging as an important 'new' drug for the treatment of refractory VT or VF. - In their current guidelines for cardiopulmonary resuscitation and emergency cardiovascular care, the American Heart Association and the International Liaison

642

Committee on Resuscitation have made major changes to the recommendations for advanced cardiovascular life support. One of these changes is that a single 40IU dose of intravenous vasopressin can be used as an alternative to adrenaline for the management of ventricular fibrillation /pulseless VT. This change is based on limited but reasonably persuasive evidence that vasopressin is an effective vasoconstrictor and has a good safety profile; it is unlikely to cause beta stimulation and or postarrest adrenergic storm. However, adrenaline also remains acceptable. See: Forrest P. Vasopressin and shock.Anaesth Intensive Care. 2001 Oct;29(5):46372. The American Heart Association and the International Liaison Committee on Resuscitation (ILCOR). Guidelines 2000 for cardiopulmonary resuscitation and emergency cardiovascular care. An international consensus on science. Circulation. 2000;102(suppl 1):1-384. Number: 29 A 70 kg adult is persistently hypotensive following recent coronary artery grafting. The systemic vascular resistance has been measured and been found to be extremely low. Current therapy includes the use of high dose noradrenaline. The decision is made to switch the vasopressor to vasopressin. Used in these circumstances: 1. An appropriate initial dose rate for infusion of the drug would be 0.1 IU/minute. 2. The drug is likely to have potent anti-diuretic effects. 3. The drug is unlikely to cause arrhythmias. 4. The drug is likely to cause hyperglycaemia. A: 1,2,3 Correct B: 1,3 Correct C: 2,4 Correct D: 4 Correct E: All Correct ABCDE Correct Answer: B Forrest has recently written an excellent, comprehensive review of the role of vasopressin in the treatment of shock. According to this author: " The vasopressin doses typically used post CPB were </= 0.1 IU/minute." In contrast to DDAVP, the drug is not likely to have potent anti-diuretic effects when used in this way. Again, according to Forrest: "The antidiuretic properties of vasopressin result from an increase in the capillary permeability of the distal tubule and collecting duct of the nephron. The vasopressin analogue desmopressin (DDAVP) is primarily a V-2 agonist, with only 0.05% of the pressor activity of arginine

643

vasopressin. Although vasopressin is antidiuretic, it has been shown to increase urine output during haemorrhage, sepsis` and severe heart failure. Possible explanations are that vasopressin may increase renal perfusion pressure secondary to increased systemic pressure and it may also increase glomerular filtration rate by selectively constricting the glomerular efferent arteriole. Finally, renal sympathetic innervation is also involved in the modulation of the renal vascular response to vasopressin. In response to renal denervation, vasopressin produces a marked constrictor response." The absence of major side-effects of the drug has been commented on by many authors. In particular, cardiac arrhythmias have rarely been reported when the drug is used in the circumstances described above. The drug does not interfere with glucose metabolism. See: Forrest P. Vasopressin and shock.Anaesth Intensive Care. 2001 Oct;29(5):46372. Number: 129 The best single indicator of the severity of carbon monoxide (CO) poisoning is: A. Alveolar:arterial oxygen tension difference. B. Mixed venous PO2. C. Arterial base deficit. D. Carboxyhaemoglobin level. E. Arterial PCO2. Select the single best answer ABCDE Correct Answer: C There is no really satisfactory measure of the severity of CO poisoning. COHb levels do not correlate particulalry well - possibly because, under some circumstances, the half-life of COHb breathing 100% oxygen at 1 atmosphere can be as short as 30 minutes. Turner et al have recently concluded that "that initial acidosis is a better predictor of treatment requirements and severity than initial COHb." See: J Accid Emerg Med 1999 Mar;16(2):96-8: Carbon monoxide poisoning treated with hyperbaric oxygen: metabolic acidosis as a predictor of treatment requirements. Turner M, Esaw M, Clark RJ. Number: 129 The best single indicator of the severity of carbon monoxide (CO) poisoning is: A. Alveolar:arterial oxygen tension difference.

644

B. Mixed venous PO2. C. Arterial base deficit. D. Carboxyhaemoglobin level. E. Arterial PCO2. Select the single best answer ABCDE Correct Answer: C There is no really satisfactory measure of the severity of CO poisoning. COHb levels do not correlate particulalry well - possibly because, under some circumstances, the half-life of COHb breathing 100% oxygen at 1 atmosphere can be as short as 30 minutes. Turner et al have recently concluded that "that initial acidosis is a better predictor of treatment requirements and severity than initial COHb." See: J Accid Emerg Med 1999 Mar;16(2):96-8: Carbon monoxide poisoning treated with hyperbaric oxygen: metabolic acidosis as a predictor of treatment requirements. Turner M, Esaw M, Clark RJ. Number: 466 In patients who are successfully resuscitated from ventricular fibrillation, the proportion who require 3 shocks or less is approximately: A. 30% B. 40% C. 60% D. 80% E. 90% Select the single best answer ABCDE Correct Answer: D 80% BMJ 1991 Jun 22;302(6791):1517-20 "Heartstart Scotland"--initial experience of a national scheme for out of hospital defibrillation. Cobbe SM, Redmond MJ, Watson JM, Hollingworth J, Carrington DJ OBJECTIVE--To determine the outcome of out of hospital defibrillation in Scotland during the year after the introduction of automated external defibrillators in October 1988.

645

DESIGN--Retrospective analysis of ambulance service reports and hospital records. SETTING--Scottish Ambulance Service and acute receiving hospitals throughout Scotland. MAIN OUTCOME MEASURES--Delay from cardiac arrest to first defibrillator shock; vital state on arrival at hospital accident and emergency department; survival to hospital discharge. RESULTS--During the study period 268 defibrillators were purchased by public subscription and 96% of the 2000 ambulance crew underwent an eight hour training programme in cardiopulmonary resuscitation and defibrillation. A total of 1111 cardiac arrests were recorded, and defibrillation was indicated and undertaken in 602 (54%) patients, mean age 63 (range 14-92) years. A spontaneous pulse was present on arrival at hospital in 180 (30%) of the defibrillated patients, and 75 (12.5%) were subsequently discharged alive. As expected, the likelihood of survival was inversely related to the delay from the onset of cardiac arrest to the time of the first shock and was greater in the case of witnessed arrest. If ventricular fibrillation occurred after the arrival of the ambulance, survival to discharge was 33%. CONCLUSIONS--An effective scheme for out of hospital defibrillation can be introduced rapidly, and with limited training implications and costs, by the use of automated external defibrillators in ambulances. See also: Tunstall-Pedoe H. et al, Survey of 3765 cardiopulmonary recuscitations in British hospitals (the BRESUS study). BMJ 1992;304:1347-51. Type: A Number: 467 Score: 0 Attempted: 0 Category: Trauma / Resuscitation The Australian Resuscitation Council recommendation for an initial dose of amiodarone given by intravenous injection during a cardiac arrest is: A. 1 mg/kg. B. 2 mg/kg. C. 3 mg/kg. D. 4 mg/kg. E. 5 mg/kg. Select the single best answer ABCDE Correct Answer: E 5 mg/kg. A recent review of the role of amiodarone has been published by Gonzalez.

646

Intravenous amiodarone for ventricular arrhythmias: overview and clinical use. Resuscitation 1998 Oct-Nov;39(1-2):33-42 Gonzalez ER, Kannewurf BS, Ornato JP Numerous pharmacological agents with varying cellular electrophysiological effects are available to treat cardiac arrhythmias. Amiodarone is predominantly a Vaughan Williams Class III agent, but also possesses electrophysiological characteristics of the other three Vaughan Williams classes (Class I and IV and minor Class II effects). Amiodarone's primary mechanism is to prolong the cardiac action potential and repolarization time leading to an increased refractory period and reduced membrane excitability. The efficacy and tolerability of intravenous (IV) amiodarone for acute treatment of recurrent and refractory ventricular tachycardia and ventricular fibrillation has been demonstrated in clinical trials. The ARREST trial, a randomized trial comparing IV amiodarone to placebo, found a significant improvement in the proportion of patients surviving to the emergency department following out-of-hospital cardiac arrest in amiodarone-treated patients. Intravenous amiodarone is an effective anti-arrhythmic agent for the acute treatment of life-threatening ventricular arrhythmias and represents an important treatment option for emergency anti-arrhythmic therapy for patients suffering from cardiac arrest. Number: 469 In a patient whose temperature has fallen to 31 degrees centigrade, the activity of clotting factors will be reduced: A. Not at all B. By about 20% C. By about 50% D. By about 80% E. None of the above Select the single best answer ABCDE Correct Answer: D Hypothermia induces marked reductions in the activity of all clotting factors. At 31 degrees, the activity of the various factors is: II - 17%. V - 22%. VII - 34%. VIII - 16%. IX - 7%. X - 20%. XI - 16%. XII - 10%.

647

By the time temperature has fallen to 25, all factors are virtually inactive. Johnson TD, Chen Y, Reed RL: Functional equivalence of hypothermia to specific clotting factor deficiencies. J Trauma 37:413, 1994. Number: 503 The Glascow Coma Score of a head-injured patient who opens his eyes and withdraws his arm in response to a painful stimulus and who is groaning unintelligibly is: A. 6 B. 7 C. 8 D. 9 E. 10 Select the single best answer ABCDE Correct Answer: C Eye Opening: 4 Spontaneous, 3 To Voice, 2 To Pain 1 Nil Verbal Response: 5 Orientated, 4 Confused, 3 Words, 2 Groans, 1 Nil Motor Response: 6 Obeys Commands, 5 Localises Pain, 4 Withdraws from Pain, 3 Abnormal Flexion, 2 Extension, 1 Nil Number: 540 An unconscious man is brought into hospital suffering from methyl alcohol poisoning: 1. Kussmaul's breathing could be expected to be a feature of the condition. 2. Papilloedema would be consistent with this form of intoxication. 3. His plasma bicarbonate might be very low. 4. The methyl alcohol would metabolised to acetaldehyde. A: 1,2,3 Correct B: 1,3 Correct C: 2,4 Correct D: 4 Correct E: All Correct ABCDE Correct Answer: A

648

Early manifestations of poisoning are caused by methyl alcohol, whereas the late manifestations are due to the metabolites formaldehyde and formic acid. The late manifestations include an increased anion gap, metabolic acidosis, and retinal injury. Impairment of vision due to methyl alcohol intoxication is abrupt in onset and is characterised by large symmetric central scotomata. The lesion is in the retinal ganglion cells and their axons, which constitute the optic nerve. Papilloedema is often present. The acidosis of methyl alcohol poisoning is profound, in part because it is metabolised to formaldehyde and formic acid. Number: 562 A patient presents to an emergency room with a six hour history of chest pain suggestive of myocardial infarction. Which of the following tests is the most sensitive and specific predictor of myocardial necrosis? A. Troponin I B. CK MB fraction C. Troponin T D. 12 lead ECG. E. Sestamibi scan. Select the single best answer ABCDE Correct Answer: A The recent development of monoclonal antibodies to cardiac troponin I and troponin T has resulted in cardiac-specific assays which appear to be the most sensitive and specific indicators of myocardial necrosis. Troponin I appears to be slightly more precise than Troponin T. See, for example: N Engl J Med 1997 Dec 4;337(23):1648-53 Emergency room triage of patients with acute chest pain by means of rapid testing for cardiac troponin T or troponin I. Hamm CW, Goldmann BU, Heeschen C, Kreymann G, Berger J, Meinertz T BACKGROUND: Evaluation of patients with acute chest pain in emergency rooms is time-consuming and expensive, and it often results in uncertain diagnoses. We prospectively investigated the usefulness of bedside tests for the detection of cardiac troponin T and troponin I in the evaluation of patients with acute chest pain.

649

METHODS: In 773 consecutive patients who had had acute chest pain for less than 12 hours without ST-segment elevation on their electrocardiograms, troponin T and troponin I status (positive or negative) was determined at least twice by sensitive, qualitative bedside tests based on the use of specific monoclonal antibodies. Testing was performed on arrival and four or more hours later so that one sample was taken at least six hours after the onset of pain. The troponin T results were made available to the treating physicians. RESULTS: Troponin T tests were positive in 123 patients (16 percent), and troponin I tests were positive in 171 patients (22 percent). Among 47 patients with evolving myocardial infarction, troponin T tests were positive in 44 (94 percent) and troponin I tests were positive in all 47. Among 315 patients with unstable angina, troponin T tests were positive in 70 patients (22 percent), and troponin I tests were positive in 114 patients (36 percent). During 30 days of follow-up, there were 20 deaths and 14 nonfatal myocardial infarctions. Troponin T and troponin I proved to be strong, independent predictors of cardiac events. The event rates in patients with negative tests were only 1.1 percent for troponin T and 0.3 percent for troponin I. CONCLUSIONS: Bedside tests for cardiac-specific troponins are highly sensitive for the early detection of myocardial-cell injury in acute coronary syndromes. Negative test results are associated with low risk and allow rapid and safe discharge of patients with an episode of acute chest pain from the emergency room. Myocardial perfusion imaging with technetium 99m sestamibi, will reveal a defect in most patients during the first few hours after development of a transmural infarct. However, although perfusion scanning is extremely sensitive, it cannot distinguish acute infarcts from chronic scars and thus is not appropriate for the diagnosis of acute myocardial infarction. Number: 592 With regard to the use of adrenaline during cardio-pulmonary resuscitation (CPR): 1. The typical initial dose in adults is 0.5 - 1.0 mg. 2. It is occasionally useful to administer doses of up to 10 mgs in divided doses to an adult. 3. The typical initial dose in children is 10 micrograms / kg 4. Adrenaline's beta effects appear to be more important that its alpha effects during CPR. A: 1,2,3 Correct B: 1,3 Correct C: 2,4 Correct D: 4 Correct E: All Correct ABCDE Correct Answer: A

650

During CPR, the improved perfusion as a result of epinephrine's alpha-mediated constriction is far more valuable than its beta-receptor stimulation. In fact, the betaadrenergic effects are potentially deleterious since they increase oxygen consumption. The typical initial dose in adults is 0.5-1.0 mg, whereas that in children is 10 micrograms / kg. Unusually large doses of adrenaline may be required in beta-blocked patients, those in whom receptor down-regulation has occurred (eg asthmatics), or adults with obstructed coronary vessels. Number: 657 A 17 year old is admitted to a large, inner city emergency department having collapsed at a 'rave' party. On admission, he has a pulse rate of 180 bpm and a blood pressure of 80/50 mm Hg. Shortly after admission he goes into ventricular fibrillation which responds to treatment with a counter-shock of 300 Joules and a single dose of lignocaine. An ECG performed after this shows sinus tachycardia with occasional ventricular ectopic beats. A transthoracic echo suggests mild, global hypokinesia, but no other obvious abnormality. The most likely cause for this scenario is: A. Hypertrophic Obstructive Cardiomyopathy. B. Mitral Valve Prolapse. C. Cocaine-induced cardiac toxicity. D. Viral myocarditis. E. Wolff-Parkinson-White syndrome. Select the single best answer ABCDE Correct Answer: C All of these options can be associated with sudden cardiac arrest. Given the context, cocaine is probably the most likely. - The subject has been recently reviewed (Richman PB, Nashed AH. The etiology of cardiac arrest in children and young adults: special considerations for ED management. Am J Emerg Med. 1999 May;17(3):26470.). Acute doses of cocaine suppress myocardial contractility, reduce coronary calibre and coronary blood flow, induce electrical abnormalities in the heart, and in conscious patients increase heart rate and blood pressure. These effects will decrease myocardial oxygen supply and may increase demand (if heart rate and blood pressure rise). Thus, myocardial ischaemia and/or infarction may occur, the latter leading to large areas of confluent necrosis. Increased platelet aggregability may contribute to ischaemia and/or infarction. Young patients who present with acute myocardial infarction, especially without other risk factors, should be questioned regarding use of cocaine. Acute depression of LV

651

function by cocaine may lead to the presence of a transient cardiomyopathic presentation. Chronic cocaine use can lead to the above problems as well as to acceleration of atherosclerosis. Direct toxic effects on the myocardium have been suggested, including scattered foci of myocyte necrosis (and in some but not all studies, contraction band necrosis), myocarditis, and foci of myocyte fibrosis. These abnormalities may lead to cases of cardiomyopathy. Left ventricular hypertrophy associated with chronic cocaine recently has been described. Arrhythmias and sudden death may be observed in acute or chronic use of cocaine. Miscellaneous cardiovascular abnormalities include ruptured aorta and endocarditis. Most of the cardiac toxicity with cocaine can be traced to two basic mechanisms: one is its ability to block sodium channels, leading to a local anesthetic or membrane-stabilizing effect; the second is its ability to block reuptake of catecholamines in the presynaptic neurons in the central and peripheral nervous system, resulting in increased sympathetic output and increased catecholamines. Other potential mechanisms of cocaine cardiotoxicity include a possible direct calcium effect leading to contraction of vessels and contraction bands in myocytes, hypersensitivity, and increased platelet aggregation (which may be related to increased catecholamine). The correct therapy for cocaine cardiotoxicity is not known. Calcium blockers, alpha-blockers, nitrates, and thrombolytic therapy show some promise for acute toxicity. Beta-Blockade is controversial and may worsen coronary blood flow. In patients who develop cardiomyopathy, the usual therapy for this entity is appropriate. Number: 672 Which of the following signs is NOT suggestive of a cervical spinal cord injury? A. Flaccidity. B. Increased rectal sphincter tone. C. Diaphragmatic breathing. D. Priapism. E. Hypotension with warm extremities. Select the single best answer ABCDE Correct Answer: B Signs of injury to the cervical spinal cord include flaccidity (particularly of the rectal sphincter), diaphragmatic breathing, priapism, and hypotension. Number: 721 A 20 year old female weighing about 55 kgs is admitted to an emergency department having consumed 10 g of paracetamol (acetaminophen), together with alcohol, 6 hours earlier. An urgent paracetamol level is reported as 400 micrograms/ml. With regard to this scenario:

652

1. Gastric lavage and administration of activated charcoal is indicated. 2. Administration of N-acetylcysteine is indicated. 3. Abnormalities of liver function are likely to be present at this time. 4. Hepatotoxicity is likely to occur. A: 1,2,3 Correct B: 1,3 Correct C: 2,4 Correct D: 4 Correct E: All Correct ABCDE Correct Answer: C Paracetamol causes severe centrilobular hepatic necrosis when ingested in large amounts. A single dose of 10 to 15 g, occasionally less, may produce clinical evidence of liver injury. Fatal fulminant disease is usually associated with ingestion of 25 g or more. Blood levels of paracetamol correlate with the severity of hepatic injury (levels above 300 ug/ml 4 hours after ingestion are predictive of the development of severe damage, while levels below 150 ug/ml suggest that hepatic injury is highly unlikely). Nausea, vomiting, diarrhoea, abdominal pain, and shock are early manifestations occurring 4 to 12 hours after ingestion. The features of hepatic injury do not usually become apparent until about 24 hours after ingestion. Maximal abnormalities and hepatic failure may not be evident until 4 to 6 days after ingestion, and aminotransferase levels approaching 10,000 units are not uncommon. Paracetamol hepatotoxicity is mediated by a toxic reactive metabolite formed from the parent compound by the cytochrome P450 mixed-function oxidase system of the hepatocyte. The toxic metaboliteis believed to be N-acetylimidoquinone which is detoxified by binding to glutathione. When excessive amounts of the metabolite are formed, glutathione levels in the liver fall, and the metabolite is covalently bound to nucleophilic hepatocyte macromolecules. This process is believed to lead to hepatocyte necrosis. Hepatic injury may be potentiated by prior administration of alcohol or other drugs or by conditions that stimulate the mixed-function oxidase system. In chronic alcoholics, the toxic dose of paracetamol may be as low as 2 g. N-acetylcysteine appears to act by providing a reservoir of sulfhydryl groups to bind the toxic metabolites or by stimulating synthesis and repletion of hepatic glutathione.

653

Therapy should be begun within 8 hours of ingestion but may be effective even if given as late as 24 to 36 hours after overdose. The Rumack-Matthew nomogram can be used in deciding whether or not administration of N-acetylcysteine is indicated. Gastric lavage and administration of activated charcoal is unlikely to be of benefit more than 1-2 hours after ingestion. In a recent review from Pittsburgh, acetaminophen toxicity was the second most common cause of acute liver failure (~20% of cases) - after viral hepatitis (~30% of cases). Number: 797 The Australian Resuscitation Council recommend that external cardiac compression (ECC) in neonate be carried out at a rate of: A. 140 bpm. B. 120 bpm. C. 100 bpm. D. 80 bpm. E. 60 bpm. Select the single best answer ABCDE Correct Answer: C The Australian Resuscitation Council (and the Pediatric Working Group of the International Liaison Committee on Resuscitation) recommend a rate of 100 bpm. The patient should be placed on a firm surface, and compression directed to the lower sternum to a depth approximating a third of the anteroposterior diameter of the chest, or at a depth of 2-3 cm and rate of 100/min for a newborn or infant; depth of 3-4 cm and rate of 100/min for a small child; depth of 4-5 cm and rate of 80-100/min for a large child. ECC for a newborn or infant can be performed with two fingers, although a better technique is to encircle the chest with both hands, compressing the sternum anteriorly with the thumbs while stabilising the vertebral column posteriorly with the fingers. The rescuer's hands must encircle the chest freely and not restrict chest expansion. ECC for a small child can be performed with the heel of one hand and, for a large child or teenager, with two hands. A cycle should be 50% chest compression and 50% relaxation. Number: 805 The half-life of carboxyhaemoglobin (COHb t(1/2)) in carbon monoxide (CO) poisoned patients treated with 100% oxygen at atmospheric pressure is approximately:

654

A. 10 minutes. B. 30 minutes. C. 75 minutes. D. 125 minutes. E. 250 minutes. Select the single best answer ABCDE Correct Answer: C Weaver et al recently examined a group of 93 CO-poisoned patients and concluded that the COHb t(1/2) when treated with 100% O(2) at atmospheric pressure was 74 +/25 min. This was considerably shorter than the COHb t(1/2) reported in prior clinical reports (approximately 130 +/- 130 min) and was influenced only by the patient's PaO2. See: Chest 2000 Mar;117(3):801-8: Carboxyhemoglobin half-life in carbon monoxidepoisoned patients treated with 100% oxygen at atmospheric pressure. Weaver LK, Howe S, Hopkins R, Chan KJ. A similar result has also been reported by Takeuchi et al. This group also examined the effect of normocarbic hyperventilation on COHb t(1/2) and were able to show that the half-life could be reduced from 78 to 30 minutes when the technique was used. See: Am J Respir Crit Care Med 2000 Jun;161(6):1816-9: A simple "new" method to accelerate clearance of carbon monoxide. Takeuchi A, Vesely A, Rucker J, Sommer LZ, Tesler J, Lavine E, Slutsky AS, Maleck WH, Volgyesi G, Fedorko L, Iscoe S, Fisher JA. Number: 806 A young man is found unconscious in a motor vehicle. The engine is running and a hose has been connected from the exhaust pipe to the interior of the vehicle. He is brought to the emergency department by ambulance while breathing 100% oxygen. On arrival he is conscious but drowsy. An arterial blood gas analysis performed at this time shows: PaO2: 560 mm Hg PaCO2: 33.7 mm Hg pH: 7.13 BXS: -16.3 COHb: 25% Hb: 150 Gm/L Given this scenario: 1. Hyperbaric therapy is definitely indicated. 2. Neuropsychiatric sequelae are unlikely to occur.

655

3. The COHb level suggests that the poisoning is severe. 4. The base deficit suggests that the poisoning is severe. A: 1,2,3 Correct B: 1,3 Correct C: 2,4 Correct D: 4 Correct E: All Correct ABCDE Correct Answer: D The role of hyperbaric oxygen therapy (HBO) in the treatment of CO poisoning appears to be controversial. Scheinkestel et al have recently concluded that "HBO therapy did not benefit, and may have worsened, the outcome." and a recent metaanalysis has also cast doubt upon the use of HBO as a means of preventing neurological sequelae. See: Med J Aust 1999 Mar 1;170(5):203-10: Hyperbaric or normobaric oxygen for acute carbon monoxide poisoning: a randomised controlled clinical trial. Scheinkestel CD, Bailey M, Myles PS, Jones K, Cooper DJ, Millar IL, Tuxen DV. Cochrane Database Syst Rev 2000;(2):CD002041: Hyperbaric oxygen for carbon monoxide poisoning. Juurlink DN, Stanbrook MB, McGuigan MA. (See website) If HBO is used, it should probably be implemented as early as possible in patients with a carboxyhaemoglobin level of 40% or greater; or in those with a history of loss of consciousness, persistent neurological deficits (including coma), or cardiac abnormalities. Management should consist of two treatments separated by 24 hours. 2. The incidence of neuropsychiatric sequelae following this degree of intoxication is very high - probably in the order of 30-50% who survive the insult. See: Anaesth Intensive Care 1992 Aug;20(3):311-6: A longitudinal study of 100 consecutive admissions for carbon monoxide poisoning to the Royal Adelaide Hospital. Gorman DF, Clayton D, Gilligan JE, Webb RK. Smith JS, Brandon S: Morbidity from acute carbon monoxide poisoning at three-year follow-up. Br Med J 1973;1:318-321. 3. The COHb level has been consistently found to be an unreliable indicator of the severity of intoxication. The fact that the patient has been unconcscious is of much greater significance. - It should be remembered that the decrease in the oxygen carrying capacity of blood is only one of several pathophysiological 'lesions' which develop when a person is exposed to CO. Others include: Alteration of the

656

dissociation characteristics of oxyhaemoglobin, further decreasing oxygen delivery to the tissues; Decrease in cellular respiration by binding with cytochrome a3; Binding to myoglobin, potentially causing myocardial and skeletal muscle dysfunction. Furthermore, a significant 'reperfusion injury' probably occurs as the COHb level falls. 4. A large base deficit is certainly suggestive of severe intoxication. See, for example: J Accid Emerg Med 1999 Mar;16(2):96-8: Carbon monoxide poisoning treated with hyperbaric oxygen: metabolic acidosis as a predictor of treatment requirements. Turner M, Esaw M, Clark RJ. Number: 895 The most appropriate first-line anticonvulsant therapy for a known epileptic in status epilepticus is: A. Diazepam 0.15 mg/kg. B. Lorazepam 0.1 mg/kg. C. Phenobarbital 5 mg/kg. D. Phenytoin 18 mg/kg. E. Thiopentone 5 mg/kg. Select the single best answer ABCDE Correct Answer: B See the excellent review by Chapman, Smith and Hirsch. These authors make the point that "Unfortunately, there is no ideal drug and a compromise between effective therapy and its inevitable side-effects often has to be accepted. " However, of the options which are presented, Lorazepam 0.1 mg/kg is likely to be the most effective. (Note that the correct dose of phenobarbital is 15 mg/kg.) See: Chapman MG, Smith M, Hirsch NP. Status epilepticus. Anaesthesia. 2001 Jul;56(7):648-59. and: Treiman DM, Meyers PD, Walton NY et al. A comparison of four treatments for generalized convulsive status epilepticus. Veterans Affairs Status Epilepticus Cooperative Study Group. New England Journal of Medicine 1998; 339: 7928. Number: 922 The plasma half-life of vasopressin is approximately: A. 20 seconds. B. 2 minutes. C. 10 minutes. D. 20 minutes. E. 1 hour. Select the single best answer

657

ABCDE Correct Answer: D The plasma half-life of vasopressin has been reported to be 24 minutes. See: Baumann G, Dingman JF. Distribution, blood transport, and degradation of antidiuretic hormone in man. J Clin Invest. 1976 May;57(5):1109-16. See also, the excellent, comprehensive review of the role of vasopressin in the treatment of shock by Forrest. ( Forrest P. Vasopressin and shock.Anaesth Intensive Care. 2001 Oct;29(5):463-72.) Number: 935 The likelihood of the diagnosis of acute coronary ischaemia being missed by staff of the emergency department (ED) in a major metropolitan hospital is approximately: A. 0.2% B. 2% C. 5% D. 10% E. 20% Select the single best answer ABCDE Correct Answer: B Pope et al recently reviewed the records of over 10000 patients who attended the ED's in the US with presenting symptoms suggestive of acute coronary ischaemia. (Chest, left-arm, jaw, or epigastric pain or discomfort; shortness of breath; dizziness; palpitations; syncope; or other symptoms suggestive of acute ischemia. ). Of these patients, 17 percent ultimately met the criteria for acute cardiac ischaemia. Of the 889 patients with acute myocardial infarction, the rate of missed diagnosis 2.1 percent. Of the 966 patients with unstable angina, the rate of missed diagnosis was 2.3 percent. Women were at particular risk of misdiagnosis. See: Pope JH, Aufderheide TP, Ruthazer R, Woolard RH, Feldman JA, Beshansky JR, Griffith JL, Selker HP. Missed diagnoses of acute cardiac ischemia in the emergency department. N Engl J Med. 2000 Apr 20;342(16):1163-70.

658

Number: 978 A 70 year old man presents to an emergency department with full-thickness burns to 50% of his body In the region of the head and thorax. He has clinical evidence of an airway burns. The predicted mortality for this patient is approximately: A. 10%. B. 30%. C. 50%. D. 70%. E. 90%. Select the single best answer ABCDE Correct Answer: E Ryan et al conducted a retrospective review of 1665 patients with acute burn injuries admitted from 1990 to 1994 to Massachusetts General Hospital and the Shriners Burns Institute in Boston. Using logistic-regression analysis, they developed probability estimates for the prediction of mortality based on a minimal set of well-defined variables. They identified three important risk factors for death: Age greater than 60 years; More than 40 percent of body-surface area burned; and inhalation injury. These factors predicted 0.3 percent, 3 percent, 33 percent, or approximately 90 percent mortality, depending on whether zero, one, two, or three were present. The presence of an airway burn was of particular importance in predicting mortality. See: Ryan CM, Schoenfeld DA, Thorpe WP, Sheridan RL, Cassem EH, Tompkins RG. Objective estimates of the probability of death from burn injuries. N Engl J Med. 1998 Feb 5;338(6):362-6. Number: 985 A 45 year old patient suffers a subarachnoid haemorrhage and becomes clinically brainstem dead. Tests of brainstem death are carried out in compliance with the guidelines issued by the Conference of Medical Royal Colleges of the United Kingdom Given this scenario, which of the following statements are correct? 1. If an EEG is performed, there will be no activity. 2. The patient will have diabetes insipidus. 3. The patient will not develop hypertension in response to a surgical incision. 4. The oculovestibular reflexes will be absent. A: 1,2,3 Correct B: 1,3 Correct

659

C: 2,4 Correct D: 4 Correct E: All Correct ABCDE Correct Answer: D Readers are referred to the editorial by Young and Matta in the journal 'Anaesthesia'. Although brainstem death (properly diagnosed by independent experts under appropriate conditions) is widely accepted as one of the criteria which must be fulfilled before organ donation can be undertaken, it should be understood that such patients may exhibit considerable evidence of other neurological activity. For example, Grigg et al reviewed 56 'brain dead' patients and found EEG activity in about 20% of cases. Howlett et al examined anterior and posterior pituitary function in 31 brainstem dead patients and found that diabetes insipidus was NOT present in about 25% of the patients. Wetzel et al examined the haemodynamic response to organ donation in 10 brainstem dead patients and found that systolic pressure increased by a mean of 31 torr, diastolic pressure by 16 torr, and heart rate by 23 beats/min in response to surgical stimulation. The oculovestibular reflexes MUST be absent before the diagnosis of brainstem death can be made.. See: 1. Young PJ, Matta BF. Anaesthesia for organ donation in the brainstem dead--why bother? Anaesthesia. 2000 Feb;55(2):105-6. 2. Grigg MM, Kelly MA, Celesia GG, Ghobrial MW, Ross ER. Electroencephalographic activity after brain death. Arch Neurol. 1987 Sep;44(9):94854. 3. Howlett TA, Keogh AM, Perry L et al. Anterior and posterior pituitary function in brain-stem-dead donors: a possible role for hormone replacement therapy. Transplantation 1989; 47: 82834. 4. Wetzel RC, Setzer N, Stiff JL et al. Hemodynamic responses in brain dead organ donor patients. Anesthesia and Analgesia 1985; 64: 125-8.

660

TRIVIAL PURSUITS TESTS

Number: 181 Who is the 'Odd Man Out' A. A Coqueros Indian. B. The larva of Eloria Noyesi. C. The Pharoah Ant. D. William Halsted. E. Fleischl von Markow. Select the single best answer ABCDE Correct Answer: C With the exception of the Pharoah Ant, all the others had a predilection for Cocaine. The Coqueros Indians chew the leaves of Erythroxolum Coca constantly and the larvae of the lymantriid butterfly Eloria Noyesi will eat nothing else! Halsted - the " father " of American Surgery became addicted to the drug shortly after he carried out the first Brachial Plexus block in the 1880's. The physiologist Fleischl von Markow had become addicted to Morphine after he developed a painful neuroma on the amputated thumb of his right hand (The amputation had been carried out by Billroth and later re-operation was performed by Trendelenburg). At the suggestion of Sigmund Freud, he tried to wean himself from morphine by using Cocaine. The weaning process was successful, but the patient remained addicted to cocaine for the rest of his life! In contrast, the Pharaoh Ant Monomorium Pharaonis - which is a predator of many plants in the High Andes, is repelled by cocaine and give the Erythroxolum Coca a wide berth. This presumably confers a survival advantage on the plant. . Number: 188 Who are the contenders for having given the first anaesthetic in Australia? 1. Dr William Russ Pugh of Launceston, Tasmania. 2. Drs. John Belisario and Charles Nathan of Sydney. 3. Dr John White of Sydney. 4. Dr Colin Buchanan of Stroud, NSW. A: 1,2,3 Correct B: 1,3 Correct C: 2,4 Correct D: 4 Correct E: All Correct

661

ABCDE Correct Answer: C The two serious contenders are Drs John Belisario/ Charles Nathan in Sydney using ether in the first week of June 1847 for dental procedures, and Dr Colin Buchanan. Buchanan was the surgeon to the Australian Agricultural Co. at their headquarters in Stroud, NSW, and possibly used ether for resuscitation of a popliteal aneurysm in late May 1847. Dr William Pugh was a surgeon in Launceston performed his anaesthetic on 7th June 1847 and wrote it up the same day to be published in July 1847 issue of the Australian Medical Journal. Dr John White was Surgeon General to the First Fleet and may have been a serious contender but was a little before his time. References WILSON, G; " The first anaesthetics in Australia: an historical update. ", Med J Aust,157, 1992, pp 781-784. WILSON, G; Anaes & Intens Care, 1, 1972, pp9-26. Number: 189 Which of the following drugs are acronymic? 1. EMLA. 2. EUSOL. 3. WARFARIN. 4. NEMBUTAL. A: 1,2,3 Correct B: 1,3 Correct C: 2,4 Correct D: 4 Correct E: All Correct ABCDE Correct Answer: E EMLA - Eutectic Mixture of Local Anaesthetic (lignocaine & prilocaine) EUSOL - Edinburgh University Solution Of Lime WARFARIN - Wisconsin Alumni Research Foundation & coumARIN NEMBUTAL - Na Ethyl Methyl BUTyl barbiturate (AL).

662

Number: 190 Which of the following plants have anticholinergic effects? 1. Hyoscyamus niger. 2. Scopolia carniolica. 3. Atropa belladonna. 4. Chondrodendron tomentosum. A: 1,2,3 Correct B: 1,3 Correct C: 2,4 Correct D: 4 Correct E: All Correct ABCDE Correct Answer: A Hyoscine was first isolated from scopolia carniolica (hence the name scopolamine) but is now obtained from hyoscyamus niger (henbane). Atropine is from atropa belladonna. Curare is obtained from chondrodendron tomentosum. Number: 191 Who obtained the first M.D. for a thesis in Anaesthesia in Australia? A. Robert B. Wade. B. Geoffrey Kaye. C. Robert Orton. D. Edward H. Embley. E. Michael Cousins. Select the single best answer ABCDE Correct Answer: D Edward H. Embley of the Melbourne Hospital gained his MD from the University of Melbourne in 1901. He established an international reputation by eliciting conclusively the cardiac effects of chloroform - " The Causation of death by the administration of chloroform " British Medical Journal, 1902, April 5th,12th & 19th. This work helped to quash many false ideas about chloroform and drew attention to the dangers of the drug. Robert B. Wade (later Sir Robert) was appointed " Registrar and Anaesthetist " to the Royal Alexandra Hospital for Children, Sydney in 1901. In 1904 he gained his M.D. from the University of Sydney for his thesis " Ether Anaesthesia in Children ". (Australasian Med Gaz, 22, 1902, pp 149-151.)

663

Number: 192 Which of the following anaesthetic gases is lighter than air? A. Argon. B. Xenon. C. Cyclopropane. D. Nitrous oxide. E. Ethylene. Select the single best answer ABCDE Correct Answer: E The specific gravity of ethylene is 0.97 - however explosiveness was also a disadvantage. It was first used in the 1930's. Number: 193 Who described the circle system for anaesthetic circuits? A. Ayre. B. Boyle. C. Sword. D. Magill. E. Mapleson. Select the single best answer ABCDE Correct Answer: C Phillip Ayre introduced the T piece in 1937. This was modified by G. Jackson Rees by adding a 500 ml open ended reservoir bag to the expiratory limb. Henry F. G. Boyle described his machine in 1917. It was an English adaptation of James Gwathmey's nitrous oxide/oxygen machine, and was subsequently modified many times over the years. Boyle received an OBE for his anaesthetic services in France in WW1. Brian Sword of North Carolina described the circle or two-phase system in 1926. Sir Ivan W. Magill developed and described many anaesthetic components: eg. Magill's forceps, bronchial blockers and endobronchial tubes. He also developed a low resistance spring loaded expiratory valve and this was used in various combinations on semiclosed circuits. It was classified by William W. Mapleson in 1954

664

References SWORD, B.C;.Curr Res Anaesth Analg , 9, 1930, pp198. AYRE, T.P; Lancet , 1, 1931, p561. REES, G.J; BJA , 32, 1960, p132. MAGILL, I.W; BMJ , 2 , 1917, p 653. MAPLESON,W.W; BJA , 26, 1954, p323. Number: 194 Who is the oldest of the three fates of Greek mythology? A. Atropos. B. Morpheus. C. Clothos. D. Lachesis. E. Somnus. Select the single best answer ABCDE Correct Answer: A Atropos was the oldest and severed the thread of life (atropa belladonna-atropine is named after her). Clothos span the thread and Lachesis allocated it. Morpheus was the Greek god of dreams (morphine). Somnus was the Roman (not Greek) god of sleep with several drugs named after him eg. papaver somniferum - the opium poppy and somnifaine - the first barbiturate (a combination of diethyl & diallyl barbituric acids 1924). Number: 196 Complete the sequence: 85, 200, ??? , 1400, 3500, 7000. A. 300. B. 800. C. 1200. D. 1000. E. 400. Select the single best answer ABCDE Correct Answer: E

665

400 litres is the NOMINAL capacity of a size C cylinder which fits to standard anaesthetic machines - if it is oxygen then it contains approx 440 litres, and if N2O then approx 950 litres. The others in order of size are A (85), B,C,D,E, and G (7000) - there is no size F. Number: 197 Who first suggested the use of nitrous oxide to relieve the pain of surgery and named it " laughing gas "? A. Horace Wells. B. Jospeh Priestley. C. William TG Morton. D. Sir Humphry Davy. E. Crawford Long. Select the single best answer ABCDE Correct Answer: D Humphry Davy (1778 - 1829) experimented with nitrous oxide at the age of 17. In 1798 he became the superintendent of the Pneumatic Institute in Bristol and in 1800 he wrote " Researches, Chemical and Philosophical; chiefly concerning nitrous oxide " - this has been called one of the most remarkable books in the history of science. Davy stated " on the day when the inflammation was most troublesome, I breathed three large doses of nitrous oxide. The pain always diminshed after the first four or five respirations ". Priestley discovered nitrous oxide in 1772. Horace Wells was a dentist in Hartford, Conneticut, who observed the analgesic effects of N2O and used them in dentistry. Number: 199 Which is the " Odd Man Out "? A. Papaver Somniferum. B. Mandragora Officinaron. C. Chondrodendron Tomentosum. D. Strychnos Toxifera. E. Erythroxolum Coca. Select the single best answer ABCDE Correct Answer: D All except Strychnos Toxifera are represented on the Armorial Bearings of the Australian and New Zealand College of Anaesthetists. Every plant has at one time or another been a source

666

of anaesthetically important drugs. Papaver Somniferum and Mandragora Officinaron are sources of narcotic alkaloids, Chondrodendron Tomentosum and Strychnos Toxifera are sources of Curare, and Erythroxolum Coca provides Cocaine. For those with botanical leanings, there are two other plants on the shield - what are they? (See Below) References ANZCA Bulletin , 2,2, 1993, p 15. Acacia Baileyana (The Cootamundra Wattle) represents Australia. Cyathea Dealbata (The Silver Fern Tree) represents New Zealand. Number: 203 Which of the following is true of the aviator Charles Lindbergh? A. He wrote an authoritative text on the physiology of hypothermia for isolated organ preservation. B. He was a qualified anaesthetist. C. His aircraft was named "The Indefatigible Spirit" D. He wrote extensively on high altitude physiology. E. He suffered from Raynaud's phenomenon. Select the single best answer ABCDE Correct Answer: A Together with Alexis Carrel (a 'Surgical Mystic') Lindbergh was the first person to experiment seriously with the long term preservation of isolated organs. In 1938 he established the principles of organ preservation which are still used by transplant surgeons today. The aircraft in which he made his transatlantic crossing was "The Spirit of St. Louis". "The Culture of Organs" Carrel, A and Lindbergh RC, Hamish Hamilton Medical books. London. 1938 Number: 205 Who said: "After two days in hospital I took a turn for the nurse." A. Winston Churchill. B. W.C.Fields. C. Lord Lister D. William Clinton E. Groucho Marx Select the single best answer

667

ABCDE Correct Answer: B W.C.Fields. Number: 208 Who said: "There are only two sorts of doctors: Those who practise with their brains and those who practise with their tongues." A. Sir William Osler B. Rudyard Kipling C. George Bernard Shaw D. Charles Dickens E. Oliver Holmes Select the single best answer ABCDE Correct Answer: A Sir William Osler Number: 223 The Swiss surgeon Theodore Kocher gave his name to: 1. A surgical incision. 2. An anaesthesia mask. 3. An orthopaedic manoeuvre. 4. A surgical knot. A: 1,2,3 Correct B: 1,3 Correct C: 2,4 Correct D: 4 Correct E: All Correct ABCDE Correct Answer: A Theodore Kocher (1841-1917) was Professor of Surgery at Berne University. He won the Nobel prize in 1909 for improving the surgical techniques used during thyroidectomy.

668

Kocher's incision is used for access to the gall-bladder. Kocher's mask was used for open drop chloroform and ether. Kocher's manoeuvre is used to reduce a dislocated shoulder. Ball, C. "Kocher's mask" Anaesth. Intens. Care 1995; 23: 535 Number: 224 Who recommended "A cup of tea with cognac half an hour before the anaesthetic to strengthen the action of the heart" ? A. James Simpson. B. Theodore Kocher. C. William Morton. D. John Snow. E. David Waldie Select the single best answer ABCDE Correct Answer: B Theodore Kocher - Surgeons still have a healthy disregard for fasting protocols!! On a more serious note, Kocher made great contributions in many areas of surgery and anaesthesia Including regional anaesthetic techniques (using cocaine) for thyroid surgery. Number: 244 The instrument shown below was designed for: A. Removing corks from wine bottles. B. Use as a mouth gag. C. Removing stones from the hooves of the horses of itinerant anaesthetists in the 1880's. D. Removing Umbilical 'Detritus'. E. The cure of 'Fissure in ano'. Select the single best answer

669

ABCDE Correct Answer: B The instrument was known as 'Maunder's Mouth Screw' and was designed as an aid to airway management in the 1870's. Number: 319 An 8 French catheter: A. Has an external circumference of 8 mm. B. Has an external diameter of 8 mm. C. Has an external circumference of 8 / 32 of an inch. D. Has an external diameter of 8 / 32 of an inch. E. Has a cross sectional area of 8 mm2. Select the single best answer ABCDE Correct Answer: A French size specifies the external circumference in millimeters. Dividing French size by 3 provides an approximation of the external diameter in mm. Number: 494 This American surgeon was also a pioneer of local anaesthesia. He was: A. Theodor Kocher. B. William Halstead. C. Joseph Pennybacker. D. Harvey Cushing. E. William Osler.

670

Select the single best answer

ABCDE Correct Answer: D Harvey Cushing (1869 - 1939) Cushing was a pioneering neurosurgeon who developed many of the basic techniques and procedures in use today. He had an early interest in infiltrative analgesia and in addition to his clinical writings was awarded the Pulitzer prize in 1926 for his book entitled "The Life of Sir William Osler." Number: 651 Sexual intercourse will increase the chances of a 50-year old, non-smoking, non-diabetic man suffering a myocardial infarction (MI) to about: A. 2 chances in a million. B. 5 chances in a million. C. 10 chances in a million. D. 50 chances in a million. E. 100 chances in a million. Select the single best answer

671

ABCDE Correct Answer: A Data from the Framingham Heart Study indicates that the baseline risk that a 50-year old, non-smoking, non-diabetic man will experience an MI is about 1% per year, or 1 chance in a million in any hour. Since the relative risk of MI sexual activity is about 2, engaging in intercourse increases his risk to 2 in a million. (But only for a two-hour period.) Is it worth the risk?? Number: 661 Score: 0 Attempted: 1 Category: Trivial pursuit Which of the following conditions did Braithwaite and Shugg suggest that the model used by Rembrandt van Rijn in his painting 'Bathsheba at her toilet' was suffering from? A. Staphylococcal breast abscess. B. Tuberculous breast abscess. C. Cysto-sarcoma phylloides. D. Breast carcinoma. E. Sarcoidosis. Select the single best answer

ABCDE Correct Answer: D Braithwaite and Shugg have attributed the shadowing of Bathsheba's left breast to a late stage carcinoma. This is certainly conventional wisdom, although it has recently been questioned by Bourne.

672

See: Braithwaite PA, Shugg D. Rembrandt's Bathsheba: the dark shadow of the left breast. Ann R Coll Surg Engl. 1983 Sep;65(5):337-8. And: Bourne RG. Did Rembrandt's Bathsheba really have breast cancer? Aust N Z J Surg. 2000 Mar;70(3):231-2. Number: 663 Who said "Alcohol is the anaesthesia by which we endure the operation of life."? A. Karl Marx. B. George Bernard Shaw. C. Groucho Marx. D. Robert Macintosh. E. John F. Kennedy. Select the single best answer ABCDE Correct Answer: B George Bernard Shaw Number: 664 Who said "Once ... in the wilds of Afghanistan, I lost my corkscrew, and we were forced to live on nothing but food and water for days."? A. Karl Marx. B. George Bernard Shaw. C. Groucho Marx. D. W.C.Fields. E. John F. Kennedy. Select the single best answer ABCDE Correct Answer: D W.C.Fields. Who also observed: Horse sense is the thing a horse has which keeps it from betting on people. I am free of all prejudice. I hate everyone equally.

673

Number: 681 Who said: "The desire to take medicine is, perhaps, the greatest feature which distinguishes man from animals." ? A. William Osler. B. Robert Macintosh. C. George Bernard Shaw. D. Hippocrates. E. Christopher Wren. Select the single best answer ABCDE Correct Answer: A Sir William Osler (1849-1919 ) was the best-known physician in the English-speaking world at the turn of the century, and has been called the "most influential physician in history". He was born on July 12,1849 at Bond Head, Canada West (now Ontario) and died on Dec 29, 1919, in Oxford. He trained in medicine at the Univeristy of Toronto and McGill and obtained an MD at McGill in1872. For the last 15 years of his life he was the regius professor of medicine at Oxford. Number: 732 The first successful blood transfusion from one human to another was performed by: A. Christopher Wren (1660). B. Richard Lower (1666). C. James Blundell (1829). D. Karl Landsteiner (1901). E. Bernard Fantus (1937). Select the single best answer ABCDE Correct Answer: C Around 1660 the English architect and astronomer Christopher Wren began to experiment with the transfusion of blood between animals and intravenous injections (including red wine!) into animals. In 1666, Oxford physician Richard Lower experimented with animal-to-animal transfusion using apparatus described by Wren in the 'Philosophical Transactions of the Royal Society of London' in 1665. In June of the following year, Jean-Baptiste Denys, court physician to Louis XIV, received a 16-year-old boy tormented with fever and terribly weak. Other doctors had bled the boy 20

674

times, and Denys, probably correctly, felt the loss of blood was responsible for the boy's weakened condition. Denys found a gentle lamb, bared its neck, and, using a silver tube, transfused about 9 ounces of its blood into the vein of the patient. The boy felt a very great heat rise up his arm, but survived. Not to be outdone by Denys' work, Lower and his colleague Edmund King transfused lamb's blood into a man in England shortly thereafter. Denys continued to perform animal-to-human transfusions, and not surprisingly, given that issues of blood compatibility were unknown inevitably experienced a fatality, for which he was charged. Eventually Denys was exonerated the patient had died of arsenic poisoning at the hand of his wife but all transfusions involving humans were outlawed by the Parliament of Paris. England and large parts of Europe under papal rule soon followed suit. It would be almost 150 years before the first documented human-to-human blood transfusion. By about 1818, James Blundell, a London obstetrician, became interested in trying to replace the blood of patients who had postpartum haemorrhage. After a series of experiments with animals, in 1829 he saved a haemorrhaging patient with direct human-to-human transfusion. It was not until Karl Landsteiner's discovery of blood groups in 1901 that scientists could explain the phenomenon of blood group incompatibility. Landsteiner, an Austrian who later emigrated to the United States, also discovered the Rh subtypes of human blood, ultimately rendering transfusions safe. In 1930, Moscow physician Serge Yudin saved a patient by transfusing him with blood drawn from the cadaver of an old man who had died six hours earlier. He was able to preserve the blood for later use, and within the next eight years, had transfused 2,500 people using stored cadaver blood. Although cadaver blood transfusion did not catch on in the United States, Dr. Bernard Fantus modified the Soviet idea by preserving blood from healthy living persons. Drawing on earlier work involving preservatives and anticoagulants, Fantus added the element of refrigeration and in 1937 established the first blood bank at Chicago's Cook County Hospital. Number: 836 How many mls of Xenon are found in an average sized room? A. 0.4 mls. B. 4 ml. C. 40 mls. D. 400 mls. E. 4000 mls. Select the single best answer ABCDE Correct Answer: B

675

Xenon constitutes 0.0000087% of the atmosphere, which is estimated to contain around 400 million tonnes. An average room contains about 4 ml. Number: 992 Which of the following literary figures was most likely to have suffered from Obstructive Sleep Apnoea (OSA)? A. Samuel Pickwick. B. John Falstaff. C. Ondine. D. King Alfred of Wessex. E. Austin Powers Select the single best answer ABCDE Correct Answer: B The topic of OSA has recently been expertly reviewed by Loadsman and Hillman. The term 'Pickwickian Syndrome' in fact derives from Charles Dickens portrayal of 'Joe the Fat Boy' in 'The Posthumous Papers of the Pickwick Club', not from Samuel Pickwick himself. Sir John Falstaff was almost certainly a sleep apnoeic. For example: "Poins: Falstaff!fast asleep behind the arras, and snorting like a horse. Prince Henry: Hark, how hard he fetches breath . . . " Act II, Scene IV King Henry IV Part I Ondine's curse was a curse laid upon her husband (Sir Lawrence) in retribution for his infidelity. (Having been caught, by her, asleep with another woman). Sir Lawrence was thus afflicted with a form of central hypoventilation, while Ondine herself retained perfect ventilatory drive. The term is now used to describe patients with Primary Alveolar Hypoventilation. Such patients maintain rhythmic respiration when awake, although the level of ventilation is below normal. However, during sleep, when breathing is critically dependent on the metabolic control system, there is typically a further deterioration in ventilation with frequent episodes of central hypopnoea or apnoea. King Alfred of Wessex was entrusted with minding the cooking of some cakes following a battle with the Danes in 877 AD. He fell asleep on the job and the cakes were burnt. There is, however, no suggestion that he suffered from OSA. Austin Powers' father was a narcoleptic - "My father was a relentlessly self-improving boulangerie owner from Belgium with low grade narcolepsy and a penchant for buggery. My

676

mother was a fifteen year old French prostitute named Chloe with webbed feet. My father would womanize, he would drink. He would make outrageous claims like he invented the question mark. Sometimes he would accuse chestnuts of being lazy. The sort of general malaise that only the genius possess and the insane lament." - However, Austin himself indulged in nothing more stressful than heavy breathing. See: Loadsman JA, Hillman DR. Anaesthesia and sleep apnoea. Br J Anaesth. 2001 Feb;86(2):254-66. Number: 1017 The only mammal which produces a venom capable of significant envenomation in man is the: A. Koala bear (Phascolarctos cinereus). B. Duck-billed platypus (Ornithorhynchus anatinus). C. South american three toed sloth (Bradypus tridactylus). D. Siberian tiger (Panthera tigris altaica). E. Siberian weasel (Mustela sibirica). Select the single best answer ABCDE Correct Answer: B The spurs on the hind legs of the male duck-billed platypus produce a venom which causes persistent, severe pain when injected into man. The substance is a C-type natriuretic peptide of considerable interest to pain researchers. There are at least two case reports of these animals savaging unssuspecting humans. Tonkin and Negrine have, not unreasonably, suggested that "Warning signs should be erected at air and sea ports warning tourists of the dangers of these venomous Australians." The Siberian tiger has a nasty bite.

X RAY TESTS Number: 388 Score: A 60 year old man with a 25 pack year history of smoking presents with haemopytysis. The CT is shown. The most likely diagnosis is: A. Squamous cell carcinoma of the lung, B. Adenocarcinoma of the lung, C. Wegener's granulomatosis. D. Tuberculosis. E. Necrotising sarcoidosis Select the single best answer

677

ABCDE Correct Answer: A The CT shows a spiculated, thick-walled cavitary mass in the right upper lobe. The lesion is surrounded by emphysema. The spiculated appearance is highly suggestive of malignancy. 10-15% of lung malignancies cavitate, the vast majority being squamous cell carcinomas. Also worth consideration for a cavitary upper lobe mass, though much less likely are: Wegener's granulomatosis; necrotising sarcoidosis, and tuberculosis. Anaerobic abscesses (eg bacteroides) are usually located in dependant bronchopulmonary segments. Mucor and Aspergillus invade blood vessels and cause pleural-based, wedge-shaped areas of pulmonary infarction which may subsequently cavitate. Number: 389 A middle-aged female presents with cough and haemoptysis. The most likely diagnosis is: A. Right middle lobe pneumonia. B. Right lower lobe pneumonia. C. Asbestosis. D. Encysted interlobar effusion. E. Pulmonary embolism, Select the single best answer

678

ABCDE Correct Answer: A Right middle lobe pneumonia. There is density in the lower half of the right lung field. It is well defined superiorly at the level of the anterior end of the right 4th rib. The right heart border is blurred. It has lost its silhouette and for this reason, the right middle lobe must be involved. The well-defined upper limit in the PA. view marks the site of the horizontal fissure. The organism which is classically associated with lobar (rather than confluent) distribution is the pneumococcus. Number: 390 55 year old woman admitted unconscious after a overdose of sleeping tablets. Required intubation in casualty. Arterial saturation noted to be 80%. The most likely cause is: A. Aspiration. B. Pneumothorax.

679

C. Adult Respiratory Distress Syndrome. D. Bronchopneumonia. E. None of the above. Select the single best answer

ABCDE Correct Answer: E Right endobronchial intubation: The tip of the endotracheal tube lies in the right lower lobe bronchus. The right lung is large volume, but there is a triangular density at its apex, extending to the right hilum, lying adjacent to the tracheal stripe and blurring its silhouette. This corresponds to the location of the right upper lobe. Although it is displaced, the mediastinum is probably normal. Endotracheal tubes can move in the trachea by +/- 2cms with changes in head posture. 680

Alterations in endotracheal tube position during general anaesthesia. Yap SJ; Morris RW; Pybus DA Anaesth Intensive Care, 22:586-8, 1994 Oct Number: 391 Patient with recently inserted pacemaker complaining of dyspnoea 3 hours after insertion.The most likely diagnosis is: A. Complete heart block. B. Pneumothorax. C. Pacemaker malfunction. D. Air embolism. E. Acute left ventricular failure. Select the single best answer

ABCDE

681

Correct Answer: B Pneumothorax. There is a single chamber pacemaker system in situ. A curvilinear density marks the limit of visible lung architecture, beyond which a featureless crescent of air can be seen in the left upper zone. The incidence of pneumothorax is quoted at between 1% and 2% when the subclavian route is used for pacemaker insertion. Number: 392 2 year old child with respiratory distress the most likely diagnosis is: A. Bronchopneumonia. B. Pneumothorax. C. Epiglottitis. D. Inhaled foreign body. E. Lobar pneumonia. Select the single best answer

682

ABCDE Correct Answer: D Foreign body right main bronchus. The right lung field is large and the gas transradiance has a margin, convex to the left. The right lungs vessels are spread out and obscured by the transradiance, except behind the liver. The mediastinal contents are displaced to the left. The left lung is compressed, but the silhouettes of its vessels are preserved. Peanut aspiration into the right main bronchus with 'ball-valve' obstructive emphysema of the right lung. Aspiration of foreign bodies is a common problem, particularly in children 1 to 3 years old. More than 80% of the aspirated foreign bodies are organic material. Peanuts being the most frequently aspirated objects. Other vegetable derivatives are also often aspirated. Nonorganic inert objects such as beads represent about 17% of inhaled objects. The location of the foreign body depends on the patient's posture at the time of aspiration. The right lung is most often involved because the anatomy of the right mainstem bronchus is more in line with the trachea. If the patient was upright at the time of aspiration, the right lower lobe is most frequently affected. If the patient was supine, the right upper lobe is most often affected. The left lung is affected in 20% to 40% of patients.

Number: 393 A 73 year old woman admitted with a history of increasing confusion. The most likely diagnosis is: A. Acute extradural haematoma. B. Acute subdural haematoma. C. Chronic subdural haematoma. D. Haemorrhagic cerebrovascular accident. E. Meningioma. Select the single best answer

683

ABCDE Correct Answer: C Chronic subdural haematoma. There is a semilunar lesion on the right side of the skull, which comprises an anterior low fluid density (old, liquified haematoma) and a posterior higher density (fresh haemorrhage). In chronic subdural haematoma, a history of preceding trauma is often unclear; 20 to 30 percent of patients fail to give a history of injury, particularly elderly patients or those with a bleeding diathesis. The causative injury may be trivial and is often forgotten because it was remote in time. A period of weeks, or even months, follows when headaches, slowed thinking, confusion, changes in personality, seizures, and/or a mild hemiparesis emerges. Fluctuation in the severity of the headache is typical, often with positional changes. Many chronic subdural haematomas are bilateral and give particularly misleading clinical syndromes.

684

Number: 434 A 45 yr old Australian sheep farmer from central NSW presents with this finding on a routine CXR. He is asymptomatic. Which diagnosis / diagnoses is / are most likely? 1. Lung abscess. 2. Hydatid cyst. 3. Tuberculosis. 4. Pericardial cyst. A: 1,2,3 Correct B: 1,3 Correct C: 2,4 Correct D: 4 Correct E: All Correct

ABCDE Correct Answer: C There is a well defined mass lesion, which lies in the right lower zone. The right hemidiaphragm margin is preserved. The lesion would do for either a hydatid or pericardial cyst. It is, in fact, the former.

685

Slowly enlarging echinococcal cysts generally remain asymptomatic until their expanding size or their space-occupying effect in an involved organ elicits symptoms. Liver and lung are the most common sites. Since 5 to 20 years often elapses before cysts enlarge sufficiently to cause symptoms, the cysts may be discovered as an incidental finding on a routine x-ray or ultrasound study. Pulmonary hydatid cysts may rupture into the bronchial tree or peritoneal cavity and produce cough, chest pain, or hemoptysis. Pericardial ('Springwater') cysts appear as rounded or lobulated deformities of the cardiac silhouette, most commonly at the right cardiophrenic angle. They do not cause symptoms, and their major clinical significance lies in the possibility of confusion with a tumour, ventricular aneurysm, or massive cardiomegaly. Number: 435 A 60 year old woman, who had a sigmoid colectomy 3 years ago. The most likely diagnosis is: A. Primary lung cancer. B. Secondary colonic cancer. C. Lung abscess. D. Encysted pleural effusion. E. Tuberculosis. Select the single best answer

686

ABCDE Correct Answer: B Two lesions (one more obvious than the other) are seen in the right middle zone. The most likely diagnosis is metastatic colonic cancer. Population-screening studies and autopsy surveys have revealed that adenomatous polyps may be found in the colons of about 30 percent of middle-aged or elderly people. Based on this prevalence and the known incidence of colorectal cancers, it appears that less than 1 percent of polyps ever become malignant. Most polyps produce no symptoms and remain clinically undetected.

687

Number: 461 The ellipse (shown in red) on this adult sternum most appropriately represents the surface projection of which intracardiac structure? A. The pulmonary valve. B. The aortic valve. C. The mitral valve. D. The tricuspid valve. E. None of the above. Select the single best answer

ABCDE Correct Answer: A The pulmonary valve. Note that this point does not correspond to the auscultation area of the valve.

688

Number: 493 The most likely diagnosis of this plain X-Ray of the abdomen is: A. Vesico-colic fistula. B. Caecal volvulus. C. Ovarian cyst. D. Acute gastric dilatation. E. None of the above. Select the single best answer

ABCDE Correct Answer: A This plain abdominal X-ray shows an air / fluid level within the urinary bladder. The commonest cause is a vesico-colic fistula. Note that the 'cyst' is arising from the pelvis. The commonest causes are: Diverticular disease (~60%) Colonic carcinoma (~25%) Inflammatory bowel disease (<10%) Radionecrosis following radiotherapy Bladder cancer See also Dis Colon Rectum 1979 Jan-Feb;22(1):27-30 Steele M, Deveney C, Burchell M Diagnosis and management of colovesical fistulas. 689

ECG TESTS Number: 206 This cardiac rhythm is: A. 1st degree Heart Block B. Mobitz type 1 block. C. Mobitz type 2 block. D. Complete heart block. E. None of the above. Select the single best answer

ABCDE Correct Answer: B Second degree Mobitz type 1 block. Note the progressive lengthening of the PR interval until eventually a QRS complex is dropped. Number: 245 A 50 yr old man admitted with a two hour history of central chest pain. This ECG shows: 1. Left Bundle Branch Block. 2. Left Axis Deviation. 3. Ventricular Extrasystoles. 4. Evidence of recent (less than 6 hours) myocardial infarction. A: 1,2,3 Correct B: 1,3 Correct C: 2,4 Correct D: 4 Correct E: All Correct 690

ABCDE Correct Answer: B The axis is normal. There is no evidence of recent myocardial infarction although the history suggests it strongly. The LBBB may mask acute changes. Number: 246 A 50 yr old man admitted with a two hour history of central chest pain. This ECG strip shows: 1. Sinus rhythm with ventricular extrasystoles. 2. Evidence of acute inferior myocardial infarction. 3. Ventricular fibrillation. 4. ECG evidence of hyperkalaemia. A: 1,2,3 Correct B: 1,3 Correct C: 2,4 Correct D: 4 Correct E: All Correct

691

ABCDE Correct Answer: A Although the full ECG montage is not presented, it appears that the VF which is precipitated by the 2nd ventricular ectopic in a classical 'R on T' manner, is due to an acute, inferior myocardial infarction. Number: 247 An 85 year old man presents to a casualty department complaining of exercise induced chest pain and extreme shortness of breath. His blood pressure is 90/60. The most likely diagnosis is: A. Aortic stenosis. B. Acute myocardial infarction. C. Accelerated hypertension. D. Aortic incompetence. E. Ischaemic cardiomyopathy. Select the single best answer

692

ABCDE Correct Answer: A The ECG shows marked left ventricular hypertrophy. In view of the low blood pressure, aortic stenosis should be excluded. There is no evidence of acute myocardial ischaemia although chronic ischaemia is difficult to exclude. Number: 248 The ECG of an asymptomatic 55 year old taken at an insurance examination. It shows: 1. Left axis deviation. 2. Bifascicular block. 3. Right bundle branch block. 4. Left bundle branch block. A: 1,2,3 Correct B: 1,3 Correct C: 2,4 Correct D: 4 Correct E: All Correct

693

ABCDE Correct Answer: A This ECG is compatible with block of the right bundle and anterior fascicle of the left bundle. - So called 'Bifascicular block'. The patient is totally reliant on the posterior fascicle of the left bundle for conduction. Under some circumstances a permanent pacemaker may be indicated for this type of lesion. Number: 413 This rhythm strip is most likely to represent a pacemaker functioning as: A. DOO B. VOO C. VVI D. DDD E. AOO Select the single best answer

694

ABCDE Correct Answer: D In this case, a ventricular spike is seen after every 'P' wave. This implies that atrial systole is being correctly sensed. Therefore, even though only one spike is apparent, the pacemaker must be a dual chamber device in 'non-asynchronous' mode. Of the suggested alternatives, only 'DDD' is compatible with this. A five-letter ICHD (Inter-Society Commission for Heart Disease Resources) code is widely used as shorthand notation to indicate modes of pacing. The first position designates the chamber paced (A = atrium, V = ventricle, or D = dual [both A and V]), the second position the chamber sensed (A, V, D, or 0 = none), and the third position the response to sensed events (I = inhibited, T = triggered, D = double [both I and T possible], or 0 = none). The fourth and fifth positions designate programmable and special antitachycardia functions, but these are rarely used by physicians to describe modes of pacing. However 'R' is commonly seen after the third position (eg DDD-R) to indicate sensor-modulated, rate-adaptive pacing. Rate-adaptive pacing is used in patients with atrioventricular (AV) heart block, and means that to pacemaker automatically adjusts its stimulation rate to meet increased metabolic demands. Number: 453 The ECG / Pressure trace below represents a period of diathermy in a patient with a permanent indwelling pacemaker. The pacemaker mode is most likely to be: A. VOO B. DOO C. DDD D. VVI E. DDI Select the single best answer

695

ABCDE Correct Answer: B Note that the diathermy interference has not inhibited the pacemaker and that both an atrial and ventricular spikes are present. The pacemaker is most likely to be in DOO mode. If unipolar diathermy must be used in a pacemaker dependent patient, the pacemaker should be re-programmed to VOO or DOO. However, it should be noted that this alone does NOT guarantee the safety of the patient. A five-letter ICHD (Inter-Society Commission for Heart Disease Resources) code is widely used as shorthand notation to indicate modes of pacing. The first position designates the chamber paced (A = atrium, V = ventricle, or D = dual [both A and V]), the second position the chamber sensed (A, V, D, or 0 = none), and the third position the response to sensed events (I = inhibited, T = triggered, D = double [both I and T possible], or 0 = none). The fourth and fifth positions designate programmable and special antitachycardia functions, but these are rarely used by physicians to describe modes of pacing. However 'R' is commonly seen after the third position (eg DDD-R) to indicate sensor-modulated, rate-adaptive pacing. Rate-adaptive pacing is used in patients with atrioventricular (AV) heart block, and means that to pacemaker automatically adjusts its stimulation rate to meet increased metabolic demands. Number: 455 The ECG below is most compatible with: A. Acute inferoposterior myocardial infarction. B. Acute anterolateral infarction. C. Acute inferior myocardial infarction. D. Acute inferior myocardial infarction with subendocardial ischemia of anterolateral wall. E. Acute posterior myocardial infarction. Select the single best answer

696

ABCDE Correct Answer: A Acute inferoposterior myocardial infarction: Acute inferior myocardial infarction is evident. Although the ST segments are significantly depressed in leads I and aVL these are merely reciprocal changes of the ST segment elevation in the inferior leads, and do not represent subendocardial ischemia or infarction of the high lateral wall. However, the precordial leads are not the reciprocal leads of the inferior wall, but of the posterior ventricular wall. The ST segment depression in these leads therefore represents a mirror image of ST elevation in the posterior ventricular wall. Number: 456 A 30 year old male with AIDS is admitted to casualty complaining of lethargy and weakness. His ECG is shown below. The most likely cause of these changes is: A. Acute Pericarditis. B. Acute Anteroseptal Infarction. C. Hyperkalemia. D. Anteroseptal Infarction and Hyperkalemia. E. Zidovudine toxicity. Select the single best answer

697

ABCDE Correct Answer: C Hyperkalemia. It is not widely appreciated that hyperkalemia can produce marked ST elevation and can do so selectively, with ST depression elsewhere, so that it looks like a localised infarct. When the serum potassium level was measured it was found to be 8.9 mmol/L and the diagnosis of hypoadrenalism was later confirmed. His T waves are typically "tented" in several leads, which strongly suggest potassium intoxication. Clinical suspicion of adrenal insufficiency should be high in patients with AIDS. Cytomegalovirus regularly involves the adrenal glands (so-called CMV necrotising adrenalitis), and Mycobacterium avium-intracellulare, Cryptococcus, and Kaposi's sarcoma involvement of the adrenals also have been reported. While many patients experience fatigue, malaise, nausea, and headache upon the initiation of zidovudine therapy these side effects often subside over time. The major adverse effect of zidovudine is on the bone marrow with patients often developing a macrocytic anaemia.

698

Number: 458 This ECG shows evidence of two electrolyte abnormalities. They are likely to be: A. Hyperkalaemia and hypocalcaemia. B. Hypokalaemia and hypocalcaemia. C. Hyperkalaemia and hypercalcaemia. D. Hyponatraemia and hypocalcaemia. E. Hyperkalaemia and hypercalcaemia. Select the single best answer

ABCDE Correct Answer: A Hyperkalaemia and hypocalcaemia. The QT interval is prolonged which is the typical feature of hypocalcaemia. In addition, the T waves are tall, narrow and pointed, indicating hyperkalaemia. This combination of electrolyte disturbances is frequently seen in patients suffering from acute renal failure (ARF). ARF impairs renal excretion of sodium, potassium, and water; divalent cation homeostasis; and urinary acidification mechanisms. As a result, ARF is frequently complicated by intravascular volume overload, hyponatraemia, hyperkalaemia, hyperphosphataemia, hypocalcaemia, hypermagnesaemia, and metabolic acidosis. Hypokalaemia is associated with T wave flattening, U waves, and ST segment depression, while hypercalcaemia is associated with tachycardia, and a shortened ST segment and QT interval. Isolated hyponatraemia tends to dispose to intraventricular conduction disorders and reentrant arrhythmias 699

Number: 468 The lateral chest lead shown below is most likely to represent: A. Sinus tachycardia. B. Ventricular tachycardia. C. Atrial fibrillation. D. Atrial flutter with 1:1 response. E. Atrial flutter with 2:1 response. Select the single best answer

ABCDE Correct Answer: D Atrial flutter with 1:1 response. A narrow complex tachycardia at this rate almost always represents atrial flutter with 1:1 conduction. Number: 484 A two-year-old boy comes to medical attention because he has lost consciousness four times over a period of five months. The child is totally deaf. His ECG is shown. The most likely diagnosis is: A. Jervell and Lange-Nielsen syndrome . B. Romano-Ward Syndrome C. Familial hypokalemic periodic paralysis D. Lown-Ganong-Levine syndrome E. Wolff Parkinson White syndrome Select the single best answer

700

ABCDE Correct Answer: A These findings are consistent with the autosomal recessive form of the congenital long-QT syndrome (called the Jervell and Lange-Nielsen syndrome). This syndrome, along with the more common Romano-Ward form of the long-QT syndrome (autosomal dominant inheritance, normal hearing), is associated with sudden cardiac death. Syncopal episodes are probably due to nonsustained polymorphic ventricular tachycardia (torsade de pointes). See also: Paediatr Anaesth 1999;9(2):156-8 Baines DB, Murrell D Preoperative hypoglycaemia, propranolol and the Jervell and Lange-Nielsen syndrome.

Number: 500 ST segment elevation on the ECG may be seen in: 1. Diastolic overload 2. Athletic hearts 3. Myocardial contusion 4. Hyperkalaemia A: 1,2,3 Correct B: 1,3 Correct C: 2,4 Correct D: 4 Correct E: All Correct 701

ABCDE Correct Answer: E 1. Compare with ST depression (strain pattern) seen in systolic overload. 2. Particularly the precordial leads and associated with prominent J waves and deep, narrow Q waves. 4. It is not widely appreciated that hyperkalaemia can produce marked ST elevation. See also: Question 456 and Curr Opin Cardiol 1998 Jul;13(4):248-53 Zalenski RJ, Shamsa FH Diagnostic testing of the emergency department patient with chest pain. Number: 533 In a standard 12 lead ECG in an asymptomatic patient: 1. Right Bundle Branch Block is usually indicative of cardiac pathology. 2. Mobitz type 1 heart block (Wenckebach) is an indication for pacing. 3. Left Bundle Branch Block is usually indicative of cardiac pathology. 4. An inverted T wave in aVR is usually due to lateral ischaemia. A: 1,2,3 Correct B: 1,3 Correct C: 2,4 Correct D: 4 Correct E: All Correct ABCDE Correct Answer: B RBBB often indicates problems in the right side of the heart, but RBBB patterns with a normal duration of the QRS complex are quite common in healthy people. Mobitz type I second-degree AV block (AV Wenckebach block) is characterised by progressive PR interval prolongation prior to block of an atrial impulse. The pause that follows is less than fully compensatory (i.e., is less than two normal sinus intervals), and the PR interval of the first conducted impulse is shorter than the last conducted atrial impulse prior to the blocked P wave. This type of block is almost always localised to the AV node and associated with a normal QRS duration. It is seen most often as a transient abnormality with inferior wall infarction or with drug intoxication, particularly digitalis, beta blockers, and occasionally calcium channel antagonists. This type of block also can be observed in normal individuals with heightened vagal tone. Although Mobitz type I block can progress to complete heart block, this is uncommon.

702

LBBB is always an indication of heart disease. In normal ECGs the T wave is always inverted in VR, but is usually upright in all other leads. Number: 696 In the normal ECG: 1. The PR interval can be up 0.24 sec. in duration. 2. There is a Q wave in V6. 3. The T wave is upright in aVR. 4. The S wave is of greater amplitude than the R wave in V1. A: 1,2,3 Correct B: 1,3 Correct C: 2,4 Correct D: 4 Correct E: All Correct ABCDE Correct Answer: C 1. The upper limit of normal for the PR interval is 0.2 sec. 2. A small Q wave normally occurs in V6 and represents depolarisation of the interventricular septum. 3. Ventricular repolarisation (represented by the T wave), takes place in the opposite direction to that of the major component of the QRS complex. 4. V1 is a right-sided lead and the bulk of ventricular depolarisation (represented by the S wave), is directed away from this lead. Number: 706 In a patient with a normal ECG at a pulse rate of 72 bpm: 1. The QT interval should be less than 0.44 sec. 2. The frontal plane QRS axis should be between - 30 and 100 degrees. 3. The duration of the QRS complex should be less than 0.12 sec. 4. The PR interval should be less than 0.2 sec. A: 1,2,3 Correct B: 1,3 Correct C: 2,4 Correct D: 4 Correct E: All Correct

703

ABCDE Correct Answer: E The QT interval represents the time for a complete ventricular depolarisation and repolarisation cycle. In total it should be less than 0.44 sec. The normal QRS axis should be between - 30 and 100 degrees. A value of less than -30 is left axis deviation and one of more than a 100 right axis deviation. The duration of the QRS complex should be less than 0.12 sec. The PR interval should be less than 0.2 sec. Number: 776 The abnormalites in this ECG are most consistent with a diagnosis of: A. Right Bundle Branch Block. B. Lown-Ganong-Levine Syndrome. C. Hypothermia. D. Wolff-Parkinson-White Syndrome. E. Hypercalcaemia. Select the single best answer

ABCDE Correct Answer: D This is typical of a patient with Wolff-Parkinson-White Syndrome. Note the short PR interval and 'delta' wave on the upstroke of the QRS complex. 704

This picture is not consistent with right bundle branch block (RBBB). - In RBBB, the terminal QRS vector is oriented anteriorly and to the right (rSR' in V1 and qRS in V6, typically). The Lown-Ganong-Levine Syndrome is a pre-excitation syndrome characterised by a short PR interval without the presence of 'delta' waves. Hypothermia leads to the appearance of 'J' or 'Osborn' waves on the downslope of the QRS complex. Hypercalcaemia typically shortens the QT interval (ST portion), while hypocalcaemia lengthens it. Number: 896 Which of the following is/are true of Wolff-Parkinson-White (WPW) syndrome: 1. The P-R interval is typically less than 0.12 sec. 2. It is caused by an aberrant conduction pathway between atrium and ventricle. 3. It may mimic acute myocardial infarction. 4. It may be associated with sudden death. A: 1,2,3 Correct B: 1,3 Correct C: 2,4 Correct D: 4 Correct E: All Correct

ABCDE Correct Answer: E The WPW syndrome occurs when an accessory bundle between the atria and ventricle exists such that conduction occurs via the atrioventricular (AV) node and accessory pathway (AP or Bundle of Kent). A variety of permutations exist resulting in different baseline and episodic ECGs. The classic baseline ECG appearance occurs when conduction proceeds anterograde through both pathways (faster through the AP), such that there is preexcitation of the ventricle (PR< 0.12 sec), which commences depolarization earlier (AP) and completes it at the usual time (AV node). This is manifest as a widened QRS complex with a delta notch on the upstroke. Tachyarrhythmic episodes encompass a variety of arrhythmias. When SVT occurs, the QRS complexes appear normal if the re-entrant rhythm is conducted anterogradely through the AV node and retrogradely though the AP. This occurs commonly. If it is conducted anterogradely through the AP, then the ventricle is depolarised from an ectopic focus. In this event, the ECG appearances are that of a wide complex tachyarrhythmia. This is called SVT with aberrant

705

conduction and differentiating it from left bundle branch block, ventricular tachycardia (VT), or acute infarction may be difficult. 15-20% experience AF. If conduction occurs anterogradely through the AP during AF, then the normal effect of the AV node to limit ventricular response will not occur. This scenario may be associated with actual VT and sudden death. The majority of patients have normal hearts but it may be associated with Ebsteins Anomaly, hypertrophic cardiomyopathy (HOCM), and others conditions. WPW is most commonly manifest as paroxysmal supraventricular tachycardia, although this may vary depending upon the nature of conduction through the accessory pathway. During periods of normal sinus rhythm (SR), conduction will generally be antidromic (forward) through the AV node and AP simultaneously. The atria are activated first then the ventricles. Ventricular pre-excitation occurs as a result of more rapid conduction through the AP, causing the QRS complex to occur early (P-R <0.12 sec) and excitation is prolonged because conduction through the AV node proceeds at its normal rate. This appears as a widened QRS complex with delta notch on upstroke.The rate is normal. This pattern during sinus rhythm is referred to as a " manifest " AP. In some patients an AP exists but only ever conducts in a retrograde manner. These patients may also experience SVT but during normal SR preexcitation does not occur (no delta or short P-R). These are known as " concealed " APs. 50% will experience paroxysmal tachycardias. Underlying WPW accounts for 25% of SVTs in general. Orthodromic conduction is most common, antidromic is rare. In addition, 15-20 % experience paroxysmal AF. In orthodromic SVT there is anterograde conduction through the AV node, and retrograde through the AP. Activation of atria follow the ventricles, hence p waves follow QRSs. The ventricular activation sequences are normal, hence QRS complexes are not widened and delta waves do not occur unless aberrant conduction exists. In antidromic SVT, the AP conducts anterograde and the AVN retrogradely. The p wave immediately follows the QRS but may be difficult to detect. The ventricle is exclusively activated by the AP therefore the QRS is widened and the delta is pronounced resembling VT. If conduction through the AP is anterograde during AF, ventricular rates of >300 can occur resulting in haemodynamic compromise and degeneration to VF. This may cause sudden death. The mechanism for this is that during normal conduction through the AV node, action potentials are conducted decrementally (prolonged with increasing rate) which offers some protection with respect to ventricular response. This phenomenon does not occur in APs Therapy varies: (1) If the patient is asymptomatic and not at risk of VF it does not require treatment. (2) SVT can be treated with drugs that alter conduction through the AVN(adenosine, digoxin, B Blockers, verapamil) or AP (lignocaine, procainamide). (3)AF with a rapid ventricular response is a medical emergency and requires immediate cardioversion. AF with a moderate ventricular response may be treated with a drug causing slowing through the AP. Digoxin and verapamil may accelerate the ventricular rate and should not be used. (4)Prophylactic drug therapy and electrophysiological ablation of the AP may be used in those who are at risk of sudden death (persistent preexcitation during exercise testing as detected by electrophysiological testing- this is normally lost) References The Washington Manual-Manual of Medical Therapeutics, Little Brown & Co., 27th Ed., 1992, pp 133-5, 142-4.

706

GENERAL PRIMARY MCQS General Primary MCQ 1 1. Regarding two compartment pharmacokinetics: a) a drug is always removed from the peripheral compartment b) the central compartment is blood volume c) a drug with a high volume of distribution is likely to be lipophillic d) a drug can have a short duration of action while being eliminated very slowly e) most anaesthetic drugs are modelled well with a two-compartment model 2. Desflurane: a) is a fluorinated methylisopropyl ether b) boils at 23 degrees C c) is safe to use in patients with malignant hyperpyrexia d) stimulates the sympathetic system when inspired concentration is suddenly increased e) prolongs the duration of muscle relaxants 3. Regarding the use of suxamethonium: a) bradycardia is a complication b) phase II block occurs more commonly with neonates c) prolonged duration of action may be seen in around one patient in 40 d) can raise the serum potassium by 0.5 mmol/ L e) dose required is lower in small children 4. Concerning electroencephalography (EEG): a) voltages are in the range of 10-100 millivolts b) spontaneous EEG activity is lost when the body temperature drops below 25 degrees C c) b waves are enhanced by sedatives d) d waves only occur in brain injury e) q waves occur at a frequency of 4-7 Hz 5. Action potentials: a) are all or none signals of about 100 mV in amplitude b) are generated by leakage of K+ down their concentration gradient c) are normally conducted antidromically d) summate at high frequencies e) depend on the size of the stimulus 6. Conduction velocity of a nerve impulse: a) is greater in C fibres than in group A fibres b) is greater in large diameter nerve fibres c) is greater in unmyelinated nerve fibres because of saltatory conduction

707

d) can be as fast as 120 m/s in human nerve fibres e) is decreased in hypothermia

7. Regarding drug metabolism by cytochrome P450 isoenzymes: a) cytochrome P450 makes up 1% of total liver proteins b) the most important enzyme is CYP3A4 c) the system is responsible for most of the reductive metabolism in humans d) volatile anaesthetics are metabolised by CYP2E1 e) some isoenzymes are inhibited strongly by erythromycin 8. Atropine: a) may cause bradycardia b) dilates the pupil in premedicant dose c) has a shorter duration of action than glycopyrrolate d) increases the physiological dead space e) has both muscarinic and nicotinic effects 9. In the pulmonary circulation: a) capillary hydrostatic pressure is about 25 mmHg b) 50% of the cardiac output goes to the pulmonary circulation in the foetus c) angiotensin is broken down d) bradykinin is inactivated e) hypoxia causes vasoconstriction 10. Regarding local anaesthetic agents (LA): a) the potency of LAs is proportional to their lipid solubility b) the duration of action is dependent on protein binding c) agents with low pKa have a faster onset of action d) all local anaesthetics are vasodilators e) the depth of local anaesthetic block is increased by increasing the dose 11. Cisatracurium besylate: a) is a mixture of three stereoisomers b) in equipotent doses has a similar duration of action to vecuronium c) is less potent than atracurium d) undergoes more Hoffmans degradation than atracurium e) in equipotent doses has a similar onset time to atracurium 12. If an electric current is fed through the body: a) risk of injury is largely dependent upon the current flow b) antistatic shoes provide good protection due to their high resistance c) high frequencies are more dangerous than low frequencies

708

d) ventricular fibrillation occurs at a lower current in patients with dysrhythmias e) a tingling sensation is felt at a current strength of 1 mA 13. The countercurrent concentrating mechanism in the kidney: a) depends on active transport of sodium and chloride out of the ascending loop of Henle b) allows an osmolality of 1200 mosmoles/kg in distal tubules c) occurs predominantly in the cortical nephrons d) relies on the free movement of water and electrolytes across the walls of the vasa recta e) depends on a low concentration of urea in the medullary interstitium 14. The adverse effects of NSAIDs on the kidney: a) are reversible in normal kidneys b) are not dose related c) are mediated by inhibition of PGI2 synthesis d) may cause acute interstitial nephritis e) are counteracted by the use of ACE inhibitors 15. The following trigger the secretion of antidiuretic hormone from the posterior hypothalamus: a) a 5% reduction in extracellular fluid b) chronic renal failure c) anxiety d) supine position e) head injury 16. Concerning the measurement of oxygen: a) an oxygen electrode should be calibrated at zero and in room air b) oxygen tension in a liquid can be measured with a Clark electrode c) a polarographic electrode can be used in vivo d) oxygen measurement in a gas mixture makes use of the magnetic property of oxygen e) a fuel cell has a rapid response to change in oxygen concentration 17. Drug clearance: a) is the amount of drug removed from plasma in unit time b) is proportional to half-life c) is low in lipid-soluble drugs d) occurs only in the liver and kidney e) is calculated by dividing the dose of drug given by area under plasma concentration-time curve 18. The volume of distribution of a drug: a) is low if the drug is highly protein bound b) can be calculated by multiplying half-life by natural logarithm of 2 c) is relatively low for muscle relaxants

709

d) is proportional to half life e) is dependent on the elimination rate constant 19. Concerning composition of body fluids: a) plasma constitutes a quarter of extracellular fluid (ECF) volume b) ECF volume may be grossly depleted in intestinal obstruction c) the protein content of interstitial fluid is higher compared with intracellular fluid (ICF) and plasma d) the ratio of ECF/ICF volume is smaller in infants and children e) the normal osmolality of plasma is 280 mosmoles/kg 20. Aldosterone causes: a) a decrease in urine sodium concentration b) weight gain c) decreased serum chloride level d) increased extracellular fluid volume e) increased K+ excretion 21. The following antibiotics have good activity against anaerobic bacteria: a) vancomycin b) aztreonam c) metronidazole d) imipenem e) trimethoprim 22. Flumazenil: a) may induce panic attacks in susceptible patients b) has anticonvulsant activity in patients with epilepsy c) has a long duration of action d) may cause nausea and vomiting e) has inverse agonist action at benzodiazepine receptors 23. Concerning the blood brain barrier (BBB): a) it is virtually impermeable to ions and proteins b) it breaks down around brain infarcts c) a rise in serum albumin by 20 g/L will draw more H2O across the BBB than a rise in serum Na+ by 5 mmol/L d) when damaged, cytotoxic brain oedema results e) mannitol crosses the BBB easily 24. Osmolality: a) is the number of osmotically active particles per litre of solvent b) of urine is similar to that of plasma in chronic renal failure c) may be estimated by formula 2X(Na+K) + Blood sug + BUN

710

d) is measured by amount of depression of the freezing point e) is a part of colloid oncotic pressure 25. ABO compatibility is essential for transfusion of: a) SAGM blood b) haemoglobin solutions c) cryoprecipitates d) FFP e) platelets 26. The following increase during pregnancy: a) plasma volume b) fibrinogen c) gastric emptying time d) glucose tolerance e) arterial PaCO2 27. Carbon monoxide: a) binds to haemoglobin with 100 times the affinity of oxygen b) results in the oxyhaemoglobin curve shifting to the left c) poisoning can be reliably detected by pulse oximetry d) levels in normal non-smokers is 10-15% e) concentration in circle is increased during desflurane anaesthesia 28. Resting potential across the nerve membrane: a) depends largely on the ratio of K+ inside and outside the cell b) is positive inside with respect to outside c) is of the order of 0.06 volt d) decreases in magnitude during prolonged hypoxia e) is greater the larger the diameter of the nerve fibre 29. Activation of NMDA receptors: a) is important in learning and memory b) can result in neuronal damage c) is involved in the development of opioid tolerance d) causes opening of the chloride channel e) may increase the intensity of pain 30. Bacteria develop resistance to antibiotics: a) by changing permeability of porin channels in cell wall b) by producing enzymes to inactivate antibiotics c) by altering target sites (DNA gyrase and topoisomerase) for antibiotics d) by active extrusion of antibiotic once it enter the cell

711

e) easily in presence of necrotic tissue ANSWERS 1.FFTTT 2.FTFTT 3.TFFTF 4.FFTFT 5.TFFFF 6.FTFTT 7.TTFTT 8.TTTTF 9.FFTTT 10.TTTFT 11.FTFTF 12.TTFTT 13.TFFTF 14.TFTTF 15.FFTFT 16.FTTTF 17.FFFFT 18.TFTTF 19.TTFFT 20.TTFTT 21.FFTTF 22.TFFTT 23.TTFFF 24.FTFTF 25.TFTTF 26.TTTTF 27.FTFFT 28.TFTTF 29.TTFFT 30.TTTFT General Primary MCQ 2 1. P50: a) is normally 5.5 kPa b) is increased at high altitude c) is an indicator of the position of the oxygen dissociation curve d) is increased in foetal blood e) is increased in banked red blood cells 2. The work of breathing: a) is inversely related to lung compliance

712

b) increases during exercise c) normally requires 15% of total body oxygen consumption d) is increased when the subject breathes heliox instead of room air e) is calculated by integration of a pressure/volume loop 3. Alveolar - arterial oxygen difference (A-a DO2): a) is normally 2-3 kPa while breathing room air b) is increased under anaesthesia due to increased V/Q mismatch c) is decreased in one lung ventilation d) is increased in the presence of right to left intracardiac shunts e) is decreased in severe exercise 4. Lung compliance: a) is normally 0.2 L/cm H2O b) is decreased with loss of pulmonary surfactant c) is increased in emphysema d) is decreased after induction of general anaesthesia e) is different at the apices and bases of lungs 5. An arterial blood sample has the following values: pH 7.25, PCO2 4 kPa, PO2 8 kPa, base excess -5.6 mmol/L, standard bicarbonate 20 mmol/L These could result from: a) an intracardiac left to right shunt b) Acute respiratory distress syndrome c) chronic obstructive airways disease d) aspirin overdose e) diabetic ketoacidosis

6. The Magill (Mapleson A) breathing system: a) is the most efficient system for spontaneously breathing patients b) will work with minimal rebreathing at a fresh gas flow of 70% of minute volume c) makes scavenging of exhaled gases easier d) co-axial version is the Lack system e) is also efficient during controlled ventilation 7. The respiratory centre in the brain stem receives input from: a) the aortic and carotid bodies b) bronchiolar stretch receptors via vagal afferents c) oxygen-sensitive chemoreceptors in the medulla d) receptors which respond to the hydrogen ion concentration in the cerebrospinal fluid e) mechanoreceptors of the larynx 8. The carotid bodies:

713

a) have cells which respond only to decrease in PaO2 b) have the highest blood flow (ml/unit weight) in the body c) maximally stimulate the respiratory centre between 4-8 kPa d) increase the respiratory drive when mean arterial pressure decreases below 70 mmHg 9. Concerning pulmonary function tests: a) FEV1 is low in small airway disease b) FEV1 is also effort dependent c) The flow-volume loop can indentify the source of airway obstruction d) the diffusing capacity of the lung for carbon monoxide is decreased in emphysema e) FEV1 and PEFR can be used to assess the reversibility of airway obstruction 10. Mixed venous oxygen saturation: a) is normally between 65-80% b) is essential in calculations of shunt and oxygen extraction ratio c) is increased in hypothermia d) is decreased in low cardiac output states and in thyrotoxicosis e) can be continuously monitored with an oximetric Swan Ganz catheter 11. With a decrease in body temperature: a) P50 and PaCO2 are decreased b) pH is increased c) arrhythmias are increasingly common at temperatures below 30 degrees C d) blood becomes more viscous e) the EEG becomes isoelectric at about 20 degrees C 12. Concerning the pharmacokinetics of inhalational agents: a) agents with a high blood:gas partition co-efficient will keep alveolar to inspired gas (FA/FI) ratio low b) alveolar partial pressure rises faster in adults than in children c) high cardiac output slows the rate of rise of alveolar partial pressure (FA) d) diffusion hypoxia is due to the second gas effect e) the second gas effect slows the rate of rise of alveolar partial pressure (FA) 13. The following statements regarding inhalational agents are correct: a) the blood gas partition co-efficient of desflurane is 0.42 b) desflurane can cause sympathetic stimulation c) sevoflurane can degrade to compound A in soda lime d) desflurane is suitable for inhalational induction e) desflurane can be used in halothane vaporiser 14. Nitrous oxide (N2O): a) has a blood:gas partition coefficient of 0.47

714

b) is 34 times less soluble than N2 c) may be associated with postoperative hearing loss d) inactivates methionine synthetase after prolonged administration e) when inhaled at 66% can double a pneumothorax in 10 minutes 15. The speed of induction with inhalational agents: a) is quicker in patients with very high functional reserve capacity b) is quicker if the agent is very soluble in blood c) is slower in the presence of right to left shunt d) is unchanged in the presence of left to right shunt with normal systemic flow e) is quicker when used with nitrous oxide 16. With reference to the skeletal muscle myofilaments: a) actin is the major constituent of thin filaments b) myosin and tropomyosin combine to form the thick filaments c) troponin is a constituent of thin filaments d) tropomyosin prevents the interaction between actin and myosin in the resting state e) troponin C has 4 calcium binding sites 17. The following receptors are part of a ligand-gated ion channel: a) opioid m receptor b) muscarinic cholinoceptors c) nicotinic cholinoceptors d) GABA A receptor e) GABA B receptor 18. Drugs with anti-emetic activity: a) must cross the blood-brain barrier to be effective b) include agents which are selective dopamine D1 receptor antagonists c) include histamine H1 receptor agonists d) are ineffective orally e) include propofol 19. Concerning the rate of diffusion of a gas: a) it is a result of the random movement of the gas molecules b) it is proportional to the tension gradient c) it depends upon the gas temperature d) it is inversely related to the square root of the gas density at constant temperature e) carbon dioxide diffuses more rapidly than oxygen 20. Concerning the use of lasers: a) most lasers use light of wavelengths in the visible and infrared spectrum b) the CO2 laser is strongly absorbed by water, blood and tissues

715

c) the Nd-Yag laser is absorbed mostly by pigments d) nitrous oxide should be avoided during the use of a laser near the airway e) endotracheal tube cuffs should be inflated with saline or water 21. Concerning pacemakers: a) AOO is a fixed rate type of pacemaker b) VVI is the most common type of synchronous pacemaker c) failure to capture is never seen with hyperkalaemia d) unipolar electrocautery is prefered in patients with a pacemaker e) the ground plate of the electrocautery should be placed as far as possible from the pacemaker 22. Regarding defibrillation: a) the greater the time interval between onset of ventricular fibrillation and defibrillation, the less the success of defibrillation b) 5-40 joules should be applied to the heart if the chest is open c) paddles should be of 13 cm in diameter in adults d) defibrillation is most effective when the electric shock is delivered during inspiration e) the myocardium is refractory to defibrillation in hypothermia 23. Regarding acid-base balance: a) chronic obstructive airways disease patients have high serum bicarbonate levels b) the normal anion gap is 20-25 mmol/L c) mixed venous pH is always lower than arterial pH d) the pKa for bicarbonate buffer is 6.1 e) kidneys cannot produce urine with a pH <4.4 24. Baroreceptors: a) in the carotid sinus are innervated by the vagus nerve b) are stretch receptors c) reset the threshold for firing in chronic hypertension d) are made less sensitive by volatile agents e) also respond to the changes in blood pH and PaCO2 25. Ablation of the stellate ganglion causes: a) dilatation of the ipsilateral pupil b) vasodilatation of the ipsilateral arm c) posteral hypotension d) loss of consensual light reflex e) loss of ipsilateral lacrimation

716

26. Asystole: a) occurs in about 25% of hospital patients b) is likely to follow bradycardia associated with mobitz type II AV block c) only rarely follows ventricular fibrillation d) when unresponsive to high dose adrenaline (5 mg) should be treated by external pacing e) may respond to precordial thump after basic life support 27. Regarding coronary blood flow: a) subendocardium is more vulnerable to ischaemia than epicardium b) adenosine and dipyridamole are coronary vasodilators c) myocardial oxygen consumption and coronary blood flow bear a linear relationship d) coronary blood flow may be improved by slowing the heart rate e) the normal myocardial oxygen extraction ratio is 70% 28. The following are true of local anaesthetic toxicity: a) as plasma levels gradually rise, cardiotoxicity is an early likely event b) lidocaine toxicity causes its earliest symptoms at levels of 5 mg/ml c) profound vasodilatation is the mechanism of hypotension d) emergency treatment involves treating convulsions and administering antiarrhythmic agents 29. The following statements about acetylcholine are true: a) its synthesis requires ATP b) it is broken down by choline acetyltransferase in the synaptic cleft c) it is broken down to acetic acid and choline d) it is released only by preganglionic fibres in the autonomic nervous system e) it acts by diffusing through channels in the postsynaptic membrane 30. The dinamap non-invasive blood pressure monitor: a) uses a double cuff oscillotonometric system b) needs to be at the same level as the patient c) is accurate even when an abnormal rhythm is present d) gives a more accurate measure of the diastolic than systolic pressure e) is more accurate than a manual oscillotonometer ANSWERS 1.FTTFF (decreased 2-3DPG in banked RBCs causes a LEFT shift of curve and therefore DECREASED P50) 2.TTFFT 3.TTFTF 4.TTTTT 5.FTFTT 6.TTFTF

717

7.TTFTT 8.FTTT 9.TTTFT (Lung compliance increases in emphysema because of the less elastic lung tissue) 10.TTTTT 11.TFTTT 12.TFTFF (2nd gas effect SPEEDS UP the rate of rise of alveolar pp of the inhalational agent) 13.TTTFF 14.TFTTT 15.FFTTT 16.TFTTT 17.FFTTF 18.FFFFT (most do in fact cross the BBB and act directly on the Vomiting Centre but the CTZ is outside the BBB and DA and 5HT3 blockers antgonise these NTs peripherally) 19.TTFFT 20.TTTTT 21.TTFFT 22.TTTFT 23.TFTTT 24.FTTTF 25.FTFFT 26.FTFFF 27.TTTTT 28.FTTF 29.TFTFF 30.FFFFF General Primary MCQ 3 1. Hyponatraemia: a) may increase intracellular fluid volume b) may be seen in syndrome of Inappropriate antidiuretic hormone c) may increase the secretion of atrial natriuretic peptide d) may increase the plasma osmolality e) of acute onset may be associated with cerebral oedema 2. Adrenaline: a) can be nebulised b) is a bronchodilator c) may elevate the blood sugar d) has an almost equal effect on both alpha- and beta-adrenergic receptors e) tracheal administration of adrenaline should be used even in presence of IV access during cardiopulmonary resuscitation 3. Heparin: a) has a molecular weight between 3,000-60,000 daltons b) acts by binding to antithrombin III c) has antiplatelet activity

718

d) prolongs the prothrombin time e) has a shorter duration of action than low molecular weight heparin 4. Glycopyrrolate: a) can act at central cholinergic receptors b) can increase the physiological dead space c) can dilate the pupil d) is equally effective when given orally e) is five times more potent as an antisialagogue than atropine 5. Isoprenaline: a) can be given by mouth b) is absorbed from the sublingual route c) causes complete heart block d) has more beta than alpha effect on the heart e) antagonises the action of phentolamine 6. In an awake upright spontaneously breathing patient: a) ventilation per unit lung volume is smallest at the apex of lungs b) basal alveoli are smaller c) basal alveoli expand more than apical alveoli during inspiration d) pleural pressure is less negative at the base of the lungs 7. Regarding nitrates: a) they are nitric oxide donors b) they increase the level of intracellular cyclic GMP c) they have an onset of action faster than that of sodium nitroprusside d) they may cause methaemoglobinaemia e) tolerance may develop 8. Ephedrine: a) shows tachyphylaxis due to downregulation of beta receptors b) readily crosses the blood-brain barrier c) releases noradrenaline at sympathetic nerve endings d) has both alpha and beta effects e) reduces placental blood flow 9. In a normal healthy man at rest in the supine position: a) left ventricular end-diastolic volume is about 20 ml b) the first heart sound coincides with the onset of ventricular systole c) cardiac output is approximately 75 ml/beat d) left ventricular end-diastolic pressure is about 5 mmHg e) the second heart sound coincides with the end of the T wave of the ECG

719

10. The rate of diffusion of a gas (Ficks Law) across a membrane: a) is directly proportional to the area b) is directly proportional to the partial pressure gradient c) is inversely proportional to thickness of the membrane d) is directly proportional to molecular weight e) is inversely proportional to the density of the gas 11. Third generation beta-adrenergic blockers: a) include celiprolol and dilevalol b) block the beta-1 receptors c) stimulate beta-2 receptors d) appear well tolerated in asthmatics e) may reduce juxtaglomerular renin release by beta-1 blockade 12. Methaemoglobinaemia: a) may be caused by prilocaine b) may be beneficial in cyanide toxicity c) is effectively treated by methylene blue d) will not change the partial pressure of oxygen in blood e) is seen in carbon monoxide poisoning 13. Concerning the ABO blood groups: a) a person of group O is a universal recipient b) a person of group B always has anti-A agglutinins in his plasma c) in an incompatible blood transfusion reaction, donor cells are lysed by recipient antibodies d) disseminated intravascular coagulation may be a clue to mismatched transfusion e) a saline cross-matching takes 10 minutes to perform 14. Aspirin at the therapeutic dose: a) inactivates the platelet cyclo-oxygenase system temporarily b) causes sweating by a direct action on the post-ganglionic nerves c) acts in fever to reset the central thermostat d) will not antagonise the hyperthermia of exercise e) does not alter the requirement of heparin for anticoagulation 15. Oral anti-coagulant treatment: a) may be teratogenic in early pregnancy b) can cause haemorrhagic foetal death in the third trimester c) precludes breast feeding d) predisposes to particular hazards in patients undergoing prostatectomy e) is best withdrawn 5 days before elective surgery

720

16. Haemaccel: a) is a derivative of starch b) has an average molecular weight of 35,000 daltons c) is equally distributed throughout the extracellular fluid d) has a half life of 8 hours e) is hypotonic 17. Regarding blood transfusions: a) they may result in hypercalcaemia b) in an emergency, it is safe to give O+ blood to an O-ve patient c) they are contraindicated in head injury d) they may result in consumption coagulopathy e) when rapid will often cause hyperthermia 18. The haemoglobin oxygen dissociation curve is moved to the left by: a) ascending to 6000 feet over 24 hours b) carbon monoxide c) frozen red cells d) bovine haemoglobin e) anaemia 19. Amiodarone: a) prolongs the action potential duration in the SA node b) prolongs repolarisation in the AV node c) may cause photosensitivity d) may have effect on thyroid function on long-term treatment e) slows conduction velocity in His-Purkinjee system 20. Enzyme induction: a) occurs in both hepatic and extrahepatic tissues b) develops within 5 days of administration of an inducing agent c) is usually brought about by lipid-insoluble substances with a short half-life d) enables the body to adapt to varying exposure to foreign compounds e) can be a cause of loss of anticoagulant control 21. Pulmonary artery wedge pressure will be greater than left ventricular end diastolic pressure in the following situations: a) presence of mitral valve prosthesis b) positive end-expiratory pressure during mechanical ventilation c) catheter tip in Zone I d) non-compliant left ventricle e) mitral stenosis

721

22. Third generation cephalosporins: a) in general are active against gram +ve cocci b) in general are active against Escherichia coli, Klebsiella and Proteus mirabilis c) may be nephrotoxic in combination with aminoglycosides d) may have disulfiram like action e) is the drug of choice for prophylaxis against bacterial endocarditis 23. Compared with intracellular fluid, extracellular fluid has: a) a greater osmolarity b) a higher protein concentration c) a lower chloride ion concentration d) a lower hydrogen ion concentration e) a lower potassium ion concentration 24. With reference to the normal human heart: a) the most rapidly conducting fibres in the heart are the Purkinje fibres b) the last part of the ventricle to be activated is the apex c) the duration of an action potential in a ventricular muscle fibre is about the same as in a skeletal muscle fibre d) the T wave of the ECG occurs at the beginning of the absolute refractory period of the ventricle e) left axis deviation leads to abnormally large R wave in Standard Limb Lead I 25. Concerning dopamine receptors: a) D1 receptors are present in renal and mesenteric blood vessels b) D1 receptors in renal tubules may induce natriuresis c) D1 receptor stimulation may lead to renin release d) D2 receptors are present in the nigrostriatal pathway e) fenoldapam selectively acts on D1 receptors 26. Pulmonary vascular resistance a) is increased at high altitude b) is decreased by a low pH c) can be measured using a flow-directed balloon catheter d) is increased by isoprenaline e) is decreased by 5-HT 27. A plasma sodium concentration of 120 mmol/L may be a consequence of: a) Conns syndrome b) prolonged unconsciousness c) analgesic-induced nephropathy d) diabetes insipidus e) acutely decreased renal blood flow

722

28. Factors determining urinary sodium loss include: a) plasma sodium concentration b) glomerular filtration rate c) circulating renin levels d) distal tubular fluid anion concentration e) plasma hydrogen ion concentration 29. Atrial natriuretic peptide (ANP): a) is released via coronary sinus b) is a potent vasodilator c) improves renal blood flow d) increases the glomerular filteration rate e) may augment the release of antidiuretic hormone 30. Excitatory amino acids: a) include aspartate and glutamate b) act on NMDA, kainate and quisqualate receptors c) may induce uncontrolled sodium and calcium influx to the cell d) can be antagonised by ketamine e) may potentiate pain transmission ANSWERS 1.TTTFT a)see p.680, Kumar and Clark: fall in ECF osmolality causes H2O to move into cells. 2.TTTTF 3.FTTTT a) most texts quote max MW of 40,000 d) in high doses, PT is prolonged. 4.FTTFT c) hence use with caution in glaucoma. 5.TTFTF 6.TTTT 7.TTFTT 8.FTTTF 9.FTTTT 10.TTTFF 11.TTTTT 12.TTTTF 13.FTTTF 14.FFTTT 15.TTFTF 16.FTFFF 17.FTFTF 18.FTTTF 19.TTTTT 20.TFFTT 21.TTTFT 22.TTTTF

723

23.FFFTT 24.TFFFF 25.TTTTT 26.TFTFF 27.FFTFF 28.TTTTF 29.FTTTF 30.TTTTT General Primary MCQ 4 1. Severe hyperkalaemia is suggested by: a) absent p waves in the ECG b) chronic ACE inhibitor intake c) decreased serum bicarbonate level d) suxamethonium administration in immediate burn e) aldosterone deficiency 2. Glycine: a) is a non-essential amino acid b) can be used as an irrigating solution in transurethral resection of the prostate c) acts as an inhibitory neurotransmitter in the spinal cord d) toxicity can cause permanent blindness e) solution is highly ionised 3. Regarding pH and ionic dissociation: a) a weak acid will be 1000 times more ionised at a pH of 7 than at a pH of 4 b) a strong alkali will have a high pH c) the pH is inversely proportional to the hydrogen ion concentration d) a pH of 7.7 corresponds to a hydrogen ion concentration of 20 nmol/ L e) in the adult, there is a hundred fold range of H+ concentration that is compatible with life 4. Regarding 5-hydroxytryptamine (5-HT): a) it is synthesised from L-tryptophan in the enterochromaffin cells b) it is metabolised to hydroxyindole acetic acid c) it causes bronchospasm and vasodilatation by interacting with 5HT-2 receptors d) a 5HT-3 antagonist is a good anti-emetic e) it may have a platelet aggregating effect 5. Rocuronium: a) is an aminosteroid b) is stable in aqueous solution c) undergoes principally hepatic elimination d) can provide intubating conditions within 60 seconds e) does not release histamine

724

6. Propofol: a) is insoluble in water b) is bound to albumin up to 97-98% c) reduces sodium channel opening times in neuronal membranes d) is isotonic e) does not cause tachycardia 7. S(+) ketamine compared with R(-) ketamine: a) is 3-4 times more potent b) is associated with less incidence of emergence reaction c) is a better analgesic d) has a faster recovery e) has a higher affinity for NMDA receptors 8. Concerning the physiology of the stomach: a) acid secretion in response to hypoglycaemia is mediated by gastrin b) gastric emptying is quickened by sympathetic stimulation c) acid secretion is mediated by H2, M1 and gastrin receptors d) gastric pH is normally around 2-3 e) acidity is reduced by proton pump inhibitors 9. The ratio of intravascular hydrostatic pressure to colloid hydrostatic pressure (according to Starlings Forces) is greater: a) in splanchnic capillaries than in renal glomerular capillaries b) than normal in hepatic failure c) than normal in haemorrhagic shock d) than normal in capillaries where the oedema is due to venous obstruction e) in systemic than in pulmonary capillaries 10. The effects of moving from sea level to an altitude of 5000 m include: a) increased alveolar ventilation due to central chemoreceptor stimulation b) increased blood bicarbonate level c) reduced exercise tolerance d) alveolar PO2 nearly reaches PO2 of air e) increased plasma volume 11. An environmental temperature of 40 degrees C: a) is thermoneutral if there is a strong wind b) leads to vasodilatation of skin vessels c) results in heat stroke if the relative humidity is 100% d) causes the body to sweat with a lower sodium concentration e) is appropriate for people in training for heat adaptation

725

12. Longer term adjustments to altitude hypoxia include: a) hyperventilation b) kidneys slowly lose bicarbonate c) increased serum level of erythropoietin d) increased number of mitochondria e) heart rate remains elevated 13. Concerning patterns of stimulation for monitoring of neuromuscular junction: a) 100 Hz tetanic stimulation is commonly used b) frequency of train of four is 2 Hz c) first response of train of four is equivalent to single twitch response d) train of ratio of 0.7 corresponds to 100% recovery of single twitch e) double burst stimulation consists of two short tetanic stimulations separated by 750 ms 14. Non-depolarising muscle relaxants: a) demonstrate fade due to their presynaptic action b) may not show post-tetanic facilitation c) with large dose may block the diaphragm first before adductor pollicis d) have onset defined as time from administration to abolition of response to single twitch 15. The sequence of events in muscle contraction: a) action potential depolarises the T-tubules b) depolarisation of T-tubules release calcium from sarcoplasmic reticulum c) calcium binds to the troponin-tropomyosin complex d) actin combines with myosin ATP leading to cross-bridge activation e) calcium moves back into the sarcoplasmic reticulum by passive transport 16. Morphine: a) 10-30% of oral morphine reaches the systemic circulation b) has a terminal half-life of approximately 3 hours c) has morphine-3-glucoronide as principal metabolite d) morphine-6-glucoronide has no analgesic effect e) increases the secretion of antidiuretic hormone 17. Amiodarone: a) blocks the potassium channels b) prolongs the refractory period of myocardium and entire conducting tissues c) has a elimination half-life of approximately 28 days d) may result in pulmonary fibrosis on prolonged use e) may cause hypothyroidism 18. Regarding the pharmacokinetics of volatile agents: a) blood/gas partition coefficient of desflurane is 0.69

726

b) sevoflurane can be used for inhalational induction in adults c) inspired concentration influences the rate at which alveolar concentration is attained d) alveolar concentration during recovery decreases more slowly after prolonged anaesthesia 19. Breathing (reservoir) bags: a) are designed to distend enormously when subjected to pressures above 50-60 mmHg b) are of optimal size that fits neatly in one hand c) prevent wastage of fresh gas flow during expiratory pause d) provide a rough visual assessment of volume of ventilation e) act as a reservoir because anaesthetic machine can not provide the peak inspiratory flow required in normal respiration 20. Regarding breathing systems used in anaesthesia: a) Magill system is most efficient for spontaneously breathing patients even at a fresh gas flow (FGF) of 70% of minute ventilation b) D, E and F systems are all T pieces c) Bain system requires a FGF of 50-60 ml/kg during spontaneous breathing d) the rate of change of vapour concentration in circle system depends on circle system volume, the FGF rate and net gas uptake e) Magill system is inefficient during controlled ventilation because much of the gases are vented via pop-off valve 21. A typical motoneurone: a) innervates only one skeletal muscle cell b) has a myelinated axon c) would have its cell body only in the ventral horn of the spinal cord d) might itself receive an input directly from Ia afferent fibres in the spinal cord e) would be stimulated by application of glycine to its cell body 22. The following are statements about acetylcholine: a) its synthesis requires ATP b) it is broken down in the synaptic cleft by choline acetyltransferase c) it is broken down to acetic acid and choline d) it is released only by preganglionic autonomic fibres e) it acts by diffusing through channels in the postsynaptic membrane 23. The velocity of conduction of a nerve action potential: a) is inversely proportional to the cross-sectional area of the axon b) is faster in a myelinated fibre than in a non-myelinated fibre of the same diameter c) is decreased by cooling the nerve d) may be 200-250 ms in the fastest conducting mammalian fibres e) is decreased by disease which attack the myelin sheath

727

24. An excitatory post-synaptic potential (EPSP): a) is itself propagated by the post-synaptic cell b) comprises depolarisation of the membrane to zero, transient reversal of potential and then repolarisation c) is related in amplitude to the concentration of the initiating transmitter at the presynaptic membrane d) may summate both temporally and spatially with other EPSPs e) is reduced in amplitude by drugs which block the post-synaptic receptors for the transmitter substance 25. C fibres transmitting pain sensation: a) are present in lower numbers than Ad fibres in sensory nerves b) conduct at an average velocity of 2 metres/second c) conduct temperature sensation d) terminate in laminae 2 and 3 of the dorsal horn e) are the sole nociceptive afferents from viscera in the chest and abdomen 26. Concerning CNS physiology: a) the dominant cerebral hemisphere is most commonly the left b) REM sleep occurs for about 25% of the time during normal sleep in adults c) the dominant cerebral hemisphere is always on the right in left-handed individuals d) REM sleep is associated with a marked increase in muscle tone e) Neuromuscular block reduces the amplitude of somatosensory evoked potential 27. In the myocardium: a) a rise in extracellular K+ brings resting the membrane potential towards threshold b) a rise in extracellular K+ increases the membrane permeability to K+ c) hypokalaemia results in reduced ventricular action potential duration d) the slope of depolarisation (dp/dt) affects the contractility of ventricular muscle fibres e) a rise in extracellular Na+ increases membrane resting potential 28. Fibrinogen depletion: a) is diagnostic of disseminated intravascular coagulation b) occurs as a consequence of increased fibrinolysis c) should be treated by administration of cryoprecipitate d) may be a consequence of vitamin K deficiency e) occurs more commonly in men than in women 29. Insulin: a) is secreted by the alpha cells of the pancreas b) is anabolic c) has a circulating half-life of 5 minutes d) shows increased secretion during surgery e) can be used for rapid control of hyperkalaemia

728

30. Concerning absorption of carbon dioxide in breathing systems: a) soda lime granules are sized 4-8 mesh b) baralyme contains calcium hydroxide in addition to barium hydroxide c) soda lime produces more compound A during low fresh gas flow d) dry soda lime absorbs more carbon dioxide e) carbon dioxide first reacts with sodium and potassium hydroxide of soda lime ANSWERS 1.TTTFT 2.TTTFF 3.TTFTF 4.TTTTT 5.TTTTT 6.TTTTT 7.TTTTT 8.TFTTT 9.FTFTT 10.FFTFF 11.FTTTT 12.TTTTT 13.FTTTT 14.TFTT 15.TTTTF 16.TTTFT 17.TTTTT 18.FTTT 19.FFFTT 20.TTFTF 21.FTTTF 22.TFTFF 23.FTTFT 24.FTTTT 25.FFTTF 26.TTFFF 27.TTTTF 28.FTTFF 29.FTTFT 30.TTTFT

General Primary MCQ 5 1. Regarding central venous pressure monitoring: a) the tip of the catheter must be in the right atrium

729

b) cannon a waves are seen in presence of junctional rhythm c) y descent is due to opening of the tricuspid valve d) x descent occurs during ventricular systole e) a wave corresponds with QRS complex in ECG 2. Adrenaline: a) should be administered every 3 minutes during CPR b) is principally released in postsynaptic sympathetic nerve endings c) differs from noradrenaline by having an extra methyl group d) is commonly used with local anaesthetics at a concentration of 1:80,000 e) d-adrenaline is 50-100 times less potent than l-adrenaline 3. Low molecular weight heparins: a) have a molecular weight <10,000 daltons b) inhibit activated factors IX, X, XI, XII c) include tinzaparin, dalteparin and enoxaparin d) are not effective in deep-vein thrombosis prophylaxis in once-only regimen e) do not bind to thrombin and antithrombin III simultaneously 4. The following drugs have antiplatelet activity: a) Epoprostenol b) Calcium heparin c) Hydroxyethyl starch d) dipyridamole e) Remifentanil 5. Compared with that of a normal adult, the newborn infants: a) kidneys have less ability to excrete a concentrated urine b) blood-brain barrier is less permeable to bilirubin c) heat regulation is more efficient because of its ability to metabolise brown fat d) blood has a greater affinity for oxygen at low oxygen pressures e) carbohydrate reserve is greater 6. Concerning acid-base balance: a) standard bicarbonate is lower than actual bicarbonate in a chronic obstructive airways disease (COAD) patient b) metabolic alkalosis is seen with prolonged use of loop diuretics c) standard bicarbonate is low in metabolic acidosis d) about 70% of carbon dioxide is transported in plasma as bicarbonate e) the ratio of HCO3/CO2 may be normal in a stable COAD patient 7. The following statements are true regarding cardiovascular physiology: a) atrial systole contributes to 25% of ventricular filling b) pericardial effusion decreases end-diastolic volume

730

c) coronary blood flow is about 5% of the cardiac output at rest d) the pressure-volume loop consists of isovolumetric contraction, ejection, isovolumetric relaxation and rapid ventricular filling e) LVdP/dTmax is dependent on changes in preload 8. In the human heart: a) end-systolic left ventricular volume is dependent on the afterload b) left ventricular static compliance is reduced in myocardial ischaemia c) ejection fraction is the ratio of stroke volume to end-diastolic volume d) the second heart sound coincides with end of the T wave in the ECG e) pulmonary artery occlusion pressure reflects the left ventricular end-diastolic pressure 9. When measuring arterial blood pressure with a sphygmomanometer: a) cuff width should be 20% greater than the arm diameter b) a mercury column manometer may be used at an angle of up to 25 degrees from vertical c) a common source of error is blockage of the air vent d) there are five Korotkoff phases e) use of a finger to detect return of pulsation is highly inaccurate 10. In the human thyroid gland: a) iodide ions from the plasma enter the follicle cells by passive diffusion b) T4 and T3 bind to the receptors in nuclei c) thyroxine, once synthesised, is then coupled to thyroglobulin until released d) a greater proportion of tri-iodothyronine is formed when iodine is deficient e) thyroid hormones increase the number and affinity of beta-receptors in the myocardium 11. Regarding aldosterone: a) it increases the amount of Na+-K+ ATPase in the target cells b) it reduces the sodium content of the sweat c) it increases the acidity of the urine d) it increases the potassium content of the urine e) basal secretion is normal even after hypophysectomy 12. Regarding insulin: a) its release is increased by the sulphonylurea type of hypoglycaemic drugs b) it has a half-life of 5 minutes in the circulation c) it increases the number of glucose transporters in the cells d) secretion is reduced by surgical stress response e) it is normally secreted at a rate of about 40-50 U/day 13. The following statements about end-tidal CO2 are true: a) normal variation in barometric pressure at sea level is unlikely to influence end-tidal CO2 b) rebreathing can raise end-tidal CO2 if the minute ventilation remains unchanged c) a decrease in cardiac output decreases the end-tidal CO2

731

d) discontinuation of N2O at the end of anaesthesia may transiently reduce the end-tidal CO2 e) shunt has little impact on end-tidal CO2 in contrast to O2 14. A high level of calcium in the blood: a) is associated with an increased excitability of muscle and nerve cells b) may result from chronic renal failure c) may result from excessive vitamin D ingestion d) is seen after accidental hyperparathyroidectomy e) is seen following injection of gland extracts from the anterior pituitary 15. Cardiac muscle contraction: a) develops greater tension when initial fibre length is reduced b) occurs with a greater velocity of shortening when the load on the muscle is small c) develops variable force depending on the frequency of contractions d) reaches its peak at the end of the repolarisation phase of the action potential e) occurs in a graded manner due to fibre recruitment 16. PaCO2-EtCO2 gradient: a) is up to 0.7 kPa in patients without significant disease b) increases in venous air embolism c) is greater in high frequency ventilation d) is greater in high V/Q areas of the lungs e) is greater in patients with rapid respiratory rates 17. In a normal resting man, a bradycardia would be expected to occur following: a) increased carotid sinus pressure b) increased right atrial pressure c) application of pressure to the eyeball d) the release of a Valsalva manoeuvre e) inspiration 18. Which of the following are true: a) the H+ electrode relies for its action on the hydrogen ion sensitivity of a glass electrode b) A CO2 electrode is, in principle, a modified H electrode c) The H+ electrode requires no temperature compensation d) CO2 may be measured by an infrared absorption spectrometer e) N2O may be measured by an infrared absorption spectrometer 19. The capnograph trace: a) plots CO2 on the X-axis versus time on the Y-axis b) elevated phase I is due to rebreathing of CO2 c) a flattened upstroke of phase II represents slow exhalation secondary to obstruction d) phase III shows a flat curve with a slight upstroke e) undulations in phase III may be due to cardiac oscillations

732

20. In the adult human: a) albumin is synthesised only in the liver b) elevated serum lactate dehydrogenase is a good indicator of liver damage c) plasma urea concentration may be low in hepatic failure d) plasma fibrinogen is synthesised in the liver e) the pressure in portal vein is normally about 2 kPa 21. Regarding suxamethonium myalgia: a) it is more common in women b) it commonly occurs in day case patients c) a rise in plasma creatinine phosphokinase and myoglobin is related to the severity of muscle pain d) the severity of muscle pain is related to strength of muscle fasciculations e) intravenous lidocaine before suxamethonium may completely abolish the muscle pain 22. Regarding the renin-angiotensin-aldosterone mechanism: a) renin is secreted by the juxtaglomerular apparatus b) the physiological role of aldosterone is to maintain plasma volume, plasma sodium and potassium c) angiotensin II maintains the tone of the efferent arterioles of the glomeruli d) angiotensin II is the primary factor determining plasma aldosterone levels e) beta- blockers inhibit renin release 23. According to European Guidelines for Basic Life Support: a) when the pulse is absent, the first thing to do is to go and telephone for help b) the recovery position should be adopted when patient is unconscious and breathing through a clear airway c) ventricular fibrillation is the commonest primary cause of cardiac arrest d) early defibrillation is the most important determinant of success in ventricular fibrillation e) the peripheral venous route is preferred because CPR can continue and cannulation is rapid 24. Adenosine: a) is composed of adenine and d-ribose b) causes depression of cardiac contractility by binding to adenosine A1 receptors c) mediates vasodilatation by binding to low affinity adenosine A2 receptors d) is used in diagnosis of supraventricular arrhythmias e) blocks re-entry circuits for its antiarrhythmic effects 25. Concerning the renal buffering mechanism: a) H+ excreted into the tubular fluid reacts with HCO3 in the tubular fluid b) H+ excreted into tubular fluid combines with HPO4 when HCO3 is consumed c) H+ excreted into the tubular fluid reacts with NH3 when HPO4 is consumed d) the phosphate and ammonia buffers are more utilised than HCO3 in metabolic acidosis

733

e) an increased amount of HCO3 is added to blood in respiratory acidosis 26. Total body oxygen consumption can be measured by the following methods: a) oxygen loss from a closed breathing system b) subtraction of expired from inspired volume of oxygen c) ventilated hood technique d) multiplying cardiac output by arterial-mixed venous oxygen content difference e) tissue PO2 electrodes 27. Pulmonary surfactant: a) is produced by type II pneumocytes b) is a mixture of phospholipids and proteins c) prevents transudation of fluid from the blood into the alveoli d) lies in a monomolecular layer e) lowers surface tension to a large extent in a small alveolus 28. Regarding mivacurium: a) it releases more histamine than atracurium in equipotent doses b) it is broken down by plasma cholinesterase as rapidly as suxamethonium c) it is a bisquaternary benzylisoquinolinium compound d) 10% of a bolus dose is excreted in the urine over 24 hours e) at an equipotent dose it has a similar onset of action to pancuronium 29. Pulmonary vascular resistance: a) is normally 150-250 dynes.sec/cm5 b) when raised decreases the left ventricular pre-load c) is increased at an arterial PO2 of 6 kPa d) is increased by application of positive end-expiratory pressure e) is decreased by nitric oxide inhalation ANSWERS 1.FTTTT 2.TFTTT (d is true Peck & Williams and Calvey & Williams quote this dose) 3.TTTFT 4.TTTTF 5.TFFTF 6.TTTTT 7.TTTTT 8.TTTTT 9.TFTTF 10 FTTFT 11.TTTTT 12.TTTTT 13.TTTTT

734

14.FFTFT 15.FFTFF 16.TTTTT 17.TTTTT 18.TTFTT 19.FTTTT (A: is false, time is on the x-axis) 20.TFTTT 21.TTFFF 22.TTTTT 23.TTTTT 24.TTTTT 25.TTTTT 26.TTTTF 27.TTTTT 28.TFTTT 29.FTTTT (A: is false, range quoted is 25-125 dyn.sec/cm5 in A-Z, Fundamentals and Oxford Handbook of critical care). General Primary MCQ 6 1. Concerning the spectrum of electromagnetic radiation: a) the visible spectrum spreads from 390 nm to 750 nm b) X-rays and gamma rays are ionising radiations c) the CO2 laser gives infrared radiation d) ultraviolet rays are non-ionising e) wavelength is inversely related to frequency 2. Halothane: a) has a marked arrhythmogenic potential compared with other currently used volatiles b) is a racemic mixture of optical isomers c) has a minimal alveolar concentration of 0.29 in 70% of nitrous oxide d) is metabolised to the greatest extent amongst currently available agents e) sensitises the myocardium to endogenous or exogenous catecholamines 3. For intravenous anaesthetic agents: a) the end point of induction of anaesthesia is loss of the eyelash reflex b) the clearance is their rate of excretion c) lower doses will be required for induction in hypovolaemic patients d) the effects on the CNS. depend on their degree of ionisation e) their non-protein bound fraction increase in liver and renal diseases

4. Enflurane: a) is less potent than isoflurane

735

b) causes convulsive EEG pattern when used at high concentration in the presence of hypocapnia c) decreases intracranial pressure d) impairs the autoregulation e) does not affect cerebral blood flow 5. Ketamine: a) has marked analgesic properties mediated by its binding to NMDA receptors b) causes a fall in cardiac output and a rise in heart rate c) is contraindicated in patients with raised intracerebral pressure d) has active metabolites e) causes postoperative dreaming and hallucinations which are less frequent in children 6. With regard to acids and bases: a) acids can be considered as H+ ion donors b) water can act as both an acid and a base c) pH is log 10 [H+] d) physiological systems control Na+ concentration within more closely defined limits than H+ concentration e) lactic acidosis is a normal `anion-gap acidosis 7. The following characteristics of a drug intended to be given by infusion are desirable: a) short elimination half-life b) high rate of plasma clearance c) low cost d) low aqueous solubility e) administration via a central vein 8. The effects of acidosis may include: a) increased minute ventilation b) shift of the oxygen dissociation curve curve due to the Bohr effect c) negative inotropism due to higher calcium levels d) hyperkalaemia e) increased NH4 excretion 9. Concerning buffers: a) buffers are least efficient when half dissociated b) carbonic anhydrase is an important component of the physiological buffering system c) phosphate is an important extracellular buffer d) oxygenated haemoglobin is a more powerful physiological buffer than deoxygenated haemoglobin e) the imidazole ring structure of haemoglobin is the site of H+ exchange for physiological buffering 10. A competitive antagonist:

736

a) combines with the same receptor as the agonist drug b) shifts the log dose-response curve for the agonist to the right c) depresses the maximum obtainable response of the agonist d) may affect the affinity of the agonist for its receptors e) need not have a close structural similarity to the agonist 11. Concerning body fluid compartments: a) water constitutes 70% of the total body weight b) plasma constitutes a quarter of the extracellular fluid (ECF) volume c) sucrose can be used to measure the ECF volume d) the interstitial fluid volume for a 70 kg man is approximately 9 litres e) The ECF/intracellular fluid volume ratio is smaller in infants and children than it is in adults 12. When the ventilation/perfusion ratio of a lung unit increases: a) the alveolar PO2 rises b) the alveolar CO2 rises c) end capillary PO2 increases d) arterial PO2 increases e) hypoxic pulmonary vasoconstriction will compensate for any change in gas exchange 13. In the type I immune reaction in man: a) the antibody is always IgE b) an antigen-antibody reaction occurs on the surface of mast cells c) most of the clinical manifestations are due to the release of kinins from mast cells d) anaphylaxis occurs only in atopic individuals e) bronchoconstriction may follow inhalation of reagin 14. When a patient is using a monoamine oxidase inhibitor, the following drugs should be avoided: a) halothane b) pethidine c) adrenaline d) dopexamine e) phenylephrine 15. The following statements about diuretics are correct: a) amiloride causes hyperuricaemia b) thiazides improve carbohydrate tolerance c) loop diuretics reduce renal calcium loss d) loop diuretics potentiate nephrotoxicity of cephalosporins e) thiazides may prevent renal colic in patients with idiopathic hypercalcaemia

737

16. Gastric acid secretion is increased by: a) anticholinergic drugs b) histamine c) vagal stimulation d) gastrin e) sucralfate 17. Renal clearance of drugs can: a) involve zero order kinetics b) involve active secretion c) occur by filtration d) be affected by lipid solubility e) depend on protein binding 18. Peripheral nerve stimulators: a) should have an ability to generate 60-70 mA of current b) should be capable of delivering an impulse longer than 0.5 ms c) should be able to deliver monophasic and rectangular wave impulse d) should be able to deliver supramaximal stimuli e) should be used on the healthy side in hemiplegic patients 19. The following are impermeable to water: a) ascending vasa recta b) descending vasa recta c) ascending loop of Henle d) descending loop of Henle e) proximal convoluted tubule 20. A highly ionised drug: a) is well absorbed from the intestine b) is excreted mainly in the kidney c) crosses the placental barrier easily d) is reabsorbed from the renal tubule e) is highly protein bound 21. A more rapid induction of anaesthesia will occur using an inhalational agent if: a) the agent is highly soluble in blood b) the patient has a low cardiac output c) the patient is breathing 5% carbon dioxide d) the agent has a low blood/gas solubility coefficient e) a low flow circle system is used 22. Examples of active transport include:

738

a) hydrogen ion secretion from parietal cells b) endocrine gland secretion of hormones c) Na+ and K+ movement across nerve membranes d) glucose reabsorption in the proximal tubule e) voltage dependent Ca++ entry into myocardial cells 23. The following affect protein binding: a) pKa b) pH c) molecular weight d) binding site availability e) lipid solubility 24. In first order kinetics: a) the rate of excretion is proportional to the concentration of the drug present in the body at any time b) the plasma level decay is exponential c) excretion must be purely renal d) excretion must not be affected by administration of another drug e) 95% of the drug is eliminated in about three time constants 25. The following drugs have greater than 50% bioavailability after oral administration: a) warfarin b) propranolol c) atenolol d) lidocaine e) morphine 26. Lower oesophageal sphincter pressure is decreased by: a) opiates b) metoclopramide c) atropine d) gastrin e) dopamine 27. Ingested lipids: a) are mainly triglycerides b) cannot be used to produce energy for cell metabolism c) are broken down in the duodenum d) are transported as chylomicrons from the gut to the liver 28. Lidocaine: a) has a pKa of 7.7 at 37 degrees C b) is more potent than ropivacaine

739

c) is less effective orally as an antidysrhythmic because of its high first-pass metabolism d) is an acid e) can cause epileptiform fits 29. The factors affecting the rate of transport of a drug across the placenta are: a) the degree of ionisation b) the degree of protein binding c) placental blood flow d) the pK of the drug e) the lipid solubility of the drug

ANSWERS 1.TTTTT 2.TTTTT 3.FFTTT 4.TTFTF 5.TFTTT 6.TTFFF d)[H+] 36-46 nm/L [Na+] 135-145 mmol/l. Body concerned more with [H+] than [Na+] 7.TTTFF 8.TTFTT 9.FTFFT 10.TTFFT 11.FTTTF 12.TFTTF d)See West pg 53 13.TTFFT 14.FTTTT 15.FTFTF 16.FTTTF 17.FTTTT 18.TFTTT 19.FFTFF 20.TFFFF 21.FFTTF 22.TFFTF e) This is voltage mediated and no energy per se is needed the ions flow because the membrane is depolarised. 23.FTFTT d) competition for binding sites is competitive and can alter unbound fraction of each (Peck+Williams) 24.TTFTT 25.TFFFF c) 45% bioavailability in Calvey+Williams, QBASE, Peck+W BUT 50% in Fundamentals of Anaesthesia e) only 25% reaches systemic circulation 26.TFTFT 27.TFFT 28.TFTFT 29.TTTTT e) all the above are true and are listed in Fundamentals pg 533 (old edition)

740

General Primary MCQ 7 1. The following result in an increase in physiological dead space: a) positive end-expiratory pressure b) venous air embolism c) hypotension d) pregnancy e) obesity 2. Concerning gastric secretion in a normal adult: a) the lowest pH obtainable in the stomach is about 4.5 b) histamine receptors in the stomach can be pharmacologically stimulated without affecting those in the lungs c) pentagastrin injection can cause a maximal secretion of acid in the stomach d) excessive secretion is prevented by an effect originating in the antral receptors e) gastrin comes mainly from cells in the fundus of the stomach 3. The resistance of the airways: a) is not constant in a given subject b) depends on the pressure drop across the airways c) is expressed as cm H2O/L/s d) is greater in turbulent flow than in laminar flow e) is directly proportional to the density of the gases 4. In a normal man breathing quietly at rest: a) carbon dioxide in alveolar air is about twice that in room air b) carbon dioxide in mixed venous blood is greater than in alveolar air c) water vapour in alveolar air is less than half that of alveolar carbon dioxide d) oxygen in expired air is greater than that in alveolar air e) the amount of nitrogen expired is equal to that inspired 5. A shift of the oxygen dissociation curve to the right: a) occurs in the pulmonary capillaries b) is favoured by a rise in temperature c) favours the passage of oxygen from blood to tissues d) occurs when fetal blood is replaced by adult blood e) occurs in stored blood 6. Concerning glomerular filtration: a) glomerular filtration is a type of ultrafiltration b) glomerular filtrate has the same composition as lymph

741

c) blood in the efferent arterioles is more viscous than that in afferent arterioles d) glomerular filtration rate (GFR) is directly proportional to systemic arterial pressure e) tubular function is more important than GFR in determining the rate of urine production 7. Concerning local anaesthetics: a) they are absorbed more rapidly after intercostal block than after caudal administration b) in the foetus they are able to cross the placenta as readily as from the mother c) they are weak acids d) those which are esters are rapidly metabolised by liver enzymes e) pKa is the pH at which more than half of a local anaesthetic exists in non-ionised form 8. With reference to the mechanical events in the cardiac cycle in a normal adult human: a) the left ventricle ejects more blood per beat than the right ventricle b) the mitral valve opens when the left atrial pressure exceeds the left ventricular pressure c) during strenuous work, the left ventricular end-diastolic volume may be double that at rest d) the pulmonary valve opens when the right ventricular pressure reaches 20-25 mmHg e) during diastole, the left ventricular pressure is about 70 mmHg 9. Regarding peri-arrest administration of drugs: a) during ventricular fibrillation, IV access should be established before DC shock is administered b) the peripheral venous route is prefered because CPR can continue and cannulation is rapid c) central venous administration delivers drugs more rapidly than peripheral with a 20 ml push d) in adults, doses of drug for endotracheal administration should be 5 times the IV dose e) in children, intraosseous drugs should be given when the IV route cannot be established within 90 seconds 10. With reference to adjustments in exercise: a) an increase in muscle blood flow begins after the first half minute of exercise b) cerebral blood flow rises if the exercise causes systolic arterial blood pressure to rise c) body temperature may rise measurably d) lymph flow from the exercising muscle increases e) visceral blood flow decreases 11. During intra-uterine life: a) all the foetal blood returning from the placenta flows directly into the inferior vena cava b) foetal blood carries more oxygen than maternal blood at a low PO2 c) umbilical venous blood has a PO2 of 4-5 kPa d) blood on the right side of the foetal heart is slightly better oxygenated than that on the left side e) pulmonary vascular resistance is higher than after birth 12. The rotameters:

742

a) are variable orifice flowmeters b) produce a variable pressure drop across the bobbin c) produce laminar flow at low flow rates d) produce laminar flow at high flow rates e) are accurate to within +/- 0.5% 13. Cardiac output obtained by the thermodilution technique can be accurate: a) in the presence of right-sided valvular diseases b) in the presence of left to right intracardiac shunt c) in dysrhythmias d) in the presence of rapid volume infusions e) if the tip of pulmonary artery catheter is completely wedged 14. If about 500 ml of isotonic saline were to be infused into a healthy adult, the consequences would include: a) increase in cardiac stroke volume b) increase in flow of lymph from peripheral tissues c) increase in renin secretion by the kidney d) increase in cerebral blood flow e) equal distribution of the excess volume between intracellular and extracellular compartments 15. Regarding carbon dioxide: a) a rise in PaCO2 may not increase cerebral blood flow b) PaCO2 greater than 13 kPa could depress the respiratory centre c) hypercarbia will increase pulmonary vascular resistance d) chronic hypercarbia results in renal retention of bicarbonate e) bicarbonate diffuses more quickly than CO2 across the blood-brain barrier 16. Myocardial contractility: a) is the degree of inotropic state of heart independent of preload, afterload or heart rate b) determines the rate of development of ventricular pressure (dp/dt) c) can be estimated by ventricular pressure-volume loops d) is reduced by hypocalcaemia e) accounts for approximately 90% of total myocardial oxygen consumption 17. The features of non-depolarising blockade include: a) fasciculation b) sustained response to tetanic stimulation c) presence of post-tetanic potentiation d) small rapidly moving muscles are blocked first e) more than 90% of the receptors must be blocked by the relaxant for optimal relaxation

743

18. Non-depolarising relaxants: a) can cross the blood-brain barrier b) may affect the smooth muscles c) are needed at values of 2-3 times their ED95 for endotracheal intubation d) can cross tbe placenta e) are highly ionised at physiological pH 19. Concerning the chemoreceptors involved in the control of breathing: a) central chemoreceptors are located on the ventral surface of the medulla b) raised cerebrospinal fluid hydrogen ion concentration can stimulate central chemoreceptors c) increased ventilation due to raised arterial PaCO2 is mediated through both central and peripheral chemoreceptors d) denervation of carotid and aortic bodies can abolish the ventilatory response to hypoxia e) peripheral chemoreceptors mediate the ventilatory response to acidaemia 20. Which of the following are competitive antagonists: a) morphine and naltrexone b) histamine and perphenazine c) phenobarbitone and propofol d) acetyl choline and cisatracurium e) aspartate and ketamine 21. Midazolam: a) is an anticonvulsant b) is lipid soluble at physiological pH c) has no active metabolites d) has an elimination half-life of 2-4 hours e) can be administered as nasal drops for premedication 22. The following drugs cross the placenta: a) midazolam b) remifentnil c) propofol d) sevoflurane e) ropivacaine 23. A loading or priming dose: a) is smaller than the maintenance dose b) is equal to the amount of drug which is eliminated in a dose interval c) is unnecessary for benzylpenicillin (t 1/2 30 min) d) is necessary if a rapid onset of digoxin (t 1/2 36-48 hours) is required e) is given with the objective of quickly achieving the desired plasma concentration

744

24. Bioavailability: a) is indicated by the area under the plasma concentration-time curve b) of a drug which is injected intravenously must be less than 100% c) may be reduced by destruction of drug in the gut d) may be reduced by metabolism of drug in the liver e) is greater by sublingual route than enteral route 25. Regarding interactions between drugs: a) with steep dose-response curves they are unlikely to be harmful b) with small therapeutic ratios they are unlikely to be harmful c) they are described as summation if the effects of two drugs with the same action are additive d) they are described as potentiation if the action of one drug increases the effect of another e) the effect of two drugs with similar action is greater in synergism than would have been expected from summation 26. Alteration of urine pH: a) with probenecid reduces elimination of penicillin and can be therapeutically useful b) can significantly affect the elimination of drugs which do not possess ionisable chemical groups c) is valuable in overdose with barbiturates d) with alkali is useful in detecting amphetamine addicts since it increases urinary excretion of the drug e) is of major importance in the management of pethidine overdose 27. Elderly patients show increased response to standard drug dosage and an increased incidence of adverse drug reactions because they have: a) increased lean body mass b) reduced renal and hepatic function c) reduced blood flow to vital organs d) better nutrition e) less efficient homeostatic mechanisms 28. Regarding diffusion: a) the rate of diffusion of a substance across a membrane is proportional to its concentration gradient b) the diffusion across the alveolo-capillary membrane is measured using carbon monoxide c) the rate of the diffusion of a gas is proportional to its tension gradient d) pneumonectomy approximately halves the diffusing capacity e) the rate of diffusion of a gas is inversely proportional to the square root of its molecular weight

745

29. The following are idiosyncratic reactions: a) haemolysis after exposure to NSAIDs in patients with glucose-6-phosphate deficiency b) prolonged apnoea after suxamethonium due to abnormal pseudocholinesterase c) malignant hyperpyrexia after halothane d) acute porphyria following induction with thiopentone e) sulphonamide toxicity in slow acetylators 30. Phenytoin: a) plasma half-life is the same at all plasma concentrations b) is subject only to first-order kinetics c) enhances its own metabolism d) is unlikely to cause drug interactions in a patient taking other medications e) has a remarkably small range of adverse effects ANSWERS 1.TTTTF d) Increased by up to 45% (Fundamentals of Anaesthesia) 2.FTTTF 3.TTTTT 4.FTFTF 5.FTTTF 6.TFTFT 7.TFFFF 8.FTFFF 9.FTTFT 10.FFTTT 11.FTTTT 12.TFTFF 13.FFFFF 14.TTFFF 15.FTTTF 16.TTTTT a) by definition, contractility is for a GIVEN preload and afterload, i.e. it must affect it. Ganong states changes in rate or rhythm can also affect the myocardial contractility 17.FFTTT 18.TFTTT a) and d) a small amount can cross the bbb/placenta 19.TTTTT 20.TTFTT 21.TTFTT 22.TTTTT 23.FFTTT 24.TFTTT 25.FFTTT 26.FFTFF 27.FTTFF 28.TTTTT

746

29.TTTTT 30.FFTFF General Primary MCQ 8 1. Angiotensin II has a role in maintenance of effective circulatory volume by the following mechanisms: a) increasing thirst b) increased antidiuretic hormone release from posterior pituitary c) systemic vasoconstriction d) increased aldosterone release e) increased proximal tubular sodium reabsorption 2. Adenosine triphosphate: a) is synthesised when the skeletal muscle contracts b) contains two energy rich phosphate bonds c) is an integral part of the flavoprotein-cytochrome system d) is only produced during the aerobic, not anaerobic, catabolism of glucose e) is hydrolysed enzymically during the operation of the sodium pump 3. The blood-brain barrier: a) anatomically, is at the arachnoid villi b) is less permeable in the neonate c) results in the total exclusion of many certain from the brain d) results in very low catecholamine levels in the brain and spinal cord e) is functionally similar to a cell membrane 4. Sodium reabsorption in the nephron is: a) greater in the distal than in the proximal convoluted tubule b) only achieved in exchange for potassium excretion c) the only major energy consuming activity of the kidney d) the main object of the countercurrent multiplier system e) dependent on the glomerular filtration rate as well as aldosterone 5. The carotid body chemoreceptors: a) are stimulated by a fall in arterial oxygen tension b) are inhibited by a fall in arterial pH c) produce reflex peripheral vasoconstriction d) are responsible for increased ventilation in a patient with carbon monoxide poisoning e) have a very high tissue blood flow 6. Concerning pain: a) transmission occurs in the lateral spinothalamic tracts

747

b) it may be modulated at a spinal level by endorphinergic interneurones c) it is modified at a spinal level by descending fibres from the periaqueductal grey matter of the mid-brain d) fibres from the frontal cortex inhibit thalamic interpretation of pain e) surgery of the cerebral cortex is not associated with pain 7. Gas chromatography: a) depends on the partition of a substance between two phases, one stationary and one moving b) the stationary phase is an inert solid material that packs the column c) a specific detector is needed at the end of the column d) a particular gas can be identified by its specific profile e) the amount of a particular gas is calibrated against accurately known samples 8. Concerning electrical safety: a) when touching faulty apparatus with wet hands, skin impedance is increased and current flow greater than normal b) electrical shock is not a risk with potential below 24 V AC or 50 V DC c) the modern diathermy earthing plate is not connected directly with earth d) microshock may lead to ventricular fibrillation via an intracardiac catheter with currents in excess of 150 microamps e) leakage currents are induced voltages in other circuits resulting from an alternating mains current 9. Concerning flow: a) if Reynolds number exceeds 1000, turbulent flow is likely to be present b) the critical velocity at which laminar flow changes to turbulent flow varies only with the gas involved c) helium reduces the density of inspired gases and therefore the likelihood of turbulent flow within the respiratory system d) turbulent flow within blood vessels is only detectable by invasive techniques e) endotracheal tube adaptors (e.g. Magill) are specifically designed to avoid turbulent flow 10. The effects of opioids at a desired CNS receptor site depend on: a) route of administration b) volume of distribution c) ionisation and protein binding d) cerebral blood flow e) permeability of blood-brain barrier 11. Concerning the opioids used in the perioperative period: a) the volume of distribution of fentanyl is greater than that of morphine b) at physiological pH morphine is more ionised than alfentanil c) morphine is more protein bound than fentanyl d) fentanyl penetrates the blood-brain barrier more quickly than morphine e) pethidine has a metabolite with epileptogenic potency

748

12. Tachyphylaxis occurs during treatment with: a) ephedrine b) trimetaphan c) suxamethonium d) noradrenaline e) sodium nitroprusside 13. A plot of pressure against volume: a) allows compliance to be measured b) may show hysteresis c) allows a direct measurement of airway resistance d) is usually plotted on a semi-logarithmic paper e) allows an estimate to be made of respiratory work 14. Concerning protamine: a) it is a basic protein b) 1 mg antagonises 100 mg heparin c) it is a myocardial stimulant d) it is contraindicated in hepatic failure e) it is 60% protein bound 15. Vasoconstrictor agents with little or no positive inotropic effect include: a) adrenaline b) methoxamine c) isoprenaline d) dobutamine e) metaraminol 16. Opioid-induced bradycardia: a) is not seen with pethidine b) is due to stimulation of central vagal nuclei c) is not seen in patients undergone bilateral vagotomy d) is due to a direct effect of some of the opioids on the SA node e) is more commonly seen when used with vecuronium in the intraoperative period 17. Causes of the anticoagulant effect of a massive blood transfusion includes: a) deficient factor V and VIII b) inactive platelets c) microaggregates d) cold e) vitamin K availability

749

18. the oxyhaemogloin dissociation curve is: a) a curve relating quantity of oxygen combining with haemoglobin to the partial pressure of oxygen in the gas with which the blood is equilibrated b) the dissociation constant of oxygen c) a reflection of the efficiency of oxygen transport d) the same shape and position for haemoglobin A and myoglobin e) only of sigmoid shape in vivo. 19. The weight of gas dissolved in a liquid at constant ambient pressure at equilibrium depends on: a) temperature of the liquid b) partial pressure of the gas c) the diffusion coefficient d) solubility of the gas in the liquid e) critical temperature of the gas 20. Concerning opioid-induced muscle rigidity: a) it is commonly seen with very high doses of opioids b) it is more commonly seen with concomitant use of nitrous oxide c) it is often seen in young patients d) it can be reversed by naloxone e) it is seen only in chest wall muscles 21. Concerning electromagnetic radiation: a) it includes visible light b) infrared radiation only occurs from objects that are hotter than the environment c) Stefans law defines the heat radiated from a black body and includes a term of the fourth power of the absolute temperature of the body d) obeys the inverse square law e) includes ultrasound 22. Concerning the intracellular fluid: a) it is approximately twice the volume of the extracellular fluid b) the volume can be measured directly c) it is freely interchangeable with other body fluid compartments d) it contains approximately 150 mEq/L potassium e) it is a fairly constant fraction of all tissues 23. Concerning hypersensitivity: a) type I immediate hypersensitivity involves IgE b) antigen excess in type III produces the Arthus reaction c) delayed hypersensitivity is cell mediated d) type II hypersensitivity binds complement e) cell mediated hypersensitivity involves complement

750

24. The following are side-effects of suxamethonium: a) anaphylaxis b) masseter muscle rigidity c) raised intraocular pressure d) raised intragastric pressure e) myoglobinuria 25. These are true in the SI system of measurement: a) the basic unit of mass is the gram b) pico is the prefix denoting 10-12 c) the Hertz is the derived unit of frequency d) it is still allowable to use temperature on the Celsius scale e) the unit of time in the metric and SI unit system are the same 26. With increasing altitude: a) the concentration of oxygen in the atmosphere falls b) the atmospheric pressure decreases exponentially c) water will boil at a temperature of less than 100 degrees C d) PaCO2 will decrease at first e) acetazolamide can reduce the incidence of pulmonary oedema 27. The physiological dead space: a) increases with induction of anaesthesia b) decreases during controlled ventilation c) increases with a short inspiration time d) increases with the use of positive end-expiratory pressure 28. Warfarin: a) competes with vitamin K b) displaces phenylbutazone from plasma protein binding sites c) blocks prothrombin synthesis d) is contraindicated in malignant hypertension e) prevents normal fibrinolysis 29. In man, atropine causes: a) secretion of antidiuretic hormone b) an initial bradycardia c) relaxation of uterine muscle d) relaxation of ureteric muscle e) mydriasis

751

30. Concerning metabolism: a) skeletal muscles at rest utilise free fatty acids (FFA) as the energy source b) the mechanical efficiency of skeletal muscles is greater during isotonic contraction than isometric contraction c) the amount of ATP generated from FFA varies with their size d) th deamination of amino acids results in urea synthesis by the liver e) phosphoryl creatine is an energy store for skeletal muscles for ATP synthesis ANSWERS 1.TTTTT 2.FFFFT 3.FFTFT 4.FFFFT 5.TFTFT 6.TTTTT 7.TFTFT 8.FTTTT 9.FFTFF 10.TTTTT 11.TTFTT 12.TTTTT 13.TTFFT 14.TFFFF 15.FTFFT 16.TTTTT 17.TTTFF 18.TFTFF 19.TTFTF 20.TFFFF 21.TFTTF 22.TFFTF 23.TFTTF 24.TTTTT 25.FTTTT 26.FTTTT 27.TFTT 28.TTTTF 29.FTFTT 30.TTTTT

General Primary MCQ 9 1. Ideal volatile agent should have: a) analgesic properties

752

b) high oil:gas partition coefficient c) low blood:gas partition coefficient d) boiling point less than 20 degrees C e) low molecular weight 2. Regarding intracranial pressure (ICP) monitoring: a) ventricular catheter is the gold standard for monitoring of ICP b) in infants, ICP can be monitored by placing a transducer over anterior fontanelle c) the normal ICP waveform is like a miniature CVP waveform d) cerebral blood flow is zero when mean arterial blood pressure is same as ICP e) ventricular catheter does not allow compliance testing 3. Regarding temperature monitoring: a) infrared thermometers use the frequency of electromagnetic radiation of the emitting object b) thermistors measure temperature-sensitive electrical resisitance c) liquid crystal is used for measurement of skin surface temperature d) skin surface temperature corresponds to core temperature following induction of anaesthesia e) core-peripheral temperature gradient is a measure of tissue perfusion 4. PaCO2 - PEtCO2 difference is smaller in: a) ventilation with large tidal volumes b) third trimester of pregnancy c) exercise d) chronic obstructive airways disease patients ventilated with low respiratory rate compared with ventilation with a high rate e) infants and neonates without known lung disease 5. Regarding control of breathing: a) cerebrospinal fluid has a very poor buffering capacity for a given change in pH compared with blood b) carotid and aortic bodies start firing non-linearly when PaO2 falls below 13 kPa c) stretch receptors in airways are an integral part of Hering-Breuers reflex d) epithelial receptors in the larynx can precipitate laryngospasm e) J receptors are activated by pulmonary capillary engorgement 6. Dopamine: a) does not cross the blood-brain barrier b) causes renal vasodilatation via DA1 receptors c) inhibits prolactin release d) may cause nausea and vomiting via DA2 receptors e) increases atrioventricular conduction

753

7. Glyceryl trinitrate: a) dilates both resistance and capacitance vessels b) is a nitric oxide donor c) may cause cyanide toxicity d) can cause reflex tachycardia e) in combination with adrenaline can be used in low cardiac output conditions 8. Sodium nitroprusside: a) reduces both preload and afterload b) can decompose if the infusion bag is not wrapped by aluminium foil c) molecule has five cyanide groups d) combines with haemoglobin to produce cyanmethaemoglobin e) toxicity can be associated with increase in mixed venous oxygen saturation 9. Concerning allergic reactions: a) released histamine has a half-life of 2.5 minutes b) plasma tryptase level is increased during the reaction c) tryptase is a specific marker of drug-induced allergic reaction d) methylhistamine level is decreased in urine during an allergic reaction e) a skin-prick test to identify the suspected agent brings the sensitised lymphocytes to the skin surface 10. The following statements are correct: a) SI unit of pressure is mmHg b) atmospheric pressure at sea level is about 100 kPa c) a central venous pressure of 7.5 mmHg is equivalent to 10 cm H2O d) laminar flow in a blood vessel has the greatest flow rate at the centre of the blood vessel e) flow is proportional to the square root of the pressure difference in turbulent flow 11. Verapamil: a) prolongs A-V nodal refractoriness b) undergoes extensive first-pass hepatic extraction on oral administration c) is a useful antiarrhythmic agent in cases of recent myocardial infarction d) is contraindicated in Wolff-Parkinson-White syndrome e) is also useful for ventricular arrhythmias 12. The electrolyte and acid-base imbalance seen in end-stage renal failure includes: a) metabolic alkalosis b) a fall in serum magnesium c) a rise in serum phosphate d) a rise in serum calcium e) a fall in serum potassium

754

13. Nitrous oxide: a) is stored as a liquid b) is a gas above a temperature of 36 degrees C c) cylinders should have a filling ratio of 0.65-0.75 d) content in a cylinder is always reflected from its pressure gauge e) cylinder when full has a pressure of about 50 atmospheres 14. Closing volume: a) is larger than functional reserve capacity b) may be determined by single breath N2 curve following a deep breath of oxygen c) is high in young children and decreases progressively with advancing age d) if high, may be responsible for arterial hypoxaemia e) is unaffected by bronchomotor tone 15. Regarding haemodynamic changes seen following the Valsalva manoeuvre: a) the rise in mean arterial blood pressure (MAP) in phase I is due to the rise in intrathoracic pressure b) in phase II, decreased venous return is responsible for the drop in MAP c) MAP increases in phase III due to the rise in heart rate and vasoconstriction d) MAP returns to normal in phase IV e) these changes are present even in autonomic dysfunction 16. Propofol: a) has a pH of 7.0 b) is insoluble in water c) is twice as potent as thiopentone d) can be given as an infusion because its terminal half-life is less than 30 minutes e) may change the colour of urine due to propofol glucuronide 17. Hypophysectomy will result in: a) depressed thyroid function b) osteoporosis and generalised wasting c) the secretion of adrenal glucocorticoid and sex hormones to a low level d) diabetes insipidus e) normal aldosterone secretion 18. Pneumotachograph: a) lumen is broken up to a large number of smaller tubes b) has a sensitive differential pressure transducer across the resistor c) output is affected by the gas viscosity d) may give an erroneous reading in the presence of condensed water vapour e) works on the principle of Poiseuille's Law

755

19. Sympathetic stimulation affects carbohydrate metabolism because: a) adrenaline increases liver glycogenolysis b) sympathetic nerves to the pancreas regulate insulin release c) peripheral tissues require adrenaline to take up glucose d) insulin cannot act on the liver in the absence of adrenaline e) none of the above 20. Regarding work of breathing: a) inspiring a low tidal volume at a higher rate reduces the elastic component of work of breathing b) work of breathing is increased at high inspiratory flow c) the viscous component of work of breathing is increased in the presence of stiff lungs d) low respiratory rate reduces the viscous component of work of breathing e) patients naturally adopt a respiratory pattern which ensures the lowest work of breathing 21. Gastric emptying: a) can be measured by using radioactive substances b) may be measured by repeated X-rays after a barium meal c) can be measured by naso-gastric aspiration of the residues d) is commonly measured by paracetamol uptake e) time is delayed by intravenous erythromycin 22. One or more prostaglandins: a) are peptides found in prostatic secretions b) dilate the bronchial smooth muscles c) dilate the afferent arterioles in the glomeruli d) raise the intracranial pressure e) affect platelet function 23. Regarding endorphins: a) they are short chain peptides b) there is a homogenous population of receptors for them c) they are present in the brain but not the spinal cord d) concentrations are decreased by exercise e) may be antagonised by naloxone 24. Both bradykinin and histamine: a) are direct vasodilators b) increase capillary permeability c) stimulate the secretion of gastric acid d) cause pain when applied to a blister base e) cause vasodilatation in the axon reflex

756

25. Prostaglandins are: a) not naturally occurring substances b) used to induce abortion and labour c) metabolised in the pulmonary circulation d) modulators of histamine and bradykinin action in pain e) involved in protecting the gastric mucosa from hydrochloric acid 26. Normal values in the newborn include: a) blood volume of 110 ml/kg b) arterial blood pressure of 80/60 mmHg c) resting heart rate of 160-180 bpm d) PaO2 of 6 kPa e) HbF of 40% 27. Regarding thyroid hormones: a) they sensitise the myocardium to catecholamines b) they lower the plasma cholesterol c) they increase the formation of messenger RNAs d) reverse T3 may be active in some cells e) iodine is absorbed from the gastrointestinal tract in ionised form 28. Regarding antiarrhythmic drugs: a) amiodarone enhances repolarisation b) flecainide has no effect on repolarisation c) magnesium is not useful in AF and torsades de pointes d) adenosine can differentiate supraventricular from ventricular arrhythmias e) beta-blockers increase K+ permeability and decrease the rate of firing in the SA node 29. Regarding acid-base balance: a) a healthy adult eliminates 16 mmol of CO2 by lungs per day b) haemoglobin accepts more H+ on alpha-imidazole group on cooling c) pH rises when a poikilothermic animal is cooled down d) the interior of the cell may become more acidic following administration of intravenous bicarbonate e) slow administration of bicarbonate may cause intracranial haemorrhage 30. Regarding the passage of glomerular filtrate through the renal tubules: a) the thick ascending limb of the loop of Henle dilutes the filtrate by its powerful sodium pumps b) the osmolality of the filtrate at the tip of loop of Henle is about 1200 mosm/L c) the distal tubule is normally impermeable to water d) glucose is absent in the filtrate in the loop of Henle e) sodium concentration in the descending limb of the loop of Henle is the same as that in the proximal convoluted tubule

757

ANSWERS 1.TTTFF 2.TTFTF 3.TTTTT 4.TTTTT 5.TTTTT 6.TTTTT 7.TTFTT 8.TTTTT 9.TTTFT 10.FTTTT 11.TTFTF 12.FFTFF 13.TTTFT 14.FTFTF 15.TTTTF 16.FTTFT 17.TFTTT 18.TTTTT 19.TFFFF 20.TTFTT 21.TTTTF 22.FTTFT 23.FFFFT 24.TTFTF 25.FTTTT 26.FTTFF 27.TTTFT 28.TTFTT 29.TTTTF 30.TTTTT General Primary MCQ 10 1. Regarding drug administration: a) the sublingual route avoids first-pass inactivation in the liver b) suppositories expose the drug to first-pass metabolism c) distribution of nebulised drug in the respiratory tree depends on the particle size d) highly water soluble drugs are administered by the transdermal route e) drugs given by the oral route should have a very high extraction ratio 2. Regarding protein binding of drugs in the plasma: a) highly protein bound drugs have a longer biological half-life b) protein binding is markedly different in arterial and venous blood

758

c) free drug concentration is not altered in hypoalbuminaemic states d) alpha-1 acid glycoprotein principally binds to basic drugs e) the greater the protein binding, the lesser the volume of distribution 3. Alprostadil (PGE1) and Epoprostenol (PGI2): a) are principally metabolised in the lungs b) dilate ductus arteriosus to increase pulmonary blood flow in neonates c) inhibit platelet aggregation d) improve oxygenation in ARDS patients when used by the nebulised route e) do not cause apnoea in neonates 4. The following statements about drug half-life are correct: a) the half-life of a drug is shorter than its time constant b) drugs given by infusion without a bolus reach a steady state in five half-lives c) the extent of drug distribution into the total body water or extracellular fluid is unlikely to affect t1/2 d) context-sensitive halflife can be used for any drug e) about 94% of a drug is cleared from the body in four half-lives 5. First-order processes: a) apply to enzyme-mediated reactions b) are characterised by high rates of reaction when the concentrations of reacting substances are high, and vice verse c) can properly be described in terms of t1/2 d) are involved in the elimination of most drugs e) change to zero-order kinetic at very high drug doses 6. Regarding drug clearance by the body: a) this only refers to elimination by the kidney b) this refers to the volume of plasma cleared of the drug in unit time c) it cannot exceed the glomerular filtration rate d) it is dependent on its volume of distribution e) hepatic clearance is expressed as extraction ratio 7. 5-HT: a) is synthesised from tyrosine by enterochromaffin cells of the gastrointestinal tract b) is also an endogenous neurotransmitter in the brain c) is metabolised by monoamine oxidase to hydroxyindole acetic acid d) is degraded to a great extent by pulmonary endothelial cells e) can cause vasoconstriction and increased gastrointestinal motility 8. Concerning metabolism of following drugs: a) lidocaine is metabolised to monoethylglycinexylidide (MEGX) in the liver b) lithium is almost entirely excreted unchanged by the kidney

759

c) gentamicin is metabolised in the liver and excreted in the bile d) chlorpromazine is degraded via microsomal oxidative metabolism in the liver e) levodopa crosses the blood-brain barrier and is then converted to dopamine 9. Regarding arterial blood gas analysis: a) hydrogen, oxygen and CO2 electrodes measure at 37 degrees C b) PaO2 read by gas analyser would be higher than patients PaO2 at 32 degrees C c) the pH of the blood rises by 0.015 units per 10 degrees C decrease in body temperature d) the alpha-stat approach is to keep uncorrected PaCO2 and pH at normal levels e) PaCO2 of a patient at 30 degrees C will be lower than that at body temperature 10. Regarding fibrinolysis: a) streptokinase converts plasminogen to plasmin b) tranexamic acid is a potent inhibitor of plasmin c) aprotinin has antiplasmin activity d) urokinase is a plasminogen activator e) rT-PA converts plasminogen to plasmin 11. Adenosine: a) is an endogenous nucleoside and a metabolite of high energy phosphates b) has a half-life of 8-10 seconds c) has a negative inotropic effect d) can cause severe bradycardia e) has cellular protective effects during hypoxia or ischaemia 12. Breathing out against a closed glottis: a) raises intra-tracheal pressure b) heart rate slows c) right ventricular output is increased d) left ventricular output is decreased e) systolic pressure falls then rises

13. Regarding anaesthesia in a hyperbaric chamber: a) 0.4% isoflurane at 3 atmospheres will produce a similar depth of anaesthesia to that produced by 1.2% isoflurane at sea level b) rotameters may read falsely high due to an increase in gas density c) air can be used to elicit a loss of resistance of epidural space d) endotracheal tube cuffs should be inflated with saline e) the risk of oxygen toxicity is higher with high FIO2 14. The natural frequency of direct pressure measuring systems (desirable > 30 Hz) would be higher in presence of:

760

a) a wide bore cannula b) a very long catheter c) a wider catheter d) a less compliant catheter e) multiple three way stopcocks 15. On ascent to 6000 metres (1/2 ambient pressure): a) respiratory minute volume is increased b) plasma pH is increased initially c) the urine is alkaline d) hypoxia occurs e) cerebral blood flow is increased 16. Insulin secretion: a) is about 1 U/hour in the basal state b) is increased 5-10 times following ingestion of food c) is increased by glucagon d) is depressed in starvation e) is increased in the perioperative period 17. Adenyl cyclase: a) catalyses the conversion of ATP to cyclic AMP b) is linked to stimulatory and inhibitory G proteins c) is decreased by aminophylline d) release is triggered by cyclic AMP e) is an integral part of beta-adrenergic receptors 18. Concerning bilirubin: a) conjugated bilirubin is water soluble b) a conjugated bilirubin of 20 mmol/L in a neonate will cause brain damage c) the serum-conjugated bilirubin is increased in haemolysis d) barbiturates may be used to treat hyperbilirubinaemia e) urinary urobilinogen excretion is increased in obstructive jaundice 19. Regarding arterial blood pressure waveform: a) initial upstroke reflects the inotropic component of the left ventricle b) systolic and diastolic portion can be separated by dicrotic notch c) systolic peak is higher in radial artery than that in the aorta d) dicrotic notch is more pronounced in children e) area under the pressure waveform represents stroke volume 20. Motility of the gastrointestinal tract is increased by: a) vagal blockade b) mechanical obstruction

761

c) stimulation of splanchnic nerves d) blockade of ventral roots near subarachnoid space below level of T4 e) neostigmine 21. Pulmonary arterial occlusion pressure will be greater than left ventricular end-diastolic pressure in the presence of: a) severe mitral stenosis b) positive end-expiratory pressure c) left atrial myxoma d) stiff left ventricle e) premature closure of mitral valve 22. Starvation causes: a) increase in plasma glucose b) increase in urinary nitrogen excretion c) increase in plasma ketone bodies d) increase in glucose utilisation by the brain e) a metabolic alkalosis 23. Regarding isomerism of the anaesthetic drugs: a) propofol and sevoflurane are achiral compounds b) ropivacaine, rocuronium and cisatracurium are single stereoisomers c) bupivacaine and ketamine are racemic compounds d) atracurium and mivacurium have more than two stereoisomers e) there is no pharmacodynamic diference between R and S enantiomers 24. In a normal subject, the resting PaCO2 depends on: a) cardiac output b) alveolar ventilation c) transfer factor d) carbon dioxide production e) activity of medullary chemoreceptors 25. Central venous pressure is greater than pulmonary arterial occlusion pressure in: a) right ventricular failure b) pulmonary embolism c) chronic lung disease d) pulmonary hypertension e) acute pulmonary regurgitation 26. The following statements are true regarding these new drugs: a) fenoldopam, a D1 agonist, is used for treating hypertensive emergencies b) bambuterol is a prodrug, converted to terbutaline in the body c) nicorandil, a potassium channel activator, reduces both preload and afterload

762

d) nimodipine is a cerebral vasodilator e) nabilone, a synthetic cannabinoid, is used as an anti-emetic during cancer chemotherapy 27. Regarding the loop of Henle: a) the descending limb is permeable to water b) the filterate becomes hypertonic as it goes down the descending limb c) the ascending limb of loop is virtually impermeable to water d) about 15% of filtered water is absorbed in the loop of Henle e) the thick ascending limb has a higher amount of Na-K ATPase than any other part of the renal tubule 28. Concerning renal function: a) when the glomerular filtration rate is reduced by 75%, serum creatinine begins to rise b) urine osmolality reflects the tubular function c) urinary sodium <20 mmmol/L indicates hypovolaemia d) fractional excretion of sodium is greater than normal in acute tubular necrosis 29. Regarding sodium excretion by the kidney: a) 96-99% of filtered sodium is absorbed by the renal tubule b) fractional excretion of sodium is increased with a salty diet intake c) brain natriuretic peptide increases renal sodium excretion d) dopamine decreases the renal sodium excretion e) most of the filtered sodium is reabsorbed with chloride in renal tubules 30. Blood urea nitrogen is an unreliable measure of renal dysfunction as it is increased in: a) gastrintestinal haemorrhage b) dehydration c) excessive protein intake d) end-stage liver disease e) omission of essential amino acids from diet ANSWERS 1.TFTFF 2.TFFTT 3.TTTTF 4.TTFFT 5.TTFTT 6.FTFFT 7.FTTTT 8.TTFTT 9.TTTTT 10.TTTTT 11.TTTTT

763

12.TFFFF 13.TTFTT 14.TFTTF 15.TTTTT 16.TTTTF 17.TTFFT 18.TFFTT 19.TTTTT 20.FTFFT 21.TTTFF 22.FTTFF 23.TTTTF 24.TTTTT 25.TTTTT 26.TTTTT 27.TTTTT 28.TTTT 29.TTTFT 30.TTTFT General Primary MCQ 11 1. Factors which affect the peak effect of intravenous anaesthetics include: a) dose administered b) volume of distribution c) hepatic clearance d) rate of injection e) cardiac output 2. The following drugs are excreted largely unchanged by the kidney: a) lidocaine b) morphine c) propofol d) levo-bupivacaine e) vecuronium 3. The following statements are true about the flow of gases: a) gas flow is proportional to fourth power of the radius in laminar flow b) resistance is directly proportional to length of a tube c) a gas with low density is likely to develop turbulent flow d) flow is not dependent on viscosity in laminar flow e) flow is inversely proportional to the square root of pressure in turbulent flow 4. The following are correct: a) t1/2 = k/0.693

764

b) CL= k.Vd c) CL = 0.693 x Vd / t1/2 d) pH = pKa + log [base/acid] e) I x Css = CL where I = infusion rate, Css = plasma concentration at steady state, CI = clearance 5. Ketamine: a) is an imidazole derivative b) has also antanalgesic effect c) is contraindicated rectally d) has no active metabolites e) probably has no effect on intracranial pressure 6. The following statements are true regarding the opioids used in the perioperative period: a) morphine may raise plasma histamine b) pethidine does not alter the heart rate c) pethidine is more of a myocardial depressant than morphine d) sufentanil may not cause truncal rigidity in high doses e) opioid-induced bradycardia is rare in patients undergoing bilateral vagotomy 7. Regarding the pharmacokinetics of opioid analgesics: a) bioavailabilty of most of the opioid given by the oral route is about 75-85% b) highly water soluble opioids have a rapid onset of action c) the duration of action of opioids is related to their terminal half-lives d) they have flow-dependent hepatic clearance e) morphine has a terminal half-life similar to fentanyl 8. Methohexitone: a) is an oxybarbiturate b) may cause pain on injection c) is a methylated hexobarbitone d) is safer in asthma than thiopentone e) has a shorter half life than propofol 9. The following statements regarding the pharmacodynamics of opioids are true: a) naloxone is more effective at mu receptors than at other opioid receptors b) unchanged diamorphine has no affinity for opioid receptors c) pethidine penetrates the blood-brain barrier quicker than morphine d) pethidine may be used safely in patients receiving monoamine oxidase inhibitors e) the duration of action of remifentanil is prolonged by concomitant administration of anticholinesterase drugs 10. Non-selective beta-blockers have the following side-effects: a) bronchospasm

765

b) increased uterine tone c) hypoglycaemia d) decreased peripheral blood flow e) urinary retention 11. The following influence the rate of absorption of a drug given intramuscularly: a) pka b) pH c) blood flow to the muscle d) site of injection e) pharmaceutical formulation 12. Clonidine: a) is an alpha-1 adrenoceptor agonist b) reduces catecholamine release in the body c) is a commonly used antihypertensive agent d) is contraindicated via the epidural route e) does not alter the minimal alveolar concentration of volatile agents 13. Regarding partial opioid agonists: a) partial agonists are agonists at m but antagonists at k receptors b) buprenorphine has low intrinsic activity at m receptors c) partial agonists show a plateau or ceiling effect in their dose-response curve d) nalbuphine is equipotent with morphine e) buprenorphine is longer acting due to its strong receptor affinity 14. When using a nerve stimulator to monitor neuromuscular blockade: a) a minimum of 30 seconds must be left between train of four (T-O-F) stimuli b) tetanic fade suggests inadequate reversal of blockade c) recovery of 25% single twitch height is the same as return of 1st twitch of T-O-F d) depolarising block does not alter the T-O-F ratio e) ability to raise the head for 5 seconds corresponds to 50% twitch height recovery 15. Naloxone: a) is a pure antagonist b) does not produce antanalgesic effects in naive subjects c) has a terminal half-life of 2.5 hours d) can be used orally e) completely reverses the side effects of buprenorphine 16. Enoximone: a) selectively inhibits phosphodiesterase type III isoenzyme b) may even be used in hypotensive patients c) is routinely administered by IV bolus

766

d) improves cardiac index e) does not cause thrombocytopenia 17. In a patient with a low cardiac output, the following are true of inhalational anaesthetic agents: a) a lower inspired anaesthetic concentration will be required to induce anaesthesia b) it would take longer to reach a given depth of surgical anaesthesia c) the rate at which alveolar approaches inspired anaesthetic concentrations is faster d) a higher inspired anaesthetic concentration is required to reach the same brain anaesthetic concentration e) there will be no change in the inspired anaesthetic concentration to achieve the same depth of anaesthesia 18. The rate of induction using an inhalational technique is directly related to: a) the rate at which alveolar approaches the inspired fractional anaesthetic concentration b) the blood-gas solubility coefficient c) the inspired fractional anaesthetic concentration d) the alveolar ventilation e) right or left shunt 19. In the unconscious patient: a) absence of radial pulse indicates a cardiac arrest b) chest movement indicates breathing is present c) dilated pupils indicate brain damage has occurred d) the airway should be checked for obstruction e) the patients dentures should be removed 20. In normal pregnancy at term: a) the maternal haemoglobin content is increased b) the lung volume is increased c) tidal volume is increased d) the cardiac output is increased by 10-20% e) glomerular filtration rate may be increased by up to 80% 21. Aortocaval compression in the pregnant patient: a) may produce hypotension b) will only occur in the supine posture c) will only occur in patients with epidural or spinal anaesthesia d) may present solely as foetal distress e) IV ephedrine is the initial treatment of choice 22. The following cause an increase in cerebral blood flow: a) dexamethasone b) isoflurane

767

c) glyceryl trinitrate d) ketamine e) nitrous oxide 23. Infusions of sodium nitroprusside: a) do not interfere with autoregulation of cerebral blood flow b) result in cyanide toxicity if the dose exceeds 1 mcg/kg given over 3 hours c) cause an increase in renin secretion d) reduce cerebral oxygen consumption e) should be protected from light 24. A highly ionised drug: a) is well absorbed from the intestine b) is excreted mainly by the kidney c) crosses the placental barrier easily d) is reabsorbed from the renal tubules e) is highly protein bound 25. Nitric oxide: a) is beneficial in reducing pulmonary vascular resistance b) is available as compressed gas in cylinders c) side-effects are related to metabolic by-products d) is useful in treating methaemoglobinaemia e) in therapeutic concentrations can cause brain damage 26. Regarding drug pharmacokinetics: a) Vd represents the apparent volume available in the body for drug distribution b) CL reflects the ability of the body to eliminate the drug c) the terminal half-life of a drug is not dependent on Vd and CL d) the Vd of muscle relaxants is localised to the plasma and extracellular fluid e) Vd can never be greater than total body water 27. Regarding anaesthetic breathing systems: a) rebreathing does not occur in Mapleson D during controlled ventilation b) with a fresh gas flow (FGF) <1.5 L, volatile concentration in breathing system may be higher than the dial setting of the vaporiser c) in circle system with a FGF <1 L (N2O:O2=66:34), FIO2 in the circle may increase over time d) a FGF of 3 times the minute volume may be needed to prevent rebreathing in Bains coaxial system during spontaneous ventilation e) dead space gas is preserved in Magill system during spontaneous ventilation 28. The volume of distribution of a drug is not altered by: a) age

768

b) increased extracellul;ar fluid volume c) pregnancy d) renal failure e) cardiac failure 29. Regarding pharmacokinetics: a) terminal half-life provides a guide to frequency of drug administration b) for IV drugs, loading dose=CpXVd and rate of infusion = CpxCl, where Cp is the desired plasma concentration c) terminal half-life is not related to the duration of action in case of IV barbiturates d) diazepam has a high oral bioavailability e) cimetidine appears to increase the bioavailability of propranolol 30. Regarding drug metabolism: a) liver converts water-soluble to lipid-soluble drugs b) oral opioids have a negligible first-pass effect c) thiopentone in the dose used for induction will induce the hepatic enzymes d) anticonvulsants have little effect on hepatic microsomal enzymes e) metronidazole appears to induce hepatic microsomal enzymes

ANSWERS 1.TFFTT 2.FFFFF 3.TFFFF 4.FTTTF 5.FFFFF 6TFTFT 7.FFFTT 8.TTTFF 9.TTTFF 10.TTTTF 11.FFTTT 12.FTFFF 13.FTTTT 14.FTFTF 15.TFTFF 16.TFFT 17.FFTFT 18.TTTTF 19.FFFTT 20.TFTFT 21.TFFTF 22.FTTTT

769

23.FFTFT 24.FFFFF 25.TTFFF 26.TTFTF 27.FFFTT 28.FFFFF 29.TTTTF 30.FFFFF

General Primary MCQ 12 1. Albumin: a) has a molecular weight of 69,000, with a strong negative charge on its surface b) is synthesised by hepatocytes at a rate of 9-12 g/day in a healthy adult c) generates oncotic pressure because of its high molecular weight d) has a degradation half-life of 18 days e) can be present in the urine in individuals on prolonged standing 2. The following statements are correct: a) thromboxane A2 is principally produced by the endothelial cells b) prostacyclin is produced in large amounts by the platelets c) aspirin irreversibly inhibits cyclooxygenase both in platelets and endothelial cells d) nitric oxide activates soluble guanylyl cyclase in smooth muscle e) endothelins are polypeptides with half-lives of less than a minute 3. Regarding renal physiology: a) when the rate of flow through tubule increases, the glomerular filtration rate (GFR) in the same nephron decreases b) when GFR increases, the solute reabsorption in the tubule also increases c) for each H+ secreted in the renal tubule, one Na+ and one HCO3- enter the interstitial fluid d) more H+ is secreted in the proximal tubule than in the distal tubule e) urine becomes more acidic in presence of low plasma HCO34. Regarding diuretics: a) ethanol inhibits antidiuretic hormone secretion b) theophylline increases the sodium content of urine c) acetozolamide increases the sodium content of urine d) spironolactone can cause hyponatraemia e) frusemide leads to increased K+ secretion in the distal tubule

770

5. Pulmonary vascular resistance is: a) increased by low alveolar PO2 b) decreased by prostacyclin c) decreased by isoprenaline d) increased by adrenaline e) increased by metabolic and respiratory acidosis 6. Regarding the drugs acting on the autonomic nervous system: a) methyldopa forms false transmitters b) bretylium prevents norepinephrine release c) yohimbine blocks alpha-2 adrenergic receptors d) mivazerol is an alpha-2 agonist e) clonidine has analgesic action 7. Esmolol: a) is a beta-1 selective blocker b) has an elimination half-life of 9 minutes c) is metabolised by plasma pseudocholinesterase d) may cause hypotension e) is useful in the management of supraventricular arrhythmias 8. Regarding gastric acid secretion: a) when acid production is increased after a meal, urine may be less acidic b) parietal cells have H+,K+-ATPase proton pumps c) activity of the proton pump is determined by the level of cAMP in parietal cells d) parietal cells have H2, M1 and gastrin receptors e) prostaglandins inhibit the activity of proton pumps 9. Regarding the measurement of body temperature: a) electrical resistance of platinum increases with rises in temperature b) thermistor is a metal oxide whose resistance drops with rises in temperature c) copper and constanton are used for the Seeback effect in thermocouple d) infrared thermometer is increasingly used for measuring tympanic membrane temperature e) the triple point of water is at 273.160 Kelvin 10. In the pulmonary circulation: a) serotonin is almost completely removed by a single pass b) bradykinin is degraded by angiotensin-converting enzyme c) histamine is unaffected during its passage d) resistance is decreased by hypoxia e) oxygen saturation is higher in pulmonary veins than bronchial veins

771

11. At high altitude (e.g. 6000 m): a) barometric pressure is about half atmospheric b) FIO2 is 0.21 c) the saturated vapour pressure of water is 6.3 kPa at 37 degrees C d) the boiling point of water would be low e) hypoxia predominates the ventilatory drive 12. Regarding the opioids used in the perioperative period: a) papaveretum has a morphine content up to 70% b) codeine has a higher oral bioavaibility than morphine c) pethidine crosses the blood-brain barrier more rapidly than morphine d) sufentanil is 5-10 times as potent as fentanyl e) the metabolism of remifentanil is affected by concomitant administration of anticholinesterases 13. The output of a variable bypass vaporiser can be influenced by: a) rate of fresh gas flow b) temperature inside the vaporiser c) back pressure during intermittent positive pressure ventilation d) composition of carrier gases e) filling halothane vaporiser with enflurane 14. Naloxone: a) is a derivative of oxymorphone b) has a reverse agonist effect c) can cause pulmonary oedema d) has a duration of effective antagonism of 30-45 minutes e) has an antanalgesic effect in individuals not given opioids 15. Functional reserve capacity: a) increases in obstructive airways disease b) decreases in term pregnancy c) is increased by application of positive end-expiratory pressure d) is lower in morbidly obese patients e) is not altered after induction of anaesthesia with sevoflurane 16. The following are algesic substances: a) bradykinin b) histamine c) 5-hydroxytryptamine d) substance-P e) calcitonin gene-related peptide (CGRP)

772

17. Pulse pressure: a) depends on left ventricular stroke volume b) increases with increase in the velocity of blood flow c) depends on the compliance of the arterial tree d) is higher in peripheral arteries than that in the aorta e) is low in high cardiac output states 18. Pancuronium: a) is a synthetic steroid b) releases a large amount of histamine c) causes noradrenaline release at nerve endings d) can cause ganglion blockade e) has a duration of action of 45-60 minutes 19. Regarding anaesthetic gases: a) nitrous oxide can be liquified at 50 atmospheres at room temperature b) nitrogen can cause surgical anaesthesia at 12 atmospheres c) a half empty oxygen cylinder has a pressure of about 68 atmospheres d) oxygen is stored for hospital supply in liquid form at -150 degrees C e) Boyles Law does not apply to anaesthetic gases due to van der Waals attraction between the molecules 20. Regarding the neuromuscular junction: a) the junctional cleft is about 50-60 nm across b) the postjunctional membrane is thrown into folds c) the acetylcholine receptor has five protein subunits d) two alpha subunits of the acetylcholine receptor have to be activated for the channel to open e) acetylcholine is synthesised from choline and acetic acid 21. Packed red blood cells: a) one bag contains 220 ml of red cells and 80 ml of plasma b) has a packed cell volume of 0.65-0.75 c) cause fewer febrile transfusion reactions d) contain a negligible amount of platelets and white cells e) can have saline, adenine, glucose and mannitol (SAGM) added as an additive solution 22. Regarding cerebral blood flow: a) the brain receives 15% of cardiac output b) cerebral blood flow rises by 2% for each mm rise in PaCO2 c) volatile anaesthetics increase cerebral blood flow d) volatile anaesthetics can impair the autoregulation of cerebral blood flow e) cerebral blood flow is relatively constant for cerebral perfusion pressures between 50-150 mmHg

773

23. Glucagon: a) plasma level is increased in cirrhosis of the liver b) is an inotrope c) causes glycogenolysis in liver but not in muscle d) secretion is inhibited by propanolol e) can cause hyperkalaemia 24. Midazolam: a) has a half-life of less than 2 hours b) is water soluble at a pH 4 c) has an active metabolite d) can have a prolonged effect if given with erythromycin e) can be given by the intranasal route 25. Ketamine: a) raises the plasma noradrenaline level b) can cause unpleasant side-effects in adults for 24 hours after administration c) is a depressant to denervated cardiac muscle d) produces a loss of consciousness in one arm-brain circulation time e) is a bronchdilator 26. Which of the following are partial agonists at opioid receptors: a) nalbuphine b) nalorphine c) naloxone d) buprenorphine e) meptazinol 27. Enflurane: a) is a halogenated hydrocarbon with an oil:gas partition coefficient of 98 b) when metabolised is converted to form inorganic flouride ions c) at high concentration can cause seizures when associated with low PaCO2 d) depresses hypoxic pulmonary vasoconstriction e) depresses uterine tone and contractility 28. Regarding gas or liquid flow: a) in laminar flow, flow is determined by the pressure gradient divided by resistance b) in laminar flow system, resistance is constant and independent of flow c) resistance increases with flow in turbulent flow d) Poiseuilles law does not apply in turbulent flow e) flow through an orifice is always turbulent

774

29. Regarding fluid flow: a) flow is laminar in tubes that are very long compared with their diameter b) flow is turbulent when Reynolds number is < 2000 c) flow through an orifice is influenced by the fluid or gas density d) velocity is higher and pressure is lower at the point of constriction in a tube e) at low haematocrit, blood flow is better in blood vessels 30. Regarding physics applied to anaesthesia: a) the mass of a gas that dissolves in a liquid is proportional to its partial pressure b) a tube whose radius exceeds its length is called an orifice c) bipolar cautery does not need a dispersive ground plate electrode d) in unipolar cautery, if the dispersive pad is not in proper contact with the skin, burn can occur around the ECG electrode e) gases do not obey Boyles law at temperatures at which they approach their point of liquefaction BONUS QUESTION Regarding pain transmission: a) ad fibres are myelinated while C fibres are unmyelinated b) visceral pain is carried by unmyelinated sympathetic fibres c) sharp pain is transmitted by Ad fibres while dull aching pain is transmitted by C fibres d) lamina II of the dorsal horn has inhibitory interneurones which modulate pain transmission e) besides pain, Ad- and C fibres also carry touch and temperature sensation ANSWERS 1.TTFTT 2.FFTTT 3.TTTTT 4.TTTTT 5.TTTFT 6.TTTTT 7.TTFTT 8.TTTTT 9.TTTTT 10.TTTFT 11.TTTTT 12.TTTTF 13.TTTTT 14.TFTTT 15.TTTTF 16.TTTTT 17.TTTTF 18.TFTFT 19.TTTTT

775

20.TTTTF 21.TTTTT 22.TTTTT 23.TTTTT 24.TTTTT 25.TTTFT 26.TTFTT 27.TTTTT 28.TTTTF 29.TFTTT 30.TTTTT BONUS QUESTION.TTTTT MCQ A 1. Intermittent porphyria: a) Is autosomal recessive b) Can present with autonomic dysfunction c) Can present with abdominal pain and no other signs d) Can present with psychosis e) Can be started by ethanol 2. A Nitrogen washout curve is used to measure: a) Anatomical dead space b) Physiological dead space c) Vital capacity d) Uneven ventilation of different areas of lung e) Diffusion capacity 3. Amitryptiline overdose is commonly associated with: a) Convulsions b) Dysrhythmias c) Metabolic acidosis d) Hypothermia e) Hypercapnia 4. The bleeding time is increased in: a) Haemophilia b) Idiopathic thrombocytopenic purpura c) Coumarin overdose d) Vitamin C deficiency 5. Regarding the Severinghaus CO2 electrode: a) The surrounding medium is bicarbonate b) It contains CO2-sensitive glass

776

c) It is more accurate with gases than blood d) It is temperature sensitive 6. The following methods can block sensory input from the larynx: a) LA spray to the cords b) LA to the pyriform fossa c) Retrograde LA spray through cricothyroid membrane d) Superior laryngeal nerve block in hypothyroid groove e) Recurrent laryngeal nerve block

7. Pulmonary artery wedge pressure can be increased: a) After myocardial infarction b) In mitral stenosis c) In aortic incompetence d) In pulmonary fibrosis e) In pulmonary stenosis 8. In the myasthenic syndrome there is: a) Sensitivity to depolarising muscle relaxants b) Sensitivity to non-depolarising muscle relaxants c) Post-tetanic potentiation d) Improvement with repeated muscle activity e) Decreased voltage on the EMG 9. Paracetamol: a) Is fully absorbed from the stomach b) Is an aniline derivative c) Is converted to phenacetin d) Is a useful anti-inflammatory agent e) Is excreted unmetabolised 10. In phaeochromocytoma treated with beta blockers, there is: a) Decreased systolic blood pressure b) Decreased heart rate c) Miosis d) Cold peripheries e) Postural hypotension 11. Dystrophia myotonica is associated with: a) Cataracts b) Frontal baldness c) Temporalis wasting

777

d) Sternomastoid wasting e) Dysarthria f) Sensitivity to non-depolarising muscle blockers 12. The following statements concerning pulmonary function are true: a) Low forced expiratory volume/forced vital capacity occurs in restrictive conditions b) Peak expiratory flow rate can be measured using a pneumotachograph c) Peak expiratory flow rate is unrelated to effort d) A vitalograph may be used to measure the peak expiratory flow rate e) In obstructive conditions, the forced vital capacity is reduced 13. Halothane concentrations can be measured using: a) Absorption on to silicone rubber b) Ultraviolet light absorption c) Infrared light absorption d) Refractrometry e) Paramagnetism 14. The standard deviation of a sample taken from a normally distributed population: a) Is the square root of variance b) Is greater than the standard error c) Indicates the limits from the mean that contain 2/3 of the values d) Reflects the bias in selecting the original population e) Is greater than the mean 15. A Wrights respirometer: a) Measures volume directly b) Gives falsely low readings at high flow rates c) Has an internal volume of 150 ml d) May be used to measure peak flow rate e) Is unaffected by the composition of the gases being measured 16. The following endobronchial tubes have no carinal hook: a) Carlens b) Robertshaw c) McIntosh-Leatherdale d) Bryce-Smith e) White 17. In a patient with jaundice, the following suggest obstruction: a) No urobilinogen in urine b) Increased stercobilinogen c) Raised aspartate transaminase

778

d) Normal acid phosphatase e) Normal alkaline phosphatase f) Increased concentration bilirubin in urine therefore dark g) Coagulopathy 18. Creatinine phosphokinase is raised: a) 24 hours post-myocardial infarction b) In muscular dystrophy c) In myasthenia gravis d) In thyrotoxicosis e) In osteomalacia 19. The recurrent laryngeal nerve: a) Supplies some intrinsic muscles of the larynx b) Supplies the cricothyroid muscle c) Supplies the inferior constrictor muscle d) Supplies sensation below the vocal cords e) Supplies sensation to the posterior 2/3 of the tongue 20. Regarding the Fluotec Mark III vaporiser: a) The temperature control is a bellows mechanism b) The termperature control is at the vaporising chamber outlet c) When filled with isoflurane it will deliver approximately the dialled concentration d) It is not accurate at flows of less than 2 litres/minute 21. Mydriasis can occur in: a) Horners syndrome b) Neurosyphilis c) Untreated organophosphate poisoning d) Oculomotor palsy 22. Prolonged irrigation of the bladder with isotonic saline for transurethral resection TUR is associated with: a) Hyponatraemia b) Hypernatraemia c) Hypokalaemia d) Reduced osmolarity e) Haemolysis f) Confusion 30. In a patient who has had a traumatic quadriplegia for one week: a) Intermittent positive pressure ventilation can cause hypotension b) Suxamethonium sensitivity occurs c) Retention of urine occurs

779

d) Increased tendon jerks occur e) Steroids improve prognosis ANSWERS 1.FTTTT 2.TFFTF 3.TTTFT 4.FTFT 5.TFFT 6.TTTTT 7.TTTTF 8.TTTTF 9.FTFFF 10.TFFFT 11.TTTTFT 12.FTFFT 13.TTTTF 14.TTTFF 15.FFFFT 16.FTTTF 17.TFFFFTT 18.TTFFF 19.TFFTF 20.FFTF 21.FFFT 22.TFTFFT 23.TTTTT

MCQ B 1. Ritrodrine can cause: a) bradycardia b) heart block c) Left ventricular failure d) hypotension e) peripheral vasoconstriction 2. Trigeminal ganglion block causes ipsilateral analgesia of: a) lower lip b) inside the nose c) angle of the jaw d) external auditory meatus e) soft palate

780

3. A pressure cycled ventilator: a) is a minute volume divider b) is cycled from inspiration to expiration after a set time c) is cycled from expiration to inspiration after a set time d) has end expiratory pressure determined by the patients compliance e) ventilates a patient with obstructive airways disease more efficiently if operated at low flows 4. Airways resistance: a) is greater during inspiration than expiration b) can be measured by whole body plethysmography c) is increased by ephedrine d) is one factor determining the work of breathing e) is expressed in kPa/L/s 5. Regarding Maplesons breathing systems: a) D is identical to Magill b) some degree of rebreathing can occur with all systems c) A requires the same for spontaneous ventilation and IPPV d) F system is used in paediatrics e) E system is the most efficient for spontaneous ventilation 6. Recognised effects of positive end-expiratory pressure include: a) sodium retention b) fall in cardiac output c) rise in closing volume d) rise in functional residual capacity e) fall in central venous pressure 7. An elderly man given atropine becomes excited and confused; appropriate treatment includes: a) morphine b) physostigmine c) intubation and ventilation d) droperidol e) chlorpromazine 8. Causes of increased bleeding during head and neck surgery include: a) hypoxia b) hypocarbia c) respiratory obstruction d) chronic aspirin ingestion e) transfusion of excessive quantities of stored blood

781

9. In cardioversion for dysrhythmias: a) the shock is given on the upstroke of the T wave b) general anaesthesia is not required if a synchronised shock is given c) AC is safer than DC current d) no preoperative assessment is needed for elective cases e) ventricular fibrillation may result 10. Hyperventilation throughout general anaesthesia results in: a) increased anaesthetic requirements b) shift of the oxygen dissociation curve to the right c) increased arterial to venous pO2 difference d) reduced renal blood flow e) hyperkalaemia 11. In patients with haemorrhagic shock: a) physiological dead space is increased b) renal blood flow is decreased c) antidiuretic hormone secretion is increased d) the oxygen dissociation curve is shifted to the left e) O2 delivery is decreased 12. Regarding dental anaesthesia: a) oral debris inhalation is unlikely if the patient is supine b) intermittent methohexitone is a satisfactory technique for the operator anaesthetist c) the number of dental general anaesthetics given is steadily increasing d) demand flow anaesthetic systems result in economy of gases e) simple monitoring is not essential 13. The right lung has: a) one fissure b) no Sibsons fascia c) direct relationship to the azygos vein d) two pulmonary veins e) seven broncho-pulmonary segments 14. Six hours after thoracotomy for oesophageal resection: a) vital capacity is reduced b) functional residual capacity is reduced c) peak flow is reduced d) venous admixture is reduced e) pO2 on air is reduced 15. Cardioplegia:

782

a) contains a high concentration of calcium ions b) contains a high concentration of potassium ions c) contains local anaesthetic d) stops the heart in diastole e) is given at 8 degrees C 16. On assessing adequate medullary in posterior fossa surgery the following may be useful: a) delta waves on the EEG b) hypothermia c) rise in mean arterial pressure d) arrhythmias e) regular respirations 17. The critical temperature of a gas is that: a) at which the pressure is the critical pressure b) at which freezing takes place c) at which attraction between gas molecules is negligible d) above which the gas cannot be ignited e) at which Boyles law is perfectly obeyed f) above which it cannot be liquefied 18. Central venous pressure is raised and cardiac output is reduced in the following: a) Addisons disease b) pulmonary embolism c) air embolism d) tension pneumothorax e) pre-eclampsia 19. Hyaline membrane disease: a) usually occurs within 12 hours of delivery b) usually occurs after Caeserean section c) has a better prognosis if steroids are given to the infant d) is uncommon after 36 weeks' gestation e) is more common in multiple pregnancies 20. Raised left ventricular end-diastolic pressure implies: a) increased left ventricular compliance b) decreased left ventricular left ventricular stroke work index c) decreased oxygen flux d) decreased myocardial oxygen demand e) decreased myocardial oxygen supply

783

21. Internal carotid artery disease can cause: a) transient loss of vision b) tunnel vision c) paraesthesiae on the side of the face d) dysphagia e) Parkinsonss disease 22. Pulmonary capillary wedge pressure is a reliable parameter: a) in mitral stenosis b) after anterior myocardial infarction c) in pulmonary stenosis d) in aortic stenosis e) in pulmonary fibrosis 23. Vitamin B12 is appropriate treatment in: a) vegans b) pregnancy c) Crohns disease d) Partial gastrectomy e) Microcytic anaemia 24. The following are intimately related to the neck of a femoral hernia: a) public tubercle b) inguinal ligament c) inferior epigastric artery d) femoral vein e) femoral artery 25. The following result from ulnar nerve block at the wrist: a) weakness of adductor pollicis b) paraesthesiae in the palmar aspect of the little finger c) paraethesiae in the dorsal aspect of the little finger d) vasodilatation in the nerve distribution e) analgesia of C8 dermatome 26. The celiac plexus is related: a) anteriorly to the crura of the diaphragm b) anteriorly to the inferior vena cava c) anteriorly to the aorta d) to the L3 vertebra e) posteriorly to the pancreas

784

27. In cases of gastrointestinal bleeding: a) blood volume falls by 30% before a fall in blood pressure is observed b) there is a higher incidence in males than females c) the cause is more likely to be a duodenal than a gastric ulcer d) tachycardia may present before overt blood loss e) surgery must take place before resuscitation 28. Sympathetic dystrophies: a) can cause osteoporosis b) cause increase in skin temperature c) cause pain d) cause vasoconstriction e) are more common in athletes 29. For amputation at mid-thigh, the following nerves must be blocked: a) obturator b) femoral c) sciatic d) genitofemoral e) lateral cutaneous nerve of the thigh 30. The after coming twin becomes stuck following the administration of ergometrine. The following drugs will reliably relax the uterus: a) thiopentone b) suxamethonium c) ritodrine d) salbutamol e) halothane 31. Raised creatinine phospokinase: a) occurs after the administration of suxamethonium b) implies a recent myocardial infarction c) may reflect an increased risk of malignant hyperpyrexia d) is an indication of alcoholic liver disease e) is seen in acute pancreatitis 32. The following drugs may be safely used in porphyria: a) barbiturates b) bupivacaine c) lidocaine d) chlorpromazine e) sulphonamides

785

33. An aortic aneurism can be said to be leaking if there is: a) loin pain b) back pain c) anuria d) obstructed bowel sounds e) no femoral pulse f) absent bowel sounds 34. With regard to cardiopulmonary resuscitation (CPR): a) 40% of patients leave hospital b) new CPR gives better cerebral perfusion c) perfusion is achieved by the transmission of the increased intrapleural pressure to the great veins d) the presence of hypovolaemia is an indication to open the chest 35. In septic shock: a) peripheral hypothermia is associated with good prognosis b) the patients usually have an increased cardiac output c) the patients usually have a depleted circulating volume d) the causative organisms are always Gram negative e) antibiotics should not be given before blood culture results are available 36. Rheumatoid arthritis is associated with: a) pericarditis b) aortic stenosis c) tricuspid incompetence d) aortic aneurism 37. After head injury, increased intracranial pressure is indicated by: a) fall in blood pressure b) reduction in the Glasgow coma score c) increase in heart rate d) increase in pCO2 e) small pupils 38. In haemophilia: a) the mode of inheritance is X-linked b) the thromboplastin generation test may be abnormal despite a normal bleeding time c) cryoprecipitate is the most commonly used treatment d) haematomas are treated by aspiration e) disseminated intravascular coagulation is a common complication following surgery

786

39. In the diagnosis of brain death: a) lack of EEG activity is essential b) caloric tests must be performed bilaterally c) the admitting consultant must certify death d) lack of stretch reflexes in all limbs is essential e) the pupils must be fixed and dilated 40. Acute frontal sinusitis may cause: a) subdural abscess b) cerebral abscess c) spreading osteomyelitis d) orbital cellulitis e) meningitis 41. In the ECG: a) T wave indicates repolarisation b) the transmembrane potential is the ratio of K+ in to K+ out c) a single V lead requires one connection only d) QRS width is proportional to muscle mass e) negative deflection indicates movement away from the electrode 42. Diazoxide in pre-eclampsia: a) reduces the diastolic blood pressure b) relaxes the uterus c) prevents hypoglycaemia d) promotes diuresis e) causes sedation 43. Vomiting is affected by: a) age b) weight c) the choice of premedicant d) intraoperative hypotension e) the duration of anaesthesia 44. Dysphagia may be associated with: a) cervical rib b) pharyngeal pouch c) Plummer-Vinson syndrome d) achalasia e) carcinoma of the bronchus

787

45. Carcinoma of the bronchus may be associated with: a) pink palms b) thirst c) Horners syndrome d) dysphagia e) cerebellar ataxia 46. An 8-year-old child is rescued 20 minutes after drowning and has a core temperature of 30 degrees C and fixed dilated pupils. Further appropriate treatment includes: a) phenobarbitone b) rapid rewarming c) hypoventilation d) steroids e) cardiopulmonary resuscitation 47. The recurrent laryngeal nerve supplies: a) sensation below the cords b) the intrinsic muscles of the pharynx c) the cricothyroid muscle d) the inferior constrictor muscle e) the epiglottis ANSWERS 1.FFTTF 2.TTFFF 3.FTTFF 4.FTFTT 5.FTFTF 6.TTFTF 7.FTFFF 8.TFTTT 9.FTFFT 10.FFFTF 11.TTTFT 12.FFFTF 13.FFTTF 14.TTTFT 15.FTTTF 16.?F??T 17.TFFFFT 18.FTTTF 19.FFFTT 20.FFTFT 21.TFTTT 22.FTTTF 23.TFTTF 24.TTFTF

788

25.TTTTF 26.TFTFT 27.FTTTF 28.TFTTT 29.FTFFT 30.TFTTT 31.TTTFF 32.FTFTF 33.TTTFTT 34.F?TF 35.?TTFF 36.TFFF 37.FTFTF 38.TTFFF 39.FTFFF 40.TTFTT 41.TFFFT 42.TFTFF 43.TTTTF 44.FTTTT 45.FTTTT 46.FTFFT 47.TFFFF

MCQ C 1. Regarding the Bain circuit: a) it is suitable for small children b) it can act as a Mapleson C circuit c) inspiratory flow occurs through the outer tubing d) a disconnection at the machine causes increased dead space 2. Regarding the Bain coaxial circuit: a) it may function as a Mapleson C b) it requires FGF=MV to prevent rebreathing in spontaneous respiration c) it can be used in small children d) dead space increases if the inner tube becomes detached at the machine end e) FGF occurs through the outer tube 3. Regarding the Manley ventilator: a) it is a minute volume divider b) it is a pressure generator c) it is time cycled d) it is suitable for ventilating asthmatics e) it leads to rotameter inaccuracy

789

4. The following tubes used in pulmonary surgery have a carinal hook: a) Carlens b) Gordon Green c) McIntosh-Leatherdale d) Bryce-Smith e) Robertshaw f) White 5. A stellate ganglion block causes: a) pupil dilation b) enopthalmos c) anaesthesia of the superglottic part of the larynx d) increased cerebral blood flow 6. A maxillary nerve block in the pterygopalatine fossa gives anaesthesia of: a) upper molars b) upper incisors c) soft palate d) anterior 2/3 of tongue e) anterior part of nasal septum

7. Supraclavicular brachial plexus block differs from auxiliary block in that: a) there is a greater risk of intravascular injection b) analgesia of the shoulder is poor c) it is more likely to give failure to block the interosseous nerves d) it is less likely to give analgesia to the lateral forearm e) there is an increased incidence of pneumothorax f) it is less likely to give analgesia to the under side of the forearm 8. Intercostal block of one rib at the posterior angle causes: a) somatic analgesia throughout the dermatome b) visceral analgesia c) low blood pressure because of preganglionic sympathetic block d) increased blood levels of local anaesthetic e) vasoconstriction 9. Hypotension during spinal anaesthesia may be due to: a) preganglionic autonomic blockade b) venoconstriction c) block of the dorsal roots d) ischaemia of the vasomotor centre e) block of nerves to the adrenal medulla

790

10. The volume displacement of an electromaster: a) indicates the amount of liquid needed to fill it b) depends on the volume of the chamber c) depends on the stiffness of the diaphragm d) depends on the frequency response 11. Halothane concentrations can be measured by: a) ultraviolet absorption b) infrared absorption c) a refractometer d) changes in the elasticity of silicone rubber 12. Paramagnetism is used in: a) O2 analysis b) CO2 analysis c) halothane vapour analysis 13. Radiation is the emission of: a) beta particles b) alpha particles c) neutrons d) rays e) any wave form 14. Regarding gas chromatography: a) it uses carrier gas b) it uses electron deflection c) it uses the Venturi principle d) it uses haemodilution e) it uses transillumination f) a flame ionisation detector, katharometer, electron capture detector are required 15. Pressure gauges: a) work on the Bourdon principle b) can be used to regulate gas flow c) can convert gas at a high pressure into gas at a low pressure d) form part of a flow meter 16. Success of cricoid pressure in preventing aspiration depends on: a) absence of a nasogastric tube b) an intact cricoid cartilage c) the oesophagus being pressed onto the vertebral body

791

d) extension of the neck e) preoxygenation for 5 minutes 17. After massive inhalation of gastric acid, one would expect the following: a) lung abscesses b) severe hypercapnoea c) bacteraemia d) hypovolaemia e) destruction of surfactant 18. In prolonged operations, core temperature is accurately monitored at: a) the tympanic membrane b) the lower third of the oesophagus c) the muscle in the thigh d) the nasopharynx e) the rectum 19. The following may contribute to the development of a postoperative cauda equina syndrome: a) age b) the use of adrenalin in epidurals c) Trendelenburg position d) spinal barbotage 20. During anaesthesia, a patient with Parkinsons disease on levodopa should not receive: a) enflurane b) fentanyl c) morphine d) droperidol e) nitrous oxide 21. In the foetal transfer of drugs given by the epidural route: a) the placenta is an effective barrier b) foetal bradycardia may occur with local analgesics c) highly protein-bound drugs are transferred less d) amide-linked local analgesics should be avoided 22. For Caesarean section under epidural anaesthesia: a) never give more than 20 ml of 0.5% bupivacaine b) the block must extend from T5 to S1 c) syntocinon, rather than ergometrine, must be given d) a paracervical block must also be performed e) it is necessary to preload with 500 ml Hartmanns solution

792

23. Features of amniotic fluid embolus include: a) convulsions b) cardiovascular collapse c) bronchospasm d) abnormal bleeding e) pulmonary hypertension 24. Block of the trigeminal ganglion is associated with anaesthesia of: a) alar nasi b) lower lip c) angle of mandible d) soft palate e) eardrum 25. If 50% nitrous oxide is inhaled for 3 days: a) the lymphocyte count falls b) methionine synthetase activity is reduced c) megaloblastic bone marrow changes occur d) vitamin B12 deficiency anaemia develops e) peripheral neuropathy develops 26. Helium: a) is less viscous than O2 b) is in liquid form in brown cylinders c) inhalation causes voice changes d) is used to decrease the work of breathing in bronchospasm e) supports combustion 27. Methohexitone: a) has a pH less than 8 in a 1% solution b) is a methylated thiobarbiturate c) has a longer duration of action than thiopentone d) does not cause pain on injection e) is associated with fewer excitatory phenomena than thiopentone 28. Isoflurane: a) has a similar boiling point to enflurane b) has a similar saturated vapour pressure to halothane c) has a lower blood/gas coefficient than halothane d) is extensively metabolised e) has a minimum alveolar concentration of 1.68

793

29. Nitroprusside reduces blood pressure by: a) acting on alpha-receptors b) reducing the cardiac output c) causing a bradycardia d) producing cyanide ions e) a direct action on blood vessels 30. The following are chiefly eliminated by the kidney: a) d-tubocurarine b) suxamethonium c) dopamine d) gallamine e) digoxin 31. Edrophonium has the following clinical uses: a) as a test for myasthenia gravis b) for the reversal of neuromuscular blockade c) as a centrally acting respiratory stimulant d) for the relief of urinary retention e) for the treatment of myotonia congenita 32. An untoward reaction to the following may occur in a patient on monoamine oxidase inhibitors: a) adrenaline b) thiopentone c) diazepam d) tricyclic antidepressants e) amphetamine 33. A 60-year-old man develops painless jaundice, pale stools and dark urine; which of the following are true: a) this is compatible with a raised alkaline phosphatase b) the urinary urobilinogen will be raised c) an oral cholecystogram will be useful d) the absence of pain excludes malignancy e) itching may be present 34. A low serum potassium is associated with: a) Cushings disease b) hyperventilation c) carbenoxolone therapy d) triamterene therapy e) metabolic alkalosis

794

35. Hypokalaemia requiring treatment is found in the following situations: a) following cardiac bypass (open heart surgery) b) during treatment for diabetic ketoacidosis c) after major burns d) after giving depolarising relaxants 36. The following are causes of a metabolic acidosis: a) uretero-colic fistula b) vomiting c) diarrhoea d) CO2 retention 37. Hypoalbuminaemia: a) may lead to hypocalcaemia b) is a manifestation of hypopituitarism c) can cause tetany d) can be a feature of carcinoma of the sigmoid colon e) may develop in hepatic cirrhosis

ANSWERS 1.FFFT 2.FFFTF 3.TTTFT 4.TTFFFT 5.FTFT 6.TTFFT 7.FFFFTT 8.TFFTF 9.TFFFF 10.FFTF 11.TTTT 12.TFF 13.TTTTF 14.TFFFFT 15.TFFF 16.TTTTF 17.FFFTT 18.TTFFF 19.TTFF 20.FFFTF 21.FTTF 22.FFFFF 23.TTTTT

795

24.TTFTT 25.TTTFT 26.FFTFF 27.FFFFF 28.FTTFF 29.FFFFT 30.TFFTT 31.TTFFF 32.FFFTT 33.TFFFT 34.TTTFT 35.TTFF 36.TFTF 37.TFFTT

MCQ D 1. Concerning the history of anaesthesia: a) Thomas Morton gave the first public demonstration of ether b) Nieman was the first to isolate cocaine c) Waters was the first anaesthetist to use halothane d) Humphrey Davey isolated oxygen e) Snow gave ether to Queen Victoria 2. Metabolic alkalosis with hypokalaemia is caused by: a) frusemide b) Cushing's syndrome c) Addison's disease d) vomiting 3. Digoxin toxicity may cause: a) supraventricular tachycardia b) ventricular ectopics c) liver damage d) atrioventricular block e) jaundice 4. In a circle anaesthetic system: a) the soda lime canister should be equal in volume to the patient's tidal volume b) if a vaporiser is put in the circle, it should be placed between the inspiratory limb and the fresh gas flow inlet c) the volume of the reservoir bag is not critical d) the adjustable pressure-limiting valve should be between the expiratory limb and the soda lime canister 5. Signs of fat embolism include:

796

a) pyrexia b) bradycardia c) petechial rash d) bronchospasm e) mental changes 6. The following can act by competitive antagonism: a) naltrexone b) nalorphine c) neostigmine d) flumazenil e) nalbuphine 7. During cardiopulmonary resuscitation, the following are indicated: a) lidocaine as first pharmacological therapy for ventricular fibrillation b) for ventricular fibrillation, DC shock should start at 50 joules c) calcium should not be given intravenously to a patient having haemodialysis d) bicarbonate should be given every 10 minutes e) adrenaline should be given in a dose of 0.5 mg via the endotracheal tube 8. Regarding damage to the vestibular apparatus caused by gentamicin: a) it is irreversible b) it does not occur with oral gentamicin c) calorie testing is normal 9. The laryngeal mask may be useful for: a) ventilation b) resuscitation c) intermittent positive pressure ventilation d) microlaryngoscopy e) ophthalmic anaesthesia 10. During a long operation, reliable monitors of core temperature include: a) temperature probe at the tympanic membrane b) temperature probe in the rectum c) quadriceps muscle temperature d) temperature probe in the nasopharynx e) temperature probe in the oesophagus at the level of the cricoid 11. Vitamin B12 injections are appropriate for anaemia in: a) vegans b) Crohn's disease

797

c) phenytoin therapy d) alcoholism e) post-gastrectomy 12. Following head injury, signs which suggest the need for urgent craniotomy include: a) reduced conscious level b) dilated pupil c) hypotension d) convulsions e) cerebrospianl fluid rhinorrhea (persistent) f) depressed # skull 13. A patient with paraplegia of recent onset with injury at T4 may have: a) hypotension on intermittent positive pressure ventilation b) adductor spasm c) bradycardia d) hypothermia e) urinary retention 14. Side-effects of hiatus hernia are: a) iron deficiency anaemia b) aspiration c) oesophageal stricture d) Mallory-Weiss tear e) gastric carcinoma 15. A large atrioventricular shunt causes: a) increased cardiac output b) increased total systemic vascular resistance c) cold extremities d) tachycardia e) heart failure 16. Sympathectomy may be used in the treatment of: a) Renaud's disesase b) causalgia c) hyperhydrosis d) venous ulcers e) pain of intermittent claudication 17. Methods of anaesthetising the larynx include: a) topical anaesthesia to the cords b) anaesthesia of the superior laryngeal nerve through the thyrohyoid membrane c) local anaesthesia injected through the cricothyroid membrane

798

d) local anaesthesia applied to the pyriform fossa e) recurrent laryngeal nerve block 18. Breathing 50% nitrous oxide in oxygen for 3 days causes: a) megaloblasts in bone marrow b) megaloblastic anaemia c) peripheral neuropathy d) inhibition of vitamin B12 synthesis e) reduced number of lymphocytes 19. Cyanosis at birth occurs in: a) Fallot's tetralogy b) transposition of the great vessels c) pulmonary stenosis (isolated) d) patent ductus arteriosus e) ventricular septal defect 20. In aortic regurgition: a) there is a pressure gradient across the valve b) angina is only a feature if it is associated with ischaemic heart disease c) the murmur is best heard at the apex, with the patient leaning to the left d) pistol shots may be heard over the femoral arteries 21. Methods of rapidly lowering serum potassium include: a) glucose and insulin b) calcium gluconate c) frusemide d) bicarbonate e) calcium resonium resin 22. A man collapses 72 hours after a total gastrectomy. The following measurements are made: Temperature, 39 degrees C Blood pressure, 80/30 mmHg Pulse, 110 bpm Central venous pressure, +2 mmHg The diagnoses may include: a) septicaemia b) myocardial infarction c) atelectasis d) breakdown of anastomosis e) haemorrhage f) tamponade

799

23. Concerning streptokinase: a) it should not be repeated within 3 months b) it should not be given within 24 hours of an operation c) it is a tissue plasminogen activator d) it has been shown to be the most clinically useful thrombolytic agent 24. Regarding reading blood pressure by an automatic non-invasive method: a) it may over-read at high pressures b) it may over-read at low pressures c) it is affected by arrhythmias d) it may cause ulnar nerve damage e) the width of the cuff should be half the circumference of the arm 25. Treatment of overdose with amitriptyline may include: a) beta-blockade b) digitalisaion c) isoprenaline infusion d) atropine 26. Penicillins: a) are all inactivated by penicillinase b) are bacteriostatic c) at on cell wall synthesis 27. The following may attenuate the pressor response to intubation: a) enalapril b) intravenous lidocaine c) topical lidocaine to the cords d) a large dose of induction agent e) beta-blockade 28. The following cross the first rib: a) the vagus nerve b) the subclavian artery c) the supratentorial membrane d) the T1 nerve root e) the sympathetic chain 29. Concerning an interscalene approach to the brachial plexus block: a) it is at C6 b) the interscalene groove is accentuated by taking a deep breath c) pneumothorax is rare d) a total spinal may result

800

30. Midazolam is: a) metabolised to active metabolites b) acetylated in the liver c) antagonised by flumazenil d) able to produce anterograde amnesia e) water soluble 31. Concerning coagulation: a) continued bleeding from a venepuncture site suggests haemophilia b) in von Willebrand's disease, the platelet count is normal c) prothrombin time is normal in factor 5 deficiency d) prothrombin time is normal in factor 8 deficiency e) The reptilase time is abnormal if a patient is heparinised 32. Concerning stellate ganglion block: a) oesophageal perforation may occur b) diaphragmatic paralysis may result c) Horner's syndrome is essential for a successful block d) a successful block abolishes a change in skin resistance on stimulation e) a vasovagal reaction can occur while performing it 33. The following must be blocked for mid-thigh amputation: a) femoral nerve b) lateral cutaneous nerve of the thigh c) oburator nerve d) sciatic nerve e) genitofemoral nerve f) both femoral and sciatic nerves g) post-cutaneous nerve 34. Concerning pressure cycled ventilators: a) end-inspiratory pressure is determined by the patient's compliance b) they are usually cycled from inspiration to expiration by time c) they may be cycled from expiration to inspiration by time d) they are more useful for patients with airways obstruction if operated at low flows e) they are minute volume dividers 35. Concerning airway pressures: a) peak and plateau pressures increase in patients with pneumothorax b) they are high in obstructive airways disease 36. In the elderly:

801

a) chest wall compliance is decreased b) vital capacity is decreased by 20 ml each year c) closing volume is less than functional residual capacity d) PaO2 is lower than in the young e) in a 70-year-old, the alveolar/arterial oxygen difference is about 2.7 kPa 37. In a patient of 2 years old: a) fluid replacement is 100 ml/kg/day b) lung compliance is lower than in an adult c) the narrowest point of the airway is the cricoid cartilage d) blood volume is 50 ml/kg 38. A solitary nodule in the thyroid may be: a) a thyroglossal cyst b) Hashimoto's thyroiditis c) the larges nodule of a multinodular goitre d) a physiological goitre e) a colloid goitre 39. The following may be useful indications in posterior fossa surgery: a) a change in respiratory rate b) arrhythmias c) hypertension d) delta waves on the EEG e) a mill-wheel murmur 40. Desmopressin acetate may be: a) useful for bleeding post-cardiopulmonary bypass b) used to treat diabetes insipidus c) used in primary noctural enuresis d) used to treat bleeding peptic ulcers ANSWERS 1.TTFFF 2.TTFT 3.TTFTF 4.TFTFT 5.TFTTT 6.TTFTT 7.FTFFF 8.TTF 9.TTTTT 10.TFFFF 11.TTFTT 12.FTFFTT

802

13.TFTTT 14.TTTFF 15.TFTTT 16.TTTFT 17.TTTTT 18.TFFTT 19.TTFTF 20.FFFT 21.TFFFF 22.TFFTFF 23.TTTF 24.FTTTT 25.TFFF 26.FFT 27.TTTTT 28.FTFTT 29.TTTT 30.FFTTT 31.FTFTF 32.TTTTT 33.TTFFFFT 34.FTTFF 35.TT 36.TTFTT 37.TTTF 38.TFTFF 39.TTTFF 40.TTTF

MCQ E 1. Right lower lobe collapse is characterised by: a) increased A-a difference b) tachypnoea c) increased PCO2 d) decreased arterial pH e) an area of stony dullness 2. A haemoglobin of 8 g/dl and a reticulocyte count of 10% may occur in: a) aplastic anaemia b) haemolytic anaemia c) iron deficiency anaemia d) pernicious anaemia e) acute leukaemia 3. Hypokalaemia may occur with:

803

a) spironolactone treatment b) IV digoxin c) compound sodium lactate infusion d) metabolic acidosis e) intermittent positive pressure ventilation with hypocapnia 4. In gout: a) an acute attack may be precipitated by surgery b) erythrocyte sedimentation rate is normal in an acute attack c) urate deposits occur in the kidney d) cystic lesions on X-rays are urate-filled deposits e) family history occurs in less than 25% of patients with the primary variety 5. Complications of propanolol treatment are: a) congestive cardiac failure b) bronchospasm c) retinal degeneration d) hyperglycaemia e) tachycardia 6. Signs of digoxin toxicity may occur with: a) hyperkalaemia b) hypercalcaemia c) hypomagnesaemia d) acute respiratory alkalosis e) mannitol infusion 7. In the cardiovascular system: a) sudden atrial fibrillation in a patient with mitral stenosis may result in pulmonary oedema b) dilatation of the right lower lobe veins is a reliable sign of pulmonary hypertension c) tricuspid regurgitation is commonly of rheumatic origin d) cachexia is a common feature of cardiac failure e) cardiac failure due to aortic stenosis maybe irreversible due to myocardial fibrosis 8. Preoperative pacing is required in: a) Wolf-Parkinson-White syndrome b) first-degree heart block c) type two second degree heart block d) third-degree heart block e) cardiogenic shock with sinus rhythm

804

9. ECG interference: a) is reduced by screening of the leads b) is reduced by main frequency of 100 Hz c) is reduced by differential amplifiers d) is reduced by decreased skin resistance e) is reduced by the use of filters 10. Isolated pulmonary stenosis is associated with: a) arterial cyanosis b) parasternal leave c) increased A wave in the jugular venous pressure d) a loud pulmonary segment of the second heart sound e) a systolic murmur and thrill at the left upper sternal edge 11. A full nitrous oxide cyclinder: a) has a pressure of 5100 kPa at 200 degrees C b) has a filling ratio of 0.67 c) should be inverted six times before connection to an anaesthetic machine d) gives a constant pressure during usage e) has its contents estimated by weight 12. In early sepsis syndrome: a) cardiac output is normal b) a decreased white cell count is a poor prognostic sign c) adrenocorticotrophic hormone levels are low d) Insulin is raised e) PaO2 is lowered 13. Features of disseminated intravascular coagulation include: a) a lowered fibrinogen level b) a normal prothrombin time and activated partial thromboplastin time c) heparin is a reasonable treatment following placental abruption d) it may occur secondary to malaria e) it may have a compensated phase with no bleeding 14. The following are features of barbiturate overdosage: a) bullous skin reaction b) myocardial depression c) PaCO2 greater than kPa breathing spontaneously d) aspiration of gastric contents e) hypothermia

805

15. In a child who has been under water for 20 minutes and is unconscious with fixed dilated pupils, immediate management should include: a) barbiturate infusion b) rapid rewarming c) steroid therapy d) hypoventilation e) cardiac massage 16. The saturated vapour pressure of water: a) is 0 at 273 K b) is dependent on altitude c) is barometric pressure when at boiling point d) at 37 degrees C is the same as in blood 17. Supraclavicular block with respect to axillary block: a) is more likely to give blockade of the lateral aspect of the forearm b) gives better blockade of the shoulder c) has a higher incidence of intravascular injection d) gives a better block of muscles of the hand e) has a higher incidence of pneumorthorax 18. The following are good indicators of the severity of an acute severe asthma attack: a) a low PaO2 b) a high PaO2 c) scattered rales and crackles d) pyrexia e) an increased pulsus paradoxus 19. Characteristics of chronic obstructive airways disease are: a) a productive cough b) right-sided heart failure is an early feature c) abnormal arterial blood gases occur early d) breathlessness is an early feature 20. Legionnaires' disease is associated with: a) pneumonia b) encephalopathy c) a relative neutropenia d) liver disease e) conjunctivitis 21. Immediate signs of a massive pulmonary embolus include: a) ischaemic cardiac pain

806

b) pleural effusion c) lower lobe pulmonary venous congestion d) peripheral cyanosis e) haemoptysis 22. Recognised features of motor neurone disease are: a) dysphagia b) dysarthria c) urinary retention d) fasciculation e) dementia 23. In Eisenmengers syndrome: a) hypovolaemia improves R to L shunt b) hypoxia decreases R to L shunt c) a ventricular septal defect is always present d) a decrease in systemic vascular resistance reduces the R to L shunt e) pulmonary hypertension is always present 24. Ways of assessing recovery used are: a) Maddox Wing test b) p deletion test c) Romberg test d) measurement of drug levels in blood e) Stewart scoring system 25. The following effects can result from a stellate ganglion block: a) miosis b) exophthalmus c) stuffy nose d) dilatation of the cochlear artery e) recurrent laryngeal nerve block 26. The following can result in pupillary dilatation: a) trimetaphan b) nitroprusside c) stellate ganglion block d) 0.6 mg of atropine IV e) ecothiopate 27. A 3 in 1 block blocks: a) ilioinguinal nerve b) femoral nerve c) genitofemoral nerve

807

d) obturator nerve e) lateral femoral cutaneous nerve 28. Amniotic fluid embolism can cause: a) bronchospasm b) bleeding c) peripheral cyanosis d) pulmonary hypertension e) hypertension 29. The birth canal is innervated by: a) pudendal nerve b) femoral nerve c) obturator nerve d) ilioinguinal nerve e) genitofemoral nerve 30. A patient with infective endocarditis suddenly becomes dyspnoeic, with a blood pressure of 130/50 mmHg. The venous pressure is raised and systolic and diastolic murmurs become more pronounced. Which of the following may have occurred: a) rupture of an aortic cusp b) sudden occurrence of a ventricular septal defect c) inferior myocardial infarct d) late reaction to the antibiotics e) pulmonary embolism 31. Concerning hip arthroplasty: a) methyl-methacrylate is a cardiac inotrope b) hypoxia may be caused by marrow embolisation c) regional techniques give rise to a greater overall survival rate d) subcutaneous heparin will completely prevent deep vein thrombosis e) hypocapnia produced intermittent positive pressure ventilation is beneficial to the patients 32. Concerning propofol: a) it has no active metabolites b) it can cause convulsions c) it causes less nausea than thiopentone d) it causes pain on injection more often than midazolam e) it can cause metabolic alkalosis 33. Concerning alfentanyl: a) it has a higher volume of distribution than fentanyl b) it has a longer elimination half life than fentanyl c) it is more potent than sufentanil

808

d) it is more protein bound than fentanyl e) it can cause bradycardia 34. Concerning the minimum alveolar concentration of isoflurane: a) it decreases with age b) it is decreased with acute alcohol intake c) it is lower in men than in women d) it is higher in neonates compared with a two year old e) it is decreased in pregnancy 35. Causes of prolonged postoperative unconsciousness: a) acromegaly b) an intracerebral event during the operation c) myxoedema d) prolonged action of muscle relaxants e) hypoventilatoin 36. Concerning postoperative nausea and vomiting: a) it is more common in women than in men b) the incidence is 80% with general anaesthesia c) it is more common with thiopentone that with propofol d) butyrophenones can decrease the incidence e) it is more common with ear surgery 37. Concerning day case surgery: a) only ASA grade one patients are suitable b) the operation should be done in such a way that no postoperative opioids are needed c) the patient should be accompanied home with an adult d) a laparoscopic procedure is not suitable e) intubation is not appropriate 38. Agents used to decrease the pressure response to intubation include: a) ACE inhibitors b) calcium antagonists c) thiopentone d) beta-blockers e) fentanyl 39. The occurence of bronchospasm at the end of an operation to repair an umbilical hernia in an asthmatic child can be caused by: a) light anaesthesia b) irritation of a tracheal tube c) morphine in the premedication d) isoflurane

809

e) reversal by neostigmine 40. During one-lung anaesthesia, the PaO2 is influenced by: a) the amount of blood flow in the upper lung b) the cardiac output c) the mixed venous oxygen concentration d) the haematocrit e) the FIO2 41. In patients with pacemakers: a) diathermy use should be avoided b) hypovolaemia is poorly tolerated c) electrolytes should be normalised prior to surgery d) suxamethonium should be avoided e) use of volatile agents can cause deterioration of function 42. Hyponatraemia may cause the following: a) confusion b) hypertension c) convulsions d) coma e) tachycardia 43. A complete block of the ulnar nerve at the elbow will cause: a) numbness on the ulnar side of the forearm b) paralysis of the hypothenar muscles c) paralysis of all thenar muscles d) sensory loss over the little finger e) sensory loss over the lateral side of the ring finger 44. Positive end-expiratory pressure can cause: a) sodium retention b) decreased cardiac output c) increased residual volume d) decreased functional residual capacity e) decreased central venous pressure 45. The following features are essential to diagnose malignant hyperthermia: a) muscle rigidity b) hypercapnia c) renal failure d) body temperature greater than 38 degrees C e) family history

810

46. Trigeminal nerve block is associated with anaesthesia of: a) lower lip b) angle of the mandible c) soft palate d) tip of the nose 47. Hypokalaemia can be caused by: a) Addisons disease b) major burns c) alkalosis d) triamterene therapy e) diarrhoea 48. Gram-negative septicaemic shock is associated with: a) urine output (0.5 ml/kg/h) b) disseminated intravascular coagulation c) hypotension unresponsive to fluid loading d) high fever e) diminished cardiac output 49. Laryngeal motor innervation is from the following: a) glossopharyngeal nerve b) internal laryngeal nerve c) recurrent laryngeal nerve d) hypoglossal nerve e) superior laryngeal nerve 50. Uveitis is a feature of: a) ulcerative colitis b) rheumatoid arthritis c) Crohns disease d) systemic lupus erythematosus ANSWERS 1.TTFFF 2.FTFFF 3.FF?FT 4.TFTTF 5.TTFFF 6.FTTTF 7.TFFFT 8.FFTTF 9.TFTTT 10.FTTFT

811

11.FTFFT 12.TTFTT 13.TFFTT 14.TTTTT 15.FTFFT 16.TFTF 17.TTFFT 18.TTFFT 19.TFFF 20.TTFTF 21.TFFTF 22.TTFTT 23.TFFFT 24.TFFFT 25.TFTTT 26.TFFTF 27.FTFTT 28.TTTTF 29.TFFTT 30.TTFFF 31.FTFFF 32.TTTTF 33.FFFTT 34.TTFFT 35.TTTTT 36.TFTTT 37.FFTFF 38.TTTTT 39.TTFFT 40.TTTTT 41.TTTTF 42.T?TTF 43.FTFTF 44.TTTFF 45.FTFTF 46.TFFT 47.FFTFT 48.TTTFF 49.FFTFT 50.FTTT

MCQ F 1. Mitral stenosis is associated with: a) subacute bacterial endocarditis b) rheumatic fever c) pulmonary hypertension d) systolic murmur at the apex

812

2. Parathyroid adenoma may present with: a) psychiatric disturbance b) increased serum calcium c) decreased urinary calcium d) abdominal pain e) renal stones 3. Patients with malignant hypertension may have: a) increased risk of pulmonary oedema b) decreased creatinine clearance c) proteinuria d) right bundle branch block e) pulmonary hypertension 4. In the following conditions, dyspnoea is mainly due to decreased compliance: a) extensive pulmonary consolidation b) left ventricular failure c) massive pulmonary embolus d) emphysema 5. An increased A-a 02 difference is caused by: a) decreased functional reserve capacity b) increased inspired 02 c) N20 absorption d) hepatic failure e) Increased V/Q mismatch 6. There is no heart rate response to a Valsalva manoeuvre in: a) aortic incompetence b) patients on beta blockers c) autonomic neuropathy d) Horner's syndrome e) increased V/Q mismatch 7. Acute pulmonary oedema can occur with: a) aortic stenosis b) tricuspid incompetence c) myocardial infarction d) left atrial myxoma e) massive pulmonary embolus

813

8. Tension pneumothorax is associated with: a) mediastinal displacement away from the lesion b) decreased percussion note on the side of the lesion c) increased blood pressure d) stridor e) cyanosis 9. Atracurium: a) may cause profound bradycardia b) activity varies with pH c) commonly causes histamine release when used in clinical doses d) causes vagal blockade e) must not be used in liver failure 10. The laryngeal mask airway: a) ensures an unobstructed airway b) is safe to use in patients with a full stomach c) can be used with positive pressure ventilation d) can be inserted easily after induction with thiopentone e) is not associated with laryngospasm 11. Vaporisers-inside-circuit: a) are efficient b) are temperature compensated c) must be placed in the inspiratory limb of the circle d) have a small volume e) have a low resistance 12. In a pregnant woman at term: a) tidal volume is increased b) functional residual capacity is increased c) physiological dead space is decreased d) total vital capacity is reduced e) airway resistance is reduced. 13. In labour: a) entonox decreases placental blood flow b) bupivacaine crosses the placenta freely c) prilocaine causes methaemoglobinaemia in the foetus d) epidural analgesia increases delay in second stage 14. The obstetrician calls you for a stuck second twin diagnosed following administration of ergometrine. The following drugs are capable of relaxing the uterus:

814

a) halothane b) thiopentone c) suxamethonium d) salbutamol e) ritodrine 15. Suxamethonium significantly increases serum potassium concentrations in: a) quadriplegia b) myasthenic syndrome c) malignant hyperpyrexia syndrome d) adrenocortical insufficiency e) tetanus 16. Regarding the Bain circuit: a) inspiratory gas passes through the outer tube b) it can be used in small children c) when the inner tube is disconnected, a large increase in dead space is inevitable 17. Diazoxide is used in the treatment of pre-eclampsia because: a) it causes a diuresis b) given intravenously it cause hypotension c) it prevents hypoglycaemia d) it increases uterine contractility e) it provides useful sedation 18. Concerning the epidural space: a) it commences at the foramen magnum b) it is triangular in shape c) in the lumbar region, it is widest posteriorly d) no more than 20 ml of 0.5% bupivacaine should be used for a caesarean section e) for a caesarean section, the block should extend from T11 to S1 19. Complications of stellate ganglion block include: a) pneumothorax b) dural puncture c) injection into the vertebral artery d) damage to the phrenic nerve e) damage to the vagus nerve 20. Renal failure is associated with: a) normochromic normocytic anaemia b) acidosis c) hypernatraemia

815

21. The caudal space: a) allows entry into the epidural space b) contains the spinal cord c) is covered by the sacrococcygeal membrane 22. Concerning automatic blood pressure devices: a) they over-read at high systolic blood pressure b) they over-read at low systolic blood pressure c) they can cause ulnar nerve damage d) they are affected by arrhythmias e) the width of the cuff should be half the circumference of the arm f) they can cause ulnar nerve palsy 23. Concerning the Severinghaus electrode: a) sodium bicarbonate is the electrolyte solution b) it contains CO2 sensitive glass c) it is affected by temperature d) it is affected by nitrous oxide e) it is more accurate with blood than with gases 24. Helium: a) is less viscous than oxygen b) alters the voice c) can be used in the treatment of bronchospasm d) supports combustion e) is a liquid stored in brown cylinders 25. Recognised causes of urinary retention include: a) ketamine b) morphine c) amitryptyline d) ephedrine e) frusemide 26. Regarding pulmonary artery catheters in a normal person: a) the wedge pressure is about 12 mmHg b) the pulmonary artery pressure is about 20/5 mmHg c) the central venous pressure is about 5 cm H2O d) the right ventricular pressure is about 30/0 mmHg e) the internal jugular to wedged distance is about 70 cm 27. Regarding brain stem death: a) a diagnosis can be made during a convulsion

816

b) criteria cannot be met in the presence of spinal reflexes c) an EEG is required for the diagnosis d) glucose has to be normal 28. Typical total parenteral nutrition requirements for a 70 kg man include: a) 14 g of nitrogen b) glucose c) magnesium d) 1 ml of water for 1 kcal of energy e) no intralipid in hepatic failure 29. Regarding intermittent mandatory ventilation: a) it must have a mandatory breath rate of 8-10 bpm b) it should be used with positive end-expiratory pressure c) all breaths are triggered by the patient d) it is synchronised to the patients expiration e) it gives an abnormal end tidal CO2 result 30. A pressure cycled ventilator: a) is a minute volume divider b) is often cycled from expiration to inspiration by time c) is cycled from inspiration to expiration after a set time d) has end-expiratory pressure determined by the patients compliance e) ventilates a patient more efficiently with obstructive airways disease if operated at low flows 31. Concerning exponential decay: a) time constant equals a half life b) 3 x time constant equals 97% c) the rate of change is proportional to the quantity at a certain time d) at one time constant, 37% remains e) the time constant is the time at which the process would have been complete, had the initial rate of change continued ANSWERS 1.TTTF 2.TTFTT 3.TTTTT 4.TTFF 5.TTTTT 6.FTTFF 7.FFTTF 8.TFFFT 9.TTTFF 10.FFTFF 11.FFFTT

817

12.TFTFF 13.?TTT 14.TTFTT 15.TFTFT 16.FFT 17.FTTFF 18.TTFFF 19.TTTTT 20.TTF 21.TFT 22.FTTTTT 23.TFTFT 24.FTFFF 25.FTTTF 26.TFTFF 27.FFFT 28.TTTTF 29.FFFFF 30.FTTFF 31.FTTTT

MCQ G 1. The following increase intraocular pressure in the normal eye: a) hypercarbia b) acetazolamide c) atropine d) hypotension e) respiratory obstruction 2. Intraluminal gastrointestinal pressure is increased by: a) nitrous oxide anaesthesia b) epidural block to T6 c) neostigmine d) morphine e) adrenaline 3. In the elderly: a) systolic hypertension is common b) ventilatory response to CO2 is normal c) PaO2 is lower than in young adults d) upper airway reflexes are impaired e) postoperative analgesic requirements are increased

818

4. In one lung anaesthesia, PaO2 depends on: a) the amount of blood flowing through the non-ventilated lung b) FiO2 c) intraoperative haematocrit d) cardiac output e) mixed venous oxygen tension 5. Regarding bupivacaine: a) it causes cardiac toxicity before convulsions b) more than 50% is ionised at physiological pH c) more than 20% is excreted unchanged d) it is less potent than prilocaine e) it crosses the placenta more slowly than lidocaine 6. For the same amount of lidocaine (mg), a greater segmental block is achieved: a) in the pregnant near term b) in the elderly c) with higher concentration d) with rapid injection e) with addition of 1:200,000 adrenaline 7. Regarding epidural opioids: a) less than 2-3% morphine crosses into the CSF b) pethidine crosses into the CSF faster than morphine c) prior injection of local anaesthetic decreases the amount of morphine that crosses into the CSF d) relatively more diamorphine gains access to the systemic circulation than morphine e) delayed respiratory depression does not occur with the very fat soluble opioids 8. Regarding spinal anaesthesia: a) the spinal cord terminates at L4 b) barbotage increases the spread of subsequent block c) aqueous 0.5% bupivacaine is hyperbaric d) bradycardia may be treated with IV metaraminol e) high block impairs the ability to cough 9. The management of air embolism during posterior fossa surgery may include: a) positioning the patient on the right side b) administration of mannitol c) raising cerebral venous pressure d) discontinuation of nitrous oxide e) rapid infusion of fluid

819

10. Regional cerebral metabolism is increased by: a) halothane b) mannitol c) pain d) ketamine e) sodium thiopentone 11. Regarding the flow-directed multi-lumen PA catheter: a) the thermistor is situated 20 cm from the tip b) pulmonary capillary wedge is a small bronchopulmonary segment c) cardiac estimations can be performed by pressure measurement d) an open central lumen when balloon is inflated protects against distal infarction e) it is more accurate when inserted via the internal jugular vein than via the subclavian 12. Helium: a) is less viscous than oxygen b) is stored as a liquid in cylinders c) is used to decrease the work of breathing in bronchospasm d) changes the voice e) supports combustion 13. Regarding ankylosing spondylitis: a) it is more common in females than males after the age of 40 b) it is associated with bilateral hip arthropathy c) it is associated with iritis d) low grade pyrexia may be present e) may present with sciatica 14. Regarding the trachea: a) the only blood supply is from the bronchial arteries b) it starts at C4 c) it ends at T5 d) it is lined by transitional epithelium e) it is 1.5 - 2 cm wide in the adult 15. Minimum alveolar concentration: a) is greater in men than women b) decreases with age c) is lower in the neonate compared with a 2 year old d) is lower in pregnancy e) is decreased with acute alcohol intoxication

820

16. Spinal versus general anaesthesia for repair of fracture of neck of femur: a) decreases mortality b) reduced hospital stay c) decreases the incidence of thromboembolism d) provides better immediate postoperative pain relief e) decreases intraoperative blood loss 17. Regarding the Severinghaus electrode: a) the electrolyte is sodium bicarbonate b) it has CO2 sensitive glass c) it is affected by temperature d) it is more accurate for blood than gas sample anaylsis e) it is affected by nitrous oxide 18. In arterial blood gases: a) the respiratory quotient is affected by parenteral nutrition b) (A-a) O2 at FiO2 1 is less than 15 kPa normally c) more than usual amounts of heparin increase pH d) mixed venous pH is equal to arterial pH e) dissolved oxygen increases with hypothermia 19. In the anabolic phase after surgery: a) calories should be supplied in a ratio of 150:1 g nitrogen b) there is formation of 30 g protein per day c) potassium excretion increases d) tissue fats are mobilised 20. ACE inhibitors cause: a) cough b) sodium and water retention c) vasodilation d) tachycardia 21. In the anterior interscalene approach to the brachial plexus: a) it is performed at C6 level b) interscalene groove is accentuated by deep inspiration c) Horners syndrome is essential for a successful block d) total spinal anaesthesia may result e) pneumothorax is rare 22. In a 2 year old child: a) the narrowest point of the trachea is the cricoid ring b) fluid requirements are 100 ml/kg/day

821

c) blood volume is 50 ml/kg d) chest wall compliance is decreased compared with the adult e) there is increased platelet function 23. In a patient with untreated megaloblastic anaemia: a) histamine-fast achlorhydria is always present b) urine methyl malonate is increased c) there is an association with gastric cancer d) resection of ileal loops for Crohns disease may be a causative factor e) serum B12 levels may be normal 24. Prolonged exposure to 50% nitrous oxide for 3 days causes: a) megaloblastic marrow change b) peripheral neuropathy c) lymphopenia d) decreased methionine synthetase activity e) increased sensitivity to suxamethonium 25. Glycosuria may occur: a) in pregnancy b) in phaeochromocytoma c) in hypopituitarism d) in partial gastrectomy e) after subarachnoid haemorrhage 26. Hypothyroidsm is associated with: a) supraventricular tachycardia b) slow relaxation of tendon reflexes c) low serum cholesterol d) high serum protein-bound iodine 27. Regarding the radial nerve: a) it supplies the extensor compartment of the forearm b) damage is associated with sensory loss over the lateral aspect of the forearm only c) damage is associated with alcoholism d) it can be damaged by pressure on the lateral epicondyle of the humerus 28. Tissues which can rapidly regenerate after damage include: a) hepatocytes b) renal glomerular cells c) anterior horn cells d) aortic smooth muscle e) epithelial cells at the edge of a peptic ulcer

822

29. A drug that blocks dopamine receptors only will: a) be anti-arrhythmic b) decrease renal perfusion c) improve gut blood flow d) improve Parkinsons disease e) prevent motion sickness 30. Goldman criteria include: a) previous cardiac surgery b) mitral valve disease c) hypertension d) atrial fibrillation e) previous myocardial infarction 31. APACHE II utilises: a) emergency surgery b) haemoglobin c) admission after cardiac arrest d) (A a) O2 32. Coarctation of the aorta: a) is usually preductal b) is associated with cerebral aneurysms c) may be associated with differential cyanosis d) may need left heart bypass during correction e) is repaired via a right thoracotomy 33. During cardiopulmonary resuscitation: a) lidocaine should be given before adrenaline in ventricular fibrillation b) DC cardioversion with 50 J is the treatment of choice for ventricular tachycardia c) adrenaline 0.5 mg should be given via endotracheal tube d) 50 ml 8.4% bicarbonate should be given every 10 minutes e) IV calcium is indicated for patients on dialysis 34. Hypercalcaemia can be caused by: a) vitamin D deficiency b) vitamin D intoxication c) decreased serum calcitonin d) primary hyperparathyroidism e) oral potassium chelators 35. Alkaline phosphatase is increased in: a) osteitis deformans (Pagets disease)

823

b) multiple chondromata c) hyperparathyroidism d) prostatic carcinoma in situ e) liver disease 36. Creatinine phosphokinase: a) is raised 24 hours after myocardial infarction b) untreated hyperthyroidism c) osteomalacia d) duchenne muscular dystrophy e) myasthenia gravis 37. Serum sodium 125 mmol/L and serum potassium 6.25 mmol/L can be found in: a) renal failure b) Addisons disease c) hypoaldosteronism d) liver tension e) potassium deficiency 38. Serum urea of 20 mmol/L is consistent with: a) dehydration b) A gastrointestinal tract bleed c) severe hepatic impairment 39. Atrophy of the tongue occurs in: a) myotonia b) syringobulbia c) pseudobulbar palsy d) Parkinsons disease e) Multiple sclerosis 40. Metabolic acidosis is present in: a) renal failure b) respiratory acidosis c) ureterocolic anastomosis d) severe vomiting e) diarrhoea 41. In the oliguric phase of acute tubular necrosis: a) small volumes of concentrated urine are produced b) there is hyperkalaemia c) there is malignant hypertension d) there is a progressively rising ventral venous pressure e) there is a raised serum urea with a normal serum creatinine

824

ANSWERS 1.TFTFT 2.TTTFF 3.TFTTF 4.TTFTT 5.FTFFT 6.TTTFF 7.?FFTT 8.FFFFT 9.FFTTT 10.FFTTF 11.FFTFF 12.FFFTF 13.FTTTT 14.FFTFT 15.FTTTT 16.FFTTT 17.TFTTF 18.TTFFT 19.?TTT 20.TFTT 21.TTFTT 22.TTFFF 23.FTTTT 24.TFTTF 25.TTFTT 26.FTF? 27.TFTF 28.TFFFT 29.FTFFF 30.FFFTT 31.TTTTT 32.FTFTF 33.FTFFT 34.FTTTT 35.TFTFT 36.TFFTF 37.TTTTF 38.TTF 39.FTTFT 40.TFTFT 41.FTFTF

825

MCQ H 42. In severe hepatocellular failure: a) albumin levels are low b) fibrinogen levels are low c) prothrombin time is shortened d) plasma sodium levels are raised e) bilirubin is raised 43. In severe hepatic impairment, drug pharmacokinetics are altered by: a) decreased protein binding b) change in body fluid compartments c) competition for protein binding sites with toxins d) increased lipid solubility of the drug itself e) decreased albumin levels 44. Regarding gentamicin vestibular toxicity: a) it is reversible b) it is more common in the elderly c) it does not occur after oral ingestion d) caloric tests are normal e) it is more likely to occur with liver disease 45. In the diagnosis of brain stem death: a) consultation with a neurologist is needed b) an EEG must be flat for 24 hours c) convulsions preempt the diagnosis d) spinal reflexes may be present e) blood must be sent for drug screening 46. Warming blood to 37 degrees C during massive blood transfusion: a) decreases the risk of citrate toxicity b) increases plasma potassium concentration c) increases plasma carbon dioxide tension d) decreases the incidence of arrhythmia e) increases CO2 buffering capacity of cells 47. Percutaneous absorption occurs with: a) atropine b) hyoscine c) morphine d) fentanyl e) paracetamol

826

48. Salicylate overdose is associated with: a) fibrinolysis b) hypothermia c) respiratory acidosis d) polyuric renal failure e) massive gastric bleeding 49. Non-steroidal analgesic agents: a) can be given intrathecally b) cause thrombocytopenia c) block prostaglandin synthetase d) cannot be used in gastrointestinal tract disease 50. Regarding stellate ganglion block: a) it causes deafness b) it may cause phrenic nerve paralysis c) oesophageal perforation may occur d) a vasovagal reaction may occur during its performance e) if successful, it causes a change in skin resistance on stimulation 51. You are asked to attend to a woman, 30 minutes after delivery, who is fitting. The causes for this may be: a) eclampsia b) local anaesthetic toxicity c) grand mal epilepsy d) hyperventilation e) cerebral thrombosis 52. Malignancy is associated with: a) familial polyposis coli b) ulcerative colitis c) villous papilloma d) chronic constipation e) chronic diverticulitis 53. Regarding the measurement of an anaesthetic agent: a) infrared measurement is agent specific b) mass spectrometry is not agent specific c) nitrous oxide interferes with paramagnetic analysers d) acoustics can be used for measurement e) infrared analysers pick up isopropyl alcohol

827

54. Risks of electrocution are decreased by: a) conductive flooring b) the use of fuses c) the use of isolating transformers d) the use of battery power e) each part of the apparatus being attached to separate earth sockets 55. Haematuria may occur in: a) rheumatic fever b) bacterial endocarditis c) hydronephrosis d) prostatic hypertrophy e) acute glomerulonephritis 56. The following increase lower oesophageal sphincter tone: a) metoclopramide b) atropine c) cisapride d) domperidone e) morphine 57. Inhalation of gastric contents will produce: a) pulmonary oedema b) bronchoconstriction c) left ventricular failure d) hypovolaemia e) reduced activity of surfactant 58. Muscle relaxant reversal may be difficult in the presence of: a) hyperkalaemia b) fentanyl overdose c) hypermagnesaemia d) hypocapnia e) acidosis 59. Extrapyramidal effects can be seen following: a) chlorpropamide b) terfenadine c) metoclopramide d) domperidone e) perphenazine

828

60. The following statements are true: a) Cerebrospinal fluid (CSF) should be examined if a brain tumour is suspected b) CSF is characteristically normal in trigeminal neuralgia c) CSF protein greater than 1.5 g may be found in motor neurone disease d) CSF protein is raised in untreated meningococcal meningitis e) CSF cell count is increased in Guillain-Barre syndrome 61. Treatment for epilepsy includes: a) ethosuximide b) sodium valproate c) clonazepam 62. Lung compliance is increased in: a) the presence of intra-alveolar fluid b) acute respiratory distress syndrome c) idiopathic pulmonary fibrosis d) emphysema e) fibrosing alveolitis 63. In a pregnant woman at term: a) tidal volume is increased b) functional residual capacity is increased c) physiological dead space is decreased d) total vital capacity is reduced e) airway resistance is reduced 64. Suxamethonium significantly increases serum potassium concentration in: a) quadriplegia b) myasthenic syndrome c) malignant hyperpyrexia d) adrenocortical insufficiency e) tetanus 65. Coeliac plexus block: a) can be used to relieve pain from intra-abdominal malignancy b) may cause orthostatic hypotension c) may cause diarrhoea d) can be used to treat pain in acute pancreatitis e) causes constriction of the sphincter of Oddi 66. Breathing 10% oxygen in nitrogen in a fit individual causes: a) central cyanosis b) increased cardiac output

829

c) reduced stroke volume d) reduced mixed venous oxygen content e) angina pain 67. The use of oral antibiotics in the preparation for colonic surgery may cause: a) fungal infection of the bowel b) diarrhoea c) fulminant enterocolitis d) altered response to muscle relaxants e) hepatic failure 68. A raised reticulocyte count is found in: a) untreated pernicious anaemia b) aplitic anaemia c) untreated iron deficiency anaemia d) chronic lymphocytic anaemia e) congenital spherocytic anaemia 69. Regarding a prolapsed disc at L5 S1: a) it is associated withscoliosis b) it is associated with loss of sensation on the medial side of the right calf c) it is associated with loss of knee jerk d) onset of urinary incontinence requires urgent surgical treatment e) a plaster cast is the treatment of choice 70. Ulcerative colitis is associated with: a) cirrhosis b) cholangitis c) clubbing d) iritis e) arthritis 71. Neuropraxia: a) is more common after long operations b) does not occur with local anaesthetics c) does not occur with muscle relaxants d) only occurs when previous neuropathy is present e) takes a long time to recover 72. Ritodrine can cause: a) bradycardia b) heart block c) left ventricular failure d) hypotension

830

e) peripheral vasoconstriction 73. Pressure cycled ventilators: a) are minute volume dividers b) are cycled from inspiration to expiration after a set time c) may be cycled from expiration to inspiration after a set time d) have an end-expiratory pressure determined by the patients compliance e) ventilate a patient with chronic airways obstructive disease better if operated at low flows f) are not associated with a risk of barotrauma 74. In the elimination of drugs: a) alkalinisation of urine enhances elimination of weak acids b) oxidation increases elimination of polar compounds c) glucuronidation occurs at the microsomal level d) terminal half life determines drug dose intervals e) a decrease in glomerular filtration rate reduces gentamicin excretion 75. In a circle anaesthetic system: a) the soda lime canister should be equal in volume to the patients tidal volume b) the system will function satisfactorily with a single one-way valve c) if a vaporiser is put in the circle, it should be placed between the inspiratory limb and the fresh gas flow d) the volume of the reservoir bag is not critical e) the adjustable pressure limiting valve should be placed between the expiratory limb and the soda lime 76. Concerning intermittent positive pressure ventilation and airway pressures: a) peak inspiratory pressure decreases if there is a leak b) a pneumothorax increases peak and plateau pressures c) there is an increase in peak pressure with a decrease in compliance d) bronchial constriction causes a rise in plateau pressure e) peak pressures are high in obstructive airway disease 77. Concerning intra-arterial pressure transducers: a) the natural resonant frequency may be determined by square waves at 25 Hz via the intraflow system b) they are best used with long narrow catheters c) a resonant frequency of 10 Hz is ideal d) mean pressure is not affected by damping e) optimum damping occurs at 5 times the resonant frequency 78. The following may attenuate the pressor response to intubation: a) ACE inhibitors b) calcium antagonists

831

c) intravenous lidocaine d) a large dose of induction agent e) beta-blockers 79. Regarding syncope: a) the most common cause is cardiovascular disease b) there may be an association with bradycardia c) it may be preceded by a loss of vision d) it is due to a fall in cardiac output e) it may be indirectly caused by urinary retention 80. Diplopia may occur in: a) myasthenia gravis b) retrobulbar neuritis c) cerebellar hemisphere disease d) Horners syndrome 81. A Wrights respirometer: a) is essentially a turbine b) is affected by viscosity c) is temperature compensated d) is accurate at flows of <1 litre per minute e) over-reads at high flows 82. In the management of the amitryptiline overdose, the following may be useful: a) digitalisation b) beta-blockers c) alkaline diuresis d) atropine e) isoprenaline infusion 83. Infant respiratory distress syndrome: a) usually occurs within 12 hours of birth b) is more common after caesarean section c) has a better prognosis if steroids are given to the infant d) is uncommon after 36 weeks' gestation e) is more common in multiple pregnancies 84. The coeliac ganglion is: a) in front of L3 b) on the crura c) behind the pancreas d) behind the inferior vena cava e) in front of the aorta

832

85. Factors leading to hypothermia include: a) vasodilation b) exposure of abdominal contents c) neuromuscular blockers d) spinal anaesthesia e) dry gases 86. In endotracheal tubes: a) pressure is inversely related to length in laminar flow b) irregularities within the tube increase resistance c) at angled connectors, turbulence increases as gas flow rate increases d) in laminar flow, the pressure drop along the tube is linearly related to length 87. TENS: a) works via A fibres b) is useful for peripheral nerve injuries c) is useful for thalamic pain d) uses voltages of 0 to 50 volts e) uses frequencies of 1 to 100 Hz 88. Intravenous guanethidine: a) is useful in sympathetic dystrophy b) works on the parasympathetic system c) can be repeated d) requires the tourniquet to be applied for an hour e) causes hypotension ANSWERS 42.TTFFT 43.TTT?T 44.FTTFF 45.FFTTF 46.TFFTT 47.FTFTF 48.FFFFF 49.FTTT 50.FTTTT 51.TTTFF 52.TTTFF 53.FFFTT 54.FTTTF 55.FTFTT 56.TFTTF 57.TTFTT

833

58.FTTFT 59.FFTTT 60.FTFTF 61.TTT 62.FFFTF 63.TFTFF 64.TFTFT 65.TTTTF 66.TFTTF 67.?T?TF 68.FFFFT 69.FFFTF 70.TTTTT 71.TFFFF 72.FFTTF 73.FTTFFF 74.TFTTT 75.FFTFT 76.TTTFT 77.TFFTT 78.TTTTT 79.TTTTT 80.TFFF 81.TFFFT 82.FTTFF 83.FFFTT 84.FTTTT 85.TTTTT 86.FTTT 87.TTFFT 88.TFTFT

MCQ I 1. The following agents may cause pulmonary fibrosis: a) bleomycin b) cortisone hemisuccinate c) beryllium d) paraquat e) organophosphate compounds 2. In comparison with an older child, a newborn infant increases pulmonary ventilation by breathing more often rather than more deeply. This is related to: a) total respiratory compliance b) the horizontal disposition of the rib cage c) immature respiratory chemoreceptors d) non-respiratory acidosis found in neonates

834

e) coordination of respiratory effort with sucking movement 3. The most common site of laryngeal granuloma after short-term intubation is: a) the piriform fossa b) the epiglottis c) the anterior 1/3 of the vocal cords d) the posterior 1/3 of the vocal cords e) the trachea 4. Regarding measurement of anaesthetic gases: a) infra-red analysers are specific for agents b) mass spectrometry may not be specific for agents c) infra-red analysers pick up isopropyl alcohol d) acoustic techniques may be used to measure gas concentrations e) nitrous oxide interferes with paramagnetic analysers 5. Regarding myasthenia gravis: a) IgE antibodies are found in 85% of patients b) muscle weakness improves with exercise c) muscle weakness is worsened by gentamicin d) plasma exchange produces rapid remission e) thymectomy is the treatment of choice in patients over 50 years 6. A raised reticulocyte count is found in: a) untreated pernicious anaemia b) aplastic anaemia c) untreated iron deficiency anaemia d) chronic granlocytic leukaemia e) congenital spherocytic haemolytic anaemia 7. Measurement of prothrombin time is helpful in: a) haemophilia b) von Willebrands disease c) scurvy d) jaundice e) idiopathic thrombocytopenic purpura 8. Propofol: a) is suspended in an emulsion of soya bean oil and egg phosphatide b) can produce green urine c) has little effect on the cardiovascular system d) impairs ventilatory response to CO2 e) has no effect on intraocular pressure

835

9. Essential criteria for the diagnosis of brain stem death are: a) equal pupils b) absent dolls head response c) absent limb movements d) patients temperature must exceed 34 degrees C e) PaCO2 must exceed 6.5 kPa at completion of apnoea testing 10. Malignant hypertension is associated with: a) increased risk of pulmonary oedema b) right bundle branch block c) reduced creatinine clearance d) proteinuria e) pulmonary hypertension 11. Coarctation of the aorta: a) is a congenital condition b) produces upper limb hypertension c) may be associated with a displaced apex beat d) produces a diastolic murmur over the precordium e) produces skeletal abnormalities on a plain chest X-ray 12 Routine laboratory investigation of a 42-year-old man reveals an elevated creatinine phosphokinase. This finding may indicate: a) abnormal metabolism of haemoglobin b) the patient is at increased risk of malignant hyperpyrexia under anaesthesia c) hepatocellular dysfunction d) recent exercise e) recent myocardial infarction f) Duchenne muscular dystrophy g) hypothyroidism 13. Febrile collapse following a colectomy may be due to: a) pulmonary embolus b) breakdown of the anastomosis c) haemorrhage d) sepsis 14. pH 7.5, PCO2 7kPa, PO2 12 kPa, B.E. +9.5 These arterial blood gas values could represent: a) pyloric stenosis for several days b) the administration of 50 mmol of bicarbonate following cardiac arrest c) excessive deadspace ventilation d) a severe asthmatic on steroids e) chronic obstructive airways disease

836

15. A young man admitted to casualty following a road traffic accident is found to have central dislocation of the hip and is shocked. Likely causes are: a) ruptured bladder b) ruptured urethra c) blood loss d) neurogenic shock e) fat embolism 16. A patient with vomiting, respiratory distress, cyanosis, epigastric tenderness and subcutaneous emphysema in the neck may be suffering from: a) ruptured oesophagus b) ruptured diaphragm c) ruptured trachea d) spontaneous pneumothorax e) pulmonary embolus 17. The following are features of malignant hyperpyrexia: a) hypoxia b) hypocapnia c) hyperkalaemia d) prolonged neuromuscular blockade 18. In patients with porphyria: a) griseofulvin may precipitate an acute attack b) glycine should not be used during transurethral resection of the prostate c) dysautonomia may occur d) preoperative fluid restriction is beneficial e) fentanyl may safely be used 19. In carcinoid syndrome: a) urine turns pink on standing b) aortic valve lesions frequently occur c) there is an association with obstinate constipation d) diarrhoea and bronchospasm may occur e) severe flushing occurs f) ketanserin, octreotide and aprotonin should be given 20. Regarding anaphylactoid reactions: a) they are clinically indistinguishable from anaphylaxis b) widespread erythema occurs c) adrenaline is the treatment of choice d) they are not commonly caused by etomidate

837

21. For a patient suffering from Parkinsons disease on L-dopa, the following agents should not be used: a) enflurane b) droperidol c) nitrous oxide d) morphine e) fentanyl 22. During cardiopulmonary resuscitation: a) lidocaine should be used before adrenaline in ventricular fibrillation b) calcium is appropriate in patients undergoing haemodialysis c) cardioversion at 50 J is indicated for ventricular tachycardia d) adrenaline 0.5 mg is an appropriate dose for administration via an endotracheal tube e) bicarbonate should be administered every 10 minutes 23. The risk of electrocution may be reduced by: a) conductive flooring b) the use of isolating transformers c) a separate earth for each socket d) the use of fuses e) battery powered equipment 24. Ventricular arrhythmias are more common in the presence of: a) hypokalaemia b) hypoxia c) thyrotoxicosis d) cardiopulmonary bypass and digoxin treatment e) essential hypertension 25. Hypoglycaemic coma: a) occurs with hypothermia b) may cause coma in adrenal failure c) is a recognised complication of untreated thyrotoxicosis d) occurs in normal people after 48 hours of fasting 26. In arterial blood gases: a) heparin in significant amounts will cause an elevated pH b) there is an increase in dissolved oxygen with hypothermia c) when the FiO2 is 1 the alveolar-arterial pressure difference exceeds 15 kPa 27. An adult breathing 100% oxygen at sea level may suffer from: a) retrosternal chest pain b) convulsions

838

c) dizziness d) atelectasis e) permanent visual damage 28. In the measurement of cardiac output using a thermistor: a) the temperature measurement is accurate to 0.1 degree C b) the thermistor is proximal to the balloon c) results are inaccurate after the catheter has been in use for more than 48 hours d) the thermistor measures the true core temperature 29. The following are associated with ulcerative colitis: a) iritis b) cirrhosis c) cholangitis d) arthropathy e) finger clubbing 30. Features of ankylosing spondylitis include: a) it is more common over the age of 40 and in males rather than females b) low grade pyrexia c) sciatica d) hip arthropathy e) iritis 31. In L5-S1 disc prolapse with sciatica in the right leg: a) loss of the knee jerk on the right occurs b) loss of sensation in the medial right calf occurs c) incontinence requires further surgical investigation d) plaster of Paris cast is the preferred early treatment e) it is commonly associated with scoliosis 32. Cauda equine syndrome is more common in: a) old age b) use of epidural adrenaline c) lithotomy position d) cases where barbotage is used e) face down position 33. Suitable anaesthetic techniques for patients with raised intracranial pressure are: a) nitrous oxide, oxygen and fentanyl; controlled ventilation b) nitrous oxide, oxygen, thiopentone and atracurium c) ketamine d) halothane, nitrous oxide and oxygen; spontaneous respiration e) pre-medication with morphine

839

34. Epidural opiates may cause: a) nausea and vomiting b) motor block c) urinary retention d) hypotension e) respiratory depression 35. An elevated left hemidiaphragm may be caused by: a) left pneumothorax b) left pleural effusion c) situs inversus d) left phrenic nerve paralysis e) left lower lobe collapse 36. In formulating total parenteral nutrition for a 30-year-old, 70 kg man recovering from major trauma: a) insulin is not needed if less than ? of glucose per day is used b) 6000 kcal per day would be appropriate c) there is a risk of sepsis d) 0.5 mmol of phosphate is needed daily 37. In patients with permanent pacemakers: a) suxamethonium should be avoided if possible b) prophylactic antibiotics should not be used c) diathermy should preferably not be used d) electrolytes should be corrected prior to anaesthesia e) anaesthesia can lead to profound hypotension 38. Cricoid pressure: a) is effective in the presence of a nasogastric tube b) requires a complete cricoid cartilage to be effective c) should be performed with the neck extended d) should be performed after 5 minutes' pre-oxygenation e) compresses the oesophagus against the cervical vertebrae 39. Regarding EMLA cream: a) it contains 5% prilocaine and lidocaine b) it contains adrenaline c) the melting point of the agents is decreased d) it can be made with carbonated salts e) it can be made from any local anaesthetics

840

40. Regarding caudal anaesthesia: a) the posterior superior iliac crest can be used as a landmark b) absorption of local anaesthetic is greater than from lumbar epidurals c) the dura ends at the level of the posterior superior iliac spine d) subcutaneous injection can be detected using air e) negative pressure can be used to detect the space 41. The first rib: a) is crossed by the subclavian artery b) is crossed by the vagus nerve c) is crossed by the supratentorial fascia d) lies above the stellate ganglion e) is a landmark for supraclavicular brachial plexus block 42. Regarding the femoral nerve: a) it arises from the L2, -3 and -4 spinal segments b) it lies lateral to the femoral artery at the inguinal ligament c) it supplies the lateral aspect of the thigh d) total block allows arthroscopy of the knee to be performed e) block relieves pain from a fractured femur 43. Isoflurane: a) causes a dose-related decrease in systematic vascular resistance b) causes respiratory depression c) is metabolised to inorganic fluoride d) can cause a tachycardia 44. Regarding intravenous guanethidine: a) it blocks parasympathetic nerves b) it cannot be repeated c) the tourniquet must remain inflated for at least 1 hour d) it is used as a treatment for sympathetic dystrophy e) both postural hypotension and diarrhoea are common f) it causes hypotension 45. The following are true of alfentanil: a) it is more potent than sufentanil b) the volume of distribution is greater than that for fentanyl c) it is more protein bound than fentanyl d) it has a shorter elimination half-life than fentanyl e) it is more ionised than fentanyl

841

46. Pethidine is used in obstetrics because: a) it is 75% excreted unchanged b) it does not cross the placenta c) it causes less respiratory depression than an equipotent dose of morphine d) it can be used without medical supervision e) it does not affect uterine contractility 47. In the management of uterine haemorrhage, the following are used: a) IV fibrinogen b) IV ergometrine c) aortic compression d) uterine packing e) bimanual compression 48. Ritodrine causes: a) heart block b) bradycardia c) pulmonary oedema d) peripheral vasoconstriction e) increased uterine contractility 49. Concerning arterial pressure transducers: a) mean pressure is not affected by damping b) critical damping of 0.8 c) an intraflow square wave measures damping d) optimum damping occurs at 5x the resonant frequency e) they are best used with long, narrow catheters 50. Causes of respiratory distress in the neonate include: a) unilateral choanal atresia b) TDF c) diaphragmatic hernia d) necrotising enterocolitis e) myelomeningocoele 51. Neonates compared with adults have: a) decreased oxygen consumption b) decreased ability to shiver c) increased Vd/Vt d) increased body surface area/weight ratio e) increased airway resistance

842

52. Concerning the use of thiopentone in lower segment caesarean section (LSCS): a) there is a greater concentration in foetal liver compared with maternal liver b) the greatest fetal concentration occurs within 2 minutes of maternal administration c) less crosses the placenta if given at the beginning of a contraction d) neonatal depression is proportional to peak maternal concentration e) thiopentone crosses the fetal blood-brain barrier more readily 53. The following are causes of APGAR (<5) following LSCS: a) maternal hypoxia b) uterine incision-delivery time greater than 90 s c) placental transfer of muscle relaxant d) reduced uterine contractility e) reduced placental blood flow 54. Suxamethonium does not cross the placenta because of: a) placental cholinesterase b) high protein binding c) elongated molecule d) high degree of ionisation e) insufficient maternal concentration 55 Postoperative vomiting is: a) more common in males than in females b) reduced with preoperative administration of anticholinergics c) increased if pain is managed with opiates d) worse if nitrous oxide is used e) more common in operations on the abdomen than of the lower limbs 56. Cerebral ischaemia during hypotensive anaesthesia: a) is worse in a steep head-up position b) occurs with isoflurane c) is common when the blood pressure cuff is inflated frequently d) can be detected with EEG monitoring e) is avoided if normocarbia is maintained 57. In paracetamol overdose: a) respiratory alkalosis occurs b) the patient is hypothermic c) methionine is used in the treatment d) acetyl cysteine is an antidote e) coagulopathy is an early complication

843

58. Alveolar hypoventilation is associated with: a) metabolic acidosis b) asthma c) upper airway obstruction d) raised intracranial pressure e) marked ascites ANSWERS 1.TFTTF 2.TTFFF 3.FFFTF 4.FFTTF 5.FFTTF 6.FFFFT 7.FFFTF 8.TTFTF 9.FFFFT 10.TTTTF 11.TTTFT 12.FTTTTTT 13.FTFT 14.TFFFF 15.FFTFF 16.TFFFF 17.TFTF 18.TFTFT 19.FFFTTT 20.TTTT 21.FTFFF 22.FTTFF 23.FTFTT 24.TTTTT 25.FTFF 26.FTT 27.TFFTF 28.TTFF 29.TTTTT 30.FTTTT 31.FFTFF 32.TFFFF 33.FTFFF 34.TFTFT 35.TFTTT 36.FFTF 37.TFTTT 38.FTTFT 39.FFTFF 40.TTTTT

844

41.TFFFT 42.TTFFT 43.TTTT 44.FFFTTT 45.FFTTF 46.FFFTT 47.FTTTT 48.FFTFF 49.T?TTF 50.TTTTF 51.FTFTT 52.F?T?T 53.TTFFT 54.FFFTF 55.FTFTT 56.TTFTF 57.FFTTF 58.FTTTT

MCQ J 1. Recognised causes of abnormal movements of the hands include: a) respiratory failure b) renal failure c) chronic alcoholism d) liver failure e) depigmentation and sustantia nigra 2. Recognised complications of renal failure are: a) pulmonary oedema b) convulsions c) paresthesia in hands in the absence of anaemia d) acidosis e) hypokalaemia 3. Diabetes mellitus may present with: a) vulvititis b) change in refraction c) paresthesia in legs d) impotence e) acute abdominal pain 4. Acute cardiac tamponade may cause: a) progressive bradycardia b) cold cyanosed peripheries

845

c) elevated jugular venous pressure (JVP) d) giant a waves in JVP e) pulsus paradoxus 5. Causes of acute pulmonary oedema include: a) aortic stenosis b) pulmonary emboli c) atrial myxoma d) myocardial infarction e) chlorine inhalation 6. Regarding hyponatraemia: a) it should be corrected with hypertonic saline b) it always implies a disturbance in total body water c) there is abnormal aldosterone secretion d) it cannot be interpreted without clinical data e) it is associated with advanced carcinoma of the bronchus 7. DC cardioversion is indicated for: a) digitalis toxicity b) premature atrial contractions c) ventricular tachycardia d) supraventricular tachycardia e) premature ventricular contractions 8. Surgical treatment is the treatment of choice for: a) Hashimotos thyroiditis b) retrosternal goitre c) longstanding thyroid cyst d) thyrotoxicosis e) solitary thyroid nodule 9. Correct treatment for ventricular arrhythmias includes: a) procainamide b) digoxin c) lidocaine d) carotid sinus massage e) disopyramide 10. There is an increased risk of development of intestinal malignancy in: a) pernicious anaemia b) Peutz-Jeghers syndrome c) familiar polyposis coli d) ulcerative colitis

846

e) diverticular disease 11. Phaeochromocytoma is associated with: a) elevated 3-hydroxy 4-methoxy mandelic acid in urine b) bilateral tumours in 50% of cases c) medullary carcinoma of thyroid d) frequent metastases of liver e) hyperglycaemia 12. In a patient on clonidine presenting for surgery: a) clonidine should be given parenterally through to the postoperative period b) the patient is at risk of pulmonary embolism during anaesthesia c) the patient should have a noradrenaline infusion during operation d) the clonidine should be stopped before surgery e) clonidine should be stopped on the day before surgery 13. Recognised causes of glycosuria are: a) pregnancy b) partial gastrectomy c) phaeochromocytoma d) chronic renal failure e) old age f) head injury, acromegaly 14. Hiatus hernia is associated with: a) vitamin B12 deficiency b) iron deficiency anaemia c) retrosternal pain d) lower oesophageal fibrosis e) hiccoughs 15. Features of lower motor neurone disease are: a) fasciculation b) spasticity c) wasting d) increased reflexes e) upgoing plantar response 16. Immediately after complete transection of the spinal cord, the following features may be found below the lesion: a) loss of motor power, but preservation of limb reflexes b) urinary incontinence c) loss of muscle power, but preservation of sensation d) flaccid paralysis with loss of limb reflexes

847

e) loss of muscle power, but preservation of muscle joint position sense 17. Regarding acute pancreatitis: a) abdominal rigidity and guarding are characteristic early features b) the serum amylase is characteristically not raised for first 12 hours c) it may cause adult respiratory distress syndrome d) hypocalcaemia occurs within the first week of the illness e) hyperglycaemia is common 18. In a case of a Rhesus negative mother with a Rhesus positive foetus, a high level of anti-D antibody in the mother is associated with: a) foetal red cells entering maternal circulation b) antigen only crossing placenta c) neonatal anaemia d) neonatal jaundice e) sensitisation only occurs in first 3 months of pregnancy 19. Indications for an urgent craniotomy in a person with head injury include: a) sudden appearance of one fixed dilated pupil b) rapidly deteriorating conscious level c) cerebrospinal fluid rhinorrhea d) falling blood pressure and rising pulse e) generalised convulsions 20. The following are characteristic features of myasthenia gravis: a) over 90% suffer from diplopia or ptosis at some stage of their illness b) symptoms are usually symmetrical c) muscular weakness is made worse by exercise d) symptoms characteristically remit during pregnancy e) steroids and azathioprine may be effective therapy 21. Carcinoma of the lung: a) can cause Cushing's syndrome b) may present with a peripheral neuropathy c) can cause cerebellar degeneration without posterior fossa secondaries d) can produce Horners syndrome e) occurs three times more frequently in men than in women 22. Acute pancreatitis: a) may present with flank staining b) may be complicated by abscess formation c) may be precipitated by steroids d) is treated with steroids e) can precipitate tetany

848

23. The following are features of hyperparathyroidism: a) polydipsia and polyuria b) bone pain c) raised urinary calcium d) tetany e) peptic ulceration 24. Digitalis toxicity: a) may be indicated by bradycardia and a prolonged P-R internal b) may be indicated by a supraventricular tachycardia or ventricular extrasystoles c) may be reduced by giving calcium salts d) is usefully treated by a slow infusion of phenytoin e) is an absolute contraindication to beta-blockade 25. Clubbing may be seen in: a) Fallots tetralogy b) acromegaly c) ulcerative colitis d) cirrhosis e) myxoedema 26. In haemophilia A: a) factor VII levels are markedly decreased b) prothrombin and partial thromboplastin times are usually normal c) adult sufferers are usually Australian Antibody (HB3Ab) positive d) drug abuse is a frequent problem e) the dose of cryoprecipitate to be given prior to surgery is calculated from the patient's factor VIII level and estimated plasma volume 27. The treatment of adult respiratory distress syndrome should include: a) steroids b) prophylactic antibiotics c) high concentrations of oxygen d) heparin e) positive end-expiratory pressure 28. Which of the following are true of hepatitis B (serum hepatitis): a) it is spread only by blood and blood products b) most cardiac surgeons acquire the infection c) it is common in prostitutes, homosexuals and the tattooed d) it may be prevented by vaccination e) it remains the commonest cause of post-transfusion hepatitis

849

29. Acute viral bronchitis and pneumonitis: a) characteristically affects middle-aged individuals b) are typically secondary to another infection c) may be associated with the formation of epithelial giant cells in the alveoli d) often show hyaline membranes in the respiratory passages e) may be complicated by the development of lung abscesses 30. Finger clubbing is a recognised feature of: a) asbestosis b) chronic alveolitis c) fibrosing alveolitis d) fibrocaseous tuberculosis e) empyema 31. The following features are characteristic of pure mitral stenosis with atrial fibrillation: a) a loud first heart sound b) an apical pre-systolic murmur c) left ventricular hypertrophy on ECG d) an elevated left atrial pressure on cardiac catherisation e) pulsus paradoxus 32. A collapsing pulse is a recognised feature of patients with: a) malignant hypertension b) Pagets disease c) patent ductus arteriosus d) alcoholic cardiomyopathy e) atrial septal defect 33. Methaemoglobinaemia may be precipitated by: a) nitrites b) methylene blue overdosage c) aniline dyes d) ascorbic acid e) sulphonamides 34. The normal ventricular myocardium: a) has the ability to contract regularly after complete interruption of the conducting system b) has an exceptionally high mitochondrial content c) responds to an increased workload by hypertrophy d) responds to an increased workload by hyperplasia e) is of greater thickness in the right ventricle than in the left ventricle

850

35. The oxygen content of arterial blood is reduced: a) by the presence of a left to right shunt in the heart b) in patients with fibrosing alveolitis when the PaCO2 is low c) in carbon monoxide poisoning d) in methaemoglobinaemia e) in Fallots tetralogy 36. The following are recognised features of deep vein thrombosis: a) venography is the single most useful diagnostic test b) it occurs very commonly in patients who have a fractured neck of femur c) the initial treatment of choice is warfarin d) surgical intervention using an umbrella filter is almost always indicated if symptoms fail to resolve e) it is often found in association with carcinoma of the pancreas 37. The following features are commonly found in peripheral vascular disease: a) angina pectoris b) a normal cholesterol level c) occlusion at the femoral-popliteal junction d) it is more common in people from Europe and North America e) surgery using synthetic prosthesis offers a low occlusion rate 38. Superior vena cava obstruction is commonly characterised by: a) cyanosis and swelling of the head and arm b) dyspnoea c) clubbing d) Horners disease e) nausea and vomiting 39. Radiological findings of a large pulmonary artery on a chest X-ray are suggestive of: a) infundibular pulmonary stenosis b) atrial septal defect c) Fallots tetralogy d) pulmonary valve stenosis e) Eisenmenger syndrome 40. The following statements are true or false: a) starvation is characterised by loss of lean muscle mass b) low zinc levels cause insulin resistance c) L-amino acids alone are required for protein replacement d) a normal adult requires 40-50 g of protein/24 hours e) glucagon secretion is the prime response to trauma

851

41. Recognised unwanted effects of propranolol include: a) bronchospasm b) congestive cardiac failure c) retinal detachment d) tachycardia e) hyperglycaemia 42. In hypothermia: a) bradycardia is not marked b) total oxygen consumption is closely related to shivering c) a zone of dysrhythmia occurs between 26 degrees C and 28 degrees C d) the dissolved oxygen in plasma is reduced e) insulin excretion is reduced below 23 degrees C 43. Bleeding from oesophageal varices due to portal hypertension can be managed medically by: a) an infusion of 20 units of Pitressin b) propranolol c) ice-cold saline infusions into the oesophagus d) intraperitoneal aspiration e) a Sengstaken tube 44. A patient with a parathyroid tumour has a calcium level of 4 mmol/L and is admitted to hospital. Preoperative investigations and management should include: a) an intravenous pyelogram b) an infusion of normal saline c) administration of magnesium sulphate 10% d) a skull X-ray and Towne view e) vitamin D 45. A 40-year-old man is admitted after haemorrhage, dyspnoea and collapse. He is vigorously resuscitated but somewhat oliguric, and on passage of a pulmonary catheter the following results were found. RV 85/15; PA 80/30 mmHg; Wedge pressure 39 mmHg, pulse 110 bpm, systolic pressure 100 mmHg. As part of his management, this patient requires the following: a) doxapram b) dobutamine c) propranolol d) spironolactone e) nitroglycerine 46. Acute tubular necrosis is diagnosed by finding: a) casts in the urine b) a urine osmolality exceeding 500 mosm/L

852

c) a urine specific gravity of 1020 d) a urinary sodium of 10 mmol/L e) a urinary volume of 700 mol/day 47. You are asked to see a patient on the ward. He has a swinging temperature, and his sputum and chest X-ray show a right-sided collapse of part of the lung. Treatment should include: a) pleural aspiration b) bronchial lavage c) surgical biopsy of the appropriate part of the lung d) bronchography e) postural drainage 48. In severe airways obstruction: a) pulsus paradoxus is a useful prognostic guide b) helium is of benefit c) the FEV1/FVC ratio is increased d) alveolar hypoventilation is common e) the drug of choice is aminophylline 49. A 50-year-old man is admitted with features of obstructive jaundice. The following investigations are indicated to establish the diagnosis: a) barium meal b) IgM levels c) mitochondrial antibody d) fibreoptic endoscopy e) serum alkaline phosphatase 50. An elderly lady who has been on prolonged diuretic therapy presents with weakness, thirst and painful joints. The following investigations are required: a) urinary electrolytes b) serum phosphate c) an ECG d) a glucose tolerance test e) blood gases ANSWERS 1.TFTTT 2.TTFTF 3.TTTTT 4.FTTFT 5.FFTTT 6.FFFTT 7.FFTTF 8.FTFFT

853

9.TFTFT 10.FTTTF 11.TFTFT 12.TFF?F 13.TFTFFT 14.FTTTF 15.TFTFF 16.FTFTF 17.TFTTT 18.TFTTF 19.TFTFF 20.TTTFT 21.TTTTT 22.TTTFT 23.TTTFT 24.TTFTF 25.TFTTF 26.FFTTT 27.FFTFT 28.FFTTF 29.FFTTF 30.TFTTT 31.TFFTF 32.FTTFF 33.TFTFT 34.TTTFF 35.FTFTT 36.TTFFT 37.TFTTF 38.TTFFF 39.FTFFT 40.TFTFF 41.TTFFF 42.FTFFT 43.TFFFT 44.TTFFF 45.FTFFT 46.TFFFF 47.FFFFT 48.FFFFF 49.TFFTT 50.FFTTF

854

MCQ K 51. A middle-aged man presents with painless rectal haemorrhage. The following is indicated: a) barium meal b) fibreoptic colonoscopy c) a prothrombin time d) an intravenous pyelogram e) blood gas analysis 52. A patient has the following electrolyte results: Na+ 127 mEq/L, K+6.7 mEq/L. This could be caused by: a) oliguric renal failure b) hyperaldosteronism c) pituitary insufficiency d) Cushing's disease e) Meigs syndrome 53. In a patient with thyrotoxicosis: a) atrial fibrillation is common b) diabetes insipidus is a common complication c) a hoarse voice is not uncommon d) premedication with pethidine 50 mg, atropine 0.6 mg is the method of choice e) hypertensive heart disease is a common association 54. In Crohn's disease: a) the risk of subsequent malignant change is greater than in ulcerative colitis b) anal lesions are infrequent c) folate disturbances commonly cause megaloblastic anaemia d) controlled trials have demonstrated steroids to be the medical treatment of choice e) the eye signs are indistinguishable from ulcerative colitis 55. An increase in pulmonary vascular resistance is seen in: a) shock b) response to vasodilator therapy c) acidosis d) head injury e) hypocapnia 56. Pruritus is seen in association with: a) miconazole b) uraemia c) morphine overdose d) scabies e) pancreatitis

855

57. Carbon dioxide narcosis is associated with: a) papilloedema b) raised jugular venous pressure c) thready pulse d) cool periphery e) muscle twitching 58. After major abdominal surgery (e.g. abdomino-perineal resection), the following metabolic changes would be expected to occur in the first 4 postoperative days: a) an increased utilisation of glucose b) a reduction in the amount of sodium excreted in the urine c) a decrease in the free fatty acid concentration d) an increase in the amount of potassium in the urine e) an increase in the oxygen consumption per kg body weight 59. Jaundice due primarily to intrahepatic biliary obstruction (cholestatic jaundice) is a recognised complication of treatment with: a) monamine oxidase inhibitors b) C17-substituted testosterones c) erythromycin estolate (Ilosone) d) phenothiazines e) alkylating agents 60. In Crohns disease: a) the risk of subsequent malignant change of the mucosa is greater than in ulcerative colitis b) anal tensions are infrequent c) the demonstration of vitamin B12 malabsorption indicates involvement of the jejunum d) eye signs indistinguishable from those complicating ulcerative colitis are found e) controlled trials have found steroids to be helpful 61. Complications from using a central venous pressure (CVP) catheter for CVP measurement include: a) a high incidence of thrombosis and phlebitis b) a high incidence of predominantly fungal infections c) abnormal readings of CVP d) myocardial perforation of pericardial effusion e) pneumothorax, if the subclavian vein is used 62. Carcinoma of the large intestine: a) mostly originates in the ascending colon b) mostly develops from a single polyp c) may show signet ring features histologically d) characteristically metastasises to the liver before the lymph nodes

856

e) is amenable to chemotherapy 63. Finger clubbing is a recognised feature of: a) asbestosis b) chronic bronchitis c) fibrosing alveolitis d) fibrocaseous tuberculosis e) empyema 64. Carcinoma of the female breast: a) is less common than carcinoma of the cervix b) is unrelated to cystic hyperplasia of the breast c) when intraductal in type may be associated with Pagets disease of the nipple d) carries a worse prognosis when auxiliary lymph nodes show proliferation of histiocytes and plasma cells e) is prone to metastasis to the internal mammary lymph nodes 65. The following are characteristic of adrenal insufficiency: a) hypotension b) small heart c) hyperkalaemia d) increased fatiguability of striated muscle e) hypoglycaemia 66. Regarding blood transfusions: a) they are associated with an incidence of viral hepatitis of 1% or lower b) an incompatible transfusion will cause a fall in fibrinogen c) 1 unit of blood will raise the Hb value by 10% in 24 hours d) they may result in pulmonary oedema e) they are responsible for most cases of hepatitis C 67. Hypokalaemia is recognised as a complication of: a) Zollinger-Ellison syndrome b) small bowel fistula c) treatment with bumetanide d) Cushings syndrome e) infestation with round worms 68. Paroxysmal atrial tachycardia can be controlled by: a) carotid sinus pressure b) beta-blocking drugs c) atropine d) intravenous bolus of lidocaine e) stellate ganglion blockade

857

69. Radiologically, a large pulmonary artery on a chest X-ray is associated with: a) infundibular pulmonary stenosis b) atrial septal defect c) Fallots tetralogy d) pulmonary valve stenosis e) Eisenmenger syndrome 70. A man of 50 years of age presents with a left-sided pneumonia. The following is indicated: a) doxapram infusion at 2 mg/kg/h b) blood culture c) bronchoscopy d) hydrocortisone infusion e) physiotherapy with the Bird ventilator 71. A patient undergoing anaesthesia develops a blood pressure of 240/140 mmHg during the procedure. The initial treatment should include: a) digitalisation b) urinary vanillyl mandelic acid levels c) IV guanethidine d) diazoxide e) phenoxybenzamine 72. A woman, aged 30 years, who has suffered from asthma suddenly becomes increasingly breathless. The following investigations are particularly useful: a) ECG b) peak expiratory flow c) eosinophil count d) serum glucose e) clinical examination of sputum 73. Calcification on the chest X-ray in a single lesion is a feature of: a) bronchial carcinoma ANSWERS 51.FTFFF 52.TFTFT 53.TFFFF 54.FFFFT 55.TFTTF 56.TTFTF 57.TTFFT

858

58.FTFTT 59.FTTTF 60.FFFTF 61.TFTTT 62.FFTFF 63.FFTTT 64.FFTFT 65.TFTTT 66.TTTTT 67.TTTTF 68.TTFFF 69.FTFFT 70.FTTFF 71.FFFTT 72.TTFFT 73.F

Paper 3 MCQs 1. The action of the following may be prolonged in patients with abnormal plasma cholinesterase: a) prilocaine b) atracurium c) procaine d) amethocaine e) bupivacaine 2. Glyceryl trinitrate: a) increases pulmonary artery pressure b) increases central venous pressure c) produces bronchoconstriction d) increases intracranial pressure e) decreases cardiac preload 3. Sulphonylureas: a) displace bound insulin from pancreatic islet 13 cells b) exert a hypoglycaemic effect which is antagonised by thiazides c) correct ketosis d) are extensively bound to proteins e) include metoclopramide as an example 4. All anaesthetic ethers: a) are inflammable

859

b) are not metabolised in the body c) contain a halogen d) contain oxygen e) are likely to cause arrhythmias 5. Regarding a patient with severe burns: a) dangerous rises is serum potassium may occur b) a catabolic state exists for several days c) an arm represents 9% of the body surface area d) half of the fluid replacement should be given as blood e) hyperglycaemia requiring insulin may occur 6. Faulty positioning on the operating table may damage the following nerves: a) trigeminal b) radial c) ulnar d) lateral popliteal e) obturator 7. The following are recognised complications of transurethral resection of the prostate: a) haemolysis b) congestive cardiac failure c) convulsions d) hyponatraemia e) hyperkalaemia 8. Concerning postoperative myocardial infarction: a) it is often silent b) it has a lower mortality than in non-surgical patients c) postponement of elective surgery should take place if a myocardial infarct has occurred within the last 2 years d) it is associated with large intraoperative changes in blood pressure e) it occurs most commonly within 24 hours of surgery 9. The following are of use in the immediate management of an anaphylactic reaction to thiopentone: a) intravenous fluids b) 100% oxygen c) adrenaline 0.2-0.6 mg intravenously d) hydrocortisone 100 mg intravenously e) chlorpheniramine 10 mg intramuscularly 10. Regarding the innervation of the larynx: a) the recurrent laryngeal nerve supplies the only abductors of the cords

860

b) the recurrent laryngeal nerve supplies sensory innervation to the whole larynx c) the lingual surface of the epiglottis is innervated by the glossopharyngeal nerve d) the internal laryngeal nerve is a branch of the vagus nerve e) damage to one recurrent laryngeal nerve leads to life-threatening stridor 11. The following increase the rate of inhalational induction of anaesthesia: a) hyperventilation b) hypotension c) the second gas effect d) an increase in dead-space e) a high Hb concentration 12. The following are features of Cushings syndrome: a) hypokalaemia b) osteoporosis c) hypertension d) peptic ulceration e) muscle weakness 13. Dysphoria may occur with: a) pentazocine b) droperidol c) naloxone d) ketamine e) lorazepam 14. The duration of non-depolarising neuromuscular blocking agents may be prolonged by: a) acidosis b) preoperative treatment with ecothiopate c) hypothermia d) hypercarbia e) hypokalaemia 15. Angina is a recognised feature of: a) aortic stenosis b) myxoedema c) polyarteritis nodosa d) anaemia e) paroxysmal atrial tachycardia 16. Severe V/Q abnormalities are associated with: a) a small decrease in tidal volume b) a rise in the (A-a) O2 difference c) a major pulmonary embolus

861

d) a shift in the Hb-oxygen dissociation curve e) an increase in the alveolar dead-space 17. The following have blood:gas partition coefficients of < 2: a) halothane b) isoflurane c) cyclopropane d) trichlorethylene e) nitrous oxide 18. The management of life-threatening hyperkalaemia may include: a) sodium bicarbonate intravenously b) resonium orally c) calcium intravenously d) corticosteroids intravenously e) dextrose and insulin intravenously 19. Suxamethonium is contraindicated: a) during the first 24 hours following major burns b) in day-case anaesthesia c) in patients with dystrophia myotonica d) in patients with suspected epiglottitis e) in the presence of autonomic neuropathy 20. Prolonged vomiting over a 7-month period may lead to: a) vitamin B12 deficiency b) uraemia c) hypokalaemia d) hypochloraemia e) tetany 21. Regarding chlorpropamide: a) it is a biguanide b) its half-life is greater than 24 hours c) it is used in type 1 diabetes d) it prolongs the effects of warfarin e) agranulocytosis is a complication of its use 22. The following blood transfusions will result in agglutination: Donor Recipient a) O AB b) B O c) AB B

862

d) O A e) B AB 23. The following are safe in malignant hyperthermia: a) suxamethonium b) halothane c) nitrous oxide d) thiopentone e) pancuronium 24. Propranolol may cause: a) congestive cardiac failure b) bronchospasm c) hyperglycaemia d) tachycardia e) retinal detachment 25. Morphine: a) stimulates the chemoreceptor trigger zone b) is more lipid soluble than pethidine c) is a constituent of papaveretum d) causes significant depression of cardiac output e) may cause histamine release 26. Isoprenaline: a) acts at beta 1 adrenoreceptors b) increases total peripheral vascular resistance c) is excreted unchanged in the urine d) produces bronchodilatation e) increases systemic blood pressure 27. The critical temperature is: a) 273 K b) the temperature above which a gas cannot be liquified by pressure alone c) the temperature below which a gas does not vaporise d) 36.5 degrees C for nitrous oxide e) different if a substance is in a mixture rather than on its own 28. Norepinephrine reduces heart rate by: a) the baroreceptor reflex b) coronary artery vasoconstriction c) effects on beta receptors d) increased cardiac output

863

e) ganglion blockade 29. A mixture of helium and oxygen compared with air: a) is less dense b) is more flammable c) is beneficial in asthma d) has an increased blood solubility coefficient e) is used to improve turbulent flow 30. Suxamethonium: a) is metabolised in the blood b) is contraindicated in malignant hyperthermia c) is antagonised by neostigmine d) may cause hypokalaemia e) causes a transient rise in arterial blood pressure 31. There is 100 mg of lidocaine in: a) 100 ml of 1% solution b) 1 ml of 10% solution c) 50 ml of 2% solution d) 20 ml of 5% solution e) 20 ml of 0 5% solution 32. Angiotensin II: a) is produced in the lungs b) production is stimulated by renin c) production is stimulated by an increase in extracellular volume d) produces peripheral vasoconstriction e) is a mineralocorticoid 33. Psychogenic hyperventilation in a normal person causes: a) a fall in the PaCO2 b) a rise in ionised plasma calcium c) a hydrogen ion concentration of 40 nmol/L d) a rise in the PaCO2 e) a rise in the ~AO2 34. Functional residual capacity: a) is measured by helium dilution b) is decreased in restrictive lung disease c) is decreased with an increase in airways resistance d) is approximately 1200 ml in a healthy adult e) is the volume at which closure of small airways occurs

864

35. Blood group antigens: a) are on the outside of the haemoglobin b) have an autosomal inheritance pattern c) are present in group O patients d) are detectable in infants of less than 3-6 months of age e) may be detected in saliva 36. The following cause delayed gastric emptying: a) trauma b) anxiety c) morphine d) metoclopramide e) ranitidine 37. The following changes in liver physiology occur during normal pregnancy: a) increased globulin b) decreased albumin c) decreased cholesterol d) increased transaminase e) increased alkaline phosphatase 38. Features of cor pulmonale secondary to longstanding emphysema include: a) anaemia b) an increased area of cardiac dullness c) mental confusion d) cold extremities e) a large "a" wave in the jugular venous pressure waveform 39. Acute renal failure may be precipitated by: a) gentamicin b) overtransfusion c) endotoxin d) preoperative respiratory tract infection e) obstructive jaundice 40. An increase in temperature increases: a) the vaporisation rate of volatile liquids b) blood viscosity c) the volume of a gas at constant pressure d) the solubility of a gas in a liquid e) the resistance of a thermocouple

865

ANSWERS 1. FFTTF 2. FFFTT 3. TTFTF 4. FFFTF 5. TTTFT 6. FTTTF 7. FTTTF 8. TFFTF 9. TTTFF 10 TFTTF 11. TFTFF 12. TTTTT 13. TTFTF 14. TFTTT 15. TTTTT 16. FTTFT 17. FTTFT 18. TTTFT 19. FFTTF 20. FTTTT 21. FTFFT 22. FTTTF 23. FFTFT 24. TTFFF 25. TFTFT 26. TFFTF 27. FTFTT 28. TFFFF 29. TFFFT 30. TTFFT 31. FTFFT 32. TTFTF 33. TFTFF 34. TTFFF 35. FTFTT 36. TTTFF 37. FTTFT 38. FTTFF 39. TFTFT 40. TFTFF

866

Cardiovascular MCQs 1. In the electrocardiogram at a heart rate of 80 per minute: a) the PR interval should be less than 0.2 s and greater than 0.12 s b) the QRS complex should last less than 0.02 s c) the T wave is normally greater than 1 mV d) there will be an interval of 0.75 s between the end of one complex and the beginning of the next e) the T wave is ventricular repolarisation 2. Fibrinogen degradation products are natural anticoagulants interfering with: a) polymerisation of the fibrin monomer b) platelet aggregation c) thrombin activity d) serum calcium concentrations e) intrinsic pathway activation 3. Atropine: a) has no effect on acetylcholine production or destruction b) dilates cutaneous blood vessels c) is a parasympathetic depressant d) stimulates the respiratory centre e) increases intraocular pressure 4. In the cardiac cycle: a) left ventricular volume is maximal at the end of atrial systole b) the mitral valve closes by contraction of the papillary muscles c) the left ventricular pressure is maximal just before the aortic valve opens d) the ejection fraction is about 85% e) the dicrotic notch is due to rebound of the aortic valve 5. Pulmonary vascular resistance: a) is increased in chronic hypoxia b) has a value approximately one-sixth that of the systemic circulation c) can be measured using a flow-directed balloon catheter with a thermistor tip d) is increased by isoprenaline e) is decreased by 5-hydroxytryptamine (5-HT) 6. Dopamine: a) increases cardiac output b) in high doses causes peripheral vasodilatation c) increases renal blood flow d) increases ventricular excitability e) increases splanchnic blood flow

867

7. The following are true of alpha-adrenoceptor blocking agents: a) they increase blood flow in normal skin and muscle b) they cause drowsiness c) the clinically useful drugs are competitive antagonists d) they have only alpha 1- blocking activity e) they are chronotropic agents 8. In the normal cardiac cycle: a) the period of ventricular systole is equal to the Q-T interval b) the duration of the QRS complex depends on the heart rate c) the PR interval is less than 0.22 s d) ejection occurs throughout systole c) the R-R interval may vary 9. The oxygen carrying capacity of the blood is: a) the maximum quantity of oxygen that will combine with 100 ml of whole blood b) the ratio between oxygen uptake and oxygen usage c) independent of the haemoglobin concentration d) the oxygen physically dissolved in blood c) normally of the order of 15 ml per 100 ml whole blood 10. Captropril: a) increases the rate of breakdown of angiotensin II b) inhibits the breakdown of bradykinin c) may cause an increase in plasma potassium d) can safely be given in large doses in hypertensive crisis e) urine should be checked regularly for proteinuria 11. The following are isotonic with plasma: a) 1.2% sodium bicarbonate b) 5% dextrose c) 0.9 molar NaCl d) Hartmann's solution (Ringer-Lactate) e) human plasma protein fraction (5% human albumin solution) 12. When measuring arterial blood pressure using a sphygmomanometer cuff: a) if the cuff is too small for the arm, the pressure will tend to read high b) accuracy is increased by leaving the cuff slightly inflated between readings c) the slower the deflation, the more accurate the reading d) a mercury column has a low frequency response e) diastolic pressure agrees more accurately with direct measurement than will systolic pressure

868

13. Sympathetic innervation of blood vessels: a) is mediated by alpha-adrenoceptors b) is mediated locally by noradrenaline c) implies that sympathectomy induces vasodilation d) increases flow independent of vessel diameter e) induces vasodilation in response to cold and haemorrhage 14. Venous return to the heart is decreased by: a) the Valsalva manoeuvre b) exercise c) paralysis of skeletal muscles d) femoral arteriovenous fistula c) rapid infusion of blood 15. The following are important in physiological limitation of blood clotting: a) removal of activated clotting factors by the liver b) prostacyclin c) protein C d) a factor released from the endothelial cells e) fibrinogen 16. Heart rate is slowed by: a) amphetamine b) atropine c) propranalol d) dobutamine e) nifedipine 17. Using propranolol to treat hypertension: a) may exacerbate asthma b) often produces postural hypotension c) is contraindicated in patients with high plasma renin levels d) may precipitate cardiac failure in susceptible patients e) should be avoided in a patient with Raynaud's phenomenon 18. Cardiac output may be measured by: a) thermodilution b) electromagnetic flow meter c) Doppler ultrasound d) limb plethysmography e) ballistocardiography

869

19. In pulse oximetry: a) the theoretical basis is Stefan's law b) calibration is against known in vitro standards c) carboxyhaemoglobin does not affect readings d) accuracy at readings above 90% saturation is to within 0.1% e) pulse amplitude is a good indicator of cardiac output 20. The coronary blood flow: a) is about 500 ml/min at rest b) supplies muscle that takes up 40 ml oxygen per minute at rest c) is altered directly by vagal activity d) ceases in systole e) is autoregulated ANSWERS 1.TFFFT 2.TTTFF 3.TTTFT 4.TFFFT 5.TFFFF 6.TFTTT 7.TFFFF 8.TFFFT 9.TFFFF 10.FTTFF 11.TTTTT 12.TFFTF 13.TTTFF 14.TFTFF 15.FTTTF 16.FFTFF 17.TFFTT 18.TTTFT 19.FFFFF 20.FTTFT Paper 1 MCQs 1. The following are largely metabolised in the body prior to elimination: a) midazolam b) atracurium c) halothane d) isoflurane e) gallamine

870

2. Calcium channel blockers include: a) verapamil b) beta-blockers c) nifedipine d) captopril e) hydralazine 3. Regarding the Magill breathing system: a) fresh gas flow should always be greater than 8 litres/minute b) it always contains an expiratory valve c) it has the same physical properties as a Bain system d) it is equivalent to the Mapleson C classification e) it is more efficient for spontaneous respiration than intermittent positive pressure ventilation 4. The MAC value of: a) enflurane is greater than that of isoflurane b) halothane is greater than that of cyclopropane c) nitrous oxide is greater than 1 atmosphere d) methoxyflurane is greater than that of isoflurane e) halothane is greater than that of enflurane 5. In the first 3 days following major surgery, urinary excretion of the following will be reduced: a) sodium b) chloride c) water d) potassium e) nitrogen 6. Boundaries of the epidural space include the: a) interspinous ligament b) posterior surface of the lamina c) anterior longitudinal ligament d) posterior longitudinal ligament e) sacro-coccygeal membrane 7. Morphine administered epidurally may cause: a) nausea and vomiting b) respiratory depression c) itching d) muscle weakness e) a fall in blood pressure

871

8. The following values are correct for a normal adult heart: a) left atrial pressure - 20 mmHg b) pulmonary artery pressure - 45/30 mmHg c) left ventricular pressure -120/3 mmHg d) right atrial pressure - 3-5 mmHg e) cardiac index - 3 L/min/m2 9. The presence of a slow junctional rhythm in a fit 50 year old man receiving halothane: a) requires immediate treatment b) suggests hypoxia c) suggests the patient is light d) should be treated with beta-blockers e) may be treated with glycopyrrolate 10. There is an increased incidence of gallstones in: a) haemolytic anaemia b) hypercalcaemia c) oral contraceptive pill users d) hypoparathyroidism e) conditions with an increased production of bile salts 11. Methaemoglobinaemia can be: a) caused by prilocaine b) caused by blood transfusion c) caused by carbon monoxide poisoning d) treated by methylene blue e) treated by ascorbic acid 12. Human plasma albumin: a) is the greatest contributor to plasma oncotic pressure b) is produced in the liver c) carries carbon dioxide in the blood d) is an anion at pH 7.5 e) is actively filtered by the glomerulus 13. The following are anticholinesterases: a) organophosphates b) neostigmine c) prilocaine d) ranitidine e) atropine

872

14. Dopamine antagonists cause: a) renal artery vasodilatation b) extrapyramidal effects c) antiemetic effects d) a rise in heart rate e) a rise in the apnoeic threshold for CO2 15. Oxygen: a) is prepared by fractional distillation b) has a critical temperature of 36.5 degrees C c) in the presence of oil can lead to explosion d) under hyperbaric conditions can produce convulsions e) therapy can lead to bone marrow depression with prolonged exposure 16. Headache after spinal anaesthesia: a) is less likely with a 26G than with a 22G needle b) is due to an increase in cerebrospinal fluid pressure c) is unlikely to develop after 24 hours d) may be accompanied by a 6th cranial nerve palsy e) is more frequent in the elderly 17. The following ions have a higher intracellular than extracellular concentration: a) sodium b) calcium c) potassium d) chloride e) bicarbonate 18. Pressure: a) is force per unit area b) can be expressed as the height of fluid in a column c) is force multiplied by distance d) is work per unit time e) has potential difference as its electrical analogue 19. The sympathetic nervous system: a) has cholinergic post-ganglionic fibres b) has fibres in the vagus c) causes miosis d) has acetylcholine as the ganglionic neurotransmitter e) has its outflow from the spinal cord from T1 to L2

873

20. Methohexitone: a) is more potent than thiopentone b) commonly causes pain on injection c) is a thiobarbiturate d) causes involuntary movements e) is prepared in a solution of pH 7.4 21. A respirometer can be used to measure: a) functional residual capacity b) residual volume c) vital capacity d) inspiratory reserve volume e) closing volume 22. The following can influence the height of a spinal block: a) strength of local anaesthetic solution b) volume of local anaesthetic solution c) baricity of local anaesthetic solution d) the interspace used e) addition of adrenaline to the local anaesthetic solution 23. Features of Mendelsons syndrome include: a) urticarial rash b) bronchospasm c) hypoxia d) hypotension e) aspiration of at least 100 ml of gastric contents 24. 21 day old stored blood: a) is a good source of platelets b) is a good source of clotting factors c) may have a serum potassium concentration of 10 mmol/L d) may have a pH of 7.0 e) is likely to have an excess of 2,3-diphosphoglycerate 25. Tracheal deviation to the right can be caused by: a) a large left pleural effusion b) right upper lobe fibrosis c) a right pneumothorax d) a retrosternal goitre e) a previous left pneumonectomy

874

26. The following should decrease the risk of severe barotrauma: a) a compliant reservoir bag b) a pressure relief valve set at 70 cm H20 c) non-interchangeable connections in a circle system d) draw-over anaesthesia e) use of cylinders rather than piped gas supplies 27. When administering oxygen: a) a Ventimask delivers a fixed oxygen percentage independent of fresh gas flow b) nasal cannulae are less efficient than an MC mask c) a constant oxygen percentage is delivered by an MC mask d) significant rebreathing is likely with an MC mask e) chronic bronchitic patients should never receive more than 28% oxygen 28. Signs of a Horners syndrome include: a) dry forehead b) conjuctival injection c) ptosis d) exophthalmos e) miosis 29. In a patient who fails to breathe 1 hour after administration of 80 mg suxamethonium: a) the pseudocholinesterase gene is likely to be homozygous atypical b) a serum potassium of 2 5 mmol/L is a possible cause c) 2.5 mg neostigmine should be given d) the metabolism of lidocaine may well be impaired e) the same response would be expected in all siblings 30. Regarding anticholinergic agents: a) atropine is a stronger antisialogogue than hyoscine b) hyoscine produces more tachycardia than atropine c) glycopyrrolate produces marked mydriasis d) glycopyrrolate crosses the blood-brain barrier e) hyoscine-induced confusion in the elderly is reversed by procyclidine 31. The cerebral blood flow of a normal person is increased: a) when the arterial PCO2 increases from 5.3 to 8.0 kPa b) when placed in the head-down position c) if the mean arterial blood pressure rises from 90 to 110 mmHg d) if the intracranial pressure is increased e) during physiological sleep

875

32. Features of aortic regurgitation include: a) right ventricular hypertrophy b) wide pulse pressure c) a mid-diastolic murmur at the apex d) pulsus paradoxus e) cardiac failure 33. With regard to kidney function: a) sodium is mainly reabsorbed from the proximal tubule b) urinary potassium content is regulated by the distal tubule c) glucose is completely reabsorbed if the blood concentration is normal d) a rise in blood volume results in an increase in antidiuretic hormone secretion e) acidosis results in an increased excretion of ammonium chloride 34. Concerning nitrous oxide cylinders: a) they are colour-coded blue with white shoulders b) the cylinder pressure is directly proportional to the quantity of N20 it contains c) rapid emptying of a cylinder will cause its temperature to fall d) pressure regulators must be used to reduce the pressure of the gas supplied from the cylinder before it enters the patient circuit e) the pressure in a full cylinder at 20 degrees C is approximately 137100 kPa 35. The Mapleson A breathing system: a) is a non-rebreathing system b) must have the expiratory valve close to the patient c) requires a fresh gas flow equal to the dead space ventilation d) requires a higher fresh gas flow than a Mapleson D system in manual ventilation e) was modified by Magill from Maplesons original design 36. Activated charcoal, as used in the Cardiff Aldasorber: a) does not absorb nitrous oxide b) is effective in absorbing isoflurane c) increases expiratory resistance significantly d) can be reactivated by heating to remove the absorbed agents e) produces toxic substances if used with trichlorethylene 37. The oxygen-haemoglobin dissociation curve is moved to the right by: a) acidosis b) raised body temperature c) passage through the pulmonary capillaries d) ageing e) anaemia

876

38. At the neuromuscular junction: a) the resting potential is -45 mV b) single twitch height is reduced if 60% of receptors are occupied by a non-depolarising muscle relaxant (NDMR) c) acetylcholine reduces the permeability of the post-junctional membrane to sodium d) neostigmine directly displaces NDMR from the motor end plate e) hypokalaemia increases sensitivity to an NDMR 39. The following act on peripheral alpha-adrenergic receptors: a) clonidine b) phenoxybenzamine c) trimetaphan d) droperidol e) phentolamine 40. Isoflurane: a) is not metabolised b) can be used with accuracy in halothane vaporisers c) is less respiratory depressant than halothane d) has a calcium antagonist action e) causes a fall in heart rate in elderly subjects ANSWERS 1. TTFFF 2. TFTFF 3. FTFFT 4. TFTFF 5. TTTFF 6. FFFTT 7. TTTFF 8. FFTTT 9. FFFFT 10. TFTFF 11. TFFTT 12. TTFTF 13. TTFFF 14. FTTFF 15. TFTTF 16. TFFTF 17. FFTFF 18. TTFFT 19. TFFTF 20. TTFTF 21. FFTTF 22. TTTTF

877

23. FTTFF 24. FFTTF 25. TTFTF 26. TTTTF 27. TTFFF 28. TFTFT 29. TFFFF 30. FFFFF 31. TFFFT 32. FTFFT 33. TTTFT 34. FFTTF 35. FFFTF 36. TTFTF 37. TTFFT 38. FFFFT 39. FTFTT 40. FTFTF Paper 2 MCQs 1. Donor blood in the UK is routinely screened for evidence of infection by: a) Cytomegalovirus b) HIV c) Hepatitis A d) Hepatitis B e) Hepatitis C 2. Peripheral nerve injury is a recognised complication of a: a) fracture of the upper fibula b) fracture of both forearm bones c) dislocated shoulder d) tibial fracture e) fracture shaft of humerus 3. Etomidate: a) is an imidazole derivative b) contains an ester group c) causes more postoperative nausea and vomiting than propofol d) is associated with adrenal medullary suppression e) has a low incidence of allergy 4. The pain of angina typically: a) is stabbing in nature b) is aggravated by hot weather conditions c) occurs mainly on the left side of the chest

878

d) is relieved by rest within 2 or 3 seconds of stopping exercise e) is associated with nausea 5. The following drugs may be used effectively via a tracheal tube: a) atropine b) sodium bicarbonate c) adrenaline d) lidocaine e) calcium carbonate 6. Postoperative jaundice may occur: a) as a consequence of biliary surgery b) following premedication with papaveretum c) after perioperative blood transfusion d) following the use of nitrous oxide for a prolonged surgical procedure e) due to incidental viral hepatitis 7. In the Venturi oxygen mask: a) the delivered oxygen concentration falls when the flow is decreased from 5 to 2 L/min b) rebreathing is likely c) total flow should exceed 20 L/min d) increasing the diameter of the constriction increases the oxygen concentration e) occluding the holes in the side of the mask decreases the oxygen concentration 8. Features of depolarising block include: a) fasciculations b) long duration of action c) fade d) antagonism by anticholinesterases e) a rise in intraocular pressure 9. Medicolegally: a) an 18 year old can give consent for his/her own surgery b) the Sister in charge of theatre must check the patients identification c) the anaesthetist is not responsible for checking the side of the operation d) a signed consent form is necessary before surgery e) a complication occurring with an incidence of at least 1% should be explained to a patient preoperatively 10. A canister of soda-lime: a) contains mostly calcium carbonate b) requires water for the absorption process c) contains 50% by weight of sodium hydroxide d) will absorb nitrous oxide

879

e) when packed will be 50% filled with granules 11. In a patient with rheumatoid arthritis, expected problems include: a) cardiac failure b) anaemia c) difficulty with intubation d) poor venous access e) hypotension 12. An elective operation should be postponed if the patient has: a) a haemoglobin concentration of 10.8 g/dl b) acute intermittent porphyria c) a white cell count of 8 x 109/L d) unexpected glycosuria e) a potassium concentration of 5.0 mmol/L 13. Gallstones are associated with: a) intestinal obstruction b) peritonitis c) pancreatitis d) carcinoma of the gall bladder e) cholangitis 14. Cardiac output: a) varies with oxygen consumption b) increases with a rise in left ventricular end-diastolic volume c) increases with a rise in afterload d) increases with a rise in myocardial contractility e) can be estimated with echocardiography 15. Induction with ketamine is contraindicated: a) in an asthmatic patient b) for caesarean section c) in a patient with facial burns d) in a hypertensive patient e) in a shocked patient 16. The oesophagus: a) lies in the anterior mediastinum b) is lined by columnar epithelium c) begins at the level of C6 d) extends to approximately 20 cm from the teeth e) receives its motor innervation from the vagus

880

17. The following drugs can be used to treat bronchospasm occurring during anaesthesia: a) aminophylline b) halothane c) salbutamol d) sodium dicromoglycate e) propranolol 18. The partial pressure of carbon dioxide in arterial blood: a) rises during sleep b) rises when acclimatised at an altitude of 5000 metres c) is halved when the minute ventilation is doubled (with constant production of CO2) d) differs from that of mixed venous blood by about 0.8 kPa (6 mmHg) e) normally rises at about 0.8 kPa (6 mmHg) per minute during apnoea

19. In the treatment of established malignant hyperthermia, the following are recognised as part of the treatment: a) chlorpromazine b) EDTA sodium c) sodium bicarbonate d) magnesium sulphate e) glucose and insulin 20. The following are constituents of raw opium: a) heroin b) methadone c) morphine d) papaverine e) codeine 21. The following are recognised features of Fallots tetralogy: a) squatting b) pulmonary oligaemia c) cyanosis occurring only with exercise d) a loud systolic murmur due to a ventricular septal defect e) syncope 22. The risk of passive regurgitation can be increased by: a) the presence of a nasogastric tube b) neostigmine c) metoclopramide d) a non-depolarising muscle relaxant

881

e) placing a patient in the lithotomy position 23. Regarding the innervation of the eye: a) the sensory supply is via the facial nerve b) the motor supply to orbicularis oculi is via the oculomotor nerve c) the sympathetic supply is via the superior cervical ganglion d) the ciliary ganglion lies within the extraocular muscle cone e) parasympathetic stimulation produces mydriasis 24. The following are produced in the anterior pituitary: a) somatotrophin b) oxytocin c) aldosterone d) thyrotrophin (TSH) e) prolactin 25. Stridor following thyroidectomy may be caused by: a) recurrent laryngeal nerve damage b) wound haematoma c) thyroid storm d) hypocalcaemia e) tracheomalacia 26. An acute intracranial extradural haematoma occurs usually: a) after a lucid interval b) in association with contralateral pupillary dilatation c) in the elderly d) in association with a skull fracture e) as a result of arterial bleeding 27. A size E cylinder of nitrous oxide: a) contains the equivalent of 1800 litres of gas (at STP) when full b) has a pressure of 137 Bar at 20 degrees C when full c) may develop frost on the outside during normal use d) is coloured blue, with blue and white quartered shoulders e) contains more liquid than gas when full 28. When using a circle system with absorber and a vaporiser outside the circle: a) a fresh gas flow of at least 2/3 of the minute volume is required b) the actual inspired vapour concentration during maintenance may be higher than the vaporiser setting c) the actual inspired oxygen concentration may be lower than that of the fresh gas at low fresh gas flows

882

d) sevoflurane is a suitable agent to use e) the system can only be used for patients breathing spontaneously 29. The intraocular pressure: a) is normally less than 5 mmHg b) is lowered by the administration of suxamethonium c) increases with coughing d) may be normal in closed-angle glaucoma e) can be lowered by the use of acetazolamide 30. Tachycardia may be expected following administration of: a) pancuronium b) ketamine c) edrophonium d) gallamine e) d-tubocurarine 31. Digoxin toxicity: a) is potentiated by hypokalaemia b) is an idiosyncracy of the drug c) produces atrioventricular block d) is less likely to occur in the elderly e) is associated with vomiting 32. The following drugs are readily soluble in water: a) propofol b) midazolam c) diazepam d) ketamine e) etomidate 33. The Hb-oxygen dissociation curve is shifted to the: a) right by a raised arterial PCO2 b) right by a raised arterial pH c) left by hypothermia d) left in 14 day old stored blood e) right if the P50 is increased 34. P waves are absent in: a) atrial fibrillation b) ventricular tachycardia c) massive pulmonary embolism d) first degree heart block e) nodal rhythm

883

35. The pupil is dilated by the following being put into the conjunctival sac: a) ephedrine b) timolol c) homatropine d) cocaine e) chloramphenicol 36. Nerve impulses: a) are slower in large diameter nerve fibres than in small ones b) are transmitted via electrical impulses c) are stopped if the nerve is frozen d) result from ionic movement across the membrane e) cannot be transmifted during the absolute refractory period 37. Halothane: a) is the only inhalational agent implicated in precipitating malignant hyperpyrexia b) has a lower minimum alveolar concentration (MAC) than isoflurane c) undergoes a greater degree of metabolism in the liver than does isoflurane d) is a halogenated ether e) sensitises the myocardium to the arrhythmogenic action of adrenaline, even at a concentration of 1 MAC 38. Reduced central venous pressure may be due to: a) intermittent positive pressure ventilation b) constrictive pericarditis c) increased venous capacitance d) tricuspid incompetence e) reduced myocardial contractility 39. A tall R wave in lead V1 is typically present with: a) left bundle branch block b) right ventricular hypertrophy c) inferior myocardial infarction d) mitral stenosis e) hyperkalaemia 40. Lymph: a) contains no cells b) contains less protein than plasma c) circulates at a rate of 1 litre per day d) clots on standing e) carries important enzymes

884

ANSWERS 1. FTFTT 2. TFTFT 3. TTTFT 4. FFFFT 5. TFTTF 6. TFTFT 7. FFTTF 8. TFFFT 9. TFFFT 10. FTFFT 11. FTTTF 12. FFFTF 13. TTTTT 14. FTFTT 15. FFFTF 16. FFTFT 17. TTTFF 18. TFTTF 19. FFTFT 20. FFTTT 21. TTFTF 22. TFFFT 23. FFTTF 24. TFFTT 25. TTFTT 26. TFFTT 27. TFTFT 28. FFTFF 29. FFTTT 30. TTFTF 31. TFTFT 32. FTFTF 33. TFTTT 34. TTFFT 35. TFTTF 36. FTTTT 37. FTTFT 38. FFTFF 39. FTFFF 40. FTFTT

885

Paper 4 MCQs 1. The following act by blocking cholinergic receptors: a) trimetaphan b) hexamethonium c) ouabain d) benzhexol e) physostigmine 2. Esmolol: a) has a negative inotropic effect b) has no intrinsic sympathomimetic activity c) causes a dose-dependent fall in heart rate d) increases airways resistance e) may prolong the duration of action of suxamethonium 3. Enflurane: a) is a halogenated methyl-ethyl ether b) has a blood/gas solubility coefficient of 1.43 c) lowers intracranial pressure d) increases the tone of the pregnant uterus e) causes a decrease in systemic vascular resistance 4. Morphine: a) causes histamine release b) has no active metabolites c) undergoes extensive first-pass hepatic metabolism d) has an elimination half-life of 3-4 hours e) may be antagonised by pentazocine 5. Uterine tone is: a) increased by ketamine b) decreased by halothane c) increased by beta-2 antagonists d) unaffected by suxamethonium e) decreased by nifedipine 6. Aspirin overdose causes: a) thrombocytopenia b) coma c) metabolic acidosis d) jaundice e) pulmonary oedema

886

7. Non-depolarising neuromuscular blockers are potentiated by: a) lithium b) trimetaphan c) diazepam d) magnesium e) suxamethonium 8. Atropine: a) crosses the blood/brain barrier b) can cause an initial bradycardia c) increases the rate of gastric emptying d) increases conduction through the A-V node e) produces bronchoconstriction 9. Hepatotoxicity has been associated with: a) diethyl ether b) chloroform c) cyclopropane d) enflurane e) halothane 10. The anticoagulant effect of warfarin: a) is exerted directly on the blood b) is slow in onset c) can be reversed by vitamin K d) is potentiated by phenylbutazone e) is potentiated by barbiturates 11. The following may be used in the treatment of digoxin toxicity: a) propranolol b) lidocaine c) phenytoin d) calcium e) potassium 12. Dobutamine: a) is structurally similar to isoprenaline b) activates adenyl cyclase c) has a selective action on beta-1 adrenoreceptors d) has a half-life of 2 minutes e) increases the left ventricular end diastolic pressure

887

13. Organophosphorus anticholinesterases: a) have an irreversible action b) phosphorylate cholinesterase c) produce more inhibition of acetyl cholinesterase than of plasma cholinesterase d) can have their action reversed in the early stages by atropine e) are readily absorbed through the skin 14. The following factors encourage passage of a substance across the cell membrane: a) high lipid solubility b) low concentration gradient c) high molecular weight d) negative hydrostatic pressure e) high degree of ionisation 15. Amitriptyline in overdose causes: a) cardiac arrhythmias b) hypertension c) restlessness d) metabolic acidosis e) jaundice 16. Low plasma cholinesterase activity: a) is related to the patients blood group b) has no effect on the action of decamethonium c) occurs with organophosphorus poisoning d) prolongs the action of esmolol e) occurs in malnutrition 17. Sulphonylureas: a) act by increasing insulin release b) tend to produce weight loss c) are suitable for use in pregnancy d) are effective in correcting ketoacidosis e) can produce hypoglycaemia 18. Side-effects of hydralazine include: a) tachycardia b) impotence c) constipation d) systemic lupus erythematosus e) bronchoconstriction

888

19. The following are measures of scatter in statistical analysis: a) mean b) standard error of the mean c) standard deviation d) range e) p of less than 0.5 20. Statistical tests are used to: a) eliminate observer bias b) eliminate placebo effect c) show that results are true d) show that the results did not occur by chance e) show that the results are clinically significant 21. The liver: a) receives most of its oxygen supply from the portal vein b) has its highest oxygen tension at the centre of a lobule c) produces heparin d) has a normal portal venous pressure of greater than 20 mmHg e) receives approximately 25% of the cardiac output 22. Skeletal muscle blood flow: a) increases with norepinephrine b) receives 50% of the cardiac output at rest c) may cease during isometric contraction d) increases with rhythmic contraction e) increases with epinephrine 23. In thermoregulation: a) respiratory heat loss is insignificant under normal conditions b) brown fat is an important source of heat production in neonates c) shivering is due to impulses conducted via autonomic efferents d) peripheral vasoconstriction increases heat production e) sweating is mediated by sympathetic adrenergic neurones 24. Epinephrine: a) is synthesised by demethylation of norepinephrine b) increases coronary blood flow c) increases free fatty acids in the blood d) mobilises glycogen stores from the liver e) is metabolised in the plasma by monoamine oxidase

889

25. Glucagon: a) is a positive inotrope b) is produced by the B cells of the pancreas c) stimulates production of free fatty acids in the blood d) release is increased in starvation e) stimulates glycogen synthesis 26. Surfactant: a) is a mucopolypeptide b) causes a decrease in surface tension c) results in the same surface tension for different sized alveoli d) causes an increase in compliance e) production is reduced after a prolonged reduction in pulmonary blood flow 27. Sinus arrhythmia: a) produces a lengthening of the P-R interval b) produces a lengthening of the R-R interval c) is maximal with breath holding d) is more marked during exercise e) is more marked in 70 year olds than in 20 year olds 28. A pressure-volume curve can be used for measuring: a) the work of breathing b) functional residual capacity c) anatomical dead space d) compliance e) respiratory quotient 29. The absolute refractory period for cardiac muscle is: a) as long as the entire action potential b) the period when no further action potential can be stimulated c) twice the length of the S-T interval d) as long as the mechanical contraction e) shorter for pacemaker tissue than for normal cardiac muscle 30. In a young normal adult: a) the glomerular filtration rate is approximately 125 ml/min b) the 24 hour urine creatinine content is approximately 800 mg c) urine specific gravity is always less than 1000 d) renal blood flow is approximately 20% of cardiac output e) over 50% of water reabsorption from the glomerular filtrate occurs in the collecting ducts

890

31. Total plasma calcium: a) increases with phosphate b) increases with a rise in albumin c) changes its degree of ionisation with pH changes d) is decreased in osteoporosis e) is affected by vitamin D 32. There is increased intestinal motility with: a) increased intraluminal pressure b) anticholinesterase drugs c) sympathetic block to T4 d) stimulation of the splanchnic nerves e) increased circulating epinephrine 33. A healthy adult breathing an FiO2 of 0.1 will: a) have a decreased cardiac output b) have a normal PaO2 c) have a changed alveolar PCO2 d) have an unchanged respiratory rate e) initially have a fall in pH 34. Acetylcholine is a neurotransmitter at: a) sweat glands b) the adrenal medulla c) the parotid gland d) parasympathetic ganglia e) the neuromuscular junction 35. Insulin: a) has the same effect on blood sugar as growth hormone b) inhibits entry of potassium into cells c) facilitates protein anabolism d) increases deposition of fats e) secretion is affected by catecholamines 36. In a normal resting subject, a bradycardia would be expected following: a) an increase in carotid sinus pressure b) an increase in right atrial pressure c) application of pressure to the eyeball d) the release of a Valsalva manoevre e) inspiration

891

37. The velocity of conduction of a nerve action potential: a) is inversely related to the cross-sectional area of the axon b) is faster in a myelinated fibre than in an unmyelinated one c) is decreased by cooling the nerve d) can exceed 100 m/s in humans e) is highest in pre-ganglionic autonomic fibres 38. The placenta: a) transports glucose from maternal to fetal blood by facilitated diffusion b) can synthesise glycogen c) actively transports oxygen from maternal to fetal blood d) allows protein molecules to pass from maternal to fetal blood by pinocytosis e) secretes oestradiol 39. Oxytocin: a) stimulates production of milk b) stimulates ejection of milk c) release is stimulated by dilatation of the cervix d) is synthesised in the anterior pituitary e) produces more powerful uterine contraction in the presence of progesterone 40. Aldosterone: a) production increases with a fall in plasma osmolality b) production decreases with a fall in blood volume c) production decreases with a rise in plasma renin level d) increases urinary potassium excretion e) may be produced by tumours of the adrenal cortex ANSWERS 1. TTFTF 2. TTTTT 3. TFFFT 4. TFTTT 5. TTTTF 6. TTTFT 7. TFTTT 8. TTFTF 9. FTFTT 10. FTTTF 11. TTTFT 12. TTTTF 13. TTFTT 14. TFFFF 15. TFTTF

892

16. FFTTT 17. TFFFT 18. TFFTF 19. FTTTF 20. FFFTF 21. FFFFT 22. FFTTT 23. TTFFF 24. FTTTF 25. TFTTF 26. FTTTT 27. FTFFF 28. TFFTF 29. FTFFT 30. TFFTF 31. FTTFT 32. TTTFF 33. FFTFF 34. TTTTT 35. FFTTT 36. TFTTF 37. FTTTF 38. TFFTT 39. FTTFF 40. FFFTT

Paper 5 MCQs 1. Dopamine: a) may produce ventricular arrhythmias b) increases mesenteric blood flow at high doses c) crosses the blood/brain barrier d) is synthesised from L-dopa e) is inactivated in alkaline solution 2. The following increase the amount of calcium in cardiac muscle: a) halothane b) adrenaline c) diltiazem d) nifedipine e) trimetaphan 3. The following can be used safely in a patient on a monoamine oxidase inhibitor: a) morphine b) pethidine c) norepinephrine

893

d) amphetamine e) epinephrine 4. Hyoscine hydrobromide causes: a) antiemesis b) somnolence c) pupillary dilatation d) tachycardia e) extrapyramidal symptoms 5. Clonidine: a) is an alpha-2 receptor agonist b) is a dopamine antagonist c) causes tachycardia d) inhibits salivation e) reduces the minimum alveolar concentration of halothane 6. Hydralazine: a) is metabolised by acetylation b) is destroyed by plasma cholinesterase c) can cause a lupus-like syndrome d) stimulates the baroreceptor reflex e) is contraindicated in pregnancy 7. The following are prodrugs: a) suxamethonium b) diamorphine c) captopril d) paracetamol e) enalapril 8. The following drugs penetrate the blood/brain barrier: a) physostigmine b) dopamine c) propranolol d) Glycopyrrolate e) norepinephrine 9. Naloxone: a) is an agonist at K receptors b) is an antagonist at mu receptors c) reverses ventilatory depression due to morphine d) may precipitate opiate withdrawal symptoms e) may cause pulmonary oedema

894

10. The following are precursors of epinephrine: a) tyrosine b) phenylalanine c) dopamine d) isoprenaline e) norepinephrine 11. The following affect gastric emptying: a) diamorphine b) diazepam c) metoclopramide d) cisapride e) omeprazole 12. Chlorpromazine: a) can cause dystonic reactions b) antagonises apomorphine-induced vomiting c) is a dopamine antagonist at the chemoreceptor trigger zone d) is a weak alpha adrenergic agonist e) undergoes extensive first pass metabolism

13. Alfentanil: a) is less lipid soluble than fentanyl b) relaxes the sphincter of Oddi c) has active metabolites d) has a large volume of distribution e) causes analgesia without sedation 14. Folic acid metabolism is impaired by: a) nitrous oxide b) sodium nitroprusside c) sulphonamides d) penicillin e) trimethoprim 15. Significant agonist activity at opioid receptors occurs with: a) clonidine b) pentazocine c) buprenorphine d) ketamine e) naloxone

895

16. Drug clearance by the body: a) only refers to elimination by the kidney b) refers to the volume of blood cleared of the drug in unit time c) cannot exceed the glomerular filtration rate d) may be influenced by renal tubular secretion e) is the same as creatinine clearance 17. Dopexamine: a) causes arterial vasoconstriction b) is an agonist at dopaminergic D1 and D2 receptors (c) increases the force of myocardial contraction (d) increases renal blood flow (e) causes arrhythmias 18. Sodium valproate: a) is effective in grand mal epilepsy b) is effective in petit mal epilepsy c) is safe in patients with liver disease d) should not be given to children e) increases brain concentrations of gamma-amino butyric acid (GABA) 19. The Chi-squared test: a) is used to compare the frequencies of occurrence b) requires the standard error of the mean to be calculated c) does not require a knowledge of the number of degrees of freedom d) should not be used for data with small groups e) does not involve the null hypothesis 20. A placebo effect: a) may occur in either treatment or control groups b) occurs only in mentally ill patients c) is likely to occur repeatedly in placebo reactors d) can occur in up to 35% of patients e) is not seen in double-blind trials 21. In calculating the shunt fraction, the following need to be measured or estimated: a) mixed venous oxygen content b) pulmonary end-capillary oxygen content c) arterial oxygen content d) alveolar partial pressure of oxygen e) haemoglobin concentration

896

22. Pulse pressure increases with an increase in: a) stroke volume b) left ventricular end-diastolic volume c) arterial partial pressure of oxygen d) systemic vascular resistance e) blood viscosity 23. Acute untreated haemorrhagic shock in a patient will lead to: a) an increase in physiological dead-space b) an increase in the arterio-venous PCO2 difference c) a fall in the pulmonary vascular volume d) an increase in antidiuretic hormone secretion e) an increase in plasma bicarbonate concentration 24. An increase in aldosterone secretion follows: a) a sodium chloride load b) a rise in blood volume c) an increase in oral potassium absorption d) trauma e) an increase in production of angiotensin II 25. Stimulation of alpha adrenergic receptors will cause: a) vasoconstriction of the coronary arteries b) increased tone in the bladder neck muscle c) increased platelet aggregation d) lipolysis e) bronchodilation 26. In the normal adult heart: a) mitral valve closure occurs before tricuspid valve closure b) pulmonary valve closure occurs before aortic valve closure c) there is isometric contraction of the left ventricle after the aortic valve opens d) atrial contraction is of more importance to ventricular filling if the heart rate increases e) the aortic valve cusps are immobile during ventricular filling 27. Resistance to laminar flow in a vessel is: a) proportional to wall thickness b) inversely proportional to the fourth power of the radius c) proportional to length d) independent of haematocrit e) proportional to the pressure drop 28. Autoregulatory mechanisms used in hypovolaemia include:

897

a) an increase in precapillary sphincter tone b) an increase in capillary hydrostatic pressure c) a decrease in baroreceptor activity d) stimulation of the juxtaglomerular apparatus e) an increase in angiotensin II 29. Myocardial contractility is increased by: a) catecholamines b) an increase in heart rate c) an increase in fibre length d) an increase in parasympathetic nervous system activity e) calcium ions 30. The carotid sinuses: a) have stretch receptors in their walls b) give afferent impulses via the glossopharyngeal nerve c) stimulate the respiratory centre d) contain chemoreceptors e) stimulate the vasomotor centre

31. The following cause a decrease in the arterial partial pressure of oxygen: a) anaemia b) carbon monoxide c) hyperventilation d) a rise in physiological dead-space e) old age 32. Iron absorption is dependent on: a) total body vitamin C b) acid in the stomach c) an intact colonic mucosa d) total body iron e) erythropoietin levels in the blood 33. If a normal person hyperventilates for 2 hours to an arterial PCO2 of 4 kPa: a) the cerebral blood flow decreases b) the standard bicarbonate decreases c) the haemoglobin-oxygen dissociation curve shifts to the left d) the ionised calcium concentration decreases e) the plasma bicarbonate increases 34. Hypoglycaemia may result from:

898

a) excessive insulin secretion b) glucagon secretion c) hypothermia 35. Ingested lipid: a) is important in prostaglandin synthesis b) increases in the faeces with a decrease in bile secretion c) is absorbed via the intestinal lymphatics d) is mainly in the form of triglycerides e) can be used as a source of ATP production 36. The following may be found in normal adult venous blood: a) 3% carboxyhaemoglobin b) 5% methaemoglobin c) 70% oxyhaemoglobin d) 2% free haemoglobin e) 2% haemoglobin F 37. In normal human lungs: a) a low PO2 produces pulmonary vasodilatation b) beta-2 agonists cause bronchoconstriction c) pulmonary vascular resistance is increased by serotonin d) pulmonary vascular resistance is decreased by histamine e) pulmonary vascular resistance is decreased by norepinephrine 38. The normal response to surgery includes: a) a decrease in urine volume b) a decrease in the urinary excretion of sodium c) a decrease in plasma cortisol level d) an increase in the urinary excretion of nitrogen e) an increase in the urinary excretion of potassium 39. Capillary permeability is increased by: a) bradykinin b) epinephrine c) calcium d) vasopressin e) histamine 40. Expected changes in a patient with a phaeochromocytoma include: a) a decreased haematocrit b) a decreased total blood volume c) a decreased serum sodium concentration

899

d) an abnormal glucose tolerance test e) a reduced metabolic rate ANSWERS 1. TFFTT 2. FTFFF 3. TFTFT 4. TTTFF 5. TFFTT 6. TFTTF 7. FTFFT 8. TFTFF 9. FTTTT 10. TTTFT 11. TFTTF 12. TTTFT 13. TFFFF 14. TFTFT 15. FTTFF 16. FTFTF 17. FFTTT 18. TTFFT 19. TFFTF 20. TFTTF 21. TTTTT 22. TTFFF 23. TFTTF 24. FFTTT 25. TTTFF 26. TFFTT 27. FTTFF 28. TFTTT 29. TFTFT 30. TTFFT 31. FFFFT 32. FTFTF 33. TFTTF 34. TFF 35. TTTTT 36. TFTFT 37. FFTTF 38. TTFTT 39. TFFFT 40. FTFTF

900

Physiology MCQS 1. The carotid bodies: a) have a low blood flow per gram of tissue b) contain baroreceptors c) respond to changes in pH d) respond to small changes in PaO2 e) are located on the external carotid arteries 2. The following are examples of active transport: a) sodium extrusion from cells b) water reabsorption from the proximal convoluted tubule c) potassium excretion in the distal convoluted tubule d) glucose absorption from the gut c) water reabsorption in the loop of Henle 3. Growth hormone and insulin have opposite effects on: a) carbohydrate uptake by muscle b) catabolism of fat c) synthesis of fat d) synthesis of protein e) somatic growth 4. Stimulation of the tenth cranial nerve causes: a) slowing of A-V conduction b) constriction of coronary vessels c) increased secretion of gastric acid d) miosis e) relaxation of the pylorus 5. An increase in sympathetic stimulation to the heart causes: a) a fall in diastolic time b) a fall in dP/dt c) an increase in stroke volume d) dilation of the coronary vasculature e) an increase in myocardial oxygen consumption 6. If oxygen is added to inspired air to increase its partial pressure from 20 kPa (150 mmHg) to 60 kPa (450 mmHg): a) dissolved oxygen will increase approximately three-fold b) the oxygen content of the blood will increase approximately three-fold c) the PaN2 will remain the same

901

d) the PaO2 will increase approximately three-fold e) hypercarbia will be prevented 7. Pulmonary vascular resistance is increased by: a) serotonin b) hypocarbia c) hypoxia d) a fall in pH e) adrenaline 8. On changing from the upright to the supine position: a) baroreceptor activity decreases b) leg vein pressure is reduced c) the blood volume in the pulmonary circulation falls d) stroke volume increases e) renin activity increases 9. The fall in urine output associated with major trauma may be caused by: a) haemorrhage b) a rise in antidiuretic hormone activity c) a fall in aldosterone activity d) an increase in the level of circulating catecholamines e) a rise in corticosteroid output 10. In the foetal circulation before birth: a) the PO2 is higher in the ductus venosus than in the ductus arteriosus b) blood can go from the right atrium to the aorta without passing through the left atrium and ventricle c) the PO2 in the aortic arch is higher than in the descending aorta d) blood flowing through the foramen ovale comes principally from the superior vena cava e) blood passes through the ductus arteriosus because of the high pulmonary vascular resistance 11. In the central venous pressure waveform: a) the a wave occurs after ventricular systole b) the v wave is caused by atrial contraction c) the a wave is absent in atrial fibrillation d) the a wave corresponds with closure of the aortic valve e) the v wave occurs during diastole 12. Renin activity is increased by: a) an increase in circulating adrenaline b) hypotension c) increased sodium ingestion

902

d) an increase in aldosterone output e) hypovolaemia 13. Acute antagonism of beta adrenergic receptors causes: a) hyperglycaemia b) peripheral vasodilatation c) suppression of uterine contractility d) pupillary dilatation e) a reduction in cardiac output 14. Unilateral transection of dorsal nerve roots C3-T2 produces: a) motor paralysis b) loss of sensation c) loss of reflexes d) loss of sympathetic and sudomotor tone e) hypotonia 15. The rate of gastric emptying is: a) delayed by fat in the duodenum b) delayed by secretin c) delayed by fat in the oesophagus d) enhanced by alcohol e) independent of volume and type of food ingested 16. An increase in aldosterone production occurs in response to: a) ingestion of sodium chloride b) an increase in blood volume c) an increased intake of potassium d) angiotensin II e) trauma 17. Cerebrospinal fluid: a) is the main source of brain nutrition b) is mainly produced by active secretion from the choroid plexus c) contains virtually no glucose d) pH changes rapidly in response to changes in plasma pH e) pressure increases with jugular venous obstruction 18. Carbonic anhydrase is found at high concentration in: a) plasma b) red blood cells c) renal tubular cells d) gastric parietal cells

903

e) cardiac muscle cells 19. The Hb-oxygen dissociation curve shifts to the right in: a) acute hypoxia b) stored blood c) metabolic acidosis d) respiratory alkalosis e) hypothermia 20. The functional residual capacity: a) is increased in the obese b) is approximately 10% higher in men than in women c) falls with general anaesthesia d) increases on changing from the supine to the standing position e) falls with increasing age ANSWERS 1. FFTFT 2. TFTTF 3. TTTFF 4. TFTFT 5. TFTTT 6. TFFTF 7. TFTTF 8. FTFTF 9. TTFTT 10. TTTFT 11. FFTFT 12. TTFFT 13 FFFFT 14. FTTFF 15. TTFFF 16. FFTTT 17. FTFFT 18. FTTTF 19. FFTFF 20. FTTTF Physiology MCQ 1 1. In calculating the shunt fraction, the following need to be measured or estimated: a) mixed venous oxygen content b) pulmonary end-capillary oxygen content c) arterial oxygen content

904

d) alveolar partial pressure of oxygen e) haemoglobin concentration 2. Pulse pressure increases with an increase in: a) stroke volume b) left ventricular end-diastolic volume c) arterial partial pressure of oxygen d) systemic vascular resistance e) blood viscosity 3. Acute untreated haemorrhagic shock in a patient will lead to: a) an increase in physiological dead-space b) an increase in the arterio-venous PCO2 difference c) a fall in the pulmonary vascular volume d) an increase in antidiuretic hormone secretion e) an increase in plasma bicarbonate concentration 4. An increase in aldosterone secretion follows: a) a sodium chloride load b) a rise in blood volume c) an increase in oral potassium absorption d) trauma c) an increase in production of angiotensin II 5. Stimulation of alpha-adrenergic receptors will cause: a) vasoconstriction of the coronary arteries b) increased tone in the bladder neck muscle c) increased platelet aggregation d) lipolysis e) bronchodilation ANSWERS 1.TTTTT 2.TTFFF 3.TFTTF 4.FFTTT 5.TTTFF

905

Physiology MCQ 2 6. In the normal adult heart: a) mitral valve closure occurs before tricuspid valve closure b) pulmonary valve closure occurs before aortic valve closure c) there is isometric contraction of the left ventricle after the aortic valve opens d) atrial contraction is of more importance to ventricular filling if the heart rate increases e) the aortic valve cusps are immobile during ventricular filling 7. Resistance to laminar flow in a vessel is: a) proportional to wall thickness b) inversely proportional to the fourth power of the radius c) proportional to length d) independent of haematocrit e) proportional to the pressure drop 8. Autoregulatory mechanisms used in hypovolaemia include: a) an increase in precapillary sphincter tone b) an increase in capillary hydrostatic pressure c) a decrease in baroreceptor activity d) stimulation of the juxtaglomerular apparatus e) an increase in angiotensin II 9. Myocardial contractility is increased by: a) catecholamines b) an increase in heart rate c) an increase in fibre length d) an increase in parasympathetic nervous system activity e) calcium ions 10. The carotid sinuses: a) have stretch receptors in their walls b) give afferent impulses via the glossopharyngeal nerve c) stimulate the respiratory centre d) contain chemoreceptors e) stimulate the vasomotor centre

ANSWERS 6.TFFTT

906

7.FTTFF 8.TFTTT 9.TFTFT 10.TTFFT Physiology MCQ Bank 3 11. The following cause a decrease in the arterial partial pressure of oxygen: a) anaemia b) carbon monoxide c) hyperventilation d) a rise in physiological dead-space e) old age 12. Iron absorption is dependent on: a) total body vitamin C b) HCl in the stomach c) an intact colonic mucosa d) total body iron e) erythropoietin levels in the blood 13. If a normal person hyperventilates for 2 hours to an arterial PCO2 of 4 kPa: a) the cerebral blood flow decreases b) the standard bicarbonate decreases c) the Hb-oxygen dissociation curve shifts to the left d) the ionised calcium concentration decreases e) the plasma bicarbonate increases 14. Hypoglycaemia may result from: a) excessive insulin secretion b) alpha-adrenergic stimulation c) beta-adrenergic stimulation d) glucagon secretion e) hypothermia 15. Ingested lipid: a) is important in prostaglandin synthesis b) increases in the faeces with a decrease in bile secretion c) is absorbed via the intestinal lymphatics d) is mainly in the form of triglycerides e) can be used as a source of ATP production

907

ANSWERS 11.FFFFT 12.FTFTF 13.TFTTF 14.TFFFF 15.TTTTT

16. The following may be found in normal adult venous blood: a) 3% carboxyhaemoglobin b) 5% methaemoglobin c) 70% oxyhaemoglobin d) 2% free haemoglobin e) 2% haemoglobin F 17. In normal human lungs: a) a low PO2 produces pulmonary vasodilatation b) beta-2 agonists cause bronchoconstriction c) pulmonary vascular resistance is increased by serotonin d) pulmonary vascular resistance is decreased by histamine e) pulmonary vascular resistance is decreased by noradrenaline 18. The normal response to surgery includes: a) a decrease in urine volume b) a decrease in the urinary excretion of sodium c) a decrease in plasma cortisol level d) an increase in the urinary excretion of nitrogen e) an increase in the urinary excretion of potassium 19. Capillary permeability is increased by: a) bradykinin b) adrenaline c) calcium d) vasopressin c) histamine 20. Expected changes in a patient with a phaeochromocytoma include: a) a decreased haematocrit b) a decreased total blood volume c) a decreased serum sodium concentration d) an abnormal glucose tolerance test e) a reduced metabolic rate

908

ANSWERS 16.TFTFT 17.FFTFF 18.TTFTT 19.TFFFT 20.FTFTF Physiology MCQ 5 1. The liver: a) receives most of its oxygen supply from the portal vein b) has its highest oxygen tension at the centre of a lobule c) produces heparin d) has a normal portal venous pressure of greater than 20 mmHg e) receives approximately 25% of the cardiac output 2. Skeletal muscle blood flow: a) increases with noradrenaline b) receives 50% of the cardiac output at rest c) may cease during isometric contraction d) increases with rhythmic contraction e) increases with adrenaline 3. In thermoregulation: a) respiratory heat loss is insignificant under normal conditions b) brown fat is an important source of heat production in neonates c) shivering is due to impulses conducted via autonomic efferents d) peripheral vasoconstriction increases heat production e) sweating is mediated by sympathetic cholinergic neurones 4. Adrenaline: a) is synthesized by demethylation of noradrenaline b) increases coronary blood flow c) increases free fatty acids in the blood d) mobilizes glycogen stores from the liver e) is metabolized in the plasma by monoamine oxidase 5. Glucagon: a) is a positive inotrope b) is produced by the beta cells of the pancreas c) stimulates production of free fatty acids in the blood d) release is increased in starvation e) stimulates glycogen synthesis

909

ANSWERS 1.TFTFT 2.FFTTT 3.TTFFT (fundamentals of anaesthesia, 2nd edition page 446) 4.FTTTF 5.TFTTF Physiology MCQ Bank 6 6. Surfactant: a) is a mucopolypeptide b) causes a decrease in surface tension c) results in the same surface tension for different sized alveoli d) causes an increase in compliance e) production is reduced after a prolonged reduction in pulmonary blood flow 7. Sinus arrhythmia: a) produces a lengthening of the P-R interval b) produces a lengthening of the R-R interval c) is maximal with breath holding d) is more marked during exercise e) is more marked in 70 year olds than in 20 year olds 8. A pressure-volume curve can be used for measuring: a) the work of breathing b) functional residual capacity c) anatomical dead space d) compliance e) respiratory quotient 9. The absolute refractory period for cardiac muscle is: a) as long as the entire action potential b) the period when no further action potential can be stimulated c) twice the length of the S-T interval d) as long as the mechanical contraction e) shorter for pacemaker tissue than for normal cardiac muscle 10. In a young normal adult: a) the glomerular filtration rate is approximately 125 ml/min b) the 24 hour urine creatinine content is approximately 800 mg

910

c) urine specific gravity is always less than 1000 d) renal blood flow is approximately 20% of cardiac output e) over 50% of water reabsorption from the glomerular filtrate occurs in the collecting ducts ANSWERS 6.FTFTT 7.FTFFF 8.TFFTF 9.FTFFT 10.TFFTF Physiology MCQ Bank 7 11. Total plasma calcium: a) increases with phosphate b) increases with a rise in albumin, c) changes its degree of ionisation with pH changes d) is decreased in osteoporosis e) is affected by vitamin D 12. There is increased intestinal motility with: a) increased intraluminal pressure b) anticholinesterase drugs c) sympathetic block to T4 d) stimulation of the splanchnic nerves e) increased circulating adrenaline 13. A healthy adult breathing an FIO2 of 0.1 will: a) have a decreased cardiac output b) have a normal PaO2 c) have a changed alveolar PCO2 d) have an unchanged respiratory rate e) initially have a fall in pH 14. Acetylcholine is a neurotransmitter at: a) sweat glands b) the adrenal medulla c) the parotid gland d) parasympathetic ganglia e) the neuromuscular junction

911

15. Insulin: a) has the same effect on blood sugar as growth hormone b) inhibits entry of potassium into cells c) facilitates protein anabolism d) increases deposition of fats e) secretion is affected by catecholamines ANSWERS 11.FTTFT 12.TTTFF 13.FFTFF 14.TTTTT 15.FFTTT

Physiology MCQ Bank 8 16. In a normal resting subject, a bradycardia would be expected following: a) an increase in carotid sinus pressure b) an increase in right atrial pressure c) application of pressure to the eyeball d) the release of a Valsalva manoevre e) inspiration 17. The velocity of conduction of a nerve action potential: a) is inversely related to the cross-sectional area of the axon b) is faster in a myelinated fibre than in an unmyelinated one c) is decreased by cooling the nerve d) can exceed 100 m/s in humans e) is highest in pre-ganglionic autonomic fibres 18. The placenta: a) transports glucose from maternal to foetal blood by facilitated diffusion b) can synthesize glycogen c) actively transports oxygen from maternal to foetal blood d) allows protein molecules to pass from maternal to foetal blood by pinocytosis e) secretes oestradiol 19. Oxytocin: a) stimulates production of milk b) stimulates ejection of milk

912

c) release is stimulated by dilatation of the cervix d) is synthesized in the anterior pituitary e) produces more powerful uterine contraction in the presence of progesterone 20. Aldosterone: a) production increases with a fall in plasma osmolality b) production decreases with a fall in blood volume c) production decreases with a rise in plasma renin level d) increases urinary potassium excretion e) may be produced by tumours of the adrenal cortex ANSWERS 16.TFTTF 17.FTTTF 18.TFFFT 19.FTTFF 20.FFFTT

Physiology MCQ Bank 9 1. Adrenaline a) is secreted by the adrenal cortex b) decreases systemic vascular resistance c) decreases pulmonary vascular resistance d) constricts the pupil e) acts only at beta-1 receptors 2. A typical mammalian motor neurone: a) innervates only one skeletal muscle cell b) is myelinated c) has its cell body in the ventral (anterior) horn of the spinal cord d) might receive an input directly from Group Ia afferent fibres in the spinal cord e) would be stimulated by application of glycine to its cell body 3. The vagus nerve: a) has little direct effect on the strength of ventricular contraction b) contains afferent and efferent fibres c) contains parasympathetic post-ganglionic fibres d) contains fibres which regulate gastric acid secretion e) has a role in bladder emptying

913

4. Vital capacity: a) is the volume of air expired from full inspiration to full expiration b) increases gradually with age in adults c) is greater in men than in women of similar age and height d) is equal to the sum of the inspiratory and expiratory reserve volumes e) may be measured by spirometry 5. Hyperventilation in a normal subject for 24 hours will produce a: a) fall in PaCO2 b) rise in PaCO2 c) rise in ionised calcium d) fall in cerebrospinal fluid bicarbonate e) rise in plasma bicarbonate ANSWERS 1.FFTFF 2.FTTTF 3.TTFTF 4.TFTFT 5.TFFFF Physiology MCQ Bank 10 6. The blood-brain barrier: a) results in certain molecules in the blood taking longer to equilibrate with tissue fluid in the brain than with tissue fluid elsewhere b) permits CO2 to pass freely c) is more permeable to water-soluble substances than fat-soluble substances d) is more permeable in neonates than in adults e) is readily crossed by dopamine 7. A reflex action: a) may be carried out by skeletal, smooth or cardiac muscle or by glands b) is not influenced by higher centres in the brain c) results from activity in at least two central nervous synapses in series d) may involve simultaneous contraction of some skeletal muscles and relaxation of others e) can be monosynaptic or polysynaptic 8. Platelets: a) are produced in the bone marrow b) increase in number after tissue damage c) have a small nucleus d) alter their shape when they make contact with collagen e) are activated by ADP and thrombin

914

9. The pressure: a) drop across the major veins is similar to that across the major arteries b) drop across the hepatic portal bed is similar to that across the splenic vascular bed c) in the hepatic portal vein is higher than that in the inferior vena cava d) drop across the vascular bed in the foot is greater when standing than when lying down e) drop across the pulmonary circulation is the same as across the systemic circulation 10. Athletes differ from normal individuals in having: a) a higher resting cardiac output b) a higher resting heart rate c) a decreased muscle mass d) a higher maximum oxygen consumption e) increased muscular efficiency at high blood lactate levels ANSWERS 6.TTFTF 7.TFFTT 8.TTFTT 9.FFTFF 10.FFFTT Physiology MCQ Bank 12 1. Stimulation of the parasympathetic nervous system: a) increases the heart rate b) decreases the rate of gastric emptying c) dilates the pupil d) causes vasoconstriction e) causes contraction of the detrusor muscle in the bladder 2. Pregnancy at term is associated with a: a) 20% decrease in red cell mass b) rise in cardiac output c) fall in PaCO2 d) fall in haematocrit e) low protein-bound iodine 3. The P5O is: a) the oxygen saturation when the arterial partial pressure of oxygen is 50 mmHg b) the arterial oxygen tension when haemoglobin is 50% saturated c) an indicator of the position of the oxygen dissociation curve d) raised in foetal blood e) lowered in chronic anaemia

915

4. In a healthy adult human heart the: a) left ventricular end-systolic volume is approximately 30 ml b) first heart sound coincides with the onset of ventricular systole c) stroke volume is approximately 70 ml d) left ventricular end-diastolic pressure is about 50 mmHg e) second heart sound is caused by closure of the aortic and pulmonary valves 5. Renin: a) is released from granules in the juxtaglomerular cells of the afferent arteriole b) levels in the blood increase in response to a fall in plasma sodium concentration c) levels in the blood decrease when renal perfusion is decreased d) release is enhanced by angiotensin II e) release is inhibited by antidiuretic hormone ANSWERS 1.FFFFT 2.FTTTF 3.FTTFF 4.TTTFT 5.TTFFT Physiology MCQ Bank 13 1. The following are secreted from the adrenal cortex: a) testosterone b) aldosterone c) angiotensin d) noradrenaline e) deoxycorticosterone 2. Aldosterone: a) secretion increases in response to a fall in blood volume b) is a polypeptide c) produces an increase in renal arterial pressure d) produces a fall in urine volume e) increases the reabsorption of sodium 3. Cerebrospinal fluid: a) is actively secreted by the choroid plexus b) is the major nutrition source of the brain c) has the same pH as arterial blood d) contains virtually no glucose e) has a higher chloride level than plasma

916

4. Red blood cell production: a) increases during acclimatisation to altitude b) can occur in the spleen c) is dependent on normal gastric secretory activity d) is stimulated by hypercarbia e) is dependent on ervthropoietin 5. Bile: a) salts contribute to the solubility of cholesterol in the bile b) contains bilirubin mainly in the unconjugated form c) contributes more than pancreatic secretion to the neutralisation of acid from the stomach d) becomes more alkaline following concentration in the gall bladder e) is produced at a rate of approximately 2000 ml/day ANSWERS 1.TTFFT 2.TFTTT 3.TFFFT 4.TTTFT 5.TFFFF Physiology MCQ Bank 14 1. Carbonic anhydrase plays a role in the: a) production of HCl by the parietal cells of the stomach b) secretion of hydrogen ions from the fluid in the renal tubules c) passage of CO2 from the pulmonary capillaries to the alveoli d) secretion of bicarbonate by the pancreas e) production of red blood cells 2. In a patient with severe hypovolaemia, the: a) physiological dead-space increases b) arterio-venous oxygen difference decreases c) alveolar-arterial oxygen difference increases d) minute volume increases e) arterial PCO2 increases 3. The group A antigen: a) is present on the red cells of a group A patient b) may occur in the saliva of a group A patient c) is transmitted as an autosomal recessive characteristic d) is more common than the group B antigen e) is the most common cause of haemolytic disease of the newborn

917

4. The action of noradrenaline maybe terminated by: a) monoamine oxidase in the nerve terminal b) catechol-O-methyltransferase in the liver c) catechol-O-methyltransferase in the nerve terminal d) dopa decarboxylase in the nerve terminal e) neuronal reuptake 5. An increase in the 2.3-DPG concentration in red blood cells occurs in: a) anaemia b) acclimatisation to altitude c) stored blood d) trained athletes e) cyanotic heart disease ANSWERS 1.TTFFF 2.TFTTF 3.TTFTF 4.TTFFT 5.TTFFT Physiology MCQ Bank 15 1. In normal cerebrospinal fluid, the: a) chloride concentration is higher than in blood b) glucose concentration is the same as in plasma c) PCO2 is higher than in mixed venous blood d) pH is the same as in arterial blood e) bicarbonate concentration is the same as in arterial blood 2. When breathing out against a closed glottis, the: a) intratracheal pressure rises b) heart rate slows transiently c) right ventricular output increases d) left ventricular output has a sustained increase e) systolic arterial pressure falls then rises 3. Changing position from standing to supine: a) increases stroke volume b) increases baroreceptor activity c) increases the pulmonary blood volume d) decreases leg vein pressure e) decreases the heart rate

918

4. Vagal stimulation produces: a) a fall in heart rate b) an increase in atrial contractility c) an increase in ventricular contractility d) slowing of A-V conduction e) a fall in stroke volume 5. The following are representative of myocardial afterload: a) mean aortic pressure b) mean pulmonary artery pressure c) left ventricular end-diastolic volume d) left ventricular end-diastolic pressure e) the rate of rise of left ventricular pressure ANSWERS 1.TFTFF 2.TTFFF 3.TTTTT 4.TFFTF 5.TTFFF Physiology MCQ Bank 16 1. On ascending to an altitude of 6000m, changes include: a) an increase in minute volume b) an initial increase in plasma pH c) a rise in urine pH d) a fall in arterial PO2 e) an increase in cerebral blood flow 2. Transferrin is: a) involved in iron uptake by the gut mucosa b) involved in iron transport across the gut mucosa c) involved in iron transport to muscle d) involved in iron transport to storage sites e) normally only 35% saturated with iron 3. Breathing 100% oxygen at atmospheric pressure for a prolonged period causes: a) retrosternal pain b) dizziness c) auditory disturbances d) convulsions e) atelectasis

919

4. The following transfusions will lead to agglutination: Donor Recipient a) B O b) AB A c) B AB d) O AB e) AB O 5. Chemoreceptors in the arterial system: a) have a higher rate of oxygen consumption per gram than brain tissue b) respond to changes in oxygen tension and not content c) respond to changes in pH d) conduct afferent information via the glossopharyngeal and vagus nerves e) are found in the carotid sinus ANSWERS 1.TTTTF 2.FFTTT 3.TFFFT 4.TTFFT 5.FTTFF Physiology MCQ Bank 17 1. In the healthy heart, an increase in stroke volume is seen with an increase in: a) dP/dT b) aortic systolic pressure c) left ventricular end-diastolic volume d) left ventricular end-systolic pressure e) heart rate 2. The a-wave of the central venous pressure waveform: a) is caused by atrial contraction b) is not seen in atrial fibrillation c) is caused by atrial filling during ventricular contraction d) decreases with inspiration e) is followed by the v-wave 3. In the electrocardiogram, the: a) P-R interval is equivalent to the A-V nodal conduction time b) T-wave is equivalent to ventricular repolarisation c) Q-T interval is equivalent to the duration of ventricular contraction d) U-wave represents sinoatrial node repolarisation e) duration of a normal P-wave is 0.2 seconds

920

4. Ptosis results from: a) parasympathetic block b) sympathetic block c) facial nerve block d) trigeminal nerve block e) oculomotor nerve block 5. The following are precursors of adrenaline: a) tyrosine b) phenylalanine c) noradrenaline d) dopamine e) isoprenaline ANSWERS 1.TFTFF 2.TTFFF 3.TTTFF 4.FTTFT 5.TTTTF Physiology MCQ Bank 18 1. The following lead to an increase in insulin secretion: a) glucagon b) adrenaline c) growth hormone d) starvation e) major trauma 2. Adenyl cyclase: a) increases the conversion of ATP to cyclic AMP b) is closely linked to alpha- and beta-adrenergic receptors c) is inhibited by aminophylline d) release is triggered by cyclic AMP e) acts at a mitochondrial level 3. Surfactant: a) contains phospholipids b) prevents oedema formation in the alveolar wall c) reduces surface tension by approximately 30% d) produces a monomolecular layer e) stabilises the size of an alveolus

921

4. The oxyhaemoglobin dissociation curve is shifted to the left by: a) an increase in arterial PCO2 b) acidosis c) chronic anaemia d) carbon monoxide e) a fall in temperature 5. In the adult, growth hormone stimulates: a) glucose uptake into cells b) calcium absorption from the gut c) protein synthesis d) fat synthesis e) bone growth ANSWERS 1.TFTFT 2.TTFFF 3.TTFFT 4.FFFTT 5.FFTFT

Pharmacology MCQS 1. Cimetidine: a) lowers plasma creatinine b) prolongs the prothrombin time c) hastens gastric emptying d) may potentiate the action of warfarin e) may impair cardiac conduction 2. Acute intermittent porphyria: a) is exacerbated by digoxin b) is precipitated by sulphonamides c) is precipitated by ethyl alcohol d) is treated by parenteral lidocaine e) contraindicates the use of thiopentone 3. The following are features of digoxin toxicity: a) headache b) nausea c) abdominal pain d) convulsions

922

e) coupled beats 4. Lidocaine can cause: a) sedation b) convulsions c) slowed A-V conduction d) prolongation of the cardiac action potential e) shortening of the refractory phase 5. Features of severe aspirin overdose include: a) tinnitus b) metabolic acidosis c) a reduction in the platelet count d) haemolysis e) hyperventilation 6. Atropine antagonises the action of: a) some of the effects of morphine b) acetylcholine at the neuromuscular junction c) acetylcholine at sweat glands d) trimetaphan at the autonomic ganglia e) pilocarpine on the pupil 7. Toxic effects of sodium nitroprusside: a) are not related to dose b) are due to cyanide ions c) are due to thiocyanate d) are due to liver rhodanese e) may be treated with vitamin B12 8. Pethidine: a) causes less miosis than morphine b) is metabolised to active substances with analgesic properties c) has a longer duration of action than morphine d) possesses atropine-like activity e) has some local anaesthetic activity 9. The following are useful in the suppression of ventricular ectopic beats: a) digoxin b) amiodarone c) mexiletine d) verapamil e) bretylium

923

10. The following act directly on vascular smooth muscle by altering calcium transport: a) nicardipine b) sodium nitroprusside c) hydralazine d) bendrofluazide c) dantrolene 11. Metoclopramide: a) has an action on gastric emptying opposed by atropine b) causes prolactin release c) is a phenothiazine d) lowers the blood glucose e) is a dopamine antagonist 12. Otoxicity can follow the administration of: a) ethacrynic acid b) gentamicin c) frusemide d) cefuroxime e) streptomycin 13. Poisoning by organophosphorus compounds causes: a) increased bronchial secretions b) constipation c) miosis d) tetanus e) tachycardia 14. Phenobarbitone: a) is used in the treatment of grand mal epilepsy b) may aggravate petit mal epilepsy c) can produce skin rashes d) is rapidly metabolised e) has its effects terminated by redistribution 15. Testosterone derivatives are: a) androgenic b) diabetogenic c) progestogenic d) anabolic e) carcinogenic

924

16. Buprenorphine: a) causes little nausea and vomiting b) has a respiratory depressant action antagonised by naloxone c) is mostly metabolised in the liver d) may cause withdrawal symptoms in morphine addicts e) must be given parenterally 17. The following should not be discontinued abruptly: a) cimetidine b) propranolol c) clonidine d) amiodarone e) digoxin 18. Sulphonylureas: a) are used in maturity-onset diabetes b) can cause hypoglycaemia c) are suitable for the treatment of ketoacidosis d) are an effective treatment for hyperglycaemia following total pancreatectomy e) can cause lactic acidosis 19. Hyoscine: a) causes tachycardia b) causes sedation c) causes mydriasis d) is an antiemetic e) has a weaker antisialagogue effect than atropine 20. Salbutamol can cause: a) tremor b) an increase in uterine contractility c) a worsening of intermittent claudication d) hypokalaemia e) tachycardia ANSWERS 1. FFFTT 2. FTTFT 3. TTTTT 4. TTTFT 5. TTFFT 6. FFTFT 7. TTFFF 8. FTFTT

925

9. FTTFT 10. TTTFF 11. TTFFT 12. TTTFT 13. TTTFT 14. TTTFF 15. TTFTT 16. FFTTF 17. FTTFF 18. TTFFF 19. TTTTF 20. TFFTT Pharmacology MCQ 1 1. Dopamine: a) may produce ventricular arrhythmias b) increases mesenteric blood flow at high doses c) crosses the blood-brain barrier d) is synthesised from L-dopa e) is inactivated in alkaline solution 2. The following increase the amount of calcium in cardiac muscle: a) halothane b) adrenaline c) diltiazem d) nifedipine e) trimetaphan 3. The following can be used safely in a patient on a monoamine oxidase inhibitor: a) morphine b) pethidine c) noradrenaline d) amphetamine e) adrenaline 4. Hyoscine hydrobromide causes: a) antiemesis b) somnolence c) pupillary dilatation d) tachycardia followed by bradycardia e) extrapyramidal symptoms

926

5. Clonidine: a) is an alpha-2 receptor agonist b) is a dopamine antagonist c) causes tachycardia d) inhibits salivation e) reduces the minimum alveolar concentration of halothane ANSWERS 1.TTFTT 2.FTFFF 3.TFTFT 4.TTTTF 5.TFFTT Pharmacology MCQ Bank 2 6. Hydralazine: a) is metabolised by acetylation b) is destroyed by plasma cholinesterase c) can cause a lupus-like syndrome d) stimulates the baroreceptor reflex e) is contraindicated in pregnancy 7. The following are prodrugs: a) suxamethonium b) diamorphine c) captopril d) paracetamol e) enalapril 8. The following drugs penetrate the blood-brain barrier: a) physostigmine b) dopamine c) propranolol d) glycopyrrolate e) noradrenaline 9. Naloxone: a) is an agonist at kappa receptors b) is an antagonist at mu receptors c) reverses ventilatory depression due to morphine d) may precipitate opiate withdrawal symptoms e) may cause pulmonary oedema

927

10. The following are precursors of adrenaline: a) tyrosine b) phenylalanine c) dopamine d) isoprenaline e) noradrenaline ANSWERS 6.TFTTF 7.FTFFT 8.TFTFF 9.FTTTT 10.TTTFT Pharmacology MCQ Bank 3 11. The following affect gastric emptying: a) diamorphine b) diazepam, c) metoclopramide d) cisapride e) omeprazole 12. Chlorpromazine: a) can cause dystonic reactions b) antagonises apomorphine-induced vomiting c) is a dopamine antagonist at the chemoreceptor trigger zone d) is a weak alpha-adrenergic agonist e) undergoes extensive first-pass metabolism 13. Alfentanil: a) is less lipid soluble than fentanyl b) relaxes the sphincter of Oddi c) has active metabolites d) has a large volume of distribution e) causes analgesia without sedation 14. Folic acid metabolism is impaired by: a) nitrous oxide b) sodium nitroprusside c) sulphonamides d) penicillin e) trimethoprim

928

15. Significant agonist activity at opioid receptors occurs with: a) clonidine b) pentazocine c) buprenorphine d) ketamine e) naloxone ANSWERS 11.TFTTF 12.TTTFT 13.TFFFF 14.TFTFT 15.FTTFF Pharmacology MCQ Bank 5 1. The following act by blocking cholinergic receptors: a) trimetaphan b) hexamethonium c) ouabain d) benzhexol e) physostigmine 2. Esmolol: a) has a negative inotropic effect b) has no intrinsic sympathomimetic activity c) causes a dose-dependent fall in heart rate d) increases airways resistance e) may prolong the duration of action of suxamethonium 3. Enflurane: a) is a halogenated methyl-ethyl ether b) has a blood/gas solubility coefficient of 1.43 c) lowers intracranial pressure d) increases the tone of the pregnant uterus e) causes a decrease in systemic vascular resistance 4. Morphine: a) causes histamine release b) has no active metabolites c) undergoes extensive first-pass hepatic metabolism d) has an elimination half-life of 3-4 hours e) may be antagonised by pentazocine

929

5. Uterine tone is: a) increased by ketamine b) decreased by halothane c) increased by beta-2 antagonists d) unaffected by suxamethonium e) decreased by nifedipine ANSWERS 1.TTFTF 2.TTTTT 3.TFFFT 4.TFTTT 5.TTTTF Pharmacology MCQ Bank 6 6. Aspirin overdose causes: a) thrombocytopenia b) coma c) metabolic acidosis d) jaundice e) pulmonary oedema 7. Non-depolarising neuromuscular blockers are potentiated by: a) lithium b) diazepam c) trimetaphan d) magnesium e) suxamethonium 8. Atropine: a) crosses the blood-brain barrier b) can cause an initial bradycardia c) increases the rate of gastric emptying d) increases conduction through the AV node e) produces bronchoconstriction 9. Hepatotoxicity has been associated with: a) diethyl ether b) chloroform c) cyclopropane d) enflurane e) halothane

930

10. The anticoagulant effect of warfarin: a) is exerted directly on the blood b) is slow in onset c) can be reversed by vitamin K d) is potentiated by phenylbutazone c) is potentlated by barbiturates ANSWERS 6.TTTFT 7.TFTTT 8.TTFTF 9.FTFTT 10.FTTTF Pharmacology MCQ 7 11. The following may be used in the treatment of digoxin toxicity: a) propranolol b) lidocaine c) phenytoin d) calcium e) potassium 12. Dobutamine: a) is structurally similar to isoprenaline b) activates adenyl cyclase c) has a selective action on beta-1 adrenoreceptors d) has a half-life of 2 minutes e) increases the left ventricular end-diastolic pressure 13. Organophosphorus anticholinesterases: a) have an irreversible action b) phosphorylate cholinesterase c) produce more inhibition of acetyl cholinesterase than of plasma cholinesterase d) can have their action reversed in the early stages by atropine e) are readily absorbed through the skin 14. The following factors encourage passage of a substance across the cell membrane: a) high lipid solubility b) low concentration gradient c) high molecular weight d) negative hydrostatic pressure e) high degree of ionisation

931

15. Amitriptyline in overdose causes: a) cardiac arrhythmias b) hypertension c) restlessness d) metabolic acidosis e) jaundice ANSWERS 11.TTTFT 12.TTFTF 13.TTFTT 14.TFFFF 15.TFTTF Pharmacology MCQ 8 16. Low plasma cholinesterase activity: a) is related to the patient's blood group b) has no effect on the action of decamethonium c) occurs with organophosphorus poisoning d) prolongs the action of esmolol e) occurs in malnutrition 17. Sulphonylureas: a) acts by increasing insulin release b) tends to produce weight loss c) is suitable for use in pregnancy d) is effective in correcting ketoacidosis e) can produce hypoglycaemia 18. Side-effects of hydralazine include: a) tachycardia b) impotence c) constipation d) systemic lupus erythematosus e) bronchoconstriction 19. The following are measures of scatter in statistical analysis: a) mean b) standard error of the mean c) standard deviation d) range e) p of less than 0.5

932

20. Statistical tests are used to: a) eliminate observer bias b) eliminate placebo effect c) show that results are true d) show that the results did not occur by chance e) show that the results are clinically significant ANSWERS 16.FFTTT 17.TFFFT 18.TFFTF 19.FTTTF 20.FFFTF Pharmacology MCQ Bank 9 1. Nifedipine: a) causes tachycardia b) causes tremor c) causes hypoglycaemia d) can be given sublingually e) dilates skeletal muscle arterioles 2. Tetracycline: a) may be hepatotoxic in large doses b) is extensively metabolised c) is contraindicated in pregnancy d) potentiates non-depolarising muscle relaxants e) absorption is decreased by magnesium trisilicate 3. Neostigmine: a) is eliminated entirely by the liver b) causes foetal bradycardia when given to a pregnant woman c) plasma levels correlate well with reversal of neuromuscular blockade d) causes miosis e) is a quaternary ammonium compound 4. The following drugs readily cross the placenta: a) atracurium b) lidocaine c) bupivacaine d) neostigmine e) propranolol

933

5. In acute intermittent porphyria, the following, drugs may be safely used: a) etomidate b) ketamine c) midazolam d) pethidine e) cotrimoxazole ANSWERS 1.TFFTT 2.TFTTT 3.FFFTT 4.FTFFT 5.FTTTF Pharmacology MCQ Bank 10 6. The following drugs are mainly eliminated from the body by hepatic metabolism: a) isoflurane b) morphine c) atracurium d) suxamethonium e) dopamine 7. The following have an elimination half-life of greater than 24 hours: a) diazepam b) midazolam c) methadone d) gelofusine e) hydroxyethyl starch 8. Phenytoin: a) shows first-order elimination kinetics b) induces enzymes c) causes hypotension d) causes vitamin B2 deficiency e) has a half-life of about 4 hours 9. L-dopa: a) is more lipid soluble than dopamine b) may produce postural hypotension c) may cause nausea d) may cause abnormal movements e) is ineffective via the oral route

934

10. Toxicity from hyperbaric oxygen therapy can cause: a) painful joints b) pulmonary oedema c) acute tubular necrosis d) convulsions e) bradycardia ANSWERS 6.FTFFF 7.TFTFT 8.TTTFF 9.TTTTF 10.FTFTF Pharmacology MCQ Bank 11 11. Atropine: a) increases gut motility b) is a mixture of d- and l-isomers c) is equipotent with hyoscine d) can cause bradycardia e) crosses the blood-brain barrier 12. Beclomethasone: a) can be given by aerosol inhaler b) can cause candidiasis c) is a fludrocortisone d) increases the circulating cortisol concentration e) is a bronchodilator 13. Dopamine receptor antagonists: a) increase heart rate b) are antiemetics c) cause hypertension d) cause extrapyramidal side-effects e) increase lower oesophageal sphincter tone 14. The volume of stomach contents is reduced by: a) sodium citrate b) metoclopramide c) cimetidine d) atropine e) pyridostigmine

935

15. The following drugs relax the pregnant uterus: a) salbutamol b) propranolol c) noradrenaline d) acetylcholine e) dopamine ANSWERS 11.FTFTT 12.TTFFT 13.FTFTT 14.FTTFT 15.TFFFF Pharmacology MCQ 12 16. Carbonic anhydrase inhibitors: a) cause an alkaline urine to be produced b) cause a metabolic acidosis c) are more effective when given continuously than when given intermittently d) may lead to the formation of renal calculi e) cause the intraocular pressure to rise 17. Osmotic diuretics: a) are only effective if completely reabsorbed from the renal tubule b) reduce intracranial pressure primarily by inducing a diuresis c) produce a urine flow limited by glomerular filtration rate d) can lead to pulmonary oedema if renal function is impaired e) have a molecular weight of between 40,000 and 65,000 daltons 18. The following are ethers: a) sevoflurane b) halothane c) desflurane d) trichlorethylene e) enflurane 19. Warfarin: a) has a rapid onset of action b) is readily excreted in the urine c) is antagonised by salicylates d) is potentiated by metronidazole e) is antagonised by barbiturates

936

20. Salbutamol: a) is a beta-1 selective adrenoreceptor agonist b) acts on the lung only when given by the inhalational route c) reduces the forced expiratory volume in asthmatics d) produces vasodilatation in skeletal muscle e) can produce hypokalaemia ANSWERS 16.TTFTF 17.FFFTF 18.TFTFT 19.FFFTT 20.FFFTT

OSCE and Viva

Welcome to our section containing Primary FRCA OSCE and Viva questions. We have split the resources into relevant sections as per the exam format. In order to be eligible for a Viva a minimum of 1+ is required from the written MCQ paper. To pass overall, a minimum of 1+,2,2,2 is required. (1 poor fail, 1+ a fail, 2 pass, 2+ outstanding pass). Format of Primary FRCA OSCE Viva sections VIVAs Morning: Viva 1 1. Clinical Anaesthesia Viva (15 minutes) 2. Equipment and safety, physics and clinical measurement (15 minutes) Afternoon: Viva 2 1. Physiology and biochemistry (15 minutes) 2. Pharmacology and Statistics (15 minutes) OSCEs The OSCES consists of a series of stations around which the candidates rotate, spending 1 1/2 minutes at a preparation area before each station proper. The areas tested are resuscitation, technical skills, anatomy, history-taking, physical examination, communication skills, data interpretation, stats, monitoring and measurement and anaesthetic hazards.

937

Need minimum 2,2,2,1+ to pass the Primary FRCA.

Anatomy Primary Viva 1 (Anatomy) Primary Viva 2 (Anatomy) Pharmacology Primary Viva 1 (Pharmacology) Primary Viva 2 (Pharmacology) Primary Viva 3 (Pharmacology) Primary Viva 4 (Pharmacology) Primary Viva 5 (Pharmacology) Primary Viva 6 (Pharmacology) Primary Viva 7 (Pharmacology) Primary Viva 8 (Pharmacology) Primary Viva 9 (Pharmacology) Primary Viva 10 (Pharmacology) Primary Viva 11 (Pharmacology) Primary Viva 12 (Pharmacology) Physiology Primary Viva 1 (Physiology) Primary Viva 2 (Physiology) Primary Viva 3 (Physiology) Primary Viva 4 (Physiology) Primary Viva 5 (Physiology) Primary Viva 6 (Physiology) Primary Viva 7 (Physiology) Primary Viva 8 (Physiology) Primary Viva 9 (Physiology) Primary Viva 10 (Physiology) Primary Viva 11 (Physiology) Primary Viva 12 (Physiology) Clinical Primary Viva 1 (Clinical) Primary Viva 2 (Clinical) Primary Viva 3 (Clinical) Primary Viva 4 (Clinical) Primary Viva 5 (Clinical)

938

Primary Viva 6 (Clinical) Primary Viva 7 (Clinical) Primary Viva 8 (Clinical) Primary Viva 9 (Clinical) Primary Viva 10 (Clinical) Primary Viva 11 (Clinical) Primary Viva 12 (Clinical) Primary Viva 13 (Clinical) Physics Primary Viva 1 (Physics) Primary Viva 2 (Physics) Primary Viva 3 (Physics) Primary Viva 4 (Physics) Primary Viva 5 (Physics) Primary Viva 6 (Physics) Primary Viva 7 (Physics) Primary Viva 8 (Physics) Primary Viva 9 (Physics) Primary Viva 10 (Physics) Primary Viva 11 (Physics) Primary Viva 12 (Physics) Primary Viva 13 (Physics) Primary Viva 14 (Physics) Primary Viva 15 (Physics) OSCEs Primary OSCE 1 Primary OSCE 2 Primary OSCE 3 Primary OSCE 4 Primary OSCE 5 Primary OSCE 6 Primary OSCE 7 Primary OSCE 8 Primary OSCE 9 Primary OSCE 10 Primary OSCE 11 Primary OSCE 12 Primary OSCE 13 Primary OSCE 14 Primary OSCE 15 Primary OSCE 16 Primary OSCE 17 Primary OSCE 18 Primary OSCE 19

939

Primary OSCE 20 Primary OSCE 21 Primary OSCE 22 Primary OSCE 23 Primary OSCE 24 Primary Viva 1 (Anatomy) QUESTIONS 1. Discuss the sensory innervation of the larynx. 2. Draw the circle of Willis. 3. Look at a coronary angiogram and label each of the vessels. Which parts of the heart do the vessels supply? 4. Which roots do the brachial plexus come from? Which nerves do they support? 5. Show on an actor how you would perform an ankle block. What five nerves would you be trying to block? 6. Point out the coeliac plexus on a CT scan. What does it supply and what roots does it come from? How would you block it? 7. What layers do you pass through when performing spinal anaesthesia? 8. What is a 3 in 1 block and how is it performed? 9. What is your favourite block and how do you perform it? 10. Describe the course of the internal jugular vein. Primary Viva 2 (Anatomy) 1. Discuss the coronary circulation 2. Draw the anatomy of the coronary arteries 3. Which arteries supply the relevant parts of the heart? 2. What factors affect the coronary circulation? Primary Viva 1 (Pharmacology)

940

QUESTIONS 1. List the properties of an ideal anaesthetic agent (Click here for a tutorial on ideal anaesthetic agents). 2. Discuss how isoflurane measures up to these ideals. 3. Discuss the concept of deliberate hypotension. 4. Discuss the use of beta-blockers 5. Discuss the use of sodium nitroprusside. 6. Discuss ganglion blockade. 7. Discuss the different types of interactions that exist between drugs. Primary Viva 2 (Pharmacology) QUESTIONS 1. Discuss the characteristics of an ideal inhalational agent (Click here for a tutorial on this) 2. Define each of the following and explain why they are important: Oil-gas coefficients Minimum alveolar concentration (MAC) 3. What is the oil-gas coefficient and MAC of: Isoflurane Desflurane Sevoflurane Click here for a tutorial on the physical properties of the inhalational agents. 4. Discuss intraoperative antihypertensive agents and classify them into their appropriate drug groups. 5. Compare the effects of nitroprusside and nitroglyerine. 6. Discuss cyanide toxicity. Primary Viva 3 (Pharmacology) QUESTIONS 1. Isomers What is the definition and classification of isomers?

941

Give an example of a structural isomer. In the case of the example above, draw both isomers and state their physical properties (effects on boiling point (bp), saturated vapour pressure [SVP] and minimal alveolar concentration [MAC]). What is the relationship between bp and SVP? What is the bp/SVP of ether and methoxyflurane? Give a definition of stereoisomers and tautomers, and give examples of each. 2. Opioid receptors What is the classification, action and distribution of each type of receptor? How do specific agonists work at each receptor? How do opioids work at a molecular level? How does nalbuphine act? What is your favourite opioid? Discuss this in further detail. Why do morphine, fentanyl and alfentanil have such different onset times and durations of action? What is the significance of their volumes of distribution? You may like to read more about opioids by clicking here. Primary Viva 4 (Pharmacology) QUESTIONS 1. How do drugs work? 2. Define a ligand. 3. What is the natural ligand for adrenoceptors? 4. Sketch the shape of doseresponse and log doseresponse curves. 5. What is potency and ED50? Sketch the doseresponse curves of a drug plus a less potent and less efficacious drug and show the effects of competitive and non-competitive antagonists. 6. Draw the structures of propofol and thiopentone. 7. What properties does the S atom confer to thiopentone? What molecule is formed when the S atom is replaced by an O atom? 8. Describe the pharmacodynamic differences between propofol and thiopentone. 9. Where in the central nervous system do propofol and thiopentone act? 10. Discuss anticholinesterase inhibitors. What uses do they have in clinical practice? Besides anaesthesia, how else are they used?

942

11. Draw a schematic diagram of the anticholinesterase enzyme. Point out the two binding sites. What are they called? Primary Viva 5 (Pharmacology) QUESTIONS 1. Describe the properties of suxamethonium? 2. What is the mechanism of action of suxamethonium? 3. Discuss the concept of receptors in relation to pharmacology, with a particular focus on GABA receptors and adrenoceptors. 4. Discuss genetic conditions involving drug metabolism. 5. What is the ideal agent for total intravenous anaesthesia (TIVA)? Explain why. 6. Describe the pharmacokinetics of propofol? 7. Discuss routes of drug administration. Primary Viva 6 (Pharmacology) 1. Draw the concentrationtime and the log concentrationtime curves for propofol. Describe (in detail) how the plasma concentration is calculated. Discuss giving drugs by infusion. 2. What is the mode of action of non-steroidal anti-inflammatory drugs (NSAIDs)? What are the standard doses for drugs in this class? Compare and contrast the properties and uses of ketorolac and ibuprofen. 3. List the various types of anti-epileptic agents and describe their interactions with anaesthetic drugs. Primary Viva 7 (Pharmacology) 1. What are the characteristics of an ideal inhalational anaesthetic agent? 2. How do sevoflurane and isoflurane measure up to these characteristics? 3. What type of compound is compound A?

943

4. What are its effects in man? 5. Why is it nephrotoxic in animals but not in humans? 6. Which compounds form CO with dry soda lime, and why? 7. How do heparin and warfarin work? 8. Where is heparin found in humans? 9. Is it an acidic or a basic compound? 10. How are its actions reversed? 11. What are the side-effects of protamine? 12. What are the side-effects of heparin administration? 13. What are the advantages of low molecular weight heparins (LMWHs)? 14. Describe what is meant by MichaelisMenton kinetics. 15. Draw a graph of reaction velocity vs substrate concentration, and define first order/zero order kinetics. 16. Draw the structure of GABA receptors, showing their various binding sites. 17. Describe the structure of the NMDA receptor. 18. What would be your actions if you inadvertently gave a patient a syringe of suxamethonium? 19. What are the modes of action of the common anti-asthmatic drugs? 20. Discuss the pharmacology of adrenoreceptors. 21. Discuss the action of steroids. 22. How do antiepileptic agents work? (include mention of carbamazepine, phenytoin, benzodiazepines and their receptors) 23. How would you anaesthetise a sick diabetic patient for amputation of an infected leg? 24. What complications of diabetes have an effect on the administration of anaesthesia? 25. Discuss the technical aspects of spinal anaesthesia

944

Primary Viva 8 (Pharmacology) 1. Discuss the uptake of inhalational anaesthetics (include blood gas solubility, factors which modulate the rate of uptake of anaesthetics). 2. Describe the classification of non-depolarising muscle relaxants. What structural similarities account for both aminosteroids and benzyl isoquinolonium esters acting at the neuromuscular junction? 3. Discuss the mechanisms of the renal excretion of drugs. Muscle relaxation 4. What is the mode of action of non-depolarising muscle relaxants? 5. Explain the mode of action of reversal agents. 6. How are reversal agents classified? 7. What are the advantages/disadvantages of each type of reversal agent?

Primary Viva 9 (Pharmacology) Part I Answer the following questions on local anaesthetics: 1. Compare and contrast lidocaine and bupivacaine. How do they compare with ropivacaine? 2. What is the definition and significance of pKa with regard to the onset of action and duration of action of local anaesthetics? 3. What is the significance of the molecular weight of a local anaesthetic? 4. Discuss the influence of plasma protein binding in relation to the metabolism, duration of effects, side-effects and transplacental transfer of local anaesthetics. 5. What concentrations of local anaesthetics are used in spinal and epidural anaesthesia and in obstetrics? 6. Draw the chemical structure of lidocaine. 7. What are the differences between esters and amides, including the advantages and disadvantages of both? 8. Discuss the toxicity of local anaesthetics to the cardiovascular system, and explain how this should be managed. 9. What are the differences between esters and amides in terms of their central nervous system toxicities? Part II 1. Discuss the different types of isomerism, giving examples. 2. What are the clinical differences between isomers with regard to pharmacokinetics and side-effects? 3. What are the differences between atropine, and atracurium and cisatracurium?

945

Part III 1. Define bioavailability and explain how it is calculated for oral administration of a drug. Primary Viva 10 (Pharmacology) 1) Pharmacokinetics Define exponential Why are the logarithms of concentrations plotted? Explain the information that can be obtained from a log concentrationtime graph, including definitions. Define two-compartment model, zero and 1st order reactions, and non-linear kinetics. 2) Opioids What are their routes of administration? Describe the classification of opioid receptors. Draw the structure of morphine. Compare the characteristics of morphine, fentanyl, alfentanil and pethidine. 3) Regarding the treatment of diabetes What is the WHO/ADA classification? What do you know about the DCCT and UKPDS trials? Draw the structure of an insulin molecule.

Primary Viva 11 (Pharmacology) 1. Describe the appearance of a plasma concentrationtime graph for a drug. 2. What do you understand by the term exponential? 3. How else could these data be presented? 4. What information could you gain from the graph? Derive the volume of distribution (Vd), clearance (CL), elimination half-life (t) and the elimination rate constant (K)

946

What is the context-sensitive half time? Discuss how to calculate dosing intervals in relation to the data derived. Discuss one-, two- and three-compartment models, comparing muscle relaxants and propofol as an example. 5. Discuss the uses, presentation and mode of action of non-steroidal anti-inflammatory drugs (NSAIDs). What is the difference between COX-1 and COX-2 inhibition, and the respective sideeffects? Discuss prostaglandins, thromboxanes, prostacyclin and leukotrienes. Primary Viva 12 (Pharmacology) 1. Classify the neuromuscular blockers 2. Classify the non-depolarising neuromuscular blockers 3. How are the non-depolarising neuromuscular blockers metabolised? 4. Describe the metabolism of each of the neuromuscular blockers; what are the relative percentages excreted by the liver and the kidney? 5. How is atracurium metabolised? 6. What is Hoffmans degradation? 7. What are the advantages of atracurium? 8. Which muscle relaxant would you use in myasthenia gravis and at what dose? 9. How would you assess the blockade? 10. How is mivacurium metabolised? 11. What are the disadvantages of mivacurium? 12. How can the effects of mivacurium be reversed? 13. How does neostigmine work? 14. What are benzodiazepines? 15. Classify the benzodiazepines, giving examples

947

16. Describe the various effects of benzodiazepines 17. Describe the mechanism of action of the benzodiazepines 18. Describe the subtypes of GABA receptors and the mechanisms associated with them 19. How are the benzodiazepines metabolised? 20. What is the enterohepatic circulation? 21. What are the advantages of midazolam? 22. What are the principles of treating benzodiazepine poisoning? 23. Name a specific antidote for benzodiazepine overdose 24. What is the mechanism of action and half-life of flumazenil? 25. Describe the various uses of benzodiazepines 26. Explain how antiarrhythmic agents are classified Primary Viva 1 (Physiology) QUESTIONS 1. Discuss the cardiac action potential. 2. Discuss the pacemaker action potential. 3. Compare and contrast these action potentials, in terms of ionic changes, the various ion channels and the effect of autonomic innervation on electrical changes. 4. Discuss the autonomic innervation of the heart. 5. Discuss arterial blood gases (with reference to respiratory acidosis). 6. Discuss acidbase balance. What is the HendersonHasselbach equation? 7. Discuss how PCO2 affects PO2. 8. What is the alveolar gas equation? 9. What is the respiratory quotient? How does it change in chronic respiratory disease? 10. What are the non-respiratory functions of the lung?

948

Primary Viva 2 (Physiology) QUESTIONS 1. Discuss the pacemaker action potential. 2. Discuss both slow and fast calcium channels. 3. Describe the various phases of these slow and fast channels. Why do they occur? What are they? 4. Discuss the cardiac action potential. 5. Discuss the autonomic innervation of the heart. 6. How do autonomic changes occur? 7. Discuss the innervation of the sino-atrial (SA) and atrioventricular (AV) node. 8. Discuss the non-respiratory functions of lung. 9. Discuss the metabolic functions of the lung. 10. Discuss the immunological functions of the lung. 11. What pulmonary vasodilators do you know? 12. What is hypoxic pulmonary vasoconstriction (HPV)? 13. What happens to pO2 in the apex and base of the lung? You are now presented with blood gas results, showing: [H+], 7.1 nmol/L PCO2, 9.0 kPa; PaO2, 15.1 kPa. QUESTIONS 1. What do these values show? 2. Is oxygenation satisfactory in this patient? Does the patient need supplementary oxygen? 3. How does the body compensate in this situation? 4. What is the Henderson-Hasselbach equation? Primary Viva 3 (Physiology) QUESTIONS 1. Oxygen carriage How is oxygen carried in the blood? Draw a diagram showing the relationship between oxygen content and pO2. Re-label the following graph, which shows SaO2/pO2, and mark the significant points. What causes the slope of the graph to shift to the right/left? What causes movement of the venous point? 2. How does the body respond to an acute rise in blood pressure? Discuss this in further detail. 3. Describe in detail the endocrine component of the stress response to surgery.

949

Primary Viva 4 (Physiology) QUESTIONS 1. Define the Valsalva manoeuvre. Who was Valsava? 2. Sketch the mean arterial pressure (MAP) and heart rate (HR) changes in healthy and sympathectomised patients and in patients with congestive cardiac failure (CCF). 3. What reflex causes these changes and what nerves carry afferents in these conditions? 4. Where are the receptors mediating these responses, and where in the brain do the afferents relay to? 5. How many nuclei does the vagus nerve have? 6. What is the cerebral blood flow rate in ml/kg/min? 7. Does cerebral blood flow (CBF) vary from white to grey matter? Give values for these regions. 8. How is CBF measured? (you may like to consider the Kety-Schmidt technique and Doppler) 9. Define the Doppler effect. 10. Where is the jugular bulb? 11. Describe MonroKellie doctrine. 12. Sketch the autoregulation curve in a healthy person and in a patient with chronic hypertension. 13. Write an equation for intracranial pressure (ICP) and sketch curves for CBF-O2 and CBFCO2. Primary Viva 5 (Physiology) QUESTIONS 1. Discuss the classification of nerve fibres. 2. Discuss pain pathways. 3. Subdivide A, B and C nerve fibres what role do each of these play? 4. Discuss the renal handling of glucose/water.

950

Primary Viva 6 (Physiology) 1) Draw and compare a pacemaker and ventricular muscle action potential. (Draw each to scale and superimpose them) Explain each phase. Why are the resting membrane potentials different? How are the action potentials affected by: a) Sympathetic stimulation b) Lidocaine c) A beta-blocker d) A calcium channel blocker e) Amiodarone 2) What is the Nernst equation? Compare the concentrations of ions intra- and extracellularly. What is the GibbsDonnan effect? 3) Draw a graph of CO2 content versus PaCO2. How is CO2 carried around the body? Explain the Haldane effect. Does the graph cut the zero point? Draw O2 content versus PaO2 on same graph. Why does this have a different shape? Primary Viva 7 (Physiology) 1. Give a detailed description of the pain pathway that becomes activated if you prick your finger with a pin. What happens in the thalamus and cerebral cortex? Describe the neurotransmitters involved. 2. Compare the body fluid compartments between an 18-year-old and an elderly person Describe what happens when 2 litres of water are drunk.

951

Primary Viva 8 (Physiology) 1. What happens when you breathe from a bag containing 95% O2 and 5% CO2? 2. What are the reasons for a high haemoglobin level? 3. What is the mechanism for polycythaemia at altitude? 4. How long will it take for polycythaemia to occur? 5. In indigenous people living at high altitude, is this an adaptive or genetic response? 6. Where is haemoglobin (Hb) formed? 7. Where does the iron for Hb formation come from? 8. What are the mechanisms of iron absorption and transfer? 9. What is Hb broken down into? 10. What is the life span of a red blood cell? 11. Aside from acute blood loss, what are the causes of a reduced haemoglobin? 12. How could one differentiate between the different causes? 13. Where do vitamin B12/folate exert their effects? 14. What events occur to prepare the body for digestion when one sees a plate of food? 15. How is gastric acid secretion regulated? 16. Which cells secrete gastric acid? 17. What is the cellular mechanism for the secretion of stomach acid? 18. What are secretin, cholecystokinin (CCK) and gastrin secreted in response to? 19. What effects do secretin, CCK and gastrin have in regulating stomach acid secretion? 20. What is the role of histamine in gastric acid secretion?

952

Primary Viva 9 (Physiology) 1. Discuss how the body handles glucose during exercise. 2. You are given a basic blood gas profile (pH, pO2, pCO2). - Explain each term and then draw a Davenport diagram to aid interpretation of this profile. - What other values might the blood gas results yield? 3. Discuss the mechanisms for water balance in the body (include osmoreceptors, antidiuretic hormone, HenryGauer reflex and the reninangiotensinaldosterone mechanism). Buffers 4. What is a buffer? 5. What buffers are used in the body? 6. How would the body react to a bolus of acid? 7. Which is the first buffer that would be encountered? 8. What is the role of the kidney as a buffer? Primary Viva 10 (Physiology) Part I 1. What are the causes of tissue hypoxia? 2. How would you calculate the dissolved oxygen content of the blood? 3. What is the significance of mixed venous saturation? What mixed venous saturations would be found in different types of hypoxia? 4. Draw an oxygen dissociation curve and mark the points of interest (use kPa units) Part II Describe the differences between neonatal and adult physiology, in terms of the following parameters: 1. The cardiovascular system, including heart rate, blood pressure, cardiac output and systemic vascular resistance (SVR). Why is there a difference in SVR? 2. The respiratory system, including functional residual capacity and closing volume. Why do neonates have a high respiratory rate? 3. Metabolic rate and oxygen consumption 4. The liver and renal systems At what stage, approximately, do these systems in neonates approach adult levels? 5. The musculoskeletal system Why do neonates fatigue easily Part III 1. Describe the location and function of the vasomotor centre, including discussion on the afferent and efferent nerves.

953

2. Describe the location, function and nerve supply of the carotid and aortic sinuses. 3. Describe the location and function of the low-pressure baroreceptors in the cardiovascular system. 4. Discuss the hormonal control of blood pressure, including the mechanism of action and function of the angiotensin system and atrionatriuretic peptide. Primary Viva 11 (Physiology) 1. How is alveolar oxygen calculated? 2. What is the respiratory quotient? 3. What do you understand by the autonomic nervous system? 4. Discuss the sympathetic nervous system, including outflow, functions, ganglia and neurotransmitters. 5. Discuss the parasympathetic nervous system, including cranial nuclei, and the course and distribution of the vagus nerve. 6. What is the thyroid gland? Describe its anatomical location, the hormones it produces, the hypothalamuspituitary axis and hyper- and hypothyroidism. Primary Viva 12 (Physiology)

1. What is a buffer? 2. Explain the importance of buffers 3. How are buffers classified? 4. Which is the most important buffer in the body? 5. What is the equation for the formation of bicarbonate? 6. Which enzyme is essential for the formation of bicarbonate? 7. How does bicarbonate buffer the hydrogen ions? 8. How are hydrogen ions produced in the body? 9. What concentration of hydrogen ions are produced per day? 10. How does the body handle an acid load? 11. How are hydrogen ions buffered in protein and haemoglobin?

954

12. How does the kidney handle hydrogen ions? 13. How is metabolic acidosis compensated in chronic renal failure? 14. What would be the blood findings in chronic renal failure? 15. Describe the anatomy of the adrenal gland 16. Describe the various layers of the adrenal cortex and their secretions 17. What functions are carried out by the adrenal medulla? 18. List the various the amino acid precursors and describe the steps in their synthesis 19. In healthy adults, which is produced in greater quantity: adrenaline or noradrenaline? 20. Describe the features of right-axis deviation in the heart. 21. In the heart, which ventricle is normally dominant? 22. What is the ratio of the size of left ventricle to right ventricle? Viva 1 (Clinical) An 80-year-old man presents with an acute abdomen for an emergency laparotomy. He is in atrial fibrillation at a rate of 80 bpm and has not passed urine for 8 hours. He is also mildly breathless. QUESTIONS 1. How would you optimise the patient for surgery? 2. How would you assess his volume status? 3. Discuss fluid management, differential diagnoses and cardiac assessment. 4. How would you anaesthetise him? During surgery, the patient develops frequent ectopics, becomes compromised and subsequently progresses to ventricular tachycardia (VT).

QUESTIONS 1. How would you treat this emergency situation? The patient then develops ventricular fibrillation (VF).

955

QUESTIONS 1. How would you treat this situation now? 2. How is blood loss assessed? 3. How do you manage different severities of haemorrhage? Primary Viva 2 (Clinical) A 26-year-old male attends an accident and emergency department with a compound fracture of the left tibia. The orthopaedic team want to take him to the operating theatre. His girlfriend says that he is hepatitis B positive. QUESTIONS 1. Would you take him for surgery? 2. Why is there a need for an urgent operation? 3. What is the significance of saying he is hepatitis B positive? 4. List the types of hepatitis 5. What other non-hepatitis group viruses could cause jaundice? 6. How is hepatitis transmitted? 7. What risks are involved when anaesthetising a patient with hepatitis B? 8. What risks would he pose to staff? 9. How can you avoid risks what precautions would you take? 10. What are the universal precautions that should be taken? 11. How would you resuscitate the patient? Would blood loss have occurred? 12. What investigations would you ask for? 13. What anaesthetic technique would you use? 14. What monitors would you use? Critical incident: postoperatively, the nurse says that he has breathing difficulty and he is cyanosed. QUESTIONS 1. What would you do? 2. What are the various causes of cyanosis? 3. How is cyanosis diagnosed? 4. How would you diagnose and differentiate between opiate overdose and non-reversal of neuromuscular blockade? 5. What would you look for on the nerve stimulator in this patient? 6. How would you treat the various causes of cyanosis?

956

Primary Viva 3 (Clinical) Print friendly page A 22-year-old female presents to an accident and emergency department with right abdominal lower quadrant pain. She is given an injection of morphine, 10 mg. Her blood pressure is now 85/60 mmHg and her heart rate is HR 120 bpm. QUESTIONS 1. What is the differential diagnosis? 2. What are the possible medical causes? 3. What investigations would you perform in this patient and why? 4. Does injection of morphine, given for pain relief, affect your decision or management? 5. Should the patient go straight to the operating theatre or should she undergo preoperative optimisation first? If you feel she requires optimisation, how, where, when, why and what are the end points? 6. When she has been taken to the operating theatre, discuss the maintenance drugs and anaesthetic technique you would use. 7. A diagnosis of ruptured ectopic pregnancy is made; discuss how you would manage blood loss. 8. Complications of blood transfusion: a) Discuss the problem of mismatched blood. b) What are the causes, types and management? c) Explain how a mismatched transfusion can lead to acute renal failure. d) List some other causes of acute renal failure, other than blockage of renal tubules with haemoglobin. e) Why should Rhesus positive blood not be given to females, and why is it only the Rhesus group that causes problems with females? Why not the ABO system?

957

Primary Viva 4 (Clinical) QUESTIONS 1. What are the problems associated with anaesthetising an obese patient? 2. What specific points would you look for in the history of a patient with ischaemic heart disease? 3. Discuss the airway differences between a neonate and an adult. 4. Discuss the management of inhalational burns. 5. Discuss the calculation of fluid requirements in adults and children. 6. How would you confirm that an endotracheal tube has been placed correctly? 7. Discuss Maplesons classifications. What other classification systems are there? Click here for an interactive Mapleson A circuit. 8. What criteria must be fulfilled for day surgery eligibility? 9. List the causes of hypotension at the following stages: Preoperatively Intraoperatively Postoperatively 10. Why is humidification important and what potential problems are associated with it? Click here for an article on humidity. 11. What is in soda lime and with which agents should it be used with caution? Click here for an article on soda lime. 12. Draw a circle system. 13. What are the problems of anaesthetising a patient with diabetes? Click here for an article on the problems of anaesthetising patients with diabetes. 14. What drugs are dangerous in porphyria? 15. Explain how a laser works.

958

Primary Viva 5 (Clinical)

QUESTIONS 1. Before your viva, you are given a card to read, which says the following: You have been called to A&E to provide anaesthesia for reduction of a Colles fracture. The patient is a 68-year-old heavy smoker and drinker, who has been involved in a house fire; she has burns to her face, chest and arms. What assessment of the patient would you make? Discuss airway assessment, the significance of SaO2 and any other investigations you would perform(COHb). What are the indications for intubation? What fluid requirements will the patient have? What fluid would you give, when would you give it and why? Discuss analgesia are burns painful? Would you give an anaesthetic for the fracture? Where should the patient be looked after? 2. What would you do if an anaesthetised patient suddenly became hard to ventilate? Discuss your management in this critical situation. What are the signs of a pneumothorax? How should a pneumothorax be treated?

Primary Viva 6 (Clinical) A 24-year-old male haemophiliac presents for wisdom tooth extraction.

959

QUESTIONS 1. What are the risks of dental surgery in a healthy individual? 2. Discuss the genetics of haemophilia. 3. How would you anaesthetise the above patient? 4. Is this patient a suitable day case? 5. What investigations would you perform and what clotting results would you expect? 6. Would the bleeding time be prolonged? 7. What blood products would you have ready? 8. Would you give factor VIII preoperatively?

Primary Viva 7 (Clinical) A 15-year-old black boy presents to an accident and emergency department with painful testis. Surgeons want to take him straight to theatre. QUESTIONS 1. What is the differential diagnosis? 2. Which particular questions are important when taking the patients history? 3. Discuss the assessment of hydration in this patient. 4. Discuss paediatric fluid management. 5. Discuss sickle cell disease. Primary Viva 8 (Clinical) 1. You are invited to anaesthetise a 60-year-old female for a retinal detachment. She has had asthma for many years and takes inhaled steroids. How would you assess her, as regards her asthma? (Her history includes taking oral steroids; on examination, she has pulsus paradoxus) What would you use to provide analgesia for her preoperatively?

960

How would you anaesthetise her? Discuss the effects of the drugs you mention on this patients asthma. What would you consider to be essential monitoring in the operating theatre? During anaesthesia, an 'astute medical student' draws your attention to the fact that the endtidal CO2 has risen to 12 kPa. What would you do? 2. You are presented with a young female primip of 3840 weeks gestation, who is to have a Caesarean section. What issues would you address in your preoperative assessment? Primary Viva 9 (Clinical) 1. A 25-year-old woman who is 12 weeks pregnant presents with right iliac fossa pain and pyrexia for appendicectomy. Discuss the differential diagnoses. How would you change your anaesthetic practice in the light of her pregnancy? Primary Viva 10 (Clinical) An 8-year-old girl is involved in a road traffic accident and is brought into Casualty intubated and ventilated. 1. How would you estimate her weight? 2. How would you assess her circulation? 3. What methods would you use to attain venous access? 4. How would you assess her neurological status? 5. How would you anaesthetise her to take her to the CT scanner for her suspected head injury? The CT shows no intracranial pathology, so the child is taken to theatre for surgical repair of a fractured femur. 6. How would you anaesthetise her for this? Surgery is uneventful but towards the end you notice that the patient is beginning to develop an unexplained tachycardia. You also notice that her end tidal CO2 is beginning to rise and she is very hot to the touch.

961

7. What could be the cause of this and how would you treat it? 8. How could you confirm the diagnosis in the child and her family? 9. Where is the malignant hyperthermia research laboratory? Primary Viva 11 (Clinical) 1. An 80-year-old man presents with intestinal obstruction for an emergency laparotomy. He has not passed urine for eight hours, he is slightly breathless while sitting up in bed, his blood pressure is 100/60 mmHg and his radial pulse is irregular. What might be of particular concern regarding his condition, and what further information would you need to know? How would you assess him on the ward? Demonstrate how you would perform a capillary refill test. Explain what constitutes a positive test. Why might the patient not have been passing urine? How would you rate his blood pressure? What is the likeliest cause for this? How would you manage the patients fluid status? If the patients radial pulse rate is 90 bpm and apical pulse rate is 120 bpm, what would you do? If the apical rate now drops to 110 bpm, how would you respond? What dose of digoxin would you administer? Once the patient is in the operating theatre, how would you anaesthetise him? 2. A man is undergoing a laparoscopy. During the procedure, you notice that his airway pressures have been gradually rising and are now very high. What would you do? What are the likely causes for this increase in pressure? 3. How is temperature measured? Name the different sites in the body where it is measured, and discuss the advantages and disadvantages of these different sites. In the operating theatre, when would a patients temperature be measured? Primary Viva 12 (Clinical) A 68-year-old alcoholic is found unconscious in a burning house, with burns to his face, chest and limbs. How would you manage this man, scheduled for repair of Colles' fracture? How would you calculate how much fluid to give? What type of fluid would you give? Will the burns be painful? If so, what painkiller should be used? How would you diagnose carbon monoxide poisoning?

962

Is there a level of carboxyhaemoglobin (COHb) that is not diagnostic? What are the indications for hyperbaric therapy? What is the half-life of COHb in air, oxygen, hyperbaric oxygen? How would you anaesthetise for repair of Colles fracture? How is a Biers block performed? Critical incident You are in theatre, operating on a patient with a bleeding peptic ulcer. The patient suddenly becomes tachycardic, hypertensive and has ST depression. What would you do? Primary Viva 13 (Clinical) An 8-year-old child is brought in to the accident and emergency department with a fracture to the forearm, having been pulled across the road by his mother. - What questions would you ask in your preoperative visit? - What premedication would you give? - How would you anaesthetise this child? - How would you respond if, during surgery, the child suddenly becomes bradycardic? - What might be the causes of the bradycardia? Primary Viva 1 (Physics) 1. Pulse oximetry How does it work? Is it fast or slow as a monitor of oxygen saturation? What problems are associated with it? 2. You are now shown a graph of light absorption of haemoglobin and oxyhaemoglobin. Please comment on this graph, mentioning all the salient features. You may like to read more about pulse oximetry by clicking here.

963

3. Humidity Define relative and absolute humidity. What is an appropriate level of humidity for the operating theatre and why? How does saturated vapour pressure (SVP) change with altitude and temperature? Primary Viva 2 (Physics) Scavenging QUESTIONS 1. What are the methods for reducing pollution in the operating theatre? 2. What is scavenging? 3 Describe the different types of scavenging. 4. What are the differences and advantages between them? 5. What toxicity do the gases from active scavenging cause? 6. What are the maximal permissible levels for N2O and volatile agents? 7. Where does toxicity occur (apart from the operating theatre)? You may like to think about the Labour ward, and the accident and emergency and physiotherapy departments. Equipment QUESTIONS 1. How does an ECG machine work? 2. What are amplifiers? 3. Define bandwidth and gain. 4. What is differential amplification? 5. What is the frequency of the ECG? 6.What is diathermy? how would you avoid interference caused by diathermy? 7. What is the role of impedance? 8. You are shown circuit diagrams. Explain each of them.

964

Primary Viva 3 (Physics) QUESTIONS 1. How do rotameters operate? What are their underlying physical principles? 2. Explain the Venturi Principle and how it would be applied in anaesthetic practice. 3. Are there any ventilators that work on the Coanda effect? Discuss these ventilators and the principle underlying their function. 4. Define the following: Critical temperature Critical pressure Critical flow 5. What is the filling ratio? 6. Discuss the pressure changes in a nitrous oxide cylinder as it is discharged. 7. Is there a difference between gauge pressure and absolute pressure? How can they be measured in practice? 8. Discuss all aspects of a ventilator of your choice. 9. There are two types of diathermy. What are the main differences between them and how are these differences achieved? Draw a waveform for both. 10. What equipment in the operating theatre uses a capacitor? What is the importance of capacitors in practice? 11. Draw a Wheatstone bridge. You may like to access this article on the Wheatstone bridge to see if you were correct. 12. Draw a scavenging system and explain the pressures involved. 13. List the basic SI units. 14. Explain how a laser works.

965

Primary Viva 4 (Physics) Explosions and safety QUESTIONS 1. What is a stoichiometric mixture? 2. Under what circumstances would you use a capnograph? Why is it so useful? 3. List the safety features of a standard anaesthetic machine. 4. Describe the different types of sterilisation techniques. 5. You are shown a picture of an autoclaved laryngeal mask airway (LMA). What do the stripes signify? 6. Draw a picture of an oxygen disconnection alarm. Primary Viva 5 (Physics) QUESTIONS 1. Flow Discuss laminar and turbulent flow profiles. Draw a diagram of each. Discuss the HagenPoiseuille equation. Why is it clinically relevant? Discuss the use of flowmeters and rotameters. How can you decrease the work of breathing using knowledge of flow? 2. Vaporisers Classify the various types of vaporiser. How do they work? What is the concentration of vapour leaving the vaporising chamber? How are vaporisers affected by altitude? 3. Arterial lines Discuss arterial traces (including damped/resonant). What is Fourier analysis? What is bandwidth?

966

Primary Viva 6 (Physics) You are shown photos of Venturi masks by the examiner. QUESTIONS 1. Discuss the Bernoulli principle, Venturi effect and entrainment ratio. 2. Give a value for the entrainment ratio in Venturi masks. 3. What type of oxygen delivery device do they provide? 4. When would you use one? 5. What other devices use the Venturi effect? 6. What is the Coanda effect? 7. Can you name the ventilator in which it is used? 8. How can you tell how much N2O is in a cylinder? 9. If you were using 4 L O2/min, how long would it last? 10. What is a mole? 11. Is a mole an SI unit? 12. What is Avogadros number? 13. What volume does one mole of gas occupy at standard temperature and pressure (STP)? 14. Define critical temperature and pseudocritical temperature. 15. In a cylinder of Entonox, below its pseudocritical temperature, what is the nature of the gas which is released initially? 16. Define tare and filling ratio. 17. Are there any other safety mechanisms on a nitrous oxide cylinder which is to be used in the tropics?

967

Primary Viva 7 (Physics) QUESTIONS 1. Discuss neuromuscular monitoring (for further reading, access our section on nerve stimulation) What is double burst? What is train-of-four (TOF)? 2. Can you differentiate between depolarising and non-depolarising agents? 3. What is an amplifier/gain/bandwidth/drift/valve? 4. Discuss the end-tidal CO2 trace with rebreathing. 5. Discuss the Mapleson classification. You may like to access our interactive Mapleson circuit (http://www.anaesthesiauk.com/sectioncontents.aspx?sectionid=149). Primary Viva 8 (Physics) 1. How is gas flow measured? 2. Describe the carbon dioxide electrode: What sources of error are involved with its use? How is it calibrated? Draw a calibration curve. 3. What is monitored in a breathing circuit? 4. Describe the mechanism of action of the disconnection alarm on a ventilator. Primary Viva 9 (Physics) 1. What methods do you know for assessing temperature? 2. How does a thermocouple work? 3. How does the resistance of thermistors/platinum resistance wires change with temperature? 4. You are shown photographs of a mercury thermometer, nasopharyngeal probe and a tympanic thermometer. What physical principles do these instruments work on? What is above the column of mercury? Why does the mercury column not expand linearly with temperature?

968

How is this compensated for in the thermometer? Over what temperature range can the mercury thermometer operate? 5. What instrument is used for measuring lower temperatures? 6. What are the ways in which heat is lost in the operating theatre? 7. What is the molecular explanation for the latent heat of evaporation? 8. How does radiation differ in its physical basis from convection or conduction? 9. How can heat loss be reduced in the operating theatre? 10. What other reasons are there for maintaining a reasonably high humidity in the operating theatre? 11. You are shown a picture of an Ambu-bag. What is the maximum FiO2 with this bag on air? What is the maximum FiO2 with high-flow oxygen into a reservoir bag? What sort of valve is found at the patient end of the Ambu-bag? How many leaflets does this valve have? How would you classify these valves? (High or low resistance?) Primary Viva 10 (Physics) 1. You are given 12 electrical symbols to describe. Choose one of these, and you will be asked further questions (e.g. if you pick a capacitor, you might be asked to discuss the working principles of a defibrillator). 2. Discuss the diathermy procedure, including such issues as the high frequency and density of the current used and the risk of burns. 3. You are shown a picture of the Tec6 desflurane vaporiser; describe the differences between this and the Tec5 model.

969

Primary Viva 11 (Physics) Part I 1. Discuss pressure measurement in general and non-invasive blood pressure monitoring in particular 2. Discuss the principles of the mercury sphygmomanometer. Why is it open at the top? What is the significance of cuff size, and what happens with large and small cuffs? What are the complications of cuffs? 3. Draw a diagram of the Bourdon gauge. What happens if the tube at end, near the pointer, is broken? 4. What are the units of pressure? What is the definition of a Pascal 5. Which is higher, the pressure developed in a 2 ml or a 20 ml syringe and why? In which syringe would you be able to clear a blockage? 6. How does a pressure transducer calculate pressure? (mention the word integration for measuring the mean arterial pressure!) Part II 1. You are presented with three curves and their logarithmic transformation. Explain the curves, and how they can each be identified. Give examples of particular curves which would fit these patterns and explain how you would interpret them What are the equations describing the curves? How would you calculate the various values from the curves (especially the negative exponential curve)? 2. Discuss the concepts of half-life and time constant. What are the differences between these functions? What amount of function remains at the end of 1, 2 and 3 time constants? Explain graphically the concepts of both the half-life and time constant. Why are curves logarithmical? Part III 1. What considerations should be taken before building a vaporiser? 2. You are shown a diagram of an opened up Tec vaporiser. Identify the pathways inside and discuss the structure. Show the variable bypass and explain what it means. What happens if 100% of the gas is passed through the vaporisation chamber? Primary Viva 12 (Physics) 1. How would you measure the concentration of oxygen in a gas mixture? Discussion to include the following: - Fuel cells - Clark electrode - Paramagnetic analysers - Gas chromatography - Mass spectroscopy

970

2. What are their principles of action and their drawbacks? 3. What could interfere with their function? 4. You are shown an 18G venflon, a Tuohy needle, and a Whitacre and a Sprotte needle; choose one of these and talk about it. 5. What is the difference between the Whitacre and Sprotte spinal needles, and what is the significance of this difference? 6. What types of needles are used for nerve block? 7. Discuss the measurement and units of pressure and partial pressure 8. Discuss the properties and use of mercury manometers 9. Discuss blood pressure measurement and transducers 10. What is the Bourdon gauge? 11. Discuss the interpretation of exponential curves and the significance of half (t1/2) time constants 12. Discuss the use of vaporisers, with particular reference to desflurane

Primary Viva 13 (Physics) 1. How is pressure measured? 2. Name the various units of pressure, and explain how they are related. 3. What is a Pascal? 4. What is a bar? 5. What is an atmosphere and what does it convert to in mmHg? 6. What is the gas pressure found in cylinders of O2 and N2O? 7. What is a Bourdon gauge, and how does it work? You are shown a picture of a Bourdon gauge: describe its component parts. 8. In the operating theatre, when are the principles of vacuum suction and scavenging used? 9. Why is scavenging important, and what are the effects of methionine synthase on bone marrow?

971

10. What are the safe limits of waste gases in the operating theatre? 11. How can levels of pollution in the operating theatre be reduced? 12. What is a transducer, and when might this be used in clinical practice? 13. What other methods are available for measuring blood pressure? 14. Discuss the technique of direct arterial monitoring; how does it work? What piece of equipment can be found in the detection part of the arterial pressure transducer? 15. Draw and explain the principle behind the Wheatstone bridge. Explain how this translates to an image on a cathode ray oscilloscope screen 16. Explain the concepts of resonance and damping. You are shown graphs of different waveforms; explain what you see - are they over- or under-damped? 17. How is an arterial monitor calibrated? 18. Why is there a need for a three-point calibration? 19. You are shown a graph of a calibration error; explain what is happening Primary Viva 14 (Physics) How do you measure the pressure in a gas cylinder? Discuss the use of pressure regulators. Describe the different types of scavengers and how they work. What problems are associated with each of these? What size of catheter should be use for suction, and why? Discuss the issue of pollution in the operating theatre. Which types of anaesthesia give rise to the most pollution? How can this be reduced? Is there evidence of harm arising from this pollution? Describe the various types of ventilators. Use diagrams to illustrate your answer and explain the advantages and disadvantages of each. Primary Viva 15 (Physics) 1. You are shown pictures of a thermometer, a thermistor, a thermocouple, a Bourdon gauge, Infrared tympanic membrane thermometer. How do each of these instruments work?

972

Describe the significance of graphs of resistance versus temperature You are shown a picture of liquid crystals. Describe their use in measurement of temperature 2. What is a capacitor? What is capacitance? What factors improve the ability of a capacitor to store charge? Which piece of equipment is it used in conjunction with? Primary OSCE 1 1. Equipment - Check Arterial line set-up - Assess faults: (findings: no pressure on bag, 5% dextrose in bag, unable to flush transducer, venous cannula attached, inappropriate tubing) 2. Anatomy Ankle block - Which nerves? Describe the course and distribution of tibial, saphenous, superficial peroneal - How much LA is needed? - Toxic dose of lidocaine, prilocaine and bupivacaine? 3. Counselling Jehovah's Witness, Hb 9.2 for total abdominal hysterectomy for menorrhagia, asked to get her to sign consent form for surgery without blood products - Patient brought up subject of erythropoietin and bypass, outline response 4. Equipment - Urinary catheter with transducer attached for measuring core temperature - Methods of measuring temperature, difference between thermistor + thermocouple - Which metals are used in a thermocouple? - Shown four different graphs of resistance v potential difference: identify graphs for thermistor and thermocouple 5. Resuscitation Drowning victim, has pulse but not breathing, started to breath after 10 breaths given - Fill in gaps in asystole protocol 6. Chest X-ray - 10 T/F questions

973

7. Data analysis - Sequential arterial blood gas results and observations for 21 year old with asthma - 10 T/F questions 8. Examination - Cranial nerves 3- 12 (normal actor) 9. Anatomy i) Antecubital fossa (ACF) - identify on model biceps tendon, brachial A, radial N - what is the name of aponeurosis in ACF? - which vein would you choose for a long line? - where might a line stick? ii) Wrist - identify positions of radial, ulnar and median nerves - demonstrate Allen's test - shown adult and paediatric plates, which is suitable for an adult and why? - shown picture of a patient connected to diathermy and touching metal pole; is this dangerous? Why? - discuss cutting and coagulation waveforms - shown diagram of two circuits, one with isolating transformer; which would you rather be attached to and why? - why does diathermy not cause ventricular fibrilation? 11. History 48 year-old-male for bilateral varicose vein surgery. He is an asthmatic, severe attack in the past precipitated by propranolol for an unknown arrhythmia. 12. Machine check - Check circle only - Assess faults (findings: not attached to fresh gas flow, expiratory valve stuck in open position? What would this cause?) 13. Data - Spirometry trace of patient for transurethral resection of the prostate, showing chronic obstructive pulmonary disease picture - 10 T/F questions 14. History 40-year-old female for laparoscopic cholecystectomy; no problems found 15. Equipment

974

- LMA - Demonstrate use, safe use with intermittent positive pressure ventilation - How should it be cleaned? (hint: a closer look at the LMA reveals the word autoclave!) 16. Statistics - 10 T/F questions on normal distribution / mean / mode / median / SD / SEM / confidence intervals ArticleDate:20040715 SiteSection: Article Primary OSCE 2 1. Data interpretation Following massive transfusion, a patient is found to have deranged clotting. You are asked to interpret the results. 2. History Take a full history from an arthritic patient undergoing metacarpophalangeal joint replacement. 3. Equipment - Correctly connect an HME filter and sidearm CO2 analyser to a Bain circuit - Describe the points of interest on a normal capnograph trace - Identify a trace showing disconnection and re-breathing - Why does the sidearm sampler tubing have to have a small diameter? 4. Communication A drug addict is about to undergo appendicectomy. - Talk to him about prescription methadone - Discuss his concerns about not having received methadone since admission to hospital - Address his anxiety about becoming re-addicted to opiates given for postoperative analgesia 5. Resuscitation station - Perform basic life support on a dummy - Correctly complete an algorithm to treat the rhythm shown on the cardiac monitor (ventricular fibrillation) 6. Statistics - What is the difference between nominal, ordinal and categorical data? (Type 1 and 2 errors) - What can you do to reduce type 2 error? 7. Examination Assess the Glasgow Coma Score of a patient (played by an actor) 8. Chest X-ray In the lateral view, it appears as though the patient has a hiatus hernia. - Would you give antacids and pass a nasogastric tube preoperatively in this patient?

975

9. History A 20-year-old female presents for laparoscopic sterilisation. When asked specifically, she mentions that she has recently been jaundiced but this was not investigated. She is a smoker and has a cough. 10. Anatomy - Name the layers of the heart - Where are the sinoatrial (SA) and atrioventricular (AV) nodes? - Where are the origins of the coronary arteries? - How many cusps have the mitral and aortic valves? - Where are the sinuses of Valsalva? - Describe the normal coronary angiogram (i.e. name the major vessels and the territory supplied by the left anterior descending [LAD] coronary artery) 11. Temperature measurement You are shown photographs of a nasal probe and an axillary probe. - What are these? - How do they work? - Describe resistance temperature curves for each device - What reference temperature does a thermocouple use? - What is the triple temperature of H2O? - What is the core temperature and how do you measure it? 12. Anatomy You are shown a model of the vertebral column. - Identify the pedicles, lamina, facet joints, foramina, ligament denticulatum - Describe the margins of the epidural space and the upper and lower limits of the cord - Where is the sacrococcygeal membrane? 13. Electrical hazards - What current magnitude can cause microshock? - What techniques are used to safeguard from this common earth? - Identify four electrical symbols 14. Machine check In this case, there are no volatile gases; remember to connect the O2 analyser to the circuit, as well as calibrating it. 15. ECG interpretation The ECG shows LVH and angina, and the patient is hypertensive. - Would you postpone carrying out elective surgery on this patient? Primary OSCE 3 1. Rotameters Discuss how they work. 2. Pulse oximetry Discuss the isosbestic point, p50, Myoglobin curve and causes of inaccuracies.

976

3. Resuscitation Demonstrate how you would resusciate a paediatric/adult in ventricualr fibrillation. 4. Statistics You are asked questions about a Chi square table. 5. Electricity Discuss microshock, symbols and safety features in theatre. 6. Anatomy Describe the anatomy of the orbit. 7. Anatomy Describe the anatomy of the spinal cord. 8. Data You are asked to interpret some biochemical results from a patient with diabetic ketoacidosis. 9. Skills Describe how you would perform a cricothyroidotomy. Primary OSCE 4 1. Lateral chest X-ray You are shown the lateral chest X-ray which shows diaphragmatic hernia and pleural effusion. The patient has a history of vocal cord carcinoma for rigid endoscopy. - Is the patient suitable for laryngeal mask airway? - Is an AP\PA view needed? - Is a cricoid needed? - Is there air above diaphragm? 2. Statistics You are shown a negatively skewed distribution: - Is it normally distributed? - Label the mode, median and mean - Would the Mann Whitney U test be suitable for these data ? - Are the data parametric? - If these vaues were to be squared, would this produce a a normal distribution? - Would plotting the log of these values give a normal distribution? 3. You are shown a trace of jugular venous pressure (JVP), which has been taken with the patientnot at an inclination of 45 degrees, and wearing a shirt: - Is it normal? - What is the cause of large a-waves? - What is the cause of large v-waves? - What is the cause of an absent a-wave? - how do valvular disorders affect the JVP 4. Take a preoperative history from a 54-year-old asthmatic presenting with varicose veins - Patient has aspirin-sensitive asthma - Patient is allergic to propanolol

977

6. Diathermy - Which is the earth electrode? - paediatric suitable for 40 yo? - Should unipolar or bipolar electrodes be used? - You are shown a selection of sine waves: are they for cutting or coagulation? - What are the associated risks? - What is current density? - What is frequency for cutting? - Why does this not cause ventricular fibrillation? - You are shown two circuits, with circuit A isolated: which is safer for i) the patient? ii) staff? 7. Supraclavicular block - Show the landmarks on an actor - Describe the block: needle size, depth, amount of local anaesthetic (LA), type of LA, strength of LA, etc. - What are the risks of supraclavicular block? - Show the interscalene groove - What are the risks associated with interscalene block? 8. Bain circuit check - What happens if it is disconnected at the patient/bag/at fresh gas flow (FGF)? - What is the volume of the outer tubing? - What FGF is needed for normocapnoea in a 100 kg patient (spontaneous ventilation & intermittent positive pressure ventilation)? - What is the effect of shortening the tubing? - What is the effect of lengthening the tubing? 9. You are shown a vitalograph trace - Is it calibrated for body temperature, pressure and saturation (BTPS)? - FEV1 greater than 200 - FVC greater than 250 - Normal FEV1/FVC ratio - Obstructive pattern - No risk associated with spinal - Would the patient be suitable for upper airway surgery? - What might be the cause of this type of lung disease? Primary OSCE 5 1. Oesophageal stethoscope - Discuss its uses 2. Resuscitation Demonstrate how you would resuscitate a pregnant woman, lying on the floor on a pillow. - Fill in a management flow chart - Fill in drug and electricity shock doses in the case of ventricular fibrillation

978

3. X-Ray You ares shown a plain chest X-ray of a patient with trauma - Does this show a ruptured aneurysm? 4. Statistics You are asked to answer somwe multiple choice and true/false questions on some positively skewed data. 5. Equipment Check a Bain circuit 6. History taking Take a history from a man presenting for transurethral resection of the prostate, who has had some cardiovascular problems. 7. Communication Placate a patient (payed by an actor) after accidentally knocking out one of his teeth during anaesthesia. 8. Communication Obtain consent for an epidural from a young woman with a painful back. 9. Radiology You are asked to interpret a CT scan of the coeliac plexus, how to perform a coeliac plexus block, and about the roots/innervation of the coeliac plexus. 10. Data interpretation You are asked to interpret data on the blood gases of an elderly patient with chronic obstructive airways disease presemting for an operation. 11. Examination You are asked to examine the airway of a patient (played by an actor), and you are then shown a C-spine X-ray and asked to point out the fracture. 12. Anatomy You are shown a picture of an arm hanging off the side of an operating table. - ulnar nerve palsy - What sensory changes would you expect on examination? - What treatment would you give for ulnar nerve palsy? 13. Anatomy You are shown a coronary angiogram - identify the left anterior descending coronary artery and point out where the the sinuses of Valsalva are 14. History Take a history from a patient presenting for routine surgery.

979

15. Radiology You are asked to interpret a head CT (subdural). Primary OSCE 6 1. Glasgow Coma Score (GCS) Determine the GCS on an actor with possible head injury. How would you manage him? 2. Anatomy - What structures go through the orbit holes in skull? - How long is the optic nerve? - Where does the optic nerve stop and start? - How would you perform a peribulbar block? 3. Data interpretation You are asked to interpret an ECG showing atrial flutter, and answer some questions about management. 4. Equipment - What are the safety features of lasers? - How do they work? - Describe the characteristics of different types of laser - Which types of endotracheal tubes are appropriate to use? 5. Equipment Demonstrate the correct use of a defibrillator on a dummy. 6. Anatomy You are shown some pictures of cross sections of th spine with areas marked. - What tracts are these? - Discuss the blood supply to the spinal cord - What happens if the spinal artery is blocked? - Discuss the pain and temperature pathways 7. History taking Take a history from a 65-year-old man presenting for carotid end arterectomy. He is on aspirin for previous cerebrovascular accidents but is otherwise fit. 8. Circle system check Carry out a circle system check (faults might include a sticking expiratory valve and soda lime only half full). Answer some questions on circle systems and soda lime. 9. Equipment Describe the uses of a monauricular stethoscope. Discuss how it is used for the diagnosis and management of gas embolism. 10. Data interpretation You are asked to interpret the chest X-ray of a child who has aspirated a peanut.

980

11. Statistics Discuss the concepts of correlation and regression. 12. Resuscitation A 5-year-old child has collapsed. Basic life support has been given for 2 minutes, after which the trace shows that there is no output. Give five reversible causes for this and describe how you would treat each of these causes. 13. History taking Take a history from a 50-year-old man presenting for nasal septum surgery. He has sleep apnoea, severe reflux and also a smokers' cough . 14. Equipment You are shown a photograph of a Bourdon gauge on an N2O cylinder and asked questions on pressure, critical temperature and the physical properties of nitrous oxide. 15. Communication A 50-year-old woman who is presenting for a gynaecological procedure has a mask phobia. Explain what will be involved, reassure her an explore alternative methods. Primary OSCE 7 1. Chest X-ray A 2-year-old child who has been eating peanuts suddenly develops dyspnoea and presents to the Accident & Emergency department. Answer the following true/false questions about the child's chest X-ray: - It is adequate in position - It shows pneumothorax - It shows collapse - The appropriate treatment is needle thoracostomy - Bronchoscopy is needed - The mediastinum is normal 2. ECG A 70-year-old male presents with chest pain; he is on digoxin and warfarin. Answer the following true/false questions on the patient's ECG: - The rate is 75 per min - The axis is normal - It shows atrial fibrillation - It shows bifascicular block - It shows ischaemic changes - It shows signs of hypokalaemia - It may be a feature of digoxin toxicity 3. Statistics You are shown a graph representing data from 10 children, with age plotted on the x-axis and weight on the y-axis. Answer the following true/false questions relating to correlationregression analysis:

981

- This is a regression analysis - XX is the regression factor - The correlation coefficient can be 0.7 - The correlation coefficient can be 1.3 - The correlation coefficient of 0.7 is statistically significant 4. Safety Station You are asked to demonstrate the use of defibrillator in a dummy (it is essential to check that it is safe, and to state out loud that you are shocking the patient) You are asked about the dose of shocks: - Describe the different positions and benefits of paddles - What will you do after shock? - When will you shock again? 5. Equipment You are asked about the monoaural stethoscope: - What is it? - Where should it be positioned? - For which group of patients is it used? - What does it help in diagnosing? - Give two examples of critical incidents in which this equipment is helpful 6. Venous air embolism - In which position does this occur commonly? - How would you manage this? - How is it diagnosed? - What happens to end-tidal CO2? 7. Equipment Metal endotracheal tube for laser surgery: - What is it used for? - Why is it used in laser surgery? - What is a laser? - What are the hazards of laser? - What precautions would you take if laser is used? You are shown some glass goggles which are labelled: "only for CO2" - Why is this only used for CO2? - What is the wavelength of CO2? - What laser types do you know of? - What is the wavelength range of lasers? 8. Communication A 47-year-old woman presents for a D&C surgery as a day-case. She has history of palpitations, has had an appendicectomy and is very much afraid of anaesthetics. - Ask about her fears (fear of masks) - ask about her history of anaesthetics (mask phobia still) - Explain the importance of preoxygenation - Explain what is going to happen in the anaesthetic room (patient is still unhappy about masks; she feels OK about holding the mask herself, but is still unconvinced)

982

- Explain the option of nasal prongs before operation and postoperatively - Discuss how you monitor oxygenation - Assure the patient that you will be there to take care of her - Explain premedication 9. Anatomy You are shown a diagram of the spinal cord: - You are shown various tracts and asked to name them - What volume of local anaesthetic is needed in the lumbar epidural space to block one vertebra? - What is the specific gravity of cerebrospinal fluid? - Describe the blood supply to the spinal cord (number of arteries)? - What happens if blood supply is cut off to anterior cord, and what are the features of this (anterior spinal artery syndrome)? 10. Anatomy Base of skull, periorbital block: - Point out the optic foramen ona model of the skull - What is length of the optic nerve in the foramen? - Identify the cribriform plate - What does it transmit? (olfactory nerve fibrils) - Trace the optic nerve back to the cortex - How is a periorbital block performed? - What should be the direction of needle? 11. Machine check Check the breathing system attached to the anaesthetic machine and state whether you would use this breathing system (circle absorber). Possible faults include: - fresh gas flow was not connected - inspiratory unidirectional valve not working - soda lime not filled properly (not full) You are asked questions about soda lime: - What is it? - What colour change occurs? - Would you use the circle system with the soda lime off? - What flow rates would you use? - What gas is expired out of this? 12. History taking Take a history from a middle-aged male presenting for carotid endarterectomy, with a history of reflux and cerebrovascular accident; he is currently taking Gaviscon and aspirin. 13. History taking Take a history from a middle-aged man presenting for nasal/pharyngeal surgery. He has a history of heavy snoring, and his wife has to sleep in a separate room. He does not have positional dyspnoea but gets up during sleep. He has undergone sleep breathing studies, which showed that he stops breathing in sleep. He is not on ventilators, but doctors have suggested

983

that he may need a breathing machine attached to his nose after surgery (nasal continuous positive airway pressure). He is taking atenolol and Gaviscon. 14. Physical examination Carry out a neurological examination on a middle-aged male (an actor) - What is his Glasgow coma scale score? - Asked of specific components (how do you arrive at your score?) - Examine the reactivity of his pupils 15. Equipment station You are asked about a nitrous oxide cylinder pressure gauge: - What makes this specifically a nitrous oxide gauge? - Can this be used for oxygen? Why? - What happens during usage? Why does it not fall? - What state is the nitrous oxide in? - What is the critical temperature? - What is gas and vapour? - Where is it used? what is entonox? 16. Data interpretation You are shown two graphs generated by a Swan-Ganz pumonary artery flotation catheter; the first curve is correct, while second is not. Answer the following questions: - Which is the correct wave form? - Is the wedge pressure normal? - Give five complications of pulmonary artery catheters 17. Resuscitation You find a 2-year-old child collapsed in a road; you are given an Ambu bag and mask and asked to perform basic life support: - Check that it is safe to approach - Gently check responsiveness - Shout for help - You are told that the cervical spine is normal - Check airway, breathing and circulation (ABC) - Demonstrate the correct ratio of for the paediatric age group - Carry out 1 minute of cardiopulmonary resuscitation before going to get help You are then shown a rhythm on simulator, which indicates that there is no pulse. - Give five causes for this, and suggest treatment Primary OSCE 8 Print friendly page 1. You are shown a photograph of a cylinder: - Identify entonox. - What is its composition?

984

- What precautions should be taken in regard to its storage? - What is pseudocritical temperature? - Why is it important? - What are the hazards of using it below the critical temperature? - Who can prescribe it? 2. Communication A Jehovahs witness with menorrhagia, with a haemoglobin level of Hb 8.1 g/dl presents for abdominal hysterectomy. - Obtain a special consent from the patient. 3. History A 25-year-old Afro-Caribbean man presents. He is well built, and his history reveals that he has undergone a day-case arthroscopy, and has beens the vicitim of a road traffic accident, after which he underwent a laparotomy and splenectomy. - Can you rule out sickle cell disease? 4. History A fit young woman presents for hysterectomy. Her medication history reveals that she has been told that she is taking some medicines to "increase her blood level". Ask her if she has a history of anaemia, or if there is a history of anaemia in the family. Although she has had no previous operations, you should also ask about any history of a bone marrow test. 5. Cranial nerve examination Examine cranial nerves 3 to 12 on an actor. - Give some explanation while you perform this examination. 6. Airway examination Carry out an airway assessment on an actor. - What is his Mallampati score? - Show how you would examine the patient - You are asked about any intubation problem. 7. Coronary circulation - Discuss the coronary circulation, explaining arterial and venous drainage. - What do you understand by sinus of Valsalva and what is its physiological importance? - What are the chordae tendinae and what is their physiological importance? - How many leaflets are there in the mitral valve? - What are the pressures in the heart chambers? - You are asked about an angiogram showing a stenosis of the left anterior descending artery, and asked to identify the defect. - What is the probable cause?

985

Primary OSCE 9 1. Examination - Patient with a head injury and cervical collar - Demonstrate how you assess the GCS and perform a neurological examination. 2. Ankle block anatomy - What nerves supply the ankle? - What are the anatomical relationships between deep peroneal and tibial nerves? - Describe the course of the saphenous nerve 3. Data You are asked questions on coagulation/ liver function in a patient who has had a recent stroke (1 month ago) and has atrial fibrillation. He has an International Normalised Ratio of 3.5, a thrombin time of 12 seconds, a kaolin clotting time of 60 seconds and a platelet count of 350 x 109/L - Can you give this patient spinal anaesthesia? - Does he have a liver problem? - Is he on warfarin? - Would you delay operation for 6 months? - Can you give suxamethonium (hemiplegia)? - Would you attempt to cardiovert him? 4. Pulmonary artery flotation catheter You are shown a trace and asked some questions 5. Statistics - You are asked questions about regression - You are asked questions about a graph of weight versus height 6. Chest X-ray You are shown a lateral chest X-ray, showing a massive hiatus hernia - Do you need to obtain a postero-anterior X-ray? The patient is presenting for vocal cord examination because of vocal cord paralysis - Can you perform cricoid pressure? - Are H2 blockers needed? 7. Paediatric resuscitation (non-traumatic) - What are the causes of electromechanical dissociation, and what immediate treatment should you give? 8. Lasers You are shown a steel laser endotracheal tube (ETT) and a normal ETT; answer some questions on: - the relevance of the two cuffs - Safety in theatre - Risks - Laser physics

986

- Eye protection Why are goggles only suitable for the CO2 laser? - What are lasers used for? 9. Circle check Carry out a check on a circle breathing system 10. Oscillotonometry - How does this work? - You are shown an oscillation trace: what are the associated safety issues/risks? 11. Capnography - You are shown a trace and asked to describe it - You are asked questions on respiratory physiology - What are the causes of the changes in the trace? - You are shown a trace showing a high FiCO2 and asked to comment on it 12. Safe use of defibrillator - Treatment - Abnormal rhythm 13. History taking Take a history with regard to analgesia in labour 14. History taking Take an anaesthetic history of a patient about to undergo an ear, nose and throat operation 15. Anatomy You are shown a diagram of of the cross-section of a spinal cord and asked questions about it 16. Communication Speak to an anxious woman presenting for a gynaecological operation with stable angina/hypertension who has mask phobia due to a dental experience as a child ArticleDate:20040907 SiteSection: Article Primary OSCE 10 1. History taking Take a history from a man, 68 years old, scheduled for laparotomy and hemicolectomy tomorrow. History reveals bronchitis, ischaemic heart disease, gastrointestinal bleed, history of transfusion 6 months previously (anaemia); patient is taking ibuprofen for arthritis, and is also on inhalers, diuretics and cardiac drugs. 2. History taking Take a history from an elderly man coming for hernia repair tomorrow. History reveals hypertension and ischaemic heart disease; patient is on nifedepine and aspirin. Angina increasing in frequency, but patient is responding well to glyevryl trinitrate sprays. No symptoms of heart failure.

987

3. Anatomy - Answer questions about the anatomy of antecubital fossa - Demonstrate Biceps tendon - Position of Brachial artery: how does it run in the cubital fossa (course)? - What structure lies above brachial artery? - Demonstrate position of radial nerve in cubital fossa - What test do you do to find the integrity of superficial and deep palmar arches? - Demonstrate how you would perform this test (Modified Allens Test) - Describe landmarks to block median nerve at wrist - Where do you block ulnar nerve at wrist? - Which muscle lies closer to ulnar nerve at this point? 4. Anatomy of the heart - Arterial supply of AV node: in what % of patients is this the case? You are shown a photograph of a coronary angiogram, showing the left coronary artery - Identify the branch marked A Left anterior descending artery: - What does it supply? - What is the significance of right coronary artery occlusion? - What do these patients need? - What is the nerve supply of AV node? - What is the course of the vagus nerve, either left or right (origin, course, innervation)? 5. Communication Explain a failed intubation drill to the operating department assistant who has started her training recently. On questioning, she has never seen a general anaesthetic being given and does not know what intubation is!! Explain briefly about general anaesthesia and intubation and then about failed intubation drill 6. Examination Examine the respiratory system (detailed airway examination not needed) of a 25-year-old young man with cough and chest infection. On examination, he looks a bit short of breath, with an increased respiratory rate. Carry out a systematic examination of the respiratory system . 7. Machine check Basic anaesthetic machine with the following faults: - No marks for absent pipelines - Oxygen analyser: no battery - Two empty vaporisers on back bar - isoflurane and enflurane vaporisers - No interlock mechanism. O ring missing from the enflurane vaporiser - Could not pressurise the system with enflurane vaporiser turned on. 8. Resuscitation Resusciate a pregnant woman found collapsed

988

Ventricular fibrillation scenario algorithm - Fill in the chart 9. Statistics You are shown two groups of data: GpA and Gp B, with 6 values in each; SD and n given for both. Answer 10 true/false questions, including questions concerning: - What is the mode mean and median? - Is this a normal distribution? - Does population in Gp B have SEM greater than 1 SD? - Calculation of SEM 10. Data interpretation ECG: 60-year-old man, admitted with sudden onset of chest pain and confusion. Called for urgent cardioversion. Blood pressure: 80/40 mmHg. ECG revealed ventricular tachycardia. Answer 10 true/false questions, including: - Patient is in sinus rhythm - Patient has ventricular rate of 200/min - There is evidence of left axis deviation - There is evidence of inferior wall myocardial infarction - QRS duration > 0.12 seconds - Propofol 2 mg/kg is the drug of choice for cardioversion - Inhalation induction with sevoflurane likely to get delayed - Acute myocardial infarction is a possibility - Arterial line should be inserted before cardioversion 11. Equipment You are shown a photograph of a pulse oximeter monitor - What details can be obtained from this monitor? (pulse rate, SpO2, waveform and signal strength) You are shown a picture of an oxygen dissociation curve, with marked points A and B - What is point A? (90% saturation = 60 mmHg) - What is point B? - What is P50? What are its uses? (to identify various Hb, position of ODC) You are shown another chart containing two curves - You are asked to identify them (ODCs of Myoglobin and hemoglobin) You are shown a chart of absorption characteristics of oxy- and deoxyhaemoglobin - Identify the wavelengths used in the pulse oximeter

989

- Why are there two wavelengths? Why is ther only a pulsatile component? - What are the points marked at around 800 nm? (isobestic point) - What do you obtain with light absorption at that point? (total haemoglobin) You are shown another chart - Where do you see this type of trace commonly? (blood pressure cuff going up on the same side, probe dislodged and corrected) - What are the various sources of error in pulse oximeter? 12. Blood pressure measurement - Take the blood pressure from the model and answer questions: - What is the blood pressure? - Which phase is systolic blood pressure concerned with? - Which phase is diastolic blood pressure concerned with? - What is the formula for mean arterial pressure? - Why is this formula used? - What changes occur in elderly patients? - What differences you expect between men and women? - Is the difference the same throughout life? - What is the size of the blood pressure cuff? - What is the problem of using a small sized cuff? 13. Anaesthetic hazards You are shown a photo of a patient lying on the bed connected to a central venous pressure monitor and other electronic monitors which are connected to earth. There are electrical symbols displayed here and there. - What is the magnitude of current required for microshock? - Are all items of equipment working correctly? - Is the patient not touching any metal? - Why does electrocution not occur? - What do the symbols mean? (isolated circuit, earth potential) - Is earth potential the same at all the places? - How can you make them equal? - What is the maximum allowed difference between earth potential and mains voltage of 240 V? - Select any two symbols from the card and explain what they represent 14. Data interpretation A 65-year-old man, who has had a road traffic accident, presented drowsy to Accident and Emergency, then became unconscious. His CT scan was shown on the screen. Answer 10 true/false questions, including: - Air is seen in the frontal sinus - Patient has blood collection in the left side - Patient has temporal bone fracture in the right side - No evidence of midline shift - Normal left lateral ventricle

990

The CT scan shows extradural haematoma: - Patient needs urgent evacuation of blood - Prognosis very poor at this age - Right-sided dilated pupil would be expected 15. Humidity - Define relative humidity - What is absolute humidity? - What are the units of absolute humidity? - What is the value of absolute humidity in the trachea? - Why is humidity important (give two reasons)? You are shown three charts: Regnault's, wet and dry bulb thermometer, hair hygrometer - Which is the hair hygrometer? - What is the principle of Regnaults hygrometer? You are shown chart with relative humidity lines (100%, 50% etc) Y axis is water vapour g/m3 and X axis shows temperature. - What is the absolute humidity if the dew point is at 30 degrees C 16. Data interpretation A 65-year-old man is found unconscious in car with a hose from exhaust into the car. The following are values obtained from initial investigations: PH 7.1 Temperature 33.3 degrees C pCO2 - 4.1 kPa COHb 40% pO2 9.2 kPa Base excess: 15 mmol/L Answer 10 true/false questions, including: - Pulse oximetry will show 85% saturation - Needs oxygen through Hudson mask at rate of 10 L/min - Metabolic acidosis returns to normal when hypothermia is corrected - PCO2 levels lower than actual because of hypothermia - Hyperbaric is of no use if oxygen is given effectively - Patient would require intubation - Patient requires bicarbonate administration

991

Primary OSCE 11 1. Data interpretation A 60-year-old man presents for elective transurethral resection of the prostate. He had a cerebrovascular accident one month earlier, and still has a left hemiparesis. He has been given an anticoagulation therapy. His lab results are: International Normalized Ratio (INR) 3.5, activated partial thromboplastin time (APTT) 45 seconds, thrombin clotting time (TCT) 15 seconds. Answer the following true/false questions: a. The patient is on warfarin. (T/F) b. It is safe to give this patient a spinal anaesthetic. (T/F) c. The patient will be deficient in clotting factors 2, 7, and 9. (T/F) d. The patient should be given vitamin K immediately. (T/F) e. The patients hemiparesis is a contraindication to using muscle relaxants. (T/F) 2. You are presented with a pulmonary artery flotation catheter , showing trace A and trace B Trace A shows the correct trace of a catheter going from the right atrium, right ventricle, pulmonary artery, and is wedged but the pressures are very high. Trace B shows the catheter passing back into the right atrium from the right ventricle. Answer 10 true/false questions. 3. Equipment Demonstrate safe defibrillation on a pulseless dummy in ventricular tachycardia. - Where should the paddles be placed? - Demonstrate an alternative position for the paddles - What would you do if the patient has a pacemaker? What energy levels would you apply? 4. Safety You are shown two endotracheal tubes, one is laser resistant, and the other is a standard portex endotracheal tube. - Which of these is used in ear, nose and throat surgery? - Which is safe to use in gynaecological surgery? - What safety precautions should be taken, both for patients and staff, during laser use? - Shown goggles for CO2 laser - Why are goggles required for CO2 laser surgery? 5. Anatomy You are shown a diagram of the cross-section of the spinal cord. - Identify the tracts - What is in the gray matter - Describe the blood supply to the spinal cord - What happens in the case of an anterior spinal artery thrombosis? 6. History Take a history from a pregnant woman, at 37 weeks gestation, and give her advice about pain relief during labour. 7. Examination You are asked to examine a head injury patient in the Accident and Emergency department. (Examine particularly the Glasgow Coma Score).

992

8. Anatomy Name the nerves supplying the foot. How would you perform an ankle block? Describe the course of the saphenous nerve. 9. Data interpretation. 50-year-old woman presents for a rigid bronchoscopy. Her lateral chest X-ray shows a fluid level in the chest. - Answer 10 true/false questions 10. Data interpretation Statistics: what is a correlation and regression coefficient? 11. Resuscitation Perform basic life support on a 6-year-old child who has collapsed, assuming no trauma. The monitor shows sinus rhythm but no pulse. Give five reversible causes of this rhythm and describe what immediate actions you would take. 12. History Take a history from a man presenting for nasal surgery. He has a history of hypertension, and maintains that he needs the surgery because he stops breathing at night. 13. Communication A patient you have seen a day earlier, who is on your list for minor gynaecological surgery, comes into reception, appearing nervous. She has a history of controlled hypertension. You had planned to pre-oxygenate her, but she says she is anxious about the mask, as she remembers having an anaesthetic mask held over her face as a child. 14. Machine check. Carry out a check on a circle system. - Can this be used without soda lime? 15. Monitoring You are shown some capnograph traces. - What do they show? - What can capnography be used to monitor? - What is the principle behind this equipment? 16. Equipment You are shown a picture of Dinamap - What are its operating principles? - You are shown a trace of pressure oscillations within the system; explain how you would derive the systolic blood pressure, diastolic blood pressure and mean arterial pressure from this. - What is the correct size for the blood pressure cuff?

993

Primary OSCE 12 1. History 35-year-old Mrs Rehman presents for elective hysterectomy with haemoglobin of 8.1 g/dl. She has a past medical history of six babies, but had no problems with any. She has a history of cough/sore throat and night sweats for 6 weeks. Her GP has done blood tests for the same but she does not know the results. She has also had menorrhagia for 6 months. Take a full history. 2. History A healthy African presents for day case arthroscopy. He has a previous history of road traffic accident, with fractured ribs, splenectomy, ITU admission and ventilation. He has no other significant medical history. He has been tested for thalassaemia, but does not know the results. Take a full history. 3. Communication A Jehovahs witness presents for elective hysterectomy, and you are asked to obtain consent. The patient will not accept anything containing animal products, and has refused preoperative blood donation. Ask the patient about erythropoietin etc. She is happy to defer surgery until her Hb level improves, and happy for a cell saver (her attitude being the blood must remain in circulation ). She is a very cooperative patient. 4. X-ray A thoracolumbar 3-D reconstruction of the image, easily showing a wedge compression fracture of L1 vertebra with obvious displacement of the same. Patient had fracture of both tibia and femur. Answer questions on the following: - Is the L2 nerve root intact? - What might the hypotension (96/66 mmHg) and heart rate of 130 bpm be due to? - Are steroids are indicated? - Does the patient need urgent decompression? - Can only the lumbar vertebrae be seen? - is T12 with the rib easily seen? 5) Nerve blocks and anatomy You are asked to demonstrate how to perform a 3-in-1 block, and are questioned about: - the distribution of the nerves and dermatomes - the level of analgesia - the nerve root of the femoral nerve and obturator nerve - the nerve supply of adductors - how to block the ilioinguinal and iliohypogastric nerve (their nerve root value) 6) History You are presented with an 80-year-old patient who needs four pillows to sleep; if the pillows slip, she begins to cough. Her ECG shows AF. Answer 10 true/false questions, including the following: - axis N/abnormal - left bundle branch block is present - QRS duration is normal - peribulbar block is satisfactory for this patient

994

- the ECG shows signs of digoxin toxicity 7) Capnograph trace A patient is in the prone position. You are shown the capnograph trace and asked a series of true/false questions, including: - surgery should be stopped immediately - there is a risk of cerebral hypoxia present - the slope of the inspiratory phase is dependent on tidal volume - D is the end tidal point 8. Examination of the airway A patient has a grade III Mallampati and a rather short neck. You are shown a head and neck lateral X-ray and asked to comment on whether the intubation will be difficult . The X-ray is of poor quality, but shows temporomandibular ankylosis. 9. Examination of the cranial nerves Carry out a full clinical examination of the cranial nerves. 10. Resuscitation station A young adult is seen near a pond, collapsed. The information card says to assume drowning with no trauma. Shout for help etc, and perform resuscitation for 1 minute. At the end of administering artificial breathing, demonstrate how to put the patient into the recovery position. Fill in an algorithm for the rhythm on the monitor (VF). 11. Anatomy The heart: - how many layers does it have and what are their names? - what is the function of the sinus of Valsalva? - from which aortic sinus do the coronary arteries arise? - explain venous drainage - how many cusps are there in the aortic/mitral/tricuspid valves? - what are the cordae tendinae and what is their function? - what happens if the cordae rupture? - what is the dominant artery? - which nerves supply the SA and AV nodes? You are shown a coronary angiogram: - identify the coronary artery? - what is the pathology here? - can you identify anything other than atherosclerosis? - which vessels are narrowed? - which part of the myocardium is now at risk? 12. Machine check You are asked to check a Bain circuit. There is a big hole in the outer tubing, in the undersurface, where it will attach with the adjustable pressure-limiting (APL) valve (not easily visible) - answer questions on fresh gas flow, minute volume etc. - what will happen if the bag of the circuit fell off suddenly when the patient is breathing spontaneously?

995

13. Statistics Answer 10 true/false questions (no negative marking): (Regarding different categories of data). 14. Entonox You are shown a photograph of entonox - what is the pseudocritical temperature? - what happens at this temperature? - what is the critical temperature? - does the gauge press reflect the amount left? - who else is authorised to use entonox other than doctors? - describe the 2-stage regulator 15. Anaesthesia hazards You are shown various filters, then asked questions, including: - identify an epidural filter; what is its use? - identify a blood filter; how often it should be changed? - identify a bacterial filter in an anaesthetic breathing circuit; answer questions on dead space - identify a white blood cell (WBC) filter) - what virus is transmitted by WBC? - arrange the following in order of size, starting with the smallest: platelets, red blood cells and coccus 16. Check the arterial line set-up, which might contain the following faults: - venflon cannula with side port, kinked 18G - ordinary iv tubing instead of non-compliant - glucose instead of Hepsal - non-pressurised bag - lots of air in the tubing - how would you zero? - what is high pressure zero? ArticleDate:20040907 SiteSection: Article Primary OSCE 13 1. Resuscitation You are asked to complete an asystole algorithm on an adult 2. Anaesthetic machine check You are asked to identify the following faults: - no spare O2 cylinder - half empty cylinder - no blanking plug in CO2 cylinder - vaporiser empty - no O ring

996

3. Statistics You are shown a positive skewed distribution and asked 10 questions 4. Chest X-ray You are asked 10 questions 5. Trauma You are asked about the features of tension pneumothorax 6. Hazards - examiner sitting 7. Electrical safety You are asked 15 questions on the peripheral nerve stimulator, including: - demonstrate its use - identify patterns and duration of impulse - what happens during depolarising block and non-depolarising block? 8. Agent analysis You are asked 12 questions 9. Rotameter You are shown a diagram and asked 10 questions 10. Spinal cord transaction You are asked 10 questions 11. Examination of a pregnant patient 12. Communication skills Speak to a mother whose son has suxamethonium apnoea 13. History taking station 14. History taking station 15. Base of skull You are asked questions, mainly on the trigeminal nerve ArticleDate:20040907 Primary OSCE 14 1. History taking Take a history from a woman listed for hysterectomy. She is from Bangladesh and has a history of night sweats, weight loss, low Hb and cough. She is known to have thalassaemia. 2. Examination Examine the respiratory system and also show how to use a peak flow meter. 3. Anatomy

997

- Discuss the anatomy of the heart, its blood supply and the pressures in various chambers. - You are shown a coronary angiogram - name the arterial branches pointed out. - Give a detailed description of the pathway of the Vagus nerve. 4. Equipment - Diathermy: you are shown two plates (one for a child, one for an adult); discuss the current density and frequency used. - Discuss pacemakers in detail; what codes are used (e.g. VVI or DDD etc)? - You are shown four graphs; which is the correct one for clotting and which for coagulation? 5. Anatomy - Discuss the base of the skull and the pathway of the trigeminal nerve (which holes in the skull do the branches pass through, which branches/ganglia are motor/sensory?). - Discuss trigeminal neuralgia. 6.Equipment - You are shown a stethoscope with an ear piece attached; where could it be used? - Discuss how to confirm correct intubation and what CO2 traces you would expect to see. 7.Equipment Carry out a Bain circuit check; discuss the volume of the circuit, what happens if the tubing is very long and the flow rates required. 8. Equipment Defibrillation: you are shown a mannequin attached to a defibrillator, with the monitor showing ventricular fibrillation. - Demonstrate the use of the defibrillator and discuss its use in different circumstances. 9. Anatomy Discuss an X-ray of the cervical spine. 10. MCQs 11. Equipment Rotameters: examine various pictures and discuss (flow in rotameter and function of the bobbin etc.). 11. Resuscitation You are shown a mannequin on the floor who you are told is a victim of near drowning. - Perform basic life support and go through the algorithm for ventricular fibrillation. 12. History taking Take a history from a 69-year-old man listed for transurethral resection of the prostate. He has a history of mild bronchitis. 13. Communication Explain to a very angry man that you have knocked his tooth out while trying to intubate him! 14. Data interpretation - Discuss pulmonary artery catheters.

998

- Discuss various statistical terms (regression lines etc). - Discuss the results of a cardiac output study (systemic vascular resistance/pulmonery artery pressures etc.) Primary OSCE 15 1. Anatomy of the heart - What are the three layers of the heart? - Where is the atrioventricular node (AVN) and the sinoatrial node (SAN)? - Describe the venous drainage of the heart - How many leaflets does the mitral valve have? - where does the left coronary artery/right coronary artery originate? - What are the chordae tendinae and what are they joined to? - What are the normal RV pressures? - What is the normal left atrial pressure (LAP)? You are shown an X-ray (angiogram): - what is the arrow pointing to? 2. Communication You have just anaesthetised a boxer for an MUA nose after a fight. You were unable to intubate him and had to use a laryngeal mask. You knocked off a crown during the attempt to intubate him. You are now about to see him on the ward to explain what has happened. He is very aggressive, threatening to sue, and wants the hospital to pay for dentistry. 3. Equipment Humidity: you are shown a photograph of a piece of equipment: - What is it? - Define relative humidity and absolute humidity (you are required to give a very precise definition - remember to mention and define pressure/temperature) You are shown drawings (taken from Parbrook): - Which of these drawings is the hair hygrometer? - Which of these drawings is the Regnaults hygrometer? - Which of these drawings is the wet/dry bulb hygrometer? - How does the latter equipment work? - Define the latent heat of evaporation - How would you relate the readings on the thermometers to the humidity? You have two gas samples, one at 20 degrees C, one at 37 degrees C; both have a relative humidity of 50%: - which one has the highest absolute humidity? 4. Resuscitation You find a child collapsed and unresponsive. You have a bag and mask. You are alone. Please proceed with resuscitation. - How long would you carry on giving basic life support? - What would you do then?

999

You are shown a monitor (with questions on a sheet): - What is the rhythm shown? - Fill in the algorithm 5. X-ray You are given the history of a 20-year-old female who is involved in a road traffic accident. She has facial injuries. She was initially OK, but is now becoming drowsy, with severe facial swelling. You are shown a computer screen with a 3D CT reconstruction of her skull on it: This shows fractured right maxilla, fractured right zygoma, fractured right frontal bone (through frontal sinus). Comminuted fracture of the orbit. Answer the following true/false questions: - This shows a LEFT zygomatic fracture - The arrow points to the supraorbital foramen - She is at risk of intracranial infection - She is likely to require prolonged ventilation in the ITU - She requires urgent transfer to a neurosurgical centre - She is likely to suffer a left sixth cranial nerve palsy - She requires speedy intubation as she is likely to become difficult later 6. Statistics You are shown graph of age (x-axis) versus height (y-axis) for children with scattered points on it. Answer the following true/false questions: - the line of best fit has a gradient termed the regression coefficient - the regression coefficient is denoted by the letter 'r' - it is possible to have a correlation coefficient of 1.1 - it is possible to have a correlation coefficient of minus 0.7 - an 'r' value of 0.7 would be statistically significant - it is customary to plot the dependent variable on the x-axis (abscissa) - the coordinate (mean x, mean y) would lie on the line of best fit - these data could be analysed using a non-parametric test 7. Physical examination Examine this man's radial pulse and praecordium. You should not attempt to examine any other part of the cardiovascular system. Describe what you are doing as you go along. Depending on your description of your actions, various questions will be asked; for example: - What information can you get from the pulse? - What would an irregular pulse mean? - What would a collapsing pulse mean? - Can you say anything else about the pulse? - What would radial/radial delay denote? - What is radial/femoral delay? - What would you hear better by tilting the patient?

1000

8. Anatomy You are given the front half of a skull: - What is the optic canal? - What is the superior orbital fissure? - What is the intracranial relationship of the optic nerve to the carotid artery? - What happens to the fibres of the optic nerve when entering the skull? - Do the medial fibres or the lateral fibres decussate? - If you had a bitemporal hemianopia, where would the lesion be? - What would be the most common cause of this? - What is the intracranial length of the optic nerve? - Where do the fibres of the optic nerve radiate out to? - What is the length of the optic nerve in the orbit? - What characteristics would the needle used for a retrobulbar block have? - How long would it be? - What are the bony landmarks? - Where would the needle be directed towards? - What are the complications of a peribulbar block (three wanted)? 9. History taking You are about to meet a young woman whom you are to anaesthetise tomorrow for a subtotal thyroidectomy. Take a pre-anaesthetic history. 10. Equipment You are given a blood giving set: - What is filtered by this? - What is this filter used for? - What is this type of filter called? You are shown a white blood cell (WBC) filter: - What common infectious agent is carried in WBCs? - What percentage of the population have cytomegalovirus? You are given an epidural filter: - What is this used for? - What does it filter? - What is the pore size? - How does it filter viruses? You are shown a heat-moisture exchanger: - What is this used for? - Would it be appropriate for use in a 10 kg child breathing spontaneously? You are asked to examine a microscope slide of a blood film, and see a red blood cell, a platelet and a staphylococcus aureus. - Which is biggest? - Which is smallest? 11. Machine check You are asked to carry out an equipment check on an old machine with no pipelines, two Tec5 vaporisers.

1001

Possible faults include: - O2 analyser not working - CO2 missing, no blanking plug - Both vaporisers had one O ring missing - O2 failure alarm not working 12. Data/ECG You are presented with an ECG showing a ventricular paced rhythm. Answer the following true/false questions: - The rate is 70-75 - The QRS duration is 0.12 seconds - The t wave inversion is indicative of ischaemia - The calibration spike is the usual 2 cm to 1 mv - There is evidence of spontaneous atrial activity - Bipolar diathermy would be contraindicated in this case 13. History You are about to see a woman who is to have a mastectomy tomorrow. Take a pre-anaesthetic history. Points that you should pick up: The patient has a hickman line and is on chemotherapy. She is allergic to eggs, seafoods, elastroplast and iodine. You discover that her sister has malignant hyperthermia, but the patient has been tested negative. She has had no problem with previous anaesthetics. 14. Data Clinical history: A 60-year-old man is due to undergo a transurethral resection of the prostate (TURP). He has a respiratory rate of 33 bpm and can speak six words before getting SOB. On vitalograph spirometry, he has an FEV1 of 0.3 L and FVC of 0.4 L. Answer the following questions: - Does he have a normal FEV1/FVC ratio? - Does he have primarily obstructive lung disease? - Is this corrected to btps? - He is due to have anaesthesia for elective upper abdominal surgery. Would this be precluded in this case? - Would spinal anaesthesia for the TURP have no detrimental effect on respiratory function? - Are his blood gases likely to be normal? 15. Techniques You are shown a model of a neck: - Identify the thyroid cartilage - Identify the cricoid - Identify the cricothyroid membrane - Describe how you would anaesthetise the skin for a cricothyroidostomy - Describe step by step how you would use this equipment for a cricothyroidostomy

1002

16. Measuring equipment Pulse oximetry - What information is given by a pulse oximeter? - How does a pulse oximeter work? You are shown a Hb/O2 dissociation curve - What is the pO2 at this point (90% saturation)? - What is the point at 50% saturation called? - What is the reason for measuring the p50? You are shown graph of absorbance spectra for Hb/HbO - What is the marked point? (isosbestic point) - What does this trace signify? (blood pressure cuff going up and down) - Can you give three reasons for inaccuracies? 17. Other recent OSCE questions - Coeliac plexus anatomy - Demonstration of how to do a cricothyroidotomy - Checking a Bain circuit - Thoracic surface anatomy: where to do an emergency then elective chest drain, intercostal blocks and the anatomy of the first rib - What is wrong with the arterial line set? (no pressure bag, 5% dextrose, venflon with injecting port instead of arterial cannula, etc) - Complete a cranial nerve examination and brief discussion on brainstem death testing - Pre-operative assessment of a day-case patient for removal of wisdom teeth; the patient is needle phobic and had sustained a broken nose in the past - Counselling a patient on the methods of pain relief following a cholecystectomy, with particular reference to PCAs (pros and cons of PCAs versus epidurals) Primary OSCE 16 1. Examine a patient in the Accident and Emergency Department with a head injury - Does the patient respond to commands? - Does he open his eyes to commands? - A cervical collar has been kept on what does this suggest? - Check the pupillary reflex with a torch - Check if the patient moves all four limbs in response to commands - Is the gag reflex present? - What is his Glasgow Coma Scale score? 2. Anatomy: base of skull and peribulbar block - Identify the cribriform plate, optic canal and supraorbital fissure - What structures pass through the cribriform plate? - What happens in the Leforte 3 fracture and what are the signs? - What is the length of the optic nerve in the orbit? - Describe the somatosensory supply of the eyeball - Describe how to perform a peribulbar block and its complications - Describe how to perform a retrobulbar block

1003

3. ECG True/false questions on rate and sinus rhythm - Is LVH or LV strain present? - Is a prolonged PR interval present? - Is a prolonged QRS complex present? - What could the tall T waves be due to? 4. Pulmonary artery (PA) catheter trace Short history: A patient is in pulmonary oedema, and you are inserting the PA catheter through the right internal jugular vein. You are shown the following PA catheter trace. Answer the following true/false questions: - Point A represents systolic pressure in the PA - Point B could be a giant V wave - Point C could be due to catheter whip - Point D appears after the T wave in the ECG, CPAP would decrease the cardiac output - The patient is in sinus rhythm - Nitrates will decrease the LV compliance - Frusemide might improve his condition 5. Rapid sequence induction (RSI) - Why would you perform a rapid sequence induction? - Why do we prefer to apply pressure to the cricoid cartilage? - What section of the spine should be compressed? - Name a condition in which you would not apply cricoid pressure in emergency surgery - When should the pressure be released? - What should be checked before releasing the pressure? - How much pressure is needed? - How would you demonstrate this to the operating department assistant? - Demonstrate on a manikin how much pressure should be applied 6. Electrical hazard You are shown a picture of a man lying on a surgical table connected to a central venous pressure transducer with current leaking. He is also connected to an ECG monitor: - How much current is needed to produce a microshock? - Why is the microshock current not being transmitted to the ECG? - In an isolated floating circuit, how is the patient protected from electric shock? - What happens if a 1 amp current is delivered to you? - How is the earth potential made uniform? - What do the electrical symbols represent? 7. Anatomy: Rib - Identify the rib as right/left sided - Is it an upper or lower rib? - What are the surfaces? - What structures pass under it? - In what order do these structures occur? - You are shown a first rib: identify the structures passing over it and muscles attaching to it - What are the complications of intercostal nerve block?

1004

8. History A middle-aged woman comes in for a laparoscopic cholecystectomy. Her sister had previously experienced some problems in recovery and been sent to the ICU with suxamethonium apnoea, cough, lower respiratory tract infection and regurgitation. The present patient is an ex-smoker. Take a full history and elucidate all of the salient points relevant to this patients anaesthetic. 9. Bain circuit Carry out a circuit check. 10. Intra-arterial blood pressure set-up - Identify the faults concerned with this set-up - What is the flow rate of the heparinised saline? - Why is the pressure not transmitted to the trace? - What is meant by high calibration? 11. X-ray An 8-year-old child has aspirated a peanut. Answer the following true/false questions: - There is complete collapse of left lung - The Heimlich manoeuvre is contraindicated - Urgent bronchoscopy is needed - Antibiotics and a chest drain should be given during the first 24 hours - A fracture is seen in the head of the humerus - what might this be caused by? 12. Statistics Answer a series of true/false questions. - Correlation coefficient: two graphs are presented, each one representing two cities relating height to drug dose. Describe the correlation in each case. 13. Resuscitation Paediatric basic life support. - Fill in the algorithm for asystole 14. History A patient presents for septoplasty, giving a history of snoring, hypertension and reflux 15. Communication A child undergoing an emergency appendicectomy stopped breathing after the operation and is now in recovery (suxamethonium apnoea). Explain the proceedings to the parents. 16. Equipment You are shown a photograph of a Dinamap. - What is the operating principle for this equipment - i.e. what does it actually measure and what is derived? - What is the size of the cuff? - What are the sources of error?

1005

- Why does it take at least 2 minutes to perform the measurement? - What happens when measurements are repeated? - Which nerves are affected? Primary OSCE 17 1. Explain how cardiac output is measured using a thermodilution technique. 2. Describe how the partial pressure of oxygen in a blood sample is measured using a Clark electrode. 3. Differentiate between the terms heat and temperature. 4. Briefly explain the principles of a mercury thermometer, indicating its advantages and disadvantages. 5. How does a fall in temperature influence blood gas solubility and acid-base values? 6. Explain the physical principles of ultrasound imaging. 7. Briefly explain the principles of Doppler ultrasound, used to measure cardiac output by echocardiography. 8. Briefly describe the differences between laminar and turbulent flow. 9. List the factors that increase the probability of turbulent flow. 10. Briefly describe the measurement of blood pressure using an automated oscillometric noninvasive blood pressure monitor. 11. Briefly outline the problems associated with this kind of monitor. 12. Briefly explain the causes of the differences between measured end tidal and arterial partial pressures of carbon dioxide. 13. Briefly explain the principles of the thermistor, indicating its advantages and disadvantages. 14. Outline the principles of the pneumotachograph. 15. What factors affect the accuracy of this device? 16. Compare two methods for measuring humidity. 17. Briefly discuss the factors which influence the measurement of oxygen saturation using a pulse oximeter.

1006

Primary OSCE 18 1. History taking Take a history from an elderly gentleman presenting for carotid endarterectomy with reflux and ischaemic heart disease. 2. History taking Take a history from a man presenting for septoplasty with significant sleep apnoea and reflux disease. 3. Communication Talk to an anxious patient in theatre reception who is unhappy about use of the mask for preoxygenation. 4. Anatomy You are shown an angiogram of the heart. - Describe the course of the vagus nerve 5. Anatomy Describe the anatomy of the antecubital fossa and nerve blocks which may be performed at the elbow and wrist. 6. Statistics Answer a question on the regression coefficient. 7. Data You are shown an ECG of a patient in atrial flutter. - Answer the associated questions 8. Data You are shown a chest X-ray of a child who has inhaled a peanut. - Describe the radiographic findings 9. Machine check You are shown a Bain circuit. - Answer the questions relating to the bag falling off 10. Equipment You are shown a praecordial monoauricular stethoscope and asked about its uses. 11. Equipment You are shown a Bourdon gauge. - Answer the questions concerning whether it can act as a contents gauge for N2O 12. Resuscitation - Describe basic life support for a child - List five causes and treatment of electromechanical dissociation (PEA) 13. Equipment Lasers:

1007

- What are their principles of action? - Describe safety aspects and give examples 14. Examination Neurological: - AVPU (Alert, Verbal, Painful, Unresponsive), Glasgow Coma Scale (GCS) and pupils - Management of head injury Primary OSCE 19 1. Humidity - What is a Regnault's hygrometer? - Where is it used and what is it used for? 2. Coronary circulation Describe the course of the vagus nerve. 3. Microshock - What do you understand by the term "earth"? - You are shown a list of electrical symbols and asked to identify them 4. History taking a. Take a history from a patient with a family history of malignant hyperthermia. b. Take a history from a patient who may have had suxamethonium apnoea following a previous operation. 5. Resuscitation of a pregnant woman - You are asked to identify the rhythm (shows ventricular fibrillation) - You are asked to fill in boxes concerning the management of this patient 6. Data Interpretation - Head CT: Describe the radiological findings - ECG: Describe the condition shown 7. Data Interpretation You are asked about the measurement of blood gased for hypothermia and carbon mnoxide (CO) poisoning . 8. Machine check Check a Bain circuit which is faulty. 9. Intubation drill Explain the intubation drill to a novice anaesthetist. 10. Clinical - Examine the chest of a patient with chronic obstructive airways disease - What precautions would you take when anaesthetising this patient?

1008

Primary OSCE 20 1. Examination You are asked to examine a patient: - How do you locate the femoral artery? - Where do the femoral nerve and vein lie in relation to this? - What are the roots of the femoral nerve? - Which nerves are blocked in a three-in-one block? - What do these nerves supply (motor and sensory)? - How would you go about blocking the ilioinguinal nerve? - What other nerve is blocked when you do this? - Between which two muscles are you infiltrating in this block? - You are shown a diagram: which dermatomes are highlighted here? (diagram has L2 and L3 highlighted) - Examine the third to the twelfth cranial nerves on this actor, explaining which nerves you are checking as you do so 2. Resuscitation You are walking past a river when you come across an unconscious patient who is not breathing but has a pulse. - Demonstrate how you would react - Look at the ECG shown here and fill in the blanks on the algorithm to explain how you would treat the patient 3. CT You are presented with a computer screen showing the three-dimensional CT scan of an L1 wedge fracture. This is the CT of a 27-year-old builder who has fallen from a building. He has also fractured both femurs and his left tibia. His pulse is 120 bpm and his BP is 90/60 mmHg. Answer a series of true/false questions relating to your management of the patient and take a further history. 4. ECG Answer a series of true/false questions relating to an ECG showing left ventricular hypertrophy and T-wave inversion in leads V3-V6. 5. Equipment You are presented with a diagram showing a capnography trace with two normal traces followed by a sudden absent trace. This is the capnography tracing of a 26-year-old man who is undergoing a lumbar discectomy and is currently prone and ventilated. Answer a series of true/false questions relating to the normal traces then the sudden loss of CO2 - What are the likely causes of the sudden loss of CO2? 6. History A patient is about to undergo a total abdominal hysterectomy. She does not wish to receive any blood products on religious grounds. A special consent form is provided detailing that she has refused blood products and that you have discussed this with her, and that she does not

1009

hold you or the surgeon morally responsible if she suffers injury from withholding a blood transfusion. Obtain the patients signature for this consent form. 7. History Obtain an anaesthetic history from a patient who is about to undergo a knee arthroscopy. The patient appears to be a fit and healthy 25-year-old male, whose only significant history is that he was involved in a road traffic accident three years previously, fracturing a femur, his pelvis and a couple of ribs and was ventilated overnight in the intensive care unit. 8. History Obtain an anaesthetic history from a 45-year-old female who is about to undergo a hysterectomy. She is fit and healthy but has undergone a previous caesarean section under general anaesthetic and can remember the laryngoscope being passed into her mouth. She also suffered a deep vein thrombosis, which led to what may have been a pulmonary embolism, but this was not confirmed at the time and the patient maintains that she was not treated for it. 9. Equipment check You are asked to carry out a check on anaesthetic equipment with no breathing system attached. - What type of machine is it? (Boyles) - What faults can you find? (e.g. vaporiser is empty, with no interlocking device between the O2 and N2O rotameters, preventing delivery of 100% N2O) 10. Questions on the indications and contraindications for cricoid pressure - How much force should be applied? - When should the pressure be released? - Who should perform it? - Demonstrate on a manikin how to perform it 11. Equipment You are shown a variety of filters. - Describe the use of each of these filters, and answer some supplementary questions (e.g. a leucocyte filter, a filter in a standard blood giving set, an epidural catheter filter and an HME with a bacterial filter) - What are the potential problems with using an HME in a paediatric patient? - What does the epidural filter actually filter? - Rank the following in order of size: a platelet, a bacterium and an erythrocyte filter 12. ECG - What is the usual speed of an ECG recording? - What is the calibration signal? - How can interference be reduced? 13. Anatomy of the heart - Where are the sinoatrial (SA) and atrioventricular (AV) nodes? - Where are the coronary arteries and veins? - What are the sinuses of Valsalva and what is their function? - You are shown a coronary angiogram: what is the pathological process demonstrated at the labelled points? - Which artery was this affecting?

1010

14. Statistics Answer true/false questions; for example: - What type of data is quoted? - Do non-parametric equations require equal numbers in each group? Primary OSCE 21 1. Anatomy Describe the blood supply to the heart Discuss the pacemaker activity in the heart Describe the course of the vagus nerve, from the head to the abdomen 2. Peri-arrest arrhythmia Fast atrial fibrillation What are the risk factors for patients developing this condition? What drugs should be used? What DC shock energy levels should be used? 3. Anatomy/hazards You are shown a series of pictures of possible nerve damage to the face while wearing a facemask. Describe the nerves involved. 4. Interpretation of forced expiratory volume graphs You are asked 10 true/ false questions about restrictive lung disease. 5. History station A 38-year-old woman presents for a total abdominal hysterectomy. Why might the woman be having the operation? What postoperative analgesia would you give? 6. Physical examination You are asked to perform an examination of the cardiovascular system. 7. Anatomy You are shown a diagram of a cross-section of a spinal cord, and asked to indicate the ascending and descending tracts. What would you see if you were examining gray matter under a microscope? 8. Statistics You are asked true/false questions on the Bland & Altman Plot used to compare two different ways of measuring blood pressure. 9. Humidity Define humidity What units is it measured in? Describe some methods of humidification 10. ECG You are shown an ECG showing supraventricular tachycardia.

1011

What are the risk factors for developing this condition? What treatment would you give? 11. History station Take a history from a 60-year-old man presenting for removal of a mole. 12. Communication How would you convince a needle-phobic Afro-Caribbean patient undergoing arthroscopy of the knee for sickle cell testing? 13. Viva You are asked about ECG interference and differential amplifiers. 14. Comparison of two laryngoscope blades You are asked to identify two laryngoscope blades (Macintosh and Robertshaw). Give four uses for Magill forceps 15. How to position a patient for spinal anaesthesia Choose which spinal needle to use and explain why What should you do in the event of severe hypotension while performing a spinal anaesthesia? 16. Chest X-ray You are shown a chest X-ray of interstitial fibrosis and are asked 10 true/false questions. Primary OSCE 22 1. Anatomy of the anterior cranial fossa - What passes through the cribriform plate? - Describe Le Fort fractures and the anatomy of the optic foramen - Describe the technique of performing regional eye blocks using the skull provided - Answer follow-up questions on needle characteristics/complications 2. Anatomy of the lumbar vertebrae - Discuss the atlas and axis - Describe the ligamentous relationships of the spine - Describe the anatomy of the sacral hiatus - Where does the cord end in the adult/child? - Discuss the relations of the epidural space 3. Statistics Answer questions on normal distribution curves. 4. Resuscitation - Perform basic life support with a bag/valve/mask on an adult mannekin - Interpret an ECG shown on an oscilloscope - Fill in the blanks in a supraventricular tachycardia algorithm

1012

5. Data interpretation An 86-year-old male patient is admitted for a transurethral prostatectomy. He has a history of controlled atrial fibrillation and recent hemiplegia. His blood results show an international normalised ratio (INR) of 3.5, thrombin time (TT) of 12 seconds and an activated partial thromboplastin time (APTT) of 35 seconds. - Is the patient fit for surgery? - Should he be on warfarin? - Does he have normal liver function? - Is he at risk from hypertensive stroke? 6. ECG A 25-year-old male patient is admitted for day-case herniorraphy. He has a history of palpitations, and his ECG shows that he has Wolff-Parkinson-White (WPW) syndrome. - What is the calibration time for a 1 mV spike? - Does the patient have normal atrioventricular conduction? - Is there any p-wave activity? - Is pre-operative pacing required? 7. Chest X-ray You are shown a mislabelled right lateral chest X-ray. It shows a hiatus hernia. - What is the fluid level in the chest? - Would you give pH-raising agents? - Is it safe to use cricoid pressure? - Can you see the hemi-diaphragm? 8. Equipment You are shown an oesophageal stethoscope (what are its uses?) and a urethral catheter with a thermocouple attached to it for bladder temperature monitoring. - Answer questions on temperature - What is the SI unit for temperature? - Calibration scales? - Describe the use of a thermocouple? - Can you see the beck effect? - What is the triple-point of water? An actor poses as a patient; apply peripheral nerve stimulation (PNS) electrodes to his wrist and describe their operation. You are shown pictures of train-of-four (TOF) and double-burst stimulation (DBS), showing various types of block: discuss the use and specifications of nerve stimulators. 9. Safety Answer questions on diathermy plates. 10. Equipment check You are shown a metal Miller blade and a plastic fibreoptic Macintosh blade - Discuss the differences and uses of the two pieces of equipment - What is the pressor response to intubation? - List four uses of Magills forceps

1013

11. History a) Take a history from a 35-year-old patient presenting for elective thyroidectomy. The patient is allergic to shellfish. b) Take a history from a 40-year-old patient presenting for TAH. The patient has previously been admitted to the intensive care unit with renal failure, although this information was not volunteered initially. 12. Communication Explain to a lay-person about a failed intubation drill. 13. Examination An actor poses as a patient; examine his arterial and venous pulses (he is poorly positioned for examination of the jugular venous pulse, and his left radial pulse is absent: take his blood pressure manually). Primary OSCE 23 1. Filters Discuss depth, screen, leucocyte, and epidural What is each type used for? What might each one filter out? What problems are associated with paediatric heat and moisture exchanging filters? Rank the following, from largest to smallest: red blood cells, platelets, cocci 2. Femoral nerve block Demonstrate the landmarks for this block. Describe the dermatomes of the lower limb 3. Standard Bain circuit check. Hole in tubing noted. What would happen if the bag fell off? What length should the tubing be? 4. Cranial nerve examination Given torch, cotton wool, tuning fork. Dont forget to gag! , Describe examination of the cranial nerves. 5. Cardiac vessels Where is the sinoatrial (SA) node? Where is the atrioventricular (AV) node? Where is the coronary sinus? Where is the origin of the coronary arteries? Which veins drain the heart? You are presented with a coronary angiogram; what does it show?.(stenosis of the left anterior descending artery). Which disease process gives rise to this appearance? (atherosclerosis) What is the usual ratio of depth of RV: LV? What are the sinuses of Valsalva?

1014

6. Airway assessment Carry out the following assessments: a Mallampati test; Wilson risk score; temporomandibular joint distance; Neck movements; temporomandibular joint instability How would you prepare if you anticipated a difficult intubation? 7. Data interpretation You are presented with the following patient data: K+: 6.0 mmol/L; HCO3: 14 mmol/L; pH 7.21; Urea 20; PCO2 4.0 kPa; PO2 14.0 kPa Answer the following true/false questions: This patient has a compensated metabolic acidosis This patient requires dialysis prior to a general anaesthetic These results could have been obtained as a result of sample haemolysis 8. Data interpretation You are presented with a capnograph and asked a series of true/false questions. 9. Statistics You are asked a series of true/false questions Explain the following terms: Nominal, interval, continuous data Beta error Predictive values Students t test 10. Computed tomography You are presented with a CT scan of a lumbar spine. Answer the following true/false questions: There is a crush fracture of L2 This patient could be treated with steroids This patient will have damage to the anterior longitudinal ligament This patient may be paralysed from L2 down 11. Communication A Jehovahs Witness presents for hysterectomy. Go through the consent form with the patient and explain the risks. Ascertain the products that the patient would accept (for example, is autotransfusion acceptable?) The patients current haemoglobin level is low; how would you treat this? (erythropoietin, iron, need for senior involvement). 12. History taking Take the history of a woman presenting for hysterectomy with a history of night sweats, red tongue, weight loss and foreign travel. 13. History taking Take a full anaesthetic history of a woman presenting for cataract surgery, (elicit non-insulindependent diabetes mellitus, eye and foot problems, control of diabetes). Ask the patient her weight (if the patient is overweight - e.g. 130 kg - you may have to explain that a local anaesthetic would be more appropriate)

1015

14. Resuscitation You are presented with an adult who is the victim of a drowning incident. Perform basic life support and put the patient into the recovery position; fill in a ventricular fibrillation protocol. 15. Equipment You are shown a photograph of a cylinder; what does it contain? (Entonox) What is the definition for pseudocritical temperature? Besides the anaesthetist, who else is authorized to administer entonox? Describe how the demand valve works Discuss the risks of entonox in different climates 16. Arterial line set-up You are presented with an arterial line set-up. Check the equipment for faults (e.g. glucose in the bag, no heparin, no pressure, compliant tubing). Explain how the equipment should be zeroed and calibrated Primary OSCE 24 1. You are shown various laser tubes When would you use a metal tube? Why would you not use other tubes in this situation? What risks would be incurred with other types of laser tube? Can these laser tubes be used in gynaecological procedures? Name three safety precautions that must be taken in the operating theatre when using lasers. Why are goggles needed for protection only when using a carbon dioxide laser? 2. The spinal cord What volume of epidural anaesthetic blocks one vertebral segment? You are shown a spinal cord; identify the spinocerebellar tract and the gracilis tract. Describe the blood supply to the spinal cord; where does it come from and what does it supply? Which tract carries pain and temperature? What is the normal volume of cerebrospinal fluid, and what is its specific gravity?

1016

3. History taking An Afro-Caribbean patient presents for day-case arthroscopy. He has previously undergone general anaesthesia for a splenectomy following a road traffic accident, and experienced heavy blood loss, with a 3-day stay in the intensive care unit, where he was ventilated. He was no longer taking any antibiotics or vaccines. Assess the patient for suitability for this day-case procedure. 4. Ankle block What nerve supplies the foot? (Peroneal nerve) How would you block this nerve? how would you block the tibial nerve? Discuss the cutaneous innervation of the saphenous nerve, medial plantar nerve, dorsal and superficial peroneal nerves. Describe the course of the saphenous nerve; where does it become superficial? What doses of lignocaine, bupivicaine and prilocaine are considered safe? 5. Cardiovascular examination What are you generally looking for when measuring a pulse rate? What is the cause of a collapsing pulse? Give two causes for an irregular radial pulse. What is the significance of radio-femoral delay? You are asked to examine the praecordium; carry out an inspection for visible apex beats; describe the features and assess if it is normal. Show how you would examine the mitral and aortic areas for murmurs. 6. You are shown a flow-volume loop diagram of a patient with obstructive lung disease. - Illustrate which points correspond to inspiration and expiration, and which points represents premature airway closing.

1017

7. ECG You are asked various questions about an ECG 8. Defibrillation You are presented with an ecg trace showing ventricular fibrillation (VF); what is VF, and how would you treat it? Demonstrate the proper use of a defibrillator Is the fact that the paddles are labelled apex and sternum of consequence? What precautions should be taken in regard to the pacemaker? What other paddle positions are there? When would you defibrillate during expiration or inspiration? Why? 9. History Take a history from an Asian woman who is presenting for laparoscopic sterilisation. She has had a cough for two months, night sweats, and has lost weight, although her appetite has been unaffected; she has not seen a doctor. The patient has a previous history of unexplained jaundice, from few years back, for which she did not attend hospital. She takes diazepam occasionally, which is prescribed by her GP. (Ask the patient about any previous drug injecting, which she has done on one occasion) 10. Communication You are asked to reassure a woman with a mask phobia who needs pre-oxygenation. The phobia is due to a traumatic experience as a child at the dentist. Explain that nasal cannulae, instead of a mask, can be used to administer pre-operative oxygen, and that flavoured masks can be used, to disguise the smell of the anaesthetic vapour; moreover, the patient can hold the mask on herself. Explain the need for pre-operative oxygen, and that a tight seal is necessary - if she is to hold the mask on herself, this is essential. Explain that there may also be a need for a mask postoperatively; make a deal with the patient: you will only use the mask postoperatively if it is absolutely necessary, but then she must allow it. 11. Oxygen measurement You are shown a picture of a fuel cell. What is it used for? You are then shown a circuit diagram for a fuel cell. What is solution is contained in a fuel cell?

1018

What is the anode made of? What sort of reaction occurs at the anode? Does a fuel cell need replacing after use? Why? How is a fuel cell calibrated? What is the other circuit called? polarographic electrode 12. Circle check Faults with this included: not being connected to the fresh gas flow, a stuck expiratory valve, soda lime not full. What comes out of the apl valve? Can the circle be used without a CO2 absorber? What colour are the indicators? 13. Pulse oximetry You are shown a picture of a pulse oximeter. What two pieces of information does this equipment provide? what are the values of p50 and p90? Why is the p50 value useful? You are shown graph for myoglobin: what does this represent? How does a pulse oximeter work? How does pulse oximetry differentiate between arterial, venous and tissue oxygen absorption? What is the difference between pulse oximeters and oximeters found in laboratories? 14. barium contrast films You are shown a series of barium films showing swallowing with a pharyngeal pouch. Is the cricoid cartilage essential? What is the level of origin?

1019

Is a constriction present? Can you see solid food? 15. Statistics You are shown a graph of Age (x-axis) versus Height (y-axis), and asked about Correlation and regression. What symbol is used for the regression coefficient? ( r) How is correlation determined? (line of best fit) Is it possible to have a correlation coefficient of 1.1? Is it possible to have a correlation coefficient of 0.7? Which type of test is more appropriate for analysing these data: parametric or nonparametric? (non-parametric) Is the X variable a dependant variable? (yes) Would changing the axes around have an effect on the correlation coefficient? (no) 16. Resuscitation You come across a six-year-old child who is unconscious. What would you do?

Final FRCA

Welcome to our section containing Final FRCA questions. We have split the resources into relevant sections as per the exam format. Short and long case viva can be found by on the Final FRCA section 2. In order to be eligible for a Viva a minimum of 1+ and 2 is required from the written (MCQs and SAQs). To pass overall, a minimum of 1+,2,2,2 is required. (1 poor fail, 1+ a fail, 2 pass, 2+ outstanding pass). Format of Final FRCA exam WRITTEN EXAM Morning paper

1020

SAQs (12 on clinical anaesthesia) Afternoon paper MCQ (90 questions): 20 medicine & surgery, 40 anaesthesia & pain, 10 clinical measurement, 20 ITU VIVAS Morning Viva Viva 1 (50 minutes, Clinical-long case plus short cases) Afternoon Viva Viva 2 (30 minutes, basic sciences applied to anaesthesia, ITU & pain-3 short cases) Need minimum 2,2,2,1+ to pass. Short Answer Questions (SAQs) Final 1 Final 2 Final 3 Final 4 Final 5 Final 6 Final 7 Final 8 Final 9 Final 10 Final 11 Final 12 Final 13 Final 14 Final 15 Final 16

Basic Sciences and ITU short answer questions Anaesthesia Anatomy Physiology Clinical measurement Intensive care Pain management Clinical pharmacology Multiple Choice Questions (MCQs)

1021

MCQ A MCQ B MCQ C MCQ D MCQ E MCQ F MCQ G MCQ H MCQ I MCQ J MCQ K Final 1 5th November 1996 SECTION A (Book A) 1. How would you manage a case of accidental intra-arterial injection of thiopentone in the upper limb? 2. Summarise the causes, effects and prevention of aspiration pneumonitis. 3. List the likely causes of collapse in the dental chair of a patient undergoing a procedure under local anaesthetic without sedation, with notes on the presenting signs and symptoms. Briefly state what first-aid measures can be undertaken in each case. 4. What particular problems may occur during lower abdominal surgery in a patient who suffered a traumatic transection of the spinal cord at C6 four weeks previously? Briefly indicate how you would avoid or prevent the problems you describe. 5. Describe how you would carry out an axillary brachial plexus block 6. What is your choice of anaesthesia for pericardectomy in constrictive pericarditis? Give reasons for your choice.

SECTION B (Book B) 7. A 40-year old man is admitted with an acute head injury. List the indications for intubation, ventilation and referral to a neurosurgical unit. 8. Make a simple drawing, with labels, to show the trachea, main and segmental bronchi. 9. Outline your management of an adult patient brought into the A & E department in status asthmaticus.

1022

10. List, with a brief statement on the effectiveness of each one, the means available for detecting awareness during anaesthesia. 11. What are the advantages and disadvantages of intravenous patient controlled analgesia for postoperative pain control? 12. A 60-year old man is referred to you with reflex sympathetic dystrophy following an injury at the elbow 6 months earlier. Outline the treatment. Final 2 20th May 1997 Candidates are required to answer all twelve questions. Questions numbered 1 - 6 must be answered in Book A and Questions numbered 7 - 12 must be answered in Book B. Candidates who fail to answer all twelve questions will not pass the Examination. Candidates are informed that one of the Examiners is present during the time allowed for the paper, for consultation in case any question should not appear clear. SECTION A (Book A) 1. List the dangers to the eye of general anaesthesia for elective intraocular operations. How are these prevented? 2. What are the choices for postoperative analgesia for a child aged 4 years presenting for repair of an inguinal hernia as a day case? State briefly the advantages and disadvantages of each method. 3. The first patient on your operating theatre list tomorrow morning has an implanted (permanent) cardiac pacemaker. List, with reasons, the relevant factors in your preoperative assessment. 4. What are the types of anaesthetic that should be considered for fixation of a compound ankle fracture in a patient who was briefly knocked unconscious at the accident? Outline the advantages and disadvantages of each technique. 5. An 80 year old lady with a sub-capital fractured neck of femur requires surgical fixation. She is found to be in fast atrial fibrillation. What are the important points in the preoperative preparation for anaesthesia in this case? 6. Outline the problems involved in anaesthetising an intravenous heroin abuser needing urgent surgery for incision of perianal abscess. SECTION B (Book B) 7. You have been asked to anaesthetise a 60 year old woman for ligation and stripping of varicose veins in one leg. She has a history of ischaemic heart disease. Explain briefly how suitability for her management in a day-case facility is assessed.

1023

8. A 62 year old man is admitted to the high dependency unit following a laparotomy to relieve a large bowel obstruction. He has a urinary catheter in situ. Two hours later, he has only passed 25 ml of urine. List, with reasons, the likely causes. What is going to be your initial plan of management? 9. What measurements and derived values can be made from pulmonary artery catheters used in the intensive care unit? Suggest a clinical application for each one. 10. What are the indications for performing a tracheostomy? List the complications of tracheostomy. 11. What are the problems of monitoring anaesthetised patients in the magnetic resonance imaging unit? 12. Outline your management of a fit primagravida who suffers inadvertent dural puncture with a 16 gauge Tuohy needle during attempted epidural for pain relief in the first stage of labour (cervix 4 cm dilated). Final 3 4th November 1997 9.30am to 12.30pm Candidates are required to answer all twelve questions. Questions numbered 1 - 6 must be answered in Book A and questions numbered 7 - 12 must be answered in Book B. Candidates who fail to answer all twelve questions will not pass the Examination. Candidates are informed that one of the Examiners is present during the time allowed for the paper, for consultation in case any question should not appear clear. SECTION A (Book A) 1. A 70 yr old man with chronic obstructive airways disease requires a transurethral resection of the prostate. Outline the advantages and disadvantages of intrathecal block for this patient. 2. What immunological consequences may result from homologous blood transfusion? 3. Outline the effects of old age upon morbidity and mortality in anaesthesia. 4. What hazards does a patient encounter as a result of being placed in the lithotomy position for surgery? What additional hazards are introduced by then tilting the operating table headdown? Indicate briefly how you attempt to prevent these hazards. 5. A patient on the intensive care unit has a mean arterial pressure of 130 mmHg. What drugs might be useful for reducing this to a safe level and what is the mechanism of action of each? 6. The plasma concentrations of a drug have been measured in 20 normal patients and in 20 patients with renal failure. What simple statistical tests exist to determine whether these 2 sets

1024

of observations differ at the 5% level? What assumptions are inherent in each test which you describe? SECTION B (Book B) 7. What are the pathophysiological insults which exacerbate the primary brain injury following head trauma? How can these effects be prevented or reduced? 8. What are the postoperative problems in the first 24 hours after coronary artery bypass graft? How are they prevented? 9. List, with examples, the causes of neurogenic pain. What symptoms are produced? What treatments are available? 10. Make a simple diagram, labelled to show the anatomical structures associated with the right internal jugular vein. List the complications of cannulation of this vessel, mentioning how each may be avoided. 11. Outline the possible reasons for the reduction, over the last decade, of maternal mortality associated with anaesthesia. 12. List the risk factors for venous thromboembolism and classify the current methods of prevention, with examples. Final 4 19th May 1998 Candidates are required to answer all twelve questions. Questions numbered 1 - 6 must be answered in Book A and questions numbered 7 - 12 must be answered in Book B. Candidates who fail to answer all twelve questions will not pass the Examination. Candidates are informed that one of the Examiners is present during the time allowed for the paper, for consultation in case any question should not appear clear. SECTION A (Book A) 1. Describe your procedure for cardiac life support in a child aged five years. 2. What methods are available for therapeutic nerve blockade? Explain the mechanism of action for each method. 3. You are asked to anaesthetise an 87 year old lady for diathermy of her bladder tumour on a day case basis. What are the potential problems of this case and how would you manage them? 4. What are the disadvantages of nitrous oxide in clinical practice? 5. How does a rotameter flowmeter work? Describe its advantages and limitations.

1025

6. Describe in detail how you would accurately measure a patients peak expiratory flow rate. What factors may give rise to erroneous readings? SECTION B (Book B) 7. Draw a labelled diagram of the anatomy of the anterior aspect of the wrist. How may this knowledge be used in anaesthetic practice? 8. List the factors associated with central venous catheter infections and suggest methods to limit such infections. 9. A patient with a history of obstructive sleep apnoea presents for an elective cholecystectomy. How would you assess the fitness for anaesthesia? What precautions would you take with your anaesthetic management of this patient? 10 A 60 year old man presents for a hemicolectomy. How may choice of pain management influence recovery from surgery? 11. Explain the importance of a high airway pressure alarm system during general anaesthesia. 12. List the pathophysiological and clinical features of HELLP syndrome. What are the diagnostic laboratory findings and the priorities in management? Final 5 3rd November 1998 Candidates are required to answer all twelve questions. Questions numbered 1 - 6 must be answered in Book A and questions numbered 7 - 12 must be answered in Book B. Candidates who fail to answer all twelve questions will not pass the Examination. Candidates are informed that one of the Examiners is present during the time allowed for the paper, for consultation in case any question should not appear clear. SECTION A (Book A) 1. List, with reasons, the factors which affect the incidence of perioperative myocardial infarction. 2. An adult patient is known to be severely difficult to intubate. Describe a technique of fibreoptic intubation for this case. 3. Outline the methods for prophylaxis of venous thromboembolism in routine surgical practice. 4. What are the factors contributing to unplanned awareness during general anaesthesia?

1026

5. What information would you wish to obtain from a patient at your postanaesthetic visit, the day after a total hip replacement? 6. What solutions are available for the restoration of circulating volume in a patient suffering from acute blood loss? Discuss the advantages and disadvantages of each. SECTION B (Book B) 7. Outline the clinical features and management of bupivacaine toxicity. 8. Draw a labelled diagram of the anatomical relations of the stellate ganglion. How is it blocked and what are the possible complications? 9. How would you determine the mixed venous oxygen content in the intensive care patient? What is the usefulness of this measurement? 10. A 20 year old patient, with sickle cell disease, was injured 12 hours ago. He has fractures of the metacarpal bones on his dominant hand. Elective surgical reduction and fixation is planned. Describe your anaesthetic management. 11. What safety features should be incorporated into a patient controlled analgesia (PCA) system and what is the purpose of each? What instructions would you give to the nursing staff, having set up the PCA? 12. Outline the anaesthetic management of a 2 year old child who is scheduled for therapeutic bronchoscopy following inhalation of a foreign body 2 days ago. The child does not exhibit any signs of upper airway obstruction. Final 6 18th May 1999 Candidates must attempt all twelve questions or they will fail the examination. Candidates are informed that one of the Examiners is present during the time allowed for the paper, for consultation in case any question should not appear clear. Candidates are advised to spend approximately the same amount of time on each question and ensure that the important issues are included in their answers SECTION A (Book A) 1. Outline your perioperative management of a patient with a ruptured abdominal aortic aneurysm. 2. What are the physical principles of the capnograph? Discuss the applications of capnography in anaesthetic practice. 3. A General Practitioner has contacted you for advice about a patient who may be susceptible to malignant hyperthermia. Write a letter to the General Practitioner explaining the significance of this condition for the patient and the relatives.

1027

4. Outline the possible complications of anaesthesia with a patient in the prone position. 5. What is the normal glucocorticoid response to surgery? Outline, with reasons, your perioperative corticosteroid regimens in patients: a) taking steroids at the time of surgery; b) who have stopped taking steroids several months previously. 6. How do you confirm that a double-lumen endobronchial tube has been placed correctly? Outline the possible complications associated with the use of this equipment. SECTION B (Book B) 7. What are the principles of cancer pain management? 8. Outline your technique for percutaneous tracheostomy, with particular reference to the anatomy involved. List the possible complications of this procedure. 9. A surgeon is attempting an inguinal herniorrhaphy in a fit obese young man under local anaesthetic infiltration that is proving inadequate and asks for your help. What anaesthetic strategies are available for managing this situation? 10 What are the principles of adult cardio-pulmonary bypass? What are the common complications of this procedure? 11. What are the risks for patients associated with the administration of general anaesthesia in dental surgery? How may these risks be reduced? 12. Write short notes, with reasons, on your anaesthetic management of emergency Caesarean section for cord prolapse in a fit 21 year old primagravida Final 7 2nd November 1999 CANDIDATES ARE REQUIRED TO ANSWER ALL TWELVE QUESTIONS: Candidates must attempt all twelve questions or they will fail the examination.. Candidates are informed that one of the Examiners is present during the time allowed for the paper, for consultation in case any question should not appear clear. Candidates are advised to spend approximately the same amount of time on each question and ensure that the important issues are included in their answers SECTION A (Book A) 1. A 70 year old man is to undergo an above knee amputation. What can be done to relieve any pain he may experience thereafter?

1028

2. A 27 year old man is admitted with a fracture of the cervical spine at C5/6. There are no other injuries. Describe the management of this patient in the first 48 hours after injury. 3. What is the anaesthetic management of pyloric stenosis in a 6 week old child? 4. What are the risks of carotid endarterectomy? How may the anaesthetist reduce these risks? 5. You are asked to investigate the effectiveness of a new anti-emetic agent. Briefly outline the priniciples which should guide the design of such a study. 6. What would make you suspect that a patient had sustained an air embolus during an anaesthetic? How should this situation be managed? SECTION B (Book B) 7. What factors contribute to postoperative cognitive deficits in elderly surgical patients? How may these risks be minimised? 8. What factors determine the rate of haemoglobin desaturation during a failed intubation? What can be done to maintain oxygenation in this situation? 9. List the predisposing factors for aspiration of gastric contents during general anaesthesia. How can the risk of this complication be minimised? How should it be treated? 10. What are the advantages of retaining motor power in a woman having an epidural for a normal labour? How can this be achieved and what would you check before allowing the woman to get out of bed? 11. Classify the types of heart block. Outline appropriate treatment in the intraoperative period. 12. How would you manage the transfer of a patient to a regional neurosurgical unit for evacuation of an extradural haematoma? ArticleDate:20040407 SiteSection: Article Final 8 18th May 2000 CANDIDATES ARE REQUIRED TO ANSWER ALL TWELVE QUESTIONS: Candidates must attempt all twelve questions or they will fail the examination. Candidates are informed that one of the Examiners is present during the time allowed for the paper, for consultation in case any question should not appear clear. Candidates are advised to

1029

spend approximately the same amount of time on each question and ensure that the important issues are included in their answers SECTION A (Book A) 1. A 70 year old man with chronic obstructive airways disease requires a transurethral resection of the prostate. Outline the advantages and disadvantages of intrathecal block for this patient. 2. What are the problems of monitoring anaesthetised patients in the magnetic resonance imaging unit? 3. What is the differential diagnosis of persistent headache in the puerperium of a woman who has undergone a regional anaesthetic technique? Describe the distinguishing clinical features of each cause. 4. How can jugular venous bulb oxygen saturation be measured? What factors cause its value to increase or decrease? 5. How may coagulation be assessed in the perioperative period? 6. Describe the anatomy of the coeliac plexus. What are the indications for its therapeutic blockade? SECTION B (Book B) 7. What are the advantages and disadvantages of day case anaesthesia in patients aged more than 80 years? 8. What is the role of the laryngeal mask airway in the management of difficult intubation? 9. A patient who is HIV sero-positive is scheduled to undergo a laparotomy. Discuss the factors determining the risk of transmission to theatre staff. How can this risk be reduced? 10. Describe your procedure for cardiac life support of a child aged 5 years. 11. Outline your management of a patient with status asthmaticus who is brought into the A&E Department. 12. List, with a brief statement on the effectiveness of each one, the methods described for detecting awareness during anaesthesia. Final 9 31st October 2000 Candidates are informed that one of the Examiners is present during the time allowed for the paper, for consultation in case any question should not appear clear. Candidates are advised to

1030

spend approximately the same amount of time on each question and ensure that the important issues are included in their answers SECTION A (Book A) 1. Outline the key points in the management of a patient with massive haemorrhage. 2. What diagnostic features would lead you to identify malignant hyperthermia during and immediately after anaesthesia of an adolescent presenting for appendicectomy? 3. Write brief notes on the physiological responses that constitute the stress response to surgery. 4. How does a rotameter work? Describe its advantages and limitations. 5. What safety features should be incorporated into an intravenous patient controlled analgesia (PCA) system and what is the purpose of each? What instructions would you give the nursing staff, having set up the PCA? A ten week old male infant weighing 3.5 kg is scheduled for inguinal hernia repair. He was delivered prematurely at thirty-four weeks. List the risk factors and state how these can be minimised. SECTION B (Book B) 7. You anaesthetised a sixty-four year old man for removal of a protruding disc C4/5. The patient was found to be quadriplegic in recovery. Discuss the likelihood of this being a consequence of you anaesthetic. 8. Discuss the perioperative management of the blood pressure of a patient undergoing removal of a phaeochromocytoma. 9. What are the main point that you would include in a patient information leaflet that you would submit to support an application to your local ethics committee to study a new nondepolarising muscle relaxant? 10. Describe the diagnosis and immediate assessment of a patient with smoke inhalation injury. 11. Describe the anatomy of the nerves involved for neural conduction blockade at the ankle. 12. A sixty-five year old diabetic female is to undergo a total abdominal hysterectomy. She is normally controlled by oral hypoglycaemic drugs. Describe your perioperative management of her blood sugar. ArticleDate:20040407 Final 10 Tuesday 15th May 2001 Candidates must attempt all twelve questions or they will fail the examination. Candidates are informed that one of the Examiners is present during the time allowed for the paper, for consultation in case any question should not appear clear. Candidates are advised to spend approximately the same amount of time on each question and ensure that the important issues are included in their answers SECTION A (Book A - Blue) 1. Describe the diagnosis and management of venous thromboembolism.

1031

2. Outline the anaesthetic management of an adult patient who requires surgery for a strangulated inguinal hernia. He suffers from obstructive sleep apnoea. 3. Outline the early management of a one year old child with 25% burns caused by scalding. 4. How would you recognise that a patient has regurgitated during an anaesthetic administered using a laryngeal mask airway? How would you manage the problem? 5. What is an appropriate intervertebral space at which to insert a spinal needle to administer a subarachnoid anaesthetic for a Caesarean section? Give your reasons and describe how you would locate the space. 6. Outline the methods of estimation of arterial pCO2 and their limitations. SECTION B (Book B - Pink) 7. What are the therapeutic uses of magnesium and how does it work? 8. What are the possible causes of intraoperative myocardial ischaemia in a 65 year old male undergoing major intra-abdominal surgery? Outline how the incidence of these may be minimised. 9. How can ultrasound be useful in anaesthesia and intensive care medicine? 10. When obtaining consent for an epidural for a primigravida in labour, what complications do you mention? Quote their incidence if known. What can be done to reduce the likelihood of these problems? 11. Describe the criteria and tests for brain stem death. Briefly indicate the neurological basis for each test. 12. How would you provide optimal pain relief for a 60 year old man undergoing shoulder replacement? Final 11 Tuesday 30th October 2001 Candidates must attempt all twelve questions or they will fail the examination. Candidates are informed that one of the Examiners is present during the time allowed for the paper, for consultation in case any question should not appear clear. Candidates are advised to spend approximately the same amount of time on each question and ensure that the important issues are included in their answers SECTION A (Book A - Blue) 1. Formulate a guideline for the peroperative administration of blood, explaining the reasons for your recommendations.

1032

2. What tests may be done to evaluate the adequacy of pulmonary oxygen transfer? Briefly describe how you would interpret the results. 3. A 70 year old man presents for a total hip replacement. He has no significant past medical history. At the anaesthetic assessment clinic he is noted to have a grade III ejection systolic murmur at the right sternal edge, radiating to his neck. Describe, with reasons, what investigations should be undertaken on this patient and explain how the results would affect your anaesthetic management. 4. Describe the features and management of phantom limb pain. 5. Discuss briefly the complications of placing a jugular central venous line. Where should the tip of a left internal jugular line lie and why ? 6. What are the anaesthetic considerations in a patient with autonomic neuropathy? SECTION B (Book B - Pink) 7. Draw a nephron with its blood supply. Where and how do the following exert their effects: loop diuretics, thiazide diuretics and aldosterone antagonists? 8. A patient who has undergone a heart transplant requires non-cardiac surgery. What problems may this present for the anaesthetist? 9. Discuss the ventilatory management of an adult with ARDS. 10. What nationally based audits in the UK include an examination of anaesthetic practice? Outline the methodology and recommendations of two recent reports. 11. A ten week old male infant weighing 3.5 kg is scheduled for inguinal hernia repair. He was delivered prematurely at thirty-four weeks. List the risk factors and state how these can be minimised. 12. Describe the features of the anaesthetic machine which are intended to prevent the delivery of a hypoxic mixture to the patient. ArticleDate:20040406 SiteSection: Article Final 12 Tuesday 30th April 2002 Candidates must attempt all twelve questions or they will fail the examination. Candidates are informed that one of the Examiners is present during the time allowed for the paper, for consultation in case any question should not appear clear. Candidates are advised to spend approximately the same amount of time on each question and ensure that the important issues are included in their answers

1033

SECTION A (Book A - Blue) 1. Draw a diagram of the lumbar plexus. Outline the anatomical basis of a 3 in 1 block. Explain why the block may fail to provide reliable analgesia for hip surgery. 2. Describe the principles involved in pulse oximetry. What are its limitations in clinical practice? 3. What aims and strategies are emphasised in a "Pain Management Programme"? 4. Discuss methods of applying non-invasive ventilation. What are its uses and benefits? 5. What is meta-analysis? Outline the methodology. How are the results usually presented? 6. A 78 year old male heavy smoker is admitted for laparotomy for his rectal cancer. He is treated with bendrofluazide and atenolol for hypertension (160/90 mmHg on admission). What specific actions would you take to improve the chances of a successful outcome. SECTION B (Book B - Pink) 7. Describe the anaesthetic management of a penetrating eye injury in a screaming 5 year old child. 8. Discuss the reasons for and against the use of nitrous oxide in anaesthetic practice. 9. What is disseminated intravascular coagulation. Discuss its management in the critically ill patient. 10. Write a guideline for reducing and treating postoperative nausea and vomiting. 11. Discuss the principles underlying the anaesthetic management of carotid endarterectomy. 12. List the key clinical features, and commonest causative agents, of severe anaphylaxis occurring during general anaesthesia. Outline its management. Final 13 29th October 2002 Candidates must attempt all twelve questions or they will fail the examination. Candidates are informed that one of the Examiners is present during the time allowed for the paper, for consultation in case any question should not appear clear. Candidates are advised to spend approximately the same amount of time on each question and ensure that the important issues are included in their answers SECTION A (Book A - Blue)

1034

1. List the three commonest causes of direct maternal deaths in the United Kingdom. What anatomical and physiological changes of pregnancy affect your ability to resuscitate a woman who has suffered cardiovascular collapse at full term? 2. What is pulmonary surfactant? Discuss its production in the lung, mechanism of action and function. What would be the effect of insufficient pulmonary surfactant? 3. Define contractility. Outline the methods available to the clinician to assess myocardial contractility in the perioperative period. 4. A 60 year old smoker requires non-laser surgery to the vocal cords. Outline the various anaesthetic techniques available, listing advantages and disadvantages of each. 5. Define pressure. List the methods available for measuring systemic arterial blood pressure. Outline the principles involved in one of the methods listed. 6. Outline the nerve pathways involved in the transmission and perception of a painful stimulus from the foot. SECTION B (Book B - Pink) 7. What are the possible deleterious consequences of cardiopulmonary bypass when used in coronary artery surgery? How may these be reduced? 8. Summarise the perioperative anaesthetic management of a patient who requires plating of his jaw fractured in a fight? 9. What are the principles of pain relief after surgery in a drug abuser dependent on opioids? 10. What are the indications for a preoperative chest radiograph? 11. What do you understand by the term critical incident? Following a critical incident, what information should be recorded? What sequence of events should ensue? 12. Discuss the causes of muscle weakness in a critically ill patient. How would you investigate them? ArticleDate:20040407 Final 15 23rd October 2003 Candidates are required to answer all twelve questions. Questions numbered 1 - 6 must be answered in Book A (Blue) and Questions numbered 7 - 12 must be answered in Book B (Pink). Candidates who fail to answer all twelve questions will not pass the Examination. Candidates are informed that one of the Examiners is present during the time allowed for the paper, for consultation in case any question should not appear clear. SECTION A (Book A - Blue)

1035

1. Describe the arterial blood supply to the spinal cord. How may it be compromised? 2. Describe how and why a vaporiser delivering desflurane is different from one delivering isoflurane. 3. You are called to the A and E department to review a 4 year old child who requires intubation. She has a clinical diagnosis of meningococcal sepsis. She has reduced consciousness and a petechial rash. Describe your immediate management. 4. What are the potential causes of delayed resumption of spontaneous ventilation after major intra-abdominal surgery with general anaesthesia? Discuss prevention, diagnosis and management. 5. What are the functions of cyclo-oxygenase (COX) enzymes? How are the side-effects of non-steroidal anti-inflammatory drugs related to inhibition of these enzymes? 6. What are the risks and benefits of thoracic epidural anaesthesia/analgesia for coronary artery surgery? SECTION B (Book B - Pink) 7. List the bedside tests available to predict a difficult intubation. Comment on their usefulness. 8. Draw the following diagrams (with values): A spirometer trace showing normal lung volumes, FEV1/FVC graphs and flow-volume loops. How are these altered by the following diseases: asthma, emphysema, pulmonary fibrosis, chest wall restriction and respiratory muscle disease? 9. What are the limitations and risks of intra-arterial pressure monitoring in the critically ill? How may these be minimised? 10. What potential problems and risks do you consider when planning the anaesthetic management of the delivery of twins? 11. Relate the clinical use of thiopentone and propofol to their pharmacological properties. 12. Outline the pathology of acute coronary syndromes. What pharmacological treatments are available for patients with an acute coronary syndrome? Final 16 11th May 2004 Candidates are required to answer all twelve questions. Questions numbered 1 - 3 must be answered in Book A (Blue), Questions numbered 4 - 6 must be answered in Book B (Pink), Questions 7 - 9 in Book C (Green) and Question 10 - 12 in Book D (Yellow). Candidates who fail to answer all twelve questions will not pass the Examination.

1036

An Examiner will be present during the time allowed for the paper, for consultation in case any question should not appear clear. SECTION A (Book A - Blue) 1. Describe two adjoining mid-lumbar vertebrae. Include the joints, their nerve supply and the ligaments. 2. Describe the preoperative assessment and preparation specific to an adult patient who requires a thoraco-abdominal oesophagectomy. Describe your anaesthetic plan for this operation. 3. What are the indications and contraindications for the use of an arterial tourniquet? What complications may arise from the use of such a tourniquet ? SECTION B (Book B - Pink) 4. Define capnography. Draw and label a normal capnograph trace. Why is capnography useful during general anaesthesia? Give examples of abnormal traces and their causes. 5. What procedures are associated with venous gas embolism? How can it be detected? What are the effects of a large venous gas embolus? Describe its management. 6. What advice, for and against, would you give a primagravida who is asking if she might eat and drink during her labour? Give reasons. SECTION C (Book C - Green) 7. Outline the pharmacology and clinical use of low molecular weight heparins for prophylaxis against deep vein thrombosis. 8. List the causes of perioperative atrial fibrillation. What are the dangers of acute onset atrial fibrillation? How would you manage acute atrial fibrillation in the postoperative period? 9. What problems are associated with anaesthesia for elective surgery in a patient with dialysis-dependent renal failure? SECTION D (Book D - Yellow) 10. List the indications and contraindications for Transcutaneous Electrical Nerve Stimulation (TENS)? What does the patient need to know when using a TENS machine ? 11. Outline, with reasons, your perioperative management of an otherwise healthy 4 year old admitted for tonsillectomy. 12. Define transfusion related acute lung injury (TRALI). Discuss its pathogenesis, presentation, management and outcome. Anaesthesia Questions

1037

What are the anaesthetic problems caused by morbid obesity? How would you prevent awareness under general anaesthesia? What are the advantages and limitations of the laryngeal mask airway? Under what circumstances should general anaesthesia for elective cases be postponed and why? What are the causes and management of hypoventilation immediately following anaesthesia? List the causes and briefly give the management of tachycardia in an adult during general anaesthesia. What are the causes and management of circulatory collapse at induction of anaesthesia? What is the pathophysiology of malignant hyperthermia? How would you investigate it? What factors do you consider important in selection of day case patients? How does the presence of aortic stenosis affect the management of anaesthesia? A houseman informs you that a patient's arterial pressure is raised before surgery; describe your management. What is the significance of preoperative jaundice? Describe the anaesthetic management of: perforating eye injury with full stomach; a child requiring reoperation for bleeding tonsil; infected molar tooth causing trismus; pyloric stenosis. Describe the pathophysiological processes of pre-eclamptic toxaemia of pregnancy. Describe the circle system for anaesthesia. What features of an anaesthetic machine minimise the risk of delivering hypoxic gas mixtures? Anatomy Describe the place of local analgesic nerve blocks during anaesthesia for cholecystectomy. How would you undertake ankle block for surgery of the foot? Describe the anatomy of the caudal space. What are the indications for analgesia administered by this route? Describe the anatomy of the inguinal canal and describe a technique for local anaesthesia for herniorrhaphy (excluding extradural/spinal blockade). Describe the anatomy of the diaphragm. Which factors in anaesthetic practice affect its function? Describe the anatomy of the first rib. Outline the technique for subclavian vein catheterisation.

1038

Describe the anatomy of the 9th intercostal nerve. What complications may arise following intercostal nerve block? Describe the prevention and treatment of the main complications of extradural analgesia using local analgesia. Describe the complications and technique for stellate ganglion block. Physiology What is meant by 'oxygen flux'? What factors affect it and what therapeutic measures increase it? What are the adverse effects of intermittent positive pressure ventilation? How would you minimise them? Describe the endocrine and metabolic responses to major surgery. How does anaesthesia affect them? What is physiological dead space? What factors affect it? What mechanisms are involved in anaphylactic reactions? How would you manage a patient showing signs of such a reaction? How does the physiology of children aged 1 year differ from that of adults? What factors affect cerebral blood flow? Briefly state their importance in relation to anaesthesia within 12 hours of head injury. Describe the physiological effects of hypercarbia. Describe the conducting system of the heart. How may abnormalities of cardiac conduction be revealed by the electrocardiogram? What are the causes and effects of hypothermia? Clinical measurement Describe the physical principles of a capnograph How may it be calibrated? What information can a capnograph give about an anaesthetic? What are the sources of error in a pulse oximeter? What information can be obtained from measuring central venous pressure? What arrangements are required for an adult head injured patient during transfer to a neurosurgical unit? Intensive care Describe the immediate resuscitation of a patient admitted to A&E following a fall from a 20 ft ladder. What forms of ventilatory support are used for patients undergoing mechanical ventilation? What methods of sedation of patients undergoing mechanical ventilation are available in the intensive care unit (ICU)? What factors affect the ablility to wean patients from mechanical ventilation?

1039

Describe the criteria and methods to wean such patients from mechanical support. Discuss the diagnosis for brain death. Discuss the management of a patient admitted to the ICU with: acute severe asthma; severe burns; septicaemia; Guillain Barre. Discuss the differential diagnosis of stridor in a 3-year-old child. What information can be measured or derived from a successfully placed multilumen PA catheter? Describe the alternatives to donor blood transfusion. What pathophysiological processes are involved in patients who develop acute respiratory distress syndrome? Pain management What are the advantages and disadvantages of the different ways by which opioids may be administered for postoperative pain? Write short notes on TENS ... neurolytic agents ... cryoanalgesia. What are the indications for a coeliac plexus block. Describe one approach and the complications of this technique. Describe the methods of pain relief for total abdominal hysterectomy. Write short notes on paracetamol. Clinical pharmacology What are the adverse affects of oxygen therapy? Describe the desirable and undesirable effects of nitrous oxide. What properties do you consider ideal for a neuromuscular blocking agent? Compare the cardiovascular effects of desflurane, sevoflurane and isoflurane. Discuss the mechanism of action and use of spinal opioids. What are the effects of an overdose of tricyclic antidepressant drugs? Compare and contrast gelatin-based plasma substitutes and Hartmann's solution.

1040

Final FRCA 2 Morning: 1. Viva 1 (50 minutes, Clinical-long case plus 2-3 short cases) Afternoon: 2. Viva 2 (30 minutes, basic sciences applied to anaesthesia, ITU & pain-3 short cases). Long Cases for the Clinical Viva Final clinical long case 1 Final clinical long case 2 Final clinical long case 3 Final clinical long case 4 Final clinical long case 5 Final clinical long case 6 Final clinical long case 7 Final clinical long case 8 Final clinical long case 9 Final clinical long case 10 Final clinical long case 11 Short cases for the Clinical Viva Final clinical short case 1 Final clinical short case 2 Final clinical short case 3 Final clinical short case 4 Final clinical short case 5 Final clinical short case 6 Final clinical short case 7 Final clinical short case 8 Final clinical short case 9 Final clinical short case 10 Final clinical short case 11 Final clinical short case 12 Short cases for Basic Sciences Final basic sciences short case 1 Final basic sciences short case 2 Final basic sciences short case 3 Final basic sciences short case 4 Final basic sciences short case 5

1041

Final basic sciences short case 6 Final clinical long case 1 Clinical viva 1 History A 68 year old gentleman presents for elective abdominal aortic aneurysm surgery. Hypertensive for the past 10 years. History of angina - past 2 years. Medication enalapril frusemide isosorbide Had chest pain 2 months ago. No evidence of myocardial infarction on investigation. On examination BP - 160/90 mmHg Jugular venous pressure not raised Rales on auscultation, both bases Liver just palpable below the right costal margin Investigations Positive findings White cell count = 12,000/ml Creatinine - 170 micromol/L Potassium - 3.2 mmol/L ECG - T wave inversion v3 - v6, avF X-Ray chest - mild cardiomegaly QUESTIONS Summarise the case. Do you think his blood pressure is adequately controlled? What is enalapril/how does it act/what are its side-effects? What do you think his chest pain was due to (give a differential diagnosis)? What are the implications if the chest pain had been due to myocardial infarction? What do you think about his potassium level/why is it low/do you think it is acute/will you correct it before surgery/how will you correct it/what happens if you do not correct it? Will you be happy to go ahead with the anaesthetic? What further investigations will you do/why? How will an echocardiogram help? Do you think he is in heart failure/does he have a chest infection? Differential diagnosis for hepatomegaly What do you find in his ECG? Did you determine the axis of the heart? Is there any evidence of MI in the past?

1042

What are the findings on the chest X-ray/how do you say the film is PA view/how did you determine that there is cardiomegaly/how do you measure CT ratio? What will be your anaesthetic management Why did you choose to do an epidural Will you be concerned about something (heparin) that you will be giving intraoperatively? What are the benefits (of the epidural)? What will you give in your epidural? What monitoring will you undertake and why? ArticleDate:20040408 Final clinical long case 2 Information given: 59 year old man for transurethral resection of the prostate Ex-smoker, 20 cigarettes/day Had 'shadow on lung' 2 years ago Treated with radiotherapy and drugs - no further information available Now coughing up blood Diabetic Drugs - glibenclamide 5 mg daily On examination: 1.75 m 72 kg Pulse: 80/min BP: 150/90 mmHg Respiratory rate: 20/min Chest: - trachea deviated to right - bronchial breathing and reduced breath sounds heard in the right upper chest Abdomen: - 10 cm bladder palapable

1043

Investigations: Hb 16.0 g/dl WBC 7.1 x 109/L Platelets 271 x 109/L MCV normal Differential normal Na 135 mEq/L K 3.5 mEq/L Urea 14.6 mmol/L Creatinine 237 umol/L Glucose 11 mmol/L CXR No cardiomegaly Trachea deviated to right Right upper zone shadowing and loss of volume on right Lung function tests: Predicted Measured FEV1 3.06 L 1.62 L FVC 4.18 L 3.18 L FEV1/FVC 69% 51% PEFR 533 L/min 250 L/min

QUESTIONS - Summarise the problems. - Summarise the positive findings on examination. - Does a BP of 150/90 mmHg bother you? - Comment on the investigations. - How would the raised urea and creatinine affect your management? (Give an example) - What do you think of a blood glucose of 11 mmol/L? - What does the CXR show and what is the differential diagnosis? - Do the lung function tests show a restrictive picture? - What else would you ask for regarding the lung function tests? - Would you be happy to anaesthetise this patient now?

1044

- What premedication would you prescribe and why? - What is your management of his diabetes? - What anaesthetic would you give and why? - How would you perform spinal anaesthesia? Final clinical long case 3 Clinical FRCA Viva Long case: (10 minutes to study case then questions for 20 minutes) A 24 year old male, who is a known drug addict, has recently been admitted to medical ward. He was found unconscious at home with a history of ?heroin overdose. His conscious level improved with 200 mcg naloxone and he became agitated, with a Glasgow Coma Scale of 14. He is complaining of being unable to feel his legs and of generalised weakness. His blood pressure is 80/40 mmHg and his peripheries are cool. He has a past history of depression and alcohol abuse. Investigations: Arterial blood gases post-naloxone on air: pCO2 6.0 kPa pH 7.2 HCO3- 20 mmol/L Urea and electrolytes Na 131 mEq/L K 7.8 mEq/L Ur 13.0 mmol/L Cr 331 umol/L CK 50,000 IU ECG Rate 50 bpm sinus (abnormal intermittent p waves) Broad QRS peaked T waves pO2 8.0 kPa

CXR CVP line in situ. Bilateral diffuse shadowing. R middle lobe collapse. No pneumothorax. Questions - Summarise the case. - What may have made him unconscious other than heroin? - What other drugs may he have taken? - How would you determine this?

1045

- What does the ECG show? - Why can't he feel his legs and why is he weak? - What may the cause of his raised K+ and his renal impairment? - What is rhabdomyolsis and how does it cause renal failure? - Why may he have it? - How would you resuscitate him? - How would you treat the K+ acutely and subsequently? - What is the difference between haemofiltration and dialysis? - How do they work, and which one would you use in this case, given the choice, and why? Final clinical long case 4 Clinical Viva 4 You have 10 minutes to study the case history, CXR, ECG, investigations. A 64 year old man is undergoing right total hip replacement. - He gives a history of asthma and chest pain. - He is taking oxitropium bromide. - He has been hospitalised 3 times for chest pain. - The last hospitalisation was 6 months ago. - He has a history of deep vein thrombosis after a previous operation and took Warfarin, which was stopped 3 months ago. - He also has history of hiatus hernia, diagnosed after oesophagoscopy. Examination - He is moderately obese; his weight is 95 kg, his height is 1.65 cm. - Apart from bilateral basal crepitations and diffuse wheezing, all over both lung fields are normal; first and second heart sounds normal, apex beat is not displaced. - Blood pressure is 140/90 mmHg. - Full blood count - NAD - Lung function test - FEV1 - 65% of predicted value. FEV1/FVC - 50%. PEFR - 70% of predicted value. - ECG - Bifacicular block. Inverted T waves in Leads 1, AVL, V1 to V4.

1046

- CXR - shows minimal basal effusion. Lungs are not hyperinflated. QUESTIONS 1. Summarise the case 2. Interpret the data 3. What further investigation would you carry out? 4. How would you anaesthetise this patient? 5. How would you prevent DVT? Final clinical long case 5 You are presented with a 7 year old male child with acute painful swelling in the left scrotum, vomiting and torsion of the testes. He has had a cough and been wheezing for 3 days. He had his last meal 4 hours ago. He has been asthmatic for the past 3 years and is currently taking salbutamol and beclomethasone inhalers. On examination: The child is found to be very anxious and in pain. He is mildly dehydrated, and weighs 19 kg. Respiratory signs Respiratory rate: 28/min Left lower zone: air entry decreased Percussion note decreased Bilateral rhonchi Abdominal examination: scrotum nothing abnormal detected Cardiovascular signs Heart rate: >120 bpm Blood pressure: 110/60 mmHg Auscultation: nothing abnormal detected The result of investigations are as follows: Haemoglobin: 10.6 g/dl White blood cell count: 14,600/ml Neutrophil count: 80 x 106 ml (Normal range 2-7) lymphocyte count: 20 x 106 ml (Normal range 1.5-4) Packed cell volume: 40%

1047

Other investigations carried out: Chest X-Ray Urea and electrolytes Na+ K+ Creatinine QUESTIONS (answers below) 1. Summarise the patient and identify the problems associated with this case. 2 Give the differential diagnosis. 3. Discuss childhood asthma. 4. How do you assess severity of asthma? 5. Comment on investigations. 6. How would you manage this child? 7. How would you anaesthetise this child? ANSWERS 1. Emergency surgery - risk of ischaemia Lower respiratory tract infection Full stomach Dehydration Pain relief 2. Infection - epididymo-orchitis, abscess Trauma haematoma Obstructed hernia 3. Disappears after about 1015 years 1015% progress to have adult asthma Family history of atopy 4. Patients history: Off school/work - frequency Hospital admissions/ITU admissions Steroid courses given On examination: Heart rate: pulsus paradox Silent chest Cyanosis - very late PEFR <30% predicted 5. White blood cell count increase: could be due to epididymo-orchitis or urinary tract infection Haemoglobin decrease: could be due to malnutrition Is the child dehydrated? Weight: expected weight 22 kg [(age x 2) + 8 kg]. Why has his weight decreased?

1048

6. The following areas should be covered: Analgesia Fluids - maintenance and allowance for dehydration, signs of adequate hydration Optimise bronchodilators Treat infection; steroid cover Physiotherapy 7. Premedication EMLA/AMHOP (Eutectic mixture of LA) Salbutamol General anaesthesia Rapid sequence induction: as the child has a full stomach, suxamethonium? rocuronium Endotracheal tube size: diameter - [Age/4] + 4.5 ; length - [Age/2] + 12 Intra-operative analgesia Non-steroidal anti-inflammatory drugs Opioids Caudal anaesthesia - dose Postoperative care Pain relief Final clinical long case 6 A 68-year-old man has an implanted pacemaker and a past history of angina-like chest pain. He also has chronic obstructive pulmonary disease but maintains that he can climb stairs and has a reasonably active life. He has been smoking 20 cigarettes a day for many years. He is scheduled for an elective right nephrectomy for renal cell carcinoma and has recently undergone haemodialysis. He is on perindopril 20 mg once daily, frusemide 500 mg once daily and aspirin 75 mg twice daily. On examination, he has pitting pedal oedema and is wheezing. The result of investigations are as follows: Biochemistry Na+: 138 mEq/L K+: 3.5 mEq/L Cl: 110 mEq/L Urea: 30 mmol/L Creatinine: 524 mol/L Liver function tests: within normal limits (WNL) Haematology Haemoglobin: 7.5 g/dl Haematocrit: 25% Platelet count: 227 X 106 White blood cell count: 5.5 X 106

1049

Pulmonary function tests FEV1: 1.8 L FEV1/FVC: 55% TLCO Within normal limits Flowvolume loop: an obstructive pattern loop was provided Chest X-ray findings Implanted pacemaker Chronic venovenous haemofiltration (CVVH) Right subclavian lines Right internal jugular vein central line Cardiac/pulmonary fields within normal limits ECG findings Completely paced rhythm 60/min Left axis deviation Wide QRS complexes >3 mm ST elevation with tall T waves in V2-V5 QUESTIONS 1. Summarise the case 2. What physiological system should be discussed here? 3. The surgeon wants to operate immediately for fear of metastasis if postponed would you anaesthetise? If not, why not? When would you deem it appropriate to operate? What more information do you require? 4. Discuss the results of all of the investigations above 5. What would be your anaesthetic plan? Final clinical long case 7 A 52 year old male presents electively for transurethral resection of the prostate. He has a history of non-insulin-dependent diabetes, and 23 months ago underwent chemotherapy and radiotherapy for 'a shadow on the lung'. He has been a life-long smoker, and recently had an episode of haemoptysis. He is currently taking glibenclamide 5 mg once daily. On examination He is of average build Respiratory signs: respiratory rate: 18/min; trachea is deviated to the right; bronchial breathing can be heard in the right upper zone Cardiovascular signs: pulse: 80 bpm; blood pressure: 130/90 mmHg Gastrointestinal signs: palpable enlarged bladder Investigations - Haemoglobin 16 g/dl

1050

- White blood cell count: 8 X 106/ml - Platelets: normal - Urea 18: mmol/L - Creatinine 180 mol/L - Na+ and K+: normal - Liver function tests: normal - Chest X-ray: right upper lobe collapse - Pulmonary function tests: FEV1 1.9 L; FVC 3.75 L - FEV/FVC ratio 0.53 QUESTIONS 1. Present the salient features of this case. 2. Describe the features of prostatic hypertrophy. 3. Describe the features of renal failure. 4. How might carcinoma of the prostate be diagnosed and treated? 5. Why might this patient have right upper lobe collapse? 6. Why might this patient have a raised haemoglobin level and what is the mechanism for this? 7. Describe the relevance of the lung function results. 8. How might this patient's chest condition be optimised prior to surgery? 9. How would you anaesthetise this man? 10. What are the options for controlling a diabetic patient's glucose perioperatively? 11. Describe the types of oral hypoglycaemic agents available. 12. How would you assess a diabetic patient preoperatively? Final clinical long case 8 You are presented with a 70-year-old man 4 days postoperatively after an abdominal aortic aneurysm repair. He has a complicated past medical history, which is as follows: - Hypertension 1991 - Transient ischaemic attack (TIA) 1997 - Type 2 diabetes 1998 - Shortness of breath on exertion (SOBOE) 2000 He has presented to A+E with sudden onset shortness of breath. You, as the ITU registrar, are called down to assess him. His chest X-ray has shown pulmonary oedema, cardiomegaly and right lower lobe collapse with tracheal deviation. His ECG has shown atrial fibrillation (AF) with myocardial ischaemia. QUESTIONS 1. Please talk in depth about the management of AF in intensive care. 2. Which drugs would you use to treat his AF? 3. What are the causes of AF? 4. How would you identify the causes in this situation? 5. How would you manage this case?

1051

Final clinical long case 9 A 62 year old male with significant cardiovascular and respiratory disease presents for right total hip replacement. He has previously been admitted several times with chest pain, which has been relieved by glyceryl trinitrate (GTN), but he continues to smoke 30 cigarettes a day. He is also obese, and has a hiatus hernia, decreased exercise tolerance and a past history of deep vein thrombosis (DVT). On examination, his blood pressure is 140/85 mmHg, and he has bilateral inspiratory crackles. His ECG showed widened QRS complexes and a normal axis but did not show the classical signs of right bundle branch block. His chest X-ray showed flattened hemidiaphragms. He had a normal cardiothoracic ratio, and his lung function tests revealed an obstructive picture. The patients blood tests were all normal, with a haemoglobin level of 15 g/dl. N.B. The history, ECG etc. for this case can be found in "The Clinical Viva" by Mills and Maguire. QUESTIONS 1. Summarise the main issues regarding this case. 2. What other investigations would you undertake? 3. Which cardiovascular investigations should be carried out? 4. What can you glean from the patients baseline arterial blood gases? 5. Would you refer this patient to a cardiologist? 6. What can you say about the patients blood pressure, and how would this affect your treatment? 7. What other information would you need from his lung function tests? 8. What would you be concerned about in the full blood count? 9. What is the mechanism of the development of polycythaemia in such patients? 10. How should this patient be optimised, for example in terms of preventing DVTs? 11. How would you treat the patients hiatus hernia? 12. Would you anaesthetise him, by general or regional anaesthesia? Why? 13. Discuss the patients postoperative care.

1052

Final clinical long case 10 Long case A 62 year old male with significant cardiovascular and respiratory disease presents for right total hip replacement. He has previously been admitted several times with chest pain, which has been relieved by glyceryl trinitrate (GTN), but he continues to smoke 30 cigarettes a day. He is also obese, and has a hiatus hernia, decreased exercise tolerance and a past history of deep vein thrombosis (DVT). On examination, his blood pressure is 140/85 mmHg, and he has bilateral inspiratory crackles. His ECG showed widened QRS complexes and a normal axis but did not show the classical signs of right bundle branch block. His chest X-ray showed flattened hemidiaphragms. He had a normal cardiothoracic ratio, and his lung function tests revealed an obstructive picture. The patients blood tests were all normal, with a haemoglobin level of 15 g/dl. (N.B. The history, ECG etc. for this case can be found in "The Clinical Viva" by Mills and Maguire). QUESTIONS 1. Summarise the main issues regarding this case. 2. What other investigations would you undertake? 3. Which cardiovascular investigations should be carried out? 4. What can you glean from the patients baseline arterial blood gases? 5. Would you refer this patient to a cardiologist? 6. What can you say about the patients blood pressure, and how would this affect your treatment? 7. What other information would you need from his lung function tests? 8. What would you be concerned about in the full blood count? 9. What is the mechanism of the development of polycythaemia in such patients? 10. How should this patient be optimised, for example in terms of preventing DVTs? 11. How would you treat the patients hiatus hernia? 12. Would you anaesthetise him by general or regional anaesthesia? Why? 13. Discuss the patients postoperative care. Final clinical long case 11 Long case A patient presents for a radical nephrectomy for malignancy. He has a history of chronic obstructive pulmonary disease, ischaemic heart disease, a pacemaker and requires haemodialysis. Investigations Full blood count, urea and electrolytes, chest X-ray, ECG, echocardiogram, pulmonary function tests.

1053

Questions What is the transfer factor? (discussion re: the pulmonary function tests) How would you assess this patient's volaemic status? Would you consider an epidural? Discuss the pros vs cons How would you optimise this patient preoperatively? Final clinical short case 1 1. Discuss infantile pyloric stenosis: What is the age distribution/sex distribution/population group most commonly affected? What is the pathology? What are the signs and symptoms? How would you assess a 10 week old child with pyloric stenosis? How would you assess the degree of dehydration? What are the biochemical abnormalities? At what stage would you be prepared to proceed with the anaesthetic? How would you anaesthetise the child? How would you ventilate the child intraoperatively (mode, rate etc)? How would you provide analgesia? what postoperative problems might you encounter? 2. You are presented with an intravenous drug abuser: What are the problems you might encounter? What precautions would you take? What would you do in the case of a needle stick injury? What measures would you take if blood spills onto the floor and the anaesthetic machine? How would you anaesthetise a patient who has injected himself in the femoral artery and is scheduled to have a debridement of his leg? What would be your choice of anaesthetic, and what problems might you encounter? How would you manage postoperative analgesia if the patient is a heroin addict? 3. A patient who is a known epileptic, taking phenytoin, presents for abscess drainage in his abdomen. You are shown his ECG (2:1 block). Is the ECG normal? If not, what is the problem? How is the action potential affected? How does phenytoin cause this problem? What does it do to the action potential? What investigations would you perform? (measure phenytoin levels/electrolytes) Would you anaesthetise the patient? What would you do if it were an emergency? What problems might you encounter, and what precautions would you take? Final clinical short case 2 Short case 1

1054

What is trigeminal neuralgia? What treatments can be given? Describe the anatomy of trigeminal ganglion block. What dose/volume of glycerol should be given? How would you perform a microvascular decompression? Short case 2 A 23 year old female with severe asthma presents for elective nasal polypectomy. She is currently on inhaled and oral steroids and large doses of inhaled salbutamol. You are shown her ECG (shows atrial flutter with 3:1 block). QUESTIONS Describe the features of the ECG. How is the axis calculated? Why might the patient be in atrial flutter? How do beta-2 agonists work (beyond the G protein level)? How would you treat her, and when would you anaesthetise her? Short case 3 A 25 year old female presents for thymectomy for myasthenia gravis. QUESTIONS 1. What is myasthenia gravis? 2. Tell me as much as you know about it 3. How is it diagnosed? (include EMG) 4. What is edrophonium? 5. How does it work? 6. What dose should be used? 7. What are the symptoms? 8. How would you anaesthetise her? Final clinical short case 3 A 16 year old girl with Down syndrome presents for dental conservation as a day case. QUESTIONS 1. Outline the problems you might encounter. 2. What are the specific problems associated with Down syndrome? 3. What premedication would you give and why? 4. What anaesthetic would you give and why? Final clinical short case 4

1055

Viva 1 A 13 year old child presents with Guillain Barr syndrome. He has a history of 10 days of malaise, fatigue and weakness. What are the treatment options? (discuss plasmaphoresis versus intravenous immunoglobulin) You are shown his chest X-ray; describe the features. (discuss pneumomediastinum) Viva 2 a) A patient with end-stage renal failure presents for interval appendicectomy Discuss the management pre-, intra- and postoperatively b) A patient presents with a C2 fracture/dislocation. Discuss the anaesthetic management for the repair of acute cervical fracture. c) A patient with a 1-month history of anaphylaxis during general anaesthesia now presents for an evacuation of retained products of conception procedure (ERPC). Discuss the anaesthetic management of this patient. Discuss the investigations for anaphylaxis postoperatively. Final clinical short case 5 A 23 year old woman with Down syndrome presents with a pale, painful right leg. QUESTIONS 1. What is the likely diagnosis? 2. What is Eisenmenger's syndrome? 3. What are the problems with anaesthetising a Down syndrome patient? 4. What would the surgeon want to do? 5. What type of anaesthesia would you choose? You are shown the chest X-ray of a 26 year old female who has undergone a thoracic sympathectomy by the transaxillary approach. This shows: An endotracheal tube in situ (flexometallic) Pneumomediastinum (air around the heart and great vessels) A globular cardiac silhouette QUESTIONS 1. What is likely to be the diagnosis?

1056

2. How would you manage the patient? A patient presents with interpleural block. QUESTIONS 1. What are the indications? 2. Describe the procedure you would perform, in terms of the site, position and type of needle, and the catheter used. What would be the relation of the needle to the rib, which space would you choose? 3. What would be your choice of anaesthetic? 4. What volume of anaesthetic would you administer? What rate of infusion would you choose? 5. What complications might you encounter? Final clinical short case 6 Your Examiner describes a patient in the accident and emergency department with features of a tension pneumothorax. QUESTIONS 1. How would you treat the patient? 2. Draw the arrangement of a chest drain bottle. 3. How far does the tube go in the water and why is this? 4. Why is the bottle so large? 5. How long should the tubing be? 6. Why should the drain not be lifted above the patient? 7. Draw the arrangement of bottles used to drain a haemothorax and suction. 8. What would you do if the drain stops swinging/bubbling? Final clinical short case 7 A 21 year old primiparous Afro-Caribbean woman presents with an unbooked pregnancy of 31 weeks gestation. She is admitted onto the labour ward with foetal distress and her blood test results are: Sickledex positive Haemoglobin: 9.2 g/dL Mean cell volume: 60 fL QUESTIONS 1. What problems might you encounter? 2. What is fL the abbreviation for? 3. How would you interpret the blood test results? 4. What are the important things to consider in sickle cell disease?

1057

5. How would you anaesthetise the patient for an immediate emergency Caesarean section? Eight hours later you are called to see the patient on the ward. She is complaining of pleuritic chest pain and has a raised respiratory rate and a tachycardia. Her oxygen saturation (SpO2) is 89% and her haemoglobin is now 6.5 g/dL ,with a reticulocyte count of 16%. QUESTION What do these signs and symptoms suggest to you, and how would you manage the patient? Final clinical short case 8 You are asked to anaesthetise a fit 3 year old boy for a circumcision as a day case. QUESTIONS 1. Is this an appropriate patient for day case surgery? 2. Would you anaesthetise a 1 year old child as a day case? 3. What is your lower cut-off age for day case surgery? 4. What premedication would you give? 5. What anaesthetic would you give and why? 6. How would you administer analgesia? 7. How exactly would you perform the local block that you have chosen? 8. How would you anaesthetise this 3 year old child for a circumcision? 9. Under what circumstances would you consider deferring the operation? 10. What kinds of regional blocks might you consider? 11. How would you perform a penile block/caudal? 12. What would be your choice of local anaesthetic and how much would you use? 13. What postoperative analgesia would you prescribe, and at what dose? Final clinical short case 9 QUESTIONS 1. What non-invasive methods for estimating cardiac output do you know? 2. What is the significance of urinary output? 3. Is the darkness of the urine significant? 4. Why not? A young man presents to the accident and emergency department with an acute onset of dyspnoea. QUESTIONS 1. What course of action would you take? 2. What are the likely causes of this presentation? 3. How would you differentiate between a flail chest and a pneumothorax? 4. Describe how a chest drain is inserted.

1058

5. How far beneath the water must the tube be placed? 6. What is the significance of the depth of the under-water seal? Final clinical short case 10 QUESTIONS 1. What are the indications for a lumbar sympathectomy? 2. How would you perform this procedure? 3. What are the contraindications to a lumbar sympathectomy? 4. What substances would you inject and at what doses? 5. What strength of alcohol might you use? 6. What do you understand by the term 'total intravenous anaesthesia'? 7. What sort of patients would be suitable for such a technique, and for what sort of procedures? 8. Which agents would you use? 9. What is the half-life of propofol? 10. What are its advantages and disadvantages? Final clinical short case 11 Questions 1. What are the different reasons for being called to recovery? 2. What are the causes of low blood pressure in recovery 3. How would you approach the problem? 4. Discuss the management of acute severe asthma 5. Discuss the causes and prevention of secondary brain insult You are presented with an 80-year-old woman with diabetes mellitus and a touch of indigestion on climbing stairs. 1. Describe her preoperative assessment and perioperative management for cataract surgery. 2. Describe the control of blood pressure, both in the immediate and long term. Final clinical short case 12 1. Discuss the management of a 45-year-old female presenting with a perforated appendix and a blood glucose level of 23 mmol/L. She has been unwell for a week, and the surgeons want to take her to theatre as quickly as possible. 2. How would you manage an unanticipated difficult airway? Compare your management to the situation where a difficulty is expected. 3. How would you manage an elderly patient presenting for a quick excision biopsy of a lump in his throat? You notice that he is quite hoarse when you see him.

1059

4. How would you manage a primiparous woman, presenting for a Caesarean section, who is short of breath at rest? You notice that her booking blood pressure was at least 20 mmHg lower than its present value. 5. What neurosurgical complications do you know of? Select one complication and describe how you would manage it. 6. A 10-week-old female infant weighing 3.5 kg is scheduled for inguinal hernia repair. She was delivered prematurely at 34 weeks. List the risk factors and state how these can be minimised. 7. Discuss the perioperative management of a young patient undergoing removal of a phaeochromocytoma. 8. How would you manage a patient with 45% burns and smoke inhalation postoperatively on the intensive care unit? Final basic sciences short case 1 Topic 1 Flow-volume loop QUESTIONS 1. Draw a normal flow-volume loop. 2. Draw this in the case of an obstructive lung disorder. 3. Draw this in the case of a restrictive lung disorder. 4. Draw this in the case of an upper airway obstruction (e.g. laryngeal). 5. How is flow measured? Topic 2 Hospital gas supplies QUESTIONS 1. Draw and describe an oxygen tank. Describe the liquid, pressure and temperature Evaporating grid? In the cylinder: what should be the pressure? What volume of oxygen should it contain and what state should it be in? What type of valve is found on the cylinder? From what material is the cylinder made? 2. Describe the properties of a nitrous oxide tank, in terms of: the manifold, pressure and state? In the cylinder: what should be the values for pressure, volume and filling ratio?

1060

From what material is the cylinder made? What is the air source? What is the air pressure? What is the difference between an air supply used for anaesthesia and one used for other purposes? Topic 3 Anaesthesia and the kidneys QUESTIONS 1. Draw a normal nephron; show what gets absorbed and where. 2. What is the effect of anaesthesia on renal function? 3. What drugs or gases used by anaesthetists might precipitate acute renal failure? Topic 4 Larynx QUESTIONS 1. Draw a diagram of the larynx. 2. Describe the sensory and motor nerve supply. 3. Describe the actions of the laryngeal muscles. 4. Describe nerve block for the larynx. 5. Describe the post-surgical and non-surgical causes of stridor. Final basic sciences short case 2 QUESTIONS 1) Discuss the cytochrome p450 superfamily of enzymes; how do they affect anaesthetic drugs and what are the effects of genetic differences between individuals? 2) Discuss the strategies for avoiding the necessity of a blood transfusion in a patient. 3) Describe the anatomy of the caudal space, and discuss caudal block. 4) Discuss hyperbaric oxygen therapy. Final basic sciences short case 3 Answer the following questions on one-lung ventilation (OLV): What are the indications for OLV?

1061

What are the physiological mechanisms/causes of hypoxaemia with intrathoracic surgery? How should hypoxaemia on OLV be treated? How would you apply continuous positive airway pressure (CPAP) to the non-dependent lung? How could the surgeon assist in applying CPAP? Answer the following questions on electricity: What is Ohms law? What is the difference between direct (DC) and alternating current (AC)? Draw a diagram demonstrating the differences between AC and DC. Does Ohms law apply to AC or DC? Is the mains supply AC or DC? What is mains current frequency? Is the mains current frequency safe? If the answer to the previous question is no, why is it used? Discuss the problem of leakage current. By what mechanisms might a patient suffer? Final basic sciences short case 4 Questions 1. Discuss the safety features of the anaesthetic machine. 2. Compare sevoflurane and isoflurane. 3. Describe the sympathetic supply of the eye. 4. Discuss the management of carbon monoxide poisoning. Final basic sciences short case 5 1. What is meant by laminar/turbulent flow? What is the Hagen-Poiseuille equation? What is Reynolds number? Describe the use of the above concepts in medicine.

1062

2. Define the following terms: Log doseresponse curve Agonist Antagonist Therapeutic index Partial agonist 3. List the various types of antagonists. 4. Compare morphine and buprenorphine. 5. Describe the structure and function of the acetylcholine receptor. 6. Describe the oxygen dissociation curve in different types of hypoxia. Final basic sciences short case 6 Answer the following questions on the intercostal nerves List the branches. Where are they located? (i.e. between which intercostal muscles?) Where do they arise from? - Why are the 1st, 2nd and 12th intercostal nerves different? Which nerve supplies the skin over the back? Describe the nerve supply to the diaphragm. How is an intercostal nerve block performed (site, volume etc.)? Describe the location and arrangement of the neurovascular nerve bundle. When an intercostal nerve block is performed, where and how does the local anaesthetic spread? What are the complications of an intercostal nerve block? Answer the following questions on adrenaline: What receptors does it act on? Where is it secreted from?

1063

How is it metabolised? - How is it synthesised? - Why is it used in local anaesthesia? What are the systemic effects of adrenaline? Answer the following questions on one-lung ventilation (OLV): What are the indications for OLV? - What are the physiological mechanisms/causes of hypoxaemia with intrathoracic surgery? How should hypoxaemia on OLV be treated? How would you apply continuous positive airway pressure (CPAP) to the non-dependent lung? How could the surgeon assist in applying CPAP? Answer the following questions on electricity: What is Ohms law? What is the difference between direct (DC) and alternating (AC) current? Draw a diagram demonstrating the differences between AC and DC. Does Ohms law apply to AC or DC? Is the mains supply AC or DC? What is mains current frequency? - Is the mains current frequency safe? If the answer to the previous question is no, why is it used? Discuss the problem of leakage current. By what mechanisms might a patient suffer?

1064

You might also like